Sie sind auf Seite 1von 869

Table of contents

1 Public Administration History & Theory

1.1 Key concepts in public administration 2

1.2 History of world’s public admin in a jiffy 10

1.3 Different theories and their time frames 17

1.4 Names and theories in public administration in chronological order 22

1.5 Rethinking the identity of public administration 29

1.6 Theory and practice of public administration 54

1.7 Public choice theory 58

1.8 The development of American administration 62

1.9 Paradigms 66

1.10 PA Theory, Paradigms and the Identity Crisis 74

1.11 The SIMON-WALDO debate Empiricism vs. Normative theory 95

1.12 Major authors in PA Theory 99

1.13 Iron cage – Weber 119

1.14 Paradigmatic Progress in Public Administration: Knowledge Development 123

1.15 Outlines 133

1.16 Some Important Comps Questions and Student Answers 196

2 Administrative Management & Administrative Law

2.1 Keynesian Economics 211

2.2 Risk Management as a Strategic Posture: Public vs. Private Approaches 220

2.3 Accounting Methods and Strategic Planning / Management Issues 232

2.4 The Sarbanes-Oxley Act of 2002; Audit and Governance of Not for Profit Organizations 249

2.5 Etzioni & Lorcsh - Attempts to Combine 258

2.6 Administrative Management – Strategic Management 260

2.7 Important Comps Questions and Student Answers 284


3 Organization Theory

3.1 Perspectives of Organizational Theory 294

3.2 Organizational Theory Chart 299

3.3 Organizational Theory Matrix 304

3.4 The History of Org Theory 321

3.5 Nine Major “Schools” of Organization Theory 329

3.6 Literature Reviews 347

3.7 WALDO – ORG Theory 455

3.8 They Dyadic Environment of the Organization 456

3.9 Organization of the Environment 459

3.10 Networks In and Around Organizations 462

3.11 The Rise and Transformation of the Corporate Form 463

3.12 Changing Contours of Organizations and Org Theory 467

4 Public Budgeting

4.1 Major Authors in Public Budgeting and their theories 469

4.2 Budgeting Theory 475

4.3 The Big Names in Budgeting 505

4.4 Budgeting Terms 510

4.5 Accrual Based Accounting vs. Cost Based 524

4.6 Federal Defense Budgeting 526

4.7 Summary of Budget: Theory, Concepts, Methods, and Issues 548

4.8 Important Comps Questions and Student Answers 558

5 Public Policy Analysis

5.1 History of Public Policy 598

5.2 Public Policy Defined - Various definitions 601

5.3 Public Policy Analysis Outline 602

5.4 Definition of Budget in “Public Policymaking” 609


5.5 Public Policy Analysis in Detail 610

5.6 Policy Implementation 611

5.7 The Science of "Muddling Through" 619

5.8 "System Politics and Systems Budgeting" 622

5.9 Policy Models 625

5.10 The Bureaucracy and the Presidency 630

5.11 The Concept of Policy Evaluation 632

5.12 Five-Step Method for Policy Analysis 634

5.13 Important Comps Questions and Student Answers 635

6 Appendix A - Sample Comps Questions and Student Answers 640

7 Appendix B – NC State Comps Questions 710

8 Appendix C – Rutgers University Comps Questions 727

9 Appendix D - TSU Comps Questions 747

10 Appendix E - University of Georgia Comps Questions 771

11 Appendix F – Reading List and Online Resources 820


Chapter 1
Public Administration
History & Theory

MPA Comprehensive Exam


Study Guide
1.1 Key concepts in public administration

Accountability - Peters (2001, pg. 137) suggests that this is an element of 2nd wave of
reforms. Earlier forms of accountability focused on exceptional failures with political
embarrassment of the sitting government. There has been a shift in mechanisms of
accountability to push toward a focus on average performance and a more complete
assessment of organizational and programmatic performance. (Scorecards,
Performance Improvement)

Bureaucracy - Weber (1946, pg. 50) in his landmark article, “Bureaucracy” lists 6
characteristics of a bureaucracy:

1. There is a principle of fixed and official jurisdictional areas, which are generally
ordered by rules, that is, by laws or administration regulations,
2. The priniciples of office hierarchy and of levels of graded authority mean a
firmly ordered system of super- and subordination in which there is a
supervision of the lower offices by the higher ones,
3. The management of the modern office is based upon written documents (“the
files”), which are preserved in their original or draught form,
4. Office management, at least all specialized office management—and such
management is distinctly modern—usually presupposes thorough and expert
training,
5. When the office is fully developed, official activity demands the full working
capacity of the official, irrespective of the fact that his obligatory time in the
bureau may be firmly delimited,
6. The management of the office follows general rules, which are more or less
stable, more or less exhaustive, and which can be learned.

He also comments on the position of the Official:

1. Office holding is a “vocation”,


2. The personal position of the official is patterned in the following way:
a. enjoys a distinct social esteem as compared with the governed,
b. pure type of bureaucratic official is appointed by a superior authority
c. normally, the position of the official is held for life

2|Page
d. the official receives the regular pecuniary compensation of a normally
fixed salary and the old age security provided by a pension. Salary is
measured according to status, not wage.
e. The official is set for a “career” within the hierarchical order of the public
service.

Citizen participation: how administrators relate to citizens

Frederickson attempted to describe the public administrator as a representative


citizen attempting to bridge the chasm of self-interest that has been suggested since
the founding of the country. In this model, public administrators are employed by
their fellow citizens to carry out the work of citizenship on their behalf, and are the
primary contact between the citizens and public organizations. When the contact is
effectively carried out, the contract is reaffirmed and renewed. Frederickson offers the
following model for citizen vs. administrative participation in government:

HIGH CITIZENSHIP

HIGH C ITIZENSHIP HIGH C ITIZENSHIP

LOW ADMINISTRATION HIGH ADMINISTRATION

LOW ANCIENT A THENS ANCIENT ROME HIGH

ADMINISTRATION MODERN AMERICA ANCIENT EGYPT ADMINISTRATION

LOW ADMINISTRATION HIGH ADMINISTRATION

LOW C ITIZENSHIP LOW C ITIZENSHIP

LOW CITIZENSHIP

Table 1: Frederickson model for citizen vs. administrative participation in government

Frederickson concluded that public administrator’s must take unapologetic leadership


in making American public institutions more reflective of the communal values of

3|Page
justice and equity that are our heritage. He cautioned that the problem is in
balancing the needs and interests of individuals and groups on the one hand with the
community on the other.

Complementarities: political-administrative relations

Svara stated that political superiors and administrators were commonly thought of as
strictly separated, but there is considerable evidence that they interact extensively in a
complementary relationship with each providing important contributions to the other.
In nonprofit organizations, the interaction and shared involvement is widely
recognized. This model is in contradiction to the classic politics-administrative
dichotomy. Svara and Brunet list 3 characteristics that demonstrate
complementarities:

1. political superiors and administrators maintain distinct perspectives based


upon unique values and differences in formal positions,
2. officials have partially overlapping functions as political superiors provide
political oversight of administration and administrators are involved in policy
making, and
3. there is interdependency and reciprocal influence between political superiors
and administrators; each impacts the other.

The balance between the two sets of officials depends upon administrators meeting
certain obligations including:

1. should support the law, respect political supremacy, and acknowledge the need
for accountability. They should be loyal to the mission of their organization,
2. since administrators serve the broadest of public interests, it may bring them
into conflict with political superiors and segments of the public,
3. take responsibility for their actions,
4. should be independent with a commitment to professional values and
competence,
5. should be honest in their dealings with elected officials and deal ethically,
6. should encourage political superiors to fulfill their responsibilities.

Democracy

4|Page
Raadschelders said that democracy as rule (in Greek, krateoo) by the people (in Greek,
demos) is institutionalized as a particular division of labor between public and private
institutions as well as a division of labor between public servants and citizens. The
first division of labor refers to the balance between government intervention and self-
governance. The second concerns the balance between direct citizen participation and
indirect citizen participation. A representative democracy requires the voluntary
support of all its citizens. The citizenry is the largest possible clientele a government
can have and thus the challenge is meeting its varying needs. In contrast, a
dictatorship or government by a few (aristocracy, oligarchy), only concerns itself with
the group of people that are considered to be relevant to the government’s legitimacy.

The politics-administration dichotomy

Wilson first advanced the politics-administrative dichotomy. The reform movement


argued that public appointments should be based upon fitness and merit, rather than
partisanship, necessitating that “politics” was out of place in public service. Wilson’s
other main themes was that public administration should be premised on a science of
management (productivity and efficiency) and separate from traditional politics.

Goodnow suggested that modern administration presented a number of dilemmas


involving political and administrative functions that had supplanted the traditional
concern with the separation of powers among the various branches of government.
Politics and administration could be distinguished, he argued, as “the expression of
the will of the state and the execution of that will.” He admitted that when the
function of political decision making and administration was legally separated, there
developed a “tendency for the necessary control to develop extra-legally through the
political party system.”

Internalized/External control: complexities of accountability

Jensen and Meckling defined an agency relationship under which one or more persons
(the principal(s) engage another person (the agent) to perform some service on their
behalf which involves delegating some decision-making authority to the agent. If both
parties to the relationship are utility maximizers there is good reason to believe that
the agent will not always act in the best interests of the principal. The principal can
limit divergences from his interest by establishing appropriate incentives for the agent

5|Page
and by incurring monitoring costs designed to limit the aberrant activities of the
agent. In addition, he may pay the agent to expend resources to guarantee that he will
not take certain actions that would harm the principal or to ensure compensation if he
is harmed. “Residual loss” is the dollar reduction in welfare experience by the
principal due to divergence of interests.

Governance

Frederickson said that many scholars use the term governance as a companion or
surrogate for public administration. Salamon made the distinction that traditional
public administration focuses on management and control of government agencies
while the newer forms of public action involve elaborate partnership arrangements
with nongovernmental actors. Where traditional public administration stressed
hierarchical lines of authority and the mechanisms of command and control, the new
forms of action utilize decentralized modes of operation and the techniques of
bargaining and persuasion.

New Public Administration (NPM)

New public administration emerged from the Minnowbrook Conference on New Public
Administration. Frederickson stated that new public administration added social
equity to the classic objectives and rationale for public administration of efficient,
economical, and coordinated management of education, police, public health, fire
departments, welfare, diplomacy, the military, etc. The movement flowered briefly
during the late 1960s and early 1970s distinct from new public management. Hood
pointed that NPM was an international trend in public administration that began in
about 1975. NPM’s rise was linked to four other megatrends:

1. attempts to slow down or reverse government growth,


2. the shift to privatization and quasi-privatization,
3. the development of automation, particularly information technology, and
4. the development of a more international agenda.

Hood stated that NPM is a loose term but there are seven doctrinal components of new
public management :

1. Hands-on professional management


2. Explicit standards of and measures of performance
6|Page
3. Greater emphasis on output controls
4. Shift to disaggregation of units in the public sector
5. Shift to greater competition in the public sector
6. Stress on private-sector styles of management practice
7. Stress on greater discipline and parsimony in resource use.

NPM efforts were mainly in the direction of cutting costs and doing more for less as a
result of better-quality management and different structural design.

Professionalism

Frederickson suggested a model that relates citizenship to professionalism in public


administration:

High Citizenship High C High P


High Professionalism
Characterized by
Characterized by living
living with
with sensitivity to
sensitivity to the
professional standard
consequences of
exempt from public and
interactions with
community standards
others
Trusteeship
Civic friendship

Low Citizenship Low C Low P


Low Professionalism
Characterized by
the pursuit of self- Characterized by living
interest and with less regard for
competitive professional standards
individualism
Technocracy
Entrepreneurship

Table2: Frederickson model - citizenship to professionalism in public administration

Rational actor

7|Page
[theory of decision-making in government, that the foundation of a democracy is that of
an informed citizenry that makes decisions based upon rational logic]

Simon said that there is a triangle of limits that bounds the area of rationality
(“bounded rationality” ) of an individual in making decisions. The individual is limited
by:

1. skills, habits and reflexes which are no longer in the realm of conscious (may be
limited by speed of mental processes, elementary arithmetic, etc.),
2. values and those conceptions of purpose which influence him in making
decisions, and
3. extent of his knowledge of things relevant to his job (both fundamental
knowledge and the ability to apply it to any given situation).

References

Frederickson, H.G. (1971). Toward a new public administration. Excerpted from Toward a new
public administration: The meadowbrook perspective by Frank E. Marini. Chandler Publishing
Company.

Frederickson, H.G. (1997). The spirit of public administration. San Francisco, CA:Jossey-Bass.

Frederickson, H.G. & K. B. Smith (2003). The public administration theory primer. Boulder,
CO:Westview Press.

Goodnow, F. J. (1900). Politics and administration:A study in Government. NY:Russell &


Russell. Reprinted in Classics in Public Administration.

Hood, C. (1991). A public management for all seasons? The rise of new public management
(NPM). Public Administration (69). (pg. 503 of Classics of Public Administration)

Peters, B.G. (2001). The future of governing. Lawrence, KS:Univ Press of Kansas.

Raadschelders, J.N. (2003). Government: A public administration perspective. Armonk,


NJ:M.E. Sharpe.

Salamon, L. M. (1989). Beyond privatization: The tools of government action. Washington, D.C:
Urban Institute Press as quoted in Frederickson: The spirit of public administration.

Svara, J. (2007). Ethics primer for public administrators in government and nonprofit
organizations. Sudbury, MA: Jones and Bartlett.

8|Page
Weber, M. (1946). Bureaucracy. In From Max Weber: Essays in Sociology, ed. H.H. Gerth and
C. Wright Mills. Oxford University Press. In Classics of Public Administration.

Wilson, W. (1887). The Study of Administration. Political Science Quarterly 2 (June) as


reprinted in Classics of Public Administration.

9|Page
1.2 History of world’s public admin in a jiffy

1491 B.C. Moses: During the exodus from Egypt Moses followed the recommendation
of Jethro, his father-in-law, that he delegate authority over the tribes of Israel along
hierarchical lines.

400 B.C. Plato: Recognized management as a separate art; promoted principles of


specialization.

325 B.C. Alexander the Great: Applied the principle of line and staff to help conquer
most of the known world.

284 A.D. Diocletian: First Roman emperor to rule through genuine delegation of
authority and chain of command. He divided the empire into 101 provinces, grouped
into 13 dioceses; the dioceses, in turn, were organized into four major geographic
divisions.

1494 Pacioli: Invented double-entry bookkeeping.

1525 Machiavelli: Recognized the need for consent and cohesiveness in an effective
organization and tried to identify leadership traits.

1776 Adam Smith: Began his great economic work, Wealth of Nations, by discussing
the principle of specialization.

1789 George Washington: Began what evolved into veterans preference by selecting
many of those who had served in the Revolutionary War to fill civil service positions in
the new government.

1801 Thomas Jefferson: Began the spoils system in U.S. Government employment.

1810 Robert Owe: Recognized need for training workers and other personnel
practices.

1829 Andrew Jackson: Extended the spoils system in U.S. Government employment.

10 | P a g e
1850 John Stuart Mill: Explained concepts such as span of control, unity of
command, and wage incentives.

1856 Daniel C. McCallum: On October 5, 1841 two American passenger trains


collided head-on, making it clear that one boss could not watch everything. A well-
defined organizational structure was needed, and McCallum developed the
organization chart to show that structure.

1883 Pendleton Act: Curbed the spoils system and established the U.S. Civil Service
Commission.

1887 Woodrow Wilson: While still a practicing political scientist, Wilson called for
public administration to focus on effectiveness and efficiency - not just personnel
reform.

1900 Frederic Taylor: The "Father of Scientific Management" recognized the need for
labor-management cooperation, for controlling costs, and analyzing work methods.

1919 Boston Police Strike

1921 Budget and Accounting Act: Was passed by Congress, creating the Bureau of
the Budget (now Office of Management and Budget) and the General Accounting
Office.

1922 Max Weber: The German sociologist articulated the classical definition of the
bureaucratic form of organization. (Was not translated and published in the United
States until after World War II.)

1923 Classification Act: Began the rationalization of position classification in the


federal service.

1927 Elton Mayo: Began the famous management study at the Hawthorne Works of
the Western Electric Company near Chicago which examined the relationship between
work environment and productivity. These studies were the genesis of the human
relations school of management thought.

11 | P a g e
1930 Mary Parker Follet: Developed a management philosophy based on individual
motivation and group problem solving - a forerunner of the participatory management
idea.

1937 Brownlow Committee: Otherwise known as the President's 1937 Committee on


Administrative Management and composed of Louis Brownlow, Charles Merriam, and
Luther Gulick, made sweeping recommendations for the reorganization of the
executive branch of the U.S. Government.

1937 Luther Gulick and Lyndall Urwick: Provided the definitive statement of the
"principles" approach to management: planning, organizing, staffing, directing,
coordinating, reporting, and budgeting (in short, POSDCORB).

1938 Chester I. Barnard: Viewed organizations as cooperative systems in which the


"functions of the executive" (title of his classic work) were to maintain a balance
between the needs of the organization and the needs of the individual and to establish
effective communication.

1939 American Society for Public Administration (ASPA): A national professional


organization "to advance the science, processes, and art of public administration" was
organized.

1940 Robert K. Merton: Proclaimed that bureaucracy, which Weber (1922) had
defined so systematically, had a number of dysfunctions (that is, characteristics that
lead to inefficiency).

1943 Abraham H. Maslow: Developed a theory of human motivation in which men


and women moved up or down a needs hierarchy, as each level was satisfied or
threatened.

1946 Paul Appeleby: Asserted that processes in government organizations are


political - at least more than those in business organizations. Philip Selznick, Norton
Long, and other writers of the late 1940's were to add theoretical and empirical
support to Appeleby's most un-Wilsonian (1887) thesis.

12 | P a g e
1947 Herbert A. Simon: In his classic Administrative Behavior, Simon, like Merton
(1940), attacked the “principles” approach to management as often being inconsistent
and inapplicable. Like Barnard (1938) and influenced by him, Simon advocated a
systems approach to administration and the study of decision making.

1949 Norbert Wiener, Claude Shannon and P.M.S. Blackett: Emphasized systems
analysis, operations research, and information theory in management.

1955 Herbert Kaufman, Fred W. Riggs and Walter R. Sharp: First course on
comparative administration introduced at Yale University. This movement, which
represented a broadening of public administration to other cultures, began to wane in
later years as American foreign aid programs were scaled back.

1957 Chris Argyris and Douglas McGregor: Placed emphasis on social psychology
and research in human relations in achieving a better fit between the personality of a
mature adult and the requirements of a modern organization. Argyris developed an
open-system theory of organization, while McGregor poplarized a humanistic
managerial philosophy.

1959 Charles A. Lindblom: In his influential essay, "The Science of Muddling


Through," Lindblom attacked the rational models of decision making in government.
In reality, the model did not work; decision makers, therefore, depend heavily on
small, incremental decisions.

1961 Aaron Wildavsky: In an article, "The Political Implications of Budgetary


Reform," Wildavsky developed the concept of budgetary incrementalism and its
political nature that led to his landmark work, The Politics of the Budgetary Process.
(1964).

1962 President Kennedy: Issued Executive Order 10988 which permitted


unionization and collective bargaining in the federal service.

1964 Civil Rights Act of 1964: Title VII prohibited discrimination on the basis of
race, creed, color, sex, or national origin in private-sector employment (would be
applied to the public sector in 1972).

13 | P a g e
1964 Robert R. Blake and Jane S. Mouton: Proposed that every leader could be
categorized in terms of two variables: concern for task and concern for people. Blake
and Mouton's Managerial Grid was perhaps the best known of dozens of adaptations
of this idea, which could be traced back to the Ohio State University leadership
studies of the 1940's.

1965 Charles J. Hitch and Roland N. McKean: In the same year that President
Johnson ordered Planning-Programming-Budgeting Systems (PPBS) adopted
government wide, the "bible" of government systems analysis appeared: The
Economics of Defense in the Nuclear Age.

1966 Equality of Educational Opportunity: The Coleman Report applied the


methods of the social sciences to the analysis and evaluation of government programs.

1967 Anthony Downs: Applied economic principles to develop propositions to aid in


predicting behavior of bureaus and bureaucrats. A forerunner of the “public choice”
approach to decision making.

1967 Yehezkel Dror: Pioneered in the development of policy sciences (that is, the
analysis of the anticipated effects of a public policy and the design of better
policymaking institutions in government).

1967-73 The New Jersey Graduated Work Incentive Experiment:


First large-scale social experiment ever conducted in the U.S. This experiment
spanned 6 1/2 years (1967-1973) and cost eight million dollars.

1968 Dwight Waldo: Under the patronage of Waldo, some young scholars gathered to
critique American public administration for ignoring values and social equity and
accepting too readily the status quo. This movement was known as the "New Public
Administration".

1971-72 Alice Rivlin and Carol Weiss: Provided a comprehensive analysis of the
methodologies and difficulties of evaluating public programs in a dynamic political
environment. Since that time, the importance of evaluation has grown rapidly.

14 | P a g e
1972 Equal Employment Opportunity Act: Amended and applied Title VII of the
Civil Rights Act of 1964 to the public sector and authorized the use of "affirmative
action" to remedy the results of past dsicrimination.

1972 Griggs v. Duke Power: In this landmark opinion based on Title VII of the Civil
Rights Act of 1964, the United States Supreme Court ruled that any factor used in an
employment decision must be a bona fide occupational qualification (BFOQ) related
to the actual performance of the work.

1976 Peter F. Drucker: Addressed the problems of using management-by-objectives -


a process of mutual goalsetting between employee and supervisor for purposes of
planning and evaluation - in the public sector.

1978 Civil Service Reform Act: Significantly reorganized the Federal Civil Service.

1978 Proposition 13: Was adopted by California's voters by referendum; limited that
state's ability to levy property taxes and began what has come to be called the
"taxpayers' revolt."

1978 Regents v. Bakke: In its first major decision on affirmative action, the United
States Supreme Court ruled that race could be a factor but not the factor in university
admissions policies. This principle was later extended to employment and gender.

1980s A good way to characterize the study of public administration in the U.S. today
is in terms of three impulses: politics, management, and public policy.
University programs emphasizing politics tend to be found in departments of political
science or separate schools of public administration (e.g., Syracuse). Programs
emphasizing management tend to be found in schools of business (e.g., Stanford) or
administration (e.g., Yale and Cornell). And programs emphasizing public policy tend
to be found in schools of public affairs (e.g., Harvard and Texas). One should not view
any of these three impulses as a panacea to replace faded predecessors. To
understand better how public agencies do and should operate, one should try to blend
insights from all three approaches.

15 | P a g e
1990 Americans with Disabilities Act: Extended anti-discrimination protection to
persons with disabilities.

1991 Civil Rights Act of 1991: Attempted, inter alia, to clarify and limit certain
recent decisions of the Supreme Court that were interpreted as hostile to affirmative
action.

1993 Osborne and Gaebler publish Reinventing Government in an attempt to


"empower government officials to bring business technologies to public service."

1997 The fourth edition of Simon's classic Administrative Behavior is published on the
50th anniversary of the first.

STUDENTS: Remember what a "highlight" is. What appears above is, by no means, all you
need to know; it is merely a starting place.

16 | P a g e
1.3 Different theories and their time frame

Classical Organization Theory

 Dominated thought into the 1930s

 Structuralists--focused attention on structure or design of orgs

 Rational and closed systems pursuing the goal of efficiency

 Adam Smith, Henri Fayol, Daniel McCallum, FW Taylor, Max Weber, Gulick
&Urwick

 Organizations should work like machines, using people and capital as their
parts

 Daniel McCallum, 1856, first modern organization chart for the NY and Erie RR
Company

 Fayol's organizational principles: technical, commercial, financial, security,


accounting, managerial (greatest emphasis on managerial)

 Taylor's "one best way"

 Gulick & Urwick's POSDCORB; organize work by purpose; process; persons or


place

 Often viewed as narrow and simplistic; however, laid a foundation for all future
scholars

Neoclassical Organization Theory

 Transitional theory that revised Classical Theory by adding human element,


1950

 An organization cannot exist outside of its environment

 James March, Philip Selznick, Chester Barnard and Herbert Simon

 Barnard: individuals are what hold the organization together; thus, they must
be reduced to cooperate for success to be achieved (persuasion principle)

 Simon: openly and vehemently attacked Classical Theory, said G&U's


principles were merely proverbs, offered idea of satisfice

17 | P a g e
 Selznick: idea of cooptation

 Cyert and March: alliance-forming and coalitions

"Modern" Structural Organization Theory

 Second half of 20th Century

 Hierarchy, formal rules in place to attain goals

 Sought a return to the Structural Element, retaining the human aspects of


Neoclassicism

 "Modern" in quotations b/c it is used simply to refer to the time period--there is


little substantive difference between the Structuralists in Classical Theory and
this one, other than time frame

 Organization efficiency is the essence of organizational rationality, and the goal


of rationality is to increase the production of wealth in terms of real goods and
services

 Peter Blau and Richard Scott: all orgs consist of a formal and an informal
element and it is impossible to understand an org. without knowing each
element

 Buzz words: differentiation, specialization and integration

 Elliott Jaques: remains a lonely defender of the bureaucratic-hierarchy models

Systems Theory

 Rose to dominance in the late 1960s

 Daniel Katz and Robert Kahn: organizations are open systems

 Apply Ludwig Burtalanffy's general systems theory to organizations and use


quantitative tools and techniques to understand complex relationships among
organizational and environmental variables

 (remember inputoutput / blackbox diagram)

 search for order in complex systems, cause-and-effect oriented

 seeks optimal solutions (not "one best way")

18 | P a g e
 computers, experts, etc. are the tools necessary

 draw heavily from Neoclassicals---bounded rationality and satisficing (Simon)


and cognitive limits (Cyert and March)

 Richard Scott – rational, natural and open systems theories

Power and Politics Organization Theory

 organizations are viewed as complex systems of individuals and coalitions

 conflict in inevitable and influence is the primary weapon

 organizational goals change with shifts in the balance of power

 John Kotter: differentiate between power resulting from authority and power
resulting from being able to get job done

 Power is aimed in all directions, not just down the hierarchy

 Jeffrey Pfeffer: power and politics are fundamental concepts in defining an org

Organizational Culture

 Late 1960s--thru 70s and 80s

 Organizational culture assumes many organizational behaviors and decisions


are in effect predetermined by the patterns of basic assumptions that are held
by the members of the org.

 An org's behavior, cannot be understood and predicted by studying structural


or systemic elements but by studying its organizational culture

o Meaning (reality) is established by and among the people in organizations


(the org, culture)

o Things are not real, perceptions of them are

o People will distort the perceptions of symbols according to the need for
what is symbolized

 In the 80s, organizational culture began appearing in notable works (incl.


Thomas Peters and Robert Waterman, John Corbelly, Vijay Sathe, and Gareth
Morgan)

19 | P a g e
 TQM and "Reinventing Government" further thrust this movement onto front
pages in the 1980s and 90s

Postmodernism

 Technology and information networks have led to uncertainty and chaos is this
postmodern era

 Just as information is readily available, so is misinformation

 We are frequently seeing that we do not understand cause-effect relationships


despite abundance of information; thus, managers are abandoning their
"modern era" reliance on technical systems, turning instead to information
technology to help them into the postmodern era

 Karl Weick

o Technical system: specific set of hardware and software systems that


produce a desired outcome, products of the "modern era," designed to
accomplish desired purposes using known information and existing
technologies

o Technology: refers to the knowledge of cause-and-effect relationships


embedded in machines and methods

 Old, familiar machine analogies no longer apply. Berquist (1993), McWhinney


(1997) and Prigogine and Stengers (1984) suggest fire is most appropriate
analogy for postmodern organizations

1. fire is a second-order change process that is irreversible

2. fire is ephemeral

 Organizations must wrestle with dilemmas about how much to participate in


the information age

o Centralize or decentralize

o Outsource or produce internally

o "Regular" employees or "stringers"

20 | P a g e
o Sell products or deliver services through established networks or through
the web

o How to maintain what boundaries

Human Relations School (also, Organizational Behavior)

 People are considered to be as important, or more so, than the org itself

 Bolman & Deal (1997): organizations exist to serve humans (not the other way
around)

 Ott: themes are motivation, group behavior, leadership, empowerment

 Hawthorne Effect, Maslow's Hierarchy of Needs, McGregor's Theory X & Y,


Janis' Groupthink

 Most optimistic of all schools - under right circumstances, people and


organizations will grow and prosper together

Table3: Different theories summary

21 | P a g e
1.4 Names and theories in public administration in chronological order

Jefferson, Thomas Weak executive – bottom-up structure

Hamilton, Alexander Strong Executive branch – top-down structure

1887 Wilson, Woodrow Hierarchical-admin matters and politics shouldn’t


interfere – political neutrality and professionalism

Progressive Era Progressive reform movement

1900 Goodnow, Frank Politics and administration – expression of will of


the state and the execution of that will

1911 Taylor, Frederick Principles of Scientific Mgmt.

Methods to manage & measure individual work


elements

Study to ensure optimal placement of workers

Obtain cooperation of workers

Establish logical division between workers & mgmt.

1918 Willoughby, Wm Legislative admin school – Constitution gave admin


power to legislative branch – movement for Bud
reform

1922 Weber, Max Structural definition of bureaucracy – “ideal type”

1926 White, Leonard Defined PA as mgmt of men & materials in the


accomplishment of the proposes of the state

First textbook – Admin is a art but worthy of being


transformed

Into a science – admin is legitimized by law but is


action

22 | P a g e
1926 Follett, Mary P Participatory mgmt – power with not over

Circular behavior – feedback to sender to resolve


problems

1937 Brownlow, Louis President needs help to perform duties

 Expand White House staff

 Develop arms – budget, research, personnel


& planning

 Merit system revamped

 Centralize power under few large


departments

 Revise fiscal system

1937 Gulick, Luther Principles approach to managing orgs – POSDCORB

Looked at role of upper level mgmt.

1938 Barnard, Chester Informal orgs & their relation to formal orgs

Function of executives to maintain equilibrium


between needs of org & needs of employee

Functions of informal orgs, communication,


cohesiveness & p integrity

1943 Maslow, Abraham Theory of Human Motivation – Hierarchy of needs

1947 Simon, Herbert Logical positivism (logical reasoning & empirical


experience) –

Decision-making at true heart of admin

Bounded rationality – Humans not optimal

23 | P a g e
1947 Dahl, Robert Behavioralist in Pol Science – for PA be accepted as
a science need to:

 Recognize the complexities of human


behavior

 Deal with normative values in admin


situations

 Be comparative to the social setting

1948 Waldo, Dwight The Admin State-Economy and efficiency are


ultimate admin values

Critiqued principles of admin & the dichotomy

Headed Minnowbrook conference – PA not a


science-a profession

Populist concerned with values of democracy & the


Philpot of PA

PA is politics – partisan politics in PA is good

1949 Selznick, Philip The Cooptative Mechanism – org brings in new


elements into policymaking process in order to
prevent them from becoming a threat to the org or
its mission.

1957 Merton, Robert Bureaucratic structure & personality – a formal


rationally organized social structure involves clearly
defined patterns of activity in which every series of
actions functionally related to the purposes of the
org

1957 McGregor, Douglas Human Side of Enterprise – Mgmt assumptions


about employees – become self-fulfilling prophecies

24 | P a g e
Theory X – humans dislike work- will avoid – must
be coerced, controlled, directed, threatened

Theory Y – humans do not inherently dislike work –


source of satisfaction, seek and accept
responsibility

1966 Katz & Kahn Organizations & the Systems Concept– systems
framework for how organizations operate

Open systems = modern organizational


environments

Closed systems = traditional fixed bureaucracies

1967 Downs, Anthony Public Choice Theory – application of market logic


to PA

Life cycles of bureaus – difficult to kill a


bureaucracy

1969 Kaufman, Herbert Administration decentralization & political power

power to people not best if interference with pursuit


of national mandates for economic & social equity

1971 Allison, Graham PA is Bureaucratic politics – pluralism – Differences


in public & private mgmt

pub=planning related to political expediency

private=longer time perspective

1972 Cleveland, Harlan More governance needed – nonprofit contracting,


voluntary & other orgs used to deliver govt. services

1973 Ostrom, Vincent Intellectual crisis in PA- growing gulf between


theory & practice

25 | P a g e
Public choice theory or application of market logic
to PA

Org dynamics understood by analyzing functional


process

1977 Meyer & Rowan Institutional Theory – emphasize cultural &


institutional environmental influences – socially
constructed practices/norms

provide framework for creation of formal orgs

1978 Pfeffer & Salancik Resource Dependency Theory – All orgs exchange
resources with environ to survive

1979 Mintzberg, Henry Five basic parts of org – Mgmt. Policy Theory
=model of org w/ 5

interdependent parts: strategic apex, middle line,


operating core, techno-structure & support staff

1983 – Power Game & Players – influencers (external & internally


coalitions) that attempt to control org decisions –
must know who are to understand org behavior

1980 Lipsky, Michael Street level bureaucrat – delivery of services has


great impact

1981 Ouchi, Wm. Z Org. – Orgs proceed smoothly only w/intimacy,


subtlety & trust foster close interchange between
work & social life

1983 Rosenbloom, David Compared managerial, political & legal approaches


to PA and showed how each has separate values,
origins & structures

Collapse of admin powers in modern admin state

26 | P a g e
1987 Moe, Ronald Limits of privatization: - public admin intellectual
roots in public law

tradition not in free market economy causes


problems w/definitions

For example: FADA – private in direction but public


in rights

Concept of sovereignty must be considered –present


legal perspective

1992 Osbourne & Gaebler Reinventing Government – dichotomies of


entrepreneurial & bureaucratic approaches to PA –
Entrepreneurial=Governance

1993 Kettl Managing vs Governing – govt.’s roles of supervisors


of proxies

Public-private partnerships, contracts replacing


hierarchy

1993 Schein, Edgar Defining Org Culture – process of socializing


employees into existing org culture – dominant
attitude and accepted ways of actions

1993 Denhardt, Robert Critical Theory of PA

1995 Holzer, Mark TQM – innovative repackaging of several decades of


public sector productivity improvement – draws on
decades of industrial quality improvement in
private sector

1995 March & Olsen Democratic Governance – two perspectives on


governance

1. Exchange perspective – rational actor

27 | P a g e
assumptions, competition, markets,
bargains, winning coalitions, changed policy
Pareto-

2. improving criteria=at least one person better


off & no one worse off

1996 Frederickson, George Spirit of PA linked to New Public Mgmt. – New PA


calls for second generational behaviorism – admin
more responsive to public, more proscriptive, more
client-oriented-boundary exchange, more
normative, still scientific – Social equity necessary –
rep democracy too slow

28 | P a g e
1.5 Rethinking the identity of public administration

The world of government and public administration has traveled far since the early days
of its struggle for disciplinary independence. Lately, there has been talk of the advent of a
new spirit in the public sector, or at least expectations of its coming. Some say that such
a spirit is already here. Others aver we are witnessing only the tip of change. The world
wide globalization process supported by stronger orientations towards open markets,
open highways of information, growing levels of organizational learning and
interdisciplinary in the social sciences have also made their impact on the study of our
bureaucracies. Yet by all definitions public administration in the beginning of the 2000s
still lacks the sense of identity that other fields of the social sciences have long since
obtained. In other words, the field is looking back and down into its individuality,
searching for orientations and signs that can direct it on its way forward. Today, public
administration is already very different from what it used to be forty, thirty, and even
twenty or ten years ago. In the coming years it is going to be even more different.

This paper is based on a previous work by the author (Vigoda, 2002). It tries to portrait
the uncertain identity of public administration and possible developments waiting ahead.
Rethinking this identity we should be interested in two main questions: Which scholarly
ground are we stepping on when we talk about public administration? What is the legacy
of the field in its current phase and what are its ambitions for the future? Naturally,
these questions raise many others, for example, how to improve governments actions;
how to revitalize public administrations services; whether bureaucracies are responding
to economical/ social/ political challenges and changes ahead, and with what tools;
what is the impact of a high-technology environment and the information age on our
public agencies; how to attain the (im)possible goal of effective integration between
citizens and governments in an ultra-dynamic society; and what are the implications of
such transitions for democratic governments, their stability, and legitimization in the
eyes of citizens. I argue that in order to resolve these questions one should seek better
scholarly identity, which may be acquired through interdisciplinary analysis.

Practically, such an analysis needs to be presented gradually. Hence, I first suggest a


theoretical entry and rationality for the mixture of analytic levels, methods, and
viewpoints that are proposed by the various mother-disciplines of public administration.
More specifically I focus on the roots and foundations of public administration in both

29 | P a g e
American and Non-American cultures that furnish the background and terminology for
the discipline in its basic frame, as well as in its more advanced composition. Next, three
academic origins are discussed, namely (1) policy, politics, and political economy, (2)
sociology, culture, and community and (3) management and organizational studies. Each
represents a separate layer of investigation. The closing section suggests a synthesis and
looks to the future. It attempts to portray areas and orientations for the new generation
of public administration and for its way forward.

The dilemma of independence and interdisciplinary in public administration

For many years public administration has struggled for its independent position in the
social sciences. While in its early years it was part of the more conservative fields of
Law, Politics, and Economy, it has been developed today to a unique field,
independent in many ways but still enjoying mutual contributions of other disciplines
in the social sciences. Moreover, in the last century it has developed a theoretical but
also an impressive practical agenda that created remarkable achievements in different
ways. The public sector, both as a science and as a profession is responsible for much
of these achievements.

At the dawn of the new millennium, however, various new social problems still await
the consideration and attention of the state and its administrative system. The
question of independency of public administration as a science seems today less
important than in the past. Instead, there are many calls to take advantage of multi
disciplinary orientations in the social sciences and to find better ways to integrate
them in the current ethos of public administration. It is also suggested that such
interdisciplinary ideas, tools, and methods can help to overcome social problems and
create effective remedies for the new type of state maladies. Interdisciplinarity is also
translated into is cooperation, collaboration, and a share of information and
knowledge. The multi-level, multi-method, and multi-system analysis with a look
towards the future is the main frontiers of modern public administration.

The interdisciplinary view endeavors to provide an insight into the complexity of the
field by combining different levels of analysis into an integral whole, which better
accords with reality. This knowledge may well serve our understanding of how the
state, and its executive branches, is managed and of the obstacles to better public
performance. An important task is to illuminate cross-disciplinary principles for

30 | P a g e
greater effectiveness and efficiency of public management in future generations, when
environmental pressures will grow, together with an increase in citizens’ demands and
needs. An interdisciplinary approach to public administration may thus be of merit for
a contentious field in a state of rapid change. It may stimulate new and viable thinking
that can lead to additional positive innovation in the old type of bureaucracies.

The central assumption of this essay is that slowly and gradually, but constantly and
extensively, a change is being nurtured in public systems and in the attitudes of
public managers, politicians, and citizens to the conservative role of public
institutions. These transformations carry many challenges, as well as risks, that
citizens, governments, administrators of the future will have to confront and address.
They all represent new alternatives for the evolution of public administration as an art,
perhaps also as a science and as a profession (Lynn, 1996). Our task, as stemming
from such a perspective, is to understand better the changes ahead, which have the
potential of building bridges into the future of modern democracies. A core
assumption, as will be developed here, is that this goal can be achieved only through
cooperation among the public, private, and third sector organizations that collaborate
through mutual efforts and combined knowledge available in all the relevant social
sciences.

Public administration in transition: the evolutionary process

The foundations of modern public administration can be discerned thousands of years


ago, across cultures, and in various nations around the globe. The Bible mentions a
variety of hierarchical and managerial structures that served as prototypes for
governance of growing populations. Ancient methods of public labor distribution were
expanded by the Greeks and the Romans to control vast conquered lands and many
peoples. The Persian and Ottoman empires in the Middle East, like India and imperial
China in the Far East, and the Mesoamerica cultures paved the way for public
administration in the modern age, wherein European Christians, and later Christians
of the New World, were in the ascendant.

All these, as well as other cultures, used a remarkably similar set of concepts, ideas,
and methods for governing and administrating public goods, resources, and interests.
They all employed professionals and experts from a variety of social fields. They all
used authority and power as the cheapest control system for individuals,

31 | P a g e
governmental institutions, and processes. All of them faced administrative problems
close in type and in nature to problems of our own times: how to achieve better
efficiency, effectiveness, and economy in government, how to satisfy the needs of the
people, and how to sustain stable political hegemony despite the divergent demands
and needs of sectarian groups. Not surprisingly, all the above cultures and nations
also used similar managerial tools and methods to solve problems of this sort. They all
used, fairly effectively, division of work, professionalism, centralization and
decentralization mechanisms, accumulation of knowledge, coordination of jobs,
complex staffing processes of employees, long-range planning, controlling for
performance, and so on. Intuitively, one feels that nothing has really changed in the
managerial and administrative process of public organizations for centuries, possibly
millennia. But this feeling is of course exaggerated. Some major changes have taken
place in recent centuries to create a totally different environment and new rules, to
which rulers and citizens must adhere and by which they must adjust their operation.
In fact, a new kind of governing game has taken shape, in which public administration
plays a central role.

Despite basic similarities, the public administration of our times is entirely different
from public services in the past. These differences can be summarized in 7 key points:

(1) It is larger than ever before, and it still expanding;

(2) It is more complex than in the past, and becoming increasingly so by the day;

(3) It has many more responsibilities to citizens, and it still has to cope with increasing
demands of the people.

(4) It is acquiring more eligibilities, but must restrain its operation and adhere to
standards of equity, justice, social fairness, transparency and accountability.

(5) Modern public administration is considered a social science, a classification that


carries high esteem but also firm obligations and rigid constraints.

(6) For many individuals who decide to become public servants it is also a profession
and occupation to which they dedicate their lives and careers.

(7) Public administration is one of the highly powerful institutions in modern


democracies.

32 | P a g e
Thus, it is evident that public administration of our time wields considerable power
and influence in policy framing, policy making, and policy implementation. Hence it is
subject to growing pressures of political players, social actors, managerial
professionals, and the overall economic market.

An eclectic science

Public administration is an eclectic science. It was born towards the end of the 19th
century when the business of the state started to attract social-academic attention.
The revolution turning public administration into an independent science and
profession is traditionally related to the influential work and vision of Woodrow Wilson
(1887) and Frank J. Goodnow (1900). These scholars were among the first who
advocated the autonomy of the field as a unique area of science that drew substance
from several sources. In the first years, law, political theory of the state, and several
“hard sciences” such as engineering and industrial relations were the most
fundamental and influential mother disciplines. Over time, these fields strongly
influenced the formation and transition of public administration but the extent and
direction of the influence were not linear or consistent.

Kettl and Milward (1996:7) argued that traditional public administration, as advocated
by the progenitors of the discipline, consisted in the power of law. Representatives of
the people make the law and delegate responsibility to professional bureaucrats to
execute it properly. Highly qualified bureaucrats, supported by the best tools and
resources, are then expected to discharge the law to the highest professional
standards, which in return produces good and accountable managerial results that
best serve the people. According to Rosenbloom (1998), the legal approach views
public administration “as applying and enforcing the law in concrete circumstances”
and is “infused with legal and adjudicatory concerns” (p.33). This approach is derived
from three major interrelated sources: (1) administrative law, which is the body of law
and regulations that control generic administrative processes; (2) the judicialization of
public administration, which is the tendency for administrative processes to resemble
courtroom procedures; and (3) constitutional law, which redefines a variety of citizens’
rights and liberties. Several legal definitions argue that public administration is law in
action and mainly a regulative system, which is “government telling citizens and
businesses what they may and may not do” (Shafritz & Russell, 1997:14). However,

33 | P a g e
with the years it has become obvious that law in itself does not maintain satisfactory
conditions for quality public sector performances to emerge. Constitutional systems
furnish platforms for healthy performance of public administration, but do not
account for its effectiveness or efficiency. Put differently, good laws are necessary but
not sufficient conditions for creating a well-performing public service. They only
highlight the significance of other scholarly contributions.

One such important contribution came from the classic “hard sciences” of engineering
and industrial relations. In its very early stages public administration was heavily
influenced by dramatic social forces and long-range developments in the western
world. The ongoing industrial revolution in the early 1900s, which was accompanied
by political reforms, higher democratization, and more concern for the people’s
welfare, needed highly qualified navigators. These were engineers, industrial
entrepreneurs, and technical professionals who guided both markets and governments
along the elusive paths to economic and social prosperity. Various fields of
engineering, the subsequent evoking area of industrial studies, and other linked
disciplines such as statistical methods became popular and crucial for the
development of management science in general, and were also gradually found useful
for public arenas. The link between general management and public administration
has its roots in the understanding of complex organizations and bureaucracies, which
have many shared features. Here, much contribution was made in non-American
societies such Germany, France, and Britain. In fact, early American public
administration was influenced by the works of various European. Thus, the current
state of public administration can not be covered without adequate understanding of
the seminal works by Max Weber (1947), Henry Fayol (1925), Lindel Urwick (1928),
and others. Their ideas and theoretical development of the field are considered today
as core-stones for the emergence of modern public administration and management.

With time, dramatic changes occurred in the nature and orientation of general
organizational theory, and in its application to public administration of modern
societies. A major transition resulted from the exploration by the Hawthorn studies in
the 1920s and 1930s, conducted by a well known industrial psychologist from Chicago
University, Alton Mayo. A behavioral apparatus was used to drive a second revolution,
beyond the revolution of its original emergence, which swept the young science into its

34 | P a g e
first stages of maturity. Today, trends and developments in the public sector cannot be
fully understood without adequate attention to behavioral, social, and cultural issues.
These aspects conjoin with questions of policy making and policy evaluation, as well
as with managerial, economic, and organizational contents, better to illuminate public
systems. The human and social side of public organizations became central and
critical to all seekers of greater knowledge and comprehension of the state’s operation.
People and groups were placed at the heart of the discussion on organizational
development and managerial methods. The human side of organizations was made an
organic part of the art of administration. Still today it is an indispensable facet of the
craft of bureaucracy. All who are interested in the healthy future and sound progress
of public organizations and services both as a science and as a profession have to
incorporate humanistic views well in their basic managerial ideology.

However, major transitions still lay ahead. International conflicts during the 1930s
and the 1940s wrought immense changes in national ideology and democratic
perspectives in many western societies. Consequently, public administration and
public policy had to be transformed as well. During the Second World War theoretical
ideas were massively supported by advanced technology and higher standards of
industrialization. These were pioneered by professional managers and accompanied by
new managerial theories. Ironically, the two world wars served as facilitators of
managerial change as well as accelerators and agents of future developments and
reforms in the public sector. The political leaders and social movements of the
victorious democracies were convinced that the time had come for extensive reforms in
the management of western states. The assumed correlation of social and economic
conditions with political stability and order propelled some of the more massive
economic programs in which the state took an active part. The rehabilitation of war-
ravaged Europe involved governmental efforts and international aid, most of it from
the United States. Major attention was dedicated to the creation of better services for
the people, long-range planning, and high-performance public institutions capable of
delivering quality public goods to growing numbers of citizens. To build better societies
was the goal. A larger and more productive public sector was the tool.

In many respects the utopian vision of a better society generated by the post-war
politicians and administrators in the 1940s and 1950s inexorably crumbled and fell

35 | P a g e
during the 1960s and 1970s. A sizable number of governments in the western world
could not deliver to the people many of the social promises they had made. The
challenge of creating a new society, free of crime and poverty, highly educated and
morally superior, healthier and safer than ever before, remained an unreachable goal.
So during the 1970s and 1980s, citizens’ trust and confidence in government, and in
public administration as a professional agent of government, suffered a significant
decline. The public no longer believed that governments and public services could
bring relief to those who needed help, and that no public planning was good enough to
compete with natural social and market forces. The promises of modern
administration, running an effective public policy, seemed like a broken dream.
Political changes took place in most western states, largely stemming from deep
frustration by the public and disapproval of government policies. By the end of the
20th century the crises in public organizations and mistrust of administrators were
viewed both as a policy and managerial failure (Rainey, 1990). In addition, this
practical uncertainty and disappointment with governments and their public
administration authorities naturally diffused into the academic community.
Theoretical ideas for policy reforms in various social fields that once seemed the key to
remedying illnesses in democracies have proven unsuccessful. Within the last decade
the search for new ideas and solutions for such problems has reached its peak, as
premises originally rooted in business management have become increasingly adjusted
and applied to the public sector. Among these ventures are re-engineering
bureaucracies (Hammer and Champy, 1994), applying benchmarking strategy to
public services (Camp, 1998), re-inventing government (Osborne and Gaebler, 1992),
and the most influential movement of New Public Management (NPM: Lynn, 1998;
Stewart and Ranson, 1994). These are receiving growing attention accompanied by
large measures of skepticism and criticism.

Transformations in the academic realm

Throughout those years public administration as an academic field was also in


transition. Today, many examples exist in universities of independent public
administration units; some operate as schools and some as free-standing faculties.
But in at least an equal number of universities, public administration programs on all
levels are only part of larger units such as Political Science departments, Business

36 | P a g e
and Management schools, or even Public Affairs schools. This disciplinary
schizophrenia certainly yields a science that is more complex and heterogeneous, but
also more challenging and full of promise.

The scientific background and identity of public administration in the late 1990s and
early 2000s is still not stable and has not overcome its childhood ailments. On the
contrary identity conflicts have only intensified with the years. Some 30 years ago,
Waldo (1968) noted that ongoing transformations in public administration reflected an
identity crisis of a science in formation. During the last three decades Waldo’s
diagnostics on public administration as a science struggling with a pernicious identity
problem has not changed much. The evolution of alternative sub-disciplines inside
and around the field (e.g., policy studies, public personnel management, information
management, etc.) carried promises but also risks for its position and role as a central
field of social study. As recently noted by Peters (1996), modern public administration
greatly reflects lack of self-confidence both as a science and as a profession. This lack
is expressed in many ways, the most significant being incapacity to guide governments
through a safe circuit of public policy change. Much of the accumulated wisdom in the
science of public administration has been obtained through social experiments, the
commission of policy errors, and sometimes even learning from them about better
ways to serve the people. But mistakes cost money, much money, money from all of
us, the taxpayers. Like good customers in a neighborhood supermarket, citizens
should and have become aware of the services they deserve, of the high prices they are
asked to pay, and of governmental actions that should be taken to produce useful
changes. Demands for better operation are generally aimed at governments, but they
should be, and are, also targeted at the science and at academia. Science has the
potential of exploring new knowledge, generating better explanations for relevant
administrative problems, applying sophisticated and useful professional methods, and
most importantly directing all available resources to produce successful and practical
recommendations for professionals. Its prime goal is to design a comprehensive
theoretical view of public systems that is clear, highly efficient, effective, thrifty, and
socially oriented at the same time. This cannot be achieved without extensive
understanding of the diversity, complexity, and interdisciplinarity of the science of
public administration.

37 | P a g e
The contribution of an interdisciplinary view to public administration identity

In many ways the persistent public mistrust of governmental services and institutions,
together with the marked instability of public administration as a science, inspired us
in the present venture. The fragile status of the theory of public administration is a
port of departure for a different kind of discussion, which is broader and multi-
perceptional. Our core argument is that one can find many ways to depict the
administrative system, its functionality, and its relationship with the public. But the
identity crisis of public administration cannot be solved until many approaches are
combined and coalesce to explain the very basic constructs that modern societies
encounter at the start of the new century. A foremost assumption of this paper is that
only mutual efforts and quality combination of critical knowledge from a variety of
social disciplines and methods can yield a real opportunity for overcoming public
administration’s post-childhood problems. Such a crisis of identity, which has existed
for more than a century now, carries risks, but also promises, which must be well
isolated, assessed, analyzed, and only then fulfilled. The translation of science into
operative acts by government must rely on such wisdom, which can be sufficiently
accumulated from various social branches.

Interdisciplinary heredity

The desired comprehensive understanding of public administration, as portrayed


earlier, should rely on the accumulated wisdom and knowledge of its sister disciplines
(and not necessarily the conventional mother disciplines) in the social sciences.
Unfortunately, so far most writing on public systems has adopted a uni-dimensional
viewpoint. Public administration was frequently understood through the eyes of policy
analysts or political scientists. Alternatively it was considered a specific field of
management science or as an organizational studies domain. While the roots of the
administrative process are definitely, and with much justification, identified with
political science, policy studies, and managerial constructs of public institutions, it
would be greatly in error to point solely to these arenas in portraying the domain and
nature of public administration. An integrative approach has much merit and
potential in this case, and it must be well developed to conform to the complex reality
of serving the public.

More specifically I argue that the administrative science is a discipline in transition

38 | P a g e
that involves politics, but not only politics. It deals with policy, but reaches much
farther and deeper than policy questions. It incorporates sociological and cultural
aspects that change rapidly in a mass communicative global world, but it goes even
beyond these issues. It deals with people as workers, as citizens, as clients, and as
consumers, as leaders and managers, as well as with a variety of other human
constructs that fuse into a unique branch of knowledge. A multi-disciplinary approach
is evidently required to explain better what every scholar already knows from his or
her personal perspective: that the truth about public administration has many faces
and no monopoly exists any longer on the discipline’s status and orientations.

In light of the above we can identify three main disciplines that serve today as core
sources of knowledge in the study of public administration.

(1) Policy analysis, Political science, and Political Economy;

(2) Sociology, Cultural studies, and Community studies;

(3) Management and Organizational studies;

Policy, Politics, and Political Economy

The political approach to public administration was depicted by Rosenbloom (1998) as


stressing the values of representativeness, political responsiveness, and accountability
to the citizenry through elected officials. These values are considered necessary
requirements of democracy, and they must be incorporated into all aspects of
government and administration. Wallace (1978) argued that ultimately public
administration is a problem in political theory. It deals with the responsiveness of
administrative agencies and bureaucracies to the elected officials, and through them,
to the citizens themselves. Shafritz and Russell (1997) provide several politics-oriented
definitions of public administration: it is what government does (or does not do), it is a
phase in the policymaking cycle, it is a prime tool for implementing the public interest,
and it does collectively what cannot be done so well individually (pp. 6-13). Hence it is
impossible to conduct a politics-free discussion of public administration. This political
debate in public administration is also heavily influenced by the sub-field of political
economy. Questions of budgeting and financing the public sector (Wildavsky, 1984) as
well as bringing more economical rationality to decision making processes usually
conflict with political considerations (Jackson & Mcleod, 1982). However they also put

39 | P a g e
them under economical restraints and enhance "checks and balances" to a system
mostly monitored and controlled by politicians, political parties, and other federal or
national institutions, rather than professionals and practitioners.

Yet, politics is definitely the heart of public administration processes. Politics focuses
on citizens as members of groups or on highly institutionalized organizations that
sound the public’s voice before political officials and civil servants. The politics
approach to public administration involves strategies of negotiating and maneuvering
among political parties, public opinion, and bureaucracies. It involves an incremental
change in society, which relies on open debate, a legitimate power struggle,
distribution and redistribution of national resources and budgets, and a heavy body of
legislation and law to regulate these processes. Perhaps the most obvious linkage
between politics and public administration stems from policy making and policy
implementation processes. It is naive to distinguish political systems from professional
administration systems in regard to public policy. As Rosenbloom (1998:13)
suggested, “public administrators’ involvement in the public policy cycle makes
politics far more salient in the public sector than in private enterprise. Public
administrators are perforce required to build and maintain political support for the
policies and programs they implement. They must try to convince members of the
legislature, chief executives, political appointees, interest groups, private individuals,
and the public at large that their activities and policies are desirable and responsive”.

The theoretical contribution of political science to the study of public administration is


therefore multi-faceted. It invokes better understanding of the power relations and
influence dynamics that take place inside and among bureaucracies (Pfeffer, 1992)
and determine their operative function as well as outcomes. It also employs a rather
vast knowledge from economics and rational thinking. Party politics acknowledges
that the investigation of pressure and interest groups, and the better understanding of
conflict relationships among various players of the state, are used to build models of
decision making and policy determination that are rational and realistic. In addition,
political psychology is implemented more thoroughly to explore personality traits of
political leaders as well as public servants. For the same reasons, budgetary studies
and policy analysis methods are an integral facet of the political approach, which
assumes limited rationality as well as high constraints of time and resources on the
administrative process.

40 | P a g e
From a somewhat different perspective, Ellwood (1996: p.51) argued that political
science has simultaneously everything and little to offer public management scholars,
hence also public administration scholars. Everything, because both fields deal with
political behavior, processes, and institutions. Little, because political science deals
only with the constraints forced on the administrative process with no practical
contribution to the managerial improvement of public systems. Ellwood further
concurs that both fields rely on other academic disciplines, employing techniques of
anthropology, economics, game theory, historiography, psychology, and social
psychology, as well as sociology. In line with this it would be only natural to conclude
that the relationship between political science and public administration is described
as an on-again, off-again romance. Kettl (1993, p.409) suggested that “the importance
of administration lay at the very core of the creation of the American Political Science
Association…when five of the first eleven presidents of the association came from
public administration” and played a major role in framing the discipline. As Ellwood
puts it, with the years, public administration became public but also administration. It
shifted its focus to a more practical and client-service orientation, which necessarily
incorporated knowledge from other social disciplines like personnel management,
organizational behavior, accounting, budgeting, and so forth. The methodological
contribution of a political approach to public administration studies is also
meaningful. Here a macro analysis is necessary if one seeks an understanding of the
operation of large bureaucracies and their coexistence with political players. A political
approach delivers these goods by means of comparative studies, policy evaluation
methods, rational choice models, and simulations, as well as content-analysis
techniques and other tools useful for observation of the political sphere.

Sociology, Culture, and Community

Studying public administration is also a social issue. Thus, another approach that is
highly relevant to the understanding of public administration bodies and processes
rests on a sociological apparatus. It has a very close relationship with the political
approach, so it is sometimes defined as a socio-political view of public systems or as a
study of political culture (Shafritz & Russell, 1997:76). Yet its core prospects are
beyond the political context. The voice of society has a special role in the study of

41 | P a g e
public administration arenas not only for democratic and political reasons but also
because of its fundamental impact on informal constructs of reality such as tradition,
social norms and values, ethics, life style, work standards, and other human-cultural
interactions that are not necessarily political.

The theoretical contribution of a sociological and cultural approach to public


administration consists of several elements. An essential distinction must be drawn
between inside and outside cultural environments. An outside cultural sphere
incorporates informal activities and behaviors of small groups as well as of larger
social units which interact with the administrative system. Included in this category
are customers’ groups, private organizations, not-for-profit volunteering organizations,
and citizens at large. Considerable attention has been turned to communities and to
the idea of communitarianism (Etzioni 1994; 1995) as well as to the emergence of the
third sector as rapidly changing conventional structures and beliefs in modern
societies (Gidron, Kramer, and Salamon, 1992). An inside cultural environment is
related to internal organizational dynamics and to the behaviors of people as work
groups. Thus, it is sometimes termed organizational culture, or organizational climate
(Schein, 1985). Like the outside organizational environment, it has some observable
constructs but it mostly expresses many covert phases. In many ways, “culture is to
the organization what personality is to the individual – a hidden, yet unifying theme
that provides meaning, direction and mobilization” (Kilmann et al., 1985). It includes
basic assumptions as to what is right and what is wrong for a certain organizational
community, norms and beliefs of employees, unseen social rules and accepted codes
of behavior, as well as tradition, language, dress, and ceremonies with common
meaning to all organizational members. All these distinguish “us” from “them”,
promote group cohesiveness, and improve common interests.

Several sociological sources can be effective in analyzing public administration


dynamics. First is group theory, which is also closely related to the study of leaders
and leadership. Second are ethnic studies, which concentrate on minorities and race
questions such as equity, fair distribution of public goods, and integration in
productive public activity. Third is communication and the technological information
revolution, which have had a radical effect on society, public policy, and public
administration units and structure. Information networks and communication have

42 | P a g e
become an imminent feature of the cultural investigation of bureaucracies. For many
years a plausible approach in management science and in the study of public
administration called for the formulation of a universal theory in the field, one that is
culture-free and applicable across all nations. With the passage of time and with giant
technological developments this perception became ever more anachronistic.

Today, the goal of a universal administrative paradigm is hardly achievable. An


alternative viewpoint is more balanced and contingent. It argues that basic similarities
do exist between public organizations and public administration mechanisms, but at
the same time intra-organizational and extra-organizational culture fulfills a major
mediating role. Culture in its broad context constantly affects the operation of
bureaucracies as well as political systems that interact with them. Examples like
Theory Z of W. Ouchi (1981) and lessons from a more recent Chinese and east
European experiences stimulated the scientific community and initiated culture-
oriented ventures in general management inquiry (Hofstede, 1980). They especially
promoted the investigation of work values and culture-oriented management in private
but also in public arenas. Many scholars became convinced of the necessity of
incorporating social and cultural variables as core elements in the administrative
analysis of public arenas. A sociological and cultural approach to public
administration also made an important methodological contribution. It initiated
culture-focused surveys of individuals and groups who work in the public sector or of
citizens who receive services and goods. Culture-focused observations and analyses
possess the merit of being sensitive to people’s (as citizens or employees) norms,
values, traditions, and dispositions, and sometimes they overlap other politics and
policy-oriented studies the better to explore dynamics in public organizations.

Finally, several ethical considerations should be included under any sociological


understanding of the public sector. Ethical dilemmas are frequent in public
administration and relate to cultural aspects, to norms, and most importantly to the
individual behaviour of public servants. For example, hand-in-hand with
governmental operation, questions of ethical standards, integrity, fair and equal
treatment to clients, or appropriate criteria for rewards to public servants become
more relevant. Today, public services in Europe are wider than ever before (Gladstone,
1995; De-Leon, 1996). As a result, public servants are taking care of growing budgets.
They control the transference of more capital to and from the state treasury. This

43 | P a g e
exposes many of them to ethical dilemmas as to how to properly manage, distribute
and redistribute economical wealth. Other ethical difficulties arise as a result of the
instability between business and social requirements in the public environment. For
example, when the cost of certain medicine is too high for citizens to purchase, should
the state take responsibility and help them? When state prisons are full of convicted
prisoners, should the state release some of them to create more places for others?
Responding to such moral issues is difficult. However, public policy which neglects
considerations of ethics, equal treatment of the public, or basic justice and fairness
among its members is initiating a self-destructive process which may damage its
functioning in the long run (Wilenski, 1980).

Management and Organization studies

The third core-stone of public administration is based on knowledge from management


and organizational sciences. A managerial definition of public administration
proclaims that it is the executive function in government or a management specialty
applied in public systems (Shafritz and Russell, 1997:19-23). Although public sector
management is distinguished from private sector management, in many ways the two
systems share a surprisingly broad area of similarities (Rainey, 1990). For many years,
differences stemmed from the nature of services each sector customarily provided,
from diverse structures and functions, but mainly from discrepancies in the
environment. However, when the environment started rapidly to change, organizations
had to change as well. Modern societies have become more complex, flexible, and
dynamic. Cultural, industrial, technological, economic, and political environments of
organizations have undergone rapid transformations that are still in progress today.
On the one hand, public and private organizations have to adjust and comply with
similar changes in the environment to safeguard their interests and existence. But on
the other hand, the starting point of public organizations is far inferior and urgently
calls for rethinking and reinventing (Osborne and Gaebler, 1992).

Conventional wisdom accepted a classic assumption regarding the relatively stable


and unshakable structure of public organizations. Drawing on the Weberian approach,
hardly anyone disputed the need for large bureaucracies in modern democracies.
Moreover, the advantages and disadvantages of large bureaucracies were well known
among academics and practitioners. A weighty bureaucracy was considered an axiom

44 | P a g e
of public administration. Only with the emergence of new management trends in old
bureaucracy were these basic assumptions questioned. For example, Kettl and
Milward (1996) stated that management in the public sector matters. It matters
because citizens’ demands increase and because the standards of performance
expected from governments are higher than ever before. Performance is related in the
minds of people and in scientific studies to quality of management, quality of
managers, and the administrative process between them. Accordingly, it has much to
do with the human aspects of administration. Perhaps this perception has led to some
recent developments in public administration, making it client-oriented and more
businesslike. Scholars frequently define these shifts as the principal change in public
administration and its transition into a revised field of study named public
management.

Current trends: A public managerial reform?

What is the future of modern public administration and what new frontiers are awaiting
ahead? The wisdom of managing states and communities in the 21st century relies on
manifold disciplines and multiple sources of knowledge. The information era and the
immense technological advancement with which our nations struggle necessarily create
higher levels of accessibility, availability, and transparency to the public. The emergence
of e-government is no more a fantastic dream but blatant reality. Public administration
in America and in the world is moving through reforms and changes that are aimed at
downsizing, privatization, de-bureaucratization, higher professional managerialsim, and
above all strict dedication and aspirations to become a better "science" by improving
measurement tools and adhering with positivism and empiricism.

Since the early 1980s much work has been conducted in public administration theory
and practice that claimed to go beyond the conservative approach in the field. This
“liberalization” of public administration is recognized today as the “New Public
Management” (NPM) trend. The self-identity problem of public administration was greatly
aggravated by the launching of the idea of NPM. As noted by Kettl and Milward (1996:
vii), “public management is neither traditional public administration nor policy analysis
since it borrows heavily from a variety of disciplines and methodological approaches”.
Mainly drawing on the experience of the business/industrial/private sector, scholars
have suggested taking a more demanding attitude to the dynamics, activity, and

45 | P a g e
productivity of public organizations. However, “competing academic disciplines dueled to
establish bridgeheads or, worse, virtually ignored each other as they developed parallel
tracks on related problems” (p. 5). Consequently, a cross-fertilization, which could have
accelerated learning and improved performance of public systems, was delayed.

What are the roots of NPM, and in what way is it actually a new arena in the study of
the public sector? Several theoretical foundations, as well as practical factors, can
answer these questions. The first, and probably the deepest source of NPM emerges
from the distinction between two proximate terms or fields of research: administration
and management. As noted earlier, since the late 1880s the monopoly on the term
administration has been held by political scientists. Scholars like Goodnow and
Wilson were those who perceived public administration as a separate and unique
discipline that should consist of independent theory, practical skills, and methods.
However, the term management referred to a more general arena, used by all social
scientists and mainly by those who practice and advance theory in organizational
psychology and business studies. Consequently, conservative administration science
tends to analyze the operation of large bureaucratic systems as well as other
governmental processes aimed at policy implementation. Management, on the other
hand, refers to the general practice of empowering people and groups in various social
environments and in handling multiple organizational resources to maximize efficiency
and effectiveness in the process of producing goods or services.

NPM has indeed become extremely popular in the theory and practice of contemporary
public administration. Still, it is not clear if we can define it as a long-range revolution
in public administration theory. Some will say that NPM has only revived an old spirit
of managerialism and applied it in the public sector. Others will argue that this in
itself has been a momentous contribution to public administration as a discipline in
decline. Relying on an extensive survey of public management research in America,
Garson and Overman (1983:275) argued that this increasing popularity was due to the
more virile connotation of the term management than administration. Over the years,
a growing number of political scientists came to perceive public administration as an
old and declining discipline. It was unable to provide the public with adequate
practical answers to its demands, and moreover it left the theoreticians with epidemic
social dilemmas awaiting exploration. Interesting evidence of this process could be

46 | P a g e
found in many schools of public administration that during the 1980s and 1990s
decided to become schools of public management. Looking for alternative ideas,
management theory was proposed as the source for a new and refreshing perspective.
It was suggested that public management rather than public administration could
contribute to a new understanding of how to run the government more efficiently,
hence to surmount some of its pandemic ailments.

Thus, Perry and Kraemer (1983) stated that a greater impact of new ideas and
methods from the field of public management on the administrative science was
essential and natural. It reflected a special focus of modern public administration that
was not to be ignored. Rainey (1990:157) claimed that this process was a result of the
growing unpopularity of government during the 1960s and 1970s. Ott, Hyde, and
Shafritz (1991:1) also stated that public management was a major segment of the
broader field of public administration since it focused on the profession and on the
public manager as a practitioner of that profession. Furthermore, it emphasized well-
accepted managerial tools, techniques, knowledge, and skills that could be used to
turn ideas and policy into a (successful) program of action.

During the last two decades many definitions have been suggested for NPM. Yet
nothing seems wrong with the relatively old perception of Garson and Overman
(1983:278), who defined it as “an interdisciplinary study of the generic aspects of
administration...a blend of the planning, organizing, and controlling functions of
management with the management of human, financial, physical, information and
political resources”. As further discussed by other scholars (e.g., Lynn, 1996:38-39),
six differences exist between public administration and public management that make
the former a new field of study and practice. These are (1) the inclusion of general
management functions such as planning, organizing, control, and evaluation in lieu of
discussion of social values and conflicts of bureaucracy and democracy; (2) an
instrumental orientation favoring criteria of economy and efficiency in lieu of equity,
responsiveness, or political salience; (3) a pragmatic focus on mid-level managers in
lieu of the perspective of political or policy elites; (4) a tendency to consider
management as generic, or at least to minimize the differences between public and
private sectors in lieu of accentuating them; (5) a singular focus on the organization,
with external relations treated in the same rational manner as internal operations in

47 | P a g e
lieu of a focus on laws, institutions, and political bureaucratic processes; (6) a strong
philosophical link with the scientific management tradition in lieu of close ties to
political science or sociology.

While the emergence of NPM is frequently related to the increasing impact of positivist
behavioral science on the study of politics and government (e.g., Lynn, 1996:5-6), the
practical aspect of this process should also be considered. Practical public managers
(Golembiewski, 1995), as well as political scientists, will refer to the difficulties in
policy making and policy implementation which faced many western societies in
Europe, America, and elsewhere during the 1970s. These practical difficulties are
viewed today as an important trigger for the evolution of NPM. Reviewing two recent
books on NPM (Aucoin, 1995; Boston, Martin, Pallot, and Walsh, 1996), Khademian
(1998:269) argues that American and Westminster advocates of the field find common
ground in explaining why such reforms are necessary. The problem of an inflexible
bureaucracy that often could not respond efficiently and promptly to the public needs
conflicted with some basic democratic principles and values in these countries. Peter
Aucoin elegantly summarizes a "trinity" of broadly based challenges with which
western democracies have struggled, and will probably continue to struggle in the
future, partly through management reform. These are (1) growing demands for
restraint in public sector spending, (2) increasing cynicism regarding government
bureaucracies' responsiveness to citizens’ concerns and political authority and
dissatisfaction with program effectiveness, and (3) an international, market driven
economy that does not defer to domestic policy efforts. These challenges have
apparently led many western governments, in America, Britain, New Zealand, Canada,
and elsewhere, to the recognition that firm reforms and changes in the public service
should be made.

There is no doubt that at least some of the accumulated wisdom of the private sector
in many countries is transferable to the public sector (Pollitt, 1988; Smith, 1993). In
an attempt to liberate the public sector from its old conservative image and tedious
practice NPM was advanced as a relevant and promising alternative. NPM literature
has tried to recognize and define new criteria that may help in determining the extent
to which public agencies succeed in meeting the growing needs of the public. NPM has
continuously advocated the implementation of specific Performance Indicators (PIs)
used in private organizations to create a performance-based culture and matching

48 | P a g e
compensatory strategies in these systems. It has recommended that these indicators
be applied in the public sector (e.g., Smith, 1993; Carter, 1989) since they can
function as milestones on the way to better efficiency and effectiveness of public
agencies. Moreover, citizens' awareness of the performance of public services was
suggested as a core element of NPM since it can increase the political pressure placed
on elected and appointed public servants, thereby enhancing both managerial and
allocative efficiency in the public sector. Scholars who advocate NPM liken this
process of public accountability to stakeholders/citizens to the role adopted by
financial reporting in the private/corporate sector (Smith, 1993). As in that sector,
increasing exterior-related outcomes can have a profound impact on internal control
mechanisms, as managers and public servants become more sensitive to their duties
and highly committed to serve their public customers.

Thus, Lynn (1998:231) suggested that the NPM of the late 1990s had three
constructive legacies for the field of public administration and for democratic theory
and practice. These were (1) a stronger emphasis on performance-motivated
administration and inclusion in the administrative canon of performance-oriented
institutional arrangements, structural forms, and managerial doctrines fitted to
particular context, in other words, advances in the state of the public management
art; (2) an international dialogue on and a stronger comparative dimension to the
study of state design and administrative reform; and (3) the integrated use of
economic, sociological, social-psychological, and other advanced conceptual models
and heuristics in the study of public institutions and management, with the potential
to strengthen the field's scholarship and the possibilities for theory-grounded practice.
While the first two “legacies” are widely discussed in contemporary literature, the
third is much understudied and needs further theoretical development, empirically
guided research, and practical implementation.

Moreover, Kettl and Milward (1996) argue that one of NPM’s most significant
contributions to public administration as a discipline in transition is the focus on the
performance of governmental organizations. According to their analysis, this scientific
orientation needs to draw on “a wide variety of academic disciplines for the full and
richly textured picture required to improve the way government works. Only through
interdisciplinary cross-fertilization will the picture be rich enough to capture the

49 | P a g e
enormous variety and complexity of true public management (and administration)
puzzles” (p.6).

The journey continues

This paper has relied on previous works to describe public administration as a


discipline in transition. In many ways it has always been in continuous movement,
but not always in the same direction. Contrary to the heavy, formal, and inflexible
image of bureaucracies, public sector bodies in America, Europe, and elsewhere have
been in rapidly intensifying transition since the early 1990s. During the last century
public administration has undergone gone significant changes resulting from crises,
as well as breakthroughs in an ultra-dynamic environment. Generally speaking,
Waldo’s (1968) assertion that these ongoing transformations reflected an identity crisis
of a science in formation is also relevant today, albeit with some amendments.
Whereas in the past these crises signaled a struggle for the recognition and legitimacy
of public administration as a scholarly academic field, today the identity problem leads
to other dilemmas, which are beyond simple existence and legitimacy.

In recent decades the struggle over the nature and uniqueness of public
administration has continued, some say even intensified. From the very early days of
the discipline to the present its boundaries have been in a state of ongoing debate. To
talk of the “Public”, of “Administration”, and of the integration of the two constructs
into a useful terrain for study holds out promise as well as involving difficulties. But
consensus does exist on at least one issue: the public needs a better bureaucracy,
more flexible, working efficiently and effectively, moving quickly toward objectives, and
at the same time responding to the needs of the people without delays and with
maximum social sensitivity, responsibility, and morality. The public also expects good
and skillful administrators, versed in the mysteries of quality services and effective
management. Only they can produce better “public goods” and deliver them to all
sectors of society in minimum time and at minimum cost. These goals are
undoubtedly ambitious but they have the potential of safeguarding the structure of
democratic societies. This is a revised version of the ideal type of public administration
systems applicable to modern times.

50 | P a g e
However, reality seems far more complex. There is growing concern among scholars
today that these goals are way beyond reach. Modern states across the world face
serious problems of adhering to the public’s needs. Achieving one target is usually
accompanied by painful compromises on others, and limited resources are frequently
cited as the main reason for failure in the provision of services. Moreover, fundamental
changes are taking place in people’s lifestyles, as in their beliefs and ideologies. They
are multiplied through high technology, communication systems, new distribution of
capital, and the rise of new civic values that never existed before. All these lead
citizens to perceive government and public administration systems differently. The role
of the state and its relationship with bureaucracy and with citizens is undergoing a
substantial transformation not only in the minds of the people but also in scientific
thinking. In a rapidly changing environment, public administration has a major
function and new aims that must be clearly recognized. It remains the best tool
democracy can use to create fruitful reciprocal relationships with citizens, but on a
higher and better level. To uncover the major tasks and challenges facing the new
generation of public administration we require a cross-disciplinary strategy and
improved integration of all available knowledge in the social sciences aimed at
redefining the boundaries of public administration systems in its new era.

Today, at the beginning of the 21st century, the formation of public administration as
an interdisciplinary academic field seems certain. Still, it is unfinished business due
to the need and demand to make it more of a “harder social science”, one which is
closer to management science, economics, or even psychology. Hence, the state of the
field is in dispute among academics and practitioners from across the world who seek
higher and more extensive scientific recognition, by applying a higher level of
empirical-based paradigm. It is argued that such inputs may produce a more accurate
self-definition and better applicability of the field to rapid changes in modern life. This
process presents new challenges for public administration. Perhaps the most
important is to integrate more widely existing knowledge of the social sciences with
efficient public action and with quality governmental operation. In the coming years
public administration will be evaluated by higher standards of theory cohesiveness
and by more comprehensive performance indicators rooted in a variety of scientific
fields. The exploration of new interdisciplinary horizons for public administration is
thus essential, and inevitable for the successful passage of the field into the third

51 | P a g e
millennium. Somewhat contrary to the concerns of Waldo (1968), the identity crises in
its new form may carry a positive, not endangering, interdisciplinary merit. The
interdisciplinary orientations have the potential of pulling public administration out of
its perplexing-stagnating status and lead it towards a more solid scientific position.

In light of the above a consensus exists today among scholars and practitioners that
modern public administration decidedly benefits, and will continue to benefit, from the
seminal inputs of social and cultural motives and mainly from the impact of
managerial and organizational theory. In keeping with these, modern societies
question the current obligations of public personnel toward citizens, and urge them to
put people and social values first. These tasks can be achieved by treating citizens as
customers or clients but also through building a different value of administrative spirit
(Vigoda and Golembiewski, 2001). Yet managerial tendencies draw fire from those who
argue that a client orientation of the public sector breeds citizen passivity and lack of
individual responsibility toward the state and its agencies. It is further assumed that
today these obligations and commitments are not clearly decoded, manifested, or
satisfactory implied. Consequently they yield an identity problem of the field and strive
for redefinition of its unwritten contract with the people. Scholars are divided over the
best way to obtain missions of good-management together with good cultural order.
Still, they agree that much more can be done to improve responsiveness to citizens’
needs and demands without forgoing the active role of citizens in the administrative
process.

Moreover, the information revolution is expected to create a growing impact on public


administration of the future both as a science and as a profession. In referring to the
modern public sector Caldwell (2002) suggested that "Our task for linking information,
social issues, politics, policy and management is a challenge yet to be accomplished.
The enthusiasm for public planning, notably in the 1930s, did not survive the Second
World War. The so-called “reinvention” of government based on a market-driven model
appears to be essentially contemporaneous and superficial in relation to the multiple
challenges to be confronted in the 21st century" (p73-174) Thus, Caldwell continues to
argue that "governments and their administrators (in our time) characteristically focus
on immediate situations and pressing problems. There are few political rewards for
anticipating the long – range future. However, the advancement of science and an

52 | P a g e
apparent growth of public acceptance, however slow, of science-based forecasts may
enlarge the “educating” role of public administration. We are hardly at the end of the
expansion of knowledge and information, and there is growing although limited
recognition that we also face formidable challenges to a sustainable future. And so it is
more realistic to see the “New Public Administration” as an evolving process
continuing to become “new” as it is adapted to meeting the needs of the ever-receding
future".

Finally, in this paper I proposed that the application of multi-disciplinary approaches


(political, social, and managerial-based) to the public service is essential for somehow
resolving the identity conflict of the field. An agreement over self-identity is required
before any further development can be achieved. It is argued that some tenets of
administrative culture and democratic values need to be explored before higher levels
of social theory synthesis and integration can be reached. These may also be the
milestones on the way to better linkage, partnership, and cooperation between rulers
and citizens in modern societies. Here lies the main challenge of public administration
in the coming years: the invention of a new vitalized administrative generation that is
interdisciplinary in nature and tightly bounded together with modern participatory
democracy. The contribution of this paper is its effort to bring these views together
and to produce a multi-faceted analysis of modern public administration.

53 | P a g e
1.6 Theory and practice of public administration

In To Run a Constitution, John A. Rohr seeks to legitimize the administrative state in


accordance with the fundamental principles established by the United States
Constitution. To do so, he focuses on the nation’s heritage and tradition, rather than
the text of the constitution. Drawing upon the writings and debates of our nation’s
founders, including the debates between the Federalists and the Anti-Federalists, Rohr
finds support for his central thesis: The administrative state was contemplated by our
founders and is implicitly recognized in the framing of the Constitution. This
methodology feeds Rohr’s secondary purpose of educating public administrators on
constitutional law and history. According to Rohr, an in-depth understanding of our
founding principles is critical for administrators who must discharge their duties in
accordance with an oath to uphold the Constitution.

The book is divided into three parts. The first part emphasizes the founding of the
nation. Rohr argues that the administrative state can be justified under the
Constitution based upon three main contentions. First, the combination of executive,
legislative, and judicial powers within the administrative state does not violated the
doctrine of separation of powers. Second, the career civil service fulfills the
constitutional design of the framers by performing a balancing executive function
originally assigned to the Senate. Finally, in general, the career civil service heals the
defect of inadequate representation by reducing the ratio between the citizenry and
government officials.

In the second part of the book, Rohr turns his attention to the founding of the
administrative state “in word.” After discussing Woodrow Wilson and Frank J.
Goodnow, Rohr focuses his attention on Judge Thomas Cooley. To Rohr, Wilson and
Goodnow were not loyal to the nation’s constitutional heritage. Wilson disapproved of
the doctrine of separation of powers; Goodnow did not like the notion of popular
sovereignty. Cooley, on the other hand, provided a true example of an administrator
and constitutional scholar who was loyal in all respects to the founding principles of
the country. In 1887, Cooley, who was widely regarded as the foremost scholar of
constitutional law at the time, was tapped to lead the Interstate Commerce
Commission when it became an independent agency. Rohr believes that Cooley’s
reputation as a constitutional scholar may have led the judiciary to accept the

54 | P a g e
legitimacy of the administrative agency and redefine the requirements of due process.
Additionally, Rohr surmises that Cooley’s leadership may have inspired acceptance of
the agency in the minds of the general public: The ICC under Cooley’s leadership
provides the extraordinary example of a vigorous administrative agency explaining to
Congress its failure to use certain explicit statutory powers on the grounds that the
exercise of such powers might be in violation of the Constitution” (p. 110). He views
this “self-denying action” as an indication that the ICC considered itself accountable to
the Constitution rather than the executive or legislative branch.

The third part discusses the founding of the modern administrative state “in deeds.”
As a practical matter, the modern administrative state was founded during the New
Deal Era. This is where Rohr turns to discover a constitution-based argument to
justify the existence of the administrative state. During the New Deal Era, the United
States Supreme Court reinterpreted the Commerce Clause and the Fourteenth
Amendment in a manner that permitted more federal regulations to be adopted and
more federal agencies to be created. Rohr views this development a victory for the
administrative state that should be developed further. He suggests a that broad
interpretations of the Commerce Clause, the General Welfare Clause, and the
Necessary and Proper Clause could serve as a means of legitimizing the
administrative state.

Finally, in the Conclusion, Rohr discusses the importance of the administrator’s oath
of office. Rohr explains the profound moral significance of the administrator’s oath as
a binding commitment, a justification for administrative autonomy, and a guide for
administrative action. Succinctly stated, if administrators are going to actively seek
justification in the constitution, then certainly administrators must take their oaths
seriously. The oath represents a covenant between the administrator and society that
requires the administrator to act in accordance with constitutional commands.
Moreover, Rohr maintains that the exercise of discretion in accordance with the oath
requires a thorough understanding of the constitutional principles that undergird the
nation. In other words, public administrators should be considered, and should
consider themselves, constitutional officers.

To Run a Constitution is part of the New Public Administration movement that seeks
to advance the dialogue concerning Public Administration and replace the

55 | P a g e
conventional wisdom of bureaucrat bashing with a positive understanding of the
administrative state’s role in society. In general, this movement calls for the pursuit of
academic and legal justification for the administrative state and a redefining of the role
of the public administrator as a constitutional officer. For his part, Rohr pursues legal
justification of the administrative state. And he provides a very compelling argument.
Rohr draws upon the words of our founding fathers to explicate constitutional
provisions in a manner favorable to public administration. He makes a compelling
case that administration was always envisioned as a part of the constitutional order
and, as proof, he cites the Commerce Clause, the General Welfare Clause, and the
Necessary and Proper Clause, and the arguments surrounding their meaning at the
time the constitution was drafted.

However, Rohr’s work is not without its faults. With his contention that
administrative legitimacy may be found by characterizing the administrative state as
filling the role originally envisioned for the Senate, Rohr commits essentially the same
mistake for which he criticized Wilson and Goodnow: The contention is a very
conspicuous departure from the text of the Constitution as well as the history and
traditions that inspired the constitution. Despite the similarities that Rohr
documents, it remains that the Senate is a constitutionally-defined, representative
body within the legislative branch. Constitutionally speaking, the administrative state
is, at most, its doppelganger. Rohr’s argument that the administrative state corrects
the problems of representation befalls a similar fate. The Constitution prescribes a
system of representation through the President, the Representative, and the Senators.
Administrators cannot simply glom on to a representative role. Furthermore, and
most notably, Rohr’s thesis fails to define the critics and criticisms to which it
apparently responds, and, as a result, it deals with the subject from a very general
perspective. Consequently, Rohr seems to miss the point of so much of the criticism
of the administrative state played out in the judicial system and the court of public
opinion. Many critics of the administrative state disclaim the New Deal decisions of
the Supreme Court as ill-conceived. Hence, it seems unlikely that relying upon those
opinions as the bellwether of the legitimacy of the administrative state is unlikely to
win new friends.

Despite these drawbacks, with To Run a Constitution Rohr has made an important
contribution to the public administration literature. He is correct in noting that the

56 | P a g e
administrative state is clearly contemplated by the text and tradition of the
Constitution. This point is lost on many of the administrative state’s critics.
Moreover, his focus on the historical tradition of this country is indeed a step in the
right direction. Scholars and public administrators alike will learn much from Rohr’s
account of the founding of the nation. And Rohr is quite correct in suggesting that a
more in depth knowledge of our nation’s history will serve the profession and the
academic discipline well. This is most apropos when it comes to the administrator’s
oath of office. Administrators will be less likely to take their oaths of office for granted
when they truly understand their role as stewards of the Constitution.

57 | P a g e
1.7 Public choice theory

Public choice in economic theory is the use of modern economic tools to study
problems that are traditionally in the province of political science. (A more general
term is 'political economy', an earlier name for 'economics' that evokes its practical
and theoretical origins but should not be mistaken for the Marxian use of the same
term.)

In particular, it studies the behavior of voters, politicians, and government officials as


(mostly) self-interested agents and their interactions in the social system either as
such or under alternative constitutional rules. These can be represented a number of
ways, including standard constrained utility maximization, game theory, or decision
theory. Public choice analysis has roots in positive analysis ("what is") but is often
used for normative purposes ("what ought to be"), to identify a problem or suggest how
a system could be improved by changes in constitutional rules. A key formulation of
public choice theory is in terms of rational choice, the agent-based proportioning of
scarce means to given ends. An overlapping formulation with a different focus is
positive political theory. Another related field is social choice theory.

Origins and formation

The modern literature in Public Choice began with Duncan Black, who in 1948
identified the underlying concepts of what would become median voter theory. He also
wrote The Theory of Committees and Elections in 1958. Gordon Tullock refers to him as
the "father of public choice theory".

Special Interests

Public choice theory is often referred to when discussing how individual political
decision-making results in policy that conflict with the overall desires of the general
public. For example, many special interest and pork barrel projects are not the desire
of the overall democracy. However, it makes sense for politicians to support these
projects. It may benefit them psychologically as they feel powerful and important. It
can also benefit them financially as it may open the door to future wealth as lobbyists
(after they retire). The project may be of interest to the politician's local constituency,
increasing district votes or campaign contributions. The politician pays little to no cost

58 | P a g e
to gain these benefits, as they are spending public tax money. Special interest
lobbyists are also behaving rationally. They can gain government favors worth millions
or billions for relatively small investments. They face a risk of losing out to their
competitors if they don't seek these favors. The taxpayer is also behaving rationally.
The cost of defeating any one government give-away is very high, while the benefits to
the individual taxpayer are very small. Each citizen pays only a few pennies or a few
dollars for any given government favor, while the costs of ending that favor would be
many times higher. Everyone involved has rational incentives to do exactly what
they're doing, even though the desire of the general constituency is opposite. (It is
notable that the political system considered here is very much that of the United
States, with "pork" a main aim of individual legislators; in countries such as Britain
with strong party systems the issues would differ somewhat.)

Decision making processes and the State

One way to organize the subject matter studied by Public Choice theorists is to begin
with the foundations of the state itself. According to this procedure, the most
fundamental subject is the origin of government. Although some work has been done
on anarchy, autocracy, revolution, and even war, the bulk of the study in this area has
concerned the fundamental problem of collectively choosing constitutional rules. This
work assumes a group of individuals who aim to form a government. Then it focuses
on the problem of hiring the agents required to carry out government functions agreed
upon by the members.

The main questions are: (1) how to hire competent and trustworthy individuals to
whom day-to-day decision-making can be delegated and (2) how to set up an effective
system of oversight and sanctions for such individuals. To answer these questions it is
necessary to assess the effects of creating different loci of power and decision-making
within a government; to examine voting and the various means of selecting candidates
and choosing winners in elections; to assess various behavioral rules that might be
established to influence the behavior of elected and appointed government officials;
and to evaluate alternative constitutional and legal rights that could be reserved for
citizens, especially rights relating to citizen oversight and the avoidance of harm due to
the coercive power of government agents.

59 | P a g e
These are difficult assessments to make. In practice, most work in the field of Public
Choice has dealt with more limited issues. Extensive work has been done on different
voting systems and, more generally, on how to transform what voters are assumed to
want into a coherent "collective preference". Of some interest has been the discovery
that a general collective preference function cannot be derived from even seemingly
mild conditions. This is often called Arrow's impossibility theorem. The theorem, an
economic generalization of the voting paradox, suggests that voters have no reason to
expect that, short of dictatorship, even the best rules for making collective decisions
will lead to the kind of consistency attributed to individual choice.

Much work has been done on the loose connection between decisions that we can
imagine being made by a full contingent of citizens with zero collective decision-
making costs and those made by legislators representing different voting
constituencies. Of special concern has been logrolling and other negotiations carried
out by legislators in exercising their law-making powers. Important factors in such
legislative decisions are political parties and pressure groups. Accordingly, Public
Choicers have studied these institutions extensively. The study of how legislatures
make decisions and how various constitutional rules can constrain legislative
decisions is a major sub-field in Public Choice.

Bureaucracy

Another major sub-field is the study of bureaucracy. The usual model depicts the top
bureaucrats as being chosen by the chief executive and legislature, depending on
whether the democratic system is presidential or parliamentary. (See also presidential
system and parliamentary system.) The typical image of a bureau chief is a person on
a fixed salary who is concerned with pleasing those who appointed him. The latter
have the power to hire and fire him more or less at will. The bulk of the bureaucrats,
however, are civil servants whose jobs and pay are protected by a civil service system
against major changes by their appointed bureau chiefs. This image is often compared
with that of a business owner whose profit varies with the success of production and
sales, who aims to maximize profit, and who can hire and fire employees at will.

60 | P a g e
Rent-Seeking

A field that is closely related to public choice is "rent-seeking." This field combines the
study of a market economy with that of government. Thus, one might regard it as a
"new political economy." Its basic thesis is that when both a market economy and
government are present, government agents are a source of numerous special market
privileges. Both the government agents and self-interested market participants seek
these privileges in order to partake in the monopoly rent that they provide. When such
privileges are granted, they reduce the efficiency of the economic system. In addition,
the rent-seekers use resources that could otherwise be used to produce goods that are
valued by consumers.

Rent-seeking is broader than Public Choice in that it applies to autocracies as well as


democracies and, therefore, is not directly concerned with collective decision-making.
However, the obvious pressures it exerts on legislators, executives, bureaucrats, and
even judges are factors that Public Choicers must account for in their effort to
understand and assess collective decision-making rules and institutions. Moreover,
the members of a collective who are planning a government would be wise to take
prospective rent-seeking into account.

61 | P a g e
1.8 The development of American administration

Frederick Taylor, “Scientific Management”

1. “Scientific Management” school, 1880; factual, analytical approaches in


place of rule-of-thumb or intuitive methods; organization of rules,
procedures, methods at managerial level; integrating scientific principles into
the workforce to bring together workers and managers; advocate of “close,
personal cooperation” in tasks, or equal division of work; careful selection,
and training of personnel; friendly cooperation between management and
employees, human resource development. Application of behavioral/human
relations principles as they were then understood. Taylor’s philosophy and
methods began to attract attention in 1910s, extensively applied in new
agencies in 1930s.

2. Initiative and incentive versus one best way

3. Scientific management = practically with absolute regularity

4. Principles of Scientific Management

a. Traditional knowledge – rules, laws, formulas; development of


science organizing traditional knowledge; science to the details
in aspects of administration (one best way).

b. Scientific selection and progressive development of workers shortcomings


and possibilities; physical aspects, measurement; control of
workplace.

c. Bringing of the science and the scientifically selected and trained


workers together; organizing people; once science of management
developed, conflict will be illuminated.

d. Division of work between management and workers; managers in


control, workers happy (people who do work do not understand
work); managers implement results of science.

5. Ideas are essentially wrong, but nonetheless influential

Louis Brownlow, Charles E. Merriam, and Luther Gulick, “Report of the resident’s
Committee on Administrative Management”

62 | P a g e
1. Head of FDR’s Committee on Administrative Management, created to
improve federal civil service; committee asked to report back to FDR on
modernizing management in the executive. Favored strong executive
leadership of civil service; strong presidency grounded in reform tradition.
Constitutional role of Congress was not to nickel and dime executive
agencies, but to provide legislative oversight of whole through auditing.
Resulted in creation of Executive Office.

2. Equipping democracy for action – through the office of the President

3. Efficiency = consent of the governed and good management

4. Requires the establishment of “a responsible and effective chief executive


as the center of energy, direction, and administrative management; the
systematic organization of all activities in the hands of a qualified
personnel under the direction of the chief executive; and to aid him in
this, the establishment of appropriate managerial and staff agencies.”

5. “Administrative management concerns itself in a democracy with the


executive and his duties, with managerial and staff aides, with
organization, with personnel, and with the fiscal system because these are
the indispensable means of making good the popular will in a people’s
government.”

6. Modern managerial tools needed to “make democracy work today in our


national government; that is, to make our government an up -to-date,
efficient, and effective instrument for carrying out the will of the nation.” –
Public interest.

7. The White House Staff – “the president needs help”

a. executive assistants – division of labor and specialization

b. carry over to the departments these managerial practic es

8. Compelling argument (no executive office at time of report).

a. Control over process of administration

b. President as CEO of bureaucracy

63 | P a g e
David H. Rosenbloom, “Public Administrative Theory and the Separation of
Powers”

1. Public administrators need “constitution al competence”


2. Problem with identify crisis in PA is problem of our three -branch system
of government: hard to retrofit PA into constitutional separation of powers,
disturbs our three-dimensional thinking.
3. No accident that theoretical perspectives on PA ro le reflect each
branch of government: “managerial” role/executive; “pluralist, political”
roles/legislative, “judicialization of agencies”/legal.
4. Managerial approach: from civil service reform in late 19 th century.
a. Maximization of effectiveness, efficiency and economy.
b. Hierarchy, impersonal view of the individual.
5. Political approach: “apparent empirical reality”
a. Representativeness, political responsiveness, and accountability.
b. Representative bureaucracy, citizen participation.
c. Political pluralism; individual as part of aggregate group.
6. Legal approach: administrative law, “judicialization” and individual rights
through legal procedure; constitutional law; judges as partners in the
administrative process.
a. procedural due process, individual substantive rights (Bill of Rights,
14th amendment)
b. Adversary procedure (e.g., in public personnel management the
application of EEO and labor relations).
c. Individual as a “unique person in a unique set of circumstances.”

H. George Frederickson, “Toward a New Public Administra tion”

1. New PA was a reaction to political turbulence of the 1960s. Argument:


Given assumption of values in public administration decision -making and
inability of public administrator to achieve political neutrality, why not
require public administrator to take positive role by improving social
conditions? Moral imperative. Values important (e.g., social equity,
fairness, justice in public services). Will still achieve efficiency and
effectiveness imagined by earlier theorists, just a different version.

64 | P a g e
2. What’s New PA?
a. Classical argument: efficient, economical, and coordinated
management; New PA adds social equity.
3. Classical argument: (1) How can we offer more or better services with
available resources (efficiency)? (2) How can we maintain our level of
services while spending less money (economy)? New PA: Does this service
enhance social equity?
4. A public administration that works for changes which try to redress the
deprivation of minorities. Social equity includes activities designed to
enhance the political power and economic well-being of minorities.
5. Modified bureaucratic forms: decentralization, devolution, projects, contracts,
sensitivity training, organization development, responsibility, expansion,
confrontation, and client involvement
6. Throwing money at social issues does not solve the problem; refocus
energy on institutional arrangements to combat problem.
7. “Second generation behaviorism” (p. 318) “The Public Administrator
accepts the importance of understanding as scientifically as possible how
and why organizations behave as they do but he/she tends to be rather
more interested in the impact of that organization on its clientele and vice
versa.”
8. The second generation behaviorist is less “generic” and more
a. “public” than his forebear, less “descriptive” and more
“prescriptive,”
b. less “institution oriented” and more “client -impact oriented,” less
c. “neutral” and more “normative,” and it is hoped no less scientific.
9. Organization Theory and New PA: distributive process, integrative process,
boundary-exchange process, and the socio-emotional process.

65 | P a g e
1.9 Paradigms

What is a paradigm?

Denhardt – A paradigm is a schema upon which to hang theory.

Guba & Lincoln – A paradigm is the basic belief system or worldview that guides the
investigator ontologically, epistemologically and methodologically.

What is a Theory?

Putnam – Theories are partially interpreted calculi in which only the observation
terms are directly interpreted.

Stanley, French, Spears 2005 – A theory is inadequate as a paradigmatic choice


because it is a partial aspect of reality.

Kuhn – Definition of Paradigm:

1. “The entire constellation of beliefs, values, techniques, and so on shared by the


members of a given community”.
2. “One element in that constellation, the concrete puzzle-solutions which,
employed as models or examples, can replace explicit rules as a basis for the
solution of the remaining puzzles of normal science.”

Thomas Kuhn, one of the best-known [and most influential] historians and
philosophers of science in the 20th century, made the following summarized major
arguments (Land and Anders, 2000):

1. The field of science progress by paradigm shift – a reconstruction of prior theory


and knowledge.
2. Paradigms are criteria for indentifying legitimate problems and methods of
inquiry for a research field. It includes a worldwide (constructed theory about
how facts should be related) and two basic criteria – it attracts an enduring
group of adherents away from competing modes of scientific activity and it is
sufficiently open-ended to leave all sorts of problems for the followers to resolve.
3. Paradigms do not have to entirely inclusive. Competing paradigms can exist
within one discipline. Also, not all legitimate research activities within one
discipline have to be governed by paradigms.

66 | P a g e
4. The concept of paradigm is different from the concept of rule. Researchers
working under the guidance of a shared paradigm do not have to always agree
with one another on the set of rules that govern their research. Lack of a
standard interpretation or of an agreed reduction to rules will not prevent a
paradigm from guiding research (Kuhn, 1970).

There are three basic components to any legitimate paradigm, each one assuming a
different characteristic: the (1) ontological, (2) epistemological, and (3) methodological
arguments.

The nature of reality (the ontological argument) suggests that certain realities exist
that we can ultimately know, but there are certain things we never comprehend. It is
up to the researcher to find this reality (epistemological argument) through recognized
methods of theory development (methodological argument) that objectively or
subjectively measure partial aspects of social phenomena to determine this reality
(French, et al, 2005).

Every paradigm has ASSUMPTIONS: Ontological, Epistemological and


Methodological.

ONTOLOGY – The Nature of Reality – The Nature of that reality we are studying.
What we can know. What can be known about it? Ontological assumptions are those
which concern the very essence of the phenomena under investigation. Suggests that
certain realities exist that we can ultimately know, but there are certain things we
can never comprehend. Those questions that do not relate to matters of “real”
existence or “real” action fall outside the realm of legitimate scientific inquiry.

EPISTEMOLOGY – What Can We Know? The Philosophy of Knowledge – How and


where do we collect the data? Multiple methods and ways of thinking. What is
the relationship between the knower and what can be known? The Epistemological
assumption are assumptions about the grounds of knowledge – about how one might
begin to understand the world and communicate this as knowledge to fellow human
beings. These assumptions entail ideas about what forms of knowledge can be
obtained, and how one can sort out what is “true” from what is “false”. How do we
know what we know?

67 | P a g e
METHODOLOGY – The System of Inquiry – How can we perform examination? How
can the inquirer go about finding out whatever he believes can be known?

Five Social Science Paradigms:

POSITIVISM Popper

POST-POSITIVISM Simon

CONFLICT THEORY Feyerabend

INTERPRETIVISM Charles Taylor

CONSTRUCTIVISM Denzin & Lincoln

What is the difference between Interpretivist and Constructivist?

Interpretivists approach a study WITHOUT ANY PRECONCEPTIONS about what


they will find – simply interpret what you find.

Constructivists come into a study with a PRECONCEIVED NOTION of the particular


social constructions he might find.

GIDDENS – STRUCTURATION - The process whereby governments attempt to make


the functioning of the reproduction circuits of the state compatible with their political
objectives is called the structuration (see Giddens 1990, 64) of public administration.
The mastering of this process is the main objective of the public administration
discipline.

The structuration of public administration is not entirely determined by a


government's will. Governments have to confront the various systemic forces that
condition the nature and the role of public administration. These forces both
constrain and enable the agents in control of the administrative state in their efforts
to institutionalize patterns of social relations across historical time and territorial
space.

 The relationship between social systems within a societal totality does not
follow any rigidly determined pattern. From this perspective, the functioning of

68 | P a g e
the reproduction circuits of the administrative state, and consequently a
government's capacity to structure them, is conditioned by the interplay of the
government system, the political system, the economic system, and the internal
dynamic of the administrative state.
 The functioning of the government system, the political system, the economic
system, and the reproduction circuits of the administrative state also is
conditioned by international and transnational forces; thus, the study of the
process of structuration of public administration must take into account the
process of global interpenetration within which public administration operates
today.

Gregory A. DANEKE

Gregory Daneke’s article is an answer to Harland Cleveland and others scholars who
feel that “it is time to put behind us the idea that the politics and administration of
human endeavors are some kind of science.” Daneke feels that much of the scholarly
work that has been done in public administration in the recent past has not
contributed to advancing a new paradigm for public administration, but instead have
centered on critiques of positivism and/or neoclassical economic theory. Daneke
argues that scholars in the discipline should channel their energies into reforming
the system’s basic unifying paradigm and to advance it in the direction of a
“comprehensive design approach”, capable of enhancing the processes of adaptive
learning and institutional evolution.

Daneke then presents a new prospective paradigm called “Advanced Systems Theory”.
This emerging paradigm builds on general systems theory and contemporary
economics, while integrating a number of unique ingredients derived from advances
in the physical, biological, and cognitive sciences. By incorporating new knowledge
from works in diverse areas such as chaos theory and quantum logic, the social
sciences (and public administration in particular) may be able to overcome their
current lack of progress in paradigm development.

PA IDENTITY CRISIS

Denhardt argues that PA has been limited in the past by two important positions
deeply rooted in the history of the discipline:

69 | P a g e
1. A view of moral and political accountability conceived in hierarchical terms, in
terms of responsiveness of agencies to elected officials; and
2. A transposition of business values or at least managerial values into the
conduct of public agencies.

“The proper moral and political basis for public organizations can no longer be
encapsulated in the hierarchical relationship between agencies and legislatures.”

“We should call into question the wholesale adoption by public agencies of the values
of private organizations. Indeed, I would propose that just the opposite should
occur.”

PA theory draws its greatest strength and its most serious limitation from its
diversity. This diversity often means that the field lacks a sense of identity.

ROSENBLOOM (1983) – 3 Approaches to PA Theory: Managerial, Political & Legal.


“PA administrators have the unenviable position of having to balance the various
interests represented by these three approaches.

HART & SCOTT (1982) – American PA has been taken over by the values of American
Business Management. These values are “inappropriate” to the conduct of public
affairs. Hart & Scott argue that the conduct of public affairs should be guided by the
natural-law values of our constitutional foundation.

OSTROM (1980) – The study of PA should not be treated as strictly natural


phenomena. The methods of the natural sciences are not fully appropriate to the
study of PA. Instead, we need to look upon administrative tasks and administrative
arrangements as works of art or as artifacts.

Robert DENHARDT – In support of CRITICAL THEORY.

Denhardt begins by stating the problem: “We are in a situation in which we seem to
be somewhat unsure of the historical and philosophical grounding of our work.”
Denhardt recognizes that we can trace the roots of our heritage in many directions,
but as we do, we discover that there are “serious conflicts among our ancestors.”
(Denhardt, 1981)

In short, the problem he identifies is that our concern for making public
administration an object of “scientific” study has often been at odds with our interest

70 | P a g e
in extending the normative principles we associate with democratic governance. The
purpose of the article is to examine recent works in “critical” theory, while
addressing implications for developing a comprehensive theory of public organization.
Denhardt notes that the critical approach is suggestive of alternative conceptions of
the role of public organizations in a democratic society, as well as changing patterns
of relationships within the bureaucracy.

“It reminds us that the theory and practice of public administration is integral to the
development of the state and its allocation of values in society and therefore must
encompass far more than technical concerns”.

Following a brief review of the development of critical theory, Denhardt focuses on the
work of Jurgen Habermas, who is perhaps the best known contemporary scholar of
the critical approach.

Jurgen Habermas

The critical perspective proffered by Habermas includes the following aspects:

It examines:

1. The critique of instrumental reason


2. The scientization of political life and the reduction of the public sphere, and
3. The relationship between knowledge and human interests.

Habermas, a member of the “Frankfurt School” of thought, describes the “public


sphere” as that arena in which the various interests in society engage in discourse
related to the establishment of the normative agenda for society. “In recent times, the
public sphere has been considerably narrowed, to the point that the interests being
voiced tend to be those of hierarchical superiors in business, labor, and the
professions, mediated or administered by the mass media.” (Denhardt,1981).
Habermas seeks a “critical” approach that would aim at revealing the “false
consciousness” which binds us, thus permitting movement toward emancipation.
“Since what appear to us as causal relationships are manifestations of our particular
historical circumstance (rather than natural and unalterable “laws”), they are subject
to alteration by autonomous and responsible action on our part, and that process is
initiated in the act of critique.”

71 | P a g e
Francis X. Neumann, Jr.

Neumann criticizes a recent essay by Robert Behn (1995), entitled “The Big Questions
of Public Management”. He agrees with Behn’s assertion that “any field of science is
defined by the big questions it asks”. However, he feels that Behn incorrectly
identifies the “big questions”.

Behn proposed the following “big” questions:

1. How can public managers break the micromanagement cycle?


2. How can public managers motivate people?
3. How can public managers measure achievement?

Neumann says that these questions are not incorrect or irrelevant, but they are at the
wrong level. Neumann argues that Behn’s questions are not “big” questions at all,
but are for an argument at a much lower level. His three questions are questions of
application, not probes into the origins or basic nature of a discipline.

Neumann proposes his own “big questions” for Public Administration:

1. What is the nature of an organization? Of a “public” organization?


2. How is the public organization related to its environment?
3. What does it mean to manage or to administer the public organization?

Neumann acknowledges that the questions appear to address basic organizational


theory, and that much work has already been done in these areas to answer similar
questions. However, he feels that among the scientific disciplines, “The big questions
are never really completely answered. The big questions, by their very nature, are
multifaceted and extend into dimensions of which we are never fully cognizant at any
one time.” Neumann cautions us that although it may seem that all the major
questions have been put to rest, “in truth we have only produced the answers that
our existing vision has enabled us to find.” What is needed is a paradigm shift in
public administration.

Neumann concludes by suggesting a new paradigm based on non-linear systems


theory. He feels that this world view appears to have great implications not only for
the physical world, but for the social world as well. He adds: “the new paradigm is
that of the nonlinear system. Nonlinear systems theory, “complexity” or “chaos”

72 | P a g e
theory is providing researchers with a new view of both physical and social systems.”
Some of the characteristics of these systems might be considered to be of important
for application to public administration and public management. Neumann gives
guidelines for the development of a new paradigm based on reasons why the older
understanding of organizations may now be inadequate:

 Complex problems require complex mechanisms of solution.


 Attention to the parts of the problem may not solve the whole of the problem.
 Nonlinear systems do not necessarily tend toward equilibrium.
 Mechanisms of positive feedback are widespread and may cause unforeseen
deviation amplification.
 Complexity may develop spontaneously in a system.
 Natural complex systems contain a balance of both random and deterministic
elements.
 Accurate forecasts for future states of the nonlinear system may not be
attainable.

Related COMPS QUESTIONS:

 What is the definition of Public Administration?

 Does Public Administration have a Paradigm?

 Take a paradigmatic stance and make the case on what kind of social scientist
you are. Make a convincing argument.

 Describe in detail the “Identity Crisis” in PA.

73 | P a g e
1.10 PA theory and paradigm and identity Crisis

THE CLASSICAL APPROACH: (Dominant in the U.S. before 1940). The combination
of Wilson’s definition of the field and the Scientific Management and Departmentalist
prescriptions for organizational management and structure (heavily relying on
hierarchy as a primary mechanism for control and coordination) constituted the core
of the Classical approach to PA.

WEBER – Argued that domination is exerted through administration and that legal
domination requires bureaucracy for its exercise.

 Equal application of the law is translated into the equal (and impersonal)
application of the rule in bureaucracy.
 Weber argued that the bureaucrat should stay out of politics and limit himself
to the “impartial administration of his office,” and that he should subordinate
his personal opinion on matters of policy to his sense of duty.
 Called for bureaucrats to be the neutral servants of their political masters. (like
Wilson’s call that administrators should be responsible only for the efficient
execution of the law.)
 Called for the construction of a value-free social science.
 Believed that capitalism represents the highest stage of rationality in economic
behavior.
 Obedience is dependent on the perception of legitimacy. 3 sources of legitimacy
for domination based on authority: charisma, tradition, and legality.
 Weber identifies bureaucracy as the dominant organizational form in a legal-
rational society. Complexity breeds bureaucracy.
 Weber’s “ideal-type” bureaucracy: Administration is carried out on a
continuous basis, not simply at the pleasure of the leader. Tasks are divided
into functionally-distinct areas, each with the requisite authority and sanctions.
Offices are arranged in a hierarchy. Resources of the bureaucratic org are
distinct from those of the members as private individuals. The officeholder does
not own the office, cannot be sold or passed-on, etc. Administration is based
on written documents. Control is based on impersonally-applied rules.
 Weber believes bureaucracy to be the most rational and efficient organizational
form devised by man.

74 | P a g e
 Warned that bureaucracy can become an “Iron Cage”: a “scaffolding for
thought” that limits individual freedom and choices for action, is oppressive,
and determines how action is pursued or conducted.

WILSON (1887) The Study of Administration – Discussed the necessary separation


or dichotomy between politics and administration. Called for the serious study of the
new field of public administration.

Wilson Excerpts

 "It is getting harder to run a constitution than to frame one."


 Wilson insisted that “administration lies outside the proper sphere of politics”
and that “general laws which direct these things to be done are as obviously
outside of and above administration”. He likens administration to a machine
that functions independent of the changing mood of its leaders.
 "It is harder for democracy to organize administration than for a monarchy. The
very fact that we have realized popular rule in its fullness [sic] has made the
task of organizing that rule just so much more difficult....the people cannot
agree on something simple; advance must be made through compromise, by a
compounding of differences by a trimming of plans and a suppression of too
straightforward principles."
 "The object of administrative study is to rescue executive methods from the
confusion and costliness of empirical experiment and set them upon the
foundations laid deep in stable principle.....civil service reform is thus but a
moral preparation for what is to follow....administration lies outside the proper
sphere of politics. Administrative questions are not political questions." (18)
 "A clear view of the difference between the province of constitutional law and
the province of administrative function leave no room for misconception. Public
administration is detailed and systematic execution of public law. Every
particular application of general law is an act of administration. The broad
plans of governmental action are not administrative; the detailed execution of
such plans is administrative."
 "Monarchies and democracies, radically different as they are in other respects,
have in reality much the same business to look to.....we can borrow the science
of administration with safety and profit only if we read all fundamental

75 | P a g e
differences of condition into its essential tenet. We can never learn either our
weaknesses or our virtues by comparing ourselves with ourselves.
 We study administration as a means of making what is democratically politic
toward all administratively possible toward each...we can learn without error
what foreign systems have to teach us."
 "Our duty is to supply the best possible life to a federal organization, to systems
within systems, to make town, city, county, state and federal governments live
with a like strength and equally assured healthfulness."

Frank GOODNOW (1900) Politics and Administration: Goodnow argues that there
are “two distinct functions of government”: Politics – has to do with policies or
expressions of the state will. Administration – has to do with the execution of these
policies.

Frederick TAYLOR (1919) Father of the Scientific Management Movement:


Efficiency and finding the “ONE BEST WAY” through empirical analysis and time &
motion studies.

 Taylor identifies the basic social problem of his day as inefficiency. His
objective is to increase efficiency by capitalizing on the difference between what
can be done by a “first-class” man and by an ordinary worker.
 Taylor’s principles of Scientific Management: 1) The development of a science of
management; 2) The selection and training of the workman; 3) Bringing science
and the workman together; 4) An equal division of work and responsibility
between management and worker.
 Under Scientific Management, science would replace rules of thumb.
 Taylor sought ONE BEST WAY.
 Core of Scientific Management: Time & Motion studies; Wage-Incentive systems,
and Functional Organization.

Leonard WHITE (1926) Introduction to the Study of Public Administration:


FIRST TEXTBOOK DEVOTED IN TOTO TO THE FIELD.

White argued that politics should not intrude on administration; management lends
itself to scientific study; public administration is capable of becoming a “value-free”

76 | P a g e
science in its own right; the mission of administration is economy and efficiency,
period.

The net result of these early authors was to strengthen the notion of a distinct
politics/administration dichotomy by relating it to a corresponding value/fact
dichotomy.

WILLOUGHBY (1927) Principles of Public Administration: SECOND TEXTBOOK


DEVOTED IN TOTO TO THE FIELD.

Argued that certain scientific principles of administration were “there”, that they could
be discovered, and that administrators would be expert in their work if they learned
how to apply these principles.

Luther GULICK - The “Dean of Public Administration”.

 Adopts Wilson’s theme that a science of administration should be constructed,


and that it should be founded on basic principles applicable to both the public
and private sectors. Common objective – to achieve greater efficiency in public
sector operations.
 Gulick’s strongest emphasis is on the enhancement of executive power – both
within orgs and in the executive branch agencies. A strong executive is
required because neither the public nor the legislature is capable of the
planning needed by an effective govt.
 On Federalism: National legislators often enact policies that ignore the needs of
state and local governments. Since it is not possible to achieve a clear
separation of functions in the federal system, he argues for “pragmatic
solutions” in which functions are divided into their local, state and federal
“aspects” and responsibilities assigned accordingly.
 In relations between branches of govt., the executive should plan, propose, and
implement public policies and the legislative should be restricted to review and
approval.
 On Science of Admin: He aspires to the application of scientific methods to
administrative matters. Instead of suggesting that PA or the social sciences
adopt the research standards of the natural sciences, he advocates vigorously
for the application of Scientific Management.

77 | P a g e
 Gulick’s objective is to discover the “principles” or “immutable laws of
administration”, which can be distilled and simplified for practical application
to administrative matters.
 On Professionalism – Gulick is aware of the dangers of scientific “expertise” and
technocracy. The answer is a sense of professionalism that imposes
responsible self-discipline and recognizes that final action cannot be taken by
experts.
 Private administration and Public Administration both belong to a broad
science despite their different objectives and emphases.
 Span of control – Gulick warns that effective span of control is limited at each
level of the org by the knowledge, time, and energy of the supervisor. The span
of control can be extended where work is routine, repetitive, measurable, and
homogeneous in character.
 DICHOTOMY - Gulick breaks from the Classical authors re: the
Politics/Administration Dichotomy. Gulick believes that a separation if
impractical, impossible and undesirable. Instead, we should develop a system
that allows the fullest use of the expertise of the public administrator, including
expertise in policy matters. Administration involve the determination of major
policy (and POSDCORB)
 POSDCORB – The executive organizations should be structured around these
functions, and none should be performed outside the executive office. Gulick
attaches special importance to the Planning component (purpose translates into
programs).
 The administrator’s role is to understand and coordinate public policy and to
interpret policy directives to the operating services, but with unquestioned
loyalty to the decisions of elected officials.
 ADMIN MGMT – Discretion is vital to public officials. Discretion decreases as
one move from the elected official to the technician. The successful
administrator must understand, and be able to deal with, the strategic
dimensions of the system in which he must operate.
 Gulick felt that public and private sectors should become partners in a
cooperative enterprise serving the common good.
 The role of the state should be limited because of uncertainty about the future.

78 | P a g e
 Gulick’s emphasis on unity of command and efficiency as two of the core
principles of organization sprang from a deep-felt desire to develop a science of
administration.

GULICK & URWICK (1937) Papers on the Science of Administration: Considered


the “High-Noon of Orthodoxy” in PA. Called for separation of politics and
administration. POSDCORB – applicable to any type of enterprise. Gulick & Urwick
developed POSDCORB based on the 14 Principles of Management by FAYOL.
Advocated for “one best way” to do this or that.

Urwick and Gulick edited a 1937 publication titled Papers on the Science of
Administration, which included articles on organization theory and public
administration. Gulick isolated the responsibilities of the chief executive and
enumerated them according to the acronym POSDCORB, which stands for planning,
organizing, staffing, directing, coordinating, reporting, and budgeting. One of his main
points was that well-managed, self-contained organizations or departments are nearly
always headed by a single top manager such as a CEO. For his part, Urwick believed
that the activities necessary to achieve organizational goals should be grouped and
assigned to individuals in an impersonal way, echoing the impartial detachment of
Max Weber's bureaucracy.

Urwick also wrote about the problems of managing large numbers of employees,
identified multiple levels of supervisory management, and used a formula to determine
the minimum and maximum number of subordinates a manager can effectively
supervise. His work was an important step in synthesizing the principles of scientific
management with the thinking of Weber and Fayol.

(Wilson, Goodnow, Taylor, White, Willoughby, Gulick & Urwick)

Allof these authors called for the separation of politics and administration. As a result,
PA established itself as a distinct discipline premised on the PRINCIPLES OF
ADMINISTRATION (POSDCORB).

First real hint at conceptual challenge:

Chester BARNARD(1938)The Functions of the Executive. He was an EMPIRICIST,


not a logical positivist.

79 | P a g e
 DESCRIBED ORGANIZATION AS A SYSTEM OF EXCHANGE between the org
and each of its participants. The relationship between the individual and the
organization constitutes a free contractual arrangement.

 DISCUSSED DECISION MAKING IN ORGS. AUTHORITY IS CUMULATIVE IN


NATURE.

 AUTHORITY IS EXERCISED NOT ON THE GIVING OF AN ORDER, BUT


WHEN IT IS ACCEPTED. (BOTTOM-UP APPROACH TO AUTHORITY IN
ORGS.) Authority resides not in a position but in a relationship between a
superior and subordinate.

 Spoke about BALANCING MANAGEMENT AND EMPLOYEE NEEDS.

 Barnard’s ideas fit perfectly with Mayo & Roethlisberger, such as the
assertion that group identity and social recognition are more important
than power and money.

 Barnard describes society as a “complex of informal organizations in


which a network of formal organizations is embedded.”

 IMPORTANCE OF INFORMAL ORGANIZATION: Informal orgs serve an


important function by establishing general understandings, customs, habits,
and institutions, thus creating conditions favorable to the rise of formal
organizations.

 5 Characteristics of Complex Formal Organizations: They are systems; they are


depersonalized, they are specialized, they contain informal organizations, and
they make use of an environment-oriented decision process.

 When discussing the development of executives, Barnard de-emphasizes


intellectual ability and academic training and emphasizes intuition, know-
how, hunches and other Experiential characteristics. He had a considerable
effect on SIMON when he wrote his devastating critique of the field,
Administrative Behavior. Barnard planted the seeds of the logical positivist
perspective. He questioned the basic tenets (principles) propounded by Gulick
& Urwick and Wilson. Barnard argued that complex formal orgs evolve from
and consist of simple formal orgs.

80 | P a g e
 Leadership – Executive capacity in the form of leadership is the most important
strategic factor in human cooperation, a major task of the organization is to see
that those in executive positions are indeed leaders.

 DECISION MAKING – Barnard felt that individuals are limited in their power to
choose by physical, biological, and social factors. Simon’s SATISFICING MAN
model with its bounded rationality is firmly rooted in Barnard’s explanation of
individual behavior. Barnard argued that facts could NOT be logically
separated from values.

 Barnard stressed the need for a “science of organization or of cooperative


systems” that would complement and enhance the power of the “executive arts”.
Barnard hoped this would stimulate work among social scientists. Simon
eagerly took on the challenge.

LINDBLOM (1959) – The Science of “Muddling Through”.

 INCREMENTALISM, MUDDLING THROUGH, MUTUAL ADJUSTMENT


(SPONTANEOUS ORDER) & BARGAINING.
 Spontaneous order, or mutual adjustment, is discernible at societal and
organizational levels in the economic (market) price system, and bargaining – is
the coordinating mechanism. The analytical instrument fitting central planning
and policymaking is the rational-comprehensive model, while incrementalism
best serves as the analytical instrument for mutual adjustment.
 Echoing WEBER, Lindblom argues that the key to hierarchical exercise of
authority is not the command, but LEGITIMATION – that is, the ability to create
the perception that the leader has the right to command and the subordinate
the duty to obey.
 Lindblom contends that hierarchy runs counter to the ideology and ethos of
democracy. Hierarchical orgs are the means by which totalitarian regimes
perpetuate themselves.
 Like WEBER, Lindblom sees bureaucracy as a special case of hierarchy.
According to Lindblom, bureaucracy is an organizational form that is
hierarchical in structure, but it also embodies a bundle of characteristics,
including hierarchical organization, conscious adaptation of means to ends,

81 | P a g e
prescribed and limited discretion, specialization of skill and function, and
separation of ownership from management.
 The basic advantage of bureaucracy is that it is an effective means by which a
relatively small number of people can coordinate the activities of a relatively
large number of people.
 Bureaucratic forms may frustrate efforts at control:
o First, bureaucrats may see themselves as part of a professional,
educated corps of managers with knowledge and experience outweighing
that of their elected superiors.
o Second, bureaucrats are also a part of a particular organization with
which they are likely to strongly identify, and they may be willing to
sacrifice the greater good to the interests of their own organization.
 This thwarting of control is possible both in the public and private orgs.
 Short-term political conditions shape the incremental nature of decision making
much more than rationality.

MUTUAL ADJUSTMENT: LINDBLOM

 “If the market system is a dance, the STATE provides the dance floor and the
orchestra. Polyarchy and democracy are both directed by the same mechanism
– mutual adjustment among competing interests.
 The market system is a method of social coordination by mutual adjustment
among participants rather than by a central coordinator.
 The market offers multilateral controls in an environment characterized by
consumer sovereignty and free choice – by which individuals, driven by self-
interest, seek mutually beneficial exchanges.
 This system of exchange is moderated by the operation of a price system.
 The price system dispenses with the necessity of a central authority and
delegates decisions to a large number of individuals whose decisions are
coordinated without the aid of even a supervisory agency.
 Lindblom asserts that like all controls, those of the price system function
imperfectly – there are “market failures”. Even when the market system
operates perfectly, it may fail to produce optimal results for the following

82 | P a g e
reasons: subject to cyclical fluctuations; and the outcomes of market
transactions can be inequitable and inhumane.
 Given these imperfections, THE EXERCISE OF GOVERNMENTAL AUTHORITY
IS REQUIRED.
 Lindblom argues that all societies are mixtures of governments and markets.
The greatest distinction between one government and another is the DEGREE
to which market replaces government or government replaces market.
 THREE FORMS OF GOVERNMENTAL CONTROL OVER THE MARKET: Direct
authoritative control, indirect manipulation of the markets, and , as a special
form of indirect control, planner sovereignty.
 Lindblom’s FIRST PROBLEM OF POLITICS: How to keep our rulers from
becoming tyrants.
 It is not democracy that offers a solution to the First Problem of Politics, but
POLYARCHY, which is a rough approximation of liberal democracy.
POLYARCHY is a process in which NON-LEADERS exercise a relatively high
degree of control over leaders. POLYARCHY is distinguished from HIERARCHY
by the high degree of control exerted by NON-LEADERS.
 POLYARCHY is distinguished from DEMOCRACY in that control is not shared
equally.

INCREMENTALISM: LINDBLOM

 Incrementalism as a policymaking process unfolded as a sequence of


approximations, or in Lindblom’s terms, a method of SUCCESSIVE LIMITED
COMPARISONS that would allow the decision maker to easily revisit earlier
decisions and rectify them if necessary.
 Incrementalism serves to simplify the decision process by limiting the
conceptual space in which the decision maker operates. It simplifies the
decision process in the following ways:
o Limiting the number of alternatives that are considered, since only those
that are marginally different from previous practice are included.
o Allowing the decision maker to rely on feedback from previous experience
in rendering a decision; and

83 | P a g e
o Managing risk by making the process serial and remedial and thereby
avoiding the possibility of large, irreversible errors.

PARTISAN MUTUAL ADJUSTMENT: LINDBLOM

 Lindblom is best known for incrementalism, but THE REAL CORE OF HIS
THOUGHT is the notion of MUTUAL ADJUSTMENT, which he designated early
on as the “HIDDEN HAND IN GOVERNMENT”.
 Mutual adjustment has origins in Adam Smith’s notion that haggling in the
market is guided by the “INVISIBLE HAND” of the PRICE SYSTEM.
 Much as actors in the market seek out mutually beneficial exchanges, actors in
government seek out areas of agreement as the basis for constructing winning
coalitions. One is reminded of FOLLETT’S CIRCULAR RESPONSE,
EVOCATION, and INTEGRATIVE SOLUTIONS. The motivating force in both
instances is self-interest.
 He believes that mutual adjustment is not only more common than, but also
generally superior to, centralized decision making.
 Lindblom was the first to point out that policymaking and decision making do
not proceed hierarchically and unilaterally, but depend highly on the supportive
organization.
 The criterion for “good policy” is simply agreement. The bargaining process
requires agreement only on the policy itself, not on objectives. This limited
focus simplifies the process because it allows different parties to agree on the
same policy for different reasons.
 Bargaining treats values as central to the process rather than excluding them,
AS SIMON HAD DONE, and it takes us into a palpably political setting.
 Lindblom argues that there are identifiable benefits from the use of bargaining
in the public decision process that parallel those of the market system.
 Bargaining is good because it leads to the realization of the GENERAL
INTEREST.

84 | P a g e
IMPAIRMENT: LINDBLOM

 The problem with the American political process is IMPAIRMENT. The culprit is
BUSINESS, which occupies a privileged position in the American political
process.
 Impairment of thought is the consequence of systematic and continuous
indoctrination and manipulation. Advertisements convey non-and
misinformation, that are designed to move consumers by appealing to emotions,
that thwart rational thought, and that obfuscate.
 Lindblom argues that more than ever before, humankind is crippled in its
capacity to think critically and independently about social problems.

THE BEHAVIORAL APPROACH: Entails the study of actual behavior, usually taking
the individual as the preferred unit of analysis. This approach calls for “rigor” in the
use of scientific procedures; and it is primarily descriptive in intent. Behavioralism
incorporates a diversity of perspectives, including the Human Relations Movement,
Simon’s model of decision making, organizational humanism and contingency
theory. The major thrust of the Behavioral approach was organization structure and
management, not definition of the field. The Behavioral approach sought to modify the
hierarchical org structures so ardently espoused by the Classical authors.

Whereas the Classical approach emphasized executive decision making


responsibilities, the Behavioral approach argued for more participatory decision
making procedures. Classical argued for centralization for control and coordination,
Behavioral argued for decentralization to give more members of the organization a
greater sense of control over their own destinies.

If Follet, Mayo and Barnard were precursors to the Behavioral approach, Simon was an
important part of the movement itself. Simon’s distinctive contribution was to shift the focus of
analysis to decision making in the organization.

Mary Parker Follett – Distinguishing feature of her work is her treatment of Social
Conflict. Follett argues that conflict itself is neither good nor bad, but simply
inevitable. If it is used to produce ideas and solutions, then it is good. If it results in
domination by one side or compromise in which both sides simply yield something, the
results will be unsatisfactory.

85 | P a g e
 Social Conflict - Follett argues that we can use conflict to produce harmony,
not simply victory or accommodation.
 Means for the resolution of social conflict: Domination, Compromise, and
Integration. Domination is flawed because only one side wins in a dispute.
Compromise is also unsatisfactory because each party has to give something
up, resulting in temporary change only. Integration is Follett’s preferred
method of resolving social conflict. Circular Response is the process by
which integration is achieved. Progress is never truly achieved, but something
that advances with continued integration and reciprocal interaction.
 Power & Control: The desire for power is a predominant feature of life and
power is always unequally distributed. Rather than abolishing power,
integration through circular response both transforms power and increases it.
Power is transformed from “Power Over” to “Power With”. In an org setting,
this means that managers should give workers a chance to grow capacity
or power for themselves. This Integration, resulting in “Power With” is
akin to today’s “Empowerment” as seen in movements like TQM.
 Group Process & Group Dynamics– A central theme, Follett argues that
individuals achieve their true expression in group relationships. She considers
the State to be both a logical extension of the group process and its
highest expression; both the group and the State serve a purpose greater than
individual interests.
 The supreme function of the state is the moral ordering of social
relationships.
 MGMT – She argues that control in the organization is pluralistic and
cumulative (that is, arises from below), as opposed to the conventional view that
control should be concentrated in the apex of the org pyramid and cascade
downward.
 Authority should flow from the “Law of the Situation” (the objective
demands of the work situation), rather than being based on personal
imposition. Authority is cumulative in nature, and comes from the bottom-
up.
 Authority resides in a function (not a position), is derived from the demands of
the situation, and is the result of circular behavior.

86 | P a g e
 Leadership is the ability to create functional unity in the org through the proper
correlation of controls, rather than personal power to command based on
position.
 Circular Response – To Follett, the reality of org behavior cannot be captured
in subject or object alone, but must be seen in the interaction between subject
and object, since each in a function of the other.
 Ballot-Box democracy is not real democracy. It relies on brute numbers
rather than on a genuine union of interests; it is based on the “law of the
crowd”, which employs suggestion of persuasion as its primary technique. The
tragedy of democracy is not that we have no public opinion, but that we think
we do. What is needed is not consent, but co-action.
 On Professionalism and the Role of the Expert – Democracy requires both
expert advice and an active electorate, and the findings of experts must be
balanced against the ideas of others.
 FOLLETT CONCLUDES: “WHATEVER HEIGHTENS SELF-RESPECT
INCREASESS EFFICIENCY”. Closely related to this is pride in one’s work.
 “DEPERSONALIZING ORDERS IS ONE OF THE LARGEST CONTRIBUTIONS OF
SCIENTIFIC MANAGEMENT”. Follett shows that by changing the way we give
“orders”, or be depersonalizing orders, both manager and employee can work
more efficiently together addressing the “Law of the Situation”.

Elton MAYO – Mayo echoes two of Follett’s basic themes: The importance of the
Group Process and the Cumulative Nature of Authority in the organization.

 Founder of the Human Relations approach.


 Unlike Follett, Mayo feels that conflict is pathological and should be
avoided.
 Mayo argues that modern society suffers from a breakdown of the social
routines of traditional society. Mayo asserts that the problem of social
disorganization has been exacerbated by an economic theory that emphasizes
competition in the pursuit of individual self-interest and by a political system
that heightens the level of social conflict by playing to the fears of the masses.
Politics has become part of the problem.

87 | P a g e
 INFORMAL RELATIONSHIPS – Management has failed to perceive the
importance of informal social groups, which are a response to a basic human
need. Informal relationships, communication and cooperation are more
important than technical logic and the material interests of the individual.
 Management must learn to appeal to individual and social emotions and
attitudes in a more intimately human way. A key ingredient in this effort
is the supervisor, who is the human interface of the org, and who plays an
important role in supporting interpersonal relationships and team morale.
 According to Mayo, societies die because of ignorance. The particular ignorance
of modern society is its ignorance of human nature.
 Mayo’s empirical research led him to concentrate on the role of the social
group within the organization in determining the individual’s attitude
toward work.
 Mayo’s work was largely responsible for a major shift in the study of
organizations. His concern with the attitudes and sentiments of the
worker, the importance he attached to the social group in determining
individual behavior, and his search for “knowledge-of-acquaintance” based
on direct observation, all served as inspiration for future scholars.

Herbert SIMON (1946 ) Proverbs of Administration (pointing to the fundamental flaws


in the principles approach – and his 1947 Administrative Behavior- A devastating
critique of the field. Triggered the debate with Waldo and won him the Nobel Prize for
transforming the way economists and others perceived rational behavior. Simon said
PA has evolved as both as a basic and applied science.

Simon called for LOGICAL POSITIVISM. He was not “anti-scientific”, quite the
opposite. Simon sought to save PA from “bad science” and the pseudo-scientific
approaches of Taylorism, the Human Relations Movement and the “principles of
administration”. Simon was proposing a more credible social science approach for the
field. An important feature of Simon’s approach was its roots in the Logical Positivist
approach. Simon studied logic under Rudolph Carnap (member of the Vienna Circle).
Carnap offered a clear vision of what constitutes a “science”: the presentation of
knowledge in empirically verifiable statements untainted by the bias of values or
ethical statements. It is a position Simon adopts in the final chapter of his

88 | P a g e
dissertation Administrative Behavior: “science is interested in sentences only with
regard to their verification.”

Simon replaced the Wilsonian politics/administration dichotomy, and offered in its


place the fact-value distinction of logical positivism. He demonstrated the “principles”
to be rules-of-thumb, folklore, but held out the hope of arriving at empirically based
knowledge that would pass the test of true science.

Simon adopted major aspects of Barnard’s work, including his ideas on decision
making in orgs. He agreed with Barnard’s conceptualization of the organization
as a system of exchange and the definition of authority as a bottom-up process.

Simon called Gulick & Urwick’s “Principles” nothing but “Proverbs”. Simon effectively
demonstrated that for every “principle” of administration advocated in the literature,
there was a counter-principle, thus rendering the very idea of principles moot.

BOUNDED RATIONALITY & SATISFICING (LOGICAL POSITIVIST PERSPECTIVE).


Simon felt that at the basis of administrative organization is the concept of rationality.
He felt that absolute or pure rationality could not be achieved; only approached.
Because true rationality cannot be achieved, an individual is limited (bounded) in his
perceptions of reality. Due to their limitations, individuals find it necessary to
SATISFICE (he makes limited decisions that are merely satisfactory and sufficient for
the situation.

Simon offered an alternative to the old paradigms (politics/admin dichotomy,


principles of administration). For Simon, a new paradigm for public admin should
involve TWO KINDS OF ADMINISTRATORS WORKING IN HARMONY – Those
concerned with developing a “pure science of administration” based on “a thorough
grounding in social psychology”, and a larger group concerned with “prescribing for
public policy”. Simon agreed with Barnard that complex formal orgs evolve from
and consist of simple formal orgs. Simon’s SATISFICING MAN model with its
bounded rationality is firmly rooted in Barnard’s explanation of individual
behavior.

DAHL – 1946 – PAR article backed-up Simon by highlighting the major obstacles
facing any effort to establish a science of administration based on general principles.

89 | P a g e
Dwight WALDO – 1947

 Waldo represented the ADMINISTRATION-AS-POLITICS approach – which,


combined with the BEHAVIORAL APPROACH, constituted a devastating
critique of the CLASSICAL paradigm of public administration.
 Waldo’s “The Administrative State” attacked the “GOSPEL OF EFFICIENCY”
that dominated administrative thinking before WWII.
 WALDO DENIED THE POSSIBILITY OF CONSTRUCTING A SCIENCE OF
PUBLIC ADMINISTRATION, doubted the existence of “PRINCIPLES OF
ADMINISTRATION”, questioned the plausibility of a unified theory of
organizations, mistrusted those who would indiscriminately intermingle
POLITICS AND ADMINISTRATION, and despaired of reaching a common
agreement on a definition of the field of public administration.
 Waldo denies that politics and policy considerations can be excluded from
administration. Moreover, FACTS cannot be separated from VALUES.
Consequently, ADMINISTRATION IS BOTH AN ART AND A SCIENCE, and
perhaps more art than science.
 Also, since administration cannot be separated from politics, Waldo argues that
public administration is different from private administration, being
distinguished by the political environment in which the public administrator
must operate.

WALDO – ON THE PA DICHOTOMY

 Waldo denies that politics and policy considerations can be excluded from
administration. Moreover, FACTS cannot be separated from VALUES.
Consequently, ADMINISTRATION IS BOTH AN ART AND A SCIENCE, and
perhaps more art than science.

WALDO – CRITIQUE OF THE CLASSICAL APPROACH

 Waldo criticized TAYLOR for regarding his laborers essentially as draft animals.
 Mayo’s Hawthorne experiments resulted in a “much more subtle and
sophisticated paternalism.”

90 | P a g e
 Waldo argued that we should move toward a philosophy that encourages
cooperation among powers, be they administrative or political, not competition
among separated powers.
 Waldo criticizes both the emphasis on supposed principles or commonalities
among organizations and the rationalist bias of Classical organization theory.
Organizations should be defined and structured to meet purposes, not general
principles, and the organizational form and process actually adopted should be
suited to the specific situation confronting the organization.
 Waldo argues that the Classical approach ignores the irrational and informal
aspects of organizations. Regarding WEBER’S concept of bureaucracy, Waldo
argues that Weber placed undue emphasis on the functional side of
bureaucracy, ignored the informal and socioemotional aspects of organizations,
and elevated position over knowledge as the basis for hierarchical authority.
 Waldo is critical of the possibility of developing a science of administration. He
contends that PA must deal with thinking and valuing human beings and that
the techniques of science are inappropriate to such subjects. Values cannot be
treated scientifically, and human free will means that the principles of
mechanical cause and effect are inapplicable.
 Critique of “efficiency” – Waldo argues that the idea of efficiency itself be came
imbued with a moral significance, and the Classical approach was originally
intended to replace a moralistic approach to public administration.
 In Waldo’s judgement, the Classical approach contains much truth and
represents an intelligent response to a historical situation. Moreover, the
Classical approach is still deeply engrained in our culture, and no consensus
on an alternative has yet emerged to replace the Classical paradigm of PA.

WALDO – ORG THEORY:

 Waldo divides the development of Org Theory into three stages:


o Classical period – epitomized by the works of TAYLOR, GULICK,
FAYOL. Classical period based on the “machine model” of the
organization and emphasized the rational aspects of human behavior.
o Neo-Classical period – Began with Hawthorne experiments in the
1920s and the following Human Relations Movement. In contrast to

91 | P a g e
the Classical stage, the neoclassical approach emphasized the emotive
and sociopsychological dimensions of human behavior in orgs. The
Human Relations Movement focused on morale, perceptions, attitudes,
group relationships, informal groups, leadership and the bases for
cooperation in org behavior. The Human Relations Movement
demonstrated the limitations of perspectives such as Scientific
Management.
o Modern Org Theory – Began with publication of March & Simon’s
Organizations in 1958. Modern Org Theory is based on an “organic” or
“natural system” model of the organization and stresses organizational
growth and survival. Simon’s work on decision making is but one aspect
of modern org theory. Waldo considers Simon’s work to be a radical
departure from Classical Org Theory. A second perspective on modern
org theory is ORGANIZATIONAL HUMANISM, represented by works of
CHRIS ARGYRIS, WARREN BENNIS, and RENSIS LIKERT. Waldo
observes that the ORGANIZATIONAL HUMANISM focuses on much the
same kinds of concerns as the Human Relations Movement. From the
humanist perspective, the objective is to achieve organizational
effectiveness and self-fulfillment simultaneously, under the assumption
that the interests of the individual and the organization are compatible.
A third strand of Modern Org Theory is the “scientific and managerial”
literature (contingency theory, system perspectives).
o Waldo charges that modern org theory presents no unified “theory
of organization”. By becoming everything, systems theory runs the
danger of becoming nothing in particular.
o “A value-free theory of organizations is unattainable.”

WALDO ON PUBLIC POLICY:

 Waldo is connected with the NEW PUBLIC ADMINISTRATION movement (He


organized the 1968 MINNOWBROOK conference). The basic themes of the New
Public Administration were participation, decentralization, and representative
bureaucracy.

92 | P a g e
 Waldo argues that VALUES should be studied consciously and should be
used to give direction to empirical research.

Dwight WALDO – 1947 –The Administrative State. Came out just after Simon’s
Administrative Behavior. If Simon represented the “hard” side of the social sciences,
Waldo represented the “softer” approaches.

 As bounded rationality became dominant concept to describe human


decision making, the administrative state became the dominant concept
to describe modern government.
 Central Issue of our time: the potential conflict between bureaucracy and
democracy.
 Waldo argues that bureaucratic organization is not totally incompatible
with the concept of democracy, for bureaucracies provide important
support for democratic values, such as the universalistic criteria and
opportunity based on ability and expertise.
 “The solution to the problem is to seek an optimal mix of democracy and
bureaucracy, recognizing that while democracy is desirable, bureaucracy is
necessary.”
 Called for a return to a value-centered approach to the discipline of PA. He tried
to force the discipline away from logical positivism. While in agreement with
Simon on the shortcomings of scientific management and the “principles”
approaches, Waldo was more skeptical of efforts to rely solely on logical
positivist methods in development of a theory of PA.

Waldo was critical of the popular conception of science dealing with “facts” alone, and
that it is possible to apply the empirical methods of the physical sciences to human
affairs without taking into account the cultural and social context of the observed
phenomena.

Waldo believed that PA must give priority to carefully and critically examining
normative theories rather than generating the kind of empirical theories advocated by
logical positivist approaches.

Waldo sponsored the MINNOWBROOK I conference and resulting book – Toward a


New PA. Several attendees were frustrated that PA was being increasingly identified

93 | P a g e
as simply a profession, and they argued that PA should have full disciplinary status as
an empirical social science. The desire of obtaining disciplinary status was pervasive
at the conference, but so was the desire to maintain the normative standards central
to Waldo’s approach.

94 | P a g e
1.11 The SIMON-WALDO Debate: Empiricism vs. Normative Theory
SIMON argued that the allegedly scientific principles characteristic of the orthodox
classical tradition were contradictory proverbs.

Waldo’s description of SIMON – “Simon did not argue the futility of trying to study
administration scientifically, but rather that this was being attempted improperly.”
The basic defect of orthodox classical public administration was its lack of scientific
sophistication.

SIMON - In practice, it is impossible to separate FACTS from VALUES. Simon argued


that if we could invent procedural devices permitting a more effective separation of the
factual and ethical elements in decisions, we could better approximate government
responsibility in a democratic modern govt.

Empiricism vs. Normative Theory

See Dubnick, Melvin (1999). Early on in PA, an intellectual consensus built around
the “Classical” approach (scientific management and the “principles” of
administration). This dominated the field until the end of WWII. The postwar attack
on that consensus came in the form of criticisms from two directions:

SIMON – who sought to create a social science focused on administration and PA


as its own DISCIPLINE within the social sciences, or as a sub-field of Political
Science.

WALDO – who sought to create a normative agenda for the field – more as a
PROFESSION than a discipline.

Waldo rejected the two traditional alternative solutions: sub-disciplinary status within
political science, and status as a distinct discipline among the Social sciences.
Instead, Waldo advocates for a profession. “We try to act as a profession without
actually being one”. As an analogy –Waldo looks at the field of Medicine, where
science and art, theory and practice, study and application are included under the
umbrella of a profession. Not based on a single discipline, but utilizes many. Not
united by a single theory, and is justified and given direction by a broad social
purpose.

95 | P a g e
Any effort to resolve the identity crisis must encompass that strong
commitment to purpose.

Waldo explicitly attacks the assertion that decisions can be analyzed without
reference to values.

Simon criticized the “loose, literary style that political theorists” adopt. Simon
characterized his critics as “political theorists” whose criticisms were faulty and
lacking in rigor.

Chicago School behavioralism – Public Admin scholars trained at Univ. of Chicago.


Charles Merriam and others were advocates for an empirical political science. While
stressing the need to apply scientific methods to the study of politics and
government, the behavioralists at Chicago were also progressives and New Dealers
committed to political change.

MINNOWBROOK I – Waldo sponsored conference and resulting book – Toward a New


PA. Several attendees were frustrated that PA was being increasingly identified as
simply a profession, and they argued that PA should have full disciplinary status as
an empirical social science. The desire of obtaining disciplinary status was pervasive
at the conference, but so was the desire to maintain the normative standards central
to Waldo’s approach.

THE BLACKSBURG MANIFESTO – 1980s – maintained the Waldo-inspired aversion


to endorsing a social science disciplinary identity for the field. They blamed Simon
and the positivist/behavioralist movements in political science and organization theory
for diverting PA theory “into an intellectual cul-de-sac” and creating “tacit boundaries”
that took decades to overcome.

MERTON – Criticized WEBER’s IDEAL-TYPE BUREAUCRACY – has inhibiting


dysfunctions which prevent it from being optimally efficient. Bureaucracy also
has negative effects on people.

SELZNICK (1948) – Foundations of the Theory of Organization: TVA and the


Grass Roots.

 Orgs consist not only of positions for management to control, but of individuals
with goals and aspirations that may not coincide with the org’s formal goals.

96 | P a g e
 COOPTATION – Process of bringing in and subsuming external elements into
the org’s policy making process to prevent these external elements or orgs from
being a threat.

Rudolph Carnap – Logical Positivism (member of the Vienna Circle). Carnap offered
a clear vision of what constitutes a “science”: the presentation of knowledge in
empirically verifiable statements untainted by the bias of values or ethical statements.
It is a position Simon adopts in the final chapter of his dissertation

Administrative Behavior: “science is interested in sentences only with regard to their


verification.”

MOSHER – 1956 – Wrote about the “boundary-spanning” problems in PA. The


breadth of PA and its methodological diversity creates unusual problems in spanning
several boundaries (academic disciplines, others in PA, and to the real world). Mosher
lamented the absence of systematic ways for those in PA to keep abreast of relevant
developments in other fields and the ignorance of PA research within other fields.
Mosher wrote about the need for strengthening codes of ethics for elected officials in
the wake of the Watergate scandal.

HENRY – 1975 - 5 Paradigms. 1) The Politics/Admin. Dichotomy; 2) The Principles of


Administration; 3) Public Admin. as Political Science; 4) Public Administration as
Administrative Science; 5) Public Administration as Public Administration.

John ROHR – (1978, 1986) – Made important contributions to the understanding of


ETHICS in public service.

 In Ethics for Bureaucrats, Rohr argues that the fundamental ethical problem
facing PA was in exercising discretionary authority, a problem that could be
overcome by clearly understanding the values of the regime.
 In To Run a Constitution, Rohr performs a detailed analysis of the constitutional
legitimacy of the administrative state as a prelude to a normative theory of
public administration in a constitutional context. Rohr sees the agencies of
government as subordinate to the three branches of government but
simultaneously able to balance the various interests expressed there. Public
administrators, key actors in this pluralistic balancing act, are to uphold the

97 | P a g e
Constitution – to use “their discretionary power in order to maintain the
constitutional balance of powers in support of individual rights.”

James MARCH - 1966 – “The Power of Power”.

 Explores approaches for empirically studying social power in orgs and


communities.

CYERT & MARCH - 1963 – BEHAVIORAL THEORY OF THE FIRM.

 Focus on POWER and POLITICS of establishing organizational goals. Discussed


the formation of coalitions and negotiations to impose coalitions’ demands on
the org.
 Corporations tend to “SATISFICE” rather than engage in economically rational
profit-maximizing behavior.

98 | P a g e
1.12 Major authors in PA theory
WILSON (1887) The Study of Administration – Discussed the necessary separation
or dichotomy between politics and administration. Called for the serious study of the
new field of public administration.

 "It is getting harder to run a constitution than to frame one."


 Wilson insisted that “administration lies outside the proper sphere of politics”
and that “general laws which direct these things to be done are as obviously
outside of and above administration”. He likens administration to a machine
that functions independent of the changing mood of its leaders.

Frank Goodnow – 1900 - Politics and Administration. Goodnow argues that there are
“two distinct functions of government”: Politics – has to do with policies or
expressions of the state will. Administration – has to do with the execution of these
policies.

Taylor – 1919 – Father of the Scientific Management Movement. Efficiency and finding
the “ONE BEST WAY” through empirical analysis and time & motion studies.

Leonard White – 1926 – Introduction to the Study of Public Administration. FIRST


TEXTBOOK DEVOTED IN TOTO TO THE FIELD.

White argued that politics should not intrude on administration; management lends
itself to scientific study; public administration is capable of becoming a “value-free”
science in its own right; the mission of administration is economy and efficiency,
period.

WILLOUGHBY (1927) Principles of Public Administration. SECOND TEXTBOOK


DEVOTED IN TOTO TO THE FIELD. Argued that certain scientific principles of
administration were “there”, that they could be discovered, and that administrators
would be expert in their work if they learned how to apply these principles.

The net result of these early authors was to strengthen the notion of a distinct
politics/administration dichotomy by relating it to a corresponding value/fact
dichotomy.

99 | P a g e
GULICK – Called for a science of administration.

 Popularized the “principles” approach (orthodox) to managing organizations.


 POSDCORB

SIMON – Rational Model of Administration. Administrative Control. Logical


Positivism. Satisficing Man.

 Simon shares with the Classical approach the objective of developing a science
of administration, an effort to describe a value-free domain for the construction
of that science, a quest for general principles of administration, the acceptance
of efficiency as the criterion for decision making, and an emphasis on hierarchy
as well as its justifications (coordination, superior rationality, and the location
of responsibility).
 Simon was determined that the fact-value dichotomy was the appropriate
substitute for the politics/administration dichotomy in defining the domain for
the construction of a science of administration.
 Simon was PREOCCUPIED WITH DECISION MAKING IN THE ORGANIZATION.
 Economic Man is translated by Simon into Satisficing Man, who has features of
both Economic and Administrative Man.
 Simon argues that uncertainty pervades the decision process.
 Simon argues that the definition of RATIONALITY should be expanded to
incorporate a wider range of human behavior.
 Simon argues that hierarchy simplifies the decision making task. He contends
that hierarchy is THE adaptive form for finite intelligence to take in the face of
complexity.
 Simon was concerned about GOAL DISPLACEMENT regarding the rationality of
organizational decisions.
 Simon’s proposed foundation for the construction of a science of public
administration is the FACT-VALUE dichotomy, by which he sought to describe a
value-free domain in which scientific investigation could be conducted.

DAHL – 3 Problems in the study of public administration:

 Lack of appreciation of cultural and historical context. (studies diff. in diff


countries, not just one study of PA that transcends all boundaries).

100 | P a g e
 Managerial approach to the study and field risks forgetting the HUMAN
FACTOR.
 Managerial approach tends to downplay the NORMATIVE NATURE OF
CHOICES MADE IN THE ADMINISTRATIVE STATE.

MOSHER – (1956)

 Wrote about the “boundary-spanning” problems in PA. The breadth of PA and


its methodological diversity creates unusual problems in spanning several
boundaries (academic disciplines, others in PA, and to the real world). Mosher
lamented the absence of systematic ways for those in PA to keep abreast of
relevant developments in other fields and the ignorance of PA research within
other fields. Wrote about the need for strengthening codes of ethics for elected
officials in the wake of the Watergate scandal.

MOSHER – (1982) Democracy and the Public Service.

 Provided the foundation for understanding how public service has developed in
the U.S. and how merit and collective systems impact democracy and the public
interest.

Nicholas HENRY – 1975 - 5 Paradigms.

 The Politics/Admin. Dichotomy; 2) The Principles of Administration; 3) Public


Admin. As Political Science; 4) Public Administration as Administrative
Science; 5) Public Administration as Public Administration.

LOWI – Critical assessment of the Chicago School behavioralism.

FRANK MARINI – 1971

Toward A New Public Administration: The Minnowbrook Perspective. Noted that


debates about the Identity Crisis in PA are common.

H. George FREDERICKSON – The Public Admin Theory Primer

 Dichotomy of Politics & Admin is too simplistic. Depending on situation, may


range on a continuum from Politics-Admin.
 Scholars pushing Scientific Management and Efficiency also believe that PA is
centered on DEMOCRACY, which is one of the most inefficient methods. This is

101 | P a g e
the reason why a scientific field of administration never came about – because
no one can resolve this discrepancy.
 Governance involves the particular characteristics and activities and policy
areas involved in solving problems and addressing issues.
 LEVELS OF GOVERNANCE:
o Institutional Level – How problem is articulated - formal and informal
rules.
o Organizational Level (Managerial Level) – Admin. Discretion, measuring
performance, running bureaus, depts., commissions.
o Technical Level – solving the task at hand. Efficiency & accountability at
the individual level.

H. George FREDERICKSON - (1997)– The Spirit of Public Administration.

 Focused on the role of Public Administrators as representatives of citizens, as


in looking out for the public good. In addition, what is the role of the public
administrator as a REPRESENTATIVE CITIZEN, as they are citizens
themselves.

Graham ALLISON – (1972) Essence of Decision. –

 Book tried to explain why the governments of U.S. & Soviet Union did what they
did during the Cuban Missle Crisis. “Why do governments do what they do?”
 Allison created a Model of Bureaucratic Politics:
o Rational Actor model (Classical Model) – Focuses on single actors in
decision making.
o Organizational Process Paradigm – Focus on multiple actors.
o Bureaucratic Politics Paradigm – Combination of Bargaining and
Compromise.

Paul LIGHT – (1997) The Tides of Reform.

 The book is based on the notion that there is not too little management
reform in government, but TOO MUCH. Congress and the Presidency have
moved effortlessly from one reform philosophy to another and back again, rarely
questioning the contradictions and consequences of each separate act. Light

102 | P a g e
used the Congressional Quarterly Almanac to identify and analyze 141 federal
management statutes signed into law from 1945 to 1994.
 Four separate philosophies or Tides of Reform:
 Scientific Management – efficiency, principles of administration, experts,
executive control (Brownlow Committee & First Hoover Commission).
 War on Waste – economy, audits, investigations, generally accepted practices
(e.g. welfare fraud hearings).
 Watchful Eye – Goal: fairness, rights, information, whistle-blowers, congress
and the courts (e.g. Vietnam & Watergate).
 Liberation Management – Goal: Higher performance. Standards, evaluation,
outcomes, employees, teams (e.g. Gore’s National Performance Review)

Guy PETERS (1984) – The Politics of Bureaucracy

 Peters argues that PA is an integral part of the political process.


 The traditional Politics/Administration Dichotomy has eroded and will erode
further as the public bureaucracy plays a greater role in the policy process.
 Peters argues for a COMPARATIVE approach to PA.

PETERS – The Future of Governing.

 FOUR MODELS OF GOVERNMENT:


o MARKET
o PARTICIPATIVE
o FLEXIBLE
o DEREGULATED

John KINGDON (1984) - Policies get made into law by capitalizing on “WINDOWS” of
opportunity. TIMING IS EVERYTHING.

 Much of policy analysis has focused on the theoretical problem of agenda


building and policy development.

Mary Parker FOLLETT – (1926) The Giving of Orders.

 Follett argued for a Human Relations approach. She anticipated the


Hawthorne/Mayo findings.

103 | P a g e
 Advocated for PARTICIPATORY MANAGEMENT and the merits of exercising
“POWER WITH” as opposed to “POWER OVER”.

OSTROM – Identity Crisis in PA. The Intellectual Crisis in PA – Importance of Political


Structure.

 OSTROM & OSTROM WERE BRUCE ROGERS’ PROFESSORS AT IU.


 OSTROM – BIG, BIG NAMES IN POLITICAL ECONOMY.
 Embraced the idea of SELF-GOVERNMENT
 Ostrom is VERY critical of Bureaucracy and points out numerous failures.
 Argues that PUBLIC CHOICE THEORY is the way of the future and that PA
should embrace the concept as modern and revolutionary. (Niskanen, Tullock,
Downs – used econ & mathematical models to demonstrate bureaucratic
wastefulness and irresponsibility – calling for limiting government).
 Public Choice Theory is similar to New Public Administration and Reinventing
Government movements of the 1990s, though these movements went further by
promoting public-private partnering, outsourcing, and privatization as a means
to greater “marketization” of public goods than had been advocated by earlier
Public Choice theorists.
 Public Choice Theory: Individual is the unit of analysis. 4 Public Choice
Assumptions about individuals:
o Self-Interested
o Rational
o Benefit-Maximizing
o Uncertainty-Minimizing
 IDENTITY CRISIS IN PA:
o Ostrom argues that there is a persistent crisis in PA starting with the war
years, which provoked a challenge from which the field of PA has not
recovered. Wartime control measures were plagued by persistent
failures.
o Ostrom cites DWIGHT WALDO’s 1967 work characterizing the crisis in
confidence in PA as a CRISIS OF IDENTITY: “Both the nature and
boundaries of the subject matter and the methods of studying and
teaching become problematic. Now, two decades after the critical

104 | P a g e
attacks, the crisis of identity has not been resolved.” “What I propose is
that we try to act as a profession without actually being one.”
o Ostrom suggests a new paradigm in Public Choice Theory. Although his
book is considered a classic, Public Choice has not replaced the
traditional theory of bureaucracy.

NISKANEN – 1971 – Economic approach to bureaucratic behavior. Market advocate,


Anti-Bureaucracy.

Paul APPLEBY – Policy & Administration, Big Democracy.

GOLEMBIEWSKI (1967, 1985) – Organizational Learning and Organizational


Development.

 (1967) Men, Management, and Morality.


 (1985) Humanizing Public Organizations – Its theoretical importance to the field
of public administration lies in its analysis of the relationship between
bureaucracy and democracy, a moral and political question central to
Golembiewski’s 1967 Men, Management, and Morality.

Michael LIPSKY – (1980) Street-Level Bureaucracy.

 Lipsky first brought the term “street-level bureaucracy” to the attention of the
field by pointing out that public policy is determined not merely by legislators
and managers at high levels of government but by the police officer, the nurse,
and the welfare worker (among others), who engage in the direct delivery of
services.
 Lipsky writes that “the decisions of the street-level bureaucrats, the routines
they establish, and the devices they invent to cope with uncertainties and work
pressures, effectively become the public policies they carry out.”
 Finding that systemic constraints on the professional practices of street-level
bureaucrats result in confusion and conflicting demands, Lipsky offers several
important suggestions for reform, suggestions addressed to the concerns of
street-level bureaucrats but not without relevance to those at all levels of public
agencies.

BROWNLOW COMMISSION – (1937) Brownlow, Merriam and Gulick. (Report of the


President’s Cmte on Admin. Mgmt.)

105 | P a g e
 Eager to remove politics from administration, the report recommended a
distinction between the executive and administration.
 Culminated in the Reorganization Act of 1939, creating the Executive
Office of the President, BOB, and authorized the President to submit future
reorganization plans to Congress.

The HOOVER COMMISSION – (1949) The Report of the Commission on Organization


of the Executive Branch.

 Charge: To reduce the size of government. Instead, it aimed to strengthen the


executive branch via reorganization through purpose for each department and
better control by president.
 Unified discretion over presidential staff, strengthened Bureau of the Budget,
created an office of personnel, created a staff secretary to mediate between
President and staff.

MINNOWBROOK I – subsequent paper – The New Public Administration.

 Minnowbrook I was held to discuss the field’s desires to “move values and
norms to a central position in theory and practice.”
 The terms “The New Public Administration” or “The Minnowbrook Perspective”
refer to “a commitment to greater social equity” in the practice, theory, and
discussion of PA.

THE BLACKSBURG MANIFESTO – 1982. in Wamsley, et. al., Refounding Public


Administration.

 In response to the continuing practice of “bureaucrat bashing” by presidential


candidates, Gary Wamsley convened the Blacksburg Conference.
 The paper resulting from the meeting was The Blacksburg Manifesto.
 Calls for a shift in the current political dialogue about PA and to encourage
future work on a normative theory.
 Calls for a redefinition of the public administration to The Public
Administration, an institution with a legitimate role in governing as provided
by the Constitution and as envisioned by the Founders.
 Wamsley refers to The Blacksburg Perspective as “Minnowbrook I with
institutional grounding”.

106 | P a g e
 Calls for redefining the role of the public administrator as Constitutional
Officer, with legitimate powers similar to those found in the U.S. Senate.
 Authors encourage new thinking toward a normative theory of PA and to a
theory of the American state that is both positive in its outlook and normative
in nature.
 Public service is NOT a JOB, but a CALLING.
 If The Public Administration asserts it rightful claim to be a constitutionally
legitimate participant in the governance process, it can contribute to the
correction of a major defect in the Constitution: its unsatisfactory resolution of
the problem of representation.

John ROHR – To Run a Constitution: The Legitimacy of the Administrative State. (from
Ch. 2 in Wamsley, et. al. Refounding Public Administration).

 Central Thesis: The Administrative State was contemplated by the Founders


and is implicitly recognized in the framing of the Constitution.

 According to Rohr, an in-depth understanding of our founding principles is


critical for administrators who must discharge their duties in accordance with
an oath to uphold the Constitution.

 Rohr finds support for the ideas in the Blacksburg Manifesto in the Federalist
Papers. The concern for a sound public administration was one of the Framers’
most serious concerns.

 Rohr argues that the Founders would have been upset at the current practice
of Presidents removing subordinates from office and appointing their own
“team” without first obtaining the consent of the Senate.

 Rohr argues:

o Administrative institutions are not inconsistent with the


constitutional principle of separation of powers (considering that
they perform all three basic functions of government: making,
applying and adjudicating rules), noting that the Senate also uses all
three powers of government.

107 | P a g e
o The higher reaches of the career civil service fulfill the framers
original intent for the Senate.

o That the entire career civil service provides a remedy for a serious
defect in the Constitution (the inadequate representation that so
distressed the Anti-Federalists of 1787-1788).

 He then argues that administrators should legitimately participate in


governance, not simply as administrative managers, but as representative
decision makers for the people. “Members of Congress, like other officers of
government, derive their authority from the Constitution, not from their
election. Elections are merely a method of choosing, not a method of
authorizing.”

 Rohr concludes by proposing a normative theory of Public Administration


that is grounded in the Constitution. He argues: “The Senate, like the Public
Administration, was intended to exercise all three powers of government.
However, the Public Administration exercises all three powers in a subordinate
capacity, and must make its peculiar contribution in conformity with that
subordination. They must learn to think like judges as well as legislators and
executives, because they are all three of these.”

Charles GOODSELL – (from Ch. 3 in Wamsley, et. al. Refounding Public


Administration).

 Goodsell argues that the public interest is the standard that guides the
administrator in executing the law.
 Theorists in Public Administration have largely ignored “the public interest” in
favor of defining administration in terms of economic efficiency and managerial
control.

GOODSELL (1984) The Case for Bureaucracy.

 Proposed the POLITICAL ECONOMY approach.


 Despite its flaws, the public bureaucracy performs many important functions
and is far less oppressive to its members and clients than conventional wisdom
contends.

108 | P a g e
 Goodsell & others created the “Blacksburg Manifesto”, which seeks to develop a
more positive role for the bureaucracy in a pluralist system of governance.

Gary WAMSLEY (from Ch. 4 in Wamsley, et. al. Refounding Public Administration).

 Wamsley feels that the challenges facing the American political economy in the
future will require a more effective Public Administration than we have today,
but that this cannot take place unless a “normative guide” is created that
clearly answers questions about administrative power, and that recognizes
a level of authority and discretion essential for empowering the work of
Public Adminstrators.
 The challenges ahead require not management, but governance. Yet
governance requires a more dedicated public administration at all levels. This
cannot be achieved until respect for public institutions and the self-image of
public administrators is restored.

Orion WHITE - (from Ch.6 in Wamsley, et. al. Refounding Public Administration).

 Reframing the Authority/Participation Debate

 Regarding politics, White notes that “the prevalence and predominance of


politics in a society is a sign of ill health at the level of the individual psyche.
Political movements are symptoms of insufficiency in the pace and quality of
transformation at the individual level. Just as constitution writing is birth, so
then is politics youth, and administration maturity, in the developmental cycle
of societies.”

 White concludes with a warning of the dangers of “technicism”. “Technicism


reduces the humanly possible to the technologically available.” When life is
reduced this way, there exists a possibility of losing the human principle itself.

Camilla STIVERS - (from Ch.7 in Wamsley, et. al. Refounding Public Administration).

 Theorists of public administration must rethink the nature of theory itself, by


rejoining truth and goodness. Stivers argues that: “active citizens are a
necessary ingredient in the normative justification sought by the Blacksburg
Manifesto.” Stivers quotes Terry Cooper as saying: “Public servants are citizen-

109 | P a g e
administrators…employed as one of us to work for us and bearing the
responsibility for encouraging participation by other citizens.”

 There is a consensus among Public Administrators that the administrative state


cannot be legitimated by apathetic, ignorant, or misled citizens. Although there
may be many knowledgeable, concerned citizens, as a group they appear ill-
equipped to validate, let alone take part in public policy formation and
implementation.

 Stivers proposes a Theory of Active Citizenship in which the dialogue is


expanded between Public Administrators and citizens, noting that the agency
perspective is consistent with this aim. The missing ingredient in her mind, is
“active accountability to citizens.” To solve this problem, bureaucrats must find
opportunities to work with citizens, explain processes to them, and include
them as much as possible in the policy process.

STILLMAN (1999) Preface to Public Administration.

 “The dichotomy, which became an important instrument for the Progressive


reforms, allowed room for a new criterion for public action, based on the
insertion of professionalization, expertise and merit values into the active
directive of governmental affairs. In practice too the dichotomy served to justify
the institutional developments of such basic features of the administrative state
as the civil service system, personnel classification and planning systems as
well as the introduction of public budgets.” (p112)

STILLMAN (1999) CONT.

 No-State (Jefferson)– Adam Smith, Milton Friedman (monetarists) AND James


Buchannan, Gordon Tullock, William Niskanen, Vincent Ostrom (public choice)
o No-staters turn the PA dichotomy on its head and advocate for a clear
division between the political appointees and the careerists, with a larger
role for the political appointees
o No-staters bifurcate what is good for society (Unseen Hand of the Market)
from what is good within the government (Authoritarian, rigid hierarchy
and strict controls)

110 | P a g e
 Bold State (Hamilton)– Leonard White, Louis Browlow & Gulick are the
founders of this tradition
o Tradition is carried on by Robert Denhardt, Charles Goodsell, George
Frederickson.
o No single united front in support of one idea, or even a group of ideas
o Recognize the difference between public and private sector
administration
o Speak the language of the practitioner
o Have trouble justifying their vision, and seem to stand a bit one-sided
with bureaucracies
o Never seem willing to describe the circumstances under which the
growth of the state will stop
 Pre-State (Madison) – John Rohr, Donald Kettl.
o Recognize the uniqueness of public administration from private sector
administration
o Delve into the historical and organizational contexts when examining PA
o Philosophy is realistic and humane for the practitioner
o Works to deal with constitutional legitimacy problems in the field
o Work around the edges of change, evolution not revolution
o Criticized for descriptive only, lacking strong methodologies
 Pro-State – James Perry.
o Creatures of post-WWII American state system, globalism,
professionalism and technocracy
o Carries the “toolbag” analogy a bit too far, making this view of PA the
grand-son of Frederick Taylor’s Scientific Management
o The technocracy can solve problems with a variety of technologies,
methods and a wide-ranging set of roles and responsibilities
o This view promotes a global vision, interdisciplinary approach and shies
away from normative solutions
o Critics point to the positivist nature of the writings in this view, takes a
dim view of the non-rational problem solving that must take place,
promotes antidemocratic elitism, and places too much faith in the

111 | P a g e
technological instruments – with little regard to how those instruments
might be biased

SVARA – ETHICS IN PA.

 Basic components of administrative ethics: Ethics, Morality and Law. These


can be at odds with each other at times.
 Ethics for PA begins with DUTY. The duty-based approach to ethics derives the
responsibilities of public administrators from the nature of the office they
occupy.
 The Responsibilities of democratic public administrators:
o Place public over personal interest.
o Display a service orientation and commitment to serve.
o Exercise fiduciary responsibility
o Uphold the Law
o Support the democratic process
o Be responsive to policy goals and exercise leadership appropriate to
position.
o Public administrators have an obligation to serve superiors AND the
public.
o SVARA believes that politics and administration cannot be separated as
they overlap and intersect.
o The IDEAL for public administrators is to PROMOTE THE PUBLIC
INTEREST.
 Need to balance:
o Accountability and Independence
o Responsiveness and Neutrality
o Deference and Assertiveness in relationship to superiors

GRODZINS (1966) The American System.

 Argued to replace the antiquated traditional concept of Intergovernmental


Relations (IGR) as a “LAYER CAKE” (with three separate and autonomous
levels of government) Layer-Cake Federalism.

112 | P a g e
 Grodzins described federalism as a “MARBLE CAKE” for the cooperative
relationships among levels of govt. without boundaries, and an intermingling of
activities between and among federal, state and local levels.

FREIDRICH - FINER Debate: RESPONSIBILITY VS. ACCOUNTABILITY

Article Title: Administrative Responsibility in a Democratic Government

Author: Herman Finer

First Appeared In: Public Administration Review, Volume 19, 1941

Brief Synopsis of the Article:

This article by Herbert Finer directly responds to Freidrich’s ideas about public
administration ethics which Freidrich laid out in his 1940 book entitled "Public Policy
and the Nature of Administrative Responsibility."

In this book, FREIDRICH proposed that there should be little explicit control over
a public administrator. Instead, Freidrich believed that the administrator owed
responsibility to two dominant factors in fulfilling his/her obligation and duty to
the public. Specifically, an administrator owed a duty to 1) technological knowledge
and 2) popular sentiment. He thus believed that any policy which violates this
standard makes the official irresponsible to the public. In summary, Freidrich
believed that public administrators must exercise a moral, not a rule-bound,
responsibility to the public. Freidrich also believed that the quality of
administration depends upon an official’s sense of responsibility to the
profession (and the sense of duty to the public).

FINER believes that administrative responsibility and ethics are just as


important to democratic government as administrative effectiveness. In order to
make administrators responsible and ethical, it was necessary to have a
correction system with known punishments and sanctions for administrators
who violate the "rules." He cautioned against public administrators from using their
own discretion, instead believing that public administrators are responsible to the
elected representatives for the public. Finer advocated the use of the courts and as
well as clear lines of authority in administration (sanctions). Finer defined the public
administrator’s responsibility to the public as three–fold. First, he believed that public

113 | P a g e
administrators should work for what the public needs, not what the public wants.
Second, that public administration can only function through institutions. And third,
public administrators must be obedient to the orders of their superiors. Finer felt that
the a public administrator should have little or no flexibility or discretion and that in
order to be truly accountable to the public, administrators must be provided with
more than a sense of moral responsibility. Finer believes that moral responsibility is
likely to operate in direct proportion to the strictness and efficiency of political
responsibility and to fall into disarray if this political responsibility is not strictly
enforced via sanctions. He was firm in his belief that professional standing and a duty
to the public were not enough; sanctions were necessary to keep public administrators
ethical.

What All the Fuss Was Originally:

This debate between Finer and Freidrich over how to make public administrators
ethical actors and responsible to the public became extremely important for
administrators’ practices of how to enforce ethical behavior, whether through
sanctions or through moral obligation and guilt.

Why It Is Still a Significant Article Today:

The exchange between Finer and Freidrich is the most cited article/exchange
discussing the "best" strategy for achieving accountability in public administration.

NIETZSCHE – Was alarmed and distressed by the decline of individuality and free
expression in the machine age.

DENHARDT – 1984 –

 Denhardt’s definition of PA: “PA is concerned with managing change processes


in pursuit of publicly defined societal values.

RAADSCHELDERS – From RAAD I.

 VALUES are the very fabric of society and possibly the most basic reason for
the existence of government. Sociologists define values as part of CULTURE:
the entirety of values, norms, goals and expectations in a given society.

 The values refer to shared opinions about good and bad.

114 | P a g e
 From these values, NORMS are derived: more specific and concrete guidelines
for behavior, the written and unwritten rules that guide social interaction and
communication and that determine the ways in which we associate and act.

 LAW is a set of formalized norms.

WEBER – “Authority is the legitimate use of power”.

 Weber argued that there are 3 Pure Types of Authority:

 Charismatic – rests in the charisma of an individual ruler.

 Traditional – Based on the belief that its legitimacy is derived from a


predetermined order and stratification in society. Leader is vested with
authority by tradition of birth, family, wealth, etc.

 Legal-Rational – This type is found in modern societies. Its legitimacy rests on


the impersonal and standardized application of established rules in the most
democratic and efficient way possible.

 THE IDEAL ORGANIZATION FOR LEGAL-RATIONAL AUTHORITY TO


THRIVE IS THE BUREAUCRACY.

 Central to the notion of legal-rational authority is that people themselves can


make the laws that rule society.

Don KETTL – The Transformation of Governance. KETTL’S GOVERNANCE THEORY:

 HAMILTONIAN - Strong Executive, Top-Down hierarchical authority.


 JEFFERSONIAN - Weak Executive, Bottom-Up Devolved Power,
Responsible to Citizens. Jefferson mistrusting of executive power. He was for
LOCAL power.
 WILSONIAN - Bureaucracy-Centered. Need for good administration. Role
of Permanent Bureaucracy.
 MADISONIAN – Balance-of-Power Centered. Modern Pluralist way. Middle-
Way. Varieties of Orgs play a part.

 Kettl feels that administration needs to be strong.

 Executive needs to have a lot of latitude and power to lead effectively.

KETTL on FUTURE MANAGEMENT CHALLENGES IN PA:

115 | P a g e
 Kettl: “Americans have always been distrustful of governmental power,
and especially administrative power. They have long believed that public
administration is more inefficient and corrupt than private
administration.

MODELS OF PUBLIC DECISION MAKING (RAAD I p. 255-259)

 RATIONAL-COMPREHENSIVE MODEL – aka ECONOMIC MAN model.


Assumes the decision maker has access to all needed information, ability,
intellect, time, etc. and is able to select the ONE BEST solution. It presupposes
that individuals are out to maximize their self-interest. Values are separated
from facts.
 SIMON - ADMINISTRATIVE MAN MODEL – SIMON criticized the Economic
Man model. The reality of decision making is quite different because
information is limited and human processing capabilities are limited. Simon’s
decision maker is an ADMINISTRATIVE MAN whose analysis is based on
BOUNDED RATIONALITY and whose actions are aimed at SATISFICING rather
than maximizing. In Simon’s view, objectives and values cannot be separated.
 LINDBLOM - INCREMENTAL MODEL – LINDBLOM. “MUDDLING THROUGH”.
Decision making is a non-comprehensive process of successive and limited
comparisons. The rational-comprehensive model is fallible, too slow and costly
and can never resolve conflicts of values and interests. Lindblom’s model
provided the foundation for a more realistic analysis of BUDGETING that his
student WILDAVSKY provided in The Politics of the Budgetary Process. Values
rather than facts determine budgetary decisions (view shared by Guy PETERS).
 DROR - NORMATIVE-OPTIMUM MODEL – DROR – Dror argues that
Lindblom’s model is more realistic than Simon’s model, but is still
unsatisfactory. Dror’s model enhances rationalization, uses extra-rational
input via sensitivity sessions and brainstorming. Policy Gambling: Dror argues
that every policy is a RISK and may not work, but it is important to TRY and
Experiment.
 ETZIONI - MIXED-SCANNING MODEL – Amtai ETZIONI - Distinguishes
between contextualizing decisions (which outline the basic directions of policy

116 | P a g e
substance), and BIT-BY-BIT DECISIONS or incremental steps which prepare for
or follow a contextualizing decision.
 WILDAVSKY - IRRATIONAL POLICY ANALYSIS – WILDAVSKY – Argues that
decision making is not very rational. It is very difficult to determine empirically
the degree to which decision making processes are determined by RULE OF
THUMB, perception (definition) of the situation, bias, emotion, the need for
quick results, groupthink, stress, org and cultural history, the informal role and
position of decision makers, and the physical messages (face, expressions of
doubt, non-verbal cues, etc.) of decision makers.
 COHEN, MARCH & OLSEN - GARBAGE CAN MODEL – (Universities &
hospitals) At the individual level, one has to work with conflicting groups of
people. Organized anarchy. In this type of decision making, policies and goals
are often vague and, when clear are often conflicting. Members of the org have
little knowledge of the overall work of the organization, and decision making is
erratic. This decision making model is good for professional orgs where
individuals are highly educated, opinionated, and independent.

James MARCH - 1966 – “The Power of Power”.

 Explores approaches for empirically studying social power in orgs and


communities.

CYERT & MARCH - 1963 – BEHAVIORAL THEORY OF THE FIRM.

 Focus on POWER and POLITICS of establishing organizational goals. Discussed


the formation of coalitions and negotiations to impose coalitions’ demands on
the org.
 Corporations tend to “SATISFICE” rather than engage in economically rational
profit-maximizing behavior.

New Public Management (NPM) (a management reform movement worldwide in


1980s-1990s) – focused on productivity, marketization, service orientation,
decentralization, accountability. NPM is more ideological (Republicans &
conservatives like it). Looks to MINIMIZE government role in society with citizens
increasingly involved in policymaking.

117 | P a g e
GOVERNANCE THEORY – talks about the RELATIONSHIP between Govt. & Society.
Governance is about PROCESS, NPM is about OUTCOMES. Governance is a theory
about Politics. Tries to explain what government does and how to make it better.

BOTH NPM and GOVERNANCE THEORY believe that government is too distant from
the citizen and clientele. Capitalism is helpful in correcting deficiencies. Both are
results oriented. Both embrace the concept that government should STEER NOT
ROW.

118 | P a g e
1.13 Iron cage – Weber
Iron cage is a sociological concept introduced by Max Weber. Iron cage refers to the
increasing rationalization of human life, which traps individuals in an "iron cage" of
rule-based, rational control. He also called such over-bureaucratized social order "the
polar night of icy darkness".

The original German term is stahlhartes Gehäuse; this was translated into 'iron cage',
an expression made familiar to English language speakers by Talcott Parsons in his
1958 translation of Weber's The Protestant Ethic and the Spirit of Capitalism. Recently
some sociologists have questioned this translation, arguing that the correct term
should be 'shell as hard as steel' and that the difference from the original translation
is significant. A more literal translation from German would be "steel-hard housing."

Weber wrote: "In Baxter’s view the care for external goods should only lie on the
shoulders of the 'saint like a light cloak, which can be thrown aside at any moment.'
But fate decreed that the cloak should become an iron cage."

Weber became concerned with social actions and the subjective meaning that humans
attach to their actions and interaction within specific social contexts. He also believed
in idealism, which is the belief that we only know things because of the meanings that
we apply to them. This led to his interest in power and authority in terms of
bureaucracy and rationalization.

Rationalization and bureaucracy

Weber states, “the course of development involves… the bringing in of calculation into
the traditional brotherhood, displacing the old religious relationship.” Modern society
was becoming characterized by its shift in the motivation of individual behaviors.
Social actions were becoming based on efficiency instead of the old types of social
actions, which were based on lineage or kinship. Behavior had become dominated by
goal-oriented rationality and less by tradition and values. According to Weber, the shift
from the old form of mobility in terms of kinship to a strict set of rules was a direct
result of growth in bureaucracy and capitalism.

Effects of bureaucracies: Positive contributions

Bureaucracies were distinct from the former feudal system where people were
promoted through favoritism and bribes because now there was a set of rules that are

119 | P a g e
clearly defined; there was promotion through seniority and disciplinary control. Weber
believes that this influenced modern society and how we operate today, especially
politically.

Weber’s characteristics of an ideal bureaucracy:

1. Hierarchy of authority
2. Impersonality
3. Written rules of conduct
4. Promotion based on achievement
5. Specialized division of labor
6. Efficiency

Weber believed that bureaucracies are goal-oriented organizations that are based on
rational principles that are used to efficiently reach their goals. However, there are
constraints within this bureaucratic system.

Negative effects of bureaucracies

Bureaucracies concentrate large amounts of power in a small number of people and


are generally unregulated. Weber believed that those who control these organizations
control the quality of our lives as well. Bureaucracies tend to generate oligarchy; which
is where a few officials are the political and economic power. Because bureaucracy is a
form of organization superior to all others, further bureaucratization and
rationalization may be an inescapable fate.

Iron cage of bureaucracy

Because of these aforementioned reasons, there will be an evolution of an iron cage,


which will be a technically ordered, rigid, dehumanized society. The iron cage is the
one set of rules and laws that we are all subjected and must adhere to. Bureaucracy
puts us in an iron cage, which limits individual human freedom and potential instead
of a “technological utopia” that should set us free. It's the way of the institution, where
we do not have a choice anymore. Once came about, it was like a machine that you
were being pulled into without an alternative option; currently, whether we agree or
disagree, if you want to survive you need to have a job and you need to make money.

Laws of Bureaucracies:

1. The official is subject to authority only with respect to their official obligation
120 | P a g e
2. Organized in a clearly defined hierarchy of offices
3. Each office has a clearly defined sphere of competence
4. The official has a free contractual relationship; free selection
5. Officials are selected through technical qualification
6. The official is paid by fixed salaries
7. The office is the primary occupation of the official
8. Promotion is based on a achievement which is granted by a the judgment of
superiors
9. The official is subject to strict and systematic discipline within the office
10.Costs of bureaucracies

“Rational calculation . . . reduces every worker to a cog in this bureaucratic machine


and, seeing himself in this light, he will merely ask how to transform himself… to a
bigger cog… The passion for bureaucratization at this meeting drives us to despair.”

Loss of individuality; labor is now being sold to someone who is in control, instead of
individuals being artisans and craftsmen and benefiting from their own labor.

Loss of autonomy; others are dictating what an individual’s services are worth.

Individuals develop an obsession with moving on to bigger and better positions, but
someone else will always be determining the value of our achievements.

Lack of individual freedom; individuals can no longer engage in a society unless they
belong to a large scale organization where they are given specific tasks in return for
giving up their personal desires to conform to the bureaucracy’s goals and are now
following.

Specialization; with specialization, society becomes more interdependent and has a


less common purpose. There is a loss in the sense of community because the purpose
of bureaucracies is to get the job done efficiently.

Bureaucratic hierarchies can control resources in pursuit of their own personal


interests, which impacts society’s lives greatly and society has no control over this. It
also affects society’s political order and governments because bureaucracies were built
to regulate these organizations, but corruption remains an issue. The goal of the
bureaucracy has a single-minded pursuit that can ruin; what might be good for the
organization might not be good for the society as a whole, which can later harm the

121 | P a g e
bureaucracy’s future. Formal rationalization in bureaucracy has its problems as well.
There are issues of control, depersonalization and increasing domination. Once the
bureaucracy is created, the control is indestructible. There is only one set of rules and
procedures, which reduces everyone to the same level. Depersonalization occurs
because individual situations are not accounted for. Most importantly, the
bureaucracies will become more dominating over time unless they are stopped. In an
advanced industrial-bureaucratic society, everything becomes part of the expanding
machine, even people.

While bureaucracies are supposed to be based on rationalization, they act in the exact
opposite manner. Political bureaucracies are established so that they protect our civil
liberties, but they violate them with their imposing rules. Development and
agricultural bureaucracies are set so that they help farmers, but put them out of
business due to market competition that the bureaucracies contribute to. Service
bureaucracies like health care are set to help the sick and elderly, but then they deny
care based on specific criteria.

Debates regarding bureaucracies:

Weber argues that bureaucracies have dominated modern society’s social structure;
but we need these bureaucracies to help regulate our complex society. Bureaucracies
may have desirable intentions to some, but they tend to undermine human freedom
and democracy in the long run.

“Rationalization destroyed the authority of magical powers, but it also brought into
being the machine-like regulation of bureaucracy, which ultimately challenges all
systems of belief.”

It is important note that according to Weber, society sets up these bureaucratic


systems, and it is up to society to change them. Weber argues that it is very difficult to
change or break these bureaucracies, but if they are indeed socially constructed, then
society should be able to intervene and shift the system.

122 | P a g e
1.14 Paradigmatic Progress in Public Administration: Knowledge
Development
The Study of Public Administration in Times of Global Interpenetration: A
Historical Rationale for a Theoretical Model.

Andres Perez Baltodano (1997).

Journal of Public Administration Research and Theory, 7(4), 615-638.

The stated problem of Baltodano’s article is that scholars of public administration


have lacked a theory of the state and an explanation of the role of state bureaucracy in
the development of modern political societies.

The purpose of the article is to develop a historical interpretation of the evolution and
social significance of the administrative state and also to develop a foundation for a
theoretical model for the study of public administration in times of global
“interpenetration”.

Baltodano argues that both theorists and practitioners of public administration need
to reinterpret the administrative state in these times of global interpenetration. He
believes the study of public administration lacks a theory of the state and an
explanation of the role of state bureaucracy in the development of modern political
societies. He notes that profound changes are underway in the conventional
understanding of the general interest, social responsibility, and of the relationship
between the market and the state. The author reviews the historical evolution of the
discipline of public administration as well as the social construction of the state from
the middle ages and Monarchical absolutism to the current age of globalization with
economic, social and political interpenetration. Baltodano suggests that the discipline
needs to develop its own theoretical capacity that elucidates the relationship between
sociohistorical change and public administration. Baltodano addresses the problem
by proposing a “critical” model which incorporates the following basic propositions:

Public Administration in Times of Global Interpenetration

 The institutionalization of social relations and historical identities within the


boundaries of the modern state involves the exercise of a relational form of
power; it requires the use of an administrative state or public administration

123 | P a g e
system as a vehicle either to reproduce or change established patterns of social
order.
 The exercise of a relational form of power through the administrative state is a
purposeful event that involves the need to adjust, replace, or maintain the
structural properties of the reproduction circuits of public administration to
ensure that the formulation and implementation of public policies will not
distort a government's attempt to reproduce or change existing patterns of
social relations. The following four reproduction circuits should be considered:
the organizational design of the state apparatus; its social domain; its
organizational character; and its management.
 The process whereby governments attempt to make the functioning of the
reproduction circuits of the state compatible with their political objectives is
called the structuration (see Giddens 1990, 64) of public administration. The
mastering of this process is the main objective of the public administration
discipline.
 The structuration of public administration is not entirely determined by a
government's will. Governments have to confront the various systemic forces
that condition the nature and the role of public administration. These forces
both constrain and enable the agents in control of the administrative state in
their efforts to institutionalize patterns of social relations across historical time
and territorial space.
 The relationship between social systems within a societal totality does not
follow any rigidly determined pattern. From this perspective, the functioning of
the reproduction circuits of the administrative state, and consequently a
government's capacity to structure them, is conditioned by the interplay of the
government system, the political system, the economic system, and the internal
dynamic of the administrative state.
 The functioning of the government system, the political system, the economic
system, and the reproduction circuits of the administrative state also is
conditioned by international and transnational forces; thus, the study of the
process of structuration of public administration must take into account the
process of global interpenetration within which public administration operates
today.

124 | P a g e
In conclusion, we can agree with Baltodano that globalization represents a challenge
to the capacity of the administrative state to formulate and implement policies that
connect society’s collective experience with its expectations. Instead of choosing to
operate as a reproducer of established patterns of social order as in the past, the state
can choose to change established patterns of social order. However, to witness real
paradigmatic change there must be political support throughout the system.

Baltodano makes a valid point by emphasizing the importance of institutionalization


(see DiMaggio, 1988) and the process of structuration (see Giddens, 1984, 1990, 1991)
in the modern state. We can also agree with the author that any new overarching
theory of public administration needs to incorporate how structuration and
institutionalization are affected by the forces of globalization.

A Science of Public Administration?

Gregory A. Daneke (1990).

Public Administration Review, 50(3), 383-392.

Gregory Daneke’s article is an answer to Harland Cleveland and others scholars who
feel that “it is time to put behind us the idea that the politics and administration of
human endeavors are some kind of science.” Daneke feels that much of the scholarly
work that has been done in public administration in the recent past has not
contributed to advancing a new paradigm for public administration, but instead have
centered on critiques of positivism and/or neoclassical economic theory. Daneke
argues that scholars in the discipline should channel their energies into reforming the
system’s basic unifying paradigm and to advance it in the direction of a
“comprehensive design approach”, capable of enhancing the processes of adaptive
learning and institutional evolution.

Daneke then presents a new prospective paradigm called “Advanced Systems Theory”.
This emerging paradigm builds on general systems theory and contemporary
economics, while integrating a number of unique ingredients derived from advances in
the physical, biological, and cognitive sciences. By incorporating new knowledge from
works in diverse areas such as chaos theory and quantum logic, the social sciences

125 | P a g e
(and public administration in particular) may be able to overcome their current lack of
progress in paradigm development. Daneke gives additional suggestions of areas
scholars can look to for inspiration in developing a new paradigm for public
administration, including recent innovations in fractal geometry, theoretical
mathematics, chaos theory, evolutionary systems theory, and general systems theory.

Daneke concludes by suggesting that the best road to a “Grand Theory” should
include the work of systems theory. He concedes that systems theory has many flaws,
but that these can be addressed by developing an “Advanced Systems” paradigm.
Advanced systems would allow applied policy and administrative studies to do all that
they are currently doing, maintaining and conceptually enhancing many applied
economics and systems analytics. Yet, advanced systems would also extend concepts
and techniques to embrace insights arising from paradigmatic revolutions in other
sciences. Some of these include: resiliency, co-evolution from the new biology and
ecology; observer/observation-interaction mutual causality, potential,
complementarity and others from quantum logic and physics; dissipative structures,
order through fluctuation, and chaos theory from physical chemistry and theoretical
mathematics; theories of creativity, cognition, and intuition from computer science.

I feel that Daneke’s article is pointing to a meta-inclusive paradigm that may not only
be applicable to explaining activity and behavior in public administration, but also to
society and social networks as a whole. I feel that he is leaning toward development of
a more “science” orientation for the discipline, but this may be difficult considering the
social forces also at work.

Toward A Critical Theory of Public Organization.

Robert B. Denhardt (1981).

Public Administration Review, 41(6), 628-635.

Denhardt begins by stating the problem: “We are in a situation in which we seem to be
somewhat unsure of the historical and philosophical grounding of our work.”
Denhardt recognizes that we can trace the roots of our heritage in many directions,
but as we do, we discover that there are “serious conflicts among our ancestors.”
(Denhardt, 1981)

126 | P a g e
In short, the problem he identifies is that our concern for making public
administration an object of “scientific” study has often been at odds with our interest
in extending the normative principles we associate with democratic governance. The
purpose of the article is to examine recent works in “critical” theory, while addressing
implications for developing a comprehensive theory of public organization. Following a
brief review of the development of critical theory, Denhardt focuses on the work of
Jurgen Habermas, who is perhaps the best known contemporary scholar of the critical
approach.

Critical thinking is “motivated today by the effort really to transcend the tension and
to abolish the opposition between the individual’s purposefulness, spontaneity, and
rationality, and those work-process relationships on which society is built.” (Denhardt,
1981). The critical perspective proffered by Habermas includes the following aspects:

It examines:

 The critique of instrumental reason


 The scientization of political life and the reduction of the public sphere, and
 The relationship between knowledge and human interests.

Habermas, a member of the “Frankfurt School” of thought, describes the “public


sphere” as that arena in which the various interests in society engage in discourse
related to the establishment of the normative agenda for society. “In recent times, the
public sphere has been considerably narrowed, to the point that the interests being
voiced tend to be those of hierarchical superiors in business, labor, and the
professions, mediated or administered by the mass media.” (Denhardt, 1981).
Habermas seeks a “critical” approach that would aim at revealing the “false
consciousness” which binds us, thus permitting movement toward emancipation.
“Since what appear to us as causal relationships are manifestations of our particular
historical circumstance (rather than natural and unalterable “laws”), they are subject
to alteration by autonomous and responsible action on our part, and that process is
initiatied in the act of critique.” (Denhardt, 1981).

Denhardt speculates further about the many facets that could be included in a new
“critical approach”. He also suggests that the discipline’s current “crisis of legitimacy”
may be corrected if a new critical theory of public organizations is developed and

127 | P a g e
institutionalized. Organizational members and clients might be aided in
understanding the nature of their relationships with one another through
internalization of a critical theory of public organizations.

Denhardt concludes by noting that the critical approach is suggestive of alternative


conceptions of the role of public organizations in a democratic society, as well as
changing patterns of relationships within the bureaucracy. “It reminds us that the
theory and practice of public administration is integral to the development of the state
and its allocation of values in society and therefore must encompass far more than
technical concerns”.

We can agree wholeheartedly with Denhardt’s assessment, and feel that an


enhancement of the critical approach put forth by Habermas would be of great value
for those involved in theory development for public administration.

What Makes Public Administration A Science? Or, Are Its ‘Big Questions’ Really
Big?

Francis X. Neumann, Jr. (1996).

Public Administration Review, 56(5),409-415.

The problem addressed by Neumann’s article is “What are the appropriate basic
research questions public administration must address if it is to aspire to the status of
a science?”

Neumann criticizes a recent essay by Robert Behn (1995), entitled “The Big Questions
of Public Management”. He agrees with Behn’s assertion that “any field of science is
defined by the big questions it asks”. However, he feels that Behn incorrectly
identifies the “big questions”.

Behn proposed the following “big” questions:

 How can public managers break the micromanagement cycle?


 How can public managers motivate people?
 How can public managers measure achievement?

128 | P a g e
Neumann says that these questions are not incorrect or irrelevant, but they are at the
wrong level. Neumann argues that Behn’s questions are not “big” questions at all, but
are for an argument at a much lower level. His three questions are questions of
application, not probes into the origins or basic nature of a discipline.

Neumann proposes his own “big questions” for Public Administration:

 What is the nature of an organization? Of a “public” organization?


 How is the public organization related to its environment?
 What does it mean to manage or to administer the public organization?

Neumann acknowledges that the questions appear to address basic organizational


theory, and that much work has already been done in these areas to answer similar
questions. However, he feels that among the scientific disciplines, “The big questions
are never really completely answered. The big questions, by their very nature, are
multifaceted and extend into dimensions of which we are never fully cognizant at any
one time.” Neumann cautions us that although it may seem that all the major
questions have been put to rest, “in truth we have only produced the answers that our
existing vision has enabled us to find.” What is needed is a paradigm shift in public
administration.

Neumann concludes by suggesting a new paradigm based on non-linear systems


theory. He feels that this world view appears to have great implications not only for
the physical world, but for the social world as well. He adds: “the new paradigm is
that of the nonlinear system. Nonlinear systems theory, “complexity” or “chaos”
theory, is providing researchers with a new view of both physical and social systems.”
Some of the characteristics of these systems might be considered to be of important
for application to public administration and public management. Neumann gives
guidelines for the development of a new paradigm based on reasons why the older
understanding of organizations may now be inadequate:

 Complex problems require complex mechanisms of solution.


 Attention to the parts of the problem may not solve the whole of the problem.
 Nonlinear systems do not necessarily tend toward equilibrium.
 Mechanisms of positive feedback are widespread and may cause unforeseen
deviation amplification.

129 | P a g e
 Complexity may develop spontaneously in a system.
 Natural complex systems contain a balance of both random and deterministic
elements.
 Accurate forecasts for future states of the nonlinear system may not be
attainable.

Neumann notes that several authors (Jantsch, Ashby and Kiel and others) have
supported the idea that the tenets of chaos theory can be married with the more
traditional perspectives of political science and public administration. Under the new
paradigm, “the organization is again terra incognita, and the implications for the
discipline are truly profound. All the closed doors and all the apparently settled
questions must now be reopened. The big questions have not been satisfactorily
answered at all.” (Neumann, 1996).

In summary, Neumann is arguing that incorporating non-linear and chaotic systems


theory means that we need to start with consensus on the core definitions of the
organization, its internal dynamics, and its relationships with its environment. This
means there is a great need now for a new exploratory wave of research in this area.

We can agree with Neumann that important answers needed for further theory
development in public administration can be found in systems theory and other
theories from diverse fields. I also agree with him that a new wave of research is
needed, one that has the goal of developing a united theory for our discipline, even if it
“borrows” from the important work being done in other fields.

From Responsiveness to Collaboration: Governance, Citizens and the Next


Generation of Public Administration.

Eran Vigoda (2002).

Public Administration Review, 62(5), 527-540.

Vigoda’s article centers on the problem of illuminating the differences between


“responsiveness” and “collaboration” with regard to modern public administration. He
argues that there is an inherent “tension” between better responsiveness to citizens as
clients and effective collaboration with them as partners. “The differences between
responsiveness and collaboration/partnership are not merely conceptual or

130 | P a g e
terminological. In fact, they represent an intensifying paradox that emerges in both
the theory and the practice of contemporary public-sector management”. (Vigoda,
2002).

Vigoda argues that most of the current theoretical thinking in public administration
deals with responsiveness and collaboration separately, but never integrating them in
a useful manner. Vigoda illustrates by reviewing works from both camps. One group
highlights administrative responsiveness to citizens’ requests as the most important
value of public agencies in a businesslike arena. The other group emphasizes
partnership between citizens and government as a premise for cultural revolution in
contemporary bureaucracies.

The purpose of the article is to show that current focus on New Public Managerialism
expands the orientation of government and public administration toward
responsiveness. This is treated as a negative factor, because NPM implementation is
frequently accompanied by several pathologies such as: lower willingness to share,
participate, collaborate, and partner with citizens.

Vigoda feels that NPM and most of the other administrative reforms in recent years
focus on a relationship with citizens as customers, with all focus on satisfying the
customer in one central way - by improving responsiveness. However, this view
disguises the truth that the citizens are not just customers, they are the owners, and
should be treated as such. Vigoda argues that “neo-managerialism and New Public
Management encourage passivity among the citizenry by limiting opportunities for
inclusion in decision-making. Vigoda feels that collaboration is an indispensible part
of democracy, and that citizens are being relegated to a lower experience without their
knowledge. He recommends that future theory building work should concentrate on
the view of citizen collaboration with government, while giving up on the misguided
notions of “customer” and “responsiveness”.

The central argument is in how government and public administration view and treat
the citizenry. The choice at hand is between a view of citizens being treated as clients
and customers, or being perceived as equal partners in the process of governance.

Vigoda suggests that the “old” public administration treated citizens as subjects. With
the installation of the voter electoral system, another style of citizen-government

131 | P a g e
relationship emerged – citizens as voters. Next is the model of citizens as
clients/customers, illustrating a relationship dynamic that still exists today. On the
extreme end of the government-citizen spectrum is citizen as owner. However, Vigoda
mentions that this extreme should also be avoided.

Vigoda concludes by calling for future scholarship in public administration that


renews the values of collaboration and partnership. There needs to be movement from
a “they” spirit to a “we” spirit. Both parties (citizens & government), must be actively
engaged in the process of administrative change and reforms.

We can agree with Vigoda that the overriding theme of current policy and reforms
seems to be aimed at this notion of citizen as client or customer – a business mindset.
I also agree that this needs to change, because in reality, the rights and
responsibilities of a citizen entail much more than a “customer” mentality and passive
level of involvement in the work of government.

132 | P a g e
1.15 Outlines
FOUNDATIONS

Tensions btw Bureaucracy/Democracy

Raadschelders (2003, 316) “Democracy is the rule of the people; bureaucracy is the
rule of bureaus.”

Aristotelian/Platonic view of three types of power:

 Monarchy/Tyranny
 Aristocracy/Oligarchy
 Polis/Democracy

Raadschelders sees a fourth type: Bureaucracy/Bureau mania or Bureaucratism


(333) Bureaucracy is marginalizing politicians (and representative democracy) through
specialization and permanence.

Democracy undermines bureaucracy by advancing representativeness over


specialization, expertise or power (Note: This is inferred, not explicitly stated in
Raadschelders)

Raad says that the bureaucracy and the administrators are indirectly accountable to
the public because there is a layer of insulation (as mentioned by Peters.)

Van Braam (1986, 191) wrote on the representativeness of bureaucracy and three
qualities he believes it enhances from a democratic perspective:

 Political legitimacy – if bureaucracy is representative, then society recognizes


itself in bureaucratic action
 Political loyalty – if bureaucracy is representative, then bureaucracy will
respond to political (democratic) values in a society
 Political reward – if bureaucracy is representative, then minority rights to equal
opportunity are recognized and satisfied

Peters (1996 & 2001, 26) Bureaucracies undermine democracy in four fundamental
ways:

133 | P a g e
 Monopoly of information allows bureaucracy to have an advantage over
legislatures (as an expression of representative democracy) and to, in turn,
pursue bureaucratic self-interest versus the public interest
 Hierarchy insulates the policy decision-making core from political leaders and
citizens
 Permanence immobilizes action in a dynamic environment and halts policy
innovation by perpetuating the policy choices and governing styles to which
bureaucrats are accustomed
 Internal regulation prevents the exercise of individual discretion, allows
bureaucrats to avoid taking responsibility for administrative action by blaming
the rules

Svara (2007) suggests that administrators are ultimately responsible to its masters
and political creators.

Frederickson (1997) believes that administrator is accountable to the public directly by


suggesting the responsibility “ennobles the practice” of public administration.

Reconciliation

Frederickson (1997, 202) says that bureaucracy should seek to promote democratic
rather than its own values.

“This, then, is the model for public service – the combination of patriotism (the love of
regime values) with benevolence (the love of others) realized in action.”

Bureaucracy should be the guardian of democratic values and pursue them through
efficient, effective and equitable administration. It is in a unique position to do so
because of bureaucratic permanence and day-to-day administration of government.

Raadschelders (2003, 379-388) – Balance along several continuums is the way to


reconcile tensions between bureaucracy and democracy:

 Balance of individualism and collectivism – discretion versus control and


standardization; direct democracy versus indirect democracy; freedom versus
equality
 Balance of juridical and sociological modes of action – formal/impersonal
application of rules versus informal, discretionary application of rules

134 | P a g e
 Balance of abstract and concrete layers of understanding – long term concerns,
multi-disciplinary issues, normative planning versus a management
perspective, operational concerns, etc.

“We must accept the idea of walking the middle ground.”

If you substitute “administrative self-interest” for “bureaucracy” and “public interest”


for “democracy”, you’ve got the same argument as above…with little difference.

Still, what does it look like when the bureaucracy pursues self-interest? According to
Frederickson (1997, 29), it looks like this:

 Efficiency
 Economy
 Order
 Predictability
 Due Process
 Protecting Boundaries
 Reduction of Big Questions to Questions of Mean and Short-Term Benefits

Svara (2007) prescribes a healthy mix of bureaucratic and democratic responsibilities


for public administrators, thereby “walking the middle ground”. They are:

 Put the public interest over personal interest (democracy)


 Display a service orientation and a commitment to serve (democracy)
 Have a commitment to procedural fairness (bureaucracy)
 Exercise fiduciary responsibility (bureaucracy)
 Be bound by and uphold the law (both)
 Support the democratic process (democracy)
 Be responsive to the policy goals of political superiors while fairly examining all
policy options and exercising leadership appropriate to position (democracy)

There an appropriate level of ethics properly integrated into the organization will
greatly improve the situation.

135 | P a g e
Svara believes that no theory of public administration is complete without an ethics
component.

Svara further suggests that given the right tools, training, guidance, codes and
supervision, administrative self-interest can be minimized and thus reducing the
tension between bureaucracy/administration and democracy and the public.

Search for an Identity

This identity crisis of P.A. study is, according to Richard Stillman’s comments for a
1999 ASPA Miniplenary Session, best summed up in a 1982 paper from Dwight Waldo
entitled “Politics and Administration: A Profound Disjunction” in which he says:
“…our politics are Greek but our administration is Roman.” Essentially, Stillman
argues that Waldo identified our “thinking” or academic side as public-oriented while
our “action” or pragmatic side is administrative-oriented. Stillman argues for us to
embrace this disjunction in order to find the answer or bridge the identified gap.

Waldo (1968)

Two 1968 essays by Dwight Waldo reflect on the state of the field of PA. Waldo rejects
the two traditional solutions to the identity crisis: sub-discipline of political science or
something else OR a distinct discipline among the social sciences

Waldo advocates, instead, for the solution that “we try to act as a profession without
actually being one, and perhaps without the hope or intention of becoming one in any
strict sense.”

The profession argument uses medicine as an example of “science and art, theory and
practice, study and application” all fitting nicely under an umbrella of profession.

“It is not based on a single discipline, but utilizes many. It is not united by a single
theory, and is justified and given direction by a broad social purpose.”

Waldo agreed with Simon on the shortcomings of scientific management and the
“principles” approaches, but was more skeptical than Simon of efforts to rely solely on
logical positivist methods in development of a theory for PA.

136 | P a g e
Dubnick (1999) (a political scientist at an ASPA meeting

“Identity crisis” is but one of several labels used to illustrate the field’s problems.

Ostrom (1974) called it an “intellectual crisis”; Henry (1987) called it a “paradigmatic


quandary”; Stillman (1991) called it a “shifting” among “competing visions”.

Dubnick argued that the blame lay in the writing of the field’s intellectual leaders.

The founding of ASPA in 1930s represents a split between those in political science
that wished to establish that field as a separate social science and those committed to
maintaining the link between research and practice in governmental affairs.

Public administration involved not merely the study of government operations and
management; it inherently included a “broad social purpose” no different from that
characterizing the study of medicine. Any effort to resolve the identity crisis must
encompass that strong commitment to purpose.

Ostrom (1974)

Using Kuhn’s terminology, Ostrom argued that PA faced a “paradigmatic crisis”


because of the proliferation of prevailing theories; the methodological experimentation;
the explicit discontent among scholars; the large amount of philosophical speculation
and the debate surrounding fundamental epistemological issues.

Ostrom’s solution was to develop PA as a science of association

Golembiewski (1977a and b)

Golembiewski has suggested that the discipline of PA ought to be developed by means


of a “family of miniparadigms” such as organizational development

Raadschelders (2003)

Presents the idea that the “identity crisis” is a generic term referring to two types of
crisis: academic crisis and existential crisis

Academic crisis concerns itself with the study of PA and the practice of
PA/government, dealing with the theoretical, methodological, epistemological
weaknesses and controversies

137 | P a g e
Existential crisis concerns itself with study and practice of PA/government, dealing
with authority, legitimacy and credibility crises.

This refers in part to the legitimacy of the administrative state

The following refers to the academic crisis as it relates to Raad:

 Nicholas Henry (1975) “Paradigms of Public Administration” PAR 35(4):


o 5 paradigms
o PA Dichotomy 1900-1926
o Principles of Administration 1927-1937
o PA as Political Science 1950-1970 (PA resubmits to the domination of
PoliSci)
o PA as Administrative Science 1956-1970 (PA submits to the domination
of Administration, public and private)
o PA as PA (?) 1970 - ? (PA comes into its own, following the Simon
prescription in 1947)
 1938-1950 – Challenge to PA Dichotomy and Principles:
o 1938 Barnard’s Functions of the Executive (influenced Simon)
o 1946 Fritz Marx ed. Elements of Public Administration (questioned
dichotomy assumptions)
o 1947 Simon’s Administrative Behavior (blows the principles away)
 1947-1950 – Reaction to the Challenge:
o 1947 Simon’s “A Comment on ‘The Science of PA’” (prescribes two tracks
– scholars working on the science of admin and a larger group working
on prescriptive public policy)

Frederickson Questions

 How important that we have a paradigm?


 Are we making progress toward one?
 Which is the most promising?

Rainey Answers

 Not very
 No

138 | P a g e
 Mine!

Elaborating, Rainey says:

The importance of a paradigm is debatable – yes, we can travel in a herd, with all the
advantages that might offer BUT we are also subject to the disadvantages.

It’s difficult and dangerous to achieve and impose consensus on a field of inquiry.

Distinctions between public/private are snagging progress in the field, esp. with org
theory.

Paradigmatic Discussion:

 Five paradigms:
o Positivist
o Post-positivist (Stanley identifies with this one)
o Interpretivist
o Conflict
o Post-modern
o Every paradigm should must have three things
o An ontology – the nature of reality
o An epistemology – how to know reality
o A methodology – how to test reality

Given that, most professors don’t see Henry’s 1975 work as really being paradigms –
they are more like theories. But that really depends on your definition of paradigm,
doesn’t it?

Henry accounts for this in his article by using paradigm as “How mainstream public
administrationists have perceived their enterprise during the last 80 years”

Stanley is using the Kuhn paradigmatic definition

Rainey points out the Kuhn used “paradigm” in 21 distinct senses in 1962

French, Spears & Stanley (2005) The Fifth Paradigm of Public Administration?...

Once Public Org Theory is redefined as Public Org Efficacy, paradigmatic progress can
begin

139 | P a g e
Efficacy = efficiency, effectiveness, equity, responsiveness and accountability through
various modes of research

The last step is recognizing the quantitative and qualitative nature of inquiry, and
establishing an equal respect for both – because some problems are better solved with
numbers, some with words

Reformist Roots

Historical Overview of the Progressive Era (and around the edges)

1883 – The Pendleton Act creates the U.S. Civil Service Commission

1886 – The American Federation of Labor is formed

1887 – Woodrow Wilson publishes The Study of Administration AND the Interstate
Commerce Commission is established as the first federal regulatory commission

1894 – The National Municipal League was formed to combat local government
corruption

1900 – Frank Goodnow’s Politics and Administration provides the first definition of the
politics-administration dichotomy

1911 – Frederick Taylor publishes The Principles of Scientific Management

1912 – The Commission on Economy and Efficiency, the Taft Commission, calls for a
national executive budget

1918 – William Willoughby outlines developments that were leading to the creation of
modern budget systems

1920 – The Retirement Act creates the first federal service pension system

1920 – The Nineteenth Amendment gives women the right to vote

1921 – The Budget and Accounting Act is passed

1922 – Weber’s structural definition of bureaucracy is published posthumously

140 | P a g e
General Thoughts

A large limitation of this view is that if the Reformist movement started PA, what was
going on in America prior to the turn of the twentieth century?

The limitation of viewing PA as rooted in reform is that is does not take into account
the values discussion to which several writers continue to return

While it may be true that PA was born out of and has continuously sought civil
service/personnel reform; continued with any number of various budgetary reforms;
and has more recently involved privatization and “Reinventing Government” reforms,
to simply view PA as a series of reforms is missing the “values” component of the
equation

This, in my view, is what the scholars are getting at in the Minnowbrook Conference
and the Blacksburg Manifesto

Rosenbloom (2008)

Makes an argument that the PA dichotomy was misinterpreted from Wilson’s (and the
Progressive’s) original meaning

Rosenbloom insists that the original meaning of “politics” was narrow, meaning only
“partisan politics” and not the politics inherent in public policy formulation

Pointing to the civil service reforms right before and during the Progressive Era,
Rosenbloom demonstrates that this was the “politics” that had to be separated from
administration

Waldo, says Rosenbloom, used a broader definition of “politics” (that includes policy)
to destroy the dichotomy as a viable view of PA

Further, Rosenbloom recalls discussing the dichotomy with Waldo at Syracuse and
having Waldo label him “revisionist” but “he conceded that my interpretation was more
than viable and made more sense of the dichotomy than any other of which he was
aware”.

141 | P a g e
Peters (1995)

Civil servants can present themselves as apolitical servants of the state who are able
to serve any government. They can also claim that they are “divorced from partisan
politics” (235).

Raadschelders (2003)

“Kettl points out that evidence of savings does not necessarily mean that contractors
or private suppliers provide the same quality of service nor does it mean that they
have a focus on fairness or equity.” (p239)

Stillman (1999)

“The dichotomy, which became an important instrument for the Progressive reforms,
allowed room for a new criterion for public action, based on the insertion of
professionalization, expertise and merit values into the active directive of governmental
affairs. In practice too the dichotomy served to justify the institutional developments
of such basic features of the administrative state as the civil service system, personnel
classification and planning systems as well as the introduction of public budgets.”
(p112)

No-State (Jefferson)– Adam Smith, Milton Friedman (monetarists) AND James


Buchannan, Gordon Tullock, William Niskanen, Vincent Ostrom (public choice)

No-staters turn the PA dichotomy on its head and advocate for a clear division
between the political appointees and the careerists, with a larger role for the political
appointees

No-staters bifurcate what is good for society (Unseen Hand of the Market) from what is
good within the government (Authoritarian, rigid hierarchy and strict controls)

Bold State (Hamilton)– Leonard White, Louis Browlow & Gulick are the founders of
this tradition

Tradition is carried on by Robert Denhardt, Charles Goodsell, George Frederickson


and Anne-Marie Rizzo

No single united front in support of one idea, or even a group of ideas

142 | P a g e
Recognize the difference between public and private sector administration

Speak the language of the practitioner

Have trouble justifying their vision, and seem to stand a bit one-sided with
bureaucracies

Never seem willing to describe the circumstances under which the growth of the state
will stop

Pre-State (Madison) – John Rohr, Donald Kettl,

Recognize the uniqueness of public administration from private sector administration

Delve into the historical and organizational contexts when examining PA

Philosophy is realistic and humane for the practitioner

Works to deal with constitutional legitimacy problems in the field

Work around the edges of change, evolution not revolution

Criticized for descriptive only, lacking strong methodologies

Pro-State – James Perry

Creatures of post-WWII American state system, globalism, professionalism and


technocracy

Carries the “toolbag” analogy a bit too far, making this view of PA the grand-son of
Frederick Taylor’s Scientific Management

The technocracy can solve problems with a variety of technologies, methods and a
wide-ranging set of roles and responsibilities

This view promotes a global vision, interdisciplinary approach and shies away from
normative solutions

Critics point to the positivist nature of the writings in this view, takes a dim view of
the non-rational problem solving that must take place, promotes antidemocratic
elitism, and places too much faith in the technological instruments – with little regard
to how those instruments might be biased

143 | P a g e
A Feminist Perspective:

Stivers 2002

“The discussion aims to show how women’s work and thought were at the center of
the movement to reform city governments and how gender dynamics at the time
resulted in bifurcation between what could have been complementary impulses of
systematization and caring. The extent to which the contemporary administrative
state has roots in women’s reform work has been obscured because male reformers,
painted by party politicians as effeminate, felt the need to make public administration
masculine by making it ‘muscular’ – that is, scientific and businesslike.”

Professionalism

Waldo, Dwight. (1968)

Rejects the two traditional solutions: sub-discipline in political science or some other
discipline and a distinct discipline within social sciences

“We try to act as a profession without actually being one, and perhaps without the
hope of intention of becoming one in any strict sense.”

Uses the analogy of the field of medicine that is a combination of “science and art,
theory and practice, study and application” to illustrate the concept of a profession

Van Wart, Montgomery (1998). Changing Public Sector Values

5 Major Sources of Values for Public Administrators are (p. 8):

 Individual
 Professional
 Organizational
 Legal
 Public Interest

6 Characteristics of a Profession (p. 12):

 Self awareness
 A body of theory and knowledge
 A social ideal

144 | P a g e
 A formal organization to promote its interests
 A national academy
 Ethical standards

Van Wart’s claim is that PA is a profession only by the flimsiest of standards.

Ernest Greenwood’s 5 Attributes of a Profession (according to Van Wart p. 62)

 A body of theory/knowledge – systematic body of knowledge requiring formal


education and intellectual practice
 A professional culture – expressed in norms, symbols and a world view
 The sanction of the community – the profession controls training and
accrediting
 A regulative code of ethics – professional “disinterestedness” or neutrality
toward the client
 Substantial professional authority – lay people must trust the professionals

Van Wart applies the 5 Attributes to PA:

 A body of theory/knowledge – yes, many positions require considerable training


and knowledge
 Professional culture – yes, professional organizations pop up for all types of
public administrators
 Community sanction – yes and no, often the community/elected officials control
training and standards
 Ethics standards – yes and no, the ethical standards are often set by legislation
or by someone other than those in the profession
 Professional authority – yes, judges and social workers are an example

Conclusions:

Positive values of professionalism in PA

Heightened competence

Outside review

Job satisfaction

145 | P a g e
Innovation

Client satisfaction

Negatives associated with professionalism in PA

Limited access to profession due to educational requirements

Over-specialization

Excessive control and power

Promotion of self-interest above the public interest

Cost-benefit distortion

Wamsley, Gary; Goodsell, Charles; Rohr, John; Stivres, Camilla; White, Orion and
Wolf, James. Refounding Public Administration (a.k.a. The Blacksburg
Manifesto) (1982)

Public administration as a profession is immaterial, what matters is that public


administrators act in a professional manner by doing the following:

View themselves as “trustees”

Pursue the public interest

Adhere to the rule of law and limited government (constitutional)

Prudently accommodate powerful forces

Facilitate real citizen involvement in governance

Be responsive to valid orders

Be conscious of their own values

Be able to give reasons for their actions

Know that social problems don’t have quick, cheap or permanent solutions

Some solutions are market-based; some come from the state

Be an analyst and educator

Be committed to praxis- reflection and action – thoughtful assessment of action taken

146 | P a g e
Live for PA as a cause, not off it for less noble reasons

Goodsell’s article (1990) “Public Administration and the Public Interest”

P. 107 – professional public administrators are the “leading embodiment and


proponent of the public interest”

Public administrators should use the public interest as a verbal symbol by the
expression of 6 values or rules:

Legal-morality

Political responsiveness

Political consensus

Concern for logic

Concern for effect

Agenda awareness

Kirlin (1996) Big Questions of PA in a Democracy (in the discussion of


Professionalism of the field)

 What are are the instruments of collection action that remain responsible both
to democratically elected officials and to core societal values?
 What are the roles of nongovernmental forms of collective action in society and
how can desired roles be protected and nurtured?
 What are the appropriate tradeoffs between governmental structures based on
function and geography?
 How shall tensions between national and local political arenas be resolved?
 What decisions shall be isolated from the normal processes of politics so hat
some other rationale can be applied?
 What balance shall be struck amount neutral competence, representativeness,
and leadership?
 How can processes of societal learning be improved, including knowledge of
choices available, of consequences of alternatives and of how to achieve desired
goals, most importantly, the nurturing and development of a democratic polity?

147 | P a g e
Legitimacy of the Administrative State

Raadschelders (2003)

American government is a societal creation, founded in the U.S. Constitution with


negotiated authority and a system of checks and balances.

Legitamacy of Public Administration:

 “Is public administration a legitimate professional study?”


 Raadschelders (2003), comments that public administration is a product of
multiple disciplines, somewhat of a garbage can. Although public
administration has profited from a multipdisciplinary approach, it has also led
to a reputation of being intellectually underdeveloped. Some scholars suggest
that public administration actually has the potential to arrive at a more
complete understanding of government because it draws on so many disparate
disciplines.
 If we agree that public administration is a field of discipline, then the second
inquiry is a logical one:
 “From where does public administration draw its authority?”
 Raadschelders (2003) suggests that government is facing a legitimacy crisis that
is evidenced by declining voter participation and citizen participation in policy
making. He comments however, that today’s government is society’s creation,
founded in the U.S. Constitution with negotiated authority and a system of
human-made checks and balances. Democracy refers to a state system in
which sovereignty is formally invested in the citizens, with guaranteed civil
rights and freedoms. Democracy supports a society where individual rights are
revoked only when a citizen has in some way harmed another.

Stillman (1999)

Growth of the American state is linked to the rise of PA theory

Stillman defines state as concrete national institutions and organizations and people
that carry out the basic functions common to all modern nations, such as tax
collection, business regulation, national defense, public education, social welfare, etc.

148 | P a g e
Suggests that U.S. was “stateless” until the passage of the Pendleton Act in 1883

American PA differs greatly from European PA in that American PA does not have
authority grounded in thousands of years of “state” history

Box (2004)

Legitimacy centers on the relationship of public administration to the constitution and


the nature of the founding period. A legitimized PA would be one that is respected by
the public, has more control, authority; discretion to act independently and is given
the status of an equal partner in relation to elected leaders and other parts of
government

The framers would likely disapprove of the power of the modern federal government,
although a strong government is needed to protect individual rights

Stivers (2002)

Legitimate power is seen as flowing from the people to their elected representatives
and indirectly to appointed officials

The exercise of power by civil servants is neither elected or removable but problematic

Legitimacy premised on arguments of expertise, neutrality/objectivity and


management

New Public Management used this function to advocate “breaking through


bureaucracy” by making bureaucrats into entrepreneurs and innovators; suggests the
polity needs an entity to carry and implement the vision

Rohr (1986)

Framers left congress to decide which executive departments should be created, and
in so doing expressed their desire that some departments should be created (173).

Must be concerned how management and control which are necessary in modern
industrial society can be performed without violating commitments to individualism
and self-government (178).

149 | P a g e
While we make the case that the administrative state is compatible with constitutional
principles it is useful to remember that the constitutional principles were themselves
the object of intense debate (179).

The role of PA is to fulfill the objective of the oaths of office: to uphold the Constitution
of the United States, meaning that administrators should use discretionary powers in
order to maintain the constitutional balance of powers in support of individual rights

The Senate, in its original intended state, is the constitutional model for PA as a
balance since it was supposed to exercise all three powers of government.

PA exercises those powers, but in a subordinate capacity to the three branches of


government

Rohr (2002)

Recurrent debate over the proper role of public administrators, stemming from their
balance of demands for efficiency and for political definitions of the public good;
efficiency and a business approach currently holds the upper hand

“Nothing is more fundamental to governance than a constitution; and therefore to


stress the constitutional character of administration is to establish the proper role of
administration as governance that includes management but transcends it as well.”

Rohr promotes the ideas of administrative discretion and civil servants as


constitutional actors

1993 PAR Debate (Spicer/Terry Debate) found in PAR 53(3)

Characterized Rohr as one who romanticized the view of the founders; he and others
perpetuated a distorted view of the founders

Idealist method of historical explanation – explain actions or intentions after the face

Empathetic method of historical explanation – imagine themselves in the place of the


person involved in the event

Characterized the founders as noble men who possessed superior minds

John Hope Franklin – “created a tragically flawed revolutionary document and a


constitution that did not bestow the blessings of libery”

150 | P a g e
Herbert J. Storing – responded founders compromised on slavery

Seeks to explain the logic of the documents rather than the history

The effect of constitutional rules is to limit the discretionary power available to


government officials; insurance against risk of exploitive government

Madison’s arguments for a stronger central government are based on a role in


checking the abuse of discretionary power by majority factions in the legislature

Frederickson and NPA – public administration being used as an instrument of


repression

Rohr (1993) response to Spicer/Terry criticisms

Without the slavery compromise, it would likely have been impossible to form “the
more perfect union”

The consideration of the founding fathers as human beings is secondary to the


practical consideration that their influence still has an enormous effect on the
normative dimensions of contemporary public argument

Contention that rational individuals would, of necessity, choose a democratic


government is elitist; the founders chose as they did because of a reasoned
assessment of their peculiar historical situation

Pro-slavery criticisms may diminish respect as persons but it leaves untouched the
principles of the regime that was founded

Jefferson on slavery: “We have a wolf by the ears and we can neither hold him nor
safely let him go.”

Warren (1993)

By the end of the 1930s the courts had completely accepted the major role the
administrative state was playing in our governmental system as constitutionally
legitimate (Kenneth C. Davis)

The administrative state became a blatant reality during FDR’s administration

The administrative state is legitimate not only because our sociopolitical legal system
has sanctioned it but because the administrative state is obviously consistent with

151 | P a g e
recent tradition, established sociopolitical patters and is overwhelmingly supported by
public opinion

The more representative the public service becomes, the more legitimate as a
governing body it tends to be

New PA school held that the administrative state was illegitimate because it has long
ignored the centrality of human dignity

Others have argued that the bureaucracy is illegitimate because all parties do not
have equal access to the bureaucracy’s decision-making

Each individual has a unique perspective of what constitutes legitimate administrative


behavior on the part of the administrative state

The agonies of the bureaucratic age constitute reality and we can best deal with big
bureaucracy if we acknowledge its existence, try to understand it and then do what we
can to cope with it (Hummel, 1987)

It should be stressed that both liberal and conservative courts have upheld the power
of the administrative state by deferring time after time to agency discretion and
expertise

Congress has sanctioned the legitimacy of the administration through delegation

Wise (1993)

Federalist 15 implies that since coercion is necessary in government, the people


should be glad to choose civil rather than military coercion

When examining the contributions of the founders we should concentrate on the


relevancy of their experience as applied to real governing.

Hamilton argued in Federalist 27 that the more people experienced the actual
operation of government on the common occurrence of their lives “the more it will
conciliate the respect and attachment of the community”.

Once public administrators renounce their subordinate position in favor of an


independent position of interpreting the constitution, they forfeit the legitimacy that
the actual constitutional scheme offers

152 | P a g e
Lowi (1993)

Administration was legitimate as long as it was consistent with democracy and it was
consistent with democracy as long as it was accessible to the common person

Nowhere in the constitution is an explicit theory of separation of powers as a principle


of limitation on the national government

Article I and Article II indicate that the executive branch was to be an institution of
delegated powers

Spicer/Terry Debate (1993) The Final Response

Meier/Kaufman: Why do scholars, elected officials, political pundits, interest groups


and the general public continue to worry over the ominous specter of a powerful
imperial bureaucracy controlled by non-elected and nonpolitically appointed public
officials?

Freedman: the history of the modern administrative process can be seen as having
been marked by an extended sense of crisis. This sense of crisis is something more
serious than routine criticism and reflects a persisting uneasiness over the place and
function of the administrative process in American government

STATS/METHODS

What are the possible limits of quantitative policy analysis? Can qualitative analysis
help the policy analyst to overcome all of them?

Research in the social sciences uses several different methods to answer questions.
The experimental method is used with the quantification of data, the process of
converting data to a numerical format (Babbie, 396) and used in evaluation research.
Another method is quasi-experimental – non-rigorous inquiries somewhat resembling
controlled experiments but lacking key elements such as pre-and post-testing and/or
control groups (Babbie, 349). The third type of method is the qualitative evaluation

What are the elements of the classical experimental design?

The elements of the classical experimental design are:

153 | P a g e
 Randomization (R) – each subject or case has an equal chance of being assigned
to the experimental group or to the control group (random sampling)
 Observation or Measurement 1 (pretest)
 Administration of the experimental treatment to the experimental group, but
not the control group.
 Observation or Measurement 2 (Posttest)
 Compare the measurement of the two groups

Classical Experimental Design

Group Randomization Observation 1 Treatment Observation 2


Comparison

Experimental Re Oe1 X Oe2


Oe2 – Oe1

Control Rc Oc1 Oc2


Oc2 – Oc1

Step 1 Assign subjects to two or more groups, with at least one “experimental” and
one “control,” so that the groups are as comparable as possible. The best way to
assemble comparable groups is through random assignment of subjects to groups.

Step 2 Measure all subjects on relevant variables. Although a pre-experiment


measurement or pretest is usually administered, some experimental designs do not
require a pretest.

Step 3 Expose the experimental group(s) to a treatment or stimulus, the independent


variable. Ensure that the other control group(s) is not exposed. Exposure to the
treatment should constitute the only difference between the groups.

Step 4 Measure the groups again on the requisite variables in the postexperiment
groups.

Step 5 Compare the measurements of the groups. If the independent variable does
lead to changes in the dependent variable, this result should be evident in pretest-

154 | P a g e
posttest comparisons between the experimental and control groups. Or, if the groups
are large and known to be equivalent through random assignment, the analyst can
simply compare posttest scores between the two groups. If the causal inference is
valid, these comparisons should bear out predicted differences between the
experimental and control groups.

What are the requisites necessary for researchers to conclude that a causal

Relationship exists? Why is this the case?

The three main criteria for causal relationships in social research are 1) variables
must be correlated – there is an actual relationship, 2) the cause takes place before
the effect – time order, and 3) the variables are non-spurious – there is not a third
variable effecting the relationship (Babbie, 90). Note these definitions:

Correlation – An empirical relationship between two variables such that 1) changes in


one are associated with changes in the other or 2) particular attributes of one variable
are associated with particular attributes of the other. Correlation in and of itself does
not constitute a causal relationship between the two variables, but is one criterion of
causality (Babbie, 90).

Time order – We can’t say a causal relationship exists unless the cause precedes the

effect in time (Babbie, 90).

Spurious Relationship – a coincidental statistical correlation between tow variables,


shown to be caused by some third variable (Babbie, 91).

An alternate answer, that is, there are four criteria to conclude that a causal
relationship exists. The criteria are 1) time order – If A is the cause of B, then A must
precede B in time. Also, Changes in A must occur before changes in B. So, cause
must precede effect. 2) Covariation – means that the two variables move or vary
together. If A changes and B also changes, this covariation provides some evidence
that A is the cause of B. If changes in are never accompanied by changes in B, then A
cannot be the cause of B. 3) Non-spuriousness – a relationship is an association
between two variables that cannot be explained by a third factor. 4) Theory – Not only
must the conditions of time order, covariation, and non-spuriousness be satisfied, but
also a theoretical or substantive justification or explanation for the relationship must

155 | P a g e
be provided. Theory interprets the observed covariation; it addresses the issue of how
and why the relationship occurs (Meir, K. J. & Brudney, J.L., 32-34).

Please answer the following question in its entirety:

a. What are the requisites necessary for researchers to demonstrate causality?

b. What type of research design is most likely to enable a researcher to conclude


that a causal relationship exists? Why is this the case?

c. How do you distinguish between internal and external validity?

d. Propose an experimental research design that can be utilized by public


administration analysts. In that experimental design you are to specify:

The nature of the problem that you wish to explore and why it is amenable to research
through an experimental design.

What is the major hypothesis that you would test; how would you operationalize the
independent and dependent variables?

How would you set up the experiment?

How would you collect the data?

How would you know that the data support or do not support the hypothesis?

How would your design control specifically for threats to internal and external validity?

Why is it very difficult to utilize experimental designs in the social sciences?

Answer the following question in its entirety:

In recent years much attention has been given to the issues of equity in funding and
performance of children in public schools. The Tennessee Supreme Court in 1993's
Tennessee Small Schools systems, et. al. V. Ned Ray McWherter, et. al ruled that the:

State Constitution imposes upon the General Assembly the obligation to maintain and
support a system of free public schools that afford substantially equal educational
opportunities to all students.

156 | P a g e
The small school districts that sued the state for equity in funding were primarily
poorer counties or cities or special school districts that did not have a substantial
taxing base in their communities.

It is now nine years after the implementation of the funding equity formula and you
have been called upon to design a study to see if equity in funding has led to
comparable performance between the small school systems and the non-litigants in
the case.

How would you operationalize performance? What are the component parts that
would comprise a valid, reliable measure of performance in the context of the lawsuit?

How would you operationize equity? What are the component parts that would
comprise a valid, reliable measure of equity in the context of the lawsuit?

What type of research design would you construct to do this study? What would be
your unit of analysis? Why? What would be your sampling frame?

Babbie (2007) – see also Joy Clay study guide for first 5 chapters of Babbie defines
“theory” as “a systematic explanation for the observations that relate to a particular
aspect of life”

Social theory is more descriptive than normative

Conceptual definition – refinement and specification of abstract concepts, a bridge


from direct and indirect observables to useful constructs (theoretical creations based
on observations…)

Operational definition – specifies precisely how a concept will be measured

Kuhn (1962)

Positivist and post-positivist paradigm

The only reality we can understand is the reality that we can measure using formal
models

Can’t have a paradigm or discipline without scientific technique

157 | P a g e
Normal science; allows us to create knowledge; left up to discipline to determine how
knowledge is made

The researcher has a responsibility to declare belief in a paradigm, so a shift can be


recognized and a discipline can exist.

It’s OK for PA to know what it is; Consensus is found in the research

H. George Frederickson () caused a “scurrying” to find the paradigm in social sciences,


PA is barren

Hal G. Rainey points to Masterman’s (1970) criticism of Kuhn’s (1962)use of the term
paradigm in 21 distinct senses as an indictment of the whole “identity crisis” in PA

Yin (2003)

Case study research methods

Daneke (1990)

Advocates a mixed methods approach

The environment affects the organization (environment is mostly politics)

Mixed methods promotes comprehensive knowledge

New tech should advance sciences

Must have some consensus for a paradigm to exist

There are multiple paradigms, esp. in social sciences, but for a science to exist, must
have agreement; PA is not a paradigm; it is pre-paradigmatic; PA doesn’t have big
questions (from Stanley 5th paradigm paper) Daneke, on the other hand, indicates
that paradigmatic progress is essential for establishing an identity of public
administration. He advocates the use of advanced systems theory for his paradigmatic
choice in public administration, again suggesting that public organization theory has
dealt with this issue.

Ziman (1968)

Anyone can make an observation or conceive a hypothesis

Scientific knowledge is more than an observation.

158 | P a g e
Facts and theories must survive a period of critical study and testing by other
competent and disinterested individuals and have been found so persuasive that they
are almost universally accepted.

The objective of science is not just to acquire information not to utter all non-
contradictory notions

The goal is a consensus of rational opinion over the widest possible field.

Recognition that science knowledge must be public and consensible allows one to
trace out the complex inner relationships between its various facets.

Intellectual = attempt to discriminate between scientific and nonscientific disciplines

Psychological = role of education, the significance of scientific creativity

Sociological = Structure of the scientific community and the institutions by which it


maintains scientific standards and procedures.

Popper (1963)

Popper’s Formulated Conclusions to incompatible with certain possible results of


observation:

Easy to obtain confirmations, or verifications, for nearly every-theory

Confirmations should only count if they are the result of risky predictions

The more a scientific theory forbids certain things to happen, the better it is

A theory is nonscientific if it is not refutable by any conceivable event.

Every genuine test of a theory is an attempt to falsify it.

Confirming evidence does not count unless it is the result of a genuine test of the
theory.

Some genuinely tested theories, when found false, are still upheld by their admirers.

The criterion of the scientific status of a theory is its falsifiability, or refutability, or


testability.

159 | P a g e
The criterion of falsifibility because it states that the statements or systems, in order
to ranked as scientific, must be capable of conflicting with possible or conceivable
observations.

The beginnings of the use of a null hypothesis in testing

Feyerabend (1975)

Any ideology that breaks the hold a comprehensive system of thought has on the
minds of men contributes to the liberation of man.

Any ideology that makes man question inherited beliefs is an aid to enlightenment.

The teaching of “facts” without the attempt to awaken the critical abilities of the pupil
so that he may be able to see things in perspective.

Has Science found the correct method for achieving results?

Theories cannot be justified and their excellence cannot be shown without reference to
other theories.

Science itself is not clear, unambiguous, and precisely formulated.

Has science produced results with its methods?

Great scientific advances are due to outside interference which is made to prevail in
the face of the most basic and most “rational” methodological rules.

There does not exist a single argument that could be used to support the exceptional
role which science today plays in society.

There is no “scientific methodology” that can be used to separate science from other
ideologies.

Science is just one of many ideologies that propel society and it should be treated as
such

There must be a clear separation between science and state such as there is a clear
separation between church and state

Science may influence society to the extent that any other political or other pressure
group my influence society.

160 | P a g e
Final judgement must be left to the democratically elected bodies.

Kitcher (1982)

Evolutionists vs. Creationists

Science demands proof

Can be argued that there is no “proof” supporting either side.

“Complete certainty is best seen as an ideal toward which we strive and that is rarely,
if ever, attained.”

Fallibility is the hallmark of science

Historically, the natural sciences is “strewn with the corpses of intricately organized
theories, each of which had, in its day, considerable evidence in its favor.”

“Forlorn skepticism” - George Berkeley

From the idea of science as certain and infallible, scientists jump to a cynical
description of their endeavors.

Science is sometimes held to be a game played with arbitrary rules, an irrational


acceptance of dogma, an enterprise based ultimately on faith.

Theories win support by producing claims:

About what can be observed

That would not have seemed plausible prior to the advancement of the theory

That are in fact found to be true when we make the appropriate observations.

Theory as a collection of claims or statements

Some offer generalizations

Used to infer whose truth or falsity can be decided by observation

Theory is supported when we find that the observational consequences are true

The credentials of a theory are damaged when some of the observational consequences
are found to be false.

161 | P a g e
Successful Science:

Independent testability = achieved when possible to test auxiliary hypotheses


independently of the particular cases for which they are introduced.

Unification = result of applying a small family of problem-solving strategies to a broad


class of cases.

Fecundity = grows out of incompleteness when a theory opens up new and profitable
lines of investigation.

Hempel (1948)

Science attempts to provide explanations for phenomena occurring in the physical and
social world

“Explanations are arguments offered to establish that the event-to-be-explained had to


occur given the initial conditions and the presence of certain regularities in nature”

Laws are what distinguish scientific explanations from descriptions

If it’s a law, it will be true in all places at all times.

The typical assumption is that human behavior is unique and often un-repeatable;
therefore, it is difficult to determine causality.

Hempel suggests that even in physical sciences exact replication may not be possible.
He also purports that the concept of ‘causal explanation’ is misunderstood. All that is
needed for causality is for antecedent characteristics to be met and for those events to
repeat.

Can we explain human behavior?

Situational characteristics

History of the individual

Hempel argues that we can make generalizations based on previous experiences and
behavior.

162 | P a g e
“The determining motives and beliefs must be classified under antecedent conditions
of a motivational explanation, and there is no formal difference on this account
between motivational and causal explanation”

Denzin and Lincoln (2003)

On case studies:

As a practice one might commonly associate with qualitative analysis, Denzin and
Lincoln discuss the importance of the case study. Different types of case studies are
identified as intrinsic studies, instrumental studies and collective. The intrinsic study
is where the researcher wishes to better understand a particular situation or
occurrence versus the instrumental study where the case itself isn’t as important
because the researcher is seeking to gain insight on a problem or issue. Finally there
is the collective where the researcher examines a number of cases in order to better
understand a cohort, society, problem or phenomenon. The authors acknowledge that
these are not the only descriptions for case studies but they share the main traits that
are accepted in the field (136-138). The authors suggest that researchers only
undertake case studies that provide an understanding of other cases the field has be
underutilized and that the current methodology of the field, which stresses
contributions to generalization, further exacerbates the problem (140).

On program evaluation

The authors undertake an important discussion to public administration with their


examination of social program evaluations. The presented goals of a social program
evaluator is to improve the service and raise the public discourse on a policy or
program (590). The authors suggest there are a couple of good reasons that a
qualitative process is a good process for evaluations. First is the fact that the
epistemologies of interpretivism and constructivism were founded in the traditions of
philosophy and value pluralism. Next is the fact that these perspectives value the
stakeholders who are involved with the evaluated program giving the researcher a
“contextualized understanding (595).” The argument is presented that a qualitative
methodology will tell a story of a situation rather than just presenting data. This is
important because as the evaluator is an ethnographer they will understand the plight
and situation of those who are impacted by the program or policy that is being

163 | P a g e
evaluated. (601). This process isn’t without its detractors who will ask how good is a
particular program and whose interests are really being advanced. Because this style
of evaluation doesn’t have strong scientific method and is more philosophical in nature
there won’t be ready answers for the critics. The evaluator must keep in mind several
recognition inquiry cannons which guide all of their work in order to properly deal
with these questions: credibility, dependability, applicability and confirmability (606).
With all of this understanding the authors say that qualitative evaluations should be
conducted by individuals who have a constructivists perspective and have strong
morals so that they will have the ability to showcase honest representations to the
stakeholders.

On qualitative versus quantative

And the ability to weave these skills into research (the quilt) that can be used for the
betterment of society is not without its detractors. The work of these researchers who
use qualitative methods is considered a soft science and subjective due to its position
of not accepting that all research is going to be value-free presentation. The largest
group of detractors are those who hold the numerical and quantitative methods as the
only real study that can offer insight into reality. The specific differences between
quantitative method and qualitative method is the presentation of positivism and
postpositivisim, which share the opinions mentioned that there is one reality and it
can only be understood with the use of structured analysis (statistics.) Quantitative
analysis relies on a postmodern perspective, versus the positivist and postpostivist
perspective meaning that a postmodern researcher believes that the data gathered
from a structured analysis is but one way of representing the reality. A postmodern
researcher is looking for other ways to tell the story of the situation (i.e. make the
quilt.) A quantitative researcher is going to have the perspective of one who needs to
better understand an individual’s point of view through examining the constraints of
everyday life. Being an active part of the research is what truly separations between
qualitative and quantitative methods (13).

On Mixed Methods

Once the data has been collected in the initial survey and followed by interviews and
document review the information will then be reviewed through a series of approaches
to best capture the usefulness of examining the phenomenon. This “mixed-methods”

164 | P a g e
approach enhances the usefulness of the data for future policymakers (Marshall &
Rossman, 1995). The first method used to examine the data collected will be a
process called triangulaization. Trinangularization is a term based in surveying where
knowing any two points on a map and their distance between one another, allows the
surveyor to find the distance to any other point on the map (Patton, 2002). Much like
the three points of the triangle, using this mixed-methods approach to the data allows
the researcher more examples of what exactly occurs with these local governmental
entities and their use of performance measurement. There are four forms of
trinagulaization; data triangulation, investigator triangulation, theory triangulation
and methodological triangulation (Denzin & Lincoln, 2003). This research design has
already presented a proposal to use methodological triangulation with its use of a
survey which is more quantitative in nature and interviews and document research
which is more qualitative in nature. The research findings in the quantitative function
of the design give the researcher the directions; a roadmap as to who should be the
subject of further research. The qualitative component, documents and interviews,
will give the researcher a better understanding as to what, why and how occurs with
the local governments and their decision-making.

ADMIN MANAGEMENT

Mintzberg (1998)

“There is no ‘one best way’ to create strategy, nor is there ‘one best form’ or
organization. Quite different forms work well in particular contexts.”

Strategy as plan, ploy, pattern(consistency in behavior), position (within an


environment), and perspective (character of an organization)

Strategic Management – Making the strategic plan work

Strategic Planning (Fred David 2007)

Formulation

Identifying opportunities and threats in the environment

Attaching estimates or risk to the alternatives

Strengths and weaknesses evaluated with resources on hand

165 | P a g e
Actual or potential capacity should be estimated objectively

Preferences and politics

Ethical considerations

Implementation

Evaluation

Performance Measures – Benchmarking (for performance/production improvement)

Best Value or Best Value for Money (for accountability)

Principal-Agent Theory

Porter – Cost Leadership

Differentiation

Focus

Mintzberg – locate core, distinguish, elaborate, extend and reconcieve

BUDGETING

Budgeting Comps

Questions (from Mid-Terms and Comp Questions)

In her article treating budgeting…Naomi Caiden concludes that “Public Budgeting is


undoubtedly a discipline in the dictionary definition of the term as a “branch of
knowledge or learning.” What evidence supports this assertion? Caiden then cautions
“But if the study of pubic budgeting is to be more than a collection of disparate
concerns, it requires stronger themes, and theories that act not only to unify them but
also to reveal the philosophical assumptions underlying empirical description and
normative proposals.” Assess the state of the discipline of public budgeting.

In a renowned 1940 article, V.O. Key, Jr. lamented about “the lack of budget theory”.
Twelve years later, Vernon B. Lewis attempted to construct a normative budget theory.
In one of her numerous writings, Irene Rubin noted that “budget theory today is
fragmented and incomplete…It is in the process of being invented.” Based on the work
of scholars who address this issue, discuss the extent to which the literature of public

166 | P a g e
budgeting evidences “theory”. Assess the limitations of such a “theory of budgeting”.
Why is budget theory still viewed as incomplete or fragmented? Briefly discuss at
least three of the major competing theories of public budgeting treated in the literature
(where appropriate, distinguish between normative and descriptive theories).

Some of the readings covered in this course posit various ideas or theories to explain
public budgeting “allocation decisions”. Write an essay that assesses the validity of
the following statements: While governments have tried various “rational” decision
making systems, and while scholars have posited various explanations,
incrementalism remains the best explanation for public budget decisions.

(Hint: Begin your essay with a description of specific rational budget decision systems
adopted and later abandoned at the federal level before you explain what
incrementalism is and why many scholars still content it is the best explanation for
public budget decisions.

Trace the development of public budgeting in the United States beginning with the
Report of the Commission on Economy and Efficiency (Taft Commission) up to the
passage of the Congressional Budget and Impoundment Act of 1974. Focus on the
contributions of the key players (individual reformers and commissions) involved in
the reform effort and highlight the major legislation that shaped the development of
the federal budget process. What are the most common elements shared by the early
reformers and what are their major differences?

Cite the major federal financial and budget reforms of the past decade(or some other
time period), identifying the basic characteristics (provisions) of these reforms.
Discuss the implications these budget reforms hold for public management at the
federal, state and local levels. Discuss the implication of the value assumptions
implicit in these reforms vis-à-vis the fundamental values and beliefs which undergird
public administration.

What is performance based budgeting (PBB)? What differentiates PBB from PPBS and
ZBB? What are the similarities among the three? Explain the major opportunities
and challenges of implementing performance based budgeting. What is the purpose of
OMB’s Program Assessment Rating Tool (PART) in regard to PBB implementation?

167 | P a g e
What is a budget?

BUDGETING AS A DISCIPLINE/STATE OF THE DISCIPLINE

V.O. Key (1940) – “On what basis shall it be decided to allocate X dollars to activity A
instead of activity B?”

Key noted at the time of his writing that there were very few articles written on the
allocation of expenditures to achieve the greatest return.

Key asserts that budgeting is a form of applied economics, requiring the allocation of
scarce resources among competing demands

Sought to take the field away from focus on mechanics of budget-making

Vernon B. Lewis (1952) – Attempted to construct an economic theory of budgeting


based on three economic principles:

Return on expenditure must be worth its cost in terms of sacrificed alternatives

Incremental analysis is necessary because of the phenomenon of diminishing returns

Comparison of relative merits can be made only in terms of relative effectiveness in


achieving a common objective

Lewis’ plan involved focusing on budget alternatives (in terms of levels or quality of
service – ex. “Bare-bones; Mid-level; Cadillac”) and incremental requests for resources

Wildavsky (1961) defends Incrementalism, and asserts that we may never find a better
theory, since a budgeting theory would be the same as a political theory.

168 | P a g e
Lindbloom (1959) – “muddling through” thanks to bounded rationality, leading to
incrementalism

Schick (1966) wrote that dependent on the environment of a given time-period,


budgets emphasize three things

Financial control

Managerial improvements

Planning

Rubin (1996) adds Prioritization and Accountability to Schick’s list

Janet M. Kelly (2005) “A Century of Public Budgeting Reform: The “Key” Question

Kelly sees the answer to Key’s question in the cyclical nature of public opinion
regarding the role of government

Public opinion is in favor of business (or against govt.) the opinion is PRIVATE
REGARDING

Public opinion is in favor of govt. (or against business) the opinion is PUBLIC
REGARDING

Jacksonian (1829-1872) PRIVATE REGARDING

Progressive (1873-1921) PUBLIC REGARDING

Roaring Twenties (1921-1932) PRIVATE REGARDING

Depression and the New Deal (1933-1945) PUBLIC REGARDING

The Management Movement (1946-1958) PRIVATE REGARDING

Realism and the Great Society (1959-1969) PUBLIC REGARDING

The Modern Era (1969-1979) DISILLUSIONMENT, STILL PUBLIC REGARDING

Neo-Jacksonians (1980 to Present) PRIVATE REGARDING

Kelly also finds that incremental budgeting reflects American preferences for
incremental policy change

169 | P a g e
Traditional line-item budgeting promotes financial accountability

So, we have a theory of budget reform, which is a theory of public opinion cycles.

BUDGET THEORIES

Types of theory (from Rubin 1990)

Normative

Descriptive

Normative Theory Features

Based on narrow range of observations

Solutions may be based on values rather than observations

Weakness – if advice offered in normative theory isn’t followed, it widens the gap
between theory and practice

Evaluations of budget reforms have often been negative

Public participation tends to be short lived or non-existent (Incrementalism, Hierarchy


theories)

Normative budgeting is underestimated because evaluators look for short range


results instead of long range effects

Descriptive Theory Features

Based on close observations or participation in public sector activity

Weakness – if the explanatory power of descriptive theory is too weak, it widens the
gap between theory and practice

Budget theory has been much weaker, often unable to see the phenomena in plain
view to theorize about their meaning

Public Choice

According to Rubin (1992) and Forrester (2001)

Forrester (2001) suggests Public Choice is a response to Wilson’s PA Dichotomy

170 | P a g e
Human behavior is based on individual economic rationality and the maximization of
individual benefits (economists call this “utility”)

Emphasizes the relationship between what citizens want government to do and what
government actually does

Citizens vote in a manner that reflects their own spending priorities

Weaknesses of this theory include

Difficulty explaining change over time

Cannot explain why government has not expanded further

Impossible to arrive at a figure for aggregate individual utilities

Conflicts:

Economics framework

Methods of analysis

Bureaucratic assumptions

Value orientations

Reality

Incrementalism

According to Rubin (1992)

Budgeting occurs exclusively inside government (to the exclusion of citizens)

Based on bounded rationality (Herb Simon (1947) – “satisfice”; Lindbloom (1959)


“muddling through”)

Government is not directly or indirectly controlled by society

Interest groups exist, but do not determine outcomes

Many reforms required comprehensive evaluation of programs and specific delineation


of spending, which would have negative effects (Rubin 1990)

Weaknesses of this theory include

171 | P a g e
Underestimation of the relationship between society and budgeting

Overestimation of agency autonomy in determining budgets

Fails to account for a budget process that responds to societal ills, emerging situations
or environmental changes

Assumes budgets will be allocated the same way from year to year

Rubin and Schick believe that incrementalism as an explanatory model did not
describe the budgeting process well, noting a budgetary base is not always defined

According to Neuby (1997):

Wildavsky, Lewis and Lindbloom tend to support incrementalism

Lewis (1988) notes recent budget reforms such as ZBB and PPPB have not altered us
from a base and increments

Principal-Agent Theory

According to Forrester (2002)

When applying this theory to budgeting, the budget itself is the contract between the
principal and the agent

Focuses on the relationship between those who allocate resources and those who
provide agency services

Information management/distribution

Principals and agents manage information and both may act in their own self-interests

Managing information is challenging because of the conflicting interests

Adverse Selection: principal picked the wrong agent or has incorrectly identified the
agent’s role, responsibility or agenda

Hierarchical relationships among budget participants

Principals are setting policies and goals

Agents (service agencies) implement the programs that will address the principal’s
policies or goals

172 | P a g e
Hierarchy Theory

According to Rubin (1992)

Argues that top-level executives make decisions about broad policy issues, judge the
environment and pass that info down via the budget office to the agencies before
agencies make their requests

Similar or the same as Elite Theory

Macro-Micro Budgeting

According to Rubin (1992)

Bargaining still exists over budget strategies, but broader policy issues are explicitly
dealt with and frame the choices and outcomes of the bargaining

Economic policies, priorities, spending ceilings and assumptions about the growth of
the economy are made by the budget committees and guide the decisions of other
committees

Neo-Marxist

According to Rubin (1992)

Class interests dominate budgeting and allocation choices

Govt. is controlled by capitalists and they determine spending base on their own
priorities

Theorists call attention to military spending as a way to enrich arms manufacturers


and tax breaks that only benefit the well-off more than the poor

The major weakness of this theory is that it doesn’t explain why one groups wins out
over the other

Transaction Cost Theory (not recognized as an official budget theory in PA, but it
applies)

According to Bartle (2001)

Developed in economics and focuses on private sector org forms

173 | P a g e
Transaction Costs: the costs that occur above and beyond the purpose of exchange

How does this relate to PA budgeting?

Costs involved in budget negotiations, contracts, deals, political exchanges, what


groups have to give so that another group can have

Concepts can be applied to budgeting and finance, as budgets are the culmination of
deals and agreements

Centered on institutions and history

Transaction cost theory is general enough to apply to a variety of findings under an


over-arching framework

Under-utilized as a theoretical construct in public budgeting and finance

Public budgeting needs to further incorporate transaction cost theory into a public
budgeting framework or theory

Weakness is there’s not an established measurement for all costs of providing


government/social services

Other Theories (from Stapleford 1992)

Classical (Pre-Keynesian) – Adam Smith, capitalism, free market, ltd. Govt.

Keynesian – John Maynard Keynes (1936), govt. spending is necessary to fine-tune the
economy; performance by economy in aggregate is more important than individual;
small deficits are useful for fine-tuning

Neo-Keynesian – larger tuning of the economy; larger swings in govt. spending/taxes;


use of monetary policy (interest rate setting) to influence economy

Monetarists (Counter-Keynesian) – Milton Friedman, Alan Greenspan; monetary policy


more effective than fiscal policy; economy is inherently stable – govt. fine-tuning not
needed; money supply/interest rates controlled by Federal Reserve is important

REFORMS

Line-item (prior to 1921 was the only model)

174 | P a g e
Agencies are evaluated on program to program (line to line) to measure efficiency and
determine funding

Congress needed large staff to do this

No rational decision-making mechanism was in place

Politics substituted for rationality

Led to the re-establishment of line-item budgeting with the executive budget (1921)

PPBS (emerged in the 1950s; implemented by LBJ in 1961 for D.o.D./1965 for all
federal agencies)

Utilizes Cost Benefit Analysis and systems theory

Emphasizes the planning stage to determine and view program objectives and outputs

CBA is conducted on the outputs; if objectives are not met funding will be
discontinued

Process creates “enormous information and analytical burdens” (Schick 1973)

It is difficult to set program objectives for social policy and critique the program based
on those objectives

Military uses it to this day, though it fell out of fashion in the late 1960s

Officially died with OMB memo in 1971

Target-Based Budgeting (1980s)

Target-based budgeting is a another form of a zero-based budgeting, except that


agencies are told what the funding ceiling is going to be

Each agency will be allotted a certain amount of money to operate

The agency must prioritize those funds in the order they feel is important.

The budget is broken into two categories: operating and capital projects.

The agency may not get funding for capital, only if there is money left over

Performance-Based (Current)

175 | P a g e
Determine outcomes based on performance

Performance-based budgeting is outcome oriented

Determine allocation based on the output of that program

If output is efficient and effective, the program will receive funding

Problem with performance-based budgeting: how do you determine the efficiency and
effectiveness

Agencies define efficiency and effectiveness

Models cannot be applied to all other agencies because the output or criteria are
different

Placed each program on a system of merit, but people themselves are the ones who
defined the criteria

Zero-Based Budgeting (1977-1981)

Jimmy Carter brings this from Georgia to Washington

Every year the agency starts from zero

Agency performs a CBA – unallocated on all programs for which they are responsible

Recommendation goes up the chain of command

Takes too long, people ask for more than they need, no previous budget to refer to

HISTORY/TIMELINES

176 | P a g e
177 | P a g e
ORG. THEORY

Some words on Paradigms:

Nicholas Henry (1975) “Paradigms of Public Administration” PAR 35(4):

5 paradigms

PA Dichotomy 1900-1926

Principles of Administration 1927-1937

PA as Political Science 1950-1970 (PA resubmits to the domination of PoliSci)

PA as Administrative Science 1956-1970 (PA submits to the domination of


Administration, public and private)

PA as PA (?) 1970 - ? (PA comes into its own, following the Simon prescription in 1947)

1938-1950 – Challenge to PA Dichotomy and Principles:

1938 Barnard’s Functions of the Executive (influenced Simon)

1946 Fritz Marx ed. Elements of Public Administration (questioned dichotomy


assumptions)

1947 Simon’s Administrative Behavior (blows the principles away)

1947-1950 – Reaction to the Challenge:

1947 Simon’s “A Comment on ‘The Science of PA’” (prescribes two tracks – scholars
working on the science of admin and a larger group working on prescriptive public
policy)

Hal G. Rainey (1994) “On Paradigms, Progress, and Prospects for Public Management”
J-PART 4(1):

Frederickson Questions

How important that we have a paradigm?

Are we making progress toward one?

Which is the most promising?

178 | P a g e
Rainey Answers

Not very

No

Mine!

Elaborating, Rainey says:

The importance of a paradigm is debatable – yes, we can travel in a herd, with all the
advantages that might offer BUT we are also subject to the disadvantages

It’s difficult and dangerous to achieve and impose consensus on a field of inquiry

Distinctions between public/private are snagging progress in the field, esp. with org
theory

Paradigmatic Discussion:

Five paradigms:

Positivist

Post-positivist

Interpretivist

Conflict

Post-modern

Every paradigm should must have three things

An ontology – the nature of reality

An epistemology – how to know reality

A methodology – how to test reality

Given that, most professors don’t see Henry’s 1975 work as really being paradigms –
more like theories

But that really depends on your definition of paradigm, doesn’t it?

179 | P a g e
Henry accounts for this in his article by using paradigm as “How mainstream public
administrationists have perceived their enterprise during the last 80 years”

Stanley is using the Kuhn paradigmatic definition

Rainey points out the Kuhn used “paradigm” in 21 distinct senses in 1962

French, Spears & Stanley (2005) The Fifth Paradigm of Public Administration?...

Once Public Org Theory is redefined as Public Org Efficacy, paradigmatic progress can
begin

Efficacy = efficiency, effectiveness, equity, responsiveness and accountability through


various modes of research

The last step is recognizing the quantitative and qualitative nature of inquiry, and
establishing an equal respect for both – because some problems are better solved with
numbers, some with words

Classical Organization Theory

Authors/Works:

Fayol (1949) General Principles of Management in General and Industrial Management

Taylor(1916) Principles of Scientific Management in Bulletin of the Taylor Society

Weber (1946) Bureaucracy in Essays in Sociology

Gulick (1937) Papers on the Science of Administration

Main Points to Remember:

Dominated thought into the 1930s

Structuralists--focused attention on structure or design of orgs

Rational and closed systems pursuing the goal of efficiency

Adam Smith, Henri Fayol, Daniel McCallum, FW Taylor, Max Weber, Gulick &Urwick

Organizations should work like machines, using people and capital as their parts

Adam Smith (1776) The Wealth of Nations

180 | P a g e
Developed the concept of Division of Labor

Gives the example of the pin factory

Assembly lines would yield mass production

Mass production would increase efficiency

Frederick Taylor – Scientific mgt; legal-rational basis of govt; “one best way”; time &
motion studies

Max Weber – bureaucracy as an ideal; perpetuates p/a dichotomy

Formal rules and regulations

Division of labor

Written documents

Hierarchy

Formal Authority

Formal education/certification for employment

Daniel McCallum, 1856, first modern organization chart for the NY and Erie RR
Company

Fayol's organizational principles: technical, commercial, financial, security,


accounting, managerial (greatest emphasis on managerial)

Taylor's "one best way"

Gulick & Urwick's POSDCORB; organize work by purpose; process; persons or place

Often viewed as narrow and simplistic; however, laid a foundation for all future
scholars

NeoClassical Organization Theory

Authors/Works:

Barnard (1938)The Functions of the Executive

Merton (1957) Social Theory and Social Structure

181 | P a g e
Simon (1946) Proverbs of Administration, PAR 6 1946

Cyert and March (1963) A Behavioral Theory of the Firm

Main Points to Remember:

Transitional theory that revised Classical Theory by adding human element, 1950

An organization cannot exist outside of its environment

Robert Merton, James March, Philip Selznick, Chester Barnard and Herbert Simon

Barnard: individuals are what hold the organization together; thus, they must be
reduced to cooperate for success to be achieved (persuasion principle)

Simon: openly and vehemently attacked Classical Theory, said G&U's principles were
merely proverbs, offered idea of satisfice

Selznick: idea of cooptation (TVA study) and the Institutional Matrix

Cooptation - the process of absorbing new elements into the leadership or policy-
determining structure of an organization as a means of averting threats to is stability
or existence – balance between action and formal systems

Institutional Matrix is the combination of the Action System and the Formal System

Action system – the socialization aspect of the org

Formal system – structure, authority, procedures, process

Cyert and March: alliance-forming and coalitions; Garbage Can Model

Unclear goals

Imperfect technology

History of the organization

Input of new org members, training

All public organizations are in big garbage cans: everyone has varied backgrounds,
educational experience, demographic characteristics, and they are thrown into the
same organization together, somehow, they make it work.

182 | P a g e
Merton: informal organization among workers; latent functions of standardized
practices

Human Relations Organization Theory

Authors/Works:

Follet (1926) Scientific Foundations of Business Administration

Follet (1926) The Giving of Orders

Mayo (1933) The Human Problems of an Industrial Civilization (1st account of


Hawthorne)

Roethlisberger (1939) Managemen and Morale (definitive account of Hawthorne)

Maslow (1943) A Theory of Human Motivation, Psychological Review 50 1943

McGregor (1957) The Human Side of Enterprise

Janis (1972) Victims of Groupthink

Main Points to Remember:

People are considered to be as important, or more so, than the org itself

Elton Mayo considered founder of the Human Relations school of thought

Follet suggests superior-subordinate roles inhibit productivity

Boleman & Deal (1997): organizations exist to serve humans (not the other way
around)

Hawthorne Effect 1924-1932 (Elton Mayo studies at Western Electric plant)

Workers are part of a social group and respond to peers

Maslow's Hierarchy of Needs:

Physiological needs

Safety needs

Social needs

Esteem needs

183 | P a g e
Self-actualization

McGregor's Theory X & Y

X – people inherently dislike work and will avoid it if possible

Y – work can be a source of satisfaction for people

Janis' Groupthink:

A mode of thinking that people engage in when they are deeply involved in a cohesive
in-group, when the members' strivings for unanimity override their motivation to
realistically appraise alternative courses of action

Frederick Herzberg – psychologist concerned with mental health and work

Motivation-hygiene theory: the job versus the environment

Satisfiers/motivators: achievement, recognition, responsibility, growth

Dissatisfiers/hygiene factors: company policy; supervision; interpersonal relations;


salary status; job security; personal life

Most optimistic of all schools - under right circumstances, people and organizations
will grow and prosper together

Modern-Structural Organization Theory

Authors/Works:

Burns & Stalker (1961) The Management of Innovation

Blau & Scott (1962) Formal Organizations

Walker and Lorsch (1968) Organizational Choice: Product vs Function,

Harvard Business Review, Nov. 1968

Mintzberg (1979) The Structure of Organizations

Jaques (1990) In Praise of Hierarchy, Harvard Business Review Jan.1990

Main Points to Remember:

Second half of 20th Century

184 | P a g e
Hierarchy, formal rules in place to attain goals

Sought a return to the Structural Element, retaining the human aspects of


Neoclassicism

"Modern" in quotations b/c it is used simply to refer to the time period--there is little
substantive difference between the Structuralists in Classical Theory and this one,
other than time frame

Organization efficiency is the essence of organizational rationality, and the goal of


rationality is to increase the production of wealth in terms of real goods and services

Mintzberg and the 5 (6) Parts of an Organization:

Strategic Apex (top mgt)

Middle Line (mid mgt)

Opearting Core (operations, op processes)

Technostructure (analysts that design processes, systems)

Support Staff (supports operating workflow)

Post 1979 article – Ideology (org culture)

Peter Blau and Richard Scott: all orgs consist of a formal and an informal element and
it is impossible to understand an org. without knowing each element

Walker and Lorsch, Lawrence and Lorsch: stable environments/bureaucratic orgs;


unstable environments/non-bureaucratic orgs

Buzzwords: differentiation, specialization and integration

Elliott Jaques: remains a lonely defender of the bureaucratic-hierarchy models; in that


the structure must be fixed, not the employees

Power and Politics Organization Theory

Authors/Works:

Pfeffer (1981) Power in Organizations

185 | P a g e
Kanter (1979) Power Failure in Management Circuits, Harvard Business Review, July
1979

Mintzberg (1983) Power in and around organizations

Main Points to Remember:

Organizations are viewed as complex systems of individuals and coalitions

Conflict in inevitable and influence is the primary weapon

Organizational goals change with shifts in the balance of power

John Kotter: differentiate between power resulting from authority and power resulting
from being able to get job done

Power is aimed in all directions, not just down the hierarchy

Jeffrey Pfeffer: power and politics are fundamental concepts in defining an org

Politics defined as "the process of gaining, maintaining and exercising power.


Organizational politics involves those activities taken within organizations to acquire,
develop and use power and other resources to obtain one's preferred outcomes in a
situation where there is uncertainty or dissensus about choices."

Kanter’s Theory of Structural Power in Organizations (1977):

Work behavior and attitudes are shaped by a person’s position and situations in the
org, rather than by personal characteristics and socialization experiences

Power structures (mobilization of support, allocation of resources, gathering of


information) are accessible via the position of a person in the org (and the power
inherent in it)

The extent to which a position accesses power structures is dependent on the formal
and informal power of the person

More Kanter:

"Power is the ability to mobilize resources, to get and use whatever it is a person needs
for the goal he or she is attempting to meet."

French and Raven (1959): Description for Power in Groups:

186 | P a g e
Reward power: the power to confer or withhold rewards that others want, i.e. pay

Coercive power comes from the ability to take forceful action against another

Referent power is when a person has power over others because they see him/her as a
standard to emulate

Expert power is the control of knowledge, information and other skills

Legitimate power is where others accept the authority and ability to tell them what to
do

Systems Theory

Authors/Works:

Katz & Kahn (1966) The Social Psychology of Organizations

Thompson (1967) Organizations in Action

Richard W. Scott (2003) Organizations: Rational, Natural and Open Systems

Main Points to Remember:

Rose to dominance in the late 1960s

Daniel Katz and Robert Kahn: organizations are open systems

Apply Ludwig Bertalanffy's (1951) general systems theory to organizations and use
quantitative tools and techniques to understand complex relationships among
organizational and environmental variables

(remember inputoutput / blackbox diagram)

Search for order in complex systems, cause-and-effect oriented

Seeks optimal solutions (not "one best way")

Computers, experts, etc. are the tools necessary

Draw heavily from Neoclassicals---bounded rationality and satisficing (Simon) and


cognitive limits (Cyert and March)

187 | P a g e
Richard Scott (2003) – rational, natural and open systems theories OR Chinese
handcuffs

Rational systems do not take the environment into account

Rational systems view organizations as a means to an end goal

Natural systems focuses on the org’s survival in the environment, emphasizing the
survival of the organization for its own sake, in spite of the hostile environment

Open systems portrays the organization as a living organism with loose boundaries,
sub groups attaching and detaching, inputs and outputs moving at the same time

Open systems recognizes a changing environment

Genius of the systems theory is that competing schools of thought can be classified as
sub-theories (Scott says class, neo-class and HR are closed rational or closed natural
systems)

To complicate it further, the rational natural and open perspectives may apply to the
same organization, just at a different level of analysis

One more bit on paradigms and I promise it won’t hurt too much…

A take on Scott:

Rational Systems Scholars and One Best Way Positivist

Natural Systems Scholars and Post-Positivist – maybe


Motivation/People Interpretivist

Open Systems Scholars and Environment Interpretivist

Scott’s Big Question:

How do you assess organizational effectiveness?

Stanley’s Big Question about Scott:

What’s in the Black Box (and how would you measure it)?

188 | P a g e
ORGANIZATION THEORY: A CHRONOLOGY

400 B.C. SOCRATES – MGT IS AN ART UNTO ITSELF

360 B.C. ARISTOTLE – CULTURAL CONTEXT

1776 ADAM SMITH – OPTIMAL ORGANIZATION OF PIN FACTORY

1813 ROBERT OWEN – EMPLOYEES ARE VITAL MACHINES

1910 LOUIS BRANDEIS AND FREDERICK TAYLOR – SCIENTIFIC MANAGEMENT

1922 MAX WEBER –BUREAUCRACY AS A STRUCTURE

1937 GULICK’S POSDCORB

1940 MERTON AND THE DYSFUNCTIONS OF BUREAUCRACY

1946 SIMON ATTACKS THE PRINCIPLES APPROACH

1948 WALDO ATTACKS THE GOSPEL OF EFFICIENCY

1949 SELZNICK AND TVA’S COOPTATION

1954 DRUCKER AND MANAGEMENT BY OBJECTIVES

1957 ARGYRIS & THE CONFLICT BETWEEN PERSONALITY AND THE


ORGANIZATION

1961 THOMPSON FINDS DYSFUNCTION DUE TO ABILITY VS AUTHORITY

1962 PRESTHUS’ UPWARDMOBILES, INDIFFERENTS AND AMBIVALENTS

1964 CROZIER: BUREAUCRACY AN ORGANIZATION THAT CANNOT LEARN FROM


ERRORS

1966 BENNIS PROCLAIMS DEATH TO BUREAUCRATIC INSTITUTIONS

1968 HERZBERG – MOTIVATORS, SATISFIERS AND HYGIENE FACTORS

1972 CLEVELAND – CONTINUOUS IMPROVISATION IS REQUIRED

1976 MACCOBY AND THE GAMESMAN

1981 PFEFFER – POWER IN ORGANIZATIONS

189 | P a g e
1983 ROSABETH MOSS KANTER AND THE CHANGEMASTER

1988 ZUBOFF AND THE AGE OF THE SMART MACHINE

1990 GAGLIARDI AND SYMBOLS AND ARTIFACTS

1992 OSBORNE AND GAEBLER RE-INVENT GOVERNMENT

1997 VIRTUAL ORGANIZATIONS AND BEYOND

2000 SNOOK ANALYZES SYSTEMIC BREAKDOWN IN FRIENDLY FIRE

2002 PERROW & ORGANIZING AMERICA: WEALTH, POWER & ORIGINS

CORPORATE CAPITALISM

PUBLIC POLICY

Some scholars have argues that “public policy is public administration”. First,
evaluate whether this statement accurately captures the field to date. Then, examine
if this statement – or a replacement – explains where the field appears to be heading
for the foreseeable future.

Public policy making is made by various official and unofficial stakeholders in the
polity. Identify these stakeholders and describe the “legitimate” power that each
stakeholder possesses in the policy making process. Secondly, is policy making a
“process” or does it resemble something else? How can it be improved?

The “policy orientation” after 40 years: Review major developments and their
implications for researchers and practitioners.

What characterizes different policy arenas? Describe one typology and discuss how it
might help to conduct public policy analysis.

Discuss two views about the proper way to study policy implementation.

What is agency (principal-agent) theory and how can it be used to improve our
understanding of public policy making?

THEMES

The state of the policy field

190 | P a g e
Theories

Is policy political? (Parallel to Wildavsky’s budget assertion)

Stakeholders

Process/Streams/Open Window

Principal-Agent

STATE OF THE FIELD

(Lots of crossovers with budgeting – since budgets are a form of policy)

Birkland (2001) – the systematic study of policy is a 20th century creation dating to
Charles Merriam who sought to connect the theory and practices of politics to
understanding the actual activities of government

Koven (1994) – sees 4 foci in the literature:

 Sequential study of specific policies


 Studies of the public policy process
 Study of analytical and evaluative methods
 Cross national comparison of public policy

Lindbloom (1959) – “muddling through” thanks to bounded rationality, leading to


Incrementalism

Wildavsky (1961) defends Incrementalism, and asserts that we may never find a better
theory, since a budgeting theory would be the same as a political theory.

Anderson (2006) and the Typology Discussion:

Older typologies were done by issue; institution; or time period

Newer typologies are:

 Substantive/Procedural
 Distributive, Regulatory, Self-Regulatory and Redistributive
 Material/Symbolic
 Collective Goods/Private Goods

191 | P a g e
Thomas Dye ( ) – defines policy as “Whatever governments choose to do or not do.”

Kingdon (1995) – know for “policy stream model”: a policy agenda, supported by a
policy community, becomes public policy when the three “streams” converge at a
crucial moment

Easton (late 1960s) – Grandfather of Public Policy – Code of Relevance for policy
research

THEORIES OF POLICY STUDY

(According to Anderson and Robinson Notes)

Political Systems Theory

Group Theory

Elite Theory (Dye & Ziegler)

Institutionalism

Rational Choice Theory (a.k.a. Public Choice)

(According to Thomas Dye)

Institutional – policy as institutional output

Process – policy as political activity

Rational – policy as maximum social gain

Incremental – policy as variations on the past

Group – policy as group equilibrium

Elite – policy as elite preference (Dye & Ziegler)

Public Choice – policy as collective decision making by self-interested individuals

Game Theory – policy as rational choice in competitive situations

THEORIES OF DECISION MAKING

Rational-Comprehensive

192 | P a g e
Incremental

Mixed Scanning (Amatai Etzioni) – looking for “hot spots” while watching the “big
picture”

POLICY AS POLITICS

STAKEHOLDERS

PROCESS/STREAMS/OPEN WINDOW

PRINCIPAL-AGENT

Iron Triangle – earliest formulation by Grant McConnell (1966) in Private Power and
American Democracy

Top-Down

Bottom-Up

193 | P a g e
Hybrid

Garbage Can (Cyert, March & Olsen)

194 | P a g e
Game Theory

Public Choice – Leaders manipulate pubic into choosing their previously conceived
policy preferences

195 | P a g e
1.16 Some Important Comps Questions and Student Answers
Question #1: What is the nature of public administration, and how has it
developed as a field?

The nature of public administration is in the distillation of political ideas, legislation


and policy into administrative action and regulations with resulting effects that
liberate or restrict the actions of citizens, groups and society in general. Politicians
and legislature make policy, administrators implement those policies. However,
proper administration requires a balancing of loyalty to political bosses with ethics
and loyalty to the public interest. It is this discretion, and authority that is a constant
source of debate.

The establishment of the Interstate Commerce Commission signaled the passing of


the US from a simple, agricultural society into a highly complex and interrelated Great
Society. Waldo says this new society, based on division of labor, specialization,
systems of transportation and communication, and high technology were all based on
a new method of controlling the environment called “scientific method”.

Closing of the Frontier: Economic and social readjustment from the closing of the
frontier along with the increasing ratio of population to available resources, caused a
movement aimed at saving our natural resources (and human beings) through
adopting the idea of a “planned” and “administered” human community. Urbanization
resulted from the closing of the frontier.

Our Business Civilization: From the beginning, America has been a uniquely wealthy
country, and has become characteristically a Business Civilization. This has
influenced our methods of administration. Business methods were applied to
managing the affairs of State. Forms of public organizations historically have followed
the corporate model closely.

Raadschelders says that PA first developed as goods and services were increasingly
provided by the government instead of private concerns. We went from a Nightwatch
State with voluntary militias to having permanent police forces and a standing army.
Services that were formerly provided by church organizations were increasingly
expected to be provided by government.

196 | P a g e
In response to corruption and the Spoils System, Woodrow Wilson wrote that PA
should be buffered from politics and developed into a discipline of its own with its
basis in scientific management and employing professionals to run government.
Politicians and legislature make policy, administrators implement those policies.

Scientific Management – Gulick with POSDCORB and Taylor with Scientific


Management tried to fit the role of PA into definitive actions and a hierarchical
structure with unity of command.

Herbert Simon critiqued the Scientific Management approach by saying that PA was
more of an art than a science. He called for “bounded rationality”.

As a field of discipline, PA has not settled into a paradigm of its own. There is much
debate about creating a unified theory and a normative model.

Denhardt says that PA cannot be a discipline because it draws from so many other
fields and sciences.

Minnowbrook I – sought to make a bold synoptic approach to the discipline of PA.

Blacksburg Manifesto – Sought to change the negative dialogue surrounding PA and to


develop a normative approach to the discipline.

Minnowbrook II – saw further fragmentation of attempts to define PA.

Others argue that it should be included in business as a form of “administrative


science”, but Khun says that PA does not meet the test of being called a “normal
science” because it lacks a generally accepted paradigm and methodology.

HRM – Human relations movement focused on the informal organization that exists
under the formal structures. People understand the limits of the formal structure and
fill in gaps with NORMS.

Institutionalism – Recognizes multiple stakeholders, public-private partnerships,


coproduction, privatization, contracting-out, and fuzzy distinctions between public
and private.

Governance Theory – Rohr says administrators should legitimately participate in


governance, not just management. Governance theory talks about the dominance of
policy networks instead of traditional govt structures. The State’s declining ability to

197 | P a g e
control means we should embrace multiple approaches to administration and a
increased blending of public and private resources. Steering rather than rowing.
Blacksburg Manifesto – earliest work.

New Public Administration – Promotes Social Equity in the performance and delivery of
public services. NPA called for a change in the public discourse about PA and a
movement away from the rational model toward increased social equity. Productivity,
Marketization, Service Orientation, Decentralization, Accountability. Sweep away
programs that don’t work.

Global Professional Technocracy – Stillman sees the current state and future of PA as
increasingly global and specialized.

The implications of this are: Policy definition by little clusters of professional experts,
hyper-impermanence (lack of adherence to a stable and immutable set of principles or
concepts) and complexity surrounding the policy arena, and the blurring of the
boundaries between public and private sectors.

Question #2: Addressing the literature, what are the issues in the relationship
between the public, politicians, and administrators?

Raad says that representation of the public is not synonymous with obedience to the
public. He also says that in Western societies, the Primacy of Politics means that
Bureaucracy and Administration are subordinate to politics.

Raad says the Highest Authority in Western systems of government is not a political
institution, but the COMPROMISE and the Negotiation Process between elected
officials.

Politicians and legislature make policy, administrators implement those policies while
being accountable to both politicians and the public.

Administrators should be NON-PARTISAN in their work.

Svara says that administrators must have an understanding of the complex issues
and implement policies while keeping the ultimate goals of service to the public and
ethical behavior as guiding principles.

To whom are administrators accountable?

198 | P a g e
Svara says the public administrator should act on behalf of the public interest, but is
directly accountable to his political leaders and the organization.

Frederickson would say that the public administrator is directly responsible to the
citizens. He says the Spirit of PA is in serving the people.

Rohr says the primary moral obligation of public administrators is to be the guardian
and guarantor of the founding values of every citizen.

Peters says that administrators should be APOLITICAL in implementing public policy,


but notes that a balance must be kept betwn Pol & public interest.

Question #3: Discuss the tensions between democracy and “bureaucracy”, and
how they can be reconciled.

Raad says Democracy is the rule of the People and Bureaucracy is the rule of bureaus.

Raad views the tensions as:

Bureaucracy seeks permanence and expansion for self-serving reasons with aims at
maximizing power, income, security and prestige.

Democracy dictates that bureaucracy’s sole purpose is to administer policy with the
main goals of efficiency and serving the public interest.

A major tension over the past 30 years is the increasing use of political appointees in
bureaucracy. This politicization of bureaucracy goes against the goal democracy and
what Weber calls for neutral civil servants, who are expected to be models of integrity,
rational, politically neutral and acting in compliance with the rules.

Raad says that as a result of the politicization of bureaucracy, bureaucrats have


substantial influence on policy and decision making. Bureaucracy should seek to
promote democratic, rather than its own values.

Raad recommends how they can be reconciled:

Balance individualism and collectivism (direct democracy vs. indirect)

Balance juridicial and sociological action (formal rules vs. discretionary)

Balance of abstract and concrete layers of understanding (long-term planning vs.


immediate political & operational concerns)

199 | P a g e
Question #4: Discuss the tensions between promoting administrative self-
interest and the public interest. How can they be reconciled?

Raad says that as a result of the politicization of bureaucracy, bureaucrats have


substantial influence on policy and decision making. Bureaucracy should seek to
promote the public interest, rather than its own values.

Bureaucracy has been associated with several pathologies and seeks permanence and
expansion for self-serving reasons with aims at maximizing power, income, security
and prestige.

Budget-Maximization, permanence of agencies, perceived permanence of civil service


positions, political appointees who simply serve their masters, are all examples of
“administrative self-interest”. These are contrary to the public interest. Promoting the
public interest necessarily means reduction of the pathologies currently found.

How can they be reconciled? By

-Emphasizing professional civil service

-Putting public interest over personal or political interests

-Encouraging governance instead of management

-Adopting a view toward social equity and ideas from the New Public Administration:
NPA called for a change in the public discourse about PA and a movement away from
the rational model toward increased social equity. Productivity, Marketization, Service
Orientation, Decentralization, Accountability. Sweep away programs that don’t work.

Question #5: Discuss the normative base of public administration, i.e. values
that guide administrative practice.

Normative base includes ideas of effective administration from Scientific Management,


(efficiency, economy) as well as

Values:

SVARA:

Sense of duty

Adherence to the law

200 | P a g e
Treat all equally

Pursue public interest

Embrace accountability

Avoid Conflict of Interest

Promote Ethical Practices

Frederickson adds - Equity, Fairness, justice, ethics, responsiveness, acting in the


public interest and commitment to democratic principles.

- also responsibility to support the power of the elected officials.

Svara adds that there is tension between the administrator’s sense of acting in the
public interest and the need to serve their elected political leaders.

Question #6: Explain the influence that professionalism has had on public
administrative practitioners and discuss some of the paradoxes, problems or
controversies surrounding this subject today.

Frederickson (pg. 220-223) points out that throughout history, the practice of public
administration has shifted between societies adopting models that focus on high
citizenship (as in the case of ancient Athens), to models of high professionalism (as in
the case of Egypt).

In the case of ancient Athens, where high citizenship was prized, professionalism took
a back seat as citizens were empowered to fully participate in managing the affairs at
virtually all levels of state. In the case of ancient Egypt, professionalism was highly
valued, and the role that citizens played in participating was curtailed.

Louis Brownlow and Luther Gulick were major proponents of professionalism at all
levels of government. Mosher issued a study in 1982 labeling the period 1955 to 1970
of civil service as the “professional period” in which government is run by
professionals. Raadschelders identifies the period from 1970 to present as “the
professional public administration period”. The 1978 Civil Service Reform Act listed
merit principles and established the SES (Senior Executive Service – currently 7,900
personnel). The act emphasized the importance of government by professional
managers, already called for by the Brownlow Committee report of 1937.

201 | P a g e
The growth of government in its responsibilities and the massive growth of the
population have influenced the changes in the role and position of civil servants. The
shift to more professionalism is seen as necessary to manage the massive and complex
bureaucracy and intergovernmental networks that exist today. Mosher calculates that
almost 40% of public sector positions are professionals and technicians, while only
11% of private sector positions can be labeled as such.

Self (1979) says that a professional civil servant’s most important function in modern
times may be “intellectual appraisal”. This is in contrast to a historical view of the
civil servant administrator – characterized as simply as a blind follower of their
political leader’s wishes.

Frederickson proposes a model for combining selected views of citizenship and


professionalism in which there are four scenarios: High Citizenship, Low
Professionalism (Emphasizing Civic Friendship and Technocracy), Low
Professionalism, Low Citizenship (Emphasizing Technocracy and Entrepreneurship),
High Professionalism, Low Citizenship (Emphasizing Trusteeship and
Entrepreneurship), and High Citizenship, High Professionalism (Emphasizing Civic
Friendship and Trusteeship).

The problem for the public administrator is in balancing the needs and interests of
individuals and groups on the one hand, with the community on the other. For this
reason, Frederickson says that High Administration (or High Professionalism) is
essential.

The paradox is that American democratic government is highly responsive to the


interests of organized and well-financed forces, but the needs and rights of
individuals, and particularly of poor people, are left as the special responsibility of a
benevolent public administration. Where low public administration is the norm (as is
presently the case in many American jurisdictions), public organizations lack the
capacity to make the laws fair and just. High administration alone might work if there
were a wider consensus about what professionalism means and if its definition
included obedience to codified professional standards as well as the power to rule.

202 | P a g e
Another problem with increasing professionalism (increased value placed on
specialization, degrees, special certifications, etc.), is that less emphasis is placed on
administrators adherence to the public interest and human values. Frederickson
hopes that American public administration will be able to make the move to High
Citizenship, High Professionalism – where citizens have a voice in how government is
run, and where administrators think of themselves as Trustees of the public good. In
this way, we can realize the “moral unity of society”.

Stillman (pgs 96-106) has this to say about the influence of professionalism on PA
practice: “Professionals tend to want to establish clear-cut boundaries where they can
operate relatively free from outside pressures and set policy agendas, so they can
maintain exclusive privileges. As a result, large patches of government policy-making
processes have been handed over to clusters of professional groups in various fields,
many of whom have no particular allegiance to the public or even an organization, but
float among public, private and nonprofit sectors. At times their policy turfs overlap
and can create heated controversies, such as the turf fights among the U.S. Army,
Navy, and Air Force over control of the US defense policy.”

Stillman pg. 101:

To sum up the chief methods by which professionals influence the direction of policy
and administration within the contemporary American state system:

Professionals stake out territory within a particular policy field and governmental
activity, likely spanning across various private, public and nonprofit enterprises.

Professionals apply their expertise based on length of experience and specialized


training that serves to legitimize their claims for dominance and influence within
specific policy arenas.

Professionals create “dogmas” for correct views of the world and ways of doing their
work that provide the essential ideological glue to knit together a profession and give it
distinctiveness and claims to influence in public affairs.

Professional associations, hierarchies, and elites provide a significant degree of shared


corporate identification, policy discretion, and control over their members’ activities
through the discipline they impose and the ideals they articulate.

203 | P a g e
Professionals strive continuously for freedom from external control over their activities
and value internal accountability through peer-imposed codes of ethics or licensing
processes.

Professionals aim to link up to scientific and university bodies outside government in


order to enhance their knowledge base, prestige, and status as “authoritative experts”
within public policy arenas.

Stillman warns against the emergence of a “global professional technocracy” as


dominating postwar American state governance. The implications of this are: Policy
definition by little clusters of professional experts, hyper-impermanence (lack of
adherence to a stable and immutable set of principles or concepts) and complexity
surrounding the policy arena, and the blurring of the boundaries between public and
private sectors.

Question #7: At the close of his book, Svara reasserts that “a theory of public
administration in the political process is also a theory of ethics.” Analyze or
break down this statement and, in doing so, re-state his argument.

Svara says that without ethics, PA is merely an instrument and administrators are
simply the tools of their political masters.

His argument is that government can only work if the public views the public
administrator’s authority and role as legitimate. In order for this to happen,
administrators must have a dedication to advancing the public interest as well as
promoting democracy, ethics, justice, fairness, equity while balancing the need to
serve their political leaders.

Svara adds that there is tension between the administrator’s sense of acting in the
public interest and the need to serve their elected political leaders.

Question #8: How does public administration legitimize its activities? How does
the concept of nation figure in?

According to Rohr, administrators and the higher reaches of the civil service fulfill the
Framers’ original intent for the Senate. Rohr also says that the entire career civil
service provides a remedy for a serious defect in the Constitution (the inadequate
representation).

204 | P a g e
According to the Blacksburg Manifesto, the public administration is legitimized by the
Constitution’s plans for the Senate as well as the Founders’ view of public service as
both a calling and a “trusteeship”.

The public views it as a major social asset because of its experience, managerial skills
and commitment to advancing the public interest.

Wamsley says that PA legitimizes its activities by viewing its role as a “citizen-agent” –
standing in place of other citizens, exerting power on their behalf to achieve a
collective purpose.

How does the concept of nation figure in?

Raad defines “nation” as “a large community of people living together under one
government whose actions are considered to be representative of all.”

If we consider public administration as an extension of the Senate, solving the defect


of inadequate representation, then by legitimizing PA, we are ensuring that the
government’s actions are “representative of all”.

Question #9: Compare four of the various models of public administration and
the underlying assumptions of each.

Peters four models + Traditional Govt.

Market model: The central assumptions of the market model are:

 Monopoly is the principal source of traditional government;


 Structurally, decentralized government is preferable;
 Management can be improved through pay-for-performance and other private-
sector techniques and incentives;
 Policy making is accomplished through internal markets and market incentives;
and
 This model's primary public benefit is less costly, less intrusive government.

Participative model: Although the participative model shares the market model's view
that traditional bureaucracies impede good government, it "is almost the ideological
antithesis of the market approach". Its advocates would remove or minimize
hierarchical layers of top-down controls, opting instead to empower employees and

205 | P a g e
increase their involvement in decisions. The participative government model rests on
the popular assumption that "workers and clients closest to the actual production of
goods and services in the public sector have the greatest insight and information
about the programs". The central assumptions of the participative government model
are these:

 The principal source of traditional government's problems is hierarchy (not


monopoly, as proponents of the market model argue);
 Structurally, flatter government organizations are preferable;
 Management can be improved through greater employee involvement, for
example TQM and other team-based models;
 Policy making is accomplished through consultation and negotiation; and
 The model's primary public benefit is greater involvement.

Flexible model: Flexible government," the least clearly articulated of the four models, is
less ideological in its approach. "Flexibility" refers to "the capacity of government and
its agencies to make appropriate policy responses to environmental changes rather
than merely responding in habitual ways to inherently novel challenges". The flexible
government model's central assumptions are:

 Permanence is the principal source of problems with the traditional model of


government;
 Structurally, "virtual organizations" and networks are preferable;
 Management is improved by utilizing temporary personnel;
 Policy making is accomplished through experimentation; and
 The model's primary public benefit is low cost with coordination.

Deregulated model: In this model, "deregulation" refers to government, not industry.


Its central theme is the removal of rules, policies, and other constraints on
government action. Public managers should be freed to be entrepreneurs. For
example, this model's advocates endorse the 1991-92 congressional policy that
permitted the Department of Defense's Maintenance Centers to compete with private
industry for maintenance work, often successfully. Proponents of the deregulated
government model argue that deregulation is as important in the public sector as in

206 | P a g e
the private sector, and for the same reason: to liberate workers' entrepreneurial
energies.

Government deregulation would produce a lean, resolute civil service able to decide
and act, rather than wait and see. The model's central assumptions and
characteristics are:

 Internal regulation, such as restrictive policies and rules, is the principal source
of traditional government's
 problems;
 No particular structural arrangement is to be preferred;
 Management is improved through greater freedom;
 Policy making is accomplished through entrepreneurial government; and
 The model's primary public benefits are creativity and activism.

Stillman:

No-State (Negative State): Minimalist public administration.

Natural adjustments in the marketplace via competition and little state oversight is
best.

Sees work of administrators as low-level, technical work.

Bold-State (Positive State): Activist public administration.

the State has ability to do much good and nat govt can lead in positive planning.

Focus on organizational effectiveness, limit political intrusion on decision making and


maximize the community good and public welfare.

Pre-State (Half-way State):

 Sees PA as part of the Constitutional system, but includes a holistic approach.


 Big Picture of the issues, but no clear-cut answers to modern challenges.

Pro-State (Professional Global Technocracy):

 Higher education is key


 Govt run by groups of professionals
 Ultimate faith in scientific management and technology

207 | P a g e
 Blurring of boundaries of public and private sectors

Compared to the traditional or orthodox model, which poses the greatest promise for
the future of effectiveness in government?

If I had to choose one, I would choose Flexible Government. There is more freedom to
close down programs and agencies that are not working, while experimentation is
encouraged to find better ways to manage.

Ideally, I feel the greatest promise would be a mixture of Market Government and
Flexible Government.

In Stillman’s model – Bold-State!

the State has ability to do much good and nat govt can lead in positive planning.

Focus on organizational effectiveness, limit political intrusion on decision making and


maximize the community good and public welfare.

QUESTION 10– In attempting to define the field of public administration several


authors have commented that American PA has its origin in the progressive era
and the reform of government and/or society. The field, according to these
authors, has never strayed from these roots. If accurate, this scenario has its
limitations. Discuss how this description is or is not true today as well as
broader implications for the future of the field.

Actually, most mainstream authors accept that PA has it origin in the Progressive Era,
generally regarded as 1890 to 1920. Specifically, Woodrow Wilson’s publication of The
Study of Administration in 1887 has been reported by many prominent authors as the
beginning of public administration.

(A Little Note of the Progressive Era)

The Progressive Era was a period of reform that began in America’s urban regions
from, approximately the 1890s to the 1920s. Progressivists sought change in labor
and fiscal policies in different levels of government. The reformers were predominantly
members of the middle class. Furthermore, women came to the fore in the Progressive
era and proved their value as good workers. Although the Progressives pushed for
social justice and general equality, there was extreme diversity and contradiction

208 | P a g e
within the Progressive movement. Many reforms dotted this era, including Prohibition
with the 18th Amendment and woman’s suffrage through the Nineteenth Amendment,
both in 1920 as well as the initiation of the Income Tax with the Sixteenth Amendment
and direct election of Senators with the Seventeenth Amendment (both in 1913). The
four original goals of Progressivism were 1) protecting social welfare - YMCA, 2)
promoting moral improvement – prohibition of alcohol, 3) creating economic reform –
change of individual behavior, 4) fostering efficiency – Tayorism. President’s Theodore
Roosevelt, William H. Taft, and Woodrow Wilson served during this era.

A good number of authors would agree that PA had its origins during the Progressive
Era. This was an era characterized by attempts to accommodate or balance a life in
an increasingly urban, industrial, routinized, and multiethnic environment. The
methods for these attempts was imperialist expansion of the American West,
conservation movements, organizing labor, increasing business regulation, and
remaking the political process. Fredrick Taylor, Willoughby, and others had
established the “scientific period of public administration”. This was during the
“Classical” period of our discipline; a field dominated by Positivism theories of
epistemology. The fact that many authors contend that we have not progressed
beyond these roots is not overly surprising as many would have us retain positivist
beliefs and to a lesser degree, scientific management principles. However, for any
author to suggest our existence is continuing in an era nearly a hundred years past
would be purporting that in spite of vast technological, cultural, societal, and political
advancements we haven’t progressed is ludicrous, or is it?

Paul Light has studied reform efforts over nearly a 60 year period (1945 to 2002) and
suggests that very few of the reform efforts have had any impact on government. The
bureaucratic and hierarchical government organizations that existed in the
Progressive Era continue to exist today suggesting little change over the years in the
area of reform or organizational structure.

209 | P a g e
Chapter 2
Administrative
Management &
Administrative Law

MPA Comprehensive Exam


Study Guide
2.1 Keynesian Economics
Keynesian economics, also Keynesianism and Keynesian Theory, is an economic
theory based on the ideas of twentieth-century British economist John Maynard
Keynes. Keynesian economics promotes a mixed economy where both the state and
the private sector have important roles. Keynesian economics seeks to provide
solutions to the failures of laissez-faire economic liberalism, which advocates that
markets and the private sector operate best without state intervention.

In Keynes' theory, macroeconomic trends can overwhelm the micro-level behavior of


individuals. Instead of the economic process being based on continuous improvement
in potential output, as most classical economists had believed from the late 1700s on,
Keynes asserted the importance of aggregate demand for goods as the driving factor of
the economy, especially in periods of downturn. From this he argued that government
policies could be used to promote demand at a macro level, to fight high
unemployment and deflation of the sort seen during the 1930s. Keynes argued that
the solution to depression was to stimulate the economy ("inducement to invest")
through some combination of two approaches:

a reduction in interest rates.

Government investment in infrastructure - the injection of income results in more


spending in the general economy, which in turn stimulates more production and
investment involving still more income and spending and so forth. The initial
stimulation starts a cascade of events, whose total increase in economic activity is a
multiple of the original investment.

A central conclusion of Keynesian economics is that there is no strong automatic


tendency for output and employment to move toward full employment levels. This
conclusion conflicts with the tenets of classical economics, and those schools, such as
supply-side economics or the Austrian School, which assume a general tendency
towards a welcome equilibrium in a restrained money-creating economy. In the
'neoclassical synthesis', which combines Keynesian macro concepts with a micro

211 | P a g e
foundation, the conditions of General equilibrium allow for price adjustment to achieve
this goal.

More broadly, Keynes saw his as a general theory, in which utilization of resources
could be high or low, whereas previous economics focused on the particular case of
full utilization.
Keynesian economics by Alan S. Blinder1

Keynesian economics is a theory of total spending in the economy (called aggregate


demand) and of its effects on output and inflation. Although the term is used (and
abused) to describe many things, six principal tenets seem central to Keynesianism.
The first three describe how the economy works.

1. A Keynesian believes that aggregate demand is influenced by a host of


economic decisions—both public and private—and sometimes behaves
erratically. The public decisions include, most prominently, those on monetary
and fiscal (i.e., spending and tax) policy. Some decades ago, economists
heatedly debated the relative strengths of monetary and fiscal policy, with some
Keynesians arguing that monetary policy is powerless, and some monetarists
arguing that fiscal policy is powerless. Both of these are essentially dead issues
today. Nearly all Keynesians and monetarists now believe that both fiscal and
monetary policy affect aggregate demand. A few economists, however, believe in
what is called debt neutrality—the doctrine that substitutions of government
borrowing for taxes have no effects on total demand (more on this below).

2. According to Keynesian theory, changes in aggregate demand, whether


anticipated or unanticipated, have their greatest short-run impact on real
output and employment, not on prices. This idea is portrayed, for example, in
Phillips curves that show inflation changing only slowly when unemployment
changes. Keynesians believe the short run lasts long enough to matter. They

1
Alan S. Blinder is the Gordon S. Rentschler Memorial Professor of Economics at Princeton University. He was previously vice-
chairman of the Federal Reserve's Board of Governors, and before that was a member of President Clinton's Council of Economic
Advisers.

212 | P a g e
often quote Keynes's famous statement "In the long run, we are all dead" to
make the point.

Anticipated monetary policy (that is, policies that people expect in advance) can
produce real effects on output and employment only if some prices are rigid—if
nominal wages (wages in dollars, not in real purchasing power), for example, do
not adjust instantly. Otherwise, an injection of new money would change all
prices by the same percentage. So Keynesian models generally either assume or
try to explain rigid prices or wages. Rationalizing rigid prices is hard to do
because, according to standard microeconomic theory, real supplies and
demands do not change if all nominal prices rise or fall proportionally.

But Keynesians believe that, because prices are somewhat rigid, fluctuations in
any component of spending—consumption, investment, or government
expenditures—cause output to fluctuate. If government spending increases, for
example, and all other components of spending remain constant, then output
will increase. Keynesian models of economic activity also include a so-called
multiplier effect. That is, output increases by a multiple of the original change
in spending that caused it. Thus, a $10 billion increase in government spending
could cause total output to rise by $15 billion (a multiplier of 1.5) or by $5
billion (a multiplier of 0.5). Contrary to what many people believe, Keynesian
analysis does not require that the multiplier exceed 1.0. For Keynesian
economics to work, however, the multiplier must be greater than zero.

3. Keynesians believe that prices and, especially, wages respond slowly to


changes in supply and demand, resulting in shortages and surpluses,
especially of labor. Even though monetarists are more confident than
Keynesians in the ability of markets to adjust to changes in supply and
demand, many monetarists accept the Keynesian position on this matter.
Milton Friedman, for example, the most prominent monetarist, has written:
"Under any conceivable institutional arrangements, and certainly under those
that now prevail in the United States, there is only a limited amount of

213 | P a g e
flexibility in prices and wages." In current parlance, that would certainly be
called a Keynesian position.

No policy prescriptions follow from these three beliefs alone. And many economists
who do not call themselves Keynesian—including most monetarists—would,
nevertheless, accept the entire list. What distinguishes Keynesians from other
economists is their belief in the following three tenets about economic policy.

Keynesians do not think that the typical level of unemployment is ideal—partly


because unemployment is subject to the caprice of aggregate demand, and partly
because they believe that prices adjust only gradually. In fact, Keynesians typically see
unemployment as both too high on average and too variable, although they know that
rigorous theoretical justification for these positions is hard to come by. Keynesians
also feel certain that periods of recession or depression are economic maladies, not
efficient market responses to unattractive opportunities. (Monetarists, as already
noted, have a deeper belief in the invisible hand.)

Many, but not all, Keynesians advocate activist stabilization policy to reduce the
amplitude of the business cycle, which they rank among the most important of all
economic problems. Here Keynesians and monetarists (and even some conservative
Keynesians) part company by doubting either the efficacy of stabilization policy or the
wisdom of attempting it.

This does not mean that Keynesians advocate what used to be called fine-tuning—
adjusting government spending, taxes, and the money supply every few months to
keep the economy at full employment. Almost all economists, including most
Keynesians, now believe that the government simply cannot know enough soon
enough to fine-tune successfully. Three lags make it unlikely that fine-tuning will
work. First, there is a lag between the time that a change in policy is required and the
time that the government recognizes this. Second, there is a lag between when the
government recognizes that a change in policy is required and when it takes action. In
the United States, this lag is often very long for fiscal policy because Congress and the
administration must first agree on most changes in spending and taxes. The third lag
comes between the time that policy is changed and when the changes affect the
214 | P a g e
economy. This, too, can be many months. Yet many Keynesians still believe that more
modest goals for stabilization policy—coarse-tuning, if you will—are not only
defensible, but sensible. For example, an economist need not have detailed
quantitative knowledge of lags to prescribe a dose of expansionary monetary policy
when the unemployment rate is 10 percent or more—as it was in many leading
industrial countries in the eighties.

Finally, and even less unanimously, many Keynesians are more concerned about
combating unemployment than about conquering inflation. They have concluded from
the evidence that the costs of low inflation are small. However, there are plenty of anti-
inflation Keynesians. Most of the world's current and past central bankers, for
example, merit this title whether they like it or not. Needless to say, views on the
relative importance of unemployment and inflation heavily influence the policy advice
that economists give and that policymakers accept. Keynesians typically advocate
more aggressively expansionist policies than non-Keynesians.

Keynesians' belief in aggressive government action to stabilize the economy is based


on value judgments and on the beliefs that (a) macroeconomic fluctuations
significantly reduce economic well-being, (b) the government is knowledgeable and
capable enough to improve upon the free market, and (c) unemployment is a more
important problem than inflation.

The long, and to some extent, continuing battle between Keynesians and monetarists
has been fought primarily over (b) and (c).

In contrast, the briefer and more recent debate between Keynesians and new classical
economists has been fought primarily over (a) and over the first three tenets of
Keynesianism—tenets that the monetarists had accepted. New classicals believe that
anticipated changes in the money supply do not affect real output; that markets, even
the labor market, adjust quickly to eliminate shortages and surpluses; and that
business cycles may be efficient. For reasons that will be made clear below, I believe
that the "objective" scientific evidence on these matters points strongly in the
Keynesian direction.

215 | P a g e
Before leaving the realm of definition, however, I must underscore several glaring and
intentional omissions.

First, I have said nothing about the rational expectations school of thought (see
Rational Expectations). Like Keynes himself, many Keynesians doubt that school's
view that people use all available information to form their expectations about
economic policy. Other Keynesians accept the view. But when it comes to the large
issues with which I have concerned myself, nothing much rides on whether or not
expectations are rational. Rational expectations do not, for example, preclude rigid
prices. Stanford's John Taylor and MIT's Stanley Fischer have constructed rational
expectations models with sticky prices that are thoroughly Keynesian by my definition.
I should note, though, that some new classicals see rational expectations as much
more fundamental to the debate.

The second omission is the hypothesis that there is a "natural rate" of unemployment
in the long run. Prior to 1970, Keynesians believed that the long-run level of
unemployment depended on government policy, and that the government could
achieve a low unemployment rate by accepting a high but steady rate of inflation. In
the late sixties Milton Friedman, a monetarist, and Columbia's Edmund Phelps, a
Keynesian, rejected the idea of such a long-run trade-off on theoretical grounds. They
argued that the only way the government could keep unemployment below what they
called the "natural rate" was with macroeconomic policies that would continuously
drive inflation higher and higher. In the long run, they argued, the unemployment rate
could not be below the natural rate. Shortly thereafter, Keynesians like Northwestern's
Robert Gordon presented empirical evidence for Friedman's and Phelps's view. Since
about 1972 Keynesians have integrated the "natural rate" of unemployment into their
thinking. So the natural rate hypothesis played essentially no role in the intellectual
ferment of the 1975-85 period.

Third, I have ignored the choice between monetary and fiscal policy as the preferred
instrument of stabilization policy. Economists differ about this and occasionally
change sides. By my definition, however, it is perfectly possible to be a Keynesian and

216 | P a g e
still believe either that responsibility for stabilization policy should, in principle, be
ceded to the monetary authority or that it is, in practice, so ceded.

Keynesian theory was much denigrated in academic circles from the mid-seventies
until the mid-eighties. It has staged a strong comeback since then, however. The main
reason appears to be that Keynesian economics was better able to explain the
economic events of the seventies and eighties than its principal intellectual competitor,
new classical economics.

True to its classical roots, new classical theory emphasizes the ability of a market
economy to cure recessions by downward adjustments in wages and prices. The new
classical economists of the mid-seventies attributed economic downturns to people's
misperceptions about what was happening to relative prices (such as real wages).
Misperceptions would arise, they argued, if people did not know the current price level
or inflation rate. But such misperceptions should be fleeting and surely cannot be
large in societies in which price indexes are published monthly and the typical
monthly inflation rate is under 1 percent. Therefore, economic downturns, by the new
classical view, should be mild and brief. Yet during the eighties most of the world's
industrial economies endured deep and long recessions. Keynesian economics may be
theoretically untidy, but it certainly is a theory that predicts periods of persistent,
involuntary unemployment.

According to new classical theory, a correctly perceived decrease in the growth of the
money supply should have only small effects, if any, on real output. Yet when the
Federal Reserve and the Bank of England announced that monetary policy would be
tightened to fight inflation, and then made good on their promises, severe recessions
followed in each country. New classicals might claim that the tightening was
unanticipated (because people did not believe what the monetary authorities said).
Perhaps it was in part. But surely the broad contours of the restrictive policies were
anticipated, or at least correctly perceived as they unfolded. Old-fashioned Keynesian
theory, which says that any monetary restriction is contractionary because firms and
individuals are locked into fixed-price contracts, not inflation-adjusted ones, seems
more consistent with actual events.

217 | P a g e
An offshoot of new classical theory formulated by Harvard's Robert Barro is the idea of
debt neutrality. Barro argues that inflation, unemployment, real GNP, and real
national saving should not be affected by whether the government finances its
spending with high taxes and low deficits or with low taxes and high deficits. Because
people are rational, he argues, they will correctly perceive that low taxes and high
deficits today must mean higher future taxes for them and their heirs. They will, Barro
argues, cut consumption and increase their saving by one dollar for each dollar
increase in future tax liabilities. Thus, a rise in private saving should offset any
increase in the government's deficit. Naïve Keynesian analysis, by contrast, sees an
increased deficit, with government spending held constant, as an increase in aggregate
demand. If, as happened in the United States, the stimulus to demand is nullified by
contractionary monetary policy, real interest rates should rise strongly. There is no
reason, in the Keynesian view, to expect the private saving rate to rise.

The massive U.S. tax cuts between 1981 and 1984 provided something approximating
a laboratory test of these alternative views. What happened? The private saving rate
did not rise. Real interest rates soared, even though a surprisingly large part of the
shock was absorbed by exchange rates rather than by interest rates. With fiscal
stimulus offset by monetary contraction, real GNP growth was approximately
unaffected; it grew at about the same rate as it had in the recent past. Again, this all
seems more consistent with Keynesian than with new classical theory.

Finally, there was the European depression of the eighties, which was the worst since
the depression of the thirties. The Keynesian explanation is straightforward.
Governments, led by the British and German central banks, decided to fight inflation
with highly restrictive monetary and fiscal policies. The anti-inflation crusade was
strengthened by the European Monetary System, which, in effect, spread the stern
German monetary policy all over Europe. The new classical school has no comparable
explanation. New classicals, and conservative economists in general, argue that
European governments interfere more heavily in labor markets (with high
unemployment benefits, for example, and restrictions on firing workers). But most of
these interferences were in place in the early seventies, when unemployment was
extremely low.

218 | P a g e
Further Reading

Blinder, Alan S. Hard Heads, Soft Hearts. Chaps. 2, 3. 1987.

Blinder, Alan S. "Keynes after Lucas." Eastern Economic Journal (July-September


1986): 209-16.

Blinder, Alan S. "Keynes, Lucas, and Scientific Progress." American Economic Review
(May 1987): 130-36. (Reprinted in John Maynard Keynes (1833-1946), vol. 2, edited by
Mark Blaug. 1991.)

Gordon, Robert J. "What Is New-Keynesian Economics?" Journal of Economic Literature


28, no. 3 (September 1990): 1115-71.

Keynes, John Maynard. The General Theory of Employment, Interest, and Money. 1936.

Mankiw, N. Gregory. "A Quick Refresher Course in Macroeconomics." Journal of


Economic Literature 28 (December 1990): 1645-60.

219 | P a g e
2.2 Risk Management as a Strategic Posture: Public vs. Private Approaches
Since the events of September 11, 2001, the way governments manage risk
(especially in the United States) is changing significantly. In addition to
unforeseen events such as terrorist attacks, risk events of all types seem to
be occurring at an unprecedented pace during the infancy of our 21st century.
The risks we face range and affect the entire span of human activity - from
natural disasters such as Hurricane Katrina and earthquakes/tsunamis in
Indonesia - to deaths resulting from unregulated imports of tainted food and
poisonous products from China. As a result of this increased risk
environment, important questions are being asked about the proper role of
government in mitigating and responding to risks of all types. This paper
reviews the recent literature detailing past and present approaches for risk
management in both the private and public sectors, outlines several
suggested strategies for managing risk in the future, and concludes with an
analysis of recommendations for changes in the way that governments
manage risk.

The traditional ways in which governments and businesses managed risk centered on
either purchasing various types of insurance or simply bearing the risk without any
sort of protection. For those organizations exposed to risks that preclude the
availability of commercially-available insurance coverage, such as emergency
management, taxpayers inevitably end up footing the bill for any given response.
When disasters and other risk events occur, local, state and sometimes federal
budgets have to be adjusted in order to “find” the fiscal resources needed for a proper
response. This necessarily means that various previously-authorized programs are
cut or have their appropriations reduced.

While the need for improved risk management methods has increased, the availability
of sound, rational models for managing risk has also increased. As supported by
Werner Pfenningstorf (1977), it seems that the tools we require have always been
there, but implementation and standardization are problematic. “The available
information shows that many governments, especially small municipalities, still have
haphazard, fragmented, and inconsistent risk management policies; many rely

220 | P a g e
exclusively on the advice and the services of local insurance agents, who may not be
equipped or inclined to select the best available coverage.” (Pfenningstorf, 1977).

Although commercially-available insurance is an important piece to any risk


management strategy, it can be argued that the risk environment has changed
significantly over the past 50 years, and that a new comprehensive and more flexible
approach is needed to address the risks inherent to governments and to populations.

Risk Culture:

Bozeman and Kingsley (1998) feel that the risk management approach of any given
organization is directly related to the “risk culture” that pervades that particular
organization and its management philosophy. Bozeman and Kingsley performed a
survey of public and private organizations to measure “risk culture”. The concept
pertains to managers’ perceptions regarding their co-workers and superiors
willingness to take risks and promote risk-taking. Their findings:

“A riskier culture is positively related to the willingness of top


managers to trust employees and to the clarity of organizations’ missions.
Organizations with more red tape, weak links between promotion and
performance, and high involvement with elected officials tend to have a
less risky culture.” (Bozeman and Kingsley, 1998)

Review of Risk Management Approaches:

A literature review of risk management approaches reveals a wide array of public and
private methods that span the range from scientific-management-style programs to
community and relationship-based networks. Over the past 20 years or so, many new
approaches to managing organizational risk have been employed in both the U.S. and
abroad, with varying degrees of success. Outlined below are arguments based on
some of the most common methods found in the literature.

Budgeting as a Strategic Tool for Risk Management:

One major risk facing state and local governments involves the economic variability
associated with the business cycle. Dealing successfully with cyclic variability is

221 | P a g e
important for government officials and political leaders. Economic downturns place
greater demands on local governments, who are far less equipped to handle the effects
than agencies and officials operating at the federal level. How do budget officials
attempt to manage this source of risk? According to Michael Wolkoff (1999), four
different generic strategies are available to budget officials: they can budget
pessimistically for the period (the most conservative approach presuming the least
favorable economic circumstances for the budget period), they can make use of
savings devices (rainy day funds), they can practice financial ledgerdemain (short-run
accounting tricks that shift expenditures to later periods), or they can implement
contingency plans (pre-agreed upon actions that are triggered by economic events, i.e.
taxation and spending policies that depend on defined funding levels).

In attempting to adapt to unexpected budgetary shocks, Wolkoff notes: “The types of


actions budget officials may take will depend on the timing and magnitude of the
unexpected shock, the political environment at that time, and the tools available.”
(Wolkoff, 1999)

Risk Management in the European Union: The “Precautionary Principle”.

The United States and the European Union countries differ substantially in their
perceptions and tolerance for risks of all types. Americans seem far more willing to
employ innovations that pose serious environmental or health risks, such as
engineering genetically modified crops and organisms or infusing meats with carbon
dioxide and dangerous preservatives so they appear “fresh” longer. “The French and
many other Europeans tend to be more prudent, evaluating new technologies based on
the ‘precautionary principle’, which echoes the old adage of medicine: First, do no
harm.” (Tama, 2003) Since the U.S. has a “window dressing only” policy of regulation
and enforcement, foods and products that are refused entry into the EU are routinely
re-routed for easy sale to the United States. Recent examples include mercury-filled
shrimp from Vietnam, toys from China with poisonous plastics and lead-based paints,
cough syrup from China found to contain anti-freeze, and toothpaste from China
found to contain several poisonous substances. Thanks to the “precautionary
principle”, which is encoded into EU law through the Maastricht Treaty, and which
forms the basis for all European environmental policies, the EU seems to be far ahead

222 | P a g e
of the United States when it comes to risk management and protecting their citizens
from unscrupulous corporations whose main concern is profit, and not the public
interest.

Risk Analysis: Rational, Scientific Approaches to Risk Management

The Case of British Rail:

Computer-based simulations are established and effective aids to decision-making and


are widely used in all types of business. The risk analysis procedures used by British
Rail (BRB) involve the use of fractile analysis and Monte Carlo simulation. They were
first used in the BRB’s 1983 Corporate Plan, and because of its perceived success, is
now incorporated as a permanent feature of its corporate planning process. (Harris
and Williams, 1985) The planning process used by BRB incorporates both bottom-up
and top-down budget estimates, and incorporates several alternative scenarios and
contingency plans which can be triggered as deemed necessary. The massive amounts
of data from these plans are aggregated and risk profiles are associated with each
alternative estimate (representing the “best” and “worst” results), then the data is fed
into a simulation model. The method is known as the Continuous Distribution
Fractile Method, to give its full title, and its purpose is to establish key points for a
distribution that includes the range, median and quartiles. These are established first
for a cumulative probability curve which can then be transcribed into a probability
density curve.” (Harris and Williams, 1985) In attempting to use the simulation to
evaluate possible future financial outcomes, we then take selected data from the
fractile analysis and apply it into a Monte Carlo Simulation. “This method involves
taking a sample from each of the distribution curves and combines them to create a
set of possible financial outcomes, which represents what might happen in certain
circumstances.” (Harris and Williams, 1985) The major benefit of using this type of
process, is that when and if “shocks” occur, management can readily see the range of
financial outcomes are likely to result given a certain action, and can be proactive in
applying contingent actions to keep the organization financially on-track with regard
to the longer term.

Rethinking Risk Management in the Federal Government:

223 | P a g e
Robin Cantor (1996) is another proponent calling for a more rational, scientific
standard for managing risk at the federal level. Cantor notes that there is much
discontent with the current methods being employed in risk management, and that
there is need for radical reform. He suggests also that there needs to be an
intellectual shift in the way that managers think about risk and respond to risks.
Agencies such as the Department of Energy (DOE) and the Environmental Protection
Agency (EPA) are already using sophisticated risk analysis and risk management
systems, and Cantor feels that this will help in the push toward a uniform adoption of
risk analysis and rational processes for risk management across the federal
landscape. Cantor describes the origins of this shift in risk policy:

“The Clinton administration’s efforts on risk management have


tended to emphasize the role of risk analysis both in setting agency
priorities and in rule making. The Executive Order on Regulatory Planning
and Review (No. 12866) reinforced attention to risk-based priority setting
and the balancing of risk reduction and costs in the broader context of
regulatory decision making. This order also established a Regulatory
Working Group, which launched a concerted effort across federal agencies
to develop principles for risk assessment, risk management and risk
communication.” (Cantor, 1996)

Legal Issues related to Risk Management Approaches:

There is no disagreement on the fact that controlling the costs associated with
government operations is a challenging exercise. Strategic planning and budgeting
efforts should incorporate the risks associated with court judgments and exposure to
lawsuits brought by parties in the organization’s operating environment. It can be
argued that the more decentralized the particular government or agency is with regard
to its particular service delivery systems, the more exposed they are to potential legal
actions. Additionally, the nature of the business of an organization is related to its
degree of exposure to legal challenges and the risks associated with them. As noted by
Pfenningstorf, the federal government has always had a policy of non-insurance
(Pfenningstorf, 1977). As such, when unexpected court rulings force agencies or local
government units to pay damages, or when courts issue remedial orders that require

224 | P a g e
significant outlays, these government units must re-analyze their current plans and
re-work their budgets to accommodate these costs. There are a number of ways in
which court decisions force public managers to consider financial risk and exposure.
These unexpected costs can cause already-authorized programs to be curtailed or even
cancelled.

Risk Management and the Public-Private Issue:

In recent years, one hot topic that has generated heated debate in government circles
is the question of privatization. According to Phillip Cooper (1999), “In the
contemporary environment, issues of risk are often intertwined with other complex
problems, particularly privatization and intergovernmental relations.” What is usually
meant by privatization is the contracting-out or wholesale transference of government
service delivery in certain areas to private or non-profit firms operating under
contracts with the government. The key problems associated with privatization are
oversight and accountability.

One contentious example of the risks involved in privatization of government services


is that of correctional facilities operated by for-profit private corporations. In his 1999
article, “Courts and Fiscal Decision Making”, Phillip Cooper reviews a Tennessee case
involving the Corrections Corporation of America. In Richardson v. McNight, 1997, a
prisoner in a facility operated by the corrections Corporation of America (CCA) sued
the state, charging two guards with physical abuse that violated his constitutional
rights. Cooper notes that in normal state-run prisons, guards are protected under
immunity. CCA used this fact to claim immunity for its guards. However, as this case
proceeded to the Supreme Court, Justice Breyer argued that private sector employees
work under a different system of accountability and incentives, and they are required
to have sufficient insurance coverage to address potential liabilities such as this one.
It is clear from this case that the costs of legal services and potential remedies make
up an important part of the risk calculations used by public administrators (Cooper,
1999).

Terrorism, Homeland Security and Emergency Management:

225 | P a g e
Managing the risks associated with terrorism, homeland security and emergency
management are arguably the most challenging of the risks attempting to be managed
by U.S. government and international officials. Unfortunately, in the name of
attempting to manage these risks, our current U.S. government (G.W. Bush
administration) has viewed it as necessary to take severe and outwardly-aggressive
actions that negate the rights of individual citizens as well as those of sovereign
foreign governments in order to succeed in their current “war on terrorism”.

In addition to suspending the civil rights of individual citizens, large-scale surveillance


of the domestic population is currently taking place, involving agencies at the federal,
state and local levels. This unprecedented attack on individual freedoms domestically,
along with the incredible offenses that have been leveled against other sovereign
nations, is arguably causing the opposite effect of the one originally intended (reducing
risk) by those in charge of authorizing these operations. In addition to conducting
overtly illegal operations, the federal government has also adopted quasi-legal methods
of attempting to reduce the risks associated with terrorism and homeland security.
Consider the case of InfraGard, an FBI-led organization started in 1996, which is now
a 23,000 member-strong domestic surveillance network in the United States. This
rapidly growing group enlists business owners, banking executives, non-profit
organizations, neighborhood watch groups and other hand-selected citizens as
“deputized FBI agents with limited powers” (Rothschild, 2008).

InfraGard is essentially a partnership between the FBI and private-sector business


and citizen-members that is overseen by local FBI offices as the first line of defense
against terrorist attacks to vital infrastructure.

“Members receive secret warnings of terrorist threats before the public does –
and at least on one documented occasion, before elected officials. In return,
they provide information to the government, which understandably alarms the
ACLU. One business executive, who showed me his InfraGard card, told me
that they have permission to “shoot to kill” in the event of martial law.”
(Rothschild, 2008). InfraGard members are told to contact the FBI if they “note
suspicious activity or an unusual event.” (Rothschild, 2008)

226 | P a g e
Another highly-questionable practice used by the current administration is the illegal
“data-mining” and surveillance of domestic and foreign telephone calls, emails and
internet surfing habits of private citizens. This also is done in the name of risk
management, and has been challenged in the courts, though unsuccessfully –
arguably because of the current administration’s vast judicial political appointments,
resulting in “ideological ownership” of judges strategically positioned in the same
courts where these cases are arranged to be heard.

Emergency Management:

The demand for more effective and efficient government service delivery during and
after disasters has increased, especially in the wake of botched federal responses to
emergencies in the recent past. In terms of preparedness and risk management
practices, the emergency management communities at the state and local levels are far
ahead of their federal counterparts. After the creation of the Department of Homeland
Security (DHS) in November, 2002, billions of dollars were sent out to cities and
counties to make capital purchases that could enhance local responses to emergency
events in the future.

However, as Vicki Wilson notes, “Deciding what to fund and what not to fund before a
disaster occurs is a fine line that finance officers and elected officials must walk.”
(Wilson, 2007) Partly due to the experiences of September 11th and Hurricane
Katrina, local governments and first responders realized that they needed to
coordinate and establish relationships with organizations that may be needed in the
response efforts to various emergencies. Unlike the grass-roots efforts of InfraGard,
emergency management officials at the state and local level have now established
legitimate, and efficient networks of organizations and individuals that may be called
upon in times of emergency. Efforts such as mutual-aid agreements between states
and localities; agreements with private sector companies that could support
emergency operations; agreements with national suppliers such as Grainger, Home
Depot, and Wal-Mart; and liaison relationships with support agencies like the
Salvation Army and Red Cross, all form a cohesive network of professionals who can

227 | P a g e
increase the success of state and federal responses to emergencies. Another popular
risk management practice used the emergency management field is the establishment
of “private sector consortium groups. Building managers and owners, utility and other
infrastructure companies, healthcare organizations, hotels, as well as universities and
colleges, can play an important role in disaster management. Having previously
established relationships, contacts and agreements with these industry segments can
make life a lot easier during a disaster.” (Wilson, 2007) In addition to these efforts,
internal procurement policies can be streamlined, so that during an emergency, public
safety departments can acquire the items they need immediately, bypassing the time-
consuming regular purchasing procedures. These new approaches to managing risks
lead us directly into a discussion of the possibilities for managing risk in the future.

A More Comprehensive and Flexible Risk Management Approach:

In his 2005 article, “Governance and risk management: challenges and public
productivity”, Arie Halachmi calls for a shift from “governing to governance”, in which
governments engage private industry and the multitudes of civil society based
organizations to help elected officials address accountability issues, improve
productivity, and provide monitoring of their environments to improve government
response and risk management. Halachmi’s suggested approach to risk management
is in line with the emergency management methods outlined above by Wilson.
Halachmi’s 2005 paper recounted a recent case in which extreme management
challenges resulted from government network oversight failures during the Great
Blackout of August, 14, 2003, occurring in the Northeastern United States. Oversight
definitely becomes a complicated issue when government “devolves” responsibility and
distributes power to networks of nongovernmental entities. It can be argued that
extreme events like this cannot be guarded against because there are too many areas
under different commands, making oversight and risk management impossible.
However, according to Halachmi, increasing decentralization and devolution is a very
effective way of managing risk. It involves a paradigm shift from “governing to
governance”.

“…’governance’ is now used to depict an effort to meet the welfare needs of


citizens in a better way through partnerships with other elements of the ‘civil

228 | P a g e
society’ for the purpose of overcoming limits on action due to governmental
structures, legal issues, or administrative procedures. While some of these
partnerships are explicit (official designation of authority delegated to a
nongovernmental organization), others are implicit (examples in the U.S. include
church-based programs for adoption and placement of children or the
sponsorship and settlement of refugees). (Halachmi, 2005)

Halachmi argues that by enlisting the help of civil based society organizations and
private-sector companies, threat and risk information is more effectively and efficiently
gathered and transmitted to authorities. By incorporating street-level organizations
and companies into the management chain of government service delivery,
government can take best advantage of local-level knowledge and ideas for addressing
risks. The network of small and large organizations is far more efficient in monitoring
the environment, which translates into improved government response and overall risk
management.

Conclusion:

Bozeman and Kingsley feel that the risk management approach of any given
organization is directly related to the “risk culture” that pervades that particular
organization and its management philosophy. They argue that when the environment
is characterized by red tape, a weak relationship between promotion and performance,
and a high level of involvement with elected officials, this translates into a “less risky”
culture. Wolkoff supports the use of budgetary devices as vital risk management tools
for the public administrator. Tama argues that risks can be avoided by adopting
similar policies as the European Union’s “precautionary principle”, which is applied
universally (as a result of codification in the Maastricht Treaty) to all policies being
considered in the EU. Harris and Williams, along with Robin Cantor, are proponents
for the adoption of rational scientific approaches to risk management such as the Risk
Analysis simulation methods used by British Rail and several executive agencies
within the U.S. federal government, including DOE and EPA. Phillip Cooper finds that
legal issues in the form of court judgments and exposure to lawsuits represent
significant risks and are often intertwined with other complex problems, particularly
privatization and intergovernmental relations. As such, Cooper recommends that

229 | P a g e
public administrators consider these risks in any strategic planning they perform.
Rothschild believes that U.S. initiatives such as InfraGard and illegal wire-tapping
(domestically) and the “war on terror” (internationally), are actually increasing risks to
the country and its people, instead of successfully reducing risks. Finally, Wilson and
Halachmi are staunch proponents for a comprehensive and flexible network system of
relationships between governments, businesses and civil-based society organizations.
These arrangements are viewed as improving public productivity, helping governments
accomplish their goals, and helping in monitoring the environment for risks, while
improving communication and enhancing government responses to crises, effectively
reducing risks throughout the system.

References

Baird, Inga Skromme and Thomas, Howard “Toward a Contingency Model of Strategic
Risk Taking”, The Academy of Management Review, 10(2), 230-243.

Bettis, Richard A. (1983) “Modern Financial Theory, Corporate Strategy and Public
Policy: Three Conundrums”, The Academy of Management Review, 8(3), 406-415.

Bozeman, Barry and Kingsley, Gordon, (1998) “Risk Culture in Public and Private
Organizations”, Public Administration Review, 58(2), 109-118.

Cantor, Robin (1996) “Rethinking Risk Management in the Federal Government”,


Annals of the American Academy of Political and Social Science, Vol. 545, Challenges in
Risk Assessment and Risk Management. 135-143.

Close, Darwin B. (1974) “An Organization Behavior Approach to Risk Management”,


The Journal of Risk and Insurance, 41(3), 435-450.

Cooper, Phillip J. “Courts and Fiscal Decision Making” In Roy T. Meyers ed. (1999)
Handbook of Government Budgeting. San Francisco, Jossey-Bass, Inc. 502-526.

Gahin, Fikry S. (1967) “A Theory of Pure Risk Management in the Business Firm”, The
Journal of Risk and Insurance, 34(1), 121-129.

Halachmi, Arie (2005). “Governance and Risk Management: Challenges and Public
Productivity”, The International Journal of Public Sector Management, 18(4/5), 300-317.
230 | P a g e
Harris, D. John and Williams, D. Glyn (1985) “The Use of Risk Analysis Within British
Rail”, Managerial and Decision Economics, 6(4), 202-209.

Jemison, David B. (1987) “Risk and the Relationship among Strategy, Organizational
Processes, and Performance”, Management Science, 33(9), 1087-1101.

Ring, Peter Smith and Perry, James L. “Strategic Management in Public and Private
Organizations: Implications of Distinctive Contexts and Constraints”, The Academy of
Management Review, 10(2), 276-286.

Rothschild, Matthew “The FBI Deputizes Business” The Progressive, February 7, 2008.

Ruefli, Timothy W., Collins, James M. and Lacugna, Joseph R. (1999) “Risk Measures
in Strategic Management Research: Auld Lang Syne?”, Strategic Management Journal,
20(2), 167-194.

Wilson, Vicki “Being Prepared for Disaster: Strategies and Tactics for Finance
Managers”,

Government Finance Review, Dec. 2007; 23(6), 23-26.

Wolkoff, Michael “State and Local Government Budgeting: Coping With the Business
Cycle”, In Roy T. Meyers ed. (1999) Handbook of Government Budgeting. San
Francisco, Jossey-Bass, Inc. 178-196.

231 | P a g e
2.3 Accounting Methods and Strategic Planning / Management Issues
This paper analyzes the recent change in several government accounting
systems from cash-based to accrual-based. In the United States, the impetus for
this change started in the 1980s and resulted in the 1990s of the government
beginning procedures to adopt the generally accepted accounting principles
(GAAP). Changes to accrual-based accounting in New Zealand, Australia,
Canada, United Kingdom, and in the U.S. are reviewed and their impact on
government oversight and management. Several current costing methods used
by the private and public sectors are described and how accrual-based could
work with these. Finally, whether accrual-based accounting is beneficially
impacting the efforts of the U.S. government to improve transparency and
performance measures is considered.

Introduction

This review of the literature covers current trends in public accounting methods and
focuses on the change from cash basis to accrual basis accounting in the public sector
and how that impacts management initiatives such as activity-based costing (ABC),
cost-volume profit (CVP), and value for money (VFM). This impetus for change from
cash to accrual in the public sector began in the United States in the 1980s and by
the end of that decade it was recognized that much of the government’s financial
systems had become obsolete and did not meet generally accepted accounting
standards (Bowsher, 1990). The National Commission on the State and Local Public
Service (1993), sponsored by the Nelson A. Rockefeller Institute of Government at the
State University of New York, recommended among other things, that state and local
budgets be less line-itemed thus allowing for carry over of unused funds to meet
changing or unforeseen needs and budgets should better account for depreciation of
capital assets thus allowing managers to plan more effectively. Additionally, The
Committee on Governmental Affairs concluded that the absence of timely, relevant,
and comprehensive financial information added to the difficulty of controlling
government operations and costs (Jones & McCaffery, 1998).

It was consequently suggested that the U.S. government adopt the same accounting
principles employed by businesses and many foreign governments, namely generally
232 | P a g e
accepted accounting principles or GAAP (Mautz, 1991). GAAP recognizes liabilities as
they are incurred and associates costs of assets with the period during which they are
used, which is accrual rather than cash basis accounting (DioGuardi, 1989). The
foregoing provided a basis for audited financial statements of agencies and
departments. The Social Security Administration published its 1988 report using the
accrual basis accounting method using full disclosure of assets and liabilities and the
statement attested to the soundness of the Social Security system. Contrastingly,
when the Government Accounting Office (GAO) audited the Federal Savings and Loan
Insurance Corporation using the accrual-based accounting, it found a $13.7 billion
deficit, whereas a cash-based audit for the same period reported a substantial surplus
(Craig, 1989).

Questions for this paper include: Has this change in accounting methods beneficially
impacted strategic management issues in the public sector in the United States and
other countries? Additionally, are program and service performance measures being
captured more efficiently and effectively with accrual-based accounting procedures in
public entities? Finally, how do experts in the field of public administration see the
future prospects for the advantage of these changes in the gathering of usable
financial information?

Accrual Accounting and management issues in other countries

New Zealand had a rapid transition from the Public Finance Act of 1989 and the
setting up of the first milestones of accrual-based accounting (Marty, Trosa & Voisin,
2006). This did not happen without forethought and planning. One year before the
Public Finance Act of 1989, the State Sector Act took stock of the responsibilities that
were to fall on public managers with the change in financial reporting proposed. To
offset the added responsibilities, managers were given greater freedom in terms of the
management of their teams, such as flexible staffing and performance-based rewards.
Therefore, this preplanning stage resulted in a more rapid and smoother transition
than that in other countries.

In 1996, the Australian government introduced an accrual-based outputs and


outcomes budgeting and reporting system (Sterck, 2007). One consequence of this

233 | P a g e
change is that amounts appropriated to governmental agencies are now based on the
full cost of delivering outputs and outcomes rather than the expected cash outflow for
the year. While Parliament is receiving more information than it formerly had at its
disposal, its control has been weakened due to authorization now taking place on a
more aggregated level (i.e., outcome level) and the complexity of the budget and
accounts increasing. Parliament complains that the output information in the annual
agency reports is too aggregated and thus more difficult to get a clear view of the
agencies’ contributions to the outputs. With agencies adapting their outcome and
output structures, this instability has led to less transparency of the financial system
and has weakened Parliamentary control.

Since the late 1990s the Canadian government has attempted to institute a modified
accrual accounting system in concert with the new structure under which the
departments and agencies have to manage and report to Parliament, namely the
Program Activity Architecture [PAA] (Sterck, 2007). Under this new structure, costs are
allocated to program activities, subactivities, and subsubactivities. However, even
though this has led to the Canadian Parliament having more information on program
achievements and costs, members of Parliament have not been using this information
in any consistent fashion to influence their budget decisions to date.

The implementation of the new public accounting system in the United Kingdom (UK)
was spread over seven years between the approval of the White Paper on accounting
and budgetary policies in 1995, the Government Resources and Accounts Act in 2000,
and finally the publication on public accounts on the basis of Accrual Based
Accounting in 2002 (Marty et al., 2006). Currently, the UK Treasury has shifted its
accounting policy away from the traditional appropriations to ‘resource accounting’
involving accrual basis accounting (Wilks, 2007). This along with other reform
initiatives has led to the current emphasis on ‘corporate governance’ and the
institution of ‘board of directors’ to oversee government agencies and municipalities.

Wilkes points to both advantages and disadvantages of this development in the UK.
Advantages include: 1) location of responsibility; 2) ethical expectations; 3)
transparency; 4) consistency; 5) performance measurement; and 6) accountability.

234 | P a g e
Potential pathologies include: 1) formalistic compliance; 2) failure of mechanisms of
control; 3) creation of biased or inadequate boards; 4) lack of coordination; or 5) crude
managerialism emerging from a preoccupation with a simplified bottom line. This last
possible problem is the most concerning when the public interest should be involved
in what good performance of government entities really means.

Use of performance measures

Halachmi (2005) cautions first about the cost-benefit analysis of performance score
cards, stating the cost is always significant while the benefits in many instances may
only be tentative. Secondly, Halachmi enumerates the differences of measurements
for accountability (‘Was it done right’) as opposed to productivity (‘Was the right thing
done’) as follow:

 “Accountability is living up to performance standards that existed when the use


of resource/authority was authorized.”

 “Accountability is primarily about relationships: Who is superior to whom? Who


is answerable to whom? What must be reported and who decides it?”

 “Productivity is more than keeping with past trends or marginally improving on


them.”

 “Productivity relates to progress, innovation, and change, preferably moving to a


higher curve rather than moving to a higher point on the same productivity
curve.”

 “Productivity is about management, adaptation, creativity, and breaking away


from the past or from the group, while accountability is about staying within
the four corners of the contract.”

 “Productivity has to do with a continuous free-form process of self-examination


and an internal search for new insight, whereas accountability involves external
scrutiny and a relatively rigid use of pre-established legal or professional
standards.”

Why bring this out when writing on accounting methods relating to strategic
management issues? The above dichotomy seen by Halachmi between accountability
235 | P a g e
and productivity performance measures relate to the limitations of financial reporting
(accountability) to enable public sector managers to manage strategically to improve
performance (productivity). Strategic management needs greater insights and
adaptation than the financial bottom line can accommodate. Can any of the current
management costing systems used in conjunction with accrual accounting facilitate a
link to performance measures that will enable real progress in the use of outcome
information in government planning and budgeting?

Linking performance measures to accurate costing information

The Office of Management and Budget (OMB) has been in the process of encouraging
the development of definitive performance measures linked to accurate cost
accounting that would conform to the National Performance Review (NPR) goals (Jones
& McCaffery, 1998). In 1996 Congress passed the Federal Financial Management
Improvement Act (FFMIA), which in conjunction with the Government Performance
and Reporting Act (GPRA) of 1993 attempts to integrate financial and performance
data in a way that relates to budget accounts.

Brimson & Williams (2007) analyzed the U.S. federal government’s financial and
performance reporting models. While the current financial reporting model is single
dimensional with an accounting regulator creating a set of principles as to how
financial effects of transaction and events are to be measured, recorded and reported.
These authors also analyze the proposed process-based accounting, which is a
multidimensional approach that bridges both financial reporting and cost
management as is the intent of the FFMIA.

Can costing methods currently used by the public and private sector such as activity-
based costing (ABC), cost-volume-profit (CVP), or value for money (VFM) effectively
capture the true costs needed to link performance to cost management? First,
According to Martin (2006) designing an ABC model system requires:

1) Identifying the main activities;

2) Determining the primary and secondary drivers of these activities;

3) Aggregating activities into homogenous cost pools; and


236 | P a g e
4) Selecting activity measures to represent each pool or cost driver.

Therefore the activities are the sum of the resources that are needed to develop the
productive process and that add value to the product or service (Gonzalez-Gomez &
Morini, 2006).

Conclusions reached by Akkyol, Tuncel & Bayhan (2005), after doing a case study of
ABC implementation in a manufacturing company, were that ABC is capable of
monitoring hidden losses and profits of the traditional costing methods. Pricing
decisions can be done under the scope of these hidden losses and profits with the
company increasing or decreasing product costs to gain a more competitive advantage.
This study also found that the existence of the ABC database was an advantage to the
Balanced Score Card implementations as further performance analysis is undertaken.

Can cost-volume-profit analysis capture sufficient liability information? Kee (2007)


found it necessary to expand on traditional CVP analysis to incorporate the cost of
capital. This cost of capital is traced to products like the cost of overhead-related
resources using the principles of ABC costing. The opportunity cost of invested funds
is deducted from the product’s operating income after taxes and measures it economic
income. This CVP model based on the discounted economic income of a product
enables managers to compute a product’s breakeven sales quantity, to measure a
product’s profitability over the range of it sales and to determine the rate of change in
its profitability. The CVP also facilitates measuring the tradeoffs in alternative
investment and cost upon a product’s profitability from a program of process
improvement.

Value for money, simply put, is the price minus the cost of producing a product or
delivering a service (Shannon, 2006). To maximize the value for money, the gap
between the price and cost must be increased. Whether public or private entities, the
main emphasis to attain more value for money is to decrease the costs. The difference
is that in the public sector the price charged for services and products or the monies
from taxes cannot always be increased to account for the increased costs.

Differences in the public and private sector cost management objectives

237 | P a g e
For government contractors in a cost reimbursement contract, costs drive a
contractor’s revenues and profits (Mackie, 2006). Any decrease in cost results in
immediate profit loss. Therefore, government contractors believe a change in the way
costs are reported usually means lower revenues and decreased profits. In the private
sector, any cost savings leads to increased profits and potential increase in the market
share. Currently, a contractor has to refund any savings from lower costs to produce
a product or deliver a service for the government. Adoption of ABC management may
provide more accurate cost data with which to price items, but it may reduce
contractors’ operating margins and profits, increase the efforts of providing cost data,
and lengthen the approval process of final annual indirect cost allocations.

The cost management objectives of the public sector are not typical of those of the
private sector (Mackie, 2006). First, government contractors are primarily interested in
obtaining a fair recovery of all costs and currently believe it is fair to attach general
and administrative costs to direct labor hours using an overhead rate even though it is
not the best way to manage costs. Not every contract uses all the activities grouped
into the administrative pool, however these are still charged costs based upon their
percentage of the total cost base. Secondly, the government seeks uniformity and
consistency in cost accounting practices, whereas private sector manufacturing
methods are diverse and dynamic. In addition, government contract regulations
ensure that all contracts use the same set of cost recovery rules regardless of their
manufacturing methods and not all of these methods have the same costs. Even with
the foregoing, the government encourages the use of advanced cost management
systems, such as ABCM.

Will accrual accounting methods benefit management and planning?

Accrual-based accounting is one of the decisive stages of the attempts to


comprehensively reform public management (Marty, et al., 2006). The first objective in
the U.S. federal government’s move to accrual-based accounting was to better
management the government’s assets evident from the concomitant implementation of
policies to improve management of government real estate. This accounting valuation
of public assets can lead to further incentives for long-term management of the
natural and cultural heritage.

238 | P a g e
Secondly, the move to accrual accounting offers the possibility of defining a complete
cost. This definition of complete cost on the same accounting foundations as private
firms is essential if the public and private offers are to be compared to each other
within the framework of partnership contracts. Further, the financial impact of
commitments relating to pensions or social services will help to clarify public decision
by supplying an honest and fair picture of the financial situation and performance of
public entities.

While the benefits of accrual-based accounting for government agencies are many, it is
not without risks and potential impediments to NPM or NPR initiatives. First, the costs
of set up and training involved in switching to accrual are necessary but considerable
(Marty, et al., 2006). Second, the years of time invested to reach compliance of
accounts and reports. Third and perhaps most important, the investment relating to
the explanation of the reform to involve the staff in the process is daunting but vital to
its success.

Conclusions

The above considerations, particularly the financial cost of the transition, are among
the reasons that the Netherlands, while a ground-breaker in evaluating the cost of the
service delivered by its public administrations, has rejected the adoption of the
accrual-based accounting system for the government (Marty, et al. 2006). Has the
United States government made a wise choice in the changeover to accrual-based
accounting? If it gives managers more of the tools they need to plan for program costs
and to evaluate the returns in outcomes realized by these outlays, then it will be
advantageous. However, in analyzing the Central Banks’ move to accrual-based
accounting, Mendezela (2007) cautioned Banks to accept inherent imperfections of
accounting and reporting systems and reminded to use real-world info to improve real
world service performance and management. The next few years will bring further
insight into whether the United States’ move to accrual-based accounting in its
government institutions enhances strategic planning and management initiatives or is
an impediment to real progress.

Summary of Accounting Methods and Strategic Planning and Management Issues

239 | P a g e
Introduction

 Review of literature to cover current trend of change from cash-based to


accrual-based accounting in the public sector and how this impacts planning
and management issues

 Impetus for change with the “Winter Commission Report” (1993) on state and
local governmental reform needs and

 Committee on Governmental Affairs that called for timely, relevant and


comprehensive financial information to aid in controlling government
operations & cost (Jones & McCaffrey, 1998).

 These reports were in concert with the earlier examples of how accrual-based
accounting could give a more accurate picture of an institution or agency’s
fiscal soundness.

 Social Security Admin 1988 annual report used accrual-based accounting using
full disclosure of assets and liabilities, which attested to fiscal stability of the
system.

 In contrast, the GAO’s audit of the Fed Sav & Loan Ins. Corp using accrual-
based accounting found a $13.7 billion deficit.

Questions covered

 Has this change to accrual-based accounting beneficially impacted planning &


management issues in the US and other countries?

 Are performance measures being captured more efficiently and effectively using
accrual-based accounting procedures?

 What are the future prospects for obtaining useful financial information using
accrual accounting for public administration?

Accrual Accounting & Management Issues in Other Countries: New Zealand

240 | P a g e
 Public Finance Act - 1989 - Preplanning lead to rapid and effective transition to
accrual.

 In 1988 assessed what greater responsibilities fall on mangers

 To offset above, managers given greater freedom in terms of management of


teams with flexible staffing and performance-based rewards.

Accrual Accounting & Management Issues in Other Countries: Australia

 1996 - government went to accrual-based outputs and outcomes budgeting &


reporting systems

 Appropriations now based on full cost of delivering outputs & outcomes rather
than expected cash flow for the year

 Parliament complained that output info too aggregated and thus more difficult
to assess agency’s contributions to outputs.

 Above has led to less transparency of financial system and weakened


Parliamentary control and planning.

Accrual Accounting & Management Issues in Other Countries: Canada

 Late 1990s instituted modified-accrual accounting in concert with the Program


Activity Architecture structure for agencies reporting to Parliament

 The costs were allocated to program activities, sub-activities, etc.

 To date, this has not led to Parliament using this info consistently to influence
budgeting decisions.

Accrual Accounting & Management Issues in Other Countries:United Kingdom

 Implementation of new accrual accounting system spread over 7 yrs starting


with the White Paper on accounting & budgetary decisions in 1995 to
publication on public accounts in 2002 (Marty, et al., 2006).

241 | P a g e
 UK Treasury has shifted accounting policy from traditional appropriations to
resource accounting and has led to emphasis on corp. governance & to
institution of boards of directors for oversight of municipalities (Wilks, 2007).

 Advantages of transparency & accountability

 Potential problems - biased or inadequate boards that make decisions on the


simplified bottom-line instead of pub interest (Wilks, 2007).

Use of Performance Measures

 Halachmi (2005) cautions against confusing accountability (financial bottom-


line - Was it done right?) with productivity (strategic planning for better service
to the public interest - Was the right thing done?)

 Strategic management needs greater insights and adaptation than the financial
bottom line can accommodate

 Can any of current costing systems be used with new accounting method to
facilitate meaningful link to performance measures?

Linking Performance Measures to Accurate Costing Information

 OMB has encouraged development of definitive performance measures linked to


accurate cost accounting that would conform to the NPR goals (Jones &
McCaffrey, 1998).

 In 1996 Congress passed the Federal Financial Management Improvement Act


(FFMIA) in support of the GPRA of 1993 - attempting to integrate financial and
performance data in a way that relates to budget accounts.

 Brimson & Williams (2007) analyzed US government’s financial & performance


reporting models

 Analyzed proposed process-based accounting, which is multi-dimensional


approach that bridges both financial reporting and cost management as is
intended under FFMIA.

242 | P a g e
Current costing methods: ABC

 Activity-based costing requires:

– Id the main activities

– Determine primary & secondary drivers of activities

– Aggregate activities into homogenous cost pools

– Select activity measures to represent each pool or cost driver (Martin,


2006).

 Activities are the sum of the resources needed to develop the productive process
and add value to the product or service (Morini, 2006).

 ABC capable of monitoring hidden losses and profits of traditional costing


methods

 Pricing decisions based on the above can help gain competitive advantage

 ABC database is an advantage to the Balanced Score Card implementations

Current costing methods: CVP

 A modified Cost-Volume-Profit (CVP) adding the cost of capital that is traced to


products or services like the cost of overhead-related resources

 This CVP model based on the economic income enables manager to compute a
product’s breakeven sales quantity, its profitability over range of its sales and
determine rate of change in profitability

 Facilitates measuring tradeoffs in alternative investment and cost on a


product’s profitability from a process improvement perspective (Kee, 2007).

Current costing methods: VFM

 Value for money (VFM) simply is cost of producing a product or delivering a


service minus the price charged

243 | P a g e
 To maximize VFM gap between price & cost must be increased

 Main emphasis in VFM is decreasing cost

 In public sector - value should not just be financial return for service -

Public versus Private Cost Management Objectives

 Government contractors primarily interested in obtaining fair recovery of costs


and even though contract may not use all activities grouped into admin pool -
still charged based on % of total cost base

 Government seeks uniformity and consistency in cost accounting practices


while private sector accounting methods are diverse and dynamic

 Public contract regulations ensure all contracts use same set of cost recovery
rules regardless of manufacturing methods and not all methods the same

Accrual accounting benefits planning & management

 Accrual-based accounting one of decisive stages of the attempts to


comprehensively reform public mgmt

 First objective of US government in moving to accrual - to better manage assets


- leading to long-term management of natural & cultural heritage

 Move to accrual offers definition of complete costs on same accounting


foundation as private firms - essential with public & private partnerships

 Financial impact of commitments relating to pensions or social services help


clarify decision-making

Risks & Problems of Accrual Accounting

 Costs of set up training considerable

 Years of time invested to reach compliance of accounts & reporting

 Explanation of reform to staff

244 | P a g e
 Netherlands - ground-breaker in evaluating costs of services - rejected accrual
change - above reasons

 Basing decisions & planning on accounts receivables that may not occur -
example - taxes not able to collect

References

Alexander, S. (1999). Accrual vs. cash-based accounting. Computerworld, 33(27), 1-4.


Retrieved March 7, 2008, from the Academic Search Premier database.

Babad, Y.M. & Balachandran B.V. (1993). Cost driver optimization in activity-based
costing. The Accounting Review, 68(3), 563-575.

Baird, K. (2007). Adoption of activity management practices in public sector


organizations. Accounting and Finance, 47(4), 551-569.

Benito, B., Brusca, I., & Montesinos, V. (2007). The harmonization of government
financial information systems: the role of the IPSASs. International Review of
Administrative Sciences, 73(2), 293-317.

Bolivar, M.P., & Galera, A.N. (2007). Could fair value accounting be useful, under NPM
models, for users of financial information? International Review of Administrative
Sciences, 73(3), 473-502.

Bowsher, C.A. (1989). Comptroller General of the United States. Statement, hearings,
“Improving federal financial management,” House of Representatives, committee on
government operations, subcommittee on legislation and national Security. Washington,
D.C., Government Printing Office.

Brimson, J.A., & Williams, C.A. (2007). Process-based financial. Journal of Government
Financial Management, 56(4), 16-20.

Chen, C.C., & Jones, K.T. (2007). Management tools. CPA Journal, 77(8), 1-8.
Retrieved January 16, 2008, from the Business Source Premier database.

245 | P a g e
Christensen, M., & Yoshimi, H. (2000). A value-for-money pathway: a two-country
case study of performance reporting. International Review of Administrative
Sciences,66, 433-449.

Combs, S. (n.d.). Activity-based costing and management: The Texas pilot project.
Retrieved February 19, 2008, from
http://www.window.state.tx.us/specialrpt/abc/costmgmt.html]

Craig, L.E. (1989). Congressman. Statement, hearings, “Improving federal financial


management,” House of Representatives, committee on government operations,
subcommittee on legislation and national security. Washington, D.C., Government
Printing Office.

DioGuardi, A. (1989). Congressman. Statement, hearings, “Improving federal financial


management,” House of Representatives, committee on government operations,
subcommittee on legislation, and national security. Washington, D.C., Government
Printing Office.

Drake, A.R., Haka, S.F., & Ravenscroft, S.P. (1999). Cost system and incentive
structure effects on innovation, efficiency and profitability in teams. The Accounting
Review, 74(3), 323-345.

Grumet, L. (2007). The importance of financial transparency: swallowing a bitter (but


necessary) pill. The CPA Journal, 77(6), 7-9.

Halachmi, A. (2005). Performance measurement: test the water before you dive in.
International Review of Administrative Sciences, 71(2), 255-266.

Juras, P., & Peacock, E. (2006). Applying strategic cost analysis concepts to capacity
decisions. Management Accounting Quarterly, 8(1), 24-35.

Kamnikar, J.A., Kamnikar, E.G., & Deal, K.H. (2006). Assessing a state’s financial
condition. The Journal of Government Financial Management, 55(3), 30-37.

Kee, R. (2007). Cost-volume-profit-analysis incorporating the cost of capital. Journal of


Managerial Issues, 19(4), 478-493.

246 | P a g e
Kelley, A.G., & Ruggieri, M.P. (2007). Municipalities get a healthy dose of reality on
postemployment benefits: the effects of GASB 43 and 45 on government finances. The
CPA Journal, 77(4), 28-32.

Kennet, D.L., Durler, M.G., & Downs, A. (2007). Activity-based costing in large U.S.
cities: costs & benefits. The Journal of Government Financial Management, 56(1), 20-
31.

Levy, J. & Young, M. (2003). Caution: accounting changes ahead. Benefits Quarterly,
4, 36-46.

Marty, F., Trosa, S., & Voisin, A. (2006). The move to accrual based accounting: the
challenges facing central governments. International Review of Administrative Sciences,
72(2), 203-221.

Mautz, R.K. (1991). Generally accepted accounting principles. Public Budgeting and
Finance, 11(4), 3-11.

Mendzela, J. (2003). Central bank accounting and reporting. Accounting standards for
central banks, (pp. 1-20). Central Banking Publications.

National Commission on the State and Local Public Service. (1993). Hard truths/tough
choice: An agenda for state and local reform. Accessed March 1, 2008
fromhttp://www.rockinst.org/WorkArea/showcontent.aspx?id=9980.

New Zealand Inc. (1992). Economist, 324, 1-3. Retrieved March 7, 2008, from
Academic Search Premier database.

Shannon, P.E. (2006). When profit means value for money. Clinician in Management,
14, 175-176.

Sterck, M. (2007). The impact of performance budgeting on the role of the legislature:
a four-country study. International Review of Administrative Sciences, 73(2), 189-203.

Vercio, A. & Shoemaker, B. (2007). ABCs of batch processing. Journal of Accountancy,


204(2), 1-7. Retrieved January 16, 2008, from Business Source Premier database.

247 | P a g e
Wilks, S. (2007). Boardization and corporate governance in the U.K. as a response to
depoliticization and failing accountability. Public Policy and Administration, 22(4), 443-
460.

248 | P a g e
2.4 The Sarbanes-Oxley Act of 2002; Audit and Governance of Not for Profit
Organizations
Section I: Summary of the key provisions the Sarbanes-Oxley Act of 2002 and
their impact in the nonprofit sector

On July 30, 2002, President Bush signed into law the Sarbanes-Oxley Act of 2002,
which he characterized as "the most far reaching reforms of American business
practices since the time of Franklin Delano Roosevelt." The Act mandated a number of
reforms to enhance corporate responsibility, enhance financial disclosures and combat
corporate and accounting fraud, and created the "Public Company Accounting
Oversight Board," also known as the PCAOB, to oversee the activities of the auditing
profession (U.S. Securities and Exchange Commission). The Sarbanes-Oxley Act (SOX)
is arranged into eleven titles. As far as compliance is concerned, the most important
sections within these eleven titles are usually considered to be 302, 401, 404, 409,
802 and 906. The Act can be divided into three parts: internal controls (exercised by
management), external checks (performed by the board or external auditors), and
investigations (triggered by whistleblowers or others). Only two provisions directly
apply to nonprofit organizations: whistle-blower protection and document
preservation. These provisions require employees or volunteers of a nonprofit to be
shielded from retaliations for whistle-blowing or making reports of waste, fraud, or
abuse. Nonprofits are expected to comply with the document requirement by having a
fully functioning policy in place for the preservation, archiving, and disposal of agency
documents.

SOX creates parameters to ensure the independence of auditors from their clients by
requiring that auditors report directly to an audit committee, reporting all critical
accounting, policies, and practices used, as well an accounting of alternative
treatments of financial information shared with management. Disclosure must also
include a discussion of how the ramifications of alternate treatments might impact
stakeholders. Chief Executive or Financial Officers are required to certify the propriety
and accuracy of financial statements and periodic reports. The law requires
organizations to conduct and publish annual reviews of internal controls and process,
financial reporting procedures, and year-end results, including disclosure of recent

249 | P a g e
material changes in financial condition or operation. The law established criminal
penalties for any person who knowingly alters, destroys, or conceals documents with
the intent to obstruct justice or influence an official investigation.

In her report Responding to the Sarbanes-Oxley Act of 2002; The Financial Reporting
Practices of Nonprofits, Heinz (2003) rightly states that not for profit agencies should
be held to the highest standards as they often accept private money on behalf of
people in need. Their finances should be held open to the tightest scrutiny. Jackson
(2005) in Nonprofit Strategic Planning suggests that the spillover of SOX into the public
sector can be tied to the close alignment between business models of private sector
firms and current trends in nonprofit management. Just as a business must prove
compliance with SOX legislation before launching an Initial Public Offering (IPO), non
for profit organizations must assuage the apprehensions of potential donors or
partners by assuring that accountability standards are high and impregnable.
Oxholm (2005) suggests that, in large measure, non-profits enjoy the special benefits
they receive— exemption from tax being chief among them—because they do the
public’s business. The substantial financial assistance they receive indirectly from all
levels of governments, even without regard to grants, arguably makes them more
deserving of governmental oversight and control than publicly-traded companies,
because it is the public’s tax money, not that of private investors, that is being spent.

Whether Sarbanes-Oxley should be applied to non-profits is still being debated,


particularly in consideration of the potentially substantial costs involved that may be
burdensome to small organizations. By taking this proactive stance, however, Jackson
(2007) suggests that nonprofits will earn a competitive advantage that will inspire
confidence among stakeholders, ensuring that directing boards and executives
understand the operational importance of SOX mandates. Boude (2006) reaffirms that
when the Sarbanes-Oxley Act was adopted, the Congressional record indicated that it
was not intended to apply to any organization other than public companies. At that
time, many commentators predicted that regardless of the intent of Congress, the
guidelines would

ultimately filter down to all businesses under the guise of “best practices.” Boude’s
(2006) report found that The Sarbanes-Oxley Act continues to have a significant

250 | P a g e
impact on private organizations, with 86% of his survey respondents indicating that
SOX and other corporate governance reform requirements had impacted their
organizations.

A majority (70%) of the private organizations surveyed implement self-imposed


corporate governance reforms.

Section II: How non-profit organizations are reacting to the opportunity/threat


of greater accountability

Jackson (2007) views the SOX requirements as an opportunity for nonprofits to


improve their operational systems, procedures and methods for doing business via
better control mechanisms. Ancillary benefits are also accrued to the organization via
streamlined standard operating procedures, record keeping, and policy adoption as a
result of engaging in SOX compliance activity. Jackson finds that engaging in the
strategic planning process forces a nonprofit to evaluate how it is perceived by its
donors and the public at large as it engages external and internal stakeholders in the
development of goals and objectives that are commonly held.

Boude’s research (2006) reveals that more nonprofit survey participants cited
implementation or planned implementation of more aspects of corporate governance
reforms than for-profit organizations. In comparing the data, Boude cited three groups
consistently influencing the decision to adopt corporate governance standards: the
organizations themselves, their board members and their auditors. The influence of
these three groups fuels the continued adoption of standards as perceived “best
practices.” The research also contends that auditors generally perceive they have
more risk on audits they perform for large organizations and therefore exert more
pressure on these organizations to adopt SOX standards. The voluntary corporate
governance measures private organizations have implemented continue to come at a
cost to those organizations. Private organizations estimated an average annual price
tag of $105,000 for corporate governance procedures, representing an estimated
increase of approximately 26% over their estimated costs prior to the enactment of the
Sarbanes-Oxley Act.

251 | P a g e
Section III: How other countries are impacted by the SOX Act; Focus on EU and
China

Sound corporate governance is good for maximizing the shareholder value and
productivity of companies. In countries outside the United States, powerful mimetic
forces in international business practice drives isomorphic tendencies that create
pressure (and incentive) for organizations to adopt Sarbanes-Oxley requirements
(Lucci, Dewing & Russell, Lin). It is important to note that the Act applies only to
foreign corporations that elect to avail themselves of the benefits of American capital
markets. Lucci (2003) From its inception, Sarbanes-Oxley “caused headaches” for the
more than 1,300 foreign firms, such as British Airways and GlaxoSmithKline, that
trade on an American exchange, and left many foreign officials uncertain as to which
provisions of the Act applied to them. Lucci reports that these officials criticized the
Act for regulating foreign accounting services and corporations. Contraindications for
adoption of SOX standards cited by Lucci conclude that exempting foreign
corporations from the Act’s coverage would create economic disparities between
American and overseas firms, as one group would be required to incur compliance
costs and the other would not. Furthermore, if corporations outside the United States
are exempt from Sarbanes-Oxley’s more stringent requirements, this could provide a
perverse incentive for corporations to leave the United States to seek refuge in
countries with less stringent accounting and corporate governance standards. (p.219)
As a basis for exemption from the Acts provisions, many European executives argue
that firms are already highly regulated in their respective countries and that
Sarbanes-Oxley adds another layer of unnecessary regulation. Lucci offers as an
example why it is necessary for French companies to follow the regulations of
Sarbanes-Oxley when the country already has two of its own oversight boards.

Dewing and Russell (2004) provide an overview of EU policy developments in


accounting, auditing and corporate governance before and after the collapse of Enron.
For EU policy-makers the article identifies areas for both encouragement and concern.
It concludes that considerable progress has been made towards the harmonization of
accounting, auditing and corporate governance within the context of the Financial
Services Action Plan. However, the authors argue that to achieve this balance, the EU

252 | P a g e
“has given too much ground to US hegemony, whether by embracing US practice
masquerading as international best practice.”

Lin (2004) cites China’s securities market as well as corporate governance structure
and rules as important factors in its economic reforms. Since 1992, China has made
substantial progress in specifying rights and since entering into the World Trade
Organization (WTO) in late 2001, the Chinese government has continued to improve
its corporate governance policies. Lin’s work provides specific examples and
recommendations of how following rules form the Sarbanes-Oxley Act can be modified
and used to improve the quality of China’s company corporate governance. (Lucci,
2003) quotes Wang Xiaochu, CEO of blue-chip company China Mobile in Hong Kong,
which does business in the United States following SOX regulations: “‘Our accounts
are very transparent and accurate, we will be happy to sign the document and bear
the legal responsibility.”

By applying one set of standards to all corporations—regardless of whether they are


located in the United States or abroad—investors will be assured that each and every
corporation deriving a direct benefit from American capital markets will be subject to
the stringent protections in Sarbanes-Oxley. Corporate migration to avoid Sarbanes-
Oxley will lead to further instability in markets if companies seek refuge in
jurisdictions offering less accountability for corporate wrongdoing.

Section IV: Implications for the future – What will motivate non-profits to ensure
compliance?

Oxholm (2005) maintains that Sarbanes-Oxley is not designed to rid businesses of


corruption as it does not punish anyone for embezzling, wasting corporate assets,
excessive compensation, or anything else. Those rules, and punishments, lie
elsewhere. Instead, its goal is to encourage the earlier discovery and disclosure of
corruption. Its method is to require businesses to have enough incentives and
mechanisms in place to persuade persons who are generally lower in the
organizational chart to disclose problems and possible wrongdoing to someone with
greater authority.

253 | P a g e
Heinz (2002) finds that accounting professionals and regulators will initiate driving
forces to motivate change in the financial practices and auditing process of nonprofits.
Regulatory control of non-profits from states will as they intensify oversight of
activities through detailed reporting requirements and prosecution of illegal practices.
Heinz proposes that this effort will constitute a “collective watch dog group” that
oversees nonprofits. State initiatives mirroring SOX include the California Nonprofit
Integrity Act (SB1262), which applies to any nonprofit with a budget in excess of $2
million that solicits donations within the state.

Ostrower &Bobowick (2005) include among these driving forces IRS requirements for
posting financial statements (which are public, but the authors suggest should be
made available electronically for stakeholder viewing). Their research found that only
11% of organizations were posting their IRS returns on the agency’s site. Additional
measures to ensure compliance would be collaboration with Certified Public
Accounting protocols, real-time distribution of material, and use of plain language
documents. Non-profit boards will be called on to re-examine their composition and
possibly alter them to include ad hoc committees that represent interests vested in the
stakeholder constituency. Executives will need to provide adequate training for
management in understanding accounting and reporting measures, not only for
employees but for board members as well. Relying on board members for specific
guidance in financial and planning matters will be integral in ensuring compliance
and transparency.

Section V: Emerging best practice and strategies for governance and auditing

In 2004-2005 Senate Finance Committee Hearings, Mark W. Everson, Commissioner


of the Internal Revenue Service (IRS), testified on a more aggressive oversight and
enforcement stances for uses related to nonprofit organizations (Jackson 2004, pp. 14-
15) Addressing the concern that almost 3 million agencies qualify as tax-exempt
entities, employing about one if every four workers in the U.S. If abuses in accounting
are left unchecked by these agencies, there is a risk that Americans will lose faith in
and reduce support for charitable organizations, potentially compromising the
integrity of the national tax systems. Calls for increased transparency in the tax
exempt sector include compliance with IRS regulations, five-year reviews of tax-exempt

254 | P a g e
status based on data collected by the IRS, adoption of policies that comply with SOX
requirements, and potential agency accreditation from professional bodies that outline
industry specific standards of best practice. Jackson (2005) recommends that
nonprofits ensure that their operations are consistent with stated mission, in their
words, “that they are who they say they are and do what they say they are doing.”

Oxholm (2005) describes a process undertaken in higher education that parallels


initiatives from strategic planning and governance. “Toward ensuring that adequate
controls are in place, consultants will analyze each process to see what kinds of
controls the system already has (“analysis”) and what is missing (“gap analysis”). Then
they will determine what new controls are required (“diagnostics”), designing and
installing what it lacks (“remediation”). Finally, consultants will test them (“validation”)
and then audit them on an annual basis, comparing them to those of our peers
(“benchmarking”).”

Broude (2006) provides the most common aspects of corporate governance reform that
some private organizations have implemented or plan to implement in response to the
regulations on public companies. These are: audited financial statements,
establishing independent directors, establishment of a corporate ethical code and
audit committee oversight of auditors. Research results show that 95% of nonprofits
implement or plan to implement these reforms, compared to only 78% of for profits
Among not for profit organizations, Broude (2006) reports that more large
organizations (with annual revenue or budgets in excess of $300M) have implemented
or plan to implement more aspects of governance reform when compared to smaller
organizations. Smaller organizations typically answer to a smaller number of
constituencies that may be less concerned about corporate governance reform. Broude
sees this trend is logical because large organizations are more able to easily absorb the
costs associated with adopting aspects of governance reform.

Webb (2008) found several differences between compliance groups in terms of equity
ownership, board structure, and executive compensation schemes. However, the
strongest determinant of Section 404 compliance is firm size. This result supported
anecdotal evidence that compliance with SOX is achieved primarily by firms that can
afford it. Webb’s work suggests that firms found to be in violation of SOX are not

255 | P a g e
systematically worse when it comes to common measures of corporate governance.
The financial structure and soundness of the groups of firms are very similar to
groups in compliance. This work raises an important policy issue: Is SOX really
differentiating firms in terms of corporate governance or in terms of size?

The U.S. General Accounting Office (GAO) issued its own Auditor Independence
Standard in 2002, establishing standards for auditor services and behavior that are
similar to those passed in Sarbanes-Oxley. The GAO standard applies to federal
agencies, state and local governments, any nonprofit agency directly receiving federal
grants, and many nonprofits receiving federal funds from pass-through grants or
through purchase of service contracts with their local and state jurisdictions. As
models of governance evolve to include public-private partnerships that seek to
achieve the greatest public good with high levels of efficiency and with limited risk,
Halachmi (2005) would suggest use of market forces and non-governmental entities to
replace government agencies and regulations that are expensive and cumbersome to
implement. Sarbanes-Oxley was conceived as a rebuttal and remedy to scandal and
corruption, yet is itself subject to revision as the political tides of reform wash away
memories of past wrongdoing in the face of current economic pressures. Alternative
approaches to governance would promote managing risks to the public good by relying
less on government regulation and more on market forces that can exert safeguarding
measures through effective administrative management.

References

Broude, P.D. (2006). The impact of Sarbanes-Oxley on private & nonprofit companies.
Paper presented at the National Directors Institute of Foley & Lardner LLP,
Chicago, Illinois. International Journal of Auditing Vol. 7 Issue 3 Page 247
November 2003

Dewing, I. P. & Russell, P.O. (2004) Accounting, Auditing and Corporate Governance
of European Listed Countries: EU Policy Developments Before and After Enron
JCMS: Journal of Common Market Studies 42 (2), 289–319.

Halachmi, A. (2005 ). Governance and risk management: challenges and public


productivity. International Journal of Public Sector Management, 18(4), 300-317.
256 | P a g e
Heinz, P.A. (2003). Responding To The Sarbanes-Oxley Act Of 2002; The Financial
Reporting Practices of Nonprofits. A report published by the Alliance for Children and
Families. The Surdna Foundation, New York

Jackson, P.M. (2007). Nonprofit Strategic Planning,

Wiley Lin, T.W. (2004). Corporate Governance in China:Recent Developments, Key


Problems, and Solutions. Journal of Accounting and Corporate Governance, Vol. 1,
June, 1-23.

Lucci, J.P. (2003). Enron: The Bankruptcy Heard Around the World and the
International Ricochet of Sarbanes-Oxley. Albany Law Review, 211 (28), 211-241.

Oxholm, C. (2005) Sarbanes-Oxley in higher education: Bringing corporate America’s


“best practices” to academia. Journal of College and University Law, 31 (2) 351-
375.AN INSTITUTE

Ostrower, F. & J. Bobowick, M. J. (2005) Nonprofit Governance and the Sarbanes-Oxley


Act. A report prepared by the Urban Institute. Washington, D.C.

Publication available on the Urban Institute web site, at http://cnp.urban.org.

U.S. Securities and Exchange Commission.


http://www.sec.gov/about/laws.shtml#sox2002

Sarbanes-Oxley Act of 2002, Pub. L. No. 107-204, 116 Stat. 745 (2002) (codified in
various sections of the U.S.C.A.)

Webb, E. (2008) Sarbanes-Oxley compliance and violation: an empirical study.


Review of Accounting and Finance, 7 (1) 5 – 23.

257 | P a g e
2.5 Etzioni & Lorcsh - Attempts to Combine
Etzioni

• Structuralist model

– Combine Weber, a little Marx and natural systems

– Weber and Marx on alienation

– But why no conflict?

Authority and Power

• Source of Conflict—Marx

• Separation from the means of production-Marx

• Separation from the means of administration—Weber

• Therefore there should be conflict

Addition of Natural

• Acceptance of power by the powerless-Bernard

• Informal relations

• Limit the conflict and alienation

Lawrence and Lorsch

• Extension of contingency model

• Rational or natural dependent on the environment

• Lets talk about restaurants

– Compare the hotel with McD’s

• Hotel natural

• McD rational

Thompson’s Levels

• Technical level—production—rational

• Managerial level—controlling the production system and personal—natural

258 | P a g e
• Institutional level—relates to the larger environment—open

259 | P a g e
2.6 Admin Management – Strategic Management
STRATEGIC PLANNING – 2 BASIC APPROACHES:

1) Start with a MISSION: Given the mission, what can you do in order to carry
it out successfully? Because environment changes, we constantly adjust “what
we can do”.

2) Start with an IDEA: Start by identifying STRENGTHS & WEAKNESSES,


thus the evolving strategy (or evolving structure) will support the idea that you
can maximize your strengths, If cannot change structure, then you can
concentrate on STRATEGY.

STRATEGY IN THE PUBLIC SECTOR STARTS WITH THE MISSION!

Halachmi – “The notion of a stable environment doesn’t exist anymore”.

 With strategic planning, you don’t need to succumb to the typical life cycle
of organizations.

Several professors said to KNOW: POLITICAL ECONOMY – OSTROM, NISKANEN,


WAMSLEY & ZALD

Know these Org Theory authors for Comps: Cyert & March, Merton, Gouldner,
Parsons, Etzioni, Porter, Ronald Coase (Transaction Cost Econ).

CHANDLER – The structure of modern organization emerges from the strategy that
business pursues as it defines and capitalizes on opportunities.

MINTZBERG (1978) – Patterns in Strategy Formation.

1. Orgs experience cycles of strategy formation. The frequency of planning and


strategy formation depends on the environment and what pressures for
change exist.

2. Mintzberg asks: “Why do orgs undergo distinct periods of change &


continuity?”

3. Argues that “gestalt strategies” exist at the founding of orgs.

260 | P a g e
4. CHANDLER – Well-known edict of STRUCTURE FOLLOWS STRATEGY:

o “Changes in strategy lead to changes in org structure. Structure


should be designed to facilitate the strategic pursuit of a firm,
therefore, STRUCTURE FOLLOWS STRATEGY.

o MINTZBERG’S CRITICISM OF CHANDLER – Structure follows


Strategy must be called into question because of the influence of
bureaucratic momentum on strategy formation. “Every strategy
maker faces an impossible overload of information; as a result he can
have no optimal process to follow.”

o MILLER – “Public orgs can be characterized as low on deliberate


strategy and high on emergent strategy.” “Any manager who is
unable to relinquish intended strategies in order to pursue
emergent strategies is likely to fail.”

5. STRATEGY SHOULD FOLLOW STRUCTURE:

o The org first chooses what services/products they will provide to the
market. Based on that choice, there may exist an already-researched
optimal structure for your org.

o Sometimes facets of structure are mandated by the state (safety


officer, IG).

o At the beginning of any org you start with an idea that is shared
among a group. The group who decides to stay and entertain this
idea further is considered the initial structure. Now they interact and
develop PLANS to proceed. STRATEGY is then dictated from higher in
the structure or developed thru group and team processes. The
results of these processes may necessitate a change in structure as a
result of new strategies.

o PUBLIC SECTOR ARGUMENTS FOR “STRATEGY SHOULD


FOLLOW STRUCTURE” (See Gerard Miller article):

 Applying Chandler’s argument that structure follows strategy,


we see that some agencies begin with a basic structure that
261 | P a g e
may change very little over time (Emergency Management –
structure is dictated by State and strategies are formulated as
program initiatives and work is handed down to the States).
However, in the case of Emergency Management, when the Fed
structure changed (Dept. of Homeland Security), the missions
of State and local agencies changed in response.
STRUCTURES then changed to accommodate new programs
and mandates from above.

 To Summarize: In the public sector, normally strategy follows


structure. Agency structure is relatively stable over time. In
the case of Homeland Security, it was all new, so it was a rare
case where structure followed strategy.

o MINTZBERG – EQUAL FOOTING -

 “Structure follows strategy as the left foot follows the right in


walking. NONE TAKES PRECEDENCE; EACH ALWAYS
PRECEDES THE OTHER, AND FOLLOWS IT, EXCEPT WHEN
THEY MOVE TOGETHER, AS WHEN AN ORG JUMPS TO A
NEW POSITION.”

MINTZBERG – Strategy Formation:

 Strategy formation is not a regular, nicely sequenced process. An org may


find itself in a stable environment for years without ever needing to reassess
strategy. Then suddenly, the environment can become so turbulent that
even the very best planning techniques are of no use because of the
impossibility of predicting the kind of stability that will eventually emerge.
 In response to this kind of environment, patterns of strategic change are
never steady, but irregular and AD HOC.
 GESTALT STRATEGIES – most frequently developed at the time when the
org is founded.
 DELIBERATE, EMERGENT AND REALIZED STRATEGIES.

262 | P a g e
 EVERY STRATEGY MAKER FACES AN IMPOSSIBLE OVERLOAD OF
INFORMATION (MUCH OF IT SOFT); AS A RESULT HE CAN HAVE NO
OPTIMAL PROCESS TO FOLLOW.

Gerald MILLER – Strategy should follow structure.

 “A manager that is unable to relinquish intended strategies in order to


pursue emergent strategies is likely to fail.”
 IN THE PUBLIC SECTOR: SOME BUT NOT ALL STRATEGIES WILL WORK
IN PUBLIC ORGS.
o Strategies often emerge only after the fact.
o Strategies are constrained by forces beyond the control of those
pursuing them.
o Classic Sub-Optimization – Often occurs because the details of a
strategy call for precise evaluation measures. We may end up
OPTIMIZING the measure and fail to achieve the strategy behind the
measure.
 ADHOCRACY - Miller argues that in the public sector, there tends to be
more ADHOCRACY going on, and long-range strategic planning may not be
applicable in most areas of public management.

THIS IDEA (ADHOCRACY) WAS SUPPORTED IN 1986 BY HALACHMI – in his paper


“Strategic Planning and Management? Not Necessarily”, in which he identified a
history of problems related to adopting private-sector management tools in the public
sector.

 HALACHMI – Emergent strategies are most compatible with the adhocracy


configuration, in which many people at all levels are potentially involved in
the establishment of strategies.
 HALACHMI argued that the application of strategic planning to the public
sector is problematic:
o “In government, higher levels may set strategic goals for lower levels
in certain areas, but not in others.”

263 | P a g e
 Reserved powers of the states.
 “Home rule” provisions.
o In the public sector, the basic mission of the org is always determined
externally, by legislative bodies, and it is expressed through laws and
regulation.
o Since legislation is often vague in policy making, strategy often follows
structure in the public sector.
o Halachmi argues that managers in the public sector tend to follow the
“MUDDLING THROUGH” style of management, rather than investing
valuable time and energy into more proactive approaches such as
strategic management.
 Goals are more difficult to measure in public sector – not tied to any single
criterion like bottom-line profits.
 Halachmi adds that many public sector decisions must be made with limited
information, and for reasons of political “capital”, must produce short-term
results. This means that developing long-term plans or strategies is often a
useless exercise in the public sector, except in cases where the primary work
is technological in nature, in which case strategic planning holds promise.

CYERT & MARCH – BEHAVIORAL THEORY OF THE FIRM.

 Individuals do NOT always engage in self-interested maximizing behavior.


 STRATEGY FORMULATION IN THE PRIVATE SECTOR:
o ADAPTIVE MODE:
 Many decision makers with conflicting goals bargain amongst
themselves to produce a stream of incremental, disjointed
decisions. Some of the literature describes this as
“entrepreneurial mode”.

Philip SELZNICK – “Strategies take on value only as committed people infuse them
with energy.”

 COOPTATION – TVA AND THE GRASS ROOTS.

FUTURE MANAGEMENT CHALLENGES IN PA:

264 | P a g e
KETTL (2000) on FUTURE MANAGEMENT CHALLENGES IN PA:

 Kettl: “Americans have always been distrustful of governmental power, and


especially administrative power. They have long believed that public
administration is more inefficient and corrupt than private administration.

 In The Transformation of Governance, Kettl identifies 3 management


challenges for PA:

1. GLOBALIZATION – Instantaneous communication is accelerating govt


transformation.

2. DEVOLUTION – Increased PRIVATIZATION & CONTRACTING-OUT.

3. DEFINING THE NEW ROLE OF GOVERNMENT – In a globalized


world, what capacity does the federal govt need to play?

DENHARDT & DENHARDT (2000) – In The New Public Service: Serving rather than
Steering.

 Argues for a “New Public Service” in which public managers focus on


responsibility to serve and empower citizens while performing their work in
managing public organizations and implementing public policy.
 Denhardt feels that we desperately NEED A RENEWED SENSE OF
COMMUNITY, and the way to do this involves building a new active set of
institutions that serve the public interest.

HALACHMI (2005) – In Governance and Risk Management: Challenges and Public


Productivity.

 CALLS FOR A SHIFT FROM “GOVERNING” TO “GOVERNANCE”.

 Engage private industry and civil society organizations to help address


accountability issues, improve productivity and provide monitoring of
environments.

 Results: improve government response and risk management.

Conclusion to “the FUTURE CHALLENGES FACING PA”:

265 | P a g e
 Increasing regulation is not effective in increasing public productivity or
correcting problems with oversight.

 The best way to approach these problems is to develop a more responsive


and flexible system of GOVERNANCE in which the work of government is
accomplished through partnerships with private industry, civil based society
organizations and individual citizens.

Chris ARGYRIS – SINGLE AND DOUBLE-LOOP LEARNING.

Single-loop and double-loop learning

For Argyris and Schön (1978: 2) learning involves the detection and correction of error.
Where something goes wrong, it is suggested, an initial port of call for many people is
to look for another strategy that will address and work within the governing variables.
In other words, given or chosen goals, values, plans and rules are operationalized
rather than questioned. According to Argyris and Schön (1974), this is single-loop
learning. An alternative response is to question to governing variables themselves, to
subject them to critical scrutiny. This they describe as double-loop learning. Such
learning may then lead to an alteration in the governing variables and, thus, a shift in
the way in which strategies and consequences are framed. Thus, they came to explore
the nature of organizational learning. This is how Argyris and Schön (1978: 2-3)
described the process in the context of organizational learning:

When the error detected and corrected permits the organization to carry on its present
policies or achieve its presents objectives, then that error-and-correction process is
single-loop learning. Single-loop learning is like a thermostat that learns when it is too
hot or too cold and turns the heat on or off. The thermostat can perform this task
because it can receive information (the temperature of the room) and take corrective
action. Double-loop learning occurs when error is detected and corrected in ways that
involve the modification of an organization’s underlying norms, policies and objectives.

This process can be represented quite easily by a simple amendment of our initial
representation of theory-in-use.

266 | P a g e
Single-loop learning seems to be present when goals, values, frameworks and, to a
significant extent, strategies are taken for granted. The emphasis is on ‘techniques
and making techniques more efficient’ (Usher and Bryant: 1989: 87) Any reflection is
directed toward making the strategy more effective. Double-loop learning, in contrast,
‘involves questioning the role of the framing and learning systems which underlie
actual goals and strategies (op. cit.). In many respects the distinction at work here is
the one used by Aristotle, when exploring technical and practical thought. The former
involves following routines and some sort of preset plan – and is both less risky for the
individual and the organization, and affords greater control. The latter is more creative
and reflexive, and involves consideration notions of the good. Reflection here is more
fundamental: the basic assumptions behind ideas or policies are confronted…
hypotheses are publicly tested… processes are disconfirmable not self-seeking (Argyris
1982: 103-4).

The focus of much of Chris Argyris’ intervention research has been to explore how
organizations may increase their capacity for double-loop learning. He argues that
double-loop learning is necessary if practitioners and organizations are to make
informed decisions in rapidly changing and often uncertain contexts (Argyris 1974;
1982; 1990). As Edmondson and Moingeon (1999:160) put it:

The underlying theory, supported by years of empirical research, is that the reasoning
processes employed by individuals in organizations inhibit the exchange of relevant
information in ways that make double-loop learning difficult – and all but impossible
in situations in which much is at stake. This creates a dilemma as these are the very
organizational situations in which double-loop learning is most needed.

267 | P a g e
Peter SENGE – (1990) The Fifth Discipline. The LEARNING ORGANIZATION.

The learning organization

According to Peter Senge (1990: 3) learning organizations are:

…organizations where people continually expand their capacity to create the results
they truly desire, where new and expansive patterns of thinking are nurtured, where
collective aspiration is set free, and where people are continually learning to see the
whole together.

The basic rationale for such organizations is that in situations of rapid change only
those that are flexible, adaptive and productive will excel. For this to happen, it is
argued, organizations need to ‘discover how to tap people’s commitment and capacity
to learn at all levels’ (ibid.: 4).

While all people have the capacity to learn, the structures in which they have to
function are often not conducive to reflection and engagement. Furthermore, people
may lack the tools and guiding ideas to make sense of the situations they face.
Organizations that are continually expanding their capacity to create their future
require a fundamental shift of mind among their members.

When you ask people about what it is like being part of a great team, what is most
striking is the meaningfulness of the experience. People talk about being part of
something larger than themselves, of being connected, of being generative. It become
quite clear that, for many, their experiences as part of truly great teams stand out as
singular periods of life lived to the fullest. Some spend the rest of their lives looking for
ways to recapture that spirit. (Senge 1990: 13)

For Peter Senge, real learning gets to the heart of what it is to be human. We become
able to re-create ourselves. This applies to both individuals and organizations. Thus,
for a ‘learning organization it is not enough to survive. ‘”Survival learning” or what is
more often termed “adaptive learning” is important – indeed it is necessary. But for a

268 | P a g e
learning organization, “adaptive learning” must be joined by “generative learning”,
learning that enhances our capacity to create’ (Senge 1990:14).

The dimension that distinguishes learning from more traditional organizations is the
mastery of certain basic disciplines or ‘component technologies’. The five that Peter
Senge identifies are said to be converging to innovate learning organizations. They are:

Systems thinking

Personal mastery

Mental models

Building shared vision

Team learning

He adds to this recognition that people are agents, able to act upon the structures and
systems of which they are a part. All the disciplines are, in this way, ‘concerned with a
shift of mind from seeing parts to seeing wholes, from seeing people as helpless
reactors to seeing them as active participants in shaping their reality, from reacting to
the present to creating the future’ (Senge 1990: 69). It is to the disciplines that we will
now turn.

Systems thinking – the cornerstone of the learning organization

A great virtue of Peter Senge’s work is the way in which he puts systems theory to
work. The Fifth Discipline provides a good introduction to the basics and uses of such
theory – and the way in which it can be brought together with other theoretical devices
in order to make sense of organizational questions and issues. Systemic thinking is
the conceptual cornerstone (‘The Fifth Discipline’) of his approach. It is the discipline
that integrates the others, fusing them into a coherent body of theory and practice
(ibid.: 12). Systems theory’s ability to comprehend and address the whole, and to
examine the interrelationship between the parts provides, for Peter Senge, both the
incentive and the means to integrate the disciplines.

269 | P a g e
NEO-CLASSICAL vs. INSTITUTIONAL ECONOMICS:

 In NeoClassical, the INDIVIDUAL is the primary unit of analysis.


 In Institutional Econ, individuals operate within socially constructed
frameworks that influence their economic decision making.
 In Neo-Classical, individual is self-interested and makes rational economic
choices.
 Institutional Econ focuses on transactions and behavior between individuals in
the context of collective action.
 In Neo-Classical, individuals shape economic outcomes
 In Institutional Econ, individuals are limited by institutional constraints (govt,
legal, etc.) in their economic decision making.

TRANSACTION COST ECONOMICS:

 RONALD COASE (1937) – first asked “Why do organizations exist?” and


answered that “Sometimes the cost of managing economic exchanges across
markets is greater than the cost of managing economic exchanges WITHIN THE
BOUNDARIES OF AN ORGANIZATION.”

 WILLIAMSON (1975) – Suggests that an internal hierarchical structure may


serve as an effective arena of exchange to reduce transaction costs.

 ASSUMPTIONS OF TCE:

o Bounded Rationality (actor has limited info, cannot predict future –


departure from classical and neo-classical econ theories which assume
actor is “rational”.

o Opportunism (Self-Interested, Cheating or lying, Withholding


information)

AGENCY THEORY – aka PRINCIPAL-AGENT THEORY:

 Addresses how actors manage their respective risks in an exchange.


 Problems with Agency Theory – Moral Hazard (agent does not complete task-
shirking), Adverse Selection

270 | P a g e
MOE (1984) – LACK OF ECONOMIC ANALYSIS IN PUBLIC ORGANIZATIONS

 MOE argues that economic analysis of public organizations has received little
attention in the literature and suggests three primary areas in which economic
analysis could be applied:
o A contractual perspective on organizational relationships
o A focus on hierarchical control
o Formal analysis via principal-agent models

MOE (1987) – Exploring the Limits of Privatization.

 Need for mutual coexistence and links between public and private sectors.
 Moe views limits to privatization from a legal context (vs. economic or
managerial)
 PA should be based on a legal foundation and the concept of sovereignty.

STRATEGIC MANAGEMENT DEFINED – The ART & SCIENCE of formulating,


implementing and evaluating cross-functional decisions that enable an org to achieve
its objectives.

STRATEGIC PLANNING & PRINCIPAL-AGENT THEORY:

 To assume that when you have a strategic plan, you have a roadmap, is wrong.
Principal-Agent relationships and people working for their own interests can
change how strategic plans work out.

LINDBLOM - STRATEGY FORMULATION IN THE PRIVATE SECTOR:

 ADAPTIVE MODE:
o Many decision makers with conflicting goals bargain amongst themselves
to produce a stream of incremental, disjointed decisions. Some of the
literature describes this as “entrepreneurial mode”.

WALDO

 Waldo criticizes both the emphasis on supposed principles or commonalities


among organizations and the rationalist bias of Classical organization theory.

271 | P a g e
Organizations should be defined and structured to meet purposes, not general
principles, and the organizational form and process actually adopted should be
suited to the specific situation confronting the organization.

Graham ALLISON – 3 MODELS OF DECISION MAKING. (Know for Comps – get article)

 Allison is best known as a political scientist for his book Essence of Decision:
Explaining the Cuban Missile Crisis (1971), in which he developed two new
theoretical paradigms - an organizational process model and a bureaucratic
politics model - to compete with then-prevalent approach of understanding
foreign policy decision making using a rational actor model. Essence of Decision
swiftly revolutionized the study of decision making in political science and
beyond.
 Essence of Decision: Explaining the Cuban Missile Crisis is an analysis, by
political scientist Graham T. Allison, of the Cuban Missile Crisis. Allison used
the crisis as a case study for future studies into governmental decision-making.
The book became the founding study of the John F. Kennedy School of
Government, and in doing so revolutionized the field of international relations.

When he first wrote the book, Allison contended that political science and the study of
international relations were saturated with rational expectations theories inherited
from the field of economics. Under such a view, the actions of states are analyzed by
assuming that nations consider all options and act rationally to maximize their utility.

Allison attributes such viewpoints to the dominance of economists such as Milton


Friedman, statesmen such as Robert McNamara and Henry Kissinger, disciplines such
as game theory, and organizations such as the RAND Corporation. However, as he
puts it:

It must be noted, however, that an imaginative analyst can construct an account


of value-maximizing choice for any action or set of actions performed by a
government.

272 | P a g e
Or, to put it bluntly, this approach (which Allison terms the "Rational Actor Model")
violates the law of falsifiability. Also, Allison notes that "rational" analysts must ignore
a lot of facts in order to make their analysis fit their models.

In response, Allison constructed three different ways (or "lenses") through which
analysts can examine events: the "Rational Actor" model, the "Organizational
Behavior" model, and the "Governmental Politics" model.

To illustrate the models, Allison poses the following three questions in each section:

1. Why did the Soviet Union decide to place offensive missiles in Cuba?

2. Why did the United States respond to the missile deployment with a blockade?

3. Why did the Soviet Union withdraw the missiles?

The "Rational Actor" Model

The origin of Allison's first model is explained above. Basically, under this theory:

 Governments are treated as the primary actor.

 The government examines a set of goals, evaluates them according to their


utility, then picks the one that has the highest "payoff."

Under this theory, Allison explains the crisis like this:

1. John F. Kennedy, in 1961, revealed that the Soviet Union, despite rhetoric, had
far fewer ICBMs than it claimed. In response, Nikita Khrushchev ordered
nuclear missiles with shorter ranges installed in Cuba. In one move, the Soviets
bridged the "missile gap" while scoring points in the Cold War. Based on
Kennedy's failure to back up the Bay of Pigs invasion, they believed the U.S.
wouldn't respond harshly.

2. Kennedy and his advisors (EXCOMM) evaluated a number of options, ranging


from doing nothing to a full invasion of Cuba. A blockade of Cuba was chosen
because it wouldn't necessarily escalate into war, and because it forced the
Soviets to make the next move.
273 | P a g e
3. Because of mutually assured destruction by a nuclear war, the Soviets had no
choice but to bow to U.S. demands and remove the weapons.

The Organizational Process Model

Allison noted there were many facts that the rational model had to ignore, such as
why the Soviets failed to camouflage the nuclear sites during construction, but did so
only after U-2 flights pinpointed their locations.

He cited work by James G. March and Herbert Simon, which argue that existing
governmental bureaucracy places limits on a nation's actions, and often dictates the
final outcome. He then proposed the following "organizational process" model
propositions:

 When faced with a crisis, government leaders don't look at it as a whole, but
break it down and assign it according to pre-established organizational lines.

 Because of time and resource limitations, rather than evaluating all possible
courses of action to see which one is most likely to work, leaders settle on the
first proposal that adequately addresses the issue, which Simon termed
"satisficing."

 Leaders gravitate towards solutions that limit short-term uncertainty (emphasis


on "short-term").

 Organizations follow set "repertoires" and procedures when taking actions.

 Because of the large resources and time required to fully plan and mobilize
actions within a large organization (or government), leaders are effectively
limited to pre-existing plans.

Under this theory, the crisis is explained thus:

1. Because the Soviets never established nuclear missile bases outside of their
country at the time, they assigned the tasks to established departments, which
in turn followed their own set procedures. However, their procedures were not

274 | P a g e
adapted to Cuban conditions, and as a result, mistakes were made that allowed
the U.S. to quite easily learn of the program's existence. Such mistakes
included such gaffes as supposedly undercover Soviet troops decorating their
barracks with Red Army Stars viewable from above.

2. Kennedy and his advisors never really considered any other options besides a
blockade or air strikes, and initially, were almost unanimously in favor of the
air strikes. However, such attacks created massive uncertainty because the
U.S. Air Force couldn't guarantee it would disable all the nuclear missiles.
Additionally, although Kennedy wanted a "surgical" air strike that would
destroy the missiles without inflicting extensive damage, the existing Air Force
plan required extensive bombing that would have created more collateral
damage than Kennedy desired. Because the U.S. Navy already had considerable
strength in the field, because there was a pre-existing plan in place for a
blockade, and because Kennedy was able to communicate directly with the
fleet's captains, members fell back on the blockade as the only safe option.

3. The Soviets simply did not have a plan to follow if the U.S. took decisive action
against their missiles. Khrushchev's communications indicated a high degree of
desperation. Without any back-up plan, the Soviets had to withdraw.

The "Governmental Politics" Model

After reading works by Richard Neustadt and Samuel P. Huntington, among others,
Allison proposed a third model, which takes account of court politics (or "palace
politics"). While statesmen don't like to admit they play politics to get things done,
especially in high-stakes situations such as the Cuban missile crisis, they nonetheless
do.

Allison proposed the following propositions for this model:

 A nation's actions are best understood as the result of politicking and


negotiation by its top leaders.

 Even if they share a goal, leaders differ in how to achieve it because of such
factors as personal interests and background.

275 | P a g e
 Even if a leader holds absolute power (i.e., the President of the United States is
technically the commander-in-chief), the leader must gain a consensus with his
underlings or risk having his order misunderstood or, in some cases, ignored.

 Related to the above proposition, the make-up of a leader's entourage will have
a large effect on the final decision (i.e., an entourage of "yes men" will create a
different outcome than a group of advisors who are willing to voice
disagreement).

 Leaders have different levels of power based on charisma, personality, skills of


persuasion, and personal ties to decision-makers.

 If a leader is certain enough, they will not seek input from their advisors, but
rather, approval. Likewise, if a leader has already implicitly decided on a
particular course of action, an advisor wishing to have influence must work
within the framework of the decision the leader has already made.

 If a leader fails to reach a consensus with his inner circle (or, at least, the
appearance of a consensus), opponents may take advantage of these
disagreements. Therefore, effective leaders must create a consensus.

 Because of the possibilities of miscommunication, misunderstandings, and


downright disagreements, different leaders may take actions that the group as a
whole would not approve of.

Allison had to admit that, because the Soviets were not as open with their internal
affairs as the Americans, he simply didn't have enough data to fully interpret the crisis
with this model. Nonetheless, he made the following attempt:

1. Khrushchev came under increasing fire from the Presidium because of


Kennedy's revelation of the Soviet lack of ICBMs, as well as American successes
in the Berlin Airlift. Also, the Soviet economy was being stretched, and military
leaders were unhappy with Khrushchev's decision to cut the size of the Red
Army. Placing missiles in Cuba was a cheap and quick way for him to secure
his political base.

276 | P a g e
2. Because of the failure of the Bay of Pigs invasion, Republicans in the Congress
made Cuban policy into a major issue for the upcoming congressional elections
later in 1962. Therefore, Kennedy immediately decided on a strong response
rather than a diplomatic one. Although a majority of EXCOMM initially favored
air strikes, those closest to the president - such as his brother and Attorney
General, Robert Kennedy, and special counsel Theodore Sorensen - favored the
blockade. At the same time, Kennedy got into arguments with proponents of the
air strikes, such as Air Force General Curtis LeMay. After the Bay of Pigs fiasco,
Kennedy also distrusted the CIA and its advice. This combination of push and
pull led to the implication of a blockade.

3. With his plans thwarted, Khrushchev tried to save face by pointing to American
missiles in Turkey, a position similar to the Cuban missiles. While Kennedy
refused to move these missiles "under duress," he allowed Robert Kennedy to
reach a deal with Soviet ambassador Anatoly Dobrynin, in which the Turkish
missiles (which Kennedy ordered removed prior to the crisis) would be quietly
removed several months later. Publicly, Kennedy also agreed never to invade
Cuba.

Implications

When the book was first published, Allison's primary message was that the concept of
mutually assured destruction as a barrier to nuclear war was unfounded. By looking
at organizational and political models, such an outcome was quite possible - nations,
against what was predicted by the rational viewpoint, could indeed "commit suicide."

He pointed to several incidents in history that seemed to back this assertion. His most
salient point: prior to the attack at Pearl Harbor, Japanese military and civilian
leaders, including those responsible for making the decision, were fully aware that
they lacked the industrial capacity and military might to win a war against the U.S.
They went ahead and attacked anyway.

He also believed that the organizational model explained otherwise inexplicable gaffes
in military history. To return to 1941, he noted that the U.S. intercepted enough
evidence to indicate that Japan was about to attack Pearl Harbor, yet the commander
277 | P a g e
did not prepare. The answer, Allison revealed, was not some conspiracy, but that what
the intelligence community viewed as a "threat of attack," the commander interpreted
as a "threat of sabotage." This miscommunication, due to different viewpoints, allowed
the attack to be pulled off successfully - as Allison sarcastically noted, having U.S.
planes lined up wing-to-wing and surrounded by armed guards was a good plan for
preventing sabotage, but not for surviving an aerial attack.

Likewise, the political process model explained otherwise confusing affairs. Allison
pointed to the decision by General Douglas MacArthur to defy his orders during the
Korean War and march too far north. The reason was not a "rational" change in U.S.
intentions, but rather, MacArthur's disagreements with Harry Truman and other
policymakers, and how officials allowed MacArthur to make what they considered
unwise moves because of concerns over political backlash due to the general's public
popularity.

Above all, he described using rational actor models as dangerous. By using such
models (and modes of thinking), people made unreliable assumptions about reality,
which could have disastrous consequences. Part of what allowed the attack on Pearl
Harbor to be pulled off was the assumption that, since Japan would lose such a war,
they would never dare attack. The assumption under MAD is that nobody will ever
start a nuclear war because of its consequences. However, humans are not
inextricably bound to act in a rational manner, which history has proven time and
time again.

While Allison did not claim that any of his additional two models could fully explain
anything, he noted that policymakers and analysts alike would benefit from stepping
away from the traditional model and exploring alternate viewpoints (although this last
remark could be viewed as facetious on Allison's part).

Difference between: Classical Economics & Institutional Economics.

KORTEN (1984) – Developed the concept of STRATEGIC ORGANIZATION.

278 | P a g e
Traditional models of organization are not well-suited for the future and
STRATEGIC MANAGEMENT represents a positive alternative, one that carries
with it a “proactive commitment to the ideal that the purpose of organization is
to serve the needs of people, while facilitating the human growth of all
participants.

MOE (1987) – Exploring the Limits of Privatization.

 Need for mutual coexistence and links between public and private
sectors.

 Moe views limits to privatization from a legal context (vs. economic or


managerial)

 PA should be based on a legal foundation and the concept of


sovereignty.

HISTORY OF FINANCIAL MANAGEMENT REFORMS: See Jones, R.L. and McCaffrey


1999 – “Financial Management Reform in the Federal Government” In Roy T. Meyers
ed. (1999) Handbook of Government Budgeting. San Francisco, Jossey-Bass Inc.
Publishers, 53-81.j

 The DOCKERY ACT OF 1894 – Established and strengthened centralized


accounting function, creating a single comptroller in the Treasury Dept.

 BUDGET AND ACCOUNTING ACT OF 1921 – Created a centralized executive


budget, requiring the president to submit unified budget annually to Congress.
The effect was to separate budgeting, accounting and auditing and establish an
independent auditing arm of Congress.

 The ECONOMY ACT OF 1932 – Installed accounting systems, forms and


procedures.

 The REORGANIZATON ACT OF 1939 – Moved BOB from Treasury to the


Executive Office of the President (EOP) to assist in the preparation and
execution of the budget. BOB changed to OMB.

279 | P a g e
 BUDGET AND ACCOUNTING PROCEDURES ACT OF 1950 – Established
mandate for an adequate system of accounting and internal controls on head of
each executive agency. Amended in 1956 after 2nd Hoover Commission – called
for cost-based budgeting for federal govt.

 BUDGET AND IMPOUNDMENT ACT OF 1974 – Moved federal fiscal year to


begin October 1, created the CBO, added budget committees to both houses.

 INSPECTOR GENERAL ACT OF 1978 – Established IG offices in departments


and federal agencies. In charge of conducting audits and investigations of
executive offices and major independent agencies.

 DEBT COLLECTION ACT OF 1982 - strengthened fed govt’s ability to collect


monies owed it. Prompt Payment Act required fed govt to pay bills within 30
days.

AUDITS, IG ACTIONS AND INCREASED REPORTING ADDRESSED WEAKNESSES IN


FEDERAL SYSTEMS. HOWEVER, DEPARTMENTS FAILED TO TAKE CORRECTIVE
ACTIONS AND SCANDALS CONTINUED.

 CHIEF FINANCIAL OFFICERS ACT OF 1990 – Intended to bring the budget and
accounting functions together and to centralize financial management functions
at dept and agency levels with a CFO. OFFM – Office of Federal Financial
Management was created to assist with CFO task.

 GOVERNMENT PERFORMANCE RESULTS ACT (GPRA) 1993 – A product of


Gore’s NATIONAL PERFORMANCE REVIEW. Idea came from OSBORNE &
GAEBLER’s 1992 REINVENTING GOVERNMENT. Called for agency
STRATEGIC PLANNING and FOCUSED ON RELATIONSHIPS BETWEEN
MISSION AND OUTCOMES.

PORTER – 5 Forces

MAZLOW - Hierarchy

DRUCKER – MBO

Friedrich Hayek – (“Old Econ=Keynsian, New Econ=Hayek – Will Be On Comps)

280 | P a g e
The economic calculation problem

Hayek was one of the leading academic critics of collectivism in the 20th century.
Hayek believed that all forms of collectivism (even those theoretically based on
voluntary cooperation) could only be maintained by a central authority of some kind.
In his popular book, The Road to Serfdom (1944) and in subsequent works, Hayek
claimed that socialism required central economic planning and that such planning in
turn had a risk of leading towards totalitarianism, because the central authority would
have to be endowed with powers that would have an impact on social life as well, and
because the knowledge required for central planning is inherently decentralized.

Building on the earlier work of Mises and others, Hayek also argued that while, in
centrally planned economies, an individual or a select group of individuals must
determine the distribution of resources, these planners will never have enough
information to carry out this allocation reliably. The efficient exchange and use of
resources, Hayek claimed, can be maintained only through the price mechanism in
free markets (see economic calculation problem). In The Use of Knowledge in Society
(1945), Hayek argued that the price mechanism serves to share and synchronize local
and personal knowledge, allowing society's members to achieve diverse, complicated
ends through a principle of spontaneous self-organization. He used the term catallaxy
to describe a "self-organizing system of voluntary co-operation."

In Hayek's view, the central role of the state should be to maintain the rule of law,
with as little arbitrary intervention as possible.

Spontaneous order

Hayek viewed the free price system, not as a conscious invention (that which is
intentionally designed by man), but as spontaneous order, or what is referred to as
"that which is the result of human action but not of human design". Thus, Hayek put
the price mechanism on the same level as, for example, language. Such thinking led
him to speculate on how the human brain could accommodate this evolved behavior.
In The Sensory Order (1952), he proposed, independently of Donald Hebb, the
connectionist hypothesis that forms the basis of the technology of neural networks
and of much of modern neurophysiology.
281 | P a g e
Hayek attributed the birth of civilization to private property in his book The Fatal
Conceit (1988). He explained that price signals are the only means of enabling each
economic decision maker to communicate tacit knowledge or dispersed knowledge to
each other, in order to solve the economic calculation problem.

Investment and choice

Perhaps more fully than any other economist, Hayek investigated the choice theory of
investment involving the inter-relations between non-permanent production goods and
"latent" or potentially economic permanent resources, building on the choice
theoretical insight that, "processes that take more time will evidently not be adopted
unless they yield a greater return than those that take less time." Hayek's work on the
microeconomics of the choice theoretics of investment, non-permanent goods,
potential permanent resources, and economically adapted permanent resources mark
a central dividing point between Hayek's work on central planning, trade cycle theory,
the division of knowledge, and entrepreneurial adaptation and that of most all other
economists, most especially that of the macroeconomic "Marshallian" economists in
the tradition of John Maynard Keynes and the microeconomic "Walrasian" economists
in the tradition of Abba Lerner.

The business cycle

Capital, money, and the business cycle are prominent topics in Hayek's early
contributions to economics. Mises had earlier explained monetary and banking theory
in his Theory of Money and Credit (1912), applying the marginal utility principle to the
value of money and then proposing a new theory of industrial fluctuations based on
the concepts of the British Currency School and the ideas of the Swedish economist
Knut Wicksell. Hayek used this body of work as a starting point for his own
interpretation of the business cycle, which defended what later became known as the
"Austrian Theory of the Business Cycle". In his Prices and Production (1931) and The
Pure Theory of Capital (1941), he explained the origin of the business cycle in terms of
central bank credit expansion and its transmission over time in terms of capital
misallocation caused by artificially low interest rates.

Social and political philosophy


282 | P a g e
In the latter half of his career Hayek made a number of contributions to social and
political philosophy, which he based on his views on the limits of human knowledge,
and the idea of spontaneous order in social institutions. He argues in favor of a society
organized around a market order, in which the apparatus of state is employed almost
(though not entirely) exclusively to enforce the legal order (consisting of abstract rules,
and not particular commands) necessary for a market of free individuals to function.
These ideas were informed by a moral philosophy derived from epistemological
concerns regarding the inherent limits of human knowledge.

In his philosophy of science, which has much in common with that of his good friend
Karl Popper, Hayek was highly critical of what he termed scientism: a false
understanding of the methods of science that has been mistakenly forced upon the
social sciences, but that is contrary to the practices of genuine science. Usually
scientism involves combining the philosophers' ancient demand for demonstrative
justification with the associationists' false view that all scientific explanations are
simple two-variable linear relationships. Hayek points out that much of science
involves the explanation of complex multi-variable and non-linear phenomena, and
that the social science of economics and undesigned order compares favorably with
such complex sciences as Darwinian biology. These ideas were developed in The
Counter-Revolution of Science: Studies in the Abuse of Reason, 1952 and in some of
Hayek's later essays in the philosophy of science such as "Degrees of Explanation" and
"The Theory of Complex Phenomena".

283 | P a g e
2.7 Important Comps Questions and Student Answers
Administrative Management

What are the major approaches to measuring service quality? How applicable are they
to the case of public agencies?

There are a number of approaches to measure service quality and many are
appropriate for public organizations. While there are great similarities between public
and private organizations, there are several methods to measure service quality used
by the private sector that are clearly not compatible with public organizations.

“Early work on municipal service quality assessment recommended multiple measures


of performance from both providers and users. Citizen satisfaction surveys have
rivaled their more quantitative counterpart, administrative performance measures, in
adoption, but the implication of survey results for action is not well understood by
managers or scholars. To achieve meaningful integrated multiple measures of service
quality, we need to explore the dimensions of citizen satisfaction and review patterns
of satisfaction across localities. We also need to understand the relationship between
administrative performance measures and citizen perceptions. This cross sectional
analysis of municipal citizen satisfaction and performance benchmark data suggests
that citizen satisfaction survey results are useful to managers in conjunction with
performance-measurement programs as part of a multiple-indicator approach to
evaluating municipal service quality. However, understanding citizen perceptions
requires a different perspective than that applied to administrative service
performance measurement.”

(Kelly, J.M., Swindell, D. (2002). A multiple-indicator approach to municipal


service evaluation: correlating performance measurement and citizen satisfaction
across jurisdictions. Public Administration Review, 62, 5, 610-621.)

Other approaches to measuring service quality are:

 Customer Interaction – works for the public sector

 Self Assessments – works for the public sector

 Mystery Shopping (now here’s an idea for a public service!)


284 | P a g e
 Organizational Report Cards

Fred David (p.317-320) introduces us to a number of useful


methods to measure service/performance. For example,
comparing expected results to actual results, investigating
deviations from plans, evaluating individual performance, and
examining progress being made toward meeting stated objectives.
These are viable measures for private and public agencies. Other
procedures commonly used for measuring service quality include:
1) comparing the agency’s performance over different time periods,
2) comparing the agency’s performance to competitors’ (this seems
particularly useful with contracting out or privatization, that is,
government comparing their performance with a private entity), 3)
comparing the agency’s performance to industry averages. These
are all good measures and can be analyzed with both quantitative
and qualitative methods.

Robert K. Yin reminds us that the case study method is best used when seeking the
“how” or “why” questions asked about a contemporary set of events, over which the
investigator has little or no control. Actually, Yin recommends the use of the dual
approaches of case study and survey methods. Therefore, the case study is a
superlative method of measuring service quality. Yin (p.8) relates that one particular
method, the randomized field trial, was designed for evaluation research and is
commonly used. While this method works well for both public and private
organizations, Yin points out that randomized field trials do not work well in a number
of situations.

A related article that is excellent for this and other public administration topics to
describe the differences between public and private organizations is:

Rainey, H.G., Backoff, R. W., Levine, C.H. (1976). Comparing public and private
oganziations. Public administration Review, March/April, 233-244.

Of importance, is page 240, section III, 3. Performance Characteristics. This short


section discusses the observations of Dahl, Lindblom, Golembieski, and Downs

285 | P a g e
concerning the performance of government organizations and administrators, usually
in comparison to business.

Performance assessment is often implemented as a series of specific performance


measures associated with certain service functions or as indicators of progress toward
a mission, as in the case of service efforts and accomplishments. These may be
thought of as internal measures of service quality because they come from a definition
of effectiveness derived and monitors by administrators. In contrast, external
measures of service effectiveness come from citizens, usually through satisfaction
surveys. Results from these kinds of measures are problematic for administrators
because it is unclear what criteria the citizens are using to evaluate effectiveness. Also
questionable is how much accurate information citizens have on which to base their
evaluations.

Administrative performance measures are widely accepted as reliable indicators of


service quality though there is considerable variation in the types of measures
employed and how local governments use them. There is not so much consensus on
the utility of citizen satisfaction surveys as a reliable indictor of government
performance, which makes this topic a bit more interesting.

Describe no less than five (5) approaches/definitions/models of strategic


planning and highlight their possible advantages and limitations in general and
when applied to the public sector.

Mintzberg, Henry and Quinn, James Bryan Readings in the Strategy Process

1. The Entrepreneurial Organization---p. 244 (Mintzberg)

* simple structure
* one flamboyant leader
* little staff
* the leader creates the strategy and can adapt it as he/she deems
necessary

2. The Machine Organization---p. 265 (Mintzberg)

286 | P a g e
* more complex structure

* routine work (i.e., the postal service)

* highly standardized work processes

* very regulated, bureaucratic structure

3. The Professional Organization---p. 288 (Mintzberg)

* complex work (i.e., a university; a hospital)

* work must be carried out by professionals

* but stability is important

* standard operating procedures are used

4. The Innovative Organization---p. 309 (Mintzberg)

* must be capable of sophisticated innovation (i.e., a research firm)

* adhocracies---matrix organizations (move people around a lot)

* highly organic structure

* flexibility is the key

* experts are grouped in functional units and then deployed to teams as


needed

5. The Diversified Organization---p. 335 (Mintzberg)

* the most complex structure

* a set of semi-autonomous units called divisions

* very common in the private sector

* each unit is relatively free form headquarters control

The Model Approach --- the structure is most important

287 | P a g e
Chafee, Ellen Earle presents three models of strategy:

* Linear---rational, logical / closed system

* Adaptive---monitor the environment / open system

* Interpretive---social contracts based on free will / open system

Dutton, Jane

Categorization theory---merely labeling an event as an opportunity or a threat


affects

information processing and motivation

Ginter, Robert

Social learning theory---behavior results from interactions of persons and


Situations

Hart, Stuart

Integrative framework---focuses on integrating the 5 roles played by top


managers: command, symbolic, rational, transactive, and generative

The Process Approach---the process is most important

Mintzberg, David----strategic thinking (synthesis, creativity, intuition) is as


important as strategic planning. We need both. Mintzberg notes 3 fallacies of
planning: prediction, detachment, and formalization.

Comps Question from Syracuse University:

Pine Mountain State University

On the basis of your readings and class discussions offer an analysis of the following
case and assess the President's approach for dealing with the unfolding situation.

The administrative staff at Pine Mountain State developed a comprehensive


reorganization plan to combat specific issues of concern to the President of the

288 | P a g e
university. However, it is evident that there was no formal systematic approach to the
analysis or their strategy formulation. In addition, they neglected to include the major
stakeholders in the strategy planning and development process. This may explain why
the legislature, trustees, faculty and students are complaining about severe problems
that did not exist prior to the reorganization.

It can be argued that strategy follows structure in the typical public university
scenario. Since federal and state funding for public universities comes with
requirements, the underlying structure of a public university tends to follow a
relatively standard organizational model that serves the requirements dictated in part
by the funding authorities. In the case of Pine Mountain State, it appears that the
administration’s top levels agreed upon a new strategy, and then designed a structure
around these objectives. Instead of keeping the initial structure, which successfully
fueled the growth of the university for the last 30 years, the administration chose to
abandon it to focus mainly on external threats from the environment.

As we know from the SWOT Matrix, there are more than just external threats to be
analyzed in formulating strategy. The reading says that “after extensive study, the top
administrative staff of Pine Mountain developed a comprehensive reorganization
plan…” However, we are not told what type of analysis was employed, or if any
consideration was given to the university’s strengths, weaknesses or opportunities. By
focusing only on external threats, the administration is risking the loss of the
synergies and efficiencies enjoyed by various existing internal organizational
components.

We see from the case reading that the strategy adopted by the administration resulted
in several detrimental effects to the university (as witnessed by a freeze in additional
funding allocations from the legislature based on criticism of the new “heavy admin”
structure, and complaints from trustees, students and faculty concerning
deteriorating service levels university-wide). These problems may have arisen
because of the manner in which the reorganization was planned and managed. A
comprehensive reorganization is bound to have a disruptive effect on operations
throughout the university, as well as having detrimental effects on everyone in the

289 | P a g e
organization. However, these detrimental effects can be buffered by proper planning
that involves all interested parties.

The case noted that “the role of most of the units was redefined”. If the roles were
redefined specifically to address the external threats and no attempt was made to keep
successful structures in place, then problems will naturally arise as a result of poor
planning. Role ambiguity, combined with unexpected changes in departmental
processes may explain some of the service complaints reported by students and
faculty. New and unexpected reporting relationships can also cause stress and reduce
the positive effects of a reorganization plan.

Stakeholders

The main criticism of the President’s approach for dealing with the unfolding situation
is that he did not include the major stakeholders in the reorganization planning
process.

It is evident that the top administration was only thinking of themselves and the
problems that they face on a day-to-day basis, rather than adopting a university-wide
vision for the reorganization.

The legislature and trustees would have informed the administration that there is too
much spending in the administrative category, and that the proposed structure is not
appropriate and not in line with the structures of other similar institutions. The
faculty would have lobbied to reduce the levels of administration, thus ensuring that
access to the President and administration is not too problematic. Students and
faculty would have argued against the administration’s plans to cut funding for new
professors and parking in favor of enhancing the funding at the top administrative
levels.

Organizational Development Committee

The committee was established to monitor and receive feedback on the implementation
of the new structure. The committee was formed at the time of reorganization but has
only met three times in the past 18 months. It appears that this committee was the
“window dressing” for the reorganization effort. It sounds like an effective piece of the
290 | P a g e
new structure, but will have no effect unless the committee is run with a standard set
of rules and expectations. This would include a regular meeting schedule and
enforced reporting requirements for the committee (monthly, quarterly, etc.).

Vision/Mission Statements

The objectives which fed-into the creation of the Pine Mountain reorganization plan
were simply the issues currently being faced by the top administrative level of the
organization. In a “true” reorganization plan, there ideally will be a refinement or
reworking of the organization’s vision and mission statements. This drives the rest of
the process, as structure and strategy tie directly into the mission of the organization.
In the case of Pine Mountain State, the mission being pursued is simply that of the top
administration, neglecting the mission of the overall university and the other parts of
the organization.

Finance, Accounting and Technology

Since the reorganization was aimed at addressing the problems of declining


allocations, improving financial responsibility and accountability, and improving
internal operations of the university, the administration should have started by
instituting a new accounting/budgeting system. Since the state in which Pine
Mountain State resides is also being affected severely by an on-going national
recession, this is another reason for instituting a new university-wide accounting and
budgeting system. The President should be focusing on running the organization in a
more “lean” manner than that evidenced by the latest university organization chart,
which is heavy in the administrative layers (“Pine Mountain State spends for
administration twice the amount spent by the other major state university and four
times the amount spent by a teacher’s college”). The case for adoption of a new
accounting system is strengthened by the fact that the reorganization plan established
several new units and redefined the roles of most other units. With the adoption of a
new accounting/budgeting system, people can more easily manage the transitions to
new budgeting and reporting relationships. New technology can also make
restructuring and reform efforts more bearable for all by buffering the disruptive
effects with the positives associated with improved information resources.

291 | P a g e
Conclusion

Pine Mountain State University used no formal systematic approach to analyze their
organization’s current problems or their strategy formulation. The administration
neglected to include the major stakeholders in the strategy planning and development
process. There was no attempt to develop new vision/mission statements to
accompany the reorganization effort, and with the backdrop of severe financial
pressures, the administration should have started the reorganization process by
instituting a new accounting/budgeting system. This case is a perfect example of
what can happen when organizations undertake reorganization efforts without
performing the necessary work involved to ensure the success of the efforts. By short-
changing the strategy formulation process, they ended up short-changing the
organization, all of its stakeholders and its future prospects for success.

292 | P a g e
Chapter 3
Organization Theory

MPA Comprehensive Exam


Study Guide
3.1 Perspectives of Organizational Theory

Four Major Perspectives

1. Classical (1900s-)

Industrialism - Adam Smith - father of capitalism – 1776 - Wealth of Nations -


invisible hand - div of labor (e.g., pin factory)

Post industrialism - society and organization - Wal Mart/Saturn (no Boundaries)

Sociological stream

Emile Durkheim – sociologist - explained structural shifts from age to industrial


organizations - informal orgs focus on workers social needs

Max Weber - he liked law/structure father of bureaucracy - it is a way to rationalize


the social environment - formal rationality (means or techniques) and substantive
rationality (ends or goal)-formal rationality w/o substantive rationality leads to an iron
cage-making man a cog in a machine

Karl Marx - theory of capital-inherent antagonism between capitalists and workers


over how to divide surplus value-workers are alienated so they must organize

Classical Management stream

Frederick Taylor - father of scientific management - he attacked soldiering (workers


limiting their output on purpose - time/motion studies - one best way promoted
rationalization in orgs

Henri Fayol - span of control - “departmentation”-unit of control – hierarchy - espirit


de corps

Chester Barnard-expanded Durkheim’s informal org-integrated goals and motivation-


contributed more to org behavior than org theory

2. Modern (1950s-)

Kenneth Boulding - hierarchy of systems - anything w/ interrelated parts

294 | P a g e
 A control or cybernetic system uses feedback (e.g., a thermostat)

 A closed system does not require additional input to operate

 An open system depends on the environment for inputs to operate

Network analysis—looks at the complex web of relationships of how the org interacts
with other orgs and with its environment

The org and its environment are totally separate - there are boundaries

The general environment - social, cultural, legal, political, economic, technological and
physical components

The international and global environments also impact the org

“Buffering”-protecting the internal operations of an org from interruption by


environmental shocks such as material, labor and capital shortages

Environmental scanning is done to protect against these threats Jeffrey Pfeffer and
Gerald Salancik-Resource Dependent Theory

An org is vulnerable because of its need for resources (raw material, labor, capital,
equipment) from its environment --so the org is controlled by its environment

So we analyze the org by starting with the resources it needs and tracing them to their
source-also look at the org’s competitors for the same resources

Michael Hannan, John Freeman, Howard Aldrich - Population Ecology Theory

Orgs are dependent on the environment for resources but this theory focuses on
patterns of success and failure among all orgs - not just one- it is survival of the fittest

 variation-changes in orgs

 selection-orgs choose certain characteristics

 retention-some survival

295 | P a g e
Philip Selznick-Institutional Theory

Orgs adapt to the values of external society-i.e, when actions are repeated and given
similar meanings by self and others (Richard Scott) this is institutionalization-can lead
to “rationalized myths”.

 Rational decision making

 Define the problem

 Generate and evaluate alternatives

 Select an alternative

 Implement

 Monitor

 Evaluate

Herbert Simon-bounded rationality

When making a decision, decision makers often have

 incomplete and imperfect information

 complex problem

 limited human ability

 time pressure

 conflicting preferences

The Garbage Can Model

 The decision making process is very random

 Actors move in an out

 Problems, participants, solutions are all independent

 Thrown into the garbage can at random

296 | P a g e
Power and Politics—Jeffrey Pfeffer

 Strategy process

 Rational model-SWOT analysis-look at the org’s core competencies

 Strategy formulation precedes implementation—it is top down

 Emergent strategies can be bottom up

 Goals-interrelated with strategies

 Official-may be vague

 Operative-more specific

3. Symbolic Interpretive (1980s-)

Karl Weick-enactment theory-when you use concepts (i.e., organizations) you create
the thing you’re seeking to study-he is not pragmatic- he is an interpretivist

Conditions in the environment can’t be separated from the perception of those


conditions

4. Postmodern (1990s-)

 be careful, many of them wouldn’t like being put in a category

 this term includes a large variety of ideas-the key here is diversity

 it is relativistic-it abandons notions of universal truth-but it has some


standards

 fragmentation is a key theme-breakdowns in family, community & society

o and threats to self identity are caused by tying to play so many roles with
little separation between them

 the future will see smaller, more decentralized and informal orgs causing us to

297 | P a g e
o face more ambiguity than ever-helping (and forcing) us to adapt to more
and more change-the paradox is that science has created the means of
sharing information so quickly, making orgs all the more unpredictable
to prepare for the post modern world we must take nothing for granted -
deconstruct everything!

Philosophy includes (see Burrell and Morgan, Sociological Paradigms and Org Analysis
1979) epistemology (p.47)-how we know the world-the process by which we obtain
knowledge

 objectivists-positivists and empiricists-independent observation is required

 subjectivists-anti-positivists and idealists-all knowledge is filtered through the


observer

 a third position-the process is greatly influenced by cognitive, social and


cultural forces-language is very important-postmodern

 ontology-what can be known (the kinds of things that exist)

298 | P a g e
3.2 Organizational Theory Chart
Theory Classical Neoclassical "Modern" Systems Power & Org. Culture Postmo Human
Org. Politics dernis Relations
m

Time frame Thru 1930s 1930s-1950s 1950s on Late 1960s 1960- 60s thru 80s 1980s 1920s-?
80s on

Paradigm Positivist Pos/post-pos Positivist Positivist Post- Interpretavist Post- Interpretavist


Modern Modern
&
Conflict

Ontology Rational Rational Rational Rational Critical Relativist Critical Relativist


Structural Structural Structural Realist Realist

Epistemology Objective Objective Objective Modified Objective Subjective Subjecti Subjective


Objectivist ve

Methodology Experimental Modified/ Experimental/ Experimental Dialogic Dialogic / Dialogic Hermeneutic /


/ Experimental Manipulative / Elitist Elitist / dialectic
Manipulative Defined Defined Transfo
rmative

System Type Closed Semi-open Closed Open Open Open Open Open

Authors Smith, Fayol, March, Blau, Scott, Katz, Kahn, Kotter, Peters, Weick, Boleman, Deal,
Taylor, Selznick, Jaques Burtalanffy Pfeffer Corbelly, Berquis Ott, Follett
Weber, G&U Barnard, Waterman, t,
Simon Sathe, McWhin
Morgan ney,
Prigogin
e,
Stenger
s

Notes simplistic Transition, Return to Drew from power TQM Techni Janis, Follett,
reactionary Classical Neoclass. cal Hawthorne,
revoluti Theory X/Y
on

Classical Organization Theory

 Dominated thought into the 1930s

 Structuralists--focused attention on structure or design of orgs

 Rational and closed systems pursuing the goal of efficiency

 Adam Smith, Henri Fayol, Daniel McCallum, FW Taylor, Max Weber, G&U

 Organizations should work like machines, using people and capital as their
parts

 McCallum, 1856, first modern organization chart

299 | P a g e
 Fayol's organizational principles: technical, commercial, financial, security,
accounting, managerial (greatest emphasis on managerial)

 Taylor's "one best way"

 Gulick & Urwick's POSDCORB

 Often viewed as narrow and simplistic; however, laid a foundation for all future
scholars

Neoclassical Organization Theory

 Transitional theory that revised Classical Theory by adding human element,


1950

 An organization cannot exist outside of its environment

 James March, Philip Selznick, Chester Barnard and Herbert Simon

 Barnard: individuals are what hold the organization together; thus, they must
be educed to cooperate for success to be achieved (persuasion principle)

 Simon: openly and vehemently attacked Classical Theory, said G&U's


principles were merely proverbs, offered idea of satisfice

 Selznick: idea of cooptation

 Cyert and March: alliance-forming and coalitions

"Modern" Structural Organization Theory

 Second half of 20th Century

 Hierarchy, formal rules in place to attain goals

 Sought a return to the Structural Element, retaining the human aspects of


Neoclassicism

 "Modern" in quotations b/c it is used simply to refer to the time period--there is


little substantive difference between the Structuralists in Classical Theory and
this one, other than time frame

300 | P a g e
 Organization efficiency is the essence of organizational rationality, and the goal
of rationality is to increase the production of wealth in terms of real goods and
services

 Peter Blau and Richard Scott: all orgs consist of a formal and an informal
element and it is impossible to understand an org. without knowing each
element

 Buzz words: differentiation, specialization and integration

 Elliott Jaques: remains a lonely defender of the bureaucratic-hierarchy models

Systems Theory

 Rose to dominance in the late 1960s

 Daniel Katz and Robert Kahn: organizations are open systems

 Apply Ludwig Burtalanffy's general systems theory to organizations and use


quantitative tools and techniques to understand complex relationships among
organizational and environmental variables

 (remember inputoutput / blackbox diagram)

 search for order in complex systems, cause-and-effect oriented

 seeks optimal solutions (not "one best way")

 computers, experts, etc. are the tools necessary

 draw heavily from Neoclassicals-bounded rationality and satisficing (Simon) and


cognitive limits (Cyert and March)

Power and Politics Organization Theory

 organizations are viewed as complex systems of individuals and coalitions

 conflict in inevitable and influence is the primary weapon

 organizational goals change with shifts in the balance of power

 John Kotter: differentiate between power resulting from authority and power
resulting from being able to get job done

 Power is aimed in all directions, not just down the hierarchy


301 | P a g e
 Jeffrey Pfeffer: power and politics are fundamental concepts in defining an org

Organizational Culture

 Late 1960s--thru 70s and 80s

 Organizational culture assumes many organizational behaviors and decisions


are in effect predetermined by the patterns of basic assumptions that are held
by the members of the org.

 An org's behavior, cannot be understood and predicted by studying structural


or systemic elements but by studying its organizational culture

o Meaning (reality) is established by and among the people in organizations


(the org, culture)

o Things are not real, perceptions of them are

o People will distort the perceptions of symbols according to the need for
what is symbolized

 In the 80s, organizational culture began appearing in notable works (incl.


Thomas Peters and Robert Waterman, John Corbelly, Vijay Sathe, and Gareth
Morgan)

 TQM and "Reinventing Government" further thrust this movement onto front
pages in the 1980s and 90s

Postmodernism

 Technology and information networks have led to uncertainty and chaos is this
postmodern era

 Just as information is readily available, so is misinformation

 We are frequently seeing that we do not understand cause-effect relationships


despite abundance of information; thus, managers are abandoning their
"modern era" reliance on technical systems, turning instead to information
technology to help them into the postmodern era

 Karl Weick

302 | P a g e
o Technical system: specific set of hardware and software systems that
produce a desired outcome, products of the "modern era," designed to
accomplish desired purposes using known information and existing
technologies

o Technology: refers to the knowledge of cause-and-effect relationships


embedded in machines and methods

 Old, familiar machine analogies no longer apply. Berquist (1993), McWhinney


(1997) and Prigogine and Stengers (1984) suggest fire is most appropriate
analogy for postmodern organizations

1. fire is a second-order change process that is irreversible

2. fire is ephemeral

 Organizations must wrestle with dilemmas about how much to participate in


the information age

o Centralize or decentralize

o Outsource or produce internally

o "Regular" employees or "stringers"

o Sell products or deliver services through established networks or through


the web

o How to maintain what boundaries

Human Relations School (also, Organizational Behavior)

 People are considered to be as important, or more so, than the org itself

 Bolman & Deal (1997): organizations exist to serve humans (not the other way
around)

 Ott: themes are motivation, group behavior, leadership, empowerment

 Hawthorne Effect, Maslow's Hierarchy of Needs, McGregor's Theory X & Y,


Janis' Grouthink

 Most optimistic of all schools - under right circumstances, people and


organizations will grow and prosper together
303 | P a g e
3.3 Organizational Theory Matrix
Key Name Dates Writings Theories Approach

Barnard, 1938 Cooperative systems -- Classical/Classical


Chester individuals consent to Management
authority. stream
Structural/functional -
orgs are cooperative
systems. Workers
require inducements,
orgs compete with
environment for
workers. Expanded
Durkheim's informal
org - integrated goals
and motivation -
contributed more to
org behavior than org
theory.

Boulding, 1950s Hierarchy of systems - Modern


Kenneth anything w/
interrelated parts. A
control or cybernetic
system uses feedback
(e.g., a thermostat). A
closed system does not
require additional
input to operate. An
open system depends
on the environment for
inputs to operate.

Burrell & 1979 Sociological How we know the Postmodern


Morgan, Paradigms world-the process by
Gareth and Org which we obtain
Analysis knowledge.
Objectivists-positivists
and empiricists-
independent

304 | P a g e
observation is
required.
Subjectivists-anti-
positivists and
idealists-all knowledge
is filtered through the
observer. A third
position-the process is
greatly influenced by
cognitive, social and
cultural forces-
language is very
important. Ontology-
what can be known
(the kinds of things
that exist).

Cohen, Garbage-can model Modern


March, & and organized
Olsen anarchies: decision
making process is
random with problems,
participants, and
solutions independent.

Dahl Power in orgs - a social


relationship in which
one actor, A, can get
another actor, B, to do
something B would
otherwise not do.

Daneke On Dialectical process


Paradigmatic causes paradigm
Process formation; based on
experience and
observation.

305 | P a g e
Darwin, Natural selection,
Charles de survival of the fittest,
Tocqueville, evolution
Alexis

Denhardt, 1981 In the New PA - how


Robert B. Shadow of government and orgs
Organization have failed us. Says
ethics and moral
behavior get lost in
technical efficiency.
To improve hierarchy,
leaders must focus on
development of
individual and less on
power of the
organization. Praxis -
critical choices leading
to enlightened action.
A successful org is a
learning org (self-
actualization).
Problem with
bureaucracy is that
preoccupation with
rationality and
efficiency eliminates
moral concerns; org
control inhibits choice.

Durkheim, 1858- Sociologist - explained Classical/Sociologic


Emile 1917 structural shifts from al stream
agricultural to
industrial orgs.
Informal orgs focus on
workers' social needs.
Economic dev through
div of labor threatens
social solidarity-diff v.

306 | P a g e
integ (paradox).

Etzioni Structuralist model -


rational and natural
systems complement
eachother; control
important in both
(rational-hieracrchy
and distribution of
power and natural -
subordinates allow
superiors to have
control); formal and
informal structures are
equally important; org
dilemma - tension
between org needs and
personal needs.

Fayol, Henri 1919 Administrative theory. Classical/Classical


Classical viewpoint Management
puts control as stream
function of manager.
Top-down
rationalization. Span
of control -
"departmentation" -
unit of control -
hierarchy - espirit de
corps.

Frederickso The PA
n, George Theory
Primer

307 | P a g e
Gagliardi Secondary strategies -
model of cultural
change -- instrumental
(management of
external problems of
adaptation and
internal problems of
integration) and
expressive strategies
(symbolic to protect
stability). Cultural
incrementalism -
expands culture to
include new values.

Galbraith, 1977 Increased complexity


Jay in communication
leads to structural
complexity (increased
coordination).

Giddens, "Constitution Objectivist-


Anthony of Society" institutional
perspective.
Structuration theory --
interactions create
structure. Duality-
structure influences
actions and is made
up of actions.
“Dialectic of control”-
subordinates’
willingness to be
managed allows
superordinates to be in
control (remember
structuration).

Gouldner 1954 Conflict models -


managers and
employees see rules

308 | P a g e
differently.

Habermas, 1920s Pre-postmodernist.


Jurgen Sees 2 modes of social
action:
symbolic/communicati
ve (focus on
interaction between
individual norms and
values) and
purposive/rational
(focus on instrumental
action, technical rules
and efficiency).
Idealistic - clear
communication allows
possibility of escaping
bonds of domination.

Hannan, 1950s Population Ecology: Modern


Micheal; ? Orgs are dependent on
Freeman, the environment for
John; & resources but this
Aldrich, theory focuses on
Howard patterns of success
and failure among all
orgs-not just one-it is
survival of the fittest.
Variation-changes in
orgs; selection-orgs
choose certain
characteristics;
retention-some
survival.

Hatch, Mary 1997 Organization Symbolic-


Jo Theory: interprevist/post-
Modern, positivist. Believes in
Symbolic, multiple frames of
and reference - views

309 | P a g e
Postmodern environment as social
Perspectives construction.

Heffron, 1989 Organization


Florence Theory and
Public
Organization
s: The
Political
Connection

Jaffee, 2001 Organization Jaffee likes Scott


David Theory: (rational, natural,
Tension and open-soc. Structure,
Change participants, goals,
technology, the
environment). Jaffee
says orgs don’t have
goals, only humans
have goals. Jaffee
likes Hall’s focus on:
collectivity, boundary,
order, authority,
communication,
membership, goals
and outcomes of orgs.

Katz & Kahn Open systems model of


development of social
structures

Kuhn 1967 The Normal science,


Structure of scientific revolutions.
Scientific He put the term
Revolutions “paradigm” into
academia. Scientists
make observations and
collect data from
everyday life. But at
some point anomalies

310 | P a g e
arise that current
theories can’t address.
So the scientific
community gets into a
crisis and a new
paradigm is introduced
that is better - it
addresses the
problems. So a
wholesale conversion
to the new paradigm
takes place by the
scientists. They
abandon the prior
paradigm based on
belief, non scientific
reasons-it’s a quasi
religious experience.
Kuhn’s book led to big
controversy in the
social sciences.

Lawrence & 1967 Contingency theory:


Lorsch Rational and natural
systems refer to
different org types.
Rational orgs respond
to homogenous, stable
environments. Natural
orgs respond to
diverse, changing
environments. There
is no one best org
form. The
environment
determines which orgs
survive and thrive.
The environment was
more stable in the past
so rational systems
arose first. The open
system is the most

311 | P a g e
comprehensive model.
Orgs will adapt their
structures to adapt to
environmental
challenges.

Lewin, Kurt 1947 Model of planned


organizational change
(modernist).
Individuals change by
unfreezing (disturbing
equilibrium), moving
(introduction of
change), refreezing
(integrate change into
culture and behavior).

Lincoln & Natural paradigm -


Guba qualitative research

Marx, Karl 1818 Theory of capital - Classical/Sociologic


considered work al stream
central to human life.
Inherent antagonism
between capitalists
and workers over how
to divide surplus
value. Workers are
alienated so they must
organize. Exploitation
of labor helps
accumulate wealth but
fuels resistance by
workers (paradox).

Mayo 1945 Human relations --


change is interesting;
attention gratifying.

312 | P a g e
Meron, Manifest functions
Robert (official purposes of the
org); latent functions
(unintended); EX-ed
insts are to ed
(manifest function) but
they prepare people to
live in hierarchies
(latent function).
Structural
fundamentalism.

Meyer & 1977 Institutional


Rowan theory/rationalized
myths.

Morgan, Metaphors of orgs:


Gareth machine, organism,
brain, cultural systems
(view of reality-shared
values and beliefs),
political systems,
psychic prisons
(Denhardt),
instruments of
domination
(machine/polit), orgs
as flux and
transformation due to
fundamental tensions
and contradictions.

Ouchi Breaks control


methods into 3
categories: markets,
bureaucracy, and
clans.

313 | P a g e
Parsons, 1902- All societies carry out
Talcott 79 certain functions to
survive: A-adaptation-
gaining resources; G-
goal attainment; I-
integration-
cohesiveness of
members; L-latency-
how the org sustains
itself -transmission of
culture. Structural
fundamentalism.

Perrow, Typology helps define


Charles input or raw materials
and to explain why
differences between
public and private
orgs. "Moral
imperialism" - willing
cooperation managed
from above.

Pfeffer, 1978 Resource Resource Dependence Modern


Jeffrey Dependence Theory: An org is
by Pfeffer vulnerable because of
and Gerald its need for resources
Salancik (raw material, labor,
1978 capital, equipment)
"Power in from its environment --
Organization so the org is controlled
s" by its environment. So
we analyze the org by
starting with the
resources it needs and
tracing them to their
source-also look at the
org’s competitors for
the same resources.
Org politics is using
power to obtain

314 | P a g e
preferred outcomes
when there is
uncertainty or
dissensus.
Postmodern view.

Schein Beliefs and


assumptions form core
of org culture. Theory
of culture as
assumptions, values,
artifacts. Primary
strategy is to protect
org identity.

Scott, 2003 Organization Discusses values of Postmodern


Richard W. 1995 s: Rational, organization from a
Natural, and sociological viewpoint.
Open Grandfather of
Systems institutional theory.
Institutions Sees orga as
and collectivities, social
Organization systems with needs,
s formal and informal
structures. Identifies
org pathologies from
power and misuse.
Layered model - closed
rational systems,
closed natural models,
open natural systems,
open natural models.
Institutionalization -
process by which
social reality
constructed.
Cultural/Cognitive
structure -- beliefs and
understandings shared
by participants about

315 | P a g e
the nature of their
work and interests.
Scott proposes a
fourth perspective to
combine rational,
natural and open
models by adding the
concept of closed
systems. He also says
all four can apply to
social psychological,
structural, and
ecological levels of
analysis.

Selznik, 1949 Institutional Focuses on Modern


Philip Theory distinctiveness of orgs.
Environment is hostile
threat to stability of
orgs. Institutional
School - precursor to
org theory. Focuses
on control of an org by
creating a "committed
polity" - orgs have life
of own with rational
and non-rational
dimensions. Org
activities become
infused with value
beyond technical
requirements at hand
(institutionalization).
Orgs adapt to the
values of external
society-i.e, when
actions are repeated
and given similar
meanings by self and
others (Richard Scott).
This is
institutionalization-

316 | P a g e
can lead to
“rationalized myths.

Senge, Peter Learning org - linked


to open systems
movement.

Shaffritz & Classics of


Hyde PA

Simon, 1945 Decision Theory of Modern


Herbert Making administrative
behavior
(descriptive/social-
psychological level).
Modernist. Cynical of
Classical. He calls for
empiricism and facts.
Individuals need orgs
to be rational --
specific goals support
rational behavior.
Bounded rationality
(March and Simon
1958). Positivist.
Shifted focus from
action to analysis -
believes choice
determines
subsequent action.
Choice limited based
on management
(differs from natural
systems theorists).
Looks at formalization

317 | P a g e
- more objective and
static than
institutionalists.
Administrative Man
will satisfice - operates
within bounded
rationality (cognitive
limits and org affect
thru interaction).
Decision makers want
to be rational but can't
because attemtps at
rationality are limited.
Causes ambiguity and
uncertainty.

Smith, 1776 Wealth of Division of labor, Classical/


Adam Nations invisible hand. Father Industrialism
of capitalism.
Industrialism.

Taylor, 1911 Father of scientific Classical/Classical


Frederick management - Management
pragmatic/sociological stream
-psychological level.
Time and motion
studies/get more work
out of
workers/efficiency.
Bottom-up
rationalization. He
attacked soldiering
(workers limiting their
output on purpose).
One best way
promoted
rationalization in orgs.

318 | P a g e
Thompson Levels model --
Rational, natural and
open perspectives can
all apply to the same
org at diff levels.
Technical level-inputs
transformed into
outputs (rational);
managerial level-
design , control and
production (natural);
institutional level-Bd,
CEO relate to wider
environment (open).
An org functions as a
rational sys open to
the environment by
creating some closed
system compartments.

von 1956 Founded General


Bertlanfy, Systems Theory
Ludwig movement.

Weber, Max 1900s Behaviorist. Theory of Classical/Sociologic


(1864- bureaucracy al stream
1920) (structural and
descriptive level).
Rationalization -
impartial and efficient
decision-makers.
Believes bureaucracy
most efficient
organization (hierarchy
of authority) as it is a
way to rationalize the
social environment.
Iron cage of social
domination - humans
dominated by rational
bureaucracy. Formal

319 | P a g e
rationality (means or
techniques) and
substantive rationality
(ends or goals).
Formal rationality
without substantive
rationality leads to an
iron cage, making man
a cog in a machine.

Weick, Karl 1969 Enactment theory Symbolic/interpreti


(when you use ve
concepts {i.e.,
organizations} you
create the thing you’re
seeking to
study)/systems
theory/social
construction. Humans
organize to help reduce
information
uncertainty faced in
lives. Conditions in the
environment can’t be
separated from the
perception of those
conditions. Cognitive
processes help orgs
evolve but organized
patterns of action may
occur w/o increasing
productivity.

320 | P a g e
3.4 The History of Org Theory
MILESTONES

500 BC Sun Tsu’s The Art of War – recognizes the need for hierarchical
organization, inter org communications, and staff planning.

400 BC Socrates argues for the universality of management as an art unto itself.

360 BC Aristotle, in The Politics, states that executive power in orgs must reflect
their cultural environment.

770 AD - 1400 AD Several Muslim authors outline administrative and bureaucratic


orgs.

1513 Machiavelli urges for “unity of command”, advocates adherence to practical


rather than moral actions.

1776 Adam Smith – The Wealth of Nations – discusses the optimal organization of a
pin factory. Factory system and division of labor.

1832 Charles Babbage – anticipates the notions of scientific management movement,


division of labor.

1885 Captain Henry Metcalfe – Administration of workshops, public and private –


“science of administration”.

1902 Vilfredo Pareto – “father” of social systems concept. Precursor to Mayo human
relations movement.

1903 Frederick Taylor publishes “Shop Management”.

1910 Louis Brandeis (assoc. of Fred Taylor) coins the term Scientific Management
in testimony to ICC (apply SM to railroads).

1911 Frederick Taylor publishes The Principles of Scientific Management.

1916 In France, Henri Fayol publishes General and Industrial Management, the First
complete theory of management. Fayol believed that his concept of management was
universally applicable to every type of org.

321 | P a g e
1922 Max Weber – structural definition of bureaucracy is published. It uses an ideal
type approach to extrapolate from the real world the central core features that
characterize the most fully developed form of bureaucratic organization.

1924 The Hawthorne Studies begin – lead to new thinking about the relationships
among the work environment, human motivation, and productivity.

1926 Mary Parker Follett anticipates movement toward more participatory


management styles. She calls for “power with” as opposed to “power over”.

1933 Elton Mayo’s The Human Problems of an Industrial Civilization is the first
major report on the Hawthorne Studies and the first significant call for a human
relations movement.

1937 Luther Gulick’s “Notes on the Theory of Organization” draws attention to the
functional elements of the work of an executive with his mnemonic device POSDCORB.

1938 Chester Barnard’s “The Functions of the Executive”, a sociological analysis,


encourages the postwar revolution in thinking about org behavior.

1940 Robert K. Merton’s article “Bureaucratic Structure and Personality” proclaims


that Max Weber’s “ideal-type” bureaucracy has inhibiting dysfunctions that lead to
inefficiency and worse.

1943 Abraham Maslow’s “needs hierarchy” appears in a Psychological Review article


“A Theory of Human Motivation”.

1946 Herbert Simon’s PAR article “The Proverbs of Administration” attacks the
principles approach to management for being Inconsistent and often inapplicable.

1947 Herbert Simon’s “Administrative Behavior” urges the use of a truly scientific
method in the study of administrative phenomena. Decision making is the true heart
of administration.

1948 Dwight Waldo publishes “The Administrative State”, which attacks the “gospel
of efficiency” that dominated administrative thinking before WWII.

Norbert Weiner coins the term “cybernetics” which becomes a foundation for the
systems theories of organization.
322 | P a g e
1949 Philip Selznick in TVA and the Grass Roots, discovers “cooptation” in which
outside elements (community orgs) are subsumed into the policy- Making process in
order to prevent those elements from becoming threats.

Rufus E. Miles Jr. of the U.S. Bureau of the Budget states Miles’ Law: “Where you
stand depends on where you sit.”

Air Force captain Edsel Murphy states Murphy’s Law: “If anything can go Wrong,
it will.”

1950 George C. Homans publishes The Human Group – the first major application of
“systems” to organizational analysis.

1954 Peter Drucker’s book “The Practice of Management” popularizes the concept of
management by objectives.

1956 William H. Whyte Jr. publishes “The Organization Man” – details a man in an
organization who accepts its values and finds harmony in conforming to its policies.

Talcott Parsons, in Admin Science Quarterly – article “Suggestions for a sociological


Approach to the Theory of Organizations” defines an org as a social system that
focuses on attainment of specific subgoals and in turn contributes to the
accomplishment of goals of the larger org & society.

1957 Northcote Parkinson discovers his law that “work expands so as to fill the time
available for its completion”.

Douglas McGregor’s article “The Human Side of Enterprise”. Theory X and Theory
Y, and applies the concept of “self-fulfilling prophecies” to organizational behavior.

The History of Organization Theory – Schools and how the field has developed.

Classical Org Theory:

Classical Theme: Theorists of the classical period thought that organizations should be
based on universally applicable scientific principles.

500 BC Sun Tsu’s The Art of War – recognizes the need for hierarchical
organization, inter org communications, and staff planning.

323 | P a g e
400 BC Socrates argues for the universality of management as an art unto Itself.

360 BC Aristotle, in The Politics, states that executive power in orgs must reflect
their cultural environment.

770 AD - 1400 AD Several Muslim authors outline administrative and bureaucratic


orgs.

1513 Machiavelli urges for “unity of command”, advocates adherence to practical


rather than moral actions.

1776 Adam Smith – The Wealth of Nations – discusses the optimal organization of a
pin factory. Factory system and division of labor.

1832 Charles Babbage – anticipates the notions of scientific management movement,


division of labor.

1885 Captain Henry Metcalfe – Administration of workshops, public and private –


“science of administration”.

1902 Vilfredo Pareto – “father” of social systems concept. Precursor to Mayo human
relations movement.

1903 Frederick Taylor publishes “Shop Management”.

1910 Louis Brandeis (assoc. of Fred Taylor) coins the term Scientific Management
in testimony to ICC (apply SM to railroads).

1911 Frederick Taylor publishes The Principles of Scientific Management.

1916 In France, Henri Fayol publishes General and Industrial Management, the First
complete theory of management. Fayol believed that his concept of management was
universally applicable to every type of org.

1922 Max Weber – structural definition of bureaucracy is published. It uses an ideal


type approach to extrapolate from the real world the central core features that
characterize the most fully developed form of bureaucratic organization.

324 | P a g e
1924 The Hawthorne Studies begin – lead to new thinking about the relationships
among the work environment, human motivation, and productivity.

1926 Mary Parker Follett anticipates movement toward more participatory


management styles. She calls for “power with” as opposed to “power over”.

1933 Elton Mayo’s The Human Problems of an Industrial Civilization is the first
major report on the Hawthorne Studies and the first significant call for a human
relations movement.

1937 Luther Gulick’s “Notes on the Theory of Organization” draws attention to the
functional elements of the work of an executive with his mnemonic device POSDCORB.

Neo-Classical Org Theory: (the GIANTS include James March, Philip Selznick
and Herbert Simon.

Major Neoclassicist theme: Organizations do not and cannot exist as self-contained


islands isolated from their environments. Neoclassical Org Theory is important
because it initiated the theoretical movement away from the overly simplistic
mechanistic views of the classical school. Also it is important because neoclassicalists
initiated theories that became the foundations of most of the schools that followed.

1938 Chester Barnard’s “The Functions of the Executive”, a sociological analysis,


encourages the postwar revolution in thinking about org behavior “Individuals must
be induced to cooperate”. Persuasion.

1940 Robert K. Merton’s article “Bureaucratic Structure and Personality” proclaims


that Max Weber’s “ideal-type” bureaucracy has inhibiting cysfunctions that lead to
inefficiency and worse.

1943 Abraham Maslow’s “needs hierarchy” appears in a Psychological review article


“A Theory of Human Motivation”.

1946 Herbert Simon’s PAR article “The Proverbs of Administration” attacks the
classical approach to the “general principles of management” of Fayol and Gulick for
being inconsistent and often inapplicable to many admin situations mgrs face.

325 | P a g e
1947 Herbert Simon’s “Administrative Behavior” urges the use of a truly scientific
method in the study of administrative phenomena. Decision making should be the
focus of a new administrative science. Org theory is in fact a theory of “Bounded
Rationality” of humans who “Satisfice” because they don’t have the intellectual
capacity to maximize.

1948 Dwight Waldo publishes “The Administrative State”, which attacks the “gospel of
efficiency” that dominated administrative thinking before WWII.

Norbert Weiner coins the term “cybernetics” which becomes a foundation for the
systems theories of organization.

1949 Philip Selznick in TVA and the Grass Roots, discovers “cooptation” in which
outside elements (community orgs) are subsumed into the policy- Making process in
order to prevent those elements from becoming threats. In contrast to classical
theorists, Selznick argues that orgs consist not only of a number of positions for
management to control, but of individuals, whose goals may not coincide with those of
the organization.

Rufus E. Miles Jr. of the U.S. Bureau of the Budget states Miles’ Law: “Where you
stand depends on where you sit.”

Air Force captain Edsel Murphy states Murphy’s Law: “If anything can go wrong, it
will.”

1950 George C. Homans publishes The Human Group – the first major application of
“systems” to organizational analysis.

1954 Peter Drucker’s book “The Practice of Management” popularizes the concept
of management by objectives.

1956 William H. Whyte Jr. publishes “The Organization Man” – details a man in an
organization who accepts its values and finds harmony in conforming to its policies.

Talcott Parsons, in Admin Science Quarterly – article “Suggestions for a sociological


Approach to the Theory of Organizations” defines an org as a social system that focuses
on attainment of specific subgoals and in turn contributes to the accomplishment of
goals of the larger org & society.
326 | P a g e
1957 Northcote Parkinson discovers his law that “work expands so as to fill the time
available for its completion”.

Douglas McGregor’s article “The Human Side of Enterprise”. Theory X and Theory Y,
and applies the concept of “self-fulfilling prophecies” to organizational behavior.

Philip Selznick, in Leadership in Administration, argues that the function of an


institutional leader is to help shape the environment in which the institution operates
and to define new institutional directions through recruitment, training and
bargaining.

1958 March and Simon, in Organizations, classify the behavioral revolution in


organization theory.

1959 Charles Lindblom’s “The Science of Muddling Through”, rejects the rational
model of decision making in favor of incrementalism.

Cyert and March, in “A Behavioral Theory of Organizational objectives”, argue that


power and politics influence the formation of organizational goals. Article is a
precursor of the power and politics school of org theory.

1960 Herbert Kaufman’s The Forest Ranger examines how org and professional
socialization can develop the will and capacity of employees to conform.

1961 Amitai Etzioni, in A Comparative Analysis of Complex Organizations, argues


that org effectiveness is affected by the match between an org’s goal structure and its
compliance structure.

1962 Peter Blau and W. Richard Scott, in Formal Organizations, assert that all orgs
include both a formal and informal element, and that it is impossible to understand
the true structure of a formal org without understanding its parallel informal
organization.

1963 Cyert and James March – analyzed the impact of power and politics on the
establishment of organizational goals, and discusses the formation of coalitions and
negotiations to impose coalitions’ demands on the org. In “A Behavioral Theory of the
Firm”, they argue that corporations tend to “Satisfice” rather than engage in
economically rational profit-maximizing behavior.
327 | P a g e
1965 Robert L. Kahn’s Organizational Stress is the first major study of the mental
health consequences of organizational role conflict and ambiguity.

1966 Katz and Kahn seek to unify the findings of behavioral science on
organizational behavior through open Systems Theory in The Social Psychology of
Organizations.

James G. March, in The Power of Power, explores definitions, concepts and


approaches for empirically studying social power in orgs and communities.

328 | P a g e
3.5 Nine Major “Schools” of Organization Theory
1) CLASSICAL ORGANIZATION SCHOOL (up to ~ 1940)

SMITH, FAYOL, TAYLOR, WEBER, GULICK

2) NEOCLASSICAL ORGANIZATION SCHOOL (~ 1940 to 1960)

BARNARD, MERTON, SIMON, SELZNICK, CYERT & MARCH

3) HUMAN RESOURCE (“ORG. BEHAVIOR”) SCHOOL (~ 1955 – present)

FOLLETT, ROETHLISBERGER, MASLOW, MCGREGOR, JANIS

4) “MODERN” STRUCTURAL ORGANIZATION SCHOOL (~ 1960 to 1980)

BURNS & STALKER, BLAU & SCOTT, WALKER & LORSCH, MINTZBERG, JAQUES

5) ORGANIZATIONAL ECONOMICS SCHOOL (~ 1975 to 1990)

WILLIAMSON, JENSEN & MECKLING, RUBIN

6) POWER & POLITICS ORGANIZATION SCHOOL (~1960 – 1985)

PFEFFER, MICHELS, FRENCH & RAVEN, MARCH, KANTER, MINTZBERG

7) ORGANIZATIONAL CULTURE SCHOOL (~ 1990 to present)

SCHEIN, COOK & YANOW, TRICE & BEYER

8) ORGANIZATIONAL CULTURE REFORM MOVEMENTS (~ 1980 to present)

OUCHI, PETERS & WATERMAN, SENGE

9) SYSTEMS & ENVIRONMENTS THEORIES (~ 1965 to 1980)

KATZ & KAHN, THOMPSON, MEYER & ROWAN, PFEFFER & SALANCIK

CLASSICAL ORGANIZATION SCHOOL (up to ~ 1940)

 Rooted in the industrial revolution of 1700s and mechanical engineering,


industrial engineering, and economics

329 | P a g e
 What is best way to design and manage organizations so they achieve their
declared goals effectively and efficiently?

 Personal preferences of members are restrained by formal rules, authority and


norms of rational behavior

Tenets

 Organizations exist to accomplish production-related and economic goals

 There is one best way to organize for production, and that way can be found
through systematic, scientific inquiry

 Production is maximized through specialization and division of labor; and

 People & organizations act according to rational economic principles

ADAM SMITH (1776) “Division of Labour” – Scottish economist pin-factory, breaking


down work and specializing (ten men increase from 200 to over 48000 pins in a day)

HENRI FAYOL (1916) “General Principles of Management”

 French engineer looked from “Top-down”

 First comprehensive theory of management.

 14 general prescriptive principles include division of work, discipline, unity of


command, scalar chain (line of authority), equity, initiative...

FREDERICK TAYLOR (1916) “Scientific Management”

 Known as the “Father of Scientific Management” looked from “Bottom-up”

 Profound, revolutionary effect on business and public administration

 “One best way” to perform any task

 Increase output w/fastest, most efficient & least fatiguing production methods

 One best way to structure the organization around those methods.

MAX WEBER (pron. ‘Va-ber,’ 1922 published posthumously) “Bureaucracy”

330 | P a g e
 German sociologist, remains single most influential authority on bureaucracy

 Not prescriptive, but descriptive of “ideal-type” bureaucracy

 Identified major variables or features that characterize bureaucracy

 Hierarchy, specialization, rules/policies/procedures, “red tape”

LUTHER GULICK (1937) “Theory of Organization”

 Focused on management

 Mnemonic, POSDCoRB: seven major functions of executive management: plan,


organize, staff, direct, coordinate, report, and budget

 Analysis of management functions still continues within OT

NEOCLASSICAL ORGANIZATION SCHOOL (~ 1940 to 1960)

 “New-classical” perspective is more an “anti-school” – Anti-classical

 Not a separate body of work, could not permanently stand on its own

 Transitional, reactionary school

 Initiated movement away from overly simplistic mechanistic views of classical


school

 Issues and theories became central to foundations of most of schools that


followed

 Known for:

o humanness of organization members

o coordination among administrative units

o internal-external relations, and

o decision-making processes.

CHESTER BARNARD (1938) “The Economy of Incentives”

 Treat people as members, not machines

 Sought to create a comprehensive Theory of Organizational Behavior

331 | P a g e
 Centered on need to cooperate –cooperation holds an organization together
(thus, members must be induced to cooperate)

 Tried to reconcile top-down goals with bottom-up compliance

 Primary responsibilities of the Executive:

o Create & maintain a sense of purpose and a moral code

o Establish system of communication

o Ensure willingness of people to cooperate to achieve org. purpose

ROBERT MERTON (1940) “Bureaucratic Structure and Personality”

 One of the most influential mid-century sociologists

 “Ideal-type” bureaucracy had inhibiting dysfunctions – characteristics that


prevented it from being optimally efficient, and negative effects on the people

HERBERT SIMON (1946) “Proverbs of Administration”

 Attacked classical theory in his criticism

 Classical ideas are conflicting, inconsistent, and inapplicable to many


administrative situations facing managers

 For each “principle” of management, one can find an equally plausible,


acceptable, and [proverbial] contradictory principle

 Won Nobel Prize in Economics

PHILIP SELZNICK (1948) “Foundations of the Theory of Organization” –

 Organizations consist not simply of positions for management to control, but of


individuals w/goals & aspirations might not coincide w/formal goals

 Best known for “cooptation:” process bringing in & subsuming external


elements into its policy-making process to prevent from becoming a threat

 As in his case study: TVA and the Grass Roots

RICHARD CYERT & JAMES MARCH (1959) “Behavioral Theory of Org. Objectives”

 Colleagues of Simon at Carnegie Tech

332 | P a g e
 Focus on power and politics of establishing organizational goals

 Corporations tend to “satisfice” rather than engage in economically rational


profit-maximizing behavior

HUMAN RESOURCE (“ORG. BEHAVIOR”) SCHOOL (~ 1955 – present)

 Allow people to grow and develop and they will be satisfied

 Creativity, flexibility, prosperity will flow naturally from satisfied employees

 Then Org/Employee relationship changed from dependence to codependence

 People are as important as, or more important than, the organization itself

 Major themes: leadership, motivation, teams and groups, effects of work


environment, power and influence, and organizational change

 Criticism of HRT school:

 Called “Cow sociology:” just as satisfied cows are to give more milk, satisfied
workers are to be more productive

 Years of research has shown no clear relationship between:

o Worker satisfaction and productivity, or

o Leadership style, Job enlargement, or Decision-making involvement and


Worker satisfaction or productivity

Tenets

 Organizations exist to serve human needs (rather than the reverse)

 Organizations and people need each other - organizations need ideas, energy,
and talent; People need careers, salaries, and work opportunities

 When fit between individual and organization is poor, one or both will suffer:
individuals will be exploited, or will seek to exploit the organization, or both

 A good fit between individual and organization benefits both: humans find
meaningful/satisfying work; and orgs get human talent & energy they need

MARY PARKER FOLLETT (1926) “Giving of Orders”

333 | P a g e
 True pioneer (both as a researcher and a female)

 Orders should be depersonalized in participatory leadership

 Obey the ‘Law of the Situation’

 “Whatever heightens self-respect increases efficiency.”

FRITZ ROETHLISBERGER (1941) “Hawthorne Experiments”

 Experiments led by Elton Mayo and documented here by Roethlisberger

 Direct precursor of field of organizational behavior & human resource theory

 This work laid foundation for assumptions that displaced classical theory

ABRAHAM MASLOW (1943) “Theory of Human Motivation”

 All talk of motivation must start with Maslow

 Hierarchy of Needs:

o All humans have needs that underlie their motivations

o As lower-level needs are satisfied, they no longer ‘drive’ behavior

o Satisfied needs are not motivators

o As lower needs become satisfied, higher needs take over motivating


forces

DOUGLAS MCGREGOR (1957) “The Human Side of Enterprise”

 Managerial assumptions about employees become self-fulfilling prophecies

 Those assumptions cause employee behavior

 Theory X from scientific management: humans dislike work, will avoid it if


possible; they must be coerced/controlled/directed/ threatened

 Theory Y is the opposite: people do not inherently dislike work, it can be a


source of satisfaction; they will seek and accept responsibility

IRVING JANIS (1971) “Groupthink – Consensus at Any Cost”

334 | P a g e
 Groupthink is “mode of thinking that persons engage in when concurrence
seeking becomes so dominant in a cohesive in-group that it tends to override
realistic appraisal of alternative courses of action

 The desperate drive for consensus at any cost that suppresses any dissent

 The reason that social conformity is encountered so frequently in groups

 He examines high-level decision-makers/decision-making in national fiascoes

“MODERN” STRUCTURAL ORGANIZATION SCHOOL (~ 1960 to 1980)

 Structure refers to relationships among org. positions/groups-units/work-


processes

 Concerned w/ vertical differentiations (hierarchical levels of authority &


coordination) and horizontal differentiations (between org. units: ie,
product/service, areas, skills)

 Organizational chart is the best representation of the organization

 Highly important is “legitimate authority” (that flows down through the org.
hierarchy)

 Formal rules by those in authority ensures behavior is directed toward formal


goals

 Tend to define power as synonymous with authority

 Classical founders were structuralists (Fayol, Taylor, Weber) 1st half of 20th
century and gave this school its roots. But the “modern” theories influenced by
and greatly benefited from advancements in organization theory in 2nd half of
20th century.

TOM BURNS & G.M. STALKER (1961) “Mechanistic [vs] Organic Systems”

 Socio-technical Theory

 Stable conditions: may use mechanistic form of organization (w/ hierarchy,


formal rules/regulations, vertical communications, structured decision making

335 | P a g e
 Dynamic conditions – situations in which the environment changes rapidly–
may require organic form w/ less rigidity, more participation, and more reliance
on workers to define and redefine their positions and relationships

 Either form may be appropriate in particular situations

PETER BLAU & RICHARD SCOTT (1962) “Formal [vs Informal] Organization”

 All organizations include both formal and informal elements

 To understand true structure of formal organization, must know the informal

 Were influenced by “classical philosopher” Chester Barnard

ARTHUR WALKER & JAY LORSCH (1968) “Org.Choice: Product vs Function”

 Employee grouping

o Should all specialists in a given function be grouped under a common


boss regardless of differences in products they are involved in?

o Or should the various functional specialists working on a single product


be grouped together under the same boss?

 They look at 2 firms in same industry, 1 organized by product, 1 by function

 Either form may be appropriate in particular situations

HENRY MINTZBERG (1979) “Five Basic Parts of the Organization”

 One of most widely respected management & OT theorists in 2nd ½ of 20th c.

 Coherently synthesized many schools of organization & management theory

 Management Policy Theory: model of org. with five interdependent parts:


strategic apex; middle line; operating core; techno-structure; & support staff

ELLIOTT JAQUES (1990) “In Praise of Hierarchy” –

 Lonely defender of hierarchical-bureaucratic form of organization

 Hierarchical layers enable organizations to cope with discontinuities in mental


and physical complexities, thereby separating tasks into manageable steps

 Instead of new org. forms, we need to learn to manage hierarchies better

336 | P a g e
ORGANIZATIONAL ECONOMICS SCHOOL (~ 1975 to 1990)

 Fundamental universal problem: how to induce managers/employees to act in


best interests of ownership (or in public orgs, of who controls policy & resource
allocation)

 Minimize agency & transaction costs and maximize profit & productivity

3 Core Theories

Agency Theory

 Defines managers and other employees as “agents” of owners (“principals”), who


out of necessity must delegate some authority to agents

 Agency costs are a type of transaction cost incurred by principals to protect


interests from the possibility of loss by agents including costs of investigating,
selecting and monitoring agents and other residual losses

Transaction Cost Theory

 Core element of organizational economics (transaction costs) rests on:

o Costs of maintaining principal-agent relationship

o How to minimize the costs

o Effects on management decisions

Property Rights Theory

 Allocation of costs and rewards among organization participants

 An organization is a form of “legal fiction,” of complex relationships (ie:


contracts) between the legal fiction (the firm) and:

o Owners of labor (employees/agents)

o Owners of material and capital inputs (owners/principals)

o Consumers of outputs

337 | P a g e
OLIVER WILLIAMSON (1975) “Markets and Hierarchies: Understanding the
Employment Relation”

 Decisions on internal vs external production of goods/services

 Analyze applying economic contracts and market models to employment


relations

 Decision process for employer-employee relationship is a market transaction

MICHAEL JENSEN & WILLIAM MECKLING (1976) “Theory of the Firm:


Managerial Behavior, Agency Costs and Ownership Structure” –

 Integrative overview of the three core theory components (above)

PAUL RUBIN (1990) “Managing Business Transactions” –

 People are self-interested and opportunistic

 Impossible to write complete contracts to account for any/all possible events


which eliminate all forms of opportunism or cheating

 All other pre-/post-contract mechanisms must be used to minimize agency


costs

POWER & POLITICS ORGANIZATION SCHOOL (~1960 – 1985)

 Rejects classical school, structural school, economics school,


systems/environments school, all as being naïve and unrealistic, and of
minimal practical value

 Organizations are complex systems of individuals and coalitions:

o Each has its own interests, beliefs and values

o All of which continuously compete with each other for scarce resources

 Conflict is inevitable

 Influence is the primary “weapon” for use in competition and conflicts

 Power, politics and influence are essential and permanent facts of org. life

 Goals are result of on-going maneuvering and bargaining of individuals &


coalitions
338 | P a g e
 Coalitions are transitory: shift w/issues; often cross over vertical/horizontal
boundaries

 Sources of power:

o Legitimate authority

o Control over scarce resources

o Access to others who are perceived as having power

o A central place in a coalition

o Ability to work organization rules (knowing how to get things done)

o Personal credibility.

JEFFREY PFEFFER (1981) “Understanding the Role of Power in Decision


Making” –

 Power and politics are fundamental concepts to understand behavior in orgs

 Those who perform critical tasks have natural advantage to develop/exercise


power

 Power is first and foremost a structural phenomenon; context & relationship


specific

 One is not ‘powerful’ or ‘powerless’ in general, but only in respect to others

ROBERT MICHELS (published 1915 in German / 1962 in English) “Democracy


and the Iron Law of Oligarchy” –

 [He] who says organization says oligarchy

 People who want power try to reign in as much as possible (to point of
oligarchy)

 Orgs divided into minority of directors and majority of directed (leaders & the
led)

JOHN FRENCH & BERTRAM RAVEN (1959) “The Bases of Social Power”

339 | P a g e
 Five bases of power [“LECRR”]

o Legitimate power

o Expert power (knowledge/ability)

o Coercive power

o Reward power

o Referent power (through association)

 Power is derived from these five different bases of attraction (the recipient’s
sentiment toward the agent who uses power) and resistance to the use of power;
i.e., coercive power generally decreases attraction and causes high resistance;
while reward power increases attraction and creates minimal resistance.

JAMES MARCH (1966) “The Power of Power”

 Examines alternative definitions and concepts of social power in orgs &


communities, which extends to “boundaryless organizations,” “virtual
organizations,” and networks

 Takes three basic approaches to the study of power: experimental studies, com-
munity studies, and institutional studies

 “On the whole, power is a disappointing concept [for social research].”

ROSABETH MOSS KANTER (1979) “Power Failure in Management Circuits” –

 Executive & managerial power is a necessary ingredient for moving toward goals

 Ability of managers to lead effectively cannot be predicted by studying their


styles or traits; but by knowledge of their power sources

 Three groups are particularly susceptible to powerlessness:

o First-line supervisors

o Staff professionals

o Top executives

 Those who feel powerless tend to use more dominating/punishing forms of


influence

340 | P a g e
 Powerlessness is more of a problem to organizations than the struggle for power

HENRY MINTZBERG (1983) “The Power Game and the Players”

 “Players” are “influencers” who attempt to control organizational


decisions/actions

 To understand behavior of the organization, must know what influencers are


present, what needs each seeks to fulfill, how each is able to exercise power to
fulfill them

 Influencers come from “external coalitions” and “internal coalitions.” [Scott &
Davis book, figure 33.1 pg 339 “The Cast of Players” is the shape of a ‘box
guitar’].

ORGANIZATIONAL CULTURE SCHOOL (~ 1990 to present)

 This school is a kind of counter-culture in OT

 Rejects structural, economics, systems/environments schools (like Power &


Politics)

 Culture is shaped by many factors, such as:

o Society in which it resides

o Its technologies, markets and competition

o Personality of its present and past leaders

 Assumes that many org. behaviors and decisions are predetermined by the
patterns of basic assumptions held by members of the organization (“it worked
in the past”)

 Underlying assumptions become ingrained & totally accepted, but largely


forgotten, reasons for “the way we do things here” – even when it may no longer
be appropriate

 Org. behavior is controlled by cultural norms, values, beliefs, and assumptions

 Culture is the unseen and unobservable force that is always behind org.
activities (that can be seen and observed)

341 | P a g e
 Social energy moves people to act and provides meaning, direction, and
mobilization

 To understand or predict how an org. will behave under varying circumstances,


must know org. culture (and related symbolism), not just structure or systems

 Some organizations have strong and unified cultures whereas others are weaker
and less pervasive; and there are even cultures within cultures (sub-cultures).

EDGAR SCHEIN (1993) “Defining Organization Culture”

 Contributed significantly by his early writings focusing on the process of


socializing employees into existing organizational cultures

 To learn culture of an organization, “clinical” is better than “ethnographic”


perspective (As “helper to client” in lieu of “researcher to subject” [who wants to
be studied?]).

SCOTT COOK & DVORA YANOW (1993) “Culture and Organizational Learning”

 An organization has the capacity to learn how to do what it does, where what it
learns is possessed not by individual members, but by the aggregate itself.”

HARRISON TRICE & JANICE BEYER (1993) “Changing Organizational Cultures”

 Wrote a comprehensive treatise on organizational culture

 Offer eight “prescriptive aphorisms”

ORGANIZATIONAL CULTURE REFORM MOVEMENTS (~ 1980 to present)

 Different reform movements presented here all share a common theme: lasting
organizational reform requires changes in organizational culture.

 Replace cultures of hierarchy, rigidity, homogeneity, power based on authority,


associations in closed networks, and reliance on rules With cultures of
horizontal relations, open and accessible networks, flexibility, responsiveness,
individual and group empowerment, diversity, and customer service

 Empowered employees/teams granted autonomy and discretion to make


decisions

 Policies, procedures, and layers of hierarchy are eliminated


342 | P a g e
 Accountability to bosses is replaced by accountability to customers and clients

 In 1980s/90s, U.S. companies & government agencies lost competitiveness &


agility

 Now were forced to compete in global markets against worthy competitors who
were not all bound by same rules. “Leveling” the playing field, of worldwide
marketplace

 Ineffectiveness of government could no longer be ignored

 All of these reform movements trace back to 1950 when Deming (and later
Juran) influenced Japanese executives to adopt his methods of statistical
control; by 1975 Japan was world leader in quality and productivity

 After 1980 TV show “If Japan can, why can’t we?” quality movement exploded in
U.S

Reforms include:

 Total Quality Management (TQM) Deming, Crosby, Joiner, Juran, Walton)

 The Search For Excellence (book: In Search of Excellence) Peters & Waterman

 Quality of Work Life (QWL) Weisbond

 Productivity Measurement / Balanced Scorecard, Berman, Kaplan & Norton

 Reengineering (Process and Business), Hammer & Champy

 Reinventing Government, Gore, Osborne & Gaebler

 Feminist perspective, i.e., Gendering Organization Theory, Acker

 Multicultural perspective, i.e., Creating the Multicultural Organization, Cox

WILLIAM OUCHI (1981) “The Z Organization”

 Organizations are incompatible with formality, distance, and contractualism.


They proceed smoothly only with intimacy, subtlety, and trust

 Z-orgs more like clans than markets or bureaucracies, they foster close
interchange between work and social life

343 | P a g e
THOMAS PETERS & ROBERT WATERMAN (1982) “In Search of Excellence:
Simultaneous Loose-Tight Properties”

 From the “eight attributes of management excellence”

 [No. 8] is a summary of the other seven: “Coexistence of firm central direction


and maximum individual autonomy [is best]… Organizations that live by the
loose-tight principle are on one hand rigidly controlled, yet at the same time
allow (indeed, insist on) autonomy, entrepreneurship, and innovation from the
rank & file.”

PETER SENGE (1990) “The Fifth Discipline: A Shift of Mind”

 For orgs to learn to change, managers must detect seven “learning disabilities”
and how to use five “disciplines” to overcome them

 The 5th discipline is integrative: fuses others into a coherent body of theory &
practice

SYSTEMS & ENVIRONMENTS THEORIES (~ 1965 to 1980)

 In 1960s open systems essentially displaced the closed systems models

 Shift from internal to external dynamics of competition, interaction


interdependency

 Orgs: systems of interdependent activities embedded in/dependent on


environment

 From ENV orgs get material, financial & human resources, social support,
legitimacy

 Two kinds of organization environments (w/dynamic & interdependent


relationships)

o General:

 Societal and cultural values

 Political and legal norms

 Economic, demographic, and technological conditions

o Specific: Individuals, groups, and other organizations it interacts with


344 | P a g e
 All environments exert demands on organizations

 “Organization Darwinism” - adjusting to changes in environment in order to


survive; in turn, virtually all of org decisions and actions affect their
environment

 Systems theory also known as management science. Has two major


components:

o Apply the General Systems Theory (Bertalanffy)

o Use quantitative tools and techniques to understand complex


relationships among orgs and environmental variables and thereby to
optimize decisions

 Systems theory uses cause-&-effect relationships to find “optimal solutions”

 Search for order among complex variables has led to extensive use of
quantitative methods and models (i.e., computer analysis and operations
research)

DANIEL KATZ & ROBERT KAHN (1966) “Organizations and the System Concept”

 Were the first to articulate the concept of organizations in Open Systems Theory

 Provided intellectual basis for merging classical, neoclassical, human relations/


behavioral, “modern” structural, and systems perspectives of organizations.

 Their concept of open systems has influenced the thinking of many OT


theorists.

JAMES THOMPSON (1967) “Organizations in Action”

 Deal with environmental uncertainty by creating elements to cope with the


outside world, and to use closed system approach only at the technical level of
operations

 Three keys of his “New Tradition” Theory

o Bounded Rationality: tending to org. constraints, contingencies, variables

o Satisficing: satisfactory accomplishment rather than maximizing


efficiency
345 | P a g e
o Contingency Avoidance: practical measures taken to protect normal
operations from excessive disruption caused by environmental factors

JOHN MEYER & BRIAN ROWAN (1977) “Institutionalized Organizations: Formal


Structure as Myth and Ceremony” – Institutional Theory

 Emphasize cultural and institutional environmental influences; socially


constructed [mythical] practices/norms provide frame-work for
creation/elaboration of formal orgs

 As open systems, organizations gain legitimacy and support to extent they


accept these social norms as appropriate ways to organize

JEFFREY PFEFFER & GERALD SALANCIK (1978) “External Control of


Organizations: A Resource Dependence Perspective” – Resource Dependency
Theory

 All organizations exchange resources w/ their environment as a condition for


survival

 Importance & scarcity of resources determine extent of dependency on


environment; ie: “information” needed to reduce uncertainty & dependency, so
seek info to survive

346 | P a g e
3.6 Literature Review
Introduction

A broad base of strategy research is reviewed in this chapter to explain the


foundations for the present study as based in the existing literature. The chapter
comprises seven sections, representing a review of strategy research undertaken in
both management and marketing. The first three sections review empirical studies on
strategy in management, starting with a discussion of the contingency theory as the
root of strategy research. The second section reviews the concept of “fit”, the central
theme in contingency theory. This section discusses the dispute among scholars
regarding the role of managers to fit their organization to the environment, and on
weaknesses in the operationalization of the concept. The third section reviews
empirical studies on strategic fit. This includes discussions of strategy content studies
that focus on the content of the strategy and its relationship with environmental
factors and organizational performance, and of strategy process research, which
emphasizes the formulation process of the strategy.

The fourth section goes on to describe the important roles of the marketing manager
and marketing concept in the organizations. It discusses the role of marketing
managers in fitting organization strategy to the environment, as they have a boundary-
spanning role. It also discusses the development of a marketing concept within
organizations. The Fifth section reviews empirical studies on marketing strategy –
performance relationship. The sixth section discusses the barriers to marketing
strategy implementation, realising that not all organizations implement a marketing
strategy as prescribed in the literature, even though they know its potential benefit.
Finally, the last section summarizes some focal literatures and underlies their
limitations.

The Contingency Theory in Management

The contingency theory has been widely accepted in management discipline since the
early 60s. The emergence of the theory was the result of criticisms of the classical
theories that advocated “one best way” of organizing and managing organizations.
Contingency theorists proposed that there is no one best way to organize different
organizations working in different industries and conditions. The appropriate

347 | P a g e
management style and organizational structure depend on the environmental context
of the organization concerned.

One of the most influential studies in the emergence of the contingency theory is the
study of Burns and Stalker (1961). They investigated the relationship between internal
management practices and the external environment factors of 20 industrial
organizations in the United Kingdom to discover its affect on economic performance.
They found two different management practices in use, which they classified as
“Mechanistic” and “Organic” systems. The Mechanistic system was appropriate for
organizations that operated under stable conditions. These organizations employed
routine and well understood technology. Tasks and duties of employees were clearly
defined by heads of departments. Communication within such organizations was
designed vertically, and its content tended to be instructions from superiors. The
Organic system, on the other hand, was more suitable for organizations that worked
under an unstable, changing environment. The system enabled the organizations
concerned to adapt to environmental changes. It did not pay much attention to rules
and procedures. To cope with the changes, the organizations used lateral
communication, which resembled consultation rather than vertical command, and
hence the span of supervisory control was much wider than in the mechanistic model.
Burns and Stalker emphasized that each system is appropriate under its own specific
conditions. Neither system was superior to the other under all situations.

Similar results are also found by Woodward (1965). She investigated the relationship
between technology and organizational structure of successful organizations in South
Essex, England. Based on the techniques of production and the complexity of the
production system, she classified the organizations into three groups, these are small
batch and unit production (e.g. the custom-tailoring industry), large batch and mass
production (e.g. standard gasoline engines industry), and process and continuous
production (e.g. chemicals industry). Woodward pointed out that successful
organizations in different industries with different technologies were characterized by
different organizational structures. For example, she discovered that successful
organizations engaged in small batch and unit production had wider spans of
supervisory control and fewer levels of hierarchy than did successful organizations
with process and continuous production. This indicates that a bureaucratic-

348 | P a g e
mechanistic system is appropriate for organizations operating in stable conditions
such as the chemical industry, while the organic system is suitable for organizations
working under dynamic conditions, such as the custom-tailoring industry. This
influence of environment on organizations was highlighted further by Chandler (1962).
He conducted a comparative study of US firms, holding the simple premise that
organizational structure followed, and was guided by, strategic decisions. From this
study, he found that environmental changes, such as changes in population, income,
and technology, provided new strategic choices for the firms. The choices included
expansion in the production volume, geographic expansion (market diversification),
and product diversification. A new strategy called for a new or modified structure to
cultivate the opportunities effectively. Throughout his study, Chandler pointed out
that different strategies and environments required different organizational structures.
Centralized organization, for example, seemed to be only appropriate for firms
operating in a relatively unchanging environment. However, when the environment
changed rapidly, this structure did not enable the management to respond the
environmental changes quickly, and hence could not facilitate effective realization of
the opportunities available. In such an environment, firms that employed a
decentralized structure ended up with better performance.

Lawrence and Lorsch (1967) and Thompson (1967) further forged and refined the
theoretical foundation of the contingency perspective. Lawrence and Lorsch (1967)
studied the state of differentiation and integration in organizational systems in the
plastic, food, and plastic container industries. These industries represented the high,
medium, and low rate of growth, technology and market changes successively. They
found that successful firms in each industry had a different degree of differentiation.
The degree of differentiation in the plastic industry tended to be higher than in the
food industry. The lowest degree of the differentiation was found in the plastic
container industry, since it had the most stable environment. The successful firms in
the three industries also attained a higher degree of integration than the less
successful ones. The study revealed that the more differentiated an organization, the
more difficult is to achieve such integration. Furthermore, successful firms in each
industry employed different modes of integration, consistent with their environments.
The plastic container industry, for example, used hierarchy to resolve the conflict. In
those industries operating under less stable conditions, however, conflicts were
349 | P a g e
resolved well by appointing skilled personnel or project teams. This seminal work of
Lawrence and Lorsch refined the contingency theory by demonstrating that different
markets and technological environments require different kinds of organizations, and
that subunits or functional departments within an organization might be managed in
different ways, due to variations resulting from their sub-environments.

In addition, Thompson (1967) forged the foundation of the theory by integrating and
expanding the previous studies. He categorized the two modalities considered by
previous works as “closed system”, which sought uncertainty by only considering few
variables controllable and correlated with goal attainment, and “open system”, which
included uncertainty by acknowledging the interdependency of the organizations to
their environments to survive. Using Parson's (1960) three distinct levels of
organizational responsibility and control (technical, managerial, and institutional), he
integrated the two systems to develop what he considered a newer tradition. He
believed that a technical function should operate under certainty to achieve the
desired outcomes by reducing the number of variables operating on it. The
institutional level, on the other hand, dealt largely with environmental elements
uncontrollable by the organizations. It was best served by open management to
acknowledge the influence of environmental factors and to face up to the inevitable
resultant uncertainty. Thompson suggested that the managerial level should mediate
these two extreme levels by resolving some irregularities coming from external
environments, and pushing the technical core for modifications as environments
changed. In his newer tradition, therefore, Thompson conceived of complex
organizations as open systems faced with uncertainty that were, at the same time,
subject to a rational criterion for certain needs. As did Lawrence and Lorsch (1967), he
considered technology and environment as the major sources of uncertainty. He
further argued that differences in those dimensions resulted in different structures,
strategies, and decision processes.

Based on the above study and some others, Kast and Rosenzweig (1973) defined the
contingency theory as a mid range theory between two extreme views, which state, on
the one hand, that universal principles of organization and management exist or, on
the other, that each organization is unique and each situation must be analyzed
separately. The theory views an organization as a system composed of sub-systems

350 | P a g e
and delineated by identifiable boundaries from its external environment. It underlines
the multivariate nature of organization and attempts to understand how organizations
operate under varying conditions and specific circumstances. Kast and Rosenzweig
further emphasized that ultimately the theory is directed toward suggesting
organizational designs and managerial practices most appropriate for specific
situations.

Since then, the contingency theory has become popular in management research.
Galbraith (1973) asserted that this popularity could be attributed partly to the
assumptions that there was no one best way to manage an organization and that any
one way of organizing is not equally effective under all conditions. Criticisms and
suggestions from some writers, such as Miller (1981), Schoonhoven (1981), and Tosi
and Slocum (1984) further enhanced the theory in terms of the conceptualization of
variables and the specificity in the relationships among them. In addition, the
availability of statistical tools for multivariate analysis facilitated the proliferation of its
application. The theory is not only used for research and theory building in the fields
of organization theory, strategic management, and organizational behavior, but is also
utilized in marketing (Zeithaml, Varadarajan, and Zeithaml, 1988). It enables
researchers and managers to understand organizational needs and to provide the
basis for detailed organizational analysis, which facilitates description of detailed
patterns of organizational relations. Such analysis will generate possible solutions to
the arising problems (Morgan, 1997).

The Concept of Organization – Environment Fit

The central theme in most contingency studies is the fit between the organization and
its environment to improve effectiveness. However, in early contingency studies the
concept of fit was understood and discussed implicitly. They postulated the
organization – environment relationship using phrases such as: congruent with,
matched with, or contingent upon (Venkatraman and Prescott, 1990). Aldrich (1979)
was one theorist who explicitly states and popularizes this concept. He proposed that
organizational forms must either fit their environmental niches or fail. Using
Campbell's (1969) population ecology, rooted in the theory of biological evolution, he
developed what he called “population ecology” or the natural selection model.

351 | P a g e
Through this model, he endeavored to explain changes in organizational forms by
focusing on the nature and distribution of resources in an organization’s environment.
He defined organizational forms as specific configurations of goals, boundaries, and
activities, and classified distribution resources into six dimensions: capacity,
homogeneity-heterogeneity, stability-instability, concentration-dispersion, domain
consensus-dissensus and degree of turbulence. Various combinations of these
dimensions and other constraints created environmental niches for organizations.
Organizational forms were managed in order to exploit the environmental resources
within a niche. Aldrich highlighted that the process of organizational change meant
organizations were moving toward a better fit with the environment. He examined this
movement (organizational change) under three stages: variation, selection, and
retention. The general principle was that variation generates new material from which
environmental selection was made, while retention mechanisms preserved the selected
form.

Moreover, Aldrich (1979) pointed out the importance of environmental selection


relative to intra-organizational factors as a critical difference between his model and
the more traditional view. He acknowledged the possibility of exercising strategic
choices, but he argued that at least three environmental conditions limit the decision-
makers to realize the choices. First, organizations could not exploit many
opportunities due to economic and legal barriers. Second, individual organizations did
not have enough power to influence the environment. Third, the distortions of the
decision-makers’ perceptions of the environment limited the possible range of truly
strategic choices. These limitations severely constrained the decision-makers’ ability to
change either their environmental niches or their organizational forms.

Finally, he concluded that the natural selection model was a general one, which may
be applied to any situation where the three stages are present. When the three
conditions were met, an evolution of better fit to the selective system became
inevitable. He emphasized that a better fit did not mean that there is only one fit.
Selection was a matter of relative superiority over other forms.

Unlike Aldrich (1979) who analyzed the concept of fit at the macro/industry level and
downplayed the manager’s role in choosing strategies to attain organizational fit,
Chakravarthy (1982) explored the concept at the micro level and believes that the
352 | P a g e
latitude for experimentation available to managers determined the ability of
organizations to achieve a fit. He argued that constant pressure on short-term
performance could make the managers overlook strategic goals. In addition, the extent
of financial risk allowable for managers could determine whether or not they can be
proactive in anticipating the environmental changes. The greater the risk allowable for
managers, the more proactive strategies could be explored, and vice versa.
Chakravarthy (1982) also revealed that information-processing ability of organizations
and their material resources such as input material, finance and technology also
determined the adaptive abilities of organizations. Organizations that had high
adaptive abilities may prefer to take proactive strategies, while low- adaptive
organizations were more likely to choose defensive strategies.

The above phenomena indicate the existence of disagreements among scholars with
regard to organizational adaptation, which leads to two contradictory schools of
thought: environmental determinism and strategic choice voluntarism.

The determinism school believes that organizational life is determined by intractable


environmental constraints. It cannot easily adapt to different niches, since
environmental factors such as macroeconomic, social and political forces overpower
strategic management action in the long run. On the other hand, the voluntarism
school considers the environment as the domain in which the managers define and
enact their strategies. Astley and Van de Ven, (1983). Hrebiniak and Joyce (1985)
deemed that the environment and managerial choice were not mutually exclusive. The
two factors interacted with one another, and could be the independent variables in the
process of fit. Combining these two factors in a diagram, Hrebiniak and Joyce (1985)
identified four quadrants representing four possible conditions faced by organizations.
The first quadrant denoted a condition with low strategic choice and high environment
determinism, and was similar to the underlying assumptions of the determinism
school. In this condition, managerial action was obviously limited and constrained by
the environment. Organizations under such conditions must fit or were selected out by
the environment: this included companies operating in perfectly and imperfectly
competitive industries. The second quadrant was characterized by high levels of both
strategic choice and environmental determinism. Under these conditions, many
external forces affected and constrain decision-making; nevertheless, the organizations

353 | P a g e
concerned benefit from the availability of choice. Large companies in highly regulated
industries and multi-product or multi-divisional companies with little market and
technological relatedness were typical examples of organizations in the second
quadrant. In direct contrast to the first, the third quadrant represented conditions
with high strategic choice and low environmental determinism. Like the school of
strategic choice, organizations working under these conditions could deliberately
define and enact policies and strategies, and otherwise influence their particular
environmental domain.

The lack of environmental constraints made it easier for them to introduce innovations
and engage in proactive behavior. Finally, the fourth quadrant stood for a low level in
both strategic choice and environmental determinism. These conditions indicated that
the organizations couldn’t capitalize on even a benign and munificent environment,
due to lack of innovation, proactive behavior, internal capabilities, or inappropriate
competencies.

Hrebiniak and Joyce (1985) emphasized that the process of adaptation is dynamic.
The position of an organization might shift as a result of strategic choices or external
environmental changes. By organizational control over scarce resources, managers
were still able to exercise their strategic choices, although the nature and impact of
the actions would vary according to organization-environment context. This view
supported the concept of fit proposed by Miles and Snow (1984): a concept based on
the actual process of fit. Miles and Snow (1984) defined “fit” as a process or a state - a
dynamic search that sought to align the organization with its environment and to
arrange resources internally to support that alignment. They considered the basic
alignment as strategy and termed the internal arrangement as organizational
structure and management process. Their framework consisted of four main
possibilities, which include minimal, tight, early, and fragile fits.

Based on a previous study of Snow and Hrebiniak, (1980), they concluded that
organizations operating in a competitive environment called for minimal fit to survive.
They found only organizations classified as Defenders, Prospectors, and Analyzers
operated their strategy effectively, since they met the requirement of minimal fit, while
organizations grouped as Reactors were generally ineffective, because their strategies
were poorly articulated, unsuitable to the environment, or misaligned with
354 | P a g e
organizational structure and management systems. Unless these organizations were
protected by government regulations, they have to adjust their behavior or fail.

In addition, unlike the minimal fit, which did not guarantee an excellent performance,
organizations achieving tight fit could achieve outstanding performance. Referring to
the works of Drucker (1969) and Peters and Waterman (1983) who studied many
successful companies in the U.S., Miles and Snow (1984) concluded that excellent
performances of these companies are the result of the achievement of tight fit both
externally with the environment and internally among strategy, structure and
management process. In these conditions the strategy, structure, and process were
well understood by all members at all level of the organization. Every member from
front office to top managers clearly comprehended their roles and responsibilities in
the attainment of the ultimate goals of the organization.

However, the tight fit was not straightforward and easy to achieve. It involved complex
and long processes. It was usually preceded by an early fit that was the discovery and
articulation of a new organizational form. Miles and Snow (1984) asserted that not all
inventions could provide organizations with competitive advantages over a
considerable period of time. Some innovations, such as patenting a particular product
or technology, novel product design, or developing new distribution channels could
only offer organizations temporary competitive advantages, because sooner or later,
competitors could imitate or improve upon the innovation, which made the advantages
disappear. Success in inventing a new organizational form, on the other hand, could
enable an organization to hold the competitive advantage in the longer term.
Competitors would have some difficulties, or at least will take a long time to copy the
new form completely. Miles and Snow illustrated the success of General Motors, Sears
Roebuck, and Hewlett-Packard in applying new forms of divisional organization
structure as part of their diversification strategies. A new structure facilitated General
Motors’ steady improvement in its profit, even in the Depression and World War II, and
such innovations had delivered sustained achievement to Sears Roebuck and Hewlett-
Packard.

As any environment is dynamic and always changing, it is always possible for the
degree of organization – environment fit to weaken. Miles and Snow (1984) emphasized
that organizations must adjust their strategies, structures, or processes in response to
355 | P a g e
environmental changes. However, some organizations might be unable or unwilling to
adjust themselves to extreme environmental jolts. In such conditions, the deterioration
of fit could actually lead to a misfit. Miles and Snow further stated that the external
business environment was not the only the cause of declining fit, but an organizations’
internal processes could instigate the decline. For example, the failure of managers to
follow deliberate changes in strategies with appropriate structural and managerial
adjustments could produce misfit. This could happen when the managers did not
comprehend the strengths and limitations of alternate organizational forms. They
might develop voluntary changes in internal structure and management process
without considering their effects on strategy and market responsiveness in the long
run. Similarly, managers who did not fully understand the alternate forms might
regularly make minor changes to accommodate demands for which the systems were
not designed. However, as this happens over time, the changes might gradually
unravel the entire system. These phenomena indicate that the organization’s fit may
be quite fragile in relation to changes in the external environment and to unintended
internal unravelling.

In addition to the process of fit in regard to voluntarism-determinism, some scholars


also criticized research on the concept of “fit” with regard to its lack of
conceptualization and its simple bivariate approaches. Basically the need for
conceptual clarification in the early development stage of the concept was emphasized.
Van de Ven (1979), for example, identified four different conceptual meanings of fit,
which could alter the essence of Aldrich’s (1979) concept of the relationship of
organization and environment. Similarly, Schoonhoven (1981) argued the need to
develop a more detailed specification of fit. In the absence of this kind of specificity,
researchers tended to present different ideas of “fit” such as contingency, consistency,
match, congruence, or alignment, which, in turn, result in to inconsistent outcomes
and hampering of the theory building process (Drazin and Van de Ven, 1985; Fry and
Smith, 1987). Moreover, as organizations faced many contingency factors, it was not
possible to use simple bivariate approaches to analyze them and to make an accurate
conceptualization of fit. Organizations did not only try to fit their strategies to the
environment, but also attempted to fit the strategies to their unique competencies.
This indicated the multiple dimensionality of the concept of fit, which therefore needed
more than simple bivariate approaches (Zajac, Kraatz, and Bresser, 2000). To
356 | P a g e
overcome the above handicap Venkatraman (1989) proposed six different concepts of
fit. By reviewing research on strategic fit, he developed concepts of fit- based
relationships in terms of the degree of specificity of functional form and the choice of
anchoring specification. These included fit as moderation, mediation, matching,
covariation, profile deviation, and as gestalt. The concepts not only described the
relationship between two variables, but also portrayed the relationships among
multiple variables. For example, fit as moderation or mediation considered only two
variables, while fit as covariation, profile deviation and gestalt involved multiple
variables. In addition, Venkatraman also offered possible analytical tools for each
concept, to provide a link between the concepts and theory testing. Finally, he
encouraged the use of multiple concepts of fit to gain more useful and powerful
operationalization, data collection and analysis, and interpretation of the results.

Perspective on Strategy Research

Since the main tasks of managers are to develop and utilize a strategy that aligns the
organization’s capabilities with the opportunities and constraints present in its
environment, and to arrange resources internally to support the alignment (Miles and
Snow, 1984, 1994), the concept of fit becomes a central thrust in strategy research.
However, apart from diverse conceptualizations of fit as mentioned in the previous
section, the strategy research based on their focus can be classified into two distinct
school of thoughts: strategy content and strategy process (Jemison, 1981; Rajagopalan
and Spreitzer, 1997).

Strategy Content School

Research in strategy content specifically concentrates on what was decided (the


content of the strategy) and focuses on the relationship between strategy and
performance under different conditions/environments. In other words, strategy
content studies place more emphasis on investigation of the external business
environment as opposed to the internal one. Industrial organization studies
contributed to the existence of this school of thought. Based on the Bain/Mason
paradigm (Structure–Conduct–Performance), they investigated and promote how
industries in which the organizations operate influence their strategic choices (Porter,
1981). For example, Hambrick (1983) focused on the influence of industry

357 | P a g e
environment on strategic choice. He found organizations operating under stable
environments tended to select a cost leadership strategy. However, if organizations
work in unstable environments, they would select a differentiation strategy, as the
dynamism of the environment limited their ability to utilize a cost leadership strategy.
In addition, the environment determined the strategic orientation of an organization.
Environmental factors such as complexity, dynamism, and hostility correlated
negatively with a proactive strategic orientation, but they associated positively with a
defensive one (Tan and Litschert, 1994).

Technological changes can make the environment become hostile and uncertain. Such
changes can make the useable life of sophisticated machines shorter than anticipated,
thus forcing organizations to undertake technological innovation to stay in the market.
However, organizations use different strategies to cope with these changes. For
example, Dvir, Segev, and Senhar (1993) classifed the strategies used in electronic and
computer companies as Prospector, Analyzer, and Defender. Their study indicated
monitoring and adopting technological innovations in the “Analyzer” companies
correlated positive, significant to short term performance, while in the “Defender”
companies these technological activities associated positive, significant to both short
and long term performance. Prospector companies, on the other hand, could only gain
better long-term performance from monitoring and adopting technological innovations.
The technological innovations related negatively, but not significantly, to short-term
performance, indicating that the companies invest more aggressively in the
innovations. However, market conditions where the organizations operate can also
determine the strategic choice. Luo and Park (2001) revealed that organizations with
“Analyzer” as the strategic orientation achieved better performance than organizations
utilizing “Prospector” or “Defender”. They suggested that organizations operating in the
emerging Chinese market should be innovative and adaptive but not too proactive and
risk taking.

Moreover, the strategy of choice relates to the organizational structure. Grinyer, Yasai-
Ardekani, and Al Bazzaz (1980) suggested that while strategy positively related to
structure, environment correlates negatively with performance. As strategy reduced
the uncertainty of contingencies, organizations tended to use bureaucratic uncertainty
reduction, differentiation, or integration devices. On the other hand, they might

358 | P a g e
employ organic uncertainty reduction, differentiation, or integration devices, when the
strategy increased the uncertainty of contingencies (Miller, 1987). A better fit between
strategy and structure promoted more effective coping with environmental turbulence,
which in turn led to better performance (Adeyemi-Bello, 2000). In conclusion, studies
on strategy content indicate that to attain better performance, organizations must
align their strategy to the environment [Venkatraman and Prescott 1990). Misfit
between the strategy and structure in relation to their environment will lead to lower
performance (Naman and Slevin, 1993). Broad scanning information on customers
and competitors can help organizations to attain the right fit (Beal, 2000). However as
mentioned earlier, the studies do not take into account the internal environment of the
organizations. Table 2.1 below provides a summary of strategy content research in
more detail. Whilst not including all relevant studies, it nevertheless presents a
representative selection of known studies on strategy content.

Table 2. 1: Summary of Selected Empirical Studies on Strategy Content

Authors Subject/Researc Variables Analytical Key Findings


h studied Tools
Method

Grinyer, Senior manager of The degree of Correlation Significantly


Yasai- 48 companies in diversification Analysis positive
Ardekani, the UK. Data were (strategy), correlation
& Al- collected through degree of between strategy
Bazzaz cross sectional divisionalizatio and structure.
(1980) survey and n (structure), Good match or fit
structured company’s size, between strategy
interview. technological and structure can
change reduce the
(environment), environmental
and ROI pressure.
(performance). Environment
correlates
negatively and
significantly with
the performance.
A better fit
between strategy
and structure
promotes more
effective coping
with

359 | P a g e
environmental
turbulence which
in turn leads to
better
performance.

Hambrick 164 capital good Strategy Regression, Cost leadership


(1983) firms in the PIMS (differentiation, and Cluster strategy is found
database. cost leadership, Analysis to be a fit only in
and focus), disciplined capital
performance goods
(ROI). manufacturers
(DMs) operating
under a stable
environment. The
strategy seems
not to suit
aggressive
manufacturers of
complex capital
goods (Acs), since
their dynamic
environments
limit these
manufacturers’
abilities to
maximally utilise
the strategy. In
addition,
differentiation
strategy is not
only suitable for
the ACs but also
for the DMs to
attain high profit.
However, while
the ACs tend to
emphasize
product quality,
affinity with
users,
technological
protection, and
product
innovation to cope
with
environmental
changes, the DMs
focus on product
360 | P a g e
quality and
image.
This study
indicates that
even though
Porter’s generic
strategies can
promote good
performance, not
all generic
strategies can fit
to a particular
industry. The
study also
indicates that
there are some
variations within
the broad context
of generic
strategy. The
match between
the strategy and
its environment
determines the
performance of
the manufacturer.

Prescott 1,638 business 8 environment Cluster Supporting the


(1986) units in the PIMS variables, 4 and hypothesis,
data base strategy discriminan findings indicate
variables, and t analysis, that environment
ROI as correlation, modifies the
performance and strength, not the
variable. regression form, of the
analysis strategy –
performance
relationship

Miller 161 Senior Strategic Correlation Finds significant


(1987) Managers on variables Analysis correlation
major US firms, (complex between strategy
and 110 Senior product and structure.
Manager in the innovation, Strategy reduces
Canadian and marketing the uncertainty of
Australian firms. differentiation, contingencies so
Questionnaires breadth, and firms tend to use
were used for conservative bureaucratic
collecting data. cost control), uncertainty
structural reduction,
361 | P a g e
variables differentiation, or
(bureaucratic & integration
organic in term devices. However,
of uncertainty they may employ
reduction, organic
differentiation, uncertainty
and reduction,
integration), differentiation, or
and integration
environmental devices when
variables their strategies
(dynamism, increase the
heterogeneity, uncertainty of
and hostility)` contingencies.
Strategy
significantly
correlates to its
environment:
firms operating in
a dynamic
environment
usually utilize
complex product
innovation,
breadth
innovation, or
marketing
differentiation
strategies, while
firms facing a
more stable
environment tend
to employ
conservative cost
control.

Venktra 1638 firms (Phase 8 Ordinary Three steps of


m an & I) environmental Least systematic
Prescott And 821 firms variables, 17 Square, analysis support
(1990) (PhaseII). PIMS strategy Cluster, the hypothesis
database was the variables, and Regression, that the
source of data. performance and alignment
(ROI) Correlation between strategies
Analysis and their
environments
may significantly
increase
performance.
Replication of the
362 | P a g e
model, using
different data,
provides similar
results, indicating
further support
for the previous
findings.

Naman & 82 senior Environmental Regression, Misfit influences


Slevin managers of turbulence, Correlation the performance
(1993) manufacturing entrepreneurial Analysis, negatively and
companies. style, and significantly.
Data were organization ANOVA Since the misfit
collected structure, measures the
through mission sum of the
mailed strategy, and difference
questionnaire performance. between desired
and reported
levels of
entrepreneurship,
organizational
structure, and
mission strategy,
the authors
suggest the use of
these variables to
determine the
organization –
environment fit.

Dvir, 76 managers of Strategy Correlation, Monitoring and


Segev, & electronic and (Prospector, t-test and adopting
Shenhar computer Analyzer, and Mann- technological
(1993) companies. Data Defender), Whitney test innovations in
Collected through technological Analyzer
questionnaires progress companies,
and (monitoring, correlate
interview and adopting positively and
technological significantly to
innovation), short term
performance performance,
(short, and long while in Defender
term) companies these
technological
activities are
associated
positively and
significantly with
both short and
363 | P a g e
long term
performance.
Prospector
companies, on the
other hand, can
only gain better
long-term
performance from
their monitoring
and adopting
technological
innovations. The
technological
innovations relate
negatively but not
significantly to
short-term
performance,
indicating that
the companies
invest more
aggressively in the
innovations.

Tan & 97 managers of Environment Canonical All environmental


Litschert electronic firms in (complexity, Correlation, forces
(1994) China. dynamism, and Pearson- (complexity,
Questionnaires hostility), Correlation, dynamism, and
were used in strategic and hostility) influence
collecting data. orientation Multiple the strategic
(analysis, Regression orientation of
defensiveness, Analysis electronic firms.
futurity, While they
degrees of risk correlate
and negatively with
proactivity), proactive strategic
and orientation, they
performance associate
positively with
defensive strategic
orientation. The
match of a
strategic
orientation to its
environment
yields better
performance both
in terms of overall
performance and
364 | P a g e
profit.

Adeyemi- 187 CEO of Banks Strategy, Correlation, The degree of


Bello in US. Mailed internal and competition,
(2000) questionnaires organizational Moderated dynamic strategy,
were used to structure, Regression and organic
collect data. degree of Analysis structure
competition, influence the
and ROA. banks’ ROA. The
fit between the
degree of
competition,
strategy, and
organizational
structure
influences the
ROA positively
and significantly.

Beal 101CEOs of small Industry life Factor While the


(2000) firms. Mailed cycle Analysis scope of
questionnaires (introduction, and ANOVA scanning
were used to growth, influences
collect data maturity, and the alignment
decline), between strategy
strategy (low and environment,
cost, the frequency of
differentiation scanning does not
in innovation, indicate any effect
marketing, on the alignment.
quality, and in It seems that
service), broad scanning
scanning information of
frequency, customers and
scanning scope, competitors
and facilitates the
performance. strategy –
environment
alignment for
firms operating in
both growth and
mature
industries.

Luo & 113 Deputy and Environment Factor, Environmental


Park General Manager (complexity, Canonical, factors of the
(2001) of MNC dynamism, and and Chinese market
subsidiaries in hostility), Multiple affect the strategic

365 | P a g e
China. Data were strategic Regression orientation
collected through orientation Analysis. of MNC
questionnaire (Prospector, subsidiaries.
Analyzer, and Among the three
Defender), and strategic
performance orientations,
(ROA, sales Analyzer has the
growth, and strongest
competitive relationship with
position) the environmental
factors. This
indicates that
firms use
Analyzer as a
mechanism to fit
the operations to
their
environments.
Firms with
Analyzer as the
strategic
orientation
achieve better
performance than
did
firms with
Prospector or
Defender.
Conclusion:foreig
n firms should be
innovative and
adaptive but not
too proactive and
risk taking to
operate in the
emerging Chinese
market.

Strategy Process School

In contrast with the strategy content, studies of the strategy process concentrate
primarily on actions that lead to and support the strategy. These studies focus on the
influence of environment, organization, and the decision process on organizational
performance. However, Rajagopalan, Rasheed, and Datta, (1993) found that only
limited studies existed that investigate the direct impact of environment on strategy

366 | P a g e
decision-making. Most of the studies considered environmental influences only
implicitly, as part of the context of the decision process.

Studies of strategy process focused more on top managers as organizational factors


influencing the decision process, as they played an important role in determining the
direction of organizations. Bourgeois III (1985), for example, indicated that the
divergence of managers’ perception of environmental uncertainty (PEU) from the true
state of environmental volatility correlated negatively and significantly with economic
performance, while managers’ disagreements on the PEU and strategic goals had
positive and significant correlations with the performance. He concluded that the
variation in PEU and goals amongst managers could promote higher performance, so
long as their perception of the environment matched with the true conditions.
Supporting this finding, Priem (1994) uncovered a positive relationship between CEOs’
judgement of the strategy-structure-environment alignment and the realised strategy-
structure-environment fit of the organizations. Although, he did not find any support
for CEO characteristics and the strategy decision process as determinant factors of the
CEO judgement, he believed the judgement facilitates the attainment of better
performance.

Functional experience of the managers may affect their strategic orientation. Managers
with greater experience in marketing tend to have a greater willingness to take risks
and greater tolerance for ambiguity. Such managers fit for the successful
implementation of “Build” strategy, but not for the “Harvest” one (Gupta and
Govindarajan, 1984). Beal and Yasai-Ardekani (2000) supported this fit between
managers’ experience and their competitive strategy. They found that innovation
strategy might succeed when managers have greater experience in R & D, while
greater experience in engineering might lead to successful implementation of quality
differentiation strategy.

Accounting experience, on the other hand, became a prerequisite, combining with


other requisite expertise to facilitate execution of hybrid strategies involving low cost
leadership. In addition to functional experience, the personality of managers might
influence their strategic choice (Miller and Toulous, 1986). Flexible managers tended
to use niche strategy to market their products. They were much more reactive, risk
taking and more intuitive in formulating their strategies. They used informal structure
367 | P a g e
with some authority delegation in running the organizations. On contrary, managers
needing achievement tended to use breadth market strategy. They emphasized
analytical, proactive decision-making and a sophisticated, formal structure. Managers
with an internal locus of control, on the other hand, preferred to employ product
innovation strategy. They were more innovative, risk taking, proactive and futuristic.
They emphasized specialisation and on long-term rather than short-term results
(Miller, Kets De Vries, and Toulouse, 1982).

Moreover, the types of decision-making used determine the performance of


organizations. Frederickson (1984) found that the broad-based components of
strategic decision-making, such as situation diagnosis, alternative generation and
evaluation and decision integration led to better performance for organizations that
operated in a stable environment. Such a rational model of decision-making enables
the manager to make selective decisions to exploit the available opportunities.
However, this rational model was not appropriate for organizations working in an
unstable environment. In a changing and dynamic environment, an incremental
approach to decision-making was more likely to lead to superior performance, due to
its speed in coping with environmental changes (Frederickson and Mitchell, 1984;
Judge and Miller, 1991). To speed the decision-making, managers in a high-velocity
environment used experienced counselors to assist in accelerating the development of
alternatives and in reducing ambiguity, to engage in active conflict resolution to
achieve consensus, and to integrate the strategic decisions with one another and with
tactical plans (Eisenhardt, 1989).

In addition, acceleration of the strategy making process could also be achieved


through delegation of authority to functional managers. This authority delegation
could eliminate political behavior among the managers (Bourgeois III and Eisenhardt,
1988), since this behavior emerged when the power was centralized in the top
managers (Eisenhardt and Bourgeois III, 1988). Authority delegation also encouraged
middle managers’ participation in decision-making and helped them to implement the
strategy itself (Floyd and Wooldridge, 1994). Middle managers (MMs) had greater
influence in low risk/return decisions than high ones, and in the implementation of
strategic decisions than in their formulation. However, the longer the MMs had worked
under their superior the greater their influence on strategic decisions (Schilit, 1987).

368 | P a g e
Strengthening these findings, Floyd and Wooldridge (1 992a) indicated that the nature
of MMs’ involvement, especially in championing alternatives, facilitating adaptability,
and implementing deliberate strategy varied with strategy type. MMs in Prospector
firms reported a greater level of championing activity than MMs in Defenders. They
also carried on higher levels of facilitating and implementing activities than MMs in
both Analyzers and Defenders. In general, implementing deliberate strategy was the
highest level of activity and varies with the strategy type. Moreover, boundary
spanning MMs had higher levels of both upward and downward strategic influence
than non-boundary spanning ones, and the different is greater for upward influence.
Organizations gained higher performance levels when their MMs had more uniform
levels of downward influence and more varied levels of upward influence (Floyd and
Wooldridge 1997). However, the motivation of MMs to raise strategic issues with top
management depended on the willingness of top management to listen, the
supportiveness of the organizational culture, the level of competitive and economic
pressures, and the level of organizational change. Fear of negative consequences,
downsizing conditions, uncertainty (about the future in general, about the future of
organization and the like), and conservativeness of organizational culture might cause
the MMs to be reluctant in raising strategic issues (Dutton, Ashford, O’neill, Heyes,
and Wierba 1997).

In conclusion, studies on the strategy process agree that internal structural fit helps
organizations to attain superior performance (Powell, 1992). However most of the
studies concentrate on strategy formulation processes and tend to ignore the
implementation aspects of the strategy. The lack of the implementation research is
due to the complexity of multiple issues involved in the implementation process,
attributed to the rapid changes in the business environment and incompatibility
between he leadership/managers skills and competence, organizational structure, and
systems and processes in the organization. The most common assumption used by
researchers is that strategy implementation will be straightforward (Noble, 1999).
Table 2.2 provides a summary of empirical studies on the strategy process. It should
be noted that the table does not include all the relevant studies, but includes a
representative sample.

369 | P a g e
Table 2. 2: Summary of Selected Empirical Studies on Strategy Process

Authors Subject/Research Variables Analytical Key Findings


Method studied Tools

Miller, 33 Chief Executive’s locus Correlation All hypotheses


Kets De Executives of of control Analysis strongly
Vries, & Canadian firms. (internal & supported. The
Toulouse Data collected external), internal locus of
(1982) through structured strategy control
interviews. (innovation, risk significantly
taking, pro- correlates to
activity, and strategy,
futurity), structure, and
structure environment,
(scanning, whilst internal
technocratisation control has
, and strongest
differentiation), correlation with
and environment the strategy
(dynamism and making behavior.
heterogeneity). Executives
employing
internal control
more innovative,
risk taking,
proactive, and
futuristic than
those using
external control.
Partial correlation
analysis indicates
that relationship
between internal
control and
environment is
mediated by the
strategy. To the
degree that the
strategy variables
are controlled for,
internal control
ceases to have
significant
correlation with
the environment.
Similar analysis
is also performed

370 | P a g e
to investigate the
nature of
relationship
between internal
control and
structure, by
controlling the
influence of the
strategy: results
indicate that
internal control
relates to the
structure
indirectly.
Internal locus of
control may affect
the strategy,
which in turn has
an impact on the
structure and
environment.

Gupta & General Managers SBU’s strategy Multiple Greater


Govindar of 58 SBUs. Mail (Build, Hold, Regression experience
ajan questionnaires are Harvest, and in
(1984) used for data Divest), marketing/sales,
collections. experience in greater
marketing/sales, willingness to
willingness to take risks, and
take risk, greater tolerance
tolerance of for ambiguity
ambiguity, contribute to the
effectiveness of effectiveness of
the strategy strategy
implementation. implementation
in the case of
Build strategy
but impede in
Harvest ones.
Findings support
the view of
matching the
manager’s
characteristics to
either the
strategic mission
or stage of
product life cycle.

371 | P a g e
Frederick 109 CEOs of Comprehensiven Partial A comprehensive
son & sawmills and ess of strategic Correlation strategic
Mitchell planning firms. decision Analysis decision
(1984) Structured (situation process
interviewed were diagnosis, associates
used to collect alternative negatively and
data generation, significantly with
alternative the
evaluation, and average
decision return on
integration), assets (AROA).
performance A similar
(AROA, & association exists
% change in between the
gross sales) comprehensivene
ss of the strategy
and percentage
change in gross
sales, though this
association is
quite weak. Since
participants of
the study work in
an unstable
environment, the
authors conclude
that the
incremental
model of
decision-making
is more likely, in
a changing and
dynamic
environment, to
result in superior
performance.
Such
environments are
very complex,
unpredictable,
and prevent the
high level of
integration
needed by the
comprehensive
approach. By
employing the
incremental
model, managers
372 | P a g e
can speed
decision- making
and work flexibly
to cope with the
changes.

Frederick 152 CEOs of paint Comprehensiven Partial Situation


- son and coatings firms. ess of strategic Correlation diagnosis,
(1984) Data were decision Analysis alternative means
collected through (situation of generation and
structured diagnosis, evaluation,
interviews alternative decision
generation, integration, and
alternative overall
evaluation, and comprehensive
decision measures
integration), correlate
performance positively and
(AROA, & significantly with
% change in the average
gross sales) return on assets,
but not with the
change in sales.
The authors
conclude that
rational model of
decision-making
is characterised
by a
comprehensive
process, which is
more suitable in
a stable
environment,
since the firms
studied operate
in a stable
environment. The
managers must
make selective
decisions to
exploit the limited
opportunities.

Bourgeois 99 CEOs and Environmental Correlation The divergence


III (1985) members of top volatility Analysis of managers’
management team. (commercial and perception of
Interview, mailed technological), environmental
questionnaire were perceived uncertainty from
373 | P a g e
used to collect environmental the true state of
data. uncertainty, environmental
strategic goals, volatility
and economic correlates
performance. negatively and
significantly with
economic
performance.On
the other hand,
managers’
disagreements
on both
perceived
environmental
uncertainty (PEU)
and
strategic
goals have
positive and
significant
correlations with
performance. In
conclusion,
variation in PEU
and goals
amongst
managers can
promote higher
performance, so
long as their
perception of the
environment
matches with the
true state of
affairs.

Miller & 97 CEOs of firms CEO personality Partial Flexible CEOs


Toulouse in Quebec. (flexibility, need Correlation tend to use niche
(1986) Interviews were for achievement, and strategy to
used to collect and locus of Regression market their
data. control), strategy Analysis products. They
(innovation, are much more
marketing reactive, risk
differentiation, taking and more
and focus), intuitive in
strategy making formulating their
(future/planning, strategies. They
analysis, use informal
proactiveness, structure, with
374 | P a g e
and risk taking) some authority
structure delegation in
(delegation, running the
formalization, organization.
specialisation, Flexible CEOs
technocrats and working in small
professional, firms within a
liaison devices, stable
and control), environment can
performance, size realise superior
of firm, and performance. On
environmental contrary, CEOs
context who need
achievement tend
to use breadth
market strategy.
They emphasize
analytical,
proactive
decision- making
and a
sophisticated,
formal structure.
This dimension of
a CEO’s
personality is not
associated with
organizational
performance.
CEOs with
internal locus of
control on the
other hand,
prefer to employ a
product
innovation
strategy that
requires long
term planning for
the future of their
products. They
emphasize
specialisation and
long-term, rather
than short-term,
results.

Schilit 60 middle-level Demographic Descriptive, MLMs have


(1987) managers (MLMs) (organizational Correlation greater influence
375 | P a g e
from 57 size and sector, , and in low risk/return
organizations. functional area, Stepwise decisions than
Data were and number of Regression high ones, in the
gathered years working for Analysis implementation
through superior), of strategic
questionnaires and riskiness/return decisions than
participants’ strategic their formulation
records decisions (44 process, and in
types), upward private
influence organizations
(nature, method, than in public
and outcome) ones. In addition
the longer the
MLMs work for
their superior the
greater their
influence on the
strategic
decisions. They
may use rational
or persuasive
arguments to
influence
strategic
decisions.

Bourgeois 4 microcomputer Decentralisation Descriptive Effective


III & companies. Data of power, and executives in
Eisenhar were gathered decision process Qualitative high-velocity
dt (1988) through semi- (comprehensiven Analysis environments
structured ess, newness of employ
interviews with alternatives), comprehensive
CEOs and top decision speed, analysis,
managers, and emphasising
questionnaires, performance. product
and secondary innovation and
sources. speed of decision-
making. They
delegate authority
to functional
managers for
strategic
decision-making
and reducing
political behavior.
Comprehensive
analysis
enhances the
quality of initial
376 | P a g e
decisions, and
decentralisation
of power supports
quality further
through
adaptablity to
environmental
changes.

Eisenhar 8 microcomputer Centralisation of Descriptive Politics emerge as


dt & companies. Data power, decision and the result of
Bourgeois were gathered process (political Qualitative centralisation of
III (1988) through semi- behavior, Analysis power. In the
structured conflict, stability absence of power
interviews with of alliance), centralisation,
CEO and top performance conflicts do not
manager, (CEO’s ranking lead to the use of
questionnaires, relative to other politics. Politics
and secondary companies in are managed
sources. industry, sales through stable
growth, and alliances, based
return on sales) on demographic
characteristics,
not on issue-
specific
agreements. The
existence of high
politics within top
management
results in poor
performance,
both in economic
and decision-
process
outcomes.

Eisenhar 8 microcomputer Decision process Descriptive To speed


dt companies. Data (real- time and decision-making,
(1989) were gathered information, Qualitative top management
through semi- multiple Analysis teams in high
structured simultaneous velocity
interviews with alternatives, two- environments
CEO and top tier advice tend to use more
manager, process, information, to
questionnaires, consensus, consider more
and secondary decision simultaneous
sources. integration), alternatives for
decision speed, comparison, to
performance use experienced
377 | P a g e
counsellors to
expedite the
development of
alternatives to
help reduce
ambiguity, to
engage in active
conflict resolution
to achieve
consensus, and
to integrate
strategic
decisions with
one another and
with tactical
plans. Fast
strategic
decision-making
results in better
performance.

Judge & 86 CEOs and Top Decision speed, Correlation When a number
Miller Managers in decision Analysis of simultaneous
(1991) Biotechnology, importance, alternatives are
Hospital, and organizational considered apart
Textile companies. size, number of from the
Interviews were alternatives, environmental
used to collect data. board context, decision
experience, speed increases:
performance however, board
experience tends
to reduce the
speed. Only
biotechnology
companies
operating in a
high-velocity
environment
enjoy the
advantage of
better
performance as a
result of speedy
decision-making.
The relationships
amongst board
experience,
speed and
performance
378 | P a g e
tend to increase
as environmental
velocity
increases, while
the relationship
between speed
and the number
of alternatives
is
unchanged in
any
environmental
context.

Powell 113 CEOs of 544 Structure Correlation Organizational


(1992) firms listed in Dun’s (controls, Analysis alignment,
Million Dollar formalization, especially
Directory and standardisation, internal
Standard and Poor’s liaison devices, structural fit,
Register. Mail centralisation, size-planning,
survey was used to automation, comprehensivene
collect data. integration, and ss and a CEO
differentiation), with an internal
formal planning locus of control
(goal setting, help firms to gain
scanning, supernormal
analysis, and profits, and act
overall as sources of
comprehensive), competitive
strategy content advantage.
(production cost, Further analysis
differentiation, shows that these
innovation, and organizational
market breadth), alignments are
firm size, and not generated by
performance. chance or luck,
but are the result
of administrative
skill, thus
alignment skills
stand alongside
industry and
strategic
positioning. The
study suggests
the importance of
organizational
factors,
particularly the
379 | P a g e
alignment of key
variables, as
additional
sources of
competitive
advantage.

Floyd & 259 middle Strategic type Correlation, MMs’


Wooldri managers (MMs) (Prospectors, Factor involveme
dge Questionnaires were Analyzer, and Analysis, nt activities,
(1992) used in data Defender), and especially
collection manager MANOVA championing
involvement alternatives,
(facilitating, facilitating
synthesizing, adaptability, and
championing, implementing
and deliberate
implementing). strategies vary by
strategy type.
MMs in
Prospector firms
report a greater
level of
championing
activity than
MMs in
Defenders. They
also carry on
higher levels of
facilitating and
implementing
activities than
MMs in both
Analyzers and
Defenders.
Implementing
deliberate
strategy is the
highest level of
activity.
However, MMs’
involvement in
implementing
activity varies
according to the
type of strategy
selected.

Priem CEOs and top CEO judgment ANOVA, The CEOs


380 | P a g e
(1994) managers of 33 policies, strategy MANOVA, judgement in
manufacturing (cost leadership, Descriptive, decision-making
companies (multiple innovative Metric concerned with
responses). Data differentiation, Conjoint, the strategy-
were collected and marketing Correlation, structure-
through mailed differentiation), Multiple environment
questionnaires structure Regression, alignment is
(formulation, and Cluster strongly related
decentralisation, Analysis to the realised
specialisation, strategy-
control, and structure-
liaison devices), environment fit
strategy making of the firm. CEOs
(scanning, judgements
analysis, and following the
planning prescriptions of
process), CEO contingency
characteristics theory result in
(age, education, better
tenure) and performance. No
performance support found
for CEO
characteristics
and the strategy
making process
as determinant
factors of CEO
judgement. Even
so, rationality in
strategy-making,
represented by
scanning,
analysis, and the
planning
process, helps
CEOs to attain
better
performance.

Floyd 259 middle Boundary Multiple Findings indicate


and managers from 25 spanning Regression that boundary-
Wooldri firms. Data position, and ANOVA spanning middle
dge collected through strategic managers have
(1997) mailed influence higher levels of
questionnaires. activity, both upward and
patterns in downward
middle strategic
management influence than
strategic activity, non-boundary
381 | P a g e
performance, spanning ones,
and control and the
variables (tenure difference is
in position, size greater in regard
of organizations, to upward
and industry) influence.
Organizations
attain higher
performance
when their
middle managers
have more
uniform levels of
downward
influence and
more varied
levels of upward
influence. There
is no difference
in the
performance of
organizations
when they are
classified on the
basis of their size
and industry.

Dutton, Study I: Study I: Study I: Study I:


Ashford, 30 middle-level Perceptions of Qualitative The motivation of
O'ne managers (MLMs) of favourable and Analysis MLMs to raise
ill, Regional unfavourable Study II: strategic issues
Hayes, Telecommunication organizational Factor with top
Wierba company in environment for Analysis management
(1997) Midwest USA. selling issues to and Paired depends on the
Interviews were top management. t-Test willingness of top
used to collect data. Study II: management to
Study II: Perception of listen, the
118 middle-level conditions and supportiveness of
managers (MLMs) of factors affect the the
Regional image risk of organization’s
Telecommunication selling issues, culture,
company in demographic competitive and
Midwest USA. (age, gender, and economic
Mailed employment pressures and
questionnaires were tenure) the level of
used to collect data. organizational
change.
However, fear
of negative
382 | P a g e
consequences,
downsizing
conditions,
uncertainty
(about the future
in general, about
the future of
organization and
players, and the
like), and
conservativeness
of organizational
culture may
cause the MLMs
to be reluctant in
raising strategic
issues.
Study II:
Three factors
perceived by
MLMs to
contribute to the
risk of losing
their image: the
violation of
normal
procedures,
political
vulnerability and
distant seller-
target
relationships. No
relationship
between
demographic
characteristics
and the three
factors above,
except for age,
which is
significantly
correlated
with norm
violation
and
political
vulnerability.

383 | P a g e
Beal & 101 CEOs of small Competitive Regression The results
Yasai- manufacturing strategies (low Analysis reveal that a fit
Ardekan companies. Data cost; innovation, between CEOs’
i were collected marketing, experience and
(2000) through mailed quality, & service their competitive
questionnaires differentiations strategy results
and hybrid), CEO superior
functional performance.
experience (R & While innovation
D, marketing, strategy may
engineering, succeed as the
sales, and CEO has greater
accounting), experience in R
performance & D, greater
experience in
engineering may
lead to
successful
implementation
of quality
differentiation
strategy. In
addition,
engineering
rather than
accounting and
sales experiences
may be required
to implement low
cost and service
differentiation
strategies
effectively.
Accounting
experience
becomes a
prerequisite
combining with
other
requisite
expertise to
execute
hybrid
strategy involving
low cost
leadership.

The Roles of the Marketing Manager and Marketing Concept in Organizations

384 | P a g e
As mentioned earlier, organizations must fit with their environment or they fail
(Aldrich, 1979). However, making the organization fit to its environment depends on
the top managements’ perception of the environment, as top management makes
strategic decisions based not on the environment itself, but on their perceptions of
environmental realities. The effectiveness of strategic decisions, therefore, is subject to
the match of top managements’ perceptions of environment and resources capabilities
to their realities (Anderson and Paine, 1975; Glaister and Thwaites, 1993). To increase
the match of these perceptions to their realities, top management should improve the
quantity and quality of the environmental information (Provan, 1989). However, in a
complex and dynamic organizational environment, the top management cannot detect,
interpret, and handle the environmental changes by themselves (Walsh, 1995) as their
information capacity and the time available do not enable them to do so. They must
rely on their middle managers’ support for all strategic information (Barlett and
Goshal, 1994). The middle managers’ inputs will expose top management to the
strategic issues through the viewpoints of those closer to the actual operations of the
organization.

Since middle managers can direct and influence top managements’ strategic decision-
making by presenting the strategic issues in appropriate ways, they have to compete
with the other managers to attract the top managements’ attention (Dutton and
Ashford, 1993). However top management will not place equal value on the strategic
issues provided by their middle managers. As asserted by Pfeffer and Salancik (1978),
those coalition participants who provide crucial resources would have more influence
and control over the organization. In other words, some departments or sub
organizations will be more influential than others, as they are seen as being critical to
the success of the organization as a whole. For example, the increase of unionism in
the 1930’s enhanced the role and influence of industrial departments in large
corporations.

From this perspective, a manager who is responsible for the marketing area would
play a crucial role in providing strategic information to top management to satisfy
long-term needs of customer coalition. In other words, a marketing manager can
persuade top management to implement a marketing concept (Anderson, 1982). The
marketing concept posits that the key to profitability is not current sales volume, but

385 | P a g e
long term customer satisfaction. The only valid definition of business purpose is to
create a satisfied customer, and in doing so, any business enterprise has two basic
functions: marketing and innovation (Drucker, 1969). Top management is responsible
for creating this environment, viewpoint, attitude, and aspiration. These expressions of
marketing concept indicate that the executive must put the customer’s interests at the
top of the firm’s priorities. Its product should be tailored and modified in respond to
changing customer needs. Profit is not the objective, it is just the reward for creating
satisfied customers (Levitt, 1960).

The marketing concept became popular in business in the 1 960s. Evidence indicated
that both large and medium manufacturing firms, to a large extent, adopt the
marketing concept (Hise, 1965).

There was an inclination that large firms were more fully committed to adopting and
implementing marketing concepts than small and medium ones. Similarly, consumer
goods’ firms tended to adopt and implement the marketing concept to a greater degree
than industrial ones (McNamara, 1972). However significant variations in the response
pattern among practitioners and academicians indicated that few firms were able to
implement the marketing concept on a day-to-day basis. Ames (1970) reported that
many firms’ moves to become more marketing-oriented fell into the “trappings” of
marketing rather than the substance. There was no fundamental shift in thinking and
attitudes throughout the firm, and this was what was needed to ensure that everyone
in every functional area placed paramount importance on being responsive to market
needs. If there was no change in thinking and attitude, even most highly developed
marketing operation could not produce any real result.

These inappropriate implementations of the marketing concept not only generated


unintended results but also caused some criticism of the pertinence of the concept.
Bell and Emory (1971) suggested that the businessman's operational interpretation of
customer orientation had not approached the philosophical meaning of providing
customer satisfaction. It appeared that customer orientation had meant little more
than looking to the customer for guidance as to what can be sold for profit. This
implied that customer knowledge was simply a means to persuade or even to
manipulate the customer. The marketing concept was also blamed to the lack
competitiveness of American businesses. The implementation of the concept had led
386 | P a g e
American businesses to cut their R&D investment, resulting in the slow death of
product innovation. The concept had diverted attention away from product and its
manufacture to market research, advertising, selling, and promotion, to the suffering
of product value (Bennett and Cooper, 1981). The emergence of the corporate strategic
planning concept further lessened the adoption of the marketing concept in
organizations. The fundamental purposes of strategic planning were to maintain the
competitive strength of the firm and to improve its internal efficiency, whereas the
corporate objectives were mostly focused on the achievement of certain returns on
investment and market share (Ansoff, 1965). Webster (1988) considered that this
concept viewed market opportunities in terms of the market's growth rate and the
firm's ability to dominate its chosen market segments. In other words, it defined
market as aggregations of competitors, not as customers.

In addition, Webster (1988) believed that the strategic planning concept not only
shifted the management focus on to customers, but also removed the marketing role
in strategic decision-making. This belief was based on Ansoff’s classification of
"marketing strategy" as an operation decision, not a strategic decision. In Ansoff’s
opinion, strategic decisions involved the selection of product-market mix, products to
be offered, and markets to which the products were to be sold. He did not consider
those decisions to be marketing decisions, because he defined marketing as a broad
activity concerned with creating product acceptance, advertising, sales promotion,
selling, distributing the product (including transportation and warehousing) contract
administration, sales analysis, and very importantly servicing the product (Ansoff,
1965 :p.93). This made the role of marketing set back to its traditional role that is just
creating demand for the products. Therefore, it was not so surprising when Webster
(1981) found that many qualified marketing executives preferred to move into a
strategic planning position, than to stay in marketing one.

He also discovered that marketing people failed to think creatively to provide proper
stimulation and guidance for R & D and product development. Marketing people did
not like to take risks and were unable to approach problems in an innovative and
entrepreneurial fashion. Webster (1981) believed that these problems arose because of
pressure of short-term sale volume and financial results on marketing people.

387 | P a g e
Briggadike (1981) noticed this lack of strategic orientation of marketing in the early
80's. He asserted that marketing does have a rich basis for hypothesising about
strategic situations and a growing body of techniques to explore these hypotheses.
Most reported studies, however, involved ad hoc problem-oriented research, with little
attempt to integrate and extend the relationship to other situations. He judged that
many marketers were scientists in the solving problems at brand, or, occasionally, at
product level, but not in the theory-building sense. Supporting this judgement, Wind
and Robertson (1983) stated that marketing discipline was dominated by marketing
management, which was fundamentally concerned with the design of marketing
programs and did not focus on the mission of a firm nor on how to gain competitive or
consumer advantage. They identified seven limitations within the marketing discipline
that should be addressed and corrected. These included a fixation with the brand as
the unit analysis, the interdisciplinary isolation of marketing, the failure to examine
synergy in the design of the marketing program, marketing's short run orientation, the
lack of rigorous competitive analysis, the lack of international orientation, and the lack
of an integrated strategic framework. Similar concern was also voiced by Day (1992),
who argued that marketers were too slow in addressing some of the important issues
of the past decade, and tended to stay too long with outmoded characterisation of
strategy processes and issues. However, environmental changes in the 80s influenced
the implementation of the corporate strategic planning concept. The resulting changes
could lead to the future business environment being characterised by an
unprecedented level of diversity, knowledge richness, and turbulence (Achrol, 1991).
The main cause of these environmental changes was the rapid proliferation of
technology. Manufacturing firms must modify their production systems and patterns
with new technology. In some cases, the new technology shortened the product’s
market life cycle. Technological development has moved the world toward a borderless
marketplace: in other words, it has created global competition. Finally, technology,
especially new information technology, has made consumers better informed, more
knowledgeable, and more sophisticated in their choice processes (Denison and
McDonald, 1995; Webster, 1997). The concept of strategic planning could not cope
with these environmental changes. The adoption of the concept has made firms
become large, bureaucratic and hierarchical organizations, and hence unable to
change quickly. To cope with environmental changes, many firms developed new

388 | P a g e
organizational forms. These new organizations emphasized partnership between firms,
multiple types of ownership and partnering within the organization, teamwork among
members of the organization, and often place less emphasis on formal contracting and
managerial reporting, evaluation, and control. In addition to providing flexibility and
acceleration in responding the environmental changes, the purpose of the new forms
of organization was to build long-term strategic alliances and customer relationships
(Webster, 1992). In addition, the discovery of the importance of product quality in
determining profitability was another factor that makes the concept of corporate
strategic planning become outmoded. It was found that the higher prices associated
with higher product quality did not deter market penetration. Thus quality had
positive effect on return on investment indirectly through its influence on market
share (Phillips, Chang, and Buzzel, 1983). This discovery did not only force re-
interpretation of PIMS data that emerged quality strategy model but also drove the
emergence total quality management (TQM) that was concepts and tools for getting all
employees focused on continues product or service improvement, in the eyes of
customers (Schonberger, 1992).

The radical changes in business environment and the discovery of the strategic role of
quality have led to a rediscovery of the marketing concept. As indicated by Webster
(1988), many firms such as General Electric, GTE, 3 M, Hewlett-Packard and Ford
have redesigned their marketing organizations. McKenna (1991) restated the
importance of customer satisfaction and the marketing concept. He claimed that
marketing was not a function, but a way of doing business. Meanwhile, Webster
(1992, 1997) suggested re-definition of marketing’s role in this new business
environment. He believes that marketing obviously operates on three distinct levels of
strategy: the corporate, the business or SBU and the functional or operating level. He
also identified three dimensions of marketing - marketing as culture, marketing as
strategy, and marketing as tactic. He further explained that though each marketing
dimension was found in at each level of strategy, the emphasis accorded the separate
dimensions of marketing varies with the level of strategy and the level within the
hierarchy of the organization.

Day (1997) viewed the future role of marketing from a different standpoint. He
contended that there could be changes in the role of marketing in the future.

389 | P a g e
Marketing would become a functional fiefdom if there was no significant change in
organizational structure and marketing was already a lead function. However,
marketing would become a subordinate function if the organization successfully
implemented a hybrid structure with a strong process orientation. In this situation,
marketing was likely to occupy a subordinate role in sale support activities or as a
participant in core process teams. However, if the organization successfully executed a
hybrid structure and maintains a strong marketing orientation, marketing would hold
a central guidance function.

Finally, in the current complex business environment, marketing people must be


critical and creative in implementing their roles. As emphasized by McKenna (1991)
the marketers must be the integrators, both internally, by synthesizing technological
capability with market needs and externally, by bringing the customer into the
company as a participant in the development and adaptation of goods and services. It
is a fundamental shift in the role and purpose of marketing: from manipulation of the
customer to genuine customer involvement; from telling and selling to communicating
and sharing knowledge; from last-in-function to corporate-credibility champion. This
points out that the marketers must advance their capability to promote their
important role. They must not only be able to maximize their customers’ satisfaction,
but also give a better return on investment for their companies. Evidence indicated
that when marketers increase their knowledge and skill related to the product, service
delivery, and financial accountability, marketing become a best function to manage
the relationship between the organization and its consumer (Moorman and Rust,
1999). The main barrier of the marketing concept implementation was not because of
non-acceptance of the concept by top management but rather because of poor image,
complacency, poor integration, and lack of a secure knowledge base of marketers
themselves (Denison and McDonald, 1995). However, customer orientation is too
important to be left only to the marketing people. Delivering superior value to
customers is the ultimate responsibility of every person in the organization. If this
does not happen, the value of the firm is diminished. Marketing specialists are needed
to keep the entire organization focused on the customer (Webster, 1997).

Perspectives on Marketing Strategy Research

390 | P a g e
As mentioned in the previous section, marketing scholars utilize the contingency
theory for theory building. Zeithaml et al. (1988) revealed the common use of the
theory in the contexts of marketing organizations, strategic marketing, and marketing
behavior. Formerly, scholars in strategic marketing used the theory to investigate
environmental influences on marketing strategy. However, considering the important
role of marketing strategy to achieve superior performance in the current turbulent
business environment, marketing strategy researchers need to expand their research
focus to encompass not only environmental impacts on the content of marketing
strategy but also to take account of the influence of environment on the process of
marketing strategy formulation and implementation. As with strategy research in
general, therefore, studies of marketing strategy, based on their focus, can be
classified into content and process studies. The followings section discusses them in
more detail.

Marketing Strategy Content Studies

Besides their contribution to the selection of appropriate product-market


combinations (Jemison, 1980), marketing researchers also contributed work on the
importance of product life cycle in strategy making. As indicated by Anderson and
Zeithaml (1984), strategy should fit with the product life cycle to gain better
performance. They found difference determinants of return on investment (ROI) in the
growth and maturity stages of product life cycle. While marketing variables determined
the ROI in the growth stage, industry variables influenced the ROI in the maturity
stage. Product competition and efficiency variables affected the ROI in both stages,
even though the effect was much higher in maturity than in the growth stage.

In addition, researchers also investigate environmental and organizational influences


on the selection of marketing strategy. Burke (1984) revealed that market
attractiveness and managers’ reward systems were positively associated with build
strategy, but entry barriers and synergy had a negative impact on it. This indicated
that managers might choose a build strategy when the market is profitable, has good
future prospects, and is easy to enter. This intention might be enhanced when the
reward system for the managers emphasized short run organizational performance.
Since implementing a build strategy might require many resources, it would lead to

391 | P a g e
low levels of organizational synergy. On the other hand, managers might select a hold
or pull back strategy when they perceived that the market was not buoyant.

Lusch and Laczniak (1989) found that increases in resource constraints amplified
competitive intensity. They also discovered that this increase in competitive intensity
made organizations emphasize non-price marketing strategy, though it did not lead to
a better performance. Lusch and Laczniak argued for an insignificant relationship
between marketing strategy and performance, because a more intensive competition
market forced most organizations to engage in intensive promotion and new product
development, which suppressed short term performance.

McDaniel and Kolari (1987) investigated the relationship between strategy types and
marketing strategy orientation. They found significant differences in marketing
orientation between Defenders and Prospectors, as well as between Defenders and
Analyzers. Prospectors and Analyzers employed more proactive marketing strategy
than Defenders. They engaged more activities in new product development, promotion,
and marketing research than Defenders. McKee, Varadarajan, and Pride (1989) found
similar results. They revealed significant differences in marketing tactics among
Reactors, Defenders, Analyzers and Prospectors. Organizations with more adaptive
strategy types might focus more on marketing efforts. Prospectors, for example, tended
to use more scanning and product development efforts than the other strategy types.
However, there were no differences between Defenders and Analyzers. Significant
differences only existed between Defenders and Prospectors (Rajaratman and Chonko,
1995). Defenders tended to organize their marketing department on more functional
structure than Prospectors.

Rajaratman and Chonko (1995) also indicated centralization of power seems to exist in
Reactors, but not in the others. Prospectors tended to develop a more specialized
organization structure and seek greater market penetration and product development.
They expended greater effort in marketing than Defenders, Analyzers and Reactors. In
relation to marketing effort, Defenders spent more significantly than Reactors. Finally,
they revealed that Reactors had lower performance, either in term of earning/sales
growth rate or return on sales/ investment than the other three types of business
strategy. There were no differences in performance among these three.

392 | P a g e
Furthermore, marketing researchers also investigated the importance of the role of
marketing strategy in exporting organizations. Madsen (1989), for instance, discovered
that export marketing strategy was the key success factor of export performance in
term of export sales, growth, and profits, while market and organizational factors only
influenced export sales. Madsen also indicated the existence of inter-correlation and
association among variables, especially in regard to the characteristics of firms and
their markets. This meant that even though those variables did not have any
significant relationship with export performance, they still had an indirect influence on
it. Strengthening Madsen’s findings, Cavusgil and Zou’s (1994) revealed that product
adaptation, support to foreign distributors/subsidiaries, international competence and
commitment enhanced the performance of export ventures. They also discovered that
the internal and external organizational environments influenced export marketing
strategy. The most critical determinant of strategy was the technological orientation of
an industry. It significantly determined all variables of the marketing strategy. Export
market competitiveness also affected the strategy variables, except for price
competitiveness.

This indicated that managers did not use price as a weapon in coping with market
pressure. In addition, organizational international competence and experience with the
product determined product and promotion adaptation. Management commitment to
the venture affected only the support given to distributors and subsidiaries. Finally,
whereas product uniqueness determined product and promotion adaptations, the only
effect of cultural specificity of product was on product adaptation.

To succeed in entering global markets, entrepreneurship became a key orientation,


especially for small and medium organizations (Knight, 2000). Such organizations
made some preparations before entering the international market. They performed
such activities as international market research, commitment of appropriate
resources, and the adaptation of product and marketing dimensions. In addition,
organizations employing a marketing leadership strategy tended to respond to
globalization by being sensitive to its imperatives, modifying marketing and other
strategies as needed.

393 | P a g e
In conclusion, to gain superior performance organizations should fit their marketing
strategy to the current market environment. Centralization of power impedes the
ability of organizations to be more adaptive and leads to lower performance (Ozsomer
and Prussia, 1999; Rajaratman and Chonko, 1995). To be sensitive to environmental
changes, organizations should employ a decentralised marketing structure. This
enables marketing managers to easily adapt to environmental changes, and to achieve
strategy – environment fit. Table 2.3 below presents empirical evidence on marketing
strategy content research.

Table 2. 3: Summary of Selected Empirical Studies on Marketing Strategy Content

Authors Subject/Resea Variables Analytical Key Findings


rch studied Tools
Method

Anderson 1,234 Product life ANOVA Many strategy


& industrial cycle (growth, and variables significantly
Zeithaml manufacturing maturity, & Multiple correlate to superior
(1984) companies in decline), Regression performance.
PIMS data base strategic Analysis However there is
variables significant difference
(industry, in the determinants
product of ROI in the growth
competition, and maturity stages.
R&D, product / C.f. significant
investment, influence of
efficiency, marketing variables
vertical and industry
integration, and variables in growth
marketing and maturity stages
variables), respectively. Product
performance competition and
(ROI efficiency variables
& relative significantly affect
market share) the ROI in both
stages but the effect
is much higher in the
maturity than in the
growth stage.
Therefore, the
business strategy
should fit with the
stage of the product
life cycle to attain

394 | P a g e
superior
performance.

Burke 86 marketing Environment MANOVA, Environment and


(1984) managers. variables ANCOVA, organizational factors
Data were (market Multiple influence managers
collected attractiveness, Discrimina in selecting their
through relative nt Analysis marketing strategy.
questionnaires competitive Market attractiveness
strength, level of and reward for short
uncertainty, run performance are
exit, and entry positively associated
arriers) with build strategy,
organizational but entry barriers
variables and synergy have
(reward system, negative impact on it.
and synergy), This indicates
marketing managers may
strategy choose build strategy
(build, when market is
hold, and pull profitable, has good
back) future prospects, and
is easy to enter. This
intention may be
enhanced when the
reward system for the
managers
emphasizes short run
organizational
performance.
Because build
strategy may require
many resources, it
can lead to a low level
of organizational
synergy. Managers
may select hold or
pull back strategy
when market is not
buoyant

McDaniel 279 marketing Marketing Cluster, Significant


& Kolari managers of US environment, Multiple differences in
(1987) banks. Mailed strategy Discrimina marketing orientation
questionnaires types t Analyses, between Defenders
were used to (Prospector, MANOVA, and Prospectors, as
collect data. Analyzer, and ANOVA, well as between
Defender), and and Defenders and
marketing Duncan’s Analyzers.
395 | P a g e
strategy Multiple Prospectors and
Range Analyzers employ
Test. more proactive
marketing strategy
than Defenders. They
engage in more
activities in new
product development,
promotion, and
marketing research
than Defenders.
Conclusion:
Defenders lack
marketing orientation
and rely more on
their traditional
products.

Madsen 134 managers Market ANOVA Export marketing


(1989) of Denmark and and strategy is the key
industrial organizational Multiple success factor of
companies. characteristics, Regression export performance
Mailed Export Analysis in term of export
questionnaires marketing sales, growth, and
were used to strategy, and profits. The other two
collect data export independent
performance variables
(firm/market) only
influence export
sales. Results also
indicate that inter-
correlation and
association exist
among variables,
especially regarding
firm and market
characteristics. This
means that even
though those
variables do not have
any significant
relationship with
export performance,
they still have
indirect influence on
it. Interaction
analysis indicated
this relationship.
Of the overall
396 | P a g e
variables, product
strength is the most
important
explanatory variable
followed by planning
and control intensity,
export experience,
and export market
attractiveness.

McKee, 333 managers Market ANOVA Significant


Varadaraj of US Bank environment / and Non- differences in
an, and companies. volatility, Parametric marketing tactics
Pride Data were organization Test of among Reactors,
(1989) collected strategy Correlation Defenders, Analyzers,
through mailed type Analysis and Prospectors.
questionnaires (reactor, Organizations with
Defender, more adaptive
Analyzer, and strategy types may
Prospector), put more effort into
marketing marketing
tactics, and Prospectors, for
performance. example, tend to use
more scanning and
product development
efforts than the other
strategy types.
Market volatility has
significant direct
relation-ship to
marketing tactics. It
is not only
significantly related
to organization
performance but also
moderates strategy
types - performance
relationships.
However only in
mildly positive
volatile markets is
the role of market
volatility significant
as moderating
variable: it is not
significant in negative
and highly positive
volatile markets.

397 | P a g e
Lusch & 103 executives Resource Structural Association found
Laczniak of 500 fortune constraints, Equation between resource
(1989) companies. structural Modeling constraints and
Data were fluctuations, (LISREL) structural
collected by competitive fluctuations.
mailed intensity, However, an increase
questionnaires. marketing in resource
strategies constraints amplifies
(product, competitive intensity,
promotion, & while an increase in
distribution), structural
performance. fluctuations does not.
Increase in
competitive intensity
makes the
organizations
emphasize non-price
marketing strategy,
though it does not
lead to a better
performance.
Insignificant
relationship between
marketing strategy
and performance is
probably because a
more intensive
competition market
forces most
organizations to
engage intensive
promotion and new
product development,
suppressing short
term performance.

Cavusgil 202 marketing Industry Explorator Product


and Zou managers of US characteristics, y, adaptation,
(1994) exporting market Confirmato support to foreign
ompanies. characteristics, ry Factor, distributors/subsidia
Data were organizational and ry, international
collected characteristics, Path competence,
through in product Analyses commitment to
depth characteristics, export venture
interviews export enhance the
marketing performance of
strategy, and export ventures.
export Other export
performance marketing strategies
398 | P a g e
such as price
competitiveness and
promotion adaptation
influence the
performance
insignificantly.
Internal and external
organizational
environments
influence export
marketing variables.
The most critical
determinant of
strategy is
technological
orientation of
industry, which
significantly
determines all
variables of the
marketing strategy.
Export market
competitiveness also
affects the strategy
variables, except
price
competitiveness.
This indicated that
managers do not use
price as a weapon in
coping with the
market pressure. In
addition,
organizational
international
competence and
experience with the
product determine
product and
promotion
adaptation.
Management
commitment to the
venture affects only
support given to
distributors/subsidia
ries. Finally whereas
product
uniqueness
399 | P a g e
determines product
and promotion
adaptations, cultural
specificity of
product only
affects product
adaptation
significantly.

Rajaratna 410 marketing Organization ANOVA No difference between


m& managers of structure, and Defenders and
Chonko service marketing MANOVA Analyzers in
(1995) organizations. organization, organizing the
Data were growth strategy, marketing
collected strategic department.
through mailed orientation, and Significant
questionnaires performance differences only
exist between
Defenders and
prospectors:
Defenders tend
to organize
their
marketing
departments on a
more functional
structure than do
Prospectors. No
difference among the
four business
strategy types in
regard to
formalization and
market development.
Centralization of
power seems only to
exist in Reactors.
Prospectors tend to
carry out more
specialized
organization
structure, market
penetration and
product development,
and to expend greater
marketing effort than
Defenders, Analyzers,
and Reactors. In
relation to marketing
400 | P a g e
effort, Defenders
spend more
significantly than
Reactors.
Finally, the study
reveals that Reactors
have lower
performance, either
in term of
earning/sales growth
rate or return on
sales/ investment
than the other three
types, amongst whom
no differences can be
detected.

zsomer 45 executives of Target market Structural The contingency


& Prussia MNCs in similarities, Equation model of target
(1999) Turkey. standardized Modeling market similarities
Data were marketing (TMS), standardized
collected strategy, marketing strategy
through marketing (SMS),
interview in structure marketing
1988 and 1994. (the degree of structure (CS),
centralization of and performance
decision- relationship is
making), and significant for both
performance. periods of time (1988
and 1994). TMS
influences SMS
positively. A similar
affect exists between
SMS and CS.
However, CS has a
negative effect on
performance.
Marketing structure
is the dominant
factor in determining
organizational
performance. The
negative influence of
structure on the
performance
indicates the need of
decentralization of
marketing decision to
gain better
401 | P a g e
performance. The
decentralized
structure enables
managers to adapt
the marketing
strategy to local
environment, and to
implement the
strategy flexibly.
Marketing strategy
affects performance
indirectly through the
structure.
Contemporaneous
effect analysis
reveals that
marketing strategy is
consistently
responsive to the
current market
environment, since
the SMS 1994 is only
influenced by TMS
1994, not by TMS
1988 and SMS 1988.
The researchers
conclude that to gain
superior
performance MNCs
should align their
marketing strategy to
the current local
market environment
and decentralize
marketing decision-
making to their
subsidiary managers
to facilitate
adaptation to
environmental
changes.

Knight 216 CEOs of Globalization, Factor, Entrepreneurial


(2000) small and entrepreneurial Correlation orientation correlates
medium orientation, , and to all of the
enterprises marketing Multiple marketing
(SMEs). strategy Regression strategies:
Data were (product Analyses. product

402 | P a g e
collected specialization, specialization,
through mailed marketing, and marketing, and
questionnaires. quality quality leadership,
leadership), especially for SMEs
technology working in high
acquisition, globalization
globalization environment. In
response, addition, new
internationalizat technological
ion preparation, acquisition associates
and with these strategies,
performance while globalization
response
only correlates to
marketing leadership
strategy.
Finally globalization
response and
internationalization
preparation influence
SMEs’ performance.
There is no
relationship between
new technological
acquisition and
performance.
Conclusion:
entrepreneurship
becomes a key
orientation among
SMEs working in a
global environment.
To achieve better
performance, they
prepare for entry to
the international
market by performing
such activities as
international market
research, committing
appropriate resources
and the adaptation of
product and
marketing
dimensions. In
addition, SMEs
employing a
marketing leadership

403 | P a g e
strategy tend to
respond to
globalization by being
sensitive to its
imperatives and
modifying marketing
and other strategies
as needed.

Marketing Strategy Process Studies

As is the case with strategy process research in general, studies on the marketing
strategy process focused mainly on the formulation and implementation stages. The
studies investigate the influence of environmental, organizational and decision-
making processes on organizational performance. Phillips, Davies, and Mountinho
(2001), for example, indicated that organizations employing strategic marketing
planning with product orientation attained more effective and efficient performance
levels. This strategic planning was characterised by manager participation, planning
thoroughness, formalization and sophistication, with an emphasis on new product
development. Stratis and Powers (2001) revealed similar findings, and discovered that
strategic marketing planning determined financial performance, but planning modes
and environmental scanning individually did not influence performance. All of the
strategic marketing processes in combination affected financial performance
significantly. However, under conditions of strategic uncertainty, only strategic
marketing planning and environmental scanning had any effect on the performance.

Formalization of marketing planning improved the credibility and utilization of a


marketing plan (John and Martin, 1984). John and Martin suggested that
formalization indicated organizational commitment to certain activities. The
formalization of strategic marketing planning also indicated involvement of top
management in the planning process, and a cooperative organizational climate (Chae
and Hill, 1997). It might generate both competitive and organizational benefits. Chae
and Hill revealed that planning formality improved the effectiveness of new product
launches, cost reduction efforts, and helped organizations to enhance product quality
and market share performance. The formalization of planning efforts might also create
organizational benefits, as it enhanced understanding of priorities as well as
managerial motivation to attain better overall coordination, implementation, and
404 | P a g e
control of an organization’s activities. In addition, planning formalization might reduce
interdepartmental conflict (Morgan and Piercy 1998). It facilitated cross- functional
integration and consensus commitment (Menon, Bharadwaj, Adidam, and Edison
1999), generating better coordination of decisions throughout the organization.
Claycomb, Germain, and Droge (2000) also found positive correlation between
planning formalization and marketing specialization, indicating organizations
employing strategic marketing formalization had more specialists who direct their
efforts to a narrow set of marketing activities. Finally, in combination with
comprehensive and formal market planning positively affected the performance of
organizations operating in both stable and unstable environments (Lysonski and
Pecotich 1992). The formalized comprehensive marketing planning was likely to result
in superior performance because it usually anticipated the unexpected and lays
contingency plans accordingly.

In contrast to formalization, centralization of strategic marketing planning impeded


the credibility and utilization of the marketing plan (John and Martin 1984). In some
cases, centralization might improve resource commitment and marketing assets and
capabilities (Menon et al., 1999; Vorhies, 1999). However, it tended to generate
interdepartmental conflicts. The conflicts might not only reduce the quality of
marketing strategy formulation and implementation (Menon, Bharadwaj, and Howell,
1996), but also lowered financial and market performance (Morgan and Piercy, 1998).
Interdepartmental conflicts might also arise from failure to provide the kinds of
supports needed by other departments, unclear goals, objectives, and functional
responsibilities. Organizations might vary in their approaches to conflict resolution. As
mentioned earlier, organizations might use planning formalization to eliminate
conflict. They might employ such mechanisms as avoidance, conciliation, participatory
processes and hierarchy (Ruekert and Walker, 1987). Other approaches could include
mechanisms such as multifunctional training, cross-functional teamwork, a variety of
compensation strategies, formalization, social orientation, and spatial proximity (Maltz
and Kohli, 2000). Besides interdepartmental conflicts, behavioral problems might arise
during the planning process. Piercy and Morgan (1990) identified the existence of four
different behavioral problems that can occur. These included planning recalcitrance,
fear of uncertainty, political interest in planning, and planning avoidance. Most of
these problems correlated significantly with the perceived organizational context,
405 | P a g e
especially customer philosophy and strategic orientation. This indicated that higher
behavioral planning problems emerged when organizations were perceived to have
paid little attention to customer needs and to the requirements of different market
segments, and had been ineffective in developing and implementing acceptable
marketing strategies. The customer philosophy and strategic orientation related
significantly and positively to the credibility and utilization of a marketing plan. In the
second study, Piercy and Morgan (1994) discovered that only planning recalcitrance
influenced the credibility of marketing plan negatively. The others affected the
credibility only indirectly, through their connection with planning recalcitrance. On
the other hand, they found that thoroughness in planning and the incorporation of
appropriate plan components improved a marketing plan’s credibility. In turn,
sophisticated analytical techniques and market analyses enhanced planning
thoroughness and the plan’s components.

Moreover, the support of top management might improve the effectiveness of the
marketing planning process (Greenley and Bayus, 1994). Top management’s actions
and attitudes in coping with environmental turbulence influenced the quality of
marketing interactions. Senior managers with high tolerance of ambiguity, internal
locus of control and greater risk-taking propensity tended to adapt their marketing
strategy to environmental changes (Pitt and Kannemeyer, 2000). They might possess
good leadership qualities, needed for the effectiveness of any planning process,
especially in a turbulent environment. As indicated by Morgan and Piercy (1998),
senior management leadership had a positive effect on connectedness and
communication frequency, and affects the level of conflict negatively. They presumed
that the management’s leadership determined the organization’s performance by
facilitating employee empowerment and improving interdepartmental coordination and
cooperation.

Marketing managers had an important role to play in successful marketing strategy


implementation. Noble and Mokwa (1999) discovered that the role performance of
marketing managers determined the success of marketing strategy implementation.
Such performance was influenced by managers’ commitment, not only to marketing
strategy, but also to their career roles. However, there was no relationship between
organizational commitment and performance. The degree of role autonomy and

406 | P a g e
involvement did not affect the commitment of managers. When marketing managers
perceived their roles to be significant to the success of marketing strategy
implementation, their commitment would increase. They would commit to a strategy
when they perceived that it fit with the broader strategic direction of the organization.
This commitment would increase when managers perceive the strategy to have
potentially significant consequences for their organizations, and they received cross-
functional support. However senior management’s support and the scope of the
marketing strategy did not determine the managers’ commitment to it.

In conclusion, as with studies of the strategy process in general, most studies of


marketing strategy process, except that of Noble and Mokwa (1999), concentrated on
strategy formulation processes. They paid little attention to marketing strategy
implementation, although Sashittal and Tankersley (1997) revealed that the
formulation-implementation interface was highly responsive to market changes and to
changes in each other. Marketing managers improvised their market strategies and
implementation to fit day-to-day market changes and to achieve their marketing
objectives. However, to perform responsively in the formulation-implementation
interface, authority delegation or managers’ autonomy was a prerequisite. Table 2.4
below consolidates the findings of empirical studies of marketing strategy content
research.

Table 2. 4: Summary of Selected Empirical Studies on Marketing Strategy Process

Authors Subject / Variables studied Analytical Key Findings


Research Tools
Method

John & 292 Formalization, Correlation, Results show


Martin marketi centralization, Multiple that formalization
(1984) ng structural Regression of marketing
personnel. differences (job Analyses, planning improves
Mailed diversities, and the credibility and
questionnaires specialization, and Structural utilization of the
were used spatial dispersion), Equation marketing plan.

407 | P a g e
to credibility, and Modeling The higher the
collect data utilization of rules and
marketing plan procedures for
performing the
planning the
higher the
credibility and the
usage rate of the
plan. The
researchers
suggest the
formalization
indicates a
organization
commitment to
certain activities.
Centralization, on
the other hand,
impedes the
credibility and
utilization of the
marketing plan.
Finally, in
bivariate analysis,
higher marketing
job specialization
increases
credibility and
utilization rates,
but these
relationships turn
out to be

408 | P a g e
insignificant when
formalization is
included in
multivariate
analysis. While
spatial dispersion
only influences the
credibility
negatively, job
diversity has no
significant effect
on either
credibility or
utilization.

Ruekert & 95 marketing Business strategy Two ways Findings indicate


personnel and ANOVA, that failure to
Walker (Prospector,
21 R&D Chi- square, provide the kinds
(1987) Analyzer, &
personnel. and of supports needed
Defender),
Data were Correlation by departments,
structure
collected Analyses the degree of
(formalization),
through market response,
conflict resolution
mailed and unclear goals,
(avoidance,
questionnaire objectives, and
onciliation,
s functional
participatory,
responsibilities are
hierarchical),
the sources of the
outcome (level of
conflict. Marketing
conflict, &
personnel in
perceived
Prospector
effectiveness of
organizations
interdepartmental
experience more
relationship)

409 | P a g e
conflict with R&D
personnel than
those in Analyzer
and Defender
ones. All of the
organizations
(Prospector,
Analyzer, and
Defender) employ
similar levels of
formalization in
managing
departmental
relationships. In
addition,
organizations vary
in their ways of
dealing with
conflict resolution.
Prospectors tend
to use hierarchical
and avoidance
mechanisms,
while analyzers
are inclined to
utilize
participatory
methods to resolve
conflicts. Like
Prospectors,
Defenders are apt
to employ

410 | P a g e
hierarchical
mechanism for
resolving conflicts.
Finally, the study
discovers that
personnel in
Prospector
organizations have
less favorable
attitudes to
conflict resolution,
favoring a greater
reliance on non-
hierarchical
mechanisms.

They also perceive


the relationship
between marketing
and R&D to be
less effective than
personnel in
Analyzers and
Defenders. Further
analysis reveals
that the level of
formalization is
significantly and
positively related
to the perceived
effectiveness of
terdepartmental
relationships, but

411 | P a g e
significantly and
negatively to the
level of conflict
between the two
departments. In
addition the use of
participatory
mechanisms,
especially in
Prospectors and
Analyzers, is
negatively
associated with
the level of conflict
and positively
correlated with the
perceived
effectiveness of
relationships.
Meanwhile, the
use of hierarchical
mechanisms
relates positively
to levels of conflict
but negatively to
perceived
effectiveness.

Piercy & 144 marketing Perceived Correlation The study


Morgan managers of organizational Analysis identifies the
(1990) UK ompanies. context (customer existence of four
philosophy, different
Data were

412 | P a g e
collected marketing behavioral
through organization and problems in the
mailed information marketing
questionnaires effectiveness, planning process.
strategic These include
planning
orientation),
recalcitrance, fear
behavioral
of uncertainty,
planning
political interest in
problems, and
planning, and
marketing plan
planning
credibility and
avoidance. Most of
utilization
these problems
correlate
significantly with
the perceived
organizational
context, especially
customer
philosophy and
strategic
orientation. This
indicates that
higher behavioral
planning problems
emerge when
organizations are
perceived to give
little attention to
customer needs
and different
market segment

413 | P a g e
equirements, and
ineffective in
developing and
implementing
acceptable
marketing
strategies.
Customer
philosophy and
strategic
orientation relate
significantly and
positively to the
marketing plan’s
credibility and
utilization. Finally,
results reveal that
behavioral
problems are
associated
negatively with the
plan’s credibility
and utilization,
but their
relationships are
not statistically
significant.

Lysonski & 522 CEOs of Environmental Multiple Findings reveal


New Zealand stability, Regression formalization and
Pecotich
companies. formalization, Analyses comprehensivenes
(1992)
comprehensivenes s of marketing
Data were

414 | P a g e
collected s, and planning positively
through performance affect the
mailed performance of
questionnaires organizations
operating in both
stable and
unstable
environments. Of
the resulting
discrepancies with
the previous
study, the
researchers argue
that apart from
any environmental
conditions,
formalized and
comprehensive
marketing
planning is likely
to result in
superior
performance
because it may
provide
contingency plans
and anticipate the
unexpected.

Greenley & 175 marketing Analytical Cluster and The study


Bayus managers of techniques, Discriminan categorises four
(1994) US companies analytical systems, t Analyses different

415 | P a g e
and 106 of UK organization marketing
companies. information planning
inputs, senior processes among
Data were
management US and UK
collected
participation, and organizations.
through
the effectiveness of
mailed Most of the
marketing
questionnaires organizations
planning decision-
could be classified
making
as unsophisticated
marketing
planning decision
makers that tend
to ignore
planning
techniques and
organization
information
inputs, and use
standard
computer software
in the planning
process.

Organizations
within the second-
biggest group may
be seen as
information
seekers, likely to
use many
information inputs
in developing
planning but only

416 | P a g e
use judgment, PLC
analysis, and
standard
computer software
in planning
analysis. The third
group consist of
“gut feeler”
organizations that
use limited
information inputs
and make wide
use of judgment in
the planning
process. Only a
small number of
organizations can
be described as
sophisticated
decision makers in
their strategic
marketing
planning
processes. These
organizations tend
to use most
analytical
techniques and
several
information
inputs, but they
consider computer

417 | P a g e
software not to be
important.
However, apart
from their
differences in
marketing
planning process,
all organizations
consider senior
management
participation to be
important for the
effectiveness of the
planning process.
Likewise, there are
no differences in
the perceived level
of effectiveness of
marketing
planning
processes among
the groups. The
researchers argue
that managers,
especially the
unsophisticated
decision makers
and “gut feelers”,
may not realise
the use of a
formalized
planning process

418 | P a g e
could improve
their effectiveness.

Piercy & 220 managers Formalization and Factor and The study
Morgan of medium sophistication of Path identifies the
(1994) and large marketing Analyses existence of five
companies in planning different
UK. Mailed (analytical behavioral
questionnaires techniques, problems in the
were employed market analysis, marketing
to collect data and plan planning process:
components), these are planning
planning recalcitrance,
thoroughness, politics and
behavioral myopia, alienation
planning and uncertainty,
problems, and planning
marketing plan avoidance, and
credibility squirm factors.
From the five
behavioral
planning
problems, only
planning
recalcitrance
negatively
influences the
credibility of a
marketing plan.
The others affect
credibility
indirectly, through

419 | P a g e
planning
recalcitrance.
Likewise, findings
reveal planning
thoroughness and
plan components
improve the
marketing plan’s
credibility, while
more sophisticated
analytical
techniques and
market analyses
carried out in
marketing
planning enhance
the plan
components and
planning
thoroughness.

Menon, 236 marketing Centralization, Structural Consistent with


Bharadwaj, managers. formalization, Equation the previous
& Howell team spirit, Modeling study, findings
Data were
(1996) interdepartmental here
collected
connectedness, reveal
through
communication dysfunctional
mailed
barriers, conflict (unhealthy
questionnaires
functional conflict, behavior
dysfunctional within
conflict, the organizations)
quality of negatively

420 | P a g e
marketing influences the
strategy, and quality of
market marketing
performance strategy
formulation and
implementation,
while functional
(constructive)
conflict enhances
the quality of the
strategy. High
quality strategy
formulation and
implementation
leads to superior
market
performance. The
study also
uncovers some
causes of the
conflicts. Power
centralization and
the existence of
communication
barriers may
enhance
dysfunctional
conflict, but it may
be eliminated by
formalized
structure.
Functional

421 | P a g e
conflict, on the
other hand, may
be increased by
team spirit and
departmental
interconnectednes
s.

Sashittal & 40 marketing The processes of Qualitative Findings


Tankersley managers of market planning Analysis reveal that
small and and market planning
(1997)
midsized implementation and
industrial implementation
firms. are interrelated.
The
Data were
planning-
collected
implementation
through in
interface is highly
depth
responsive to
interviews.
market changes
and to changes in
each other.
Marketing
managers
improvise their
market plans and
implementation to
fit day-to-day
market changes
and to achieve
their marketing
objectives. The

422 | P a g e
study identifies
three major tactics
used to manage
the planning-
implementation
interface,
depending on the
extent of
environmental
changes.
Organizations
working with low-
level market
changes employ
communication
between planners
and implementers
to ensure that
their market plans
are based on the
latest market
information.
However when
environmental
changes are high,
organizations tend
to upgrade the
involvement of
planners-
implementers or to
fuse their
functions. The first

423 | P a g e
tactic promotes (a)
achievable
marketing plans,
given time and
resource
constraints faced
by the
implementers, (b)
flexibility and
realism in the
planning-
implementation
processes, and (c)
commitment
toward achieving
marketing
objectives.
Meanwhile,
organizations
employing the
fusion-of-function
tactic will, in
addition to the
three benefits
above, gain a high
degree of
sensitivity in the
planning-
implementation
function by
speeding the
response to

424 | P a g e
changes in
customer needs.
The researchers
conclude that
authority
delegation or
managers’
autonomy is a
prerequisite to the
attainment of a
highly responsive
planning-
implementation
interface.

Chae & Hill 90 CEOs of Planning formality, One Way Findings indicate
(1997) industrial and CEO involvement, ANOVA that top
consumer organizational management
companies. climate, involvement in the
environmental planning process
Mailed
complexity & and a cooperative
questionnaires
uncertainty, organizational
were used to
perceived climate increase
collect data
organizational and the formality of
competitive benefit strategic
marketing
planning. There is
no significant
influence from
organizational
structure,
environmental

425 | P a g e
complexity and
uncertainty on
planning formality.
Likewise there is
no difference in
planning formality
between industrial
and consumer
organizations.
However, formality
may generate both
competitive and
organizational
benefits. Results
reveal that
planning formality
can improve the
effectiveness of
new product
launches and cost
reduction efforts,
whilst facilitating
improved product
quality and market
share
performance. In
addition, formality
of planning efforts
may create
organizational
benefits as it
enhances

426 | P a g e
understanding of
priorities,
managerial
motivation to
attain better
overall
coordination,
implementation,
and control of the
organization’s
activities.

Vorhies 85 marketing Environmental Structural Findings indicate


(1998) managers of turbulence, equation that only
good and business strategy, modeling environmental
service organizational turbulence and
companies in structure, task task routinization
US. Data were routinization, do not significantly
collected market influence
through information marketing
mailed processing capabilities.
questionnaires capability, Business strategy
marketing has the strongest
capabilities, and effect on the
organizational marketing
effectiveness capabilities,
indicating
organizations with
a higher level of
strategy
development have
better developed

427 | P a g e
capabilities. In
addition,
marketing
information
processing
capability
enhances
marketing
capabilities. As
propermarketing
actions need
accurate
information, it
could be predicted
that organizations
with high
information
processing
capability will have
the best marketing
capabilities.
Organizational
structure, as well
as business
strategy, increases
marketing
capabilities. This
indicates more
centralized
decision-making
and more formal
rules and

428 | P a g e
procedures
encourage the
development of
marketing
capabilities.
Finally,
organizations with
high marketing
capabilities may
achieve superior
performance.

Morgan & 298 general Senior Standardize The study


Piercy managers, management d and discovers that
(1998) 351 marketing leadership, moderated senior
managers, strategy process Multiple managemen
and 398 (formalization, Regression t leadership has a
quality thoroughness, Analyses positive effect on
managers alignment, connectedness
participation), and
control system
communication
congruence,
frequency, but
environment
affects the level of
(market and
conflict negatively.
technological
The researchers
turbulence),
presume that the
interdepartmental
management
interaction
leadership
(connectedness,
determines the
communication
organization
frequency, conflict)
performance by
and performance
facilitating

429 | P a g e
employee
empowerment,
and improving
interdepartmental
coordination and
cooperation. The
influence of
strategy process
on
interdepartmental
dynamics varies.
Planning
formalization may
reduce
interdepartmental
conflict.

While the
planning
alignment and
functional
participation
improve
connectedness
between marketing
and quality
departments,
planning
thoroughness
enhances
communication
frequency between
departments.

430 | P a g e
Congruence
between the
quality control
system and quality
strategy may also
increase
communication
frequency. In
addition, findings
also indicate
various effects of
interdepartmental
dynamics on
performance. This
connectedness
may raise the
perceived quality,
whereas the
communication
frequency could
improve market
performance. On
the other hand,
lower conflict
between marketing
and quality
departments may
lead to better
financial and
market
performance.

There is no

431 | P a g e
moderating effect
of market and
technological
turbulences on the
relationship
between
interdepartmental
dynamics and
performance.

Menon, 212 managers Antecedents Correlation, Findings indicate


Bharadwaj, of Fortune (centralization, Regression, that innovative
Adidam, & 1000 formalization, & Exploratory, culture influences
Edison companies. innovative and all marketing
(1999) culture), Confirmator strategy
Data were
marketing strategy y Factor components,
collected
making (situation analyses whereas
through
analysis, centralization and
mailed
comprehensivenes formalization only
questionnaires
s, emphasis on affect some of
.
marketing assets them.
and capabilities, Centralization
cross-functional enhances the
integration, emphasis on
communication marketing assets
quality, strategy and capabilities
consensus and
commitment, resource
strategy resource commitment.
commitment),
Formalization, on
control
the other hand,
(environmental
facilitates cross-

432 | P a g e
turbulence), and functional
outcomes integration and
(creativity of consensus
strategy, commitment. In
organizational addition, there are
learning, and some variations in
market the effects of
performance) marketing
strategy-making
components on
outcomes. While
comprehensivenes
s, cross-
functional
integration, and
communication
quality increase
the creativity of
strategy, emphasis
on marketing
assets and
capabilities
impedes it. The
creativity of
strategy, in
combination with
situational
analysis,
consensus
commitment and
resource
commitment

433 | P a g e
improve
organizational
learning.

Creativity also has


a mediating role in
the relationship
between some
marketing strategy
components and
organizational
learning. It
mediates the effect
of cross-functional
integration and
communication
quality and
comprehensivenes
s and emphasis on
marketing assets
and capabilities on
organizational
learning. Finally,
comprehensivenes
s, emphasis on
marketing assets
and capabilities,
and resource
commitment may
improve market
performance, but
situational
analysis hinders

434 | P a g e
it. Further
analysis indicates
the negative
influence of
situational
analysis on market
performance only
exists in
conditions of low
environmental
turbulence.

Noble & 486 marketing Strategy factors (fit Structural The study
managers in with vision, Equation discovers that the
Mokwa
Finance and importance, scope, Modeling role performance
(1999)
goods championing, of marketing
corporation. senior managers
management determines the
Mailed
support, and buy- success of
questionnaires
in), role factors marketing
were used
(involvement, strategy
to
autonomy, and implementation.
collect data.
significance),
Role performance
dimensions of
itself is influenced
commitment
by the managers’
(organizational,
commitment to the
strategy, and role
marketing
commitments),
strategy, and to
role performance,
their career roles.
and
However,
implementation
there is no
success
relationship

435 | P a g e
between
organizational
commitment and
role performance.
When marketing
managers perceive
their roles to be
significant to the
success of
marketing strategy
implementation,
their role
commitment will
increase. The
degree of role
autonomy and
involvement do not
affect the role
commitment of the
managers. In
addition, the
managers will
commit to the
strategy when they
perceive the
strategy fits with
the broader
strategic direction
of the
organization. This
commitment will
also increase when

436 | P a g e
the managers
perceive the
strategy to have
potentially
significant
consequences for
their organizations
and have cross-
functional
support.

Finally, the
support of senior
management and
the scope of
marketing strategy
do not determine a
manager’s
commitment to a
strategy.

Claycomb, 200 managers Strategic Correlation Findings indicate


Germain, & of industrial marketing Analysis strategic
Droge companies. formalization, marketing
organizational formalization leads
(2000) Mailed
configuration, to better
questionnaires
organizational performance. This
were used
structure, formalization does
to
customer- driven not relate to the
collect data.
exchange, and configuration of a
context, and marketing
performance organization:
(market & however, it

437 | P a g e
provides for better
financial)
co-
ordination of
decisions
throughout the
organization. It
also correlates
positively to
marketing
specialization,
indicating that
organizations
employing
strategic
marketing
formalization have
more specialists,
who direct their
efforts towards a
narrow set of
marketing
activities. These
organizations
utilise internal
measurements
and competitive
benchmarking as
means of formal
performance
measurement and
evaluation.
Finally, the

438 | P a g e
formalization of
strategic
marketing
planning does not
determine the
relationship
between the
organizations and
their customers.

Pitt & 70 managers Personality traits Descriptive A significant


Kannemeye of black- (tolerance of and relationship was
r owned ambiguity, locus of correlation found between
companies control, and risk Analyses personality traits
(2000)
in taking propensity), of managers and
South Africa. and the degree of adaptation of
marketing strategy marketing
Data were
adaptation. strategy. Higher
collected
intolerance of
through
ambiguity
interview and
correlates
self-
negatively to
administered
marketing strategy
questionnaires
adaptation. In
other words,
managers who
tolerate ambiguity
are more flexible
in implementing
their marketing
strategy than
intolerant ones. In

439 | P a g e
addition,
managers with an
internal locus of
control and greater
risk-taking
propensity tend to
adapt their
marketing strategy
to environmental
changes.

Maltz & 774 of Integrating Ordinary The study finds


manufacturin mechanisms Least that only cross-
Kohli
g, R&D, (multifunctional Squares functional
(2000)
and training, cross- Regression teamwork is likely
finance functional team, Analysis to reduce manifest
managers. compensation inter-functional
variety, conflict effectively.
Data were
formalization, Inter-functional
collected
social orientation, conflict may result
through
& spatial from high internal
mailed
proximity), volatility. This
questionnaires
internal volatility, volatility
functional moderates the
interface, and relationship
manifest inter- between cross-
functional conflict functional teams.
Compensation
variety, and inter-
functional conflict.
In other words,
when volatility is

440 | P a g e
high, manifest
inter- functional
conflict may be
reduced by using
cross- functional
teams or by
lowering
compensation
variety. Further
analysis reveals
that the
integrating
mechanisms are
differentially
effective in
eliminating
marketing’s
conflict with
manufacturing,
R&D, finance
departments. In
general cross-
functional teams
seem to be
effective in
reducing conflict
in all interfaces.
Compensation
variety and
formalization, on
the other hand,
appear to be

441 | P a g e
useful in reducing
marketing’s
conflict with R&D,
but not the
conflict with
manufacturing or
finance
departments.

Phillips, 100 hotel Planning process Factor, and The study


Davies, & managers in (participation, Neural indicates that
Mountinho UK. thoroughness, Network organizations
(2001) formalization, & Analyses employing
Data were
sophistication), strategic
collected by
communication marketing
mailed
with head office, planning with
questionnaires
perceived product
competition, new orientation gain
product better performance
development, especially for
marketing culture, efficiency and
low price culture, effective
and performance erformance. This
(efficiency, strategic planning
effectiveness, & is characterized by
adaptability) manager
participation,
planning
thoroughness,
formalization,
sophistication,
and emphasizing

442 | P a g e
new product
development.
However, planning
affects adaptability
negatively,
indicating that
formally
structured
strategic
marketing
planning may
impede innovation,
creativity, and the
success of new
product
development.

Stratis & 73 bank Strategic Multiple Findings reveal


Powers managers marketing process Regression that strategic
(2001) (strategic Analysis marketing
marketing planning
planning, multiple determines
planning modes, & financial
environmental performance, while
scanning), planning modes
strategic and environmental
uncertainty, and scanning
performance individually do not
influence
performance. All
strategic
marketing

443 | P a g e
processes in
combination affect
financial
performance
significantly.
However, due to
its small â
coefficient,
excluding the
multiple planning
mode from the
equation increases
the significance
value of F. Finally
under strategic
uncertainty, only
strategic
marketing
planning and
environmental
scanning have
effects on the
performance.

Barriers to the Implementation of Marketing Strategy

As discussed in the previous section, organizations that engage in strategic marketing


planning might achieve better performance than those that did not, especially in a
highly competitive and changing environment. Implementing strategic marketing
planning is a multidimensional activity, which includes information inputs and
analyses, planning frameworks and techniques and managerial behavior, amongst
other things. Some of the evidence also indicates that implementation of strategic
planning is not as straightforward as prescribed in the literature. Verhage and Waarts
(1988), for instance, pointed out that only 38% of Dutch organizations describe
444 | P a g e
themselves as marketing-oriented companies. These organizations made annual and
long-range marketing plans, and have better performance than those that do not.
Similarly, McColl-Kennedy, Janet, You, and Keil (1990) reported that while most
Australian companies recognized the need for formal planning, not all use it. Due to
the low overall awareness and usage of planning tools, McColl-Kennedy et al.
supposed that managers might make decisions intuitively, rather than take time to
discuss and plan formally. Meanwhile, Greenley and Bayus (1994) discovered that
only small numbers of the UK and US companies they studied could be described as
sophisticated marketing planning decision-makers who used marketing strategy as
prescribed by the literature. Most organizations appeared to be unsophisticated
marketing planning decision-makers that tended to ignore planning techniques and
organization information inputs, and used standard computer software in the
planning process, whereas the others are classified as information seekers and “gut
feeler” organizations.

Many works reported failures in implementing strategic marketing planning (Cepedes


and Piercy, 1996; Harris, 1996). In general, McDonald (1996) identified two common
barriers: cultural, a lack of belief in marketing planning, and cognitive, a lack of
knowledge and skill.

Cultural Dimension

Quite often, when an organization introduces strategic changes to improve its


performance, it fails to achieve the desired result. This does not necessarily mean that
the changes are theoretically wrong or bad, rather that the changes do not fit in with
employees’ expectations and beliefs. As a result, the employees are not motivated and
may even sabotage the implementation of the changes (Anonymous, 1980). Harris
(1996) indicated that even managers might fail to initiate strategic marketing planning
when it does not match with organizational “mind set” or culture. Leppard and
McDonald (1991) asserted that an organization is not simply a conglomeration of
people and resources. It embodied a set of values and assumptions, which generated
organizational culture and climate. Organizational culture could be the major strength
of an organization when it fit the strategies. However it could also be the main
weakness when it prevented organizations from meeting competitive threats or from
adapting to environmental changes. This indicated that to smooth the process of
445 | P a g e
strategic changes, managers should first understand the peculiar ways the
organization practises or find out the reasons - which might be perfectly good ones -
why employees acted in the way they did. In other words, understanding the
organizational culture provided the managers an unfolding context of inertia, and
facilitated the execution of the changes (Martin, 1993). This perspective is based on
the premise that organizational changes cannot happen unless people or members of
the organization change (Schneider, Brief, and Guzzo, 1996). Strategic changes are
only effective when they are associated with changes in the psychology of employees.
To change organizational culture is always a challenging and difficult task for the
managers.

Deshpande and Webster (1989) distinguished between organizational culture and


climate. They defined the organization culture as the pattern of shared values and
beliefs that helped individuals understand organizational functioning and thus
provided them with the norms for behavior in the organization, while organizational
climate related to an organization’s members’ perceptions about the extent to which
the organization was currently fulfilling their expectations. The climate of an
organization is inferred by its members. Such inferences are based on the policies,
practices, procedures, and routines that they are subject to, as well as o the kinds of
behaviors that are expected, rewarded, and supported (Schneider et al., 1996).
Organizational culture and climate are interrelated. Schneider et al. (1996) believed
that culture resided at a deeper level of psychology than did climate, as culture was
concerned with the embedded values and beliefs of its members. In other words,
climate was more tangible than culture. Schneider et al. (1996) proposed that culture
could be changed through a focus of climate. Altering everyday policies, practices,
procedures, routines, and reward systems could impact on those values and beliefs of
organization members that constituted the culture. A T & T demonstrated these
changes when attempting to sell specialized services. The changing mission could not
be realized simply by sending the employee to school, or by hiring new staff. The
change was successfully implemented only as it was backed up by modifying
organizational structure and building new roles, new incentive systems, and new
reward and punishment structures into operations (Anonymous, 1980).

446 | P a g e
Moreover, Leppard and McDonald (1991) believed that a strategic marketing planning
process embodied a set of values and assumptions. It was not merely a sequential step
of actions. Organizations that successfully implemented a planning process run on
democratic principles, promoted openness and commitment to the organization, and
had a collaborative climate and a true concern for providing customer satisfactions.
This indicates that to gain a sustainable competitive advantage and to cope with
environmental changes, the implementation of the marketing planning must be
backed up by an innovative culture and climate. The term “innovative culture and
climate” refers to the extent to which organizations emphasize inventiveness, openness
to new ideas, and quick response decision- making (Menon and Varadarajan, 1992).
Top managers of organizations trying to implement marketing planning, therefore,
should create the culture and climate needed for the planning process. Without such
conditions, strategic marketing planning is never likely to come to fruition.

Cognitive Dimension

Another barrier to marketing strategy implementation may possibly arise from


managers' lack of knowledge about marketing strategy. Marketing managers often
interpret marketing strategy as having to do with the financial budget or sales
forecasting. Consequently many so-called marketing strategies have little or no
strategic content (Piercy, 1992). This may be the result of managers’ confusion
between marketing function and marketing concept, and between marketing strategy
and marketing tactics (McDonald, 1992b). Marketing concept is a philosophical
approach to managing an organization, rather than a series of functional activities.
Moreover marketing strategy differs from marketing tactics. Marketing strategy focuses
more on the quest for long term competitive advantage and consumer advantage in the
context of the organization's mission and corporate goals., while marketing tactics
concentrate more on the design of marketing mix ingredients and requirements for
operating marketing programs (Jain, 1997; Wind and Robertson, 1983).

Lack of marketing skill also inhibits marketing strategy implementation. In order to


analyse their business environments, marketing managers can use a variety of
analytical techniques, such as Ansoff matrix, market segmentation, product life cycle
analysis, portfolio management, strengths weaknesses opportunities and threats
(SWOT) analysis, and profit impact of marketing strategy (PIMS) analysis. Research
447 | P a g e
indicates that these techniques can help managers to develop strategic marketing
planning. Utilisation of suitable techniques can reduce any inclination a manager
might have towards an irrational economic approach or unstructured judgmental
processes that may be inconsistent with profit maximization, and hence can improve
the plan credibility (Piercy and Morgan, 1994). However, evidence showed that a gap
existed between theory and practice. As mentioned earlier, Greenley and Bayus (1994)
discovered that only a small number of US and UK companies could be described as
sophisticated marketing planning decision-makers. Except for this small group, few
companies seemed to use the decision-making techniques advocated in the
prescriptive literature. Similarly Reid and Hinkley, (1989) found that most of their
respondents tended to ignore the analytical technique. They did not even know the
names when questioned about their familiarity with such techniques as Ansoff matrix,
PIMS, Experience curve, and the like. This lack of utilization of appropriate marketing
techniques might cause the failure of a marketing strategy to realise its intended
results.

Problems of understanding may be the main cause of failure to use available


marketing tools. Such problems could be related to the complexity of the tools
themselves or their application. Portfolio planning models, for example, are inhibited
by difficulties in measurement of market growth rates and relative market shares.
Wind, Mahajan, and Swire (1983) identified four different definitions of market growth
and market shares. Market growth could be defined as real market growth, market
growth, forecast real market growth, or forecast real short and long-term market
growth. Meanwhile market share could be described as a company’s share of the
served market, the company’s share versus big three competitors, a company’s share
versus that of the largest competitor, or share index. The classification of any business
into a specific portfolio position such as “dog” or “star” is very sensitive to the selection
of the measurement definition. Consequently, as demonstrated by Wind et al. (1983)
different matrix methods were likely to generate different recommendations for the
same situation.

Managers also have similar problem when applying product life cycle (PLC) analysis.
Defining the product class (market) to which the product belongs is also fraught with
difficulties. Its definition is a key point in PLC analysis. McDonald (1 992a) claimed

448 | P a g e
that it would be pointless for the manager to draw a PLC of his/her product without
drawing a life cycle of its product class. McDonald (1992a) further suggested that both
academicians and practicing managers must understand not only the analytical
techniques themselves but also the nature of interrelationship among them. This
suggestion arose for two reasons. Firstly, misunderstanding of the techniques led to
their being misused. Secondly, there was no one technique on its own that could solve
the complexity of marketing problems. McDonald believed that some inputs could be
used in some models/techniques and outputs of one model could be used as inputs to
others. This integration of some models would of course raise another dimension of
complexity. However, the availability of computer-based expert systems could
overcome human weaknesses in dealing with complexity. McDonald and Wilson (1
999b) and Chan and Dandurand (1998) indicated that use of an expert system could
improve managerial decisions and organizational performance. They believed that this
system did not exclude the manager’s judgment and intuition, which were still very
important as personal inputs and control elements in decision-making. They
emphasize, however, that the manager’s judgment needed to be continually
augmented, refined and updated with current conditions.

Summary of Focal Literature and Its Gaps

Contingency theory is a very popular approach for research in the fields of


organization theory, strategic management, organizational behavior, and marketing
(Zeithaml et al., 1988). The theory enables researchers and managers to provide the
basis for organizational analyses, which generate possible solutions to the arising
problems. The fit between organization and its environment is the central theme of
contingency studies (Venkatraman and Prescott, 1990).

However, there was a dispute among scholars about the influence of environment on
organizations, particularly concerning matters related to the strategic role of managers
in their efforts to adapt their organizations to the environment (Astley and Van de Ven,
1983). Some scholars believed that environments determined the organizational life
and constrain the managers to exercise their strategic choices (Aldrich, 1979).
Whereas others believed that managers might still have the capacity to exercise their
power and enact their strategic choices to handle the organization in line with its
environments (Chakravarthy, 1982).
449 | P a g e
In an effort to resolve the dispute, Hrebiniak and Joyce (1985) claimed that the
environment and the managerial choice were not mutually exclusive. The two factors
interact one another and could function as independent variables in the process of fit.

Because of this, managers might face four different situations in their organizations.
They might confront themselves with a combination of low strategic choices and high
environmental determinism, a situation, which is similar to the underlying
assumption of the determinism school. They might face a combination of high
strategic choices and low environmental determinism, a situation reflecting the
voluntarism’s assumption. They might also have to deal with a mixture of high
strategic choices and high environmental determinism, or a blend of low strategic
choices and low environmental determinism. These different possibilities reflected that
organizational adaptation was a dynamic process, a process that was supported by
Miles and Snow (1994) who defined “fit” as a dynamic search that sought to align the
organization with its environment and to organize resources internally to support the
alignment.

Apart from the above dispute and its solution, the concept of fit also becomes the
central trust in strategy research because the main task of the managers is to develop
and utilize a strategy to fit their organization to its environment. Basically, both
studies on strategy in general and studies on marketing strategy can be classified into
two different schools: content and process (Jemison, 1981; Rajagopalan and Spreitzer,
1997). Studies in strategy content put emphasis on the exploration of external
environment’s influence upon the strategic choices, organization’s structure, and
performance. This is understandable because most of these studies emerged from
industrial organization studies, which promoted how industries influence the strategic
choices of the organizations (Porter, 1981).

Studies in strategy content indicated that environment factors such as stability of


industry (Hambrick, 1983), technological changes (Dvir, et al., 1993), complexity,
dynamism, and hostility (Lou and Park, 2001; Tan and Litschert, 1994) determined
the strategy selected by the organizations. Studies in marketing strategy content also
revealed similar results. Burke (1984) and Cavusgil and Zou (1994), for instance,
identified that market attractiveness influenced marketing managers in setting up
their marketing strategies. While technological changes in the industry determined all
450 | P a g e
elements of marketing strategy, competitive intensity influences almost all of
marketing strategies, except the pricing one (Cavusgil and Zou, 1994; Lusch and
Laczniak, 1989). This indicates that the organizations do not use the price as a
competitive tool but emphasize more on non-price marketing strategies to cope with
the competitive pressures. In addition, to deal with a turbulence environment, the
organizations should employ decentralized structure to enable the marketing
managers to adapt any environmental changes easier (Ozsomer and Prusia, 1999;
Rajaratman and Chonko, 1995). In general, content studies underline that to gain
superior performance, the organizations must fit the strategies to their environment
(Venkatraman and Prescott, 1990). Misfit between the strategy and organization
structure to their environment might lead to inferior performance (Naman and Slevin,
1993).

In contrast to the strategy content, studies on strategy process concentrate on the


strategy or marketing strategy decision process. Most of the studies consider
environmental influences implicitly, as part of the context of the decision process
(Rajagopalan et al., 1993). This might be due to the fact that administrative behavior
studies, which primarily focused on the decision processes of the strategy being
formulated and implemented, contributed more to the existence of this school of
thought (Jemison, 1981). Several studies on strategy process indicated that speed of
decision making was a crucial determinant to gain better performance, especially for
organizations working under unstable environment (Eisenhardt, 1989; Frederickson
and Mitchell, 1984; Judge and Miller, 1991). Decision making process might be
accelerated through authority delegation to the functional manager. This authority
delegation could eliminate political behavior among middle managers (Bourgeois III
and Eisenhardt, 1988). At the same time, it might also encourage them to participate
in the strategy making, which in turn might facilitate them to implement the strategy
(Floyd and Wooldridge, 1994).

In addition, studies on marketing strategy process revealed similar findings. John and
Martin (1984) demonstrated that formalization of strategic marketing planning might
increase the credibility and utilization of the marketing strategy. This formalization
might reflect the participation of top management in the planning process and
cooperative organizational climate (Chae and Hill, 1997). It might also eliminate

451 | P a g e
interdepartmental conflicts (Morgan and Piercy, 1998), and facilitate better
coordination of decisions throughout the organization (Menon et al., 1999). Finally, the
formalization of strategic marketing planning might likely attain superior performance
(Lysonski and Pecotich, 1992), provided that it could anticipate the unexpected and
lay contingency plans accordingly. On the contrary, centralization of strategic
marketing planning could hamper the credibility and utilization of marketing strategy
(John and Martin, 1984). It tended to create interdepartmental conflicts, which not
only lessened the quality of marketing strategy formulation and implementation
(Menon et al., 1996) but also decreased the performance of the organization (Morgan
and Piercy, 1998).

Furthermore, Noble and Mokwa (1999) identified that commitment of marketing


managers to the marketing strategy determined the success of marketing strategy
implementation. This commitment could increase when the managers perceive their
roles to be significant for the success of strategy implementation, and when they
perceived that the marketing strategy fits to the broader strategic direction of the
organization. In conclusion, studies on strategy and marketing strategy process
highlighted that the internal structural fit might facilitate organizations to attain
superior performance. However, most of these studies concentrated more on the
process of strategy formulation. They tended to overlook the process of strategy
implementation, assuming that the implementation could be straightforward (Noble,
1999). Meanwhile, Sashittal and Tarkersley (1997) demonstrated that formulation and
implementation of the strategy interacted each other to cope with the environmental
changes. They also pointed out that marketing managers must improvise their
marketing strategies and implementation to fit day-to-day market changes, and to
attain their marketing objectives.

Consistent with the important role of middle managers, especially the boundary
spanning managers, in current complex and dynamic business environment (Barlett
and Goshal, 1994; Dutton et al., 1997; Floyd and Wooldridge 1992a, 1994, 1997;
Schilit, 1987), marketing managers as the boundary spanners might provide top
management with strategic issues regarding customers and competitors.

These managers might also persuade the top management to implement marketing
concept, which posits long term customer satisfaction, not current sales volume, as
452 | P a g e
the key factor to profitability of organization (Anderson, 1982; Drucker, 1969; Levitt,
1960). Marketing managers, therefore, must be critical and creative in implementing
their roles. Most importantly, they must be the integrators between organization and
its customers (McKenna, 1991).

However, implementing marketing concept or strategic marketing planning was not an


easy task. McDonald (1996) identified two common barriers in the implementation of
marketing planning. These included cultural and cognitive barriers. The marketing
planning was not simply a sequential step of actions. It represented a set of values and
assumptions. To execute the planning process successfully, an organization must
promote managers empowerment, encourage openness and commitment to the
organization, and advocate a collaborative climate and a true concern for providing
customer satisfaction (Leppard and McDonald, 1991). Similarly, marketing managers
must have good marketing skills and capabilities to analyze business environment.
The use of appropriate marketing techniques, such as PIMS, SWOT analysis, PLC
analysis, market segmentation, could minimize marketing managers to use
unstructured judgmental process that may be inconsistent with profit maximization
(Piercy and Morgan, 1994). Without the existence of these two conditions, the strategic
marketing planning might not likely attain the intended results.

So far the discussion has revealed that studies on strategy research, specifically on
marketing, concentrate in two different aspects. Content school emphasizes their
investigation on the fit of the organization to its external environment, whereas the
process school places more attention on internal fit of the organization to achieve
superior performance. However, most studies representing both schools focus on
strategy formulation. They incline to ignore strategy implementation, assuming that it
is a simple aspect. Meanwhile, empirical findings indicated that most of the strategy
failures were caused by ill implementation or behavioral problems in the organizations
(Heyer and Lee, 1992; Perlitz, 1993). Bonoma (1984) even stated that inappropriate
strategies but excellent implementation would end up with better results than
excellent strategies but poor execution (discussed in more detail in the next chapter).

Considering that organizations should attain both external and internal fit to achieve
superior performance, at the same time the processes of strategy formulation and
implementation are not separable activities; there is a need for an integrative approach
453 | P a g e
that incorporates both schools of thought. An integrative approach enables
researchers not only to analyze the environmental factors that influence the content of
the organization’s strategy but also to investigate the process, in which the strategy is
formulated and implemented to attain superior performance (Jemison, 1981; Paine,
1979; White and Hammermesh, 1981). The emergence of this approach is based on
the idea that the formulation and implementation processes influence the content of a
strategy; meanwhile, the processes themselves are determined by previous strategic
decisions (Jemison, 1981).

The current study uses an integrative approach for following reasons:

 It can eliminate the weaknesses of the content and process approaches in


theory building and testing when they are applied individually (Blair and Boal,
1991).
 It provides a more comprehensive framework (Jemison, 1981).
 It enables the researcher to investigate issues related to strategy content,
strategy formulation and implementation processes, and to explore how these
factors may influence performance (Varadarajan and Jayachandran, 1999).

With all the above considerations, therefore, this study investigates how external
environment and internal organization conditions may affect the formulated marketing
strategy. It also explores the important role of marketing managers in the processes of
marketing strategy formulation and implementation to gain superior performance.

454 | P a g e
3.7 WALDO – ORG Theory
Waldo divides the development of Org Theory into three stages:

 Classical period – epitomized by the works of TAYLOR, GULICK, FAYOL.


Classical period based on the “machine model” of the organization and
emphasized the rational aspects of human behavior.

 Neo-Classical period – Began with Hawthorne experiments in the 1920s and


the following Human Relations Movement. In contrast to the Classical stage,
the neoclassical approach emphasized the emotive and sociopsychological
dimensions of human behavior in orgs. The Human Relations Movement
focused on morale, perceptions, attitudes, group relationships, informal groups,
leadership and the bases for cooperation in org behavior. The Human Relations
Movement demonstrated the limitations of perspectives such as Scientific
Management.

 Modern Org Theory – Began with publication of March & Simon’s


Organizations in 1958. Modern Org Theory is based on an “organic” or “natural
system” model of the organization and stresses organizational growth and
survival. Simon’s work on decision making is but one aspect of modern org
theory. Waldo considers Simon’s work to be a radical departure from Classical
Org Theory. A second perspective on modern org theory is ORGANIZATIONAL
HUMANISM, represented by works of CHRIS ARGYRIS, WARREN BENNIS,
and RENSIS LIKERT. Waldo observes that the ORGANIZATIONAL HUMANISM
focuses on much the same kinds of concerns as the Human Relations
Movement. From the humanist perspective, the objective is to achieve
organizational effectiveness and self-fulfillment simultaneously, under the
assumption that the interests of the individual and the organization are
compatible. A third strand of Modern Org Theory is the “scientific and
managerial” literature (contingency theory, system perspectives).

 Waldo charges that modern org theory presents no unified “theory of


organization”. By becoming everything, systems theory runs the danger
of becoming nothing in particular.

 “A value-free theory of organizations is unattainable.”

455 | P a g e
3.8 They Dyadic Environment of the Organization
Transaction Cost Economics – Ronald Coase – Conceives organizations as
structures for governing exchange relations. Central Theme is the friction between
two organizations that arises when they are mutually dependent.

The firm – a system of specialized relationships which comes into existence when the
direction of resources is dependent on an entrepreneur rather than on the price
mechanism.

Even “free” markets are not costless – the process of transacting creates its own costs
– contracts, lawyers, negotiations, etc.

Transaction costs are the costs of contracting – planning, adapting, monitoring task
completion.

Williamson – Any issue that can be formulated as a contracting problem can be


investigated to advantage in transaction cost economizing terms. Every exchange
relation qualifies. Transactions vary in their level of uncertainty and frequency. The
greater the uncertainty and frequency, the greater the transaction costs are likely to
be. The key, Williamson argues, is the degree of ASSET SPECIFICITY – High when
assets invested in a particular exchange relationship are much less valuable in other
relationships.

Simon – Bounded Rationality – Contracts can’t possibly cover every possibility, and
people are sometimes opportunistic and lie, cheat and steal, so there is a limit where
the org is better off investing in assets than continuing to contract out.

Make or Buy decisions – TCE says the greater the asset specificity involved in
producing a particular part, the more likely that part is to be made inside the
organization rather than by an outside supplier.

The M-form – Multidivisional form. TCE proposes that some org forms are better than
others form their task. Williamson argued for the benefits of the M-form in large
enterprises. Williamson calls the functional organizational form the U-form – divided
into departments such as manufacturing, sales, finance, etc. But at some level of size,
information overwhelms the orgs ability to make decisions. The M-form solves this

456 | P a g e
with each division having its own stand-alone departments (manufacturing, finance,
sales) with its own profit-loss responsibility.

Armour & Treece (1978) – study of 28 petroleum firms, evidence for the performance
benefits of the M-form.

Difficulties with TCE: In essence, firms buy everything, so the “make or buy” decision
is a misstatement – there is nothing special about one kind of contract or another.

Resource Dependence Theory – in contrast to the rational systems approach of TCE,


resource dependence offers a natural systems perspective that highlights the
organizational politics behind choices such as the make-or-buy decision.

Resource dependence theory – 3 Core Ideas:

1) Social context matters – much of what orgs do is in response to the world of


other orgs that they find in themselves in.

2) Orgs can draw on varied strategies to enhance their autonomy and pursue their
interests. – Similar to Cyert & March’s description of “the negotiated
environment”.

3) Power – not just rationality or efficiency – is important for understanding what


goes on inside orgs and what external actions they take. This emphasis on
power and the analysis of org actions to pursue it, is the distinctive
hallmark of resource dependence theory.

How is Resource Dependence Theory different than TCE? Pfeffer - Resource


Dependence argues that organizational actions are often taken “regardless of
considerations of profit or efficiency” Pfeffer focused on formulations of power and
exchange relations among organizations at the level of organizations (Emerson
studied this at the individual level)

TCE assumes “market selection” determines firm performance. Resource Dependence


assumes more “slippage” and that orgs (esp. large ones) have a great deal of discretion
to manage their environment. Also Resource Dependence (since it doesn’t rely on
arguments of market selection) readily explains behavior from orgs of any type,

457 | P a g e
covering businesses, non-profits, or governmental orgs. In short, anywhere there is
power, resource dependence will have something to say. (Pfeffer, 1987)

Bridging Mechanisms – Efforts to control or in some manner coordinate one’s actions


with those of formally independent entities. One important bridging tactic is
Cooptation.

Selznick first described – Cooptation is the incorporation of reps from external


groups into the decision-making structure of an organization. Selznick described the
significance of cooptation and its huge costs – arguing that orgs are in effect trading
sovereignty for support.

Managing Organizational Interdependence

Interlocks, Alliances, Mergers & Acquisitions (Vertical Integration, Horizontal


Mergers, Diversification).

458 | P a g e
3.9 Organization of the Environment
Organizational Ecologists – (Hannan & Freeman, Aldrich, Pfeffer, Carroll) analyze
organizational birth, change and death as the most informative dependent variables.

Ecologists focus on: 1) study all members of the population. 2) Study processes that
take place over time rather than relations among variables at a single point in time.

Ecological conception is firmly grounded in an open system model (importance of the


environment).

(Bulk of studies the have been performed at the population-level).

Organizational Populations are comprised of all organizations sharing the same


general form.

Creating New Organizational Populations

Aldrich – most entrepreneurs establish “reproducer”(routines and competencies vary


only minimally from those of existing orgs), rather than “innovator” organizations.

Organizational Population – used to define aggregates of orgs that are alike in some
respect (institutions of higher ed, newspapers).

Hannan & Freeman noted that biological species are defined in terms of genetic
structure and proposed that the appropriate analogue for organizations is to define
them in terms of their “blueprint for organizational action, for transforming inputs into
outputs.” Hannan & Freeman add that the basic key to identifying a population is
the possession of a common organizational form.

Carroll & Hannan – Forms are viewed as “a recognizable pattern that takes on rule-
like standing”.

Stinchcombe - Imprinting – Orgs are likely to retain the features acquired at their
origin. As orgs age, they are likely to become increasingly institutionalized, their
structures and routines “infused with value” and legitimacy, resisting change (a
natural system explanation).

Meyer & Rowan – The building blocks of for organizations come to be littered around
the societal landscape; it takes only a little entrepreneurial energy to assemble them
into a structure. (1977)

459 | P a g e
How are Organizations Shaped by Broader Social-Political-Cultural Processes?
Institutional Perspectives:

Scott – Institutions are composed of cultural-cognitive, normative, and regulative


elements that together with associated activities and resources, provide stability and
meaning to social life.

Institutions as Regulative Systems

North – Institutions are perfectly analogous to the rules of the game in a competitive
team sport. That is, they consist of formal written rules as well as typically unwritten
codes of conduct that underlie and supplement formal rules. The rules and informal
codes are sometimes violated and punishments are enacted. Therefore, an essential
part of the functioning of institutions is the costliness of ascertaining violations and
the severity of punishment (North 1990)

North observes that the major source of regulatory rules and enforcement
mechanisms in modern society is the nation-state. In the regulatory view of
institutions, it is assumed that the major mechanism by which compliance is effected
is coercion. Individuals and groups comply to rules and codes out of expediency – to
garner rewards or to avoid sanctions. Behavior is viewed as legitimate to the extent
that it conforms to existing rules and laws.

Institutions as Normative Systems

(Sociologists such as Cooley, Weber, Selznick, Parsons) have viewed institutions


primarily as normative structures, providing a moral framework for the conduct of
social life. Unlike externally enforced rules and laws, norms are internalized by
participants; behavior is guided by a sense of what is appropriate, by one’s social
obligations to others, by a commitment to common values.

Institutions as Cultural-Cognitive Systems

The most recent view of institutions – “The New Institutionalism in organizational


analysis” – Powell & DiMaggio emphasize the role of Cultural-Cognitive processes
in social life.

Berger & Luckmann argue that social life is only possible because and to the extent
that individuals in interaction create common cognitive frameworks and

460 | P a g e
understandings that support collective action. The process by which actions are
repeated and given similar meaning by self and others is defined as
institutionalization. It is the process by which social reality is constructed.

The most influential applications of institutional ideas to the analysis of orgs operate
at the intermediate level of focusing on the effects of societal rules and field-
specific norms and beliefs. These socially constructed realities provide a framework
for the creation and elaboration of formal organizations in every arena of social life.
(Meyer & Scott 1983, 1994)

Institutional Agents

Theorists suggest that the two major types of collective actors who generate
institutional rules and frameworks – regulatory policies, normative beliefs, and
cultural-cognitive categories – are governmental units and professional groups.

DiMaggio & Powell assert that Bureaucratization and other forms of homogenization
are effected largely by the state and the professions, which have become the great
rationalizers of the second half of the twentieth century. (1983)

Organizational Fields

DiMaggio & Powell – Fields are comprised of diverse organizational populations


and their supporting (funding) and constraining (regulatory/competitive)
partners, all of whom operate within an institutionally constructed framework of
common meanings. Fields necessarily vary among themselves and over time in their
degree of structuration – their relational and cultural coherence (DiMaggio & Powell,
Giddens).

Giddens – the original framing of his definition, that structuration stresses the
recursive interdependence of rules and relations, of schemas and resources. As
structuration increases, meanings become more widely shared: institutional logics –
“the practices and symbolic constructions which constitute a field’s organizing
principles” become more pervasive.

461 | P a g e
3.10 Networks In and Around Organizations
A network consists of “nodes” and “ties” or relationships among the nodes. It is clear
that networks have much in common with the open systems approach where we
conceived of organizations as systems of interdependent, loosely coupled parts, among
which can flow materials, energy, and information.

Formal organizations are themselves a special case of network: roles (or jobs or
participants) are nodes connected by ties such as authority relations or informal
exchanges.

Network Thinking

C. Wright Mills applied network ideas to analyze the social connections among
members of the American “power elite” – “those political, economic and military circles
which share decisions having national consequences.” Mills’ work launched a series
of empirical studies of power elites in modern societies.

Networks can be viewed at three analytical levels:

1) The ego network – consisting of a node’s direct contacts.

2) The overall network – includes all actors and relationships within a particular
domain.

3) The network position – identifies an actor’s coordinates within the “overall


network” topography.

Baker and Faulkner (1993) – Being central in a network is not necessarily a good
thing. In their study of a famous price-fixing conspiracy network, they found that
managers who were most central in their study were most likely to be found guilty in
court.

Networks are often “multiplex”, meaning that different kinds of relationships often
overlap.

462 | P a g e
3.11 The Rise and Transformation of the Corporate Form
“…organization theory seeks to provide an understanding of the intersection of
biography and history in social structure”

“The history of the development of modern society is also a history of the development of
special-purpose organizations. Organizations were both created by and helped to
produce these changes”

1) Twentieth century: organizations and nation-states are the dominant


organizational forms

2) Organizations and their structures shape the way we live

3) Social class, religion, family life has been shaped by large organizations

4) Major social organizations (prisons, schools, government agencies) have


adopted corporate organizational structures

5) Do organizations absorb society or are they becoming increasingly absorbed by


societies?

6) Movement of rational, hierarchical organizations (Ford) to loosely coupled


(Linux) organizations

Major Organizational Forms

Transitions from major organizational forms and movement from an industrial to


service economy:

 Railroads

 U-form: central management unit with several functionally organized units

 Railroads created by an upper class elite group of “robber barons”

 Helped create the emergence of stock markets and the New York Stock
Exchange

 Set the tone for interaction between business elites and governments

 Made widespread distribution of goods possible, thus helped other industries


expand
463 | P a g e
 Manufacturing firms

 Rational, hierarchical units

 Major corporations

 Guilds as a precursor to the modern corporation

 Early corporations in the U.S. were “quasi-public agencies, granted corporate


status to build canals or roads—public projects—that were too costly for
individuals to fund” (p. 347).

 Fortune 500 (U.S. businesses with largest revenues) as the institutional field of
big business

 Fortune 500 dominated by manufacturing firms until 1995

 Major corporations have had an enormous impact on social and economic life in
the U.S.

 M-form: multi-divisional structure which included a general corporate office


and product-based or regional divisions that also had different departments

Characteristics of corporations:

 vertically integrated

 multidivisional forms

 professional managers

 no particular ties to founders

 owned by dispersed and powerless shareholders

Corporations are distinguished by three features:

1. Separate legal personality (corporation can make its own contracts and own
property)

2. Unlimited life (existence under law is not dependent on any group of people)

3. Limited liability (people who own it are not responsible for corporation debts)

464 | P a g e
American Exceptionalism

 It is assumed that the development of the corporation was inevitable…it was


not--things could have gone differently.

 Corporate revolution during 1898-1903

 No other economy experienced the same development of “vertically integrated


M-forms” like the U.S. did

o Germany: cartels, smaller firms

o France: large firms owned by the state

 “In other industrialized nations, governments grew up before business; in the


U.S. the opposite was the case” (p. 355)

 If more of a federal than state influence had occurred over banks and corporate
law, we might have a different system

 Other countries may adopt our corporate forms…some are successful and some
are not

 Role of states: New Jersey and Delaware—“corporation friendly” states; tax


shelters, etc.

The “Soulful Corporation”?

 Professional managers as more “public-spirited civil servants than rapacious


robber barons”

 Responsibility to stockholders, customers, employees, public

 Firm as an institution

 Are corporations becoming more and more like nation-states?

 Corporations increasingly take on more and more social welfare roles:

o In house legal counsel

o Internal dispute and grievance procedures

o Support local arts and communities

465 | P a g e
o Support low-cost housing

o Subsidize medicines for lower income families

o Health care

o Day care

o Higher education

Are Organizations Still the Defining Structures of Society?

 Are classes or organizations the major structures in society?

 Marx: owners and non-owners/workers

 Dahrendorf: Marx’s model is too simplistic

 Social mobility is still possible while at the same time, a widening gap between
the rich and the poor continues

Discussion Questions:

Scott and Davis point out that the U.S. corporation looks more and more like a nation-
state: “Corporations have come to look more like states in the range of their activities,
while states have come to look more like business corporations, as both compete for
each other’s business”.

 Do you see the corporation as “soulful” OR as furthering the class divisions in


our society as we continue to see the widening gap between the rich and the
poor?

 Do you think the unique development of the corporation in the United States
has contributed to its role of providing an increased range of social services
more so than possibly the federal government? Should this be the role of the
corporation vs. state or federal government?

Can societal members depend or should they depend on the corporation to ensure the
social welfare of a society? Does a corporation want to fulfill this role?

466 | P a g e
3.12 Changing Contours of Organizations and Org Theory
From unitary to multi-paradigm. Scott argues that the field of org studies emerged
from the “cleft rock” provided by the scientific management and the human relations
schools. This dualistic perspective, enshrined in the rational and natural system
perspectives remains with us up to this time. From the 1960s to the 1990s, a wide
range of competing models or paradigms for studying organizations were proposed,
elaborated, and, to varying degrees, tested. We have reviewed theory and research
associated with the bounded rationality perspective, contingency theory, transaction
costs, resource dependence, sociotechnical systems, organizational ecology,
institutional theory, and network approaches to name only the main contenders.

It is now a multiparadigm world.

We have moved from studying the micro to the macro, from studying structure to
process.

467 | P a g e
Chapter 4
Public Budgeting

MPA Comprehensive Exam


Study Guide
4.1 Major authors in Public Budgeting and their Theories
Kelly suggests budget theory runs parallel to political theory. Budgeting is an open
system capable of changing to reflect public opinion; the budget is primarily an
instrument of control, either facilitating or limiting government intervention into
public and private affairs.

Willoughby on Budgeting says used as a central controlling feature of system of


financial administration. Budget as instrument of: democracy, correlate legislative and
executive action, secure administrative efficiency and economy, feature of municipal
reform.

Budgeting was a reform introduced during the Progressive Era (1890-1920) in


response to corruption, bossism, plutocracy, and declining morality. American labor
movement grew and industrialized and scientific management practices emerged
(Gulick/Urwick POSDCORB) suggesting best management practices.

President Taft - In 1909, President Taft (1909-1913; Sec. of War, 1904-1908) asked for
$100,000 to study the methods of appropriation and expenditure. The commission
concluded that one of the problems was “an inadequate provision of the methods of
getting before Congress a definite budget.” In 1912, President Taft sent the
commission’s report. He construed the constitutional privileges pertaining to receipts
and expenditures and the state of the union as authorizing a definite, well-considered
budget.

1913 Amendment to Constitution: Federal income tax. Idea was to create an executive
budget where central figure would look at it. Great immigration population moving to
larger cities — needed control by executive.

National Municipal League (NML) called for uniform municipal bookkeeping, which
facilitated statistical analyses.

New York Bureau of Research (NYB) was a corps of municipal experts with specialized
skills and a new ethic of social morality. NYB argued that improvement would be
possible with introduction of efficient business and accounting practices. New York
was first state to write budget idea into its constitution.
469 | P a g e
Before 1912, a variety of techniques were recommended for centralizing and locating
responsibility for the budget (Congress, executive, comptroller, or board). After 1912,
many reform proposals called for budget proposals from the chief executive.

Before 1912, many of the reformers wanted to take money saved by increased
efficiency and spend it on social programs and capital projects. After 1912, the focus
was on saving money to reduce the scope of government services and tax burden.

In the Taft Administration, emphasis was changed from obtaining public input into
the budget plan to requiring detailed reports from responsible administrators on what
they had already done.

NYB was founded by William Allen, Henry Bruere, and Frederick Cleveland and was
funded by donations from businessmen.

The Ideological Differences

Progressives Conservatives

Expansionary, activist government Smaller government

Cost Effectiveness Efficiency and cost cutting

Trusted the public and its No trust in the public and its
representatives representatives

Proposals had some influence, but Proposals had a major influence on the
have fallen out of favor 1921 Budget and Accounting Act

Egalitarianism; No representative had Government by elites who would ignore


the right to defeat the will of the popular demand and exercise their own
people. All men are basically equal. judgment.

Government by the people through Suspicion of legislative bodies that would


the initiative, the referendum, and give in to public sentiment.
the recall of judicial decisions.

Three Budget Reform Movements

470 | P a g e
1. Municipal Reform Movement (1870s): NML (municipalities instituting budget idea).

2. Research Bureau Movement (1906): NYB introduce good business practices.

3. Budget Reforms (1914-1920): Taft Commission on economy and efficiency.

The Budget Reformers

 Frank Goodnow
 An early progressive who became a conservative.
 In 1913, he wrote about the delegation of budgetary discretion from the
legislature to the executive, advocating that the legislature maintain
some control in case the executive did not manage efficiently.
 In 1916, as chairman of the Maryland Economy and Efficiency Commission,
he strongly endorsed the executive being responsible for preparing and
submitting a budget subject to limited legislative budgetary power.
 Frederick Cleveland
o Also a progressive who became a conservative.
o Argued that the public should be involved in all decisions concerning an
increase or decrease in taxation.
o In 1912, before the release of the Taft Commission report, was not
advocating a strong executive. His solution was to educate the public
and provide administrators with the information they need to govern
efficiently.
o In 1913, recognized the need for the chief executive to review agency
requests and present budget proposals to the legislature because the
executive is the one responsible for implementing policy. The
legislature gets a yes-or-no vote on the proposal.

Changes in the Content of Reform Proposals

In 1921, the Budget and Accounting Act established the Bureau of the Budget (BOB)
and General Accounting Office (GAO). Rational budget reforms with focus on
management was influence by the Brownlow Committee of 1937 with BOB transferred
to office of President. Brownlow Report references performance budgeting, included

471 | P a g e
expansion of government action in linking inputs to outputs. It recommended
changes to address administrative management.

The Management Movement (1946-1958)

Post WWII economic boom, high confidence to conquer problems scientifically lead to
prosperity. Hoover Commission (1949) suggested more data in the budget process
which involved performance budgeting.

TYPES OF BUDGET REFORMS

Management by Objectives (MBO) placed less emphasis on long-range planning and


program evaluation. It respected the jurisdictional lines. It encouraged a more
participative approach with line managers involved in the setting of agency objectives.
MBO consisted of three-part cycle:

1. managers and subordinates agree on measureable, results-oriented objectives;


2. milestones to be achieved to reach the objectives were established;
3. results were evaluated via a year-end report to department head. MBO failed to
establish itself as an activity essential to budgeting.

Zero-based Budgeting (ZBB): The process was result of experiments at the private,
state and local governments. President Carter brought the idea from Georgia when he
was governor. Carter felt the federal budget process was inefficient, chaotic, and
uncontrollable. ZBB would help to reduce costs and make the federal government
more efficient and effective by:

1. improving quality of budgetary information,


2. involving line managers in the decision process, and
3. emphasizing the kind of analysis that would yield better understanding on cost-
effectiveness of programs.

ZBB used four-step process:

1. organizational objectives were determined and decision units defined;


2. each decision unit was analyzed within framework of decision package;
3. prioritizing—ranking of decision packages; and

472 | P a g e
4. organization’s budget was formulated based on funding needs.

Problem with ZBB was the little evidence of its effect on outcomes. It had little impact
on budgeting in Georgia. It did not take into consideration past history. Although ZBB
went out with Carter, the Reagan administration tried zero-based for several programs
(i.e. Amtrack and Appalachian Regional Commission) but Carter restored the funds.

These reforms were unsuccessful due to inconsistencies with political nature of budget
process and workload and paperwork requirements that were overwhelming.

Management Reforms

Gramm-Rudman-Hollings (GRH) of 1985 because of large federal deficit, set ceiling for
deficit. Cannot create programs without identified revenues to support them. Gov’t is
not the solution – it’s the problem. Reduce it and you reduce problem. Reagan supply
side economics – lower taxes stimulate economic growth and induce investment,
create jobs, deficits would shrink due to increased supply of money. Cut back
budgeting, backward budgeting (aka revenue budgeting). Clinton spurred on by
Reinventing Government (Osborne and Gaebler) – National Performance Review (NPR)
and Government Performance and Results Act (GPRA), GAO – General Accounting
Office all focus on performance-based budgeting. Congressional Budget and
Impoundment Act of 1974 reasserted congressional influence over the budget.

GPRA emphasis was on outcomes. Federal agencies were to set goals, measure
performance against them and report on the progress within framework of long-term
and annual performance plans. The Government Performance Results Act (GPRA) of
1993 was a product of the National Performance Review directed by Vice President Al
Gore. It called for agency strategic planning, performance measures, focus on
customer involvement and measure outcomes of government actions. Its connection to
CFO Act is that in the future, budget numbers will accurately relate to audited
statements of government assets and liabilities.

The ability to implement performance measurement requirements of GPRA is


expensive to perform properly. Agencies that participated in the pilot process
demonstrated the effectiveness of GPRA. Strategic and performance planning enabled

473 | P a g e
review and reformulation of agency missions, achieved greater clarity of objectives and
resulted in a better understanding of relationships between mission and outcomes.

474 | P a g e
4.2 Budgeting Theory
Irene Rubin - suggests the idea of budgeting did not begin with private agencies but
with public agencies and academia. Innovation in cities spurred by NML and NYB,
which were comprised of practitioners and academics, included the adoption of budget
systems and accounting practices that were not drawn from business but were
invented for the cities (e.g. Chicago and Minneapolis). Rubin’s Real-Time Budgeting:
requires flexibility in decision-making because of environmental changes, variables
and actors. Budgeting is non-linear and non-sequential. Time is a constraint,
deadlines, timing.

Budget theory should be able to answer questions about why particular practices
should be adopted, the importance of particular tasks, and the location of particular
tasks in a larger process.

Budgeting does not currently have a theory in this sense.

Writers on budgeting do not agree on common assumptions or recommendations.

There is no widely accepted set of linked hypotheses concerning cause and effect in
budgeting.

Budget theory today is fragmented and incomplete.

Budget theory is in the process of being invented.

Factors Inhibiting Theory Development

In his renowned 1940 article, V.O. Key Jr. lamented about “the lack of budget theory.”
Twelve years later (1952) Verne B. Lewis attempted to construct a normative budget
theory. More recently in one of her numerous writings, Irene Rubin noted that “budget
theory today is fragmented and incomplete…It is in the process of being invented.”
Why is budget theory still viewed as incomplete and fragmented?

 There is no consensus as to what budgeting actually is, whether it is a political,


economic or social process.
 There is no agreement on the scope of a budget theory.

475 | P a g e
 There is no agreement on the dominant methods or techniques of budgeting.
 There is no agreement about the approach a budget theory should take.
 There is no agreement about the status of theory itself.

Wildavsky notes that budgeting must provide continuity for planning, flexibility to
respond to crises, rigidity to control spending, and openness for accountability.

Process: There is no agreement on the “type” of process budgeting is. The literature
often speaks about a “political process” or an “economic process”. However, Schick
and Seligman (1926) have noted that budgeting encompasses so much that it has
become a “social process”.

Scope of Theory: budget debate centers on the scope of theory and the number of
actors to be included in the theory.

V.O. Key question: On what basis shall it be decided to allocate X dollars to Activity A
instead of Activity B?

Suggestions offered by Key:

A careful and comprehensive analysis of the budgetary process is needed, involving


answers to the following questions:

 What forces go into the making of state budgets?


 What factors govern decisions of budgetary officials?
 Precisely what is the role of the legislature?
 Studies should be performed on congressional appropriating processes.
 Personnel in budgetary agencies should not rely solely on persons trained
primarily in accounting and fiscal procedure.
 New training schemes are needed for those in budgetary roles.

Traditional Budget Theory

Public budgeting has been viewed from 3 perspectives: economics, management and
political science.

476 | P a g e
Resource allocation processes serve to create and continuously recreate the public
organization.

Normative Theory

The problems of developing normative in a political environment are compounded in


budgeting where outcomes are direct precipitants of the clash of political values.

The organization-based approach to budget theory allows normative theorists to focus


on the organizational role of the finance official and distinctive competencies of the
profession.

The formal budget process should be open to political participation and the
underrepresented must be represented by the organization.

Descriptive Theory

Research should focus on the determinants of the elements of the budget process, as
well as the determinants of budgetary outcomes favored by political science and the
normative standards for the mix of outcomes targeted by economics.

Incrementalism Theory: Dominant budgeting method.

Rubin, LeLoup, Bozeman, Straussman and others cast aside incrementalism as a


theoretical construct. Rubin and Schick believe that as an explanatory model, it did
not describe the process well, noting that a budgetary base is not always defined.

 Argues that budgeting occurs virtually exclusively inside government.


 Based on the theory of bounded rationality.
 Government is not directly controlled or controllable by society.
 Interest groups exist, but do not determine outcomes.
 Little public involvement in budgeting.
 Weaknesses of incrementalism:
o Incrementalists underestimate the relationship between the society and
budgeting, and overestimate the autonomy of agencies in determining
budgets.

477 | P a g e
o Did not envision the budget process as responding to societal problems,
emerging situations, or environmental changes.
o Assumed that budgets will continue to be allocated the same way from year
to year.

Public Choice Theory:

The shared assumption is that human behavior is based on individual economic


rationality and the maximization of individual benefits, or what economists call utility.

Public choice theorists emphasize the very important issue of the relationship between
what the citizen taxpayers want government to do and spend and what it actually does
and spends.

They assume individuals behave in a rational and utilitarian manner, reveal


preferences in a marketplace fashion (exchange of votes). The voting mechanism then
provides a chance for citizens to choose among candidates that best reflect their own
spending priorities.

 Rooted in political economy of old.


 Pareto—if one person better off from a policy and no one is worse off, then
community as a whole is better off.
 PCT points to greedy bureaucrats and assume conflict, not cooperation.
 Weaknesses of public choice theory:
o Theory tends to be deterministic and has difficulty explaining change
over time.
o Cannot explain why government has not expanded further.
o Impossible to arrive at a figure for aggregate individual utilities.

Principal Agent Theory

 Focus on relationships between those who allocate resources (principals) and


those who provide agency services (agents).
 Principals contract with agents to provide public services and the primary focus
is the contract (or budget).

478 | P a g e
 The budget or contract is the central element for what connects the principal
and agent.
 Issues involved in the exchange of info, unexpected results in the budget may
emerge: adverse selection—principal picked the wrong agent or incorrectly
identified the agent’s role; moral hazard—agent changes his/her behavior to
where it could damage the principal.
 Principals often set the policies and goals while agents implement the programs
that will address the policies and goals.
 Problems:
o Information asymmetry – agent expert withholds info from principal.

BUDGET SYSTEMS

 Traditional Budgeting is line-item and incremental


 Condemned because:
 Mindless lines do not match programs.
 Irrational, deals with inputs instead outputs.
 Shortsighted, covers one year instead of many.
 Fragmented, only changes are reviewed.
 Conservative, changes tend to be small.
 Purpose:
o Accountability, linking expenditure activities to responsible officials.
o Control, makes sure funds are being spent for designated activities.
o Efficiency, doing things for the least cost.
o Effectiveness, achieving results.
o Economic, managing and planning.
o Political, mobilizing support to implement decisions.

Positives of traditional budgets:

 Calculations are easy, not comprehensive.


 Present is appropriated to the past and not the future.
 Choices that might cause conflict are fragmented.
 Change in objectives without challenging organization survival.

479 | P a g e
 Simpler, easier, more controllable and more flexible and adaptable.
 Incremental approach permits funding levels for agencies be set based on either
or both houses adjustment to prior year totals or budget requests (usually at
lower figure of either house).
 Problems:
o Budget involves history but may not look back far enough.
o Quick calculations maybe worse than none at all if grossly in error.
o Simplicity may become simplemindedness.
o Policy neutrality may end up with disinterest in programs.

Unit of measurement—Cash/Volume:

 Cash okay when purchasing power remains constant.


 Volume is better when value of money fluctuates (inflation) in order to provide
designated volume of activity.
 Volume budgeting may be counter-productive in fighting inflation by
accommodating price increases rather than struggling against them.

Appropriations or Treasury Budgeting:

 Treasury budgeting involves tax expenditures and mandatory entitlements.


 Tax expenditures are forms of tax reductions (home ownerships, college tuition,
medical expenses, child tax credits) taken at source.
 Mandatory entitlements provide anyone eligible for certain benefits must be
paid regardless of total amount—no cap.
 Economic management can have problems—tax expenditures may rise
independent of economic conditions.
 Treasury budgeting is popular because of its value in coping with conflict,
calculation, and economic management. Ultimately, pre-programming will be
such a large amount of available funds, there will be less flexibility.

Planning, Programming, Budgeting System (PPBS)

Robert McNamara, Secretary of Defense used planning, programming and budgeting


(PPB) and part of that was a 5-year defense plan (FYDP). The plan involved multiyear

480 | P a g e
budgetary decisions and integrated different appropriations into a single programmatic
effort.

PPB, horizontal comprehensive. Planning is the focus and important part of PPB.
Policy analysis is used to increase effectiveness. Change in one element effects change
in every element in the system. It is difficult to effect change since objectives are linked
to programs. Programs are at risk.

PPBS is rational budgeting, more in the interest of policy analysts. It uses multiple
years for budgeting. Must assess alternatives. Congress did not like this method. It did
not last.

Zero-based Budgeting (ZBB)

 ZBB, no yesterday. Nothing is taken for granted.


 Agencies were to determine the place or base at which any further drop in
funding would render the agency’s operations unviable.
 ZBB was implemented as a cost savings measure by allowing both decrements
and increments above a baseline.
 It provided for more ground-up budget building and therefore greater
participatory management.
 Problems
o ZBB was a process and processes don’t trim or expand budgets.
o Defining performance was a problem.
o A-historical increases sources of error while decreasing chances to
correct mistakes.
o Both have reason equivalent to ranking objectives. But ranking objectives
without considering resources is irrational.
o But that also yielded greater amounts of paperwork.

Performance-Based Budgeting (Outcomes Budgeting is similar)

 Government performance measurement involves a complex web of relationships


from inputs to outputs to outcomes, and contexts resources to results (Joyce).

481 | P a g e
 Any form that attempts to incorporate performance measures in the budget
process (Kong).
 A means of focusing attention on the ends to be served by the government
rather than on the dollars to be spent (Seckler-Hudson).
 PBB does not exclude Congress—agencies must consult with Congress.
 Focuses on effectiveness with outcomes.
 Early starts of PBB was part of scientific management. It was used in the
military. The Budget and Accounting Procedures Act 1950 expanded PBB to
civilian govt agencies.
 Osborne and Gaebler published info that states were pressured into
“reinventing” themselves. Al Gore continued the process with “Reinventing
Government” after GPRA was passed.
 Allen Schick – stated that efforts to budget on the basis of performance almost
always fails.
 Government Performance and Reforming Act (GPRA) 1993 passed to assure govt
accountability, efficiency and effectiveness. Established a performance
management framework for fed depts. Requires agency strategic plans, annual
performance plans, and annual performance reports. It emphasized formal
strategic planning with stakeholder consultation.
 Program Assessment Rating Tool (PART) was initiated to provide score for
effective agencies. There was a need to use performance info for budgeting.
PART addresses or subjects all programs to establishing purpose, strategic
planning, managerial performance and establishing outcomes.
 Problems
o Need to get decision makers to use the information.
o Info advises but does not direct decision-making.
o Public budgeting is intrinsically political; may apply only to management
and non-political decisions.
o Mixed result at state level—no penalties for failures.
o Unlikely that single approach to relating performance and financial
reporting fits every organization context, mission, perform planning, etc.
o Out of sync with president cycle (every 3 yrs update plans).

482 | P a g e
o Failed to approve any managerial pilots.
o Govt wide perform plans got little attention from Congress so OMB
stopped developing.

BUDGETING THEORY:

 RUBIN - THEORETICAL SCHOOLS - Each theoretical school makes its own


assumptions:
 NEO-MARXISTS: – argue that class interests dominate budgeting and allocation
choices.
 Argue that govt is controlled by capitalists, or those who own the means of
production, and that they determine spending priorities to serve their own
needs.
 Military spending enriches arms manufacturers and tax breaks routinely go to
the well-off instead of the poor.

PUBLIC CHOICE THEORY:

 Assumes that human behavior is based on individual economic rationality and


the maximization of individual benefits (utility).
 Public choice theorists emphasize the relationship between what the citizen
taxpayers want govt to do and spend and what it actually does and spends.
The voting mechanism then provides a chance for citizens to choose among
candidates that best reflect their own spending priorities.
 PROBLEM WITH PUBLIC CHOICE THEORY – Impossible to calculate a figure
for aggregate individual utilities.

INCREMENTALISM: Based on theory of BOUNDED RATIONALITY

 Argues that budgeting occurs virtually exclusively inside govt


 Govt is not directly controlled or controllable by society
 Interest groups exist, but do not determine outcomes
 Little public involvement in budgeting
 SCHICK – “Congress institutionalized incrementalism in the form of the current
services budget (president must submit to Congress a current services budget

483 | P a g e
that displays budget authority and outlays for new fiscal year as if all programs
and activities were carried on at the same level in the new fiscal year).
 Weakness of Incrementalism – did not envision the budget process as
responding to societal problems, emergencies or environmental changes.

HIERARCHY THEORY:

Argues that top-levels of the executive branch make decisions about broad policy
issues, judge the environment, and pass that info down through the budget office to
the agencies before they make their requests.

KEY – (1940) The Lack of a Budgetary Theory. Posed the key question in
budgeting:

“On what basis shall we allocate X dollars to activity A instead of activity B?”

LEWIS (1952) – He is a RATIONALIST. Response to KEY reflected the


administrative environment at the time:

 We can use science to determine how to allocate resources. The problem, Lewis
argued, is not that we lack theory, but that we lack facts.
 It can be shown that the problem in government arises out of a lack of firm
numbers rather than out of the lack of a method.
 “For X level of funding, Y level of service can be provided.” The cost of a thing is
the amount of other things we must give up for its sake.
 LEWIS concludes in the end that Key’s question is unanswerable.
 It was in this era of optimism undergirded by confidence in rationality and
science to bring about optimal decisions that Lindblom’s 1959 essay appeared.

LINDBLOM (1959) – INCREMENTALISM, MUDDLING THROUGH, MUTUAL


ADJUSTMENT OR BARGAINING.

 Argued that what KEY sought (a theory) and what LEWIS found (better facts)
were prescriptive, not descriptive.

484 | P a g e
 “POLICY is made at the margins, but not through the application of marginal
utility theory. It is a system of successive limited comparisons among a finite
number of alternatives.

LINDBLOM - STRATEGY FORMULATION IN THE PRIVATE SECTOR:

ADAPTIVE MODE:

 Many decision makers with conflicting goals bargain amongst themselves to


produce a stream of incremental, disjointed decisions. Some of the literature
describes this as “entrepreneurial mode”.
 KEY & WILDAVSKY believe that budgeting equals a full-blown theory of
government, and is a supremely POLITICAL PROCESS.

WILDAVSKY – (1961) - In response to Key’s lament on our lack of a budget


theory:

 Wildavsky advanced Lindblom’s argument by pointing out that BARGAINING


AND NEGOTIATION, not the application of science, determines public policy.
The former is associated with democracy while the latter is not.
 Incrementalism promotes stability and is permeable to the political system,
which translates public preferences into policy outcomes.
 “To have such a theory would amount to a POLITICAL THEORY, some way to
reconcile the inherent tension between the public’s desire for government action
and their resistance to revenue extractions that will make it possible.” In other
words, budgetary theory would tell us the proper role of government in society.
 Wildavsky warned against the zeal of reform: “Surely it is not asking too much
to suggest that a lot of reform be preceded by a little knowledge.” (Best way to
summarize this cycle of American politics).

WILDAVSKY – Wildavsky’s STRATEGY FOR BUDGETERS:

 OPPORTUNISM – Beyond the apparently linear logic of the public sector and the
nonlinear logic of the private sector, ONE OTHER way of thinking exists in
Wildavsky’s strategy for budgeters – Opportunism.

485 | P a g e
 His budgetary person is OPPORTUNISTIC and follows neither a linear or
nonlinear logic.
 The budgeter builds confidence, finds allies and shows results.
 This is similar to MINTZBERG’S “EMERGING STRATEGY”, which many have
found appropriate for public sector managers.
 Public organizations can be seen as low on deliberate strategy and high on
emergent and unrealized strategy.

KELLY (2005) – We have an answer to V.O. Key’s question about a theory of


budgeting. History provides it:

 A theory of budgeting is a theory of political cycles driven by changing public


opinion about the proper role of government.
 Two enduring features of budgeting practice that emerged in the past century:
Incremental budgeting reflects Americans’ preference for incremental policy
change. The traditional line-item format provides financial accountability.
 When Americans trusted the private sector more than the public sector, budget
reform was centered on cost control and improved efficiency. When Americans
turned to government to solve problems the private sector could not, budget
reform was centered on programmatic effectiveness.
 Budgeting is an OPEN SYSTEM capable of changing to reflect public opinion.
 The Budget is primarily an instrument of control, either facilitating or limiting
government intervention into public and private affairs.
 Budgeting theory has shifted with political cycles like a pendulum: On one end
is concern for the public interest and social justice, on the other end is concern
for private interest and economic prosperity.
 “We have a theory of budgeting, or at least of budget reforms, and it is a theory
of public opinion cycles”.
 Changes in our orientation toward govt (either obstacle to the public good or
agent of the public good) fuels the desire for change in the budget process. The
starting point for budget reform is ALWAYS a constant – THE TRADITIONAL,
LINE-ITEM, COST-CONTROL BUDGET. We may dress it up in different ways

486 | P a g e
(performance measures, program evaluations, cba), but THE TRADITIONAL
BUDGET IS OUR TOUCHSTONE OVER TIME.
 Nothing that followed the New York Bureau of Municipal Research’s LINE-ITEM
BUDGET was as important to the profession as IDENTIFICATION OF COST,
and nothing remains as useful in what we think of as accountability, either in
the financial or political sense.

Neuby (1997) – On A Lack of a Budget Theory.

 There is no consensus as to what budgeting actually is, whether it is a political,


economic or social process.
 No agreement on scope, dominant methods or techniques of a budget theory.
 No agreement about the status of theory itself.
 The general focus of budgeting is on the allocation of resources, but there is no
agreement on whether that is primarily a political or economic function.

Schick & Seligman (1926) – “Budgeting encompasses so much that it has become a
SOCIAL PROCESS.”

KEY & WILDAVSKY believe that budgeting equals a full-blown theory of government,
and is a supremely POLITICAL PROCESS.

SCHICK – “Congress institutionalized incrementalism in the form of the current


services budget

PRINCIPAL-AGENT THEORY: Principals and Agents manage information and both


may act in their own self-interest.

 When there are issues involved in the exchange of information, unexpected


results in the budget may emerge:
o Adverse Selection – Principal chose the wrong Agent or has incorrectly
identified the Agent’s role, responsibility or agenda.
o Moral Hazard – Agent that works for Principal may change behavior to
such an extent that it damages Principal.
o Goal Conflict is assumed to exist between agents and principals.

487 | P a g e
o Information Advantage – Agency domination (agent controls information),
Legislative or Executive Dominance.
o Issue Network – Neither principal or agent has informational advantage
and goals are not necessarily in conflict.

PUBLIC CHOICE THEORY:

Rooted in the political economy of old – study of govt. behavior and classical
economics of Smith, Mill & Marx.

Assumptions of PCT:

 Self-Interest – bureaucrats will strive to maximize their budgets (NISKANEN,


1971) or their slack (diff. between revenue and minimum cost of production.
 Science of exchange of public goods by self-interested bureaucrats and
politicians, interest groups and voters.
 Analysis should take perspective of the representative individual (OSTROM).
 Pareto-Optimal Solutions – If one person is better off and no one worse off, then
community is better off.
 Power of competitive marketplace to optimally allocate resources (Coase’s
Theorem)

FORRESTER (2001) – Public Choice Theory and Public Budgeting: Implications for
the Greedy Bureaucrat.

Public Choice Theory says self interest and the free market explain why people (and
bureaucrats) make the decisions they do. Public Budgeting is different but can learn
from Public Choice Theory.

OSTROM- Supported Public Choice Theory. Normative theorist Public Choice


Theory reflects principles of self-governance and self-interest as seen in the Federalist
Papers. If these principles are reflected in units of govt. then centralization and
hierarchy are held to a minimum.

BUDGETING SYSTEMS/METHODS:

TRADITIONAL LINE-ITEM - remains the dominant budgeting method to date.


488 | P a g e
Control-based – Executive has line-item control over expenditures.

INPUTS are the single scope of analysis

Decision units are simply objects of expenditure

INCREMENTAL policy making style

PERFORMANCE BUDGETING:

 MANAGEMENT IS KEY
 Focus on INPUTS and OUTPUTS
 Decision units – ACTIVITIES of agency
 INCREMENTAL policy making style

PLANNING PROGRAMMING BUDGETING (PPBS):

 PLANNING is KEY
 Focus is on INPUTS, OUTPUTS, PLANNING and ALTERNATIVES (Managers
must submit alternative budgets)
 Decision units – PURPOSE OF AGENCY/MISSION
 CENTRALIZED policy making style – CUTS CONGRESS OUT OF BUDGETING
PROCESS.

TBB – RUBIN – TARGET BASED BUDGETING

Must have accurate revenue projections to be useful.

PBB – PERFORMANCE-BASED BUDGETING – GPRA (1993) is the legislation that led


to adoption of PBB at fed level.

RESULTS-ORIENTED. Focus on OUTCOMES and ACCOUNTABILITY.

STRATEGIC PLANNING, MANAGEMENT and GOAL ATTAINMENT (established by top


admin.)

Focus – INPUTS, OUTPUTS and OUTCOMES.

Agencies must create: STRATEGIC PLANS, GOALS, PERFORMANCE REPORTS

489 | P a g e
UNLIKE PPBS, CONGRESS IS INCLUDED IN THE PROCESS.

WILDAVSKY, LEWIS AND LINDBLOM form the main camp in support of


incrementalism as theory. They feel that it should be the basis of theory because the
process dominates reality.

LEWIS notes that recent budgetary reforms such as ZBB and PPBS have NOT altered
the use of a base figure and increments.

RUBIN, BOZEMAN STRAUSSMAN and others cast aside incrementalism as a


theoretical construct.

RUBIN & SCHICK – believe that as an explanatory model, INCREMENTALISM does


not describe the process well, noting that a budgetary base is not always defined.

BUDGET REFORM MOVEMENTS:

HALACHMI (1997) – Reform is not a process that is based on reasoning or objective


rational analysis. In democratic and open societies, govt reform is principally about
forging a new popular consensus on national or governmental objectives.

JACKSONIAN ERA (1829-1872): PRIVATE REGARDING

Laissez-faire economics were still orthodoxy – It had been the founding principle of a
commercial republic.

Budgeting was simply accounting for revenues and expenditures.

States took the lead in financing the railroad boom early on. However the economic
panic of 1837 caused several states to default on their bonds.

PROGRESSIVE ERA (1873-1921): PUBLIC REGARDING

Depression struck in 1873 and lingered through the 80s – only to hit harder in 1893.

Citizens hurting – and very aware that great wealth was accruing to the titans of
industry. Populists called for restrictions on corporations, especially the railroad
industry.

490 | P a g e
New role of govt. redefined from an environment where private interests could flourish
to protecting the public from “big business”.

Municipal Reform Movement (1870s)

Response to over-expansion after the Civil War and a recession in 1873.

Focused on limiting costs by examining departmental estimates and buffering


government from demands by using comptrollers and/or boards, by requiring
agreement among several officials and in some cases by requiring taxpayer approval.

The single executive model fit the activist progressive era better. The NML’s model
charter (budget template) advocated the mayor proposing a budget to the council,
which could reduce it but not increase it.

ACT TO REGULATE COMMERCE – 1887 – Created the ICC – INTERSTATE


COMMERCE COMMISSION

Congress gave ICC authority to investigate complaints against the railroads.

Research Bureau Movement (1906) Bureaus of Municipal Research, National


Municipal League & the New York Bureau (1907)

Dahlberg (1966) – “Bureau founders believed that wasteful, ineffective government


could not serve democracy well.”

Bureaus were non-partisan and independent, committed to a scientific method of


administration patterned after the private sector and focused on efficiency and
economy (Dahlberg).

Bureaus researched and documented conditions that needed government action and
they surveyed cities to find the most effective form of government.

They devised and spread budget reforms that emphasized goals, costs and public
involvement.

Schick (1966) notes that the NYB tried to walk the line between the reform agenda
and the more management-oriented efficiency goal and ultimately chose control.

491 | P a g e
Budget Reforms (1914-1920)

Supported a strong chief executive who could control departmental and agency
requests, cut spending, gave tax relief to business, and provided accountability after
the fact rather than through prior restraint.

TAFT COMMISSION on Economy and Efficiency created in 1910.

TAFT COMM produced a 1912 report: “The Need for a National Budget”. In included:

President reviews estimates of the agencies and consolidates estimates into a unified
budget to Congress.

Budget would include summary financial stmt for the past year.

Summary of all proposed expenditures was to be included

Summary of proposed legislation to enable Admin to conduct public business with


greater economy & efficiency

Treasury Dept. should submit financial report to Congress showing revenues and
expenditures

A uniform system of accounts be established under the president’s authority

President should be responsible for recommending the content of appropriation bills.

FEDERAL TRADE COMMISSION ACT (1914) – Regulated working hours, working


conditions and protected consumers from unfair business practices.

ROARING TWENTIES(1921-1932): PRIVATE REGARDING

WWI redirected public attention from the Progressive agenda, for obvious reasons.

Enthusiasm for a science of administration developed during this period.

Post WWI prosperity restored American confidence in Business.

BUDGET AND ACCOUNTING ACT – 1921

492 | P a g e
Created a centralized executive budget, requiring the President to submit unified
budget annually to Congress.

Contained most of the recommendations of the TAFT COMMISSION

Established BOB and GAO.

Followed similar reforms at state and local levels (not sudden change)

Lasting effect – increased Presidential power over the budget

Lasting legacy – Institutional – both BOB & GAO influence debates over budget policy

CRASH OF 1929

Confidence in a self-regulating economy was high, which makes the stock market
crash of 1929 easier to understand.

Political response to crash of 1929 and the depression that followed was slow. Even
as unemployment climbed toward one fourth of the workforce in 1931, Herbert Hoover
still maintained that government had business interfering in private economic matters.
However, public opinion had already started shifting toward the public interest.

DEPRESSION AND THE NEW DEAL (1933-1945): PUBLIC REGARDING

RATIONAL BUDGETING REFORMS

BROWNLOW COMMITTEE (1937) Roosevelt appointed committee to study the


management of the executive branch. Recommended REORGANIZATION to address
admin mgmt. BOB transferred from Treasury to Office of the President.

The old role of budgeting was cost control and prevention of malfeasance. The new
role would be management in pursuit of public good.

GULICK & URWICK (1937) – POSDCORB- Included BUDGETING as one of the


elements.

Gulick forcefully argued that budget professionals should share the pursuit of the
public interest through the exercise of best management practices.

493 | P a g e
THE MANAGEMENT MOVEMENT (1946-1958): PRIVATE REGARDING

The Truman admin attempted to build on the New Deal, but found itself thwarted by a
conservative Congress and another shift in the American political cycle. The Allied
victory and postwar economic boom reinvigorated confidence in the ability of science
to conquer problems and confidence in the private sector to accomplish economic
prosperity. Scientific Management movement growing and scientific principles of
administration promised public sector improvement and productivity.

HOOVER COMMISSION (1949): Commission found glaring weaknesses in


departmental budget processes.

PERFORMANCE BUDGETING - More data in budget process, EFFICIENCY


ORIENTED.

Idea for performance budgeting started in local cities.

Basic idea was to adopt a budget based on functions, activities and projects. It would
enable mangers, agency heads, elected officials and citizens to have some insight into
the costs associated with govt activities.

Gave detailed breakdowns of unit costs and agency outputs. Emphasis on integration
of program information and budgeting.

PLANNING-PROGRAMMING BUDGETING (PPBS) – (1961) Instituted by DOD.

Focus on EFFECTIVENESS, results and on multi-year planning.

Competing expenditures are evaluated on their marginal benefit to the program


objective.

Started in the Defense Department.

Hailed as a way to integrate planning and budgeting by using systems theory and
cost-benefit analysis.

494 | P a g e
Superior to Performance Budgeting because in PPB, substitutable alternatives are
evaluated for their contribution to the program objective, yielding higher-quality
budget decisions.

Basic idea: Budgeting should be based on OUTPUT categories rather than INPUTS.

PPBS combined two management techniques: Program forecasting (goal-oriented) and


Systems Analysis (studies outputs by means of quantitative methods).

PROBLEMS WITH PPBS – SCHICK: “Created enormous information and analytical


burdens”.

MANAGEMENT BY OBJECTIVES (MBO) – Offered NIXON a way to align budget


activities with objectives.

Emphasis on long-range planning and program evaluation.

Encouraged a more participative approach with line managers involved in setting


agency objectives.

MBO Three-Part-Cycle: Managers & subordinates agree on measurable, results-


oriented objectives. Milestones established. Results evaluated yearly via report to
department head.

MBO failed to establish itself as a viable budgeting activity.

CONGRESSIONAL BUDGET AND IMPOUNDMENT ACT (1974)

Prior to this Act, Congress never examined or voted on overall spending, revenues or
fiscal policy.

Culmination of 30 yr effort to give Congress more influence on budget.

Provided Congress with House & Senate Budget Committees and the CBO.

Spurred by Nixon impoundment actions

Lasting effect – pendulum swinging back in direction of legislative control.

495 | P a g e
ZERO-BASED BUDGETING (ZBB) – (1977) Focused on the base rather than annual
review. President Carter brought the idea from Georgia. ZBB killed by Reagan in
1981.

These reforms were UNSUCCESSFUL due to inconsistencies with political nature of


budget process and because the workload and paperwork requirements were
overwhelming.

THE GRACE COMMISSION (1982) – Reagan’s private sector survey of government


operations FINDS WIDESPREAD INEFFICIENCIES IN THE FEDERAL GOVERNMENT.

DEFICIT-BASED BUDGET REFORMS:

GRAMM-RUDMAN-HOLLINGS (GRH) 1985- Focused on reducing the deficit and


balancing the budget.

SEEKS TO BALANCE THE FEDERAL BUDGET BY MANDATING ACROSS-THE-BOARD


CUTS OVER A PERIOD OF YEARS.

Spurred by large federal deficit – ceiling raised for first time to $2 trillion

Focused on guarantee of specific outcome and SINGLE YEAR budget projections.

BUDGET ENFORCEMENT ACT (BEA) 1990- Amended the GRH Act to require that
new spending be balanced by new taxes or spending cuts.

3 Changes – 1) Annual budget target eliminated. 2) Limits on discretionary spending


thru 1995, and 3) PAYGO – new enforcement process.

RESULTS – GRH unsuccessful. BEA was able to constrain some actions of Congress
but did not reduce deficits due to unforeseen large increases in mandatory spending
(Medicaid & Medicare)

The CHIEF FINANCIAL OFFICERS ACT (1990) – Requires federal agencies to create
a CFO position to oversee agency finances.

GOVERNMENT PERFORMANCE RESULTS ACT – GPRA (1993) – Requires agencies


to justify their budget requests on the basis of the results or outcomes to be achieved.

496 | P a g e
The aim of GPRA is to INTEGRATE PLANNING, BUDGETING, MANAGEMENT AND
PERFORMANCE ASSESSMENT. It is intended to provide for the establishment of
strategic planning and performance measurement in the federal government
(Halachmi, 2002)

Nature of Budget Reform:

 Reforms resulted in MARGINAL, not wholesale changes


 Large scale changes (i.e. PPBS) only progress as political system allows.
 Budgeting highly conservative and decreasing in flexibility

For the rest of the BUDGETING REFORM TMELINE – see article: Kelly, Janet (2005) A century of public
budgeting reform: The key question

Remnants of Reforms:

 President has more managerial control due to moving BOB/OMB


 Budgets are now more focused on the FUTURE than before PPBS
 Modified forms of Zero-Based Budgeting common (agency budget <=90% of
prior yr.)
 Program budgeting emphasis on analysis of outcomes led to increased capacity
for policy analysis used in budget process.

Appleby (1957) – Classic Article – The Role of the Budget Division – modified the
“control” idea.

Control in this context did not mean guarding against misappropriation of public
funds (as it did in the Progressive movement); however as a check on spending.

Appleby argues that some sort of equilibrium is reached when no director is able to get
everything he wants.

It was in this era of optimism undergirded by confidence in rationality and science to


bring about optimal decisions that Lindblom’s 1959 essay appeared.

LINDBLOM (1959)

497 | P a g e
Argued that what KEY sought (a theory) and what LEWIS found (better facts) were
prescriptive, not descriptive.

“POLICY is made at the margins, but not through the application of marginal utility
theory. It is a system of successive limited comparisons among a finite number of
alternatives.

WILLOUGHBY (1918) – The Movement for Budgetary Reform in the States.

Member of TAFT Commission 1912 which called for a national executive budgeting
system. (Origin of movement for adopting a federal budget).

Influenced the BUDGETING AND ACCOUNTING ACT of 1921.

Budget Reform involves:

 How budgets affect popular control


 How budgets enhance legislative and executive cooperation
 How budgets ensure administrative and management efficiency
 Budget first demanded as a feature of MUNICIPAL REFORM.
 Permanent reform not accomplished by continually throwing out officials who
use public goods for private gain.
 Problem needs to be attacked from a TECHNICAL as well as MORAL standpoint.
 NML and Bureaus of Muni Research contribute improved methods of municipal
admin.
 Budgetary system based on assumption that municipalities have same
characteristics as BUSINESS Corps and should operate as such.
 In as early as 1899, NML had a draft municipal budget plan – Mayor submits
budget to Council.

KEY – (1940) The Lack of a Budgetary Theory.

Posed the key question in budgeting:

“On what basis shall we allocate X dollars to activity A instead of activity B?”

498 | P a g e
KEY & WILDAVSKY believe that budgeting equals a full-blown theory of government,
and is a supremely POLITICAL PROCESS.

Schick & Seligman (1926) – “Budgeting encompasses so much that it has become a
SOCIAL PROCESS.”

SCHICK – (1966) The Road to PPB: The Stages of Budget Reform.

Chronicled the development of budgetary theory from the era of accountability and
control (line-item budget) to performance budgeting with its emphasis on managerial
efficiency, to PPBS, which stressed objectives, planning, and program effectiveness.

SCHICK stated: “Efforts to budget on the basis of performance almost always fails”.

SCHICK – (1969) System Politics and Systems Budgeting

PROCESS AND SYSTEMS POLITICS:

Process Politics - The activity by which BARGAINS ARE STRUCK AND ALLOCATIONS
NEGOTIATED or rules of the game for the decision of budgetary matters.

Process Politics centered on pluralistic decision making, viewed to maximize a Pareto


model.

System Politics – Unlike Process politics, is concerned with the OUTCOME or


RESULTS, not the activity of deciding.

Systems Budgeting manifests itself in the form of PPBS budgeting systems which are
concerned with results and outcomes more than anything else.

Schick argues that Process Politics dominated the literature through the mid 1960s
because resources were plentiful. He argues that in an environment of scarce
resources, Process Politics will not work.

SYSTEMS BUDGETING – Schick believes that better outcomes can be achieved


through a systems approach.

RUBIN & SCHICK – Cast aside Incrementalism as a theoretical construct.

499 | P a g e
RUBIN & SCHICK believe that as an explanatory model, Incrementalism did not
describe the process well, noting that a budgetary base is not always defined.

Irene RUBIN – (1992) – Budgeting: Theory, Concepts, Methods and Issues.

 Budgeting does not currently have a theory


 Writers on budgeting do not agree on common assumptions and
recommendations
 Budget Theory today is fragmented and incomplete
 Budget theory is in the process of being invented
 In the U.S., state and local budgeting differs significantly from federal budgeting
(size, scope, variety of functions performed by the public sector, openness of the
budget process, etc.

THEORETICAL SCHOOLS - Each theoretical school makes its own assumptions:

NEO-MARXISTS:

 Argue that class interests dominate budgeting and allocation choices.


 Argue that govt is controlled by capitalists, or those who own the means of
production, and that they determine spending priorities to serve their own
needs.
 Military spending enriches arms manufacturers and tax breaks routinely go to
the well-off instead of the poor.

PUBLIC CHOICE THEORY:

 Assumes that human behavior is based on individual economic rationality and


the maximization of individual benefits (utility).
 Public choice theorists emphasize the relationship between what the citizen
taxpayers want govt to do and spend and what it actually does and spends.
The voting mechanism then provides a chance for citizens to choose among
candidates that best reflect their own spending priorities.

PROBLEM WITH PUBLIC CHOICE THEORY – Impossible to calculate a figure for


aggregate individual utilities.

500 | P a g e
INCREMENTALISM:

 Argues that budgeting occurs virtually exclusively inside govt


 Based on theory of BOUNDED RATIONALITY
 Govt is not directly controlled or controllable by society
 Interest groups exist, but do not determine outcomes
 Little public involvement in budgeting
 Weakness of Incrementalism – did not envision the budget process as
responding to societal problems, emergencies or environmental changes.

HIERARCHY THEORY:

Argues that top-levels of the executive branch make decisions about broad policy
issues, judge the environment, and pass that info down through the budget office to
the agencies before they make their requests.

RUBIN “First, budgeting theory strongly emphasized rationality and getting the most
from each dollar; then a second school grew up to refute the maximizers, arguing that
very little rational decision making went on; and the current literature argues for a
variety of positions in between.”

RUBIN’s Budgeting Concepts:

 Public budgeting is both political and technical, influenced by interest groups


and agency heads.
 Public budgeting is open to the environment, and is influenced by the economy,
public opinion, other levels of govt., interest groups, the press, and by
politicians.
 Budgeting is a decision making process.
 Budget processes vary, both between jurisdictions and over time.
 There is a relationship between the allocation of decision making power and
budgetary outcomes.
 Shifting locus of power in budgeting processes – Executive vs. Legislative, Top-
Down vs. Bottom-Up.
 The kind of information used in budgeting decisions is highly variable.

501 | P a g e
 Line-item budgets often provide no other information than a dollar figure.
 Program budgets – allow officials to clearly choose between priorities, but does
not provide a way of evaluating the relative efficiency or effectiveness of a
program.
 Accountability – Is government doing what tax payers want it to do?
 Fund Accounting – Provides budgetary accountability by creating specific pots
of money for specific purposes (not one big budgetary piggy bank).
 Balance – One of the most fundamental concepts in budgeting. Total revenues
must match total outlays.
 We have rejected the extremes of decision making models that postulate either
economic rationality or bounded rationality, but we have barely begun to map
out the real territory that lies between the extremes.
 Each of the major efforts at budget reform has given us new ideas and new
techniques, and we have incorporated them into our budget processes.

Irene RUBIN – (2000)

“Our confidence that changing the budget process will positively affect policy outcomes
is remarkable, especially because history shows that it often does not work or does not
work in the expected way. Elected officials seem compelled to reexamine the budget
process, especially in periods of distrust of government, rather than deal with the
factors that contribute to distrust or the consequences of it for public policy.”

KELLY (2005) – “We have a theory of budgeting, or at least of budget reforms, and it is
a theory of public opinion cycles”.

“When Americans feared that govt was corrupt, the budget process was reformed to
control costs and isolate executive responsibility. When Americans believed
management and science could advance the general welfare, budget process was
reformed to be more rational and scientific. When Americans thought that there was
much more important work for govt to do, the budget was reformed to be more
programmatic. When Americans lost faith that govt action could solve difficult social
problems, the budget process was reformed to incorporate elaborate justifications for
program expenditures. When Americans believed that govt had become an obstacle to

502 | P a g e
prosperity, the budget process was reformed to hold bureaucracy accountable for
outcomes.”

Changes in our orientation toward govt (either obstacle to the public good or agent of
the public good) fuels the desire for change in the budget process. The starting point
for budget reform is ALWAYS a constant – THE TRADITIONAL, LINE-ITEM, COST-
CONTROL BUDGET. We may dress it up in different ways (performance measures,
program evaluations, cba), but THE TRADITIONAL BUDGET IS OUR TOUCHSTONE
OVER TIME.

Nothing that followed the New York Bureau of Municipal Research’s LINE-ITEM
BUDGET was as important to the profession as IDENTIFICATION OF COST, and
nothing remains as useful in what we think of as accountability, either in the financial
or political sense.

WILDAVSKY (1964/84) – The Politics of the Budgeting Process

Was the best expression of INCREMENTALISM. A DESCRIPTIVE THEORY OF


BUDGETING.

BUDGETING IS POLITICAL – CONGRESSIONAL INFLUENCE IS VERY IMPORTANT.

BUDGETING REFORM IS POLITICAL SYSTEM REFORM.

A NORMATIVE THEORY OF BUDGETING IS POSSIBLE, BUT VERY DIFFICULT.

WILDAVSKY (1969) – Rescuing Policy Analysis from PPBS.

“NO ONE KNOWS HOW TO DO PPBS!”

PPBS’ planning and analytical functions are not compatible with budgeting’s basic
aims.

WILDAVSKY - Budgeting must provide continuity for planning, flexibility to respond


to crises, rigidity to control spending, and openness for accountability.

KEY & WILDAVSKY believe that budgeting equals a full-blown theory of government,
and is a supremely POLITICAL PROCESS.

503 | P a g e
LEVINE (1978) – Organizational Decline and Cutback Management.

Old era of “Positive Budgeting” is over. New era of “Negative Budgeting” and Cutback
Management will depend on degree of political uncertainty and the magnitude of
budget shortfalls.

CAIDEN (1981) – Public Budgeting amidst Uncertainty and Instability.

The budget environment is changing and is more dominated by fiscal stress and
political uncertainty.

WILDAVSKY & CAIDEN (2001) The New Politics of the Budgetary Process

Federal budgets today are unbalanced, uncertain, and dependent on circumstances


beyond their control.

A normative theory of budgeting would a comprehensive political theory detailing what


government activities ought to be at a particular time. It would eliminate the political
struggle which helps to develop the budget, thus ending politics.

LINDBLOM, CYERT & MARCH - BUDGET FORMULATION IN THE PRIVATE


SECTOR:

ADAPTIVE MODE:

Many decision makers with conflicting goals bargain amongst themselves to produce a
stream of incremental, disjointed decisions. Some of the literature describes this as
“entrepreneurial mode”.

504 | P a g e
4.3 The Big Names in Budgeting
Brownlow, Louis 1937 Committee Report to FDR

The president needs help

Caiden, Naomi 1981 Public Budgeting Amidst Uncertainty and


Instability

Uncertainty and stress must be taken into


account in
budgeting.

Cleveland, Frederick A. Evolution of the Budget Idea in the United States


1918 (pp. 7-23 in Government Budgeting by Hyde)

This article was the justification of the Taft


Commission which led to executive budgeting.

Forrester, John P. Public Choice Theory and Public Budgeting:


2001
Implications for the Greedy Bureaucrat
(pp. 101-116 in Evolving Theories of Budgeting,
John R. Bartle, ed.)

Public choice theory says self interest and the


free market,
explain why people (including bureaucrats) make
the decisions they do. Public budgeting is
different but public budgeting can learn from
public choice theory.

Gulick, Luther POSDCORB


1937
Planning, organizing, staffing, directing,
reporting, budgeting

Harding, Warren G. Signed The Budget and Accounting Act


1921
Created the Bureau of the Budget, GAO, and
executive budgeting

505 | P a g e
Focused on accountability and control

Line item budgeting

Key, V.O. 1940 The Lack of a Budgetary Theory

“On what basis shall it be decided to allocate x


dollars to activity A instead of Activity B?”

Budgeting is “applied economics”---allocating


scarce resources

The budget must reflect the public’s interest

Keynes, John Maynard The General Theory of Employment, Interest and


1936 Money

Keynesian Theory Fiscal policy (government spending & taxing


policies) can, and should be used to influence the
economy

Lewis,Verne 1952 Towards a Theory of Budgeting (see p. 29, Hyde)

He is a rationalist---but in the end concludes that


Key’s questions is unanswerable

He said “for x level of funding, y level of service


can be provided”

Doctrine of Public Utility --- marginal utility ;


diminishing return (i.e., battleships of poor relief)

The cost of a thing is the amount of other things


we must give up for its sake.

We must make choices---i.e., 4 or 5 tires for a


car, but not 6.

Lindblom, Charles 1959 The Science of Muddling Through

Osborne, David and Gaebler, Reinventing Government,


Ted 1992
How the Entrepreneurial Spirit is Transforming

506 | P a g e
the Public Sector

Led to Government Performance and Results Act


and National Performance Review---1993

Ostrum, Vincent 1974 The Intellectual Crisis in American Public


Administration

He supported public choice theories

Rubin, Irene 1990 Budget Theory and Budget Practice: How Good
the Fit? (see p. 77, Hyde)

No single theory explains budgeting---they are


fragmented and incomplete

Budgeting is complex

Neo-Marxist, public choice, incrementalism

Rational (to maximize return), non-rational, now


combination

Schick, Allen 1966 The Road to PPB: The Stages of Budget Reform
(see p. 52, Hyde)

A classic---it traced the evolutionary cycle of


budgeting reform from line item/strict
expenditure control to performance
budget/management – work efficiency

Taft, William Howard 1912 Taft Commission presented The Need for a
National Budget

Led to Budget and Accounting Act of 1921

Wildavsky, Aaron 1964/84 The Politics of the Budgeting Process

The best expression of Incrementalism---a


descriptive theory of budgeting

Budgeting is political --- Congressional influence

507 | P a g e
is very important

Budget reform is political system reform

A normative theory of budgeting is possible but


very difficult

Redistribution is behind all budget decisions

His later writings were less incrementalism-


oriented

1969 Rescuing Policy Analysis form PPBS

No one knows how to do PPBS

Willoughby, William F. 1918 The Movement for Budgetary Reform in the


States (see p. 20 in Hyde)

The public should be involved in the budget


process

Part of the Progressive movement in the early


1900’s

Theories/Reforms of Budgeting

Line Item (incremental)---1921

Performance---1940’s-50’s Hyde:

 Management oriented system heavily focused on efficiency by relating costs to


measured outputs
 This has seldom worked

Program Budgeting---1960’s

 focus on planning and goal setting


 Little evaluation

PPBS (Planning, Programming, Budgeting System)---1960’s


508 | P a g e
 Used in public sector budgeting
 Combined planning and budgeting
 Focus was on multi - year approach
 Robert McNamara brought it from Rand Corp to Dept of Defense under LBJ
 Used systems theory and cost benefit analysis
 Rational approach
 Best linkage of goals and budgeting --- MBO

ZBB---1977

 Zero based budgeting introduced at federal level by Jimmy Carter


 Abandoned in 1981 when Reagan was elected
 Continual program justification
 Budget requests are rank ordered

Outcomes budgeting ---1993

 Results matter---GPRA---MBO
 Outcomes/effectiveness is most important
 Performance based budgeting
 Treasury budgeting (tax expenditures and entitlements= 60% of federal budget
 Target based --- set limits
 This is not necessarily rational
 Planning and budgeting are merging
 Entrepreneurial budgeting---2000
 The Evolution of Public Budgeting
 See Irene Rubin’s articles

509 | P a g e
4.4 Budgeting Terms
Accrual Method of Measuring Cost (Accrual Accounting)

Records cost when the liability is incurred. As applied to retirement benefits, cost is
recorded when the benefits are earned rather than when they are paid or at some
other time.

Authorization

An authorization is an act of Congress that establishes or continues a federal program


or agency, and sets forth the guidelines to which it must adhere.

Balanced Budget

A balanced budget occurs when total receipts equal total outlays for a fiscal year.

Budget Authority (BA)

Budget authority is the authority provided by law to incur financial obligations that
will result in outlays.

Budget Enforcement Act (BEA) of 1990

The BEA is the law that was designed to limit discretionary spending while ensuring
that any new entitlement program or tax cuts did not increase deficits. It set annual
limits on total discretionary spending and created pay-as-you-go rules for changes in
entitlements and taxes.

Budget Resolution

The budget resolution is the annual framework that Congress uses to set targets for
total spending, total revenues, and the deficit, as well as allocations, within the
spending target, for discretionary and mandatory spending. A budget resolution does
not become law and is not binding on the Executive Branch.

Cap

A “cap’’ is a legal limit on annual discretionary spending. See Discretionary Spending.

510 | P a g e
Capital Planning and Investment Control

A decision-making process for ensuring that information technology (IT) investments


integrate strategic planning, budgeting, procurement, and the management of IT in
support of agency missions and business needs.

Clinger-Cohen Act

The Clinger-Cohen Act is also known as the Information Technology Management


Reform Act of 1996. The act supplements the Paperwork Reduction Act of 1980 by
establishing a comprehensive approach for executive agencies to improve the
acquisition and management of their information resources.

Competitive Sourcing

Competitive sourcing is a management initiative to make government more market-


based, allowing the public sector to embrace the principles of competition, innovation,
and choice. It determines the most effective method of obtaining services available in
the commercial marketplace. One commonly used process is found in OMB Circular
A–76 and may result in a public-private competition or the conversion of in-house
work to the private sector.

Deficit

A deficit is the amount by which outlays exceed receipts in a fiscal year.

Discretionary Spending

Discretionary spending is what the President and Congress decide to spend through
the 13 annual appropriations bills. Examples include money for such activities as the
FBI and the Coast Guard, housing and education, space exploration and highway
construction, and defense and foreign aid. See Mandatory Spending.

E-Government

E-Government refers to the federal government’s use of information technologies (such


as Wide Area Networks, the Internet, and mobile computing) to exchange information
and services with citizens, businesses, and other arms of government.
511 | P a g e
Enterprise Architecture

Enterprise architecture is an agency-wide framework for incorporating business


processes, information flows, applications, and infrastructure to support agency goals.

Entitlement

An entitlement program is one in which the federal government is legally obligated to


make payments or provide aid to any person who meets the legal criteria for eligibility.
Examples include Social Security, Medicare, Medicaid, and Food Stamps.

FAIR Act

The Federal Activities Inventory Reform (FAIR) Act of 1998 requires federal agencies to
submit annually to OMB an inventory of all activities performed by federal employees
that are not inherently governmental in nature (i.e., that can be performed by the
private sector). After OMB review, the agency must send a copy of the inventory to
Congress and also make it available to the public.

Federal Debt

Debt Held by the Public —The cumulative amount of money the federal government
has borrowed from the public and not repaid.

Debt Held by Government Accounts —The debt Treasury owes to other accounts
within the federal government. Most of it results from the surpluses of the Social
Security and other trust funds, which are required by law to be invested in federal
securities.

Debt Limit —The maximum amount of federal securities debt that may legally be
outstanding at any time. It includes both the debt held by the public and the debt held
by government accounts. When the debt limit is reached, the government cannot
borrow more money until the Congress has enacted a law to increase the limit.

Fiscal Year

512 | P a g e
The fiscal year is the federal government’s accounting period. It begins on October 1
and ends on September 30. For example, fiscal year 2003 begins on October 1, 2002
and ends on September 30, 2003.

Full-time Equivalents (FTEs)

Civilian employment in the Executive Branch is measured on the basis of full-time


equivalents. One FTE is equal to one work year or 2,080 non-overtime hours. For
example, one full-time employee counts as one FTE, and two half-time employees also
count as one FTE.

Gross Domestic Product (GDP)

GDP is the standard measure of the size of the economy. It is the total production of
goods and services within the United States.

Human Capital

Human capital refers to the education, knowledge, skills, and competencies of the
personnel of an agency.

Mandatory Spending

Mandatory spending is authorized by permanent law rather than annual


appropriations. An example is Social Security. The President and the Congress can
change the law to change the eligibility criteria and thus the level of spending on
mandatory programs, but they don’t have to take annual action to ensure the
continuation of spending. See Discretionary Spending .

Offsetting Collections and Offsetting Receipts

Offsetting collections and offsetting receipts are monies that are deducted from
outlays, rather than counted on the receipts side of the budget. They are often paid in
return for providing goods or services. For example, payments the Postal Service
receives for stamps are offsetting collections.

Off-Budget

513 | P a g e
By law certain programs, such as Social Security and the Postal Services, are
accounted for separately from all other programs in government and are accorded this
separate treatment.

On-Budget

Those programs not legally designated as off-budget.

Outlays

Outlays are the amount of money the government actually spends in a given fiscal
year.

Pay-As-You-Go

Created by the Budget Enforcement Act (BEA), pay-as-you-go refers to requirements


that new mandatory spending proposals or tax reductions must be offset by cuts in
other mandatory spending or by tax increases, to ensure that the deficit does not rise
or the surplus does not fall. See Budget Enforcement Act.

Performance-based Budgeting

Performance-based budgeting separates programs that work from those that do not. It
allocates budgetary and human capital resources by comparing historical and
expected future performance levels with the full cost of producing desired program
outcomes as defined in the agency’s strategic goals and objectives.

Receipts

Receipts are the collections of money that result from taxes and other government
activity. Examples of receipts include income taxes, excise taxes, and customs duties.
They do not include collections from the federal government’s business-like activities,
such as the entrance fees at national parks. Business-like collections are subtracted
from total spending to calculate outlays for the year.

Surplus

A surplus is the amount by which receipts exceed outlays in a fiscal year.

514 | P a g e
Trust Funds

Trust funds are federal government accounts set up by law to record receipts and
spend them for specified purposes.

Unified Budget

The unified budget includes receipts from all sources and outlays for all programs of
the federal government. It is the most comprehensive display of the government’s
finances.

Fiscal policy- Collectively, all federal government policies on taxes, spending, and
debt management: intended to promote the nation’s macroeconomic goals, particularly
with respect to employment, gross national product, price level stability, and
equilibrium in balance of payments. The budget process is a major vehicle for
determining and implementing federal fiscal policy. The other major vehicle for
determining and implementing federal fiscal policy. The other major component of
federal macroeconomic policy is monetary policy.

Capital Budget- A budget that deals with large expenditures for capital items
normally financed by borrowing. Usually, capital items have long-range returns and
useful life spans, are relatively expensive, and have physical presence (for example,
buildings, roads, and sewage systems).

Current Services Budget- An executive budget projection that alerts the Congress-
especially the Congressional Budget Office, the budget committees, and the
appropriation committees- to anticipate specific revenue, expenditure, and debt levels,
assuming that current policy is unchanged. It also provides a baseline of comparison
to the presidential budget.

Authorizing Legislation- Substantive legislation that sets up or continues the legal


operation of a program or a program or agency, either indefinitely or for a specific
period of time, or that sanctions a particular type of obligation or expenditure within a
program. Authorizing legislation is normally a prerequisite for an appropriation. It may
place a limit on the amount of budget authority to be included in appropriations acts
or it may authorize the appropriation of "such sums as may be necessary." In some
515 | P a g e
instances, authorizing legislation may provide authority to incur debts or to mandate
payment to particular persons or political subdivisions of the country.

Zero-Based Budgeting- An approach to public budgeting in which each year’s


activities are judged anew, with no reference to the policy precedents or dollar
amounts of past years.

Rescission- The consequence of executive and legislative action that cancels budget
authority previously provided by Congress before the time when the authority would
have otherwise have lapsed (that is, when appropriated funds would have ceased to be
available for obligation). The Congressional Budget and Impoundment Control Act of
1974 specifies that whenever the president determines that all or part of any budget
authority will not be needed to carry out the full objectives or scope of programs for
which the authority was provided, the president will propose to Congress that the
funds be rescinded. Likewise, if all or part of any budget authority limited to a fiscal
year- that is, annual appropriations, or budget authority of a multiyear appropriation
in the last year of availability- is to be reserved from obligation for the entire fiscal
year, a rescission will be proposed. Budget authority mat also be proposed for
rescission for reasons of fiscal policy or other reasons. Generally, an amount proposed
for rescission is withheld for up to 45 legislative days while the proposal is considered
by Congress. All funds for rescission, including those withheld, must be reported to
Congress in a special message. If both houses have not completed action on the
rescission proposed by the president within 45 calendar days of continuous session,
any funds withheld must be made available for obligation.

Crosswalk- any procedure for expressing the relationship between different


classification of budgetary data, such as between appropriation accounts and
government policy.

Cash Accounting System - (Cash basis of accounting)-The basis of accounting


whereby revenues are recorded when received and expenditures (outlays) are recorded
when paid, without regard to the accounting period in which the transactions
occurred.

516 | P a g e
Accrual Accounting System (Accrual Basis of Accounting)- The basis of accounting
under which revenues are recorded when goods are received and services performed,
even though receipt of the of the revenue or payment of the expenditure may take
place, in whole or part, in another accounting period.

Employment Act of 1946-federal legislation that called for economic planning and for
a budget policy directed toward achieving maximum national employment and
production.

Congressional Budget Office (CBO)- Federal office responsible for presenting the
Congress with reasonable and viable forecasts of aggregate levels of spending and
revenue. The office also makes cost estimates for proposed legislation reported to the
floor and provides cost projections for all existing legislation.

Impoundment- any action or inaction by an officer or employee of the U.S.


government that precludes the obligation or expenditure of budget authority provided
by Congress. It is also the power of the president to withhold (refuse to spend) funds
that have been appropriated by Congress.

Assets-Any item of economic value owned by a government unit. The item may be
tangible (that is, physical and actual) or intangible (that is, a right to ownership,
expressed in terms of cost or some other value).

Balance Sheet- an accounting statement designed to balance total assets, total


liabilities, and fund balance.

Appropriation- A legislative authorization that permits government agencies to incur


obligations and to make payments out of the treasury for specified purposes. An
appropriation usually follows enactment of authorizing legislation. An appropriation
act is the most common means of providing budget authority, but in some cases the
authorizing legislation itself provides the budget authority. Appropriations do not
represent cash actually set aside in the treasury for purposes specified in the
appropriations act; they represent limitations of amounts that agencies may obligate
during the period of time specified in the relevant appropriations act. Several types of
appropriations are not counted as budget authority, since they do not provide

517 | P a g e
authority to incur additional obligations. Examples of these include: (a) appropriations
to liquidate contract authority; Congressional action to provide funds to pay
obligations incurred against contract authority; (b) appropriations to reduce
outstanding debt- Congressional action to provide funds for debt retirement; and (c)
appropriations for refunds of receipts. There are 13 major appropriation bills passed
by Congress that make up much of the federal budget. These appropriation bills
correspond to the subcommittees of the appropriation committees in the House and
Senate.

Continuing Resolution- If a decision has not been reached on appropriations prior to


the beginning of the current year, then Congress can pass a resolution that says that
the government can continue to obligate and spend at last year’s budget levels or the
lowest level passed by a chamber of Congress. The wording is usually framed to permit
spending at the lowest amount the legislature is likely to pass.

Expenditures- Payment of an obligation.

Planning-Program-Based Budgeting- An attempt in the federal government and some


state and local governments to bring more analysis into the budgeting process. It is
not itself an analytical technique, but it stresses the use of analytical tools in deciding
budget issues related to specific government programs.

Budget and Accounting Act of 1921- Federal legislation that provided for an
executive budget for the national government and for an independent audit of
government accounts.

Contract Authority- Statutory authority that permits obligations to be incurred in


advance of appropriations or in anticipation of receipts to be credited to a revolving
fund or other account. Contract authority is unfunded and most subsequently be
funded by an appropriation to liquidate obligations incurred under the contract
authority, or by the collection and use of receipts.

Line-Item Budget- A budget format that presents the exact amount planned to be
spent for every separate good or service to be purchased.

518 | P a g e
Transfer Payments- Money moved from one government to another or to private
persons. They often serve as automatic stabilizers built into the economy. These
payments normally rise substantially during periods of recession and fall during
periods of prosperity. For example, the unemployed receives unemployment
compensation; in recessionary times they may eventually receive welfare and food
stamps as well.

Balanced Budget Act of 1985-the Gramm-Rudman-Hollings Act, one of three major


laws passed between 1974-1992to cure the budget crisis. The provisions of each law
were complex and ineffective. It was established deficit reduction targets for specific
fiscal year. The law left major loopholes such as funding the bail-out of the 1980s
federal budget failures has meant the collapse of budget regularity, the encouragement
of deceitful accounting practices, a premium on gimmicks that produce short-term
improvement but do not ease the structural problem, strained relationships between
the president and Congress, lessening the effective leadership of the president,
Congressional congestion and frustration, overloading the ability of the political
leaders to reach an effective solution, crowding out other important national concerns,
and insufficient resources to respond to emerging issues and priorities.

Budget and Accounting Act of 1921- provided for a national budget and an
independent audit of government accounts. The law specifically required the president
to submit a budget, including estimates of expenditures, appropriations, and receipts
for the ensuing fiscal year. The new legislation created the Bureau of the Budget (BOB)
in the treasury department. This strengthened the role of the president. The Bureau of
the Budget was later moved to the Executive Office of the President and ultimately
became the Office of Management and Budget in 1970.

Bureau of the Budget- The Bureau, when directed by the president, shall make a
detailed study of the departments and establishments for the purpose of enabling the
president to determine what changes (with a view of securing greater economy and
efficiency in the conduct of the public service) should be made in (1) the existing
organization, activities, and methods of business of such departments or
establishments, (2) the appropriations, (3) the assignment of certain activities to
particular services, or (4) the regrouping of services. The results of such study shall be

519 | P a g e
embodied in a report or reports to the president, who may transmit to Congress such
report or reports or any part thereof with his recommendation on the matter covered
thereby. The bureau creates the president’s budget. Agencies were required to submit
their estimates and supporting information to BOB and were allowed no contact with
Congress. Thus ensuring the presidents power.

Office of Management and Budget- (OMB) the major implementer of fiscal policy
once decisions are made by Congress and the president. An office of the president set
up in 1970 in succession to the bureau of the budget (founded in 1921) which is
responsible for preparing the executive’s budget for presentation to Congress in
January each year. After examination by the House and Senate committees, a
concurrent resolution on the budget is announced by April 15 to be followed by
legislation by May 15. Once the budget is passed, the OMB supervises and controls its
administration and provides data on program performance.

Budget Enforcement Act of 1990- part of three major laws between 1974-1992 to
cure the public budget crisis. This law sets targets within categories.

Progressive Tax-A tax whose rate rises as income or expenditures rises. The principle
examples of these taxes are on personal and corporate incomes. Progressive taxes aim
to achieve a more equal distribution of income post-tax than pre-tax.

Flat Rate Tax- An income tax which is at the same rate for every level of income. The
justification for a tax of this kind is its simplicity and lack of disincentive effects
inherent in some forms of tax progression. However, a flat rate tax is likely to be unfair
burden on low-income groups if the rate at which it is levied is high.

Monetarism- A modern revival of the quantity theory of money, making use of modern
neoclassical economics. It regards the money supply as the most important
determinant of aggregate money income and reasserts the relevance of price theory of
economics. In practice, most monetarists use the gradualist approach of aiming for a
rate of monetary expansion, which will achieve long-term price stability. This school is
associated with economist, Milton Friedman.

520 | P a g e
Sequestration- 1. Spending cuts in the federal budget imposed under Gramm-
Rudman-Hollings Law. 2. Temporary seizure of assets under a court order, e.g. of
union funds under British employment legislation.

John Maynard Keynes- (1883-1946) In three books, he groped towards a theory


which was to dominate western macroeconomics for over thirty years. A member of the
Liberal party, he connected together the theories of the consumption function,
aggregate demand, the multiplier, the marginal efficiency of capital, liquidity
preference and expectations.

Keynesian Economics- Keynesian policy is most popularly regarded as the use of


national budget deficits to maintain full employment. An emphasis on the importance
of the investment multiplier, an assertion that the liquidity preference schedule is
stable in the long run and unaffected by the actions of central banks and the
insistence on the major importance of fiscal policy so that money and the rate of
interest are of little importance to the management of the economy.

Laffer Curve- A graphical representation of the relationship between average tax rates
and total tax revenues which asserts that above certain average tax rate of tax revenue
will decline. The curve implies that, as there is a ceiling to the amount a government
can raise, there is a limit to the public goods, which can be provided. The theory
assumes that if the tax rate were zero, no taxes would be collected. Likewise, the
theory assumes that if the tax rate were 100 percent, no tax revenues would be
generated.

Federal Reserve System- Established in 1913, it is the system that carries out the
functions of a central bank for the United States. The original aims of the system were
to give the country an elastic currency, to provide facilities for discounting commercial
paper and to improve the supervision of banking.

Prime Rate of Interest- The rate of interest that us commercial banks charge
medium and small sized firms for borrowing. Historically, this was the interest rate the
most creditworthy customers of banks were charged but with the development of the
commercial paper market, the largest customers borrow below the prime rate.

521 | P a g e
Regressive Tax- One, which falls disproportionately on lower income, groups. As
income decreases, the average rate of tax increases. Many indirect taxes, e.g. excise
duties and sales taxes, are regarded as regressive but the extent to which they are
depends on the consumption patterns of different income groups. Poll taxes are the
simplest case of regression.

Backdoor spending- Backdoor authority as well as mandatory spending legislation.


That is, legislation that mandates the payment of benefits or entitlements, such as
increases in veteran's compensation or pensions. Such mandatory legislation requires
the subsequent enactment of appropriations.

Reconciliation Bill- (appropriation bill)- A legislation bill which authorizes


expenditure for a particular purpose, e.g. defense. It has to be passed by both the
House of Representative and the Senate. Annually all bills are expected to be
reconciled by a Reconciliation Bill by the end of June; if reconciliation is impossible,
then a continuing resolution is passed which permits Departments to continue at their
current expenditure levels.

Concurrent Resolution- A resolution introduced in both houses of a legislature.

Supply-Side Economics- A major U.S. school of economics which inspired the


economic policies of the U.S. under President Reagan and of the UK under Prime
Minister Thatcher. Opposing the Keynesian view that aggregated demand is central to
determining the level of economic activity, supply-siders place emphasis on aggregate
supply. Thus there has been a revival in the respectability of Say’s Law and a concern
for the disincentive effects of taxation. The Laffer curve has been a major innovation of
the school. The adherents of supply-side economics and monetarism often coincide.
The New Classical Economists have formalized many of their insights.

Demand-Side Economics - The opposite of supply-side economics. This theoretical


framework holds that "demand" creates its own supply and views consumers to be
(rather than producers) the central actors in an economy. Demand-side economics
includes the work of John M. Keynes and the monetarist theorists.

522 | P a g e
Phillips Curve- The relationship between unemployment and inflation so named by
Samuelson and Solow after Phillips attempted to identify it in 1958 by plotting data on
changes in money wage rates for the period 1861-1957 against the national
unemployment rates. Later work on Phillips Curve sought to take into account
incomes policies and inflationary expectations. The long run Phillips Curve is vertical
at the natural rate of unemployment. In contemporary economics, the validity of the
Phillips Curve has been questioned.

Performance versus Program Budgeting- Budget formats vary in their focus.


Program Budgeting stresses management and planning to help budget officials move
beyond the constraints of line-item and incrementalism toward a more rational and
flexible decision-making that focuses on program results. Performance Budgeting is
now required in most states. The major emphasis is on activities or programs. The
idea is to focus attention on how efficiently and effectively work is being done rather
than on what things are being required. Whereas line-item focuses on inputs,
performance budgeting focuses on outputs. In short, Performance Budgeting
measures work performance and how well agencies are achieving stated objectives.

523 | P a g e
4.5 Accrual Based Accounting vs. Cost Based
Accrual based accounting refers to the method used to report earnings and income
over the fiscal accounting reporting period. For many legal entities, such as C
corporations and governments, that year ends on September 30th of each year. For
the rest of us, those years ends as the physical year ends, on December 31st each
year.

What do we mean when we use the term “accrual” basis for accounting? Accrual-
based accounting means that sales are recorded when the sale occurs; it doesn’t
matter when the money is actually received. For many government agencies, the
accrual based accounting system has never been used, but thanks to fact that
governments and other public sector entities are using private sector type financial
statements, there may be a shift in accounting methods, even for the government.

There are basically two forms of accounting methods: cash and accrual. Cash based
systems are great if your income is below one million, and you don’t have problems
collecting for your products or services. Well, by that definition alone, many of today’s
businesses should not even consider the cash based accounting methods.

What benefit does accrual based accounting provide? Actually, quite a few if you
happen to be a mid-sized business and you need to keep an accurate picture of your
company’s profitability on a regular basis. A few of the additional benefits are: greater
focus on the business output, not the input; more cost-effective and efficient use of
resources; the full cost of providing your product or service can be compared across
industry standards; improved accountability and better financial management, just to
name the most common.

What benefit does the government hope to gain, since profitability isn’t an issue? This
method of accounting also provides the business with a better perspective of
management performance and results. What does that mean for government? It’s a
new viewpoint in comparing how our government measures up against private sector
businesses, and produce better management and efficiency from the public sector of
the business world.

524 | P a g e
The greatest advantage to accrual based accounting is that it allows an information
manager, or if you’re a small business, the business owner to accurately assess
whether or not the business is generating a profit. You can better assess profit levels
because you match income to expenses, accurately when you use the accrual based
method of accounting.

Compare this to the use of your credit card. Suppose you use your credit card to
purchase an item. You don’t actually have a deduction of your personal checking
account until you pay for the credit card debt; this is cash accounting methods. If you
use your credit card, and then right a check immediately to satisfy that debt on your
credit card, regardless of the due date of the bill, you’re using accrual based
accounting. Can you see how accrual forces accountability, and also makes you
aware of your spending limits?

For some businesses, there is a need to be able to view finances from both pictures.
Access to both methods provides information managers with a better view of actual
cash flows, and keeps CEOs and CFOs more aware of the actual state of the business.

525 | P a g e
4.6 Federal Defense Budgeting
Definitions

BCP - Budget change proposal

BEA - Budget Enforcement Act

BEA - The Bureau of Economic Analysis

CBO - Congressional Budget Office

DHHS - The Department of Health and Human Services

DHS - The Department of Homeland Security

DOD - The Department of Defense

EPA - The Environmental Protection Agency

GDP - Gross Domestic Product

NIPA - National Income and Product Accounts

OMB -The Office of Management and Budget

OSD - Office of the Secretary of Defense

PPBES - Planning, programming budgeting execution systems

PPBS - Planning, programming budgeting systems

USDA - The Department of Agriculture

Executive Summary

The Department of Defense is responsible for “coordinating national security threat


assessment, long- and intermediate-range planning and programming with annual
budget formulation and execution” (Jones and McCaffery, 2005). Defense budgeting
has been highly studied and remains controversial. The most effective and efficient
way to allocate funds and distribute them in a timely manor in an environment that

526 | P a g e
changes daily is challenging at best. To that end, this paper will address the following
question: Is it realistic to assign a theory to defense budgeting?

The method used to explore theories applied to defense budgeting and proposed
defense models is a literature review and comparison of the effectiveness of each
proposal. An effort was made to obtain literature both pre and post 9/11/01 to reflect
more recent changes in national defense structure. A similar attempt sought
literature from both domestic and international sources in an effort to approach the
analysis of this topic from an unbiased perspective.

V. O. Key challenged the world for a theory of budgeting in 1940. Has anyone or any
one theory risen to the challenge? Incrementalism has correctly, yet retrospectively
described defense budgeting for specific time periods. However; incrementalism itself
has been discredited as a legitimate theory. PPBS and PPBES are still running strong
in defense budgeting according to Jones and McCaffery in their 2005 publication.
Multi-year budgeting would greatly enhance the functionality of PPBES; however this
has yet to be realized. Budgetary trade-offs are a realistic sign of limited resources. If
resources were unlimited, then trading one service for another would not be
necessary. Although budgetary trade-offs in the defense department are realistic and
necessary, the idea of a trade-off does not constitute a budgetary theory.

September 11, 2001 did not impose any earth shattering changes on defense
budgeting theory, since it can be argued that there was not a theory in place to start
with. Joyce (2005) sums up this topic nicely by stating that since 2001 the budget
outlook has worsened, the process of defense budgeting has not changed, and
Congress still fails to approve a budgets in a timely manor. This almost unrestricted
access to funding in the defense department in the form of supplemental
appropriations has made a mockery of any attempt to create a valid theory of
budgeting that could accurately provide control yet accessibility to funds when
needed. Budgeting exists simply because resources are limited. Public preference
does hold a strong influence over with defense spending, however public preference is
reactionary and does not constitute a theory of defense budgeting. Nonlinear
dynamics is the closest to being able to explain defense budgeting; however it is
presented as a qualitative method of retrospectively analyzing defense budgets, not as

527 | P a g e
a predictor of future events or theory. Nonlinear dynamics is a paradigm, a way of
thinking or viewing a system, not a theory that can guide decision making. Heuristics
are valuable as a strategic guide in an unpredictable environment, but is not useful in
determining future funding allocations.

Theory building involves prediction, which is not always possible in the world of
defense. Theory development is solidified by validation and replication (Kiel and
Elliott, 1992). Proposed defense budgeting theories are weak at best, and do not stand
the test of time (replication). It is this authors’ conclusion that a functional budgeting
theory for the federal defense department has not been applied up to this point.
Further, with the current structure of the federal budgeting system, a defense
budgeting theory is not realistic.

Abstract

Is it realistic to assign a theory to defense budgeting? This question may surprise


many, who ‘assume’ formal structured budgeting is necessary for any agency or
department to function. While the author is not disagreeing with the necessity for
accountability, the question is whether the current accountability formats are
functional. This paper will explore past and current defense budgeting methods as
well as options, such as supplemental appropriations, that were designed to be a
safety-net in times of dire need and have more recently become routine as a way to
appropriate funds more quickly. Accurately forecasting defense budgeting is not
proving to be a realistic option. Heuristics may serve as an option for strategic
decision making.

Introduction

The Department of Defense is responsible for “coordinating national security threat


assessment, long- and intermediate-range planning and programming with annual
budget formulation and execution” (Jones and McCaffery, 2005). Defense budgeting
has been highly studied and remains controversial. The most effective and efficient
way to allocate funds and distribute them in a timely manor in an environment that
changes daily is challenging at best. To that end, this paper will address the following
question: Is it realistic to assign a theory to defense budgeting?
528 | P a g e
One of the major difficulties in studying governmental budgeting is deciding and
defining what is to be studied. For example, appropriations and outlays can be
studied on the same topic and the results may be entirely different. Appropriations
indicate budgeting authority and outlays describe what funds are actually spent.
Prior to the 1974 Congressional Budget and Impoundment Reform Act, appropriations
and outlay data was not reliably or consistently available (Wlezien and Soroka, 2003).
The office of Management and Budget has retained data on defense appropriations
and outlays since 1976. Brauer (2004) calls into question the appropriateness of
using national defense outlay data, claiming that it does not accurately reflect defense
expenditures. Brauer (2004) claims that the U.S. National Income and Product
Accounts is a better reflection of defense spending, but lacks full disclosure by not
including the defense departments portion of interest payments on the accumulated
federal debt. The delays and discrepancies between appropriations and outlays in the
defense budget calls into question the utility of the budgeting process to respond to
defense needs.

Method

The method used to explore theories applied to defense budgeting and proposed
defense models is a literature review and comparison of the effectiveness of each
proposal. Most of the literature is from the mid 1970’s to present, with the exception
of the 1964 and 1966 works of Wildavsky and Davis et al. An effort was made to
obtain literature both pre and post 9/11/01 to reflect more recent changes in national
defense structure. A similar attempt sought literature from both domestic and
international sources in an effort to approach the analysis of this topic from an
unbiased perspective. The major topics to be reviewed include incrementalism, PPBS,
PPBES, trade-offs, effects of 9/11/01, supplemental appropriations, public preference,
non-linear dynamics, heuristics and the need for a defense budgeting theory. Each
topic will draw from relevant literature to provide an account of defense budgeting
theories that have been utilized, remain in effect, and defense budgeting proposals as
well as an argument against defense budgeting theory as a whole. The conclusion of
this paper will offer a summary analysis of each major topic and include the authors

529 | P a g e
opinion as to what path defense budgeting should follow to render it functional and
effective.

Results

Theoretic and Historic Literature

The first challenge for a general budgeting theory was proposed in 1940 by V. O. Key.
Wildavsky and Davis et al. (1966, 1974) answered this dare with incrementalism as
the budgeting theory. Incrementalism itself has been discredited as being applicable
to budgeting by Rubin (1988) and Berry (1990). Jones and McCaffery (2005) also
support the claims of Rubin (1988) proclaiming incrementalism as an observation, not
a theory. Yet, Kanter (1972) reports that Congress rarely changed the President’s
defense requests by more than 2% during FY1960 – FY1970 and supports the idea
that incrementalism was at hand. Incrementalism has been documented as recently
as FY2006 in Schuler’s 2005 publication on Biodefense budgeting. Kiel and Elliott
(1992) reviewed defense outlays from FY1963 to FY 1990 and illustrated a lack of
coherence to the incremental budget theory, even up to the 10% level. This study calls
into question the timing of Kanter’s study, since it covered only ten fiscal years,
compared to the twenty-eight fiscal year records studied by Kiel and Elliott.

The Department of Health and Human Services (DHHS) and the Department of
Homeland Security (DHS) are the primary recipients of defense funds, but they are
also distributed to the Environmental Protection Agency (EPA) and the Department of
Agriculture (USDA). The incremental cost doctrine (part of the Budget Enforcement
Act of 1990) required the Department of Defense (DOD) to report “only the incremental
costs of carrying out contingency or emergency operations.” (McCaffery and Godek,
2003) This resulted in a lowering of supplemental appropriations requests, as small
requests were incorporated into other programs. If only incremental costs were
reported, then depending on the data source reviewed, it could very well appear that
incrementalism was at hand.

Overall defense efforts in the form of policy choices led to citizen dissatisfaction of the
defense plans during the 1960s. According to Kanter (1972), Congressional influence
over defense policy manifested in two ways:
530 | P a g e
“(1) Congress simply could make nontrivial dollar changes in the President’s budget
request, and (2) these budget changes might be attributable to Congressional policy
concerns and program preferences rather than to other motives, especially a desire
simply to reduce the level of defense spending.”

These methods address both the fiscal impact and Congressional motives for defense
budgeting. Congressional power can implement or prevent defense policies, directly
affecting budgetary needs. Congress’s actions on defense budgeting in the 1960’s
indicate a concern for fiscal responsibility as well as programmatic budgeting (Kanter,
1972).

Carter (1989) identifies local economic effects as being a significant indicator for
defense budgeting during the Reagan administration. While Kanter argues that
defense policy shapes defense budgeting, Carter argues that defense budgeting shapes
defense policy, and Domke et al., (1983) take yet another approach that the budgetary
process does not facilitate priority setting.

The political impacts on defense budgeting are especially apparent in the 1980’s
through the 1990’s. After the Vietnam War, defense spending fell substantially. The
Regan administration increased defense spending with the help of a newly Republican-
controlled Senate. Defense spending increased from just over $50 billion in 1980 to
over $100 billion in 1987 (in constant 1995 dollars) (Goyal et al., 2002). Federal
budget reforms in the late 1980’s held defense spending to around $100 billion. By
1996, the figure had dropped to under $50 billion, with the fall of the Berlin wall in
1989 and changing presidential party control to the Democrats.

PPBS and PPBES

The Department of Defense was responsible for allocating almost half of the
discretionary portion of the U.S. federal budget in FY 2005 (Jones and McCaffery,
2005). That totals is expected to be just over $2.5 trillion for the 5 year period of FY
2004 – 2009. Planning, programming, budgeting systems (PPBS) has not been studied
specifically how it relates in the defense department, according to Jones and
McCaffery (2005), because it is too complex and it is not comparable or relevant to the
systems already established in other departments. Schick (1988), states that all
531 | P a g e
budgeting processes involve planning, management and control processes, but in
different ratios according to department-specific needs.

In 1961, Robert McNamara was appointed Secretary of Defense by President John F.


Kennedy. McNamara applied PPBS tactics to this empire of 4.5 million employees for
the first time and was successful in increasing civilian control in the budgeting and
management process. Changing control from the military to the civilian side involved
equal parts control, politics and PPBS. By 1964, PPBS was the theory running the
Department of Defense. Decisions were divided into assessment, actions and finance.
Secretary of Defense Donald Rumsfeld regained some of the power and control back to
the Office of the Secretary of Defense (OSD), while maintaining support for PPBS.
During the Cold War, it could be argued that the defense budget directed the program,
when PPBS was intended to do the opposite. The Office of Management and Budget
held influence over the Presidents recommendations and the congressional approval,
proving once again that budgeting can easily turn into politics. Jones and McCaffery
(2005), best describe the political conflicts involving the Department of Defense (DOD)
as follows:

“In DOD, this competition takes place between civilian and military assessments of
resource requirements, between planners and budgeters, between OSD and the
military departments / services, between joint war-fighter combatant commanders
and the military departments and service responsible for the hiring, training and
provisioning forces, and between other agents in a variety of contexts.”

PPBS forced the DOD to build its budget first by performing a strategic assessment of
threat. The planning stage focused on how to provide services / commodities /
manpower to address and deter perceived threats. Programming required “building a
force structure that will provide the capabilities to do so” Jones and McCaffery (2005).
Jones and McCaffery (2005) go on to say that “the key budgeting tasks in PPBS in
DOD are quite limited, seeking only to develop correct pricing and timing decisions
about force structure requirements established in the strategic planning process.”
One item to note is that strategic planning for the Department of Defense is on a
multi-year timeline. This has implications when conforming to the annual budgeting
cycle of the federal government.

532 | P a g e
The DOD had chosen to minimize management control (the process where managers
assure the efficient and effective use of resources) in lieu of planning. DOD
management never wanted to be as unprepared as it was on December 7, 1941. They
were successful until September 11, 2001. It is no surprise then that in both 2001
and 2004 the federal Comptroller issued the DOD grades of ‘A’ for planning but failing
grades for management control and fiscal management. Without management control,
a structured budgetary system beyond line-item appropriation was impossible. This
situation sets the stage for out of control supplemental appropriations, which will be
discussed in a later section.

While the Department of Defense was not happy with the results of PPBS as it relates
to budgeting, no other system appeared to offer solutions to its shortcomings. Jones
and McCaffery (2005) offer some criticisms of PPBS as it relates to the DOD including,
“PPBS in DOD failed to move the focus away form budget justification and toward
management, control and better analysis…”, “…it continued to subsist as a disjointed
long-range resource planning mechanism.”, “This has resulted in huge wastes of
taxpayers’ money and civilian and military time and energy, …”, “…inability to supply
war-fighter needs on time, and not having enough money to afford implementation of a
badly needed weapons platform…”. The Department Of Defense can best be described
as a huge, overly bureaucratic organization lacking coordination, management,
control, and worst of all, a management control system that could address these
challenges.

The Planning, Programming, Budgeting System (PPBS) was renamed in 2003 to


PPBES, or Planning, Programming, Budgeting Execution System. The purpose of
PPBES was to stress execution and replace an annual budget cycle, two-year
programming cycle, and six-year planning cycle with a singular four-year resource
planning and decision cycle. The ‘off-cycle years’ allowed time for evaluation and
execution analysis of he budget. One of the issues with PPBS was the lack of
adequate time for analysis of existing budgeting levels. Although PPBES did
incorporate time for evaluation, the effects of program based budgeting may
sometimes not be seen for many years. The main intent for PPBES was to transfer
power, allowing the program to drive the budget, instead of the other way around,

533 | P a g e
which is what resulted from PPBS. Unless a budget change proposal (BCP) was
received and approved, budgets remain the same as the previous year for the four year
cycle. Jones and McCaffery (2005) contend that the DOD would be better able to
respond than most if not all other departments of the federal government, if Congress
were to recognize, accept and implement multiple year budgeting.

Defense Budgeting and Trade-offs

Domke et al. (1983) identify the following three categories of trade-offs as they relate to
defense budgeting:

Long-term trade-offs: trends in the defense share of national resources (or budgets)
that are negatively related to trends in welfare spending;

Short term trade-offs: yearly changes in which defense expenditures are negatively
related to changes in welfare spending;

Discrete decision-point trade-offs: explicit choice of central decision makers to spend


more for defense at the expense of social programs, or vice versa.

Domke et al. (1983) and Mintz and Stevenson (1996) addressed the guns versus butter
trade-off debate, exploring the theory that increasing spending in one area requires a
decrease in spending in another. Domke et al. (1983) did not find any indication of
welfare spending acting as a trade off for defense spending in four countries with the
exception of “wartime and postwar reconstruction.” This would indicate that long-
term trade-offs affecting welfare spending only exist in special circumstances. The act
of a direct budgetary trade off of one program against another was found to be “rare or
nonexistent in democratic governments”, according to Domke et al. (1983).

This does not discount Mintz and Stevenson’s (1996) suggestion of an indirect
relationship between defense spending and the economic resources of civil society as
follows; “Budgetary decisions have economic consequences; economic performance
has an effect on government allocation decisions, therefore an “indirect link” may exist
between guns and butter.” The idea of an indirect link is difficult to disprove, since
resources are limited, any diversion to a defense focus will limit the availability of
those resources to society as a whole in the long run.
534 | P a g e
Budgetary trade-offs do exist, whether direct or indirect, and the defense of our nation
is held in the eyes of the public as a priority responsibility of the government. There
are situations where the needs of national defense can create bottlenecks in the
production of public goods, since any resource in short supply that is diverted to
defense will be temporarily unavailable for the private sector or lower priority public
programs. Mintz and Stevenson (1996) contend that dollars are more effective when
spent on civilians in this modern welfare state.

The relationship between defense expenditure and economic growth is even less clear.
According to Goldsmith (2003), other states spend proportionately more on defense
than democracies do. Military expenditure is used to measure the defense effort, or
military effort of a nation. Goldsmith (2003) found the average defense burden to be
2.75% of the Gross Domestic Product (GDP) of all studied states. North and South
America were found to spend significantly less of their nations GDP on defense when
compared to European states. Not surprisingly, the defense burden was found to be
an integral part of both the domestic political economy and the international
environment. Reports have been published supporting both a direct and indirect
relationship between defense spending and GDP spending. Gerace (2002) did not find
any evidence showing GDP spending running counter-cyclic to U.S. military defense
spending; however non-military expenditures did show this counter-cyclic relationship
to GDP growth rate. Of course, the Keynesian argument states that “military
expenditures are used as a fiscal policy tool and have a positive impact on economic
growth” (Gerace, 2002). The counter argument to the Keynesian position is that
military expenditures take human resources and capital away from emerging industry,
placing domestic advancement on hold during times of conflict.

Defense Budgeting post September 11, 2001

Prior to the terrorist attacks in 2001, large surpluses existed in the federal budget as a
result of the Clinton administration. No model or theory of defense budgeting existed
then or exists now to predict the financial outcome of such an attack. Large deficits
now exist where surpluses were expected to be. This illustrates the inability of any
budgeting theory to accurately forecast for defense budgeting. As reported by Joyce,

535 | P a g e
(2005) competition for funding between national defense and domestic government, in
the form of trade-offs, has existed since World War II.

Joyce defends three points in the 2005 analysis of post-9/11 defense budgeting. First,
since 2001 the budget outlook has worsened. Second, the process of defense
budgeting has not changed. Third, in light of the failure of Congress to approve a
budget in a timely manor has placed the process at a critical juncture and may result
in budgeting at the federal level being rendered irrelevant. Since 2001, Congress is
more likely not to challenge Presidential defense budgeting proposals. Financial
forecasts have been made by the Office of Management and Budget (OMB) and
Congressional Budget Office (CBO) indicating a deficit of close to two trillion dollars by
2007, four trillion less than their prediction made in January of 2001 (Joyce, 2005).
Even in January of 2001, 9 months prior to the terrorist attacks, the deficit prediction
was off by four trillion dollars. This is another illustration of the lack of accurate
prediction abilities with budgeting in general, and specifically defense budgeting. This
is a worst-case scenario example of how the environment can affect the best laid
plans.

Joyce (2005) explains the difference in defense budgeting pre and post 9/11 in the
following way:

“After 1998 (and continuing until approximately 9 AM eastern time on September 11,
2001), this target was replaced by a consensus that the budget should be balanced
excluding the surpluses in the Social Security funds. Now, however, there is no
consensus, and the lack of agreement means that the budget process is operating
without a notional budget constraint.”

Joyce’s statement reflects both the governments and societies priorities when it comes
to defense spending in an era of global terrorism. This statement also brings up the
point of budget constraints. If limits on overall spending are enforced, then trade-offs
will increase as unpredictable or unforeseen events continue to occur and require
funding. The other option available for unplanned funding needs is supplemental
appropriations, which will be discussed next.

Getting by on Supplementals
536 | P a g e
The purpose of a supplemental appropriation is to provide financial relief in
unforeseen emergency circumstances. These situations can be a natural or human-
caused disaster. Defense budgeting has survived on supplemental appropriations,
limping by when needs present themselves. Funds may be transferred or
reprogrammed, but some circumstances can require such large financial support, the
only an additional appropriation can meet this need. Supplemental appropriations
helped to finance some of the 50 major disaster missions of the U.S. navy from 1990
to 1997 (McCaffery and Godek, 2003). This is not the way supplementals were
originally intended to be utilized.

The 1974 Congressional Budget and Impoundment Reform Act curbed the use of
supplemental appropriations as a way to fund routine expenses. The Congressional
Budget and Impoundment Reform Act required that foreseen spending must be
requested in the president’s budget, not simply requested as a supplemental
appropriation after the fact. Of course, the critical flaw with that statement is
‘foreseen spending’. The fact that terrorism and national defense spending is unable
to be accurately predicted during a fiscal budgeting cycle is an easy argument to
make.

The 1990 Budget Enforcement Act went a step further by setting spending caps. With
this new rule, supplemental appropriations could only be approved with a matching /
offsetting spending reduction or revenue increase (McCaffery and Godek, 2003). The
exception was for ‘dire emergencies’ when appropriations would be made and applied
to the deficit. The ‘dire emergency’ clause was instituted every year from 1990 to
1999, when the overall defense supplemental expenditures netted $86 billion dollars
over recessions (McCaffery and Godek, 2003). Supplemental appropriations tend to be
approved more quickly (average 4 months) than normal appropriations bills (average
10 + months). That makes supplemental appropriations a much more attractive route
to take when funding is needed.

The 1990 Budget Enforcement Act affected defense budgeting by requiring pay-as-you-
go budgeting techniques. This theory of take-from-one to pay-for-another also allowed
the use of supplemental appropriations with the ‘dire emergency’ clause, so nothing
really changed for the defense department. During this time, efforts were made to cut

537 | P a g e
spending in other areas, but if funds were needed for defense, supplemental
appropriations remained available. This is not to be construed as an open wallet for
the Department of Defense. Congress can, and has employed techniques such as
funding half of the requested amount, asking DOD to assist with their own financing.

Not all actions in budgeting are ethical. McCaffery and Godek (2003) report that in
1992 $314 million up to $2.1 billion in 1997 of regular appropriations was classified
as a ‘dire emergency’ for the purpose of evading spending cap restrictions. In the
1970’s, prior to spending caps, Congress and the President utilized supplemental
appropriations to fund mandated programs. The change to ‘dire emergency’ spending
only was an effort to gain more control over federal budgeting. Due to the nature of
supplemental appropriations, Congress and the President usually are relying on rough
estimates of what will be required to help or fix a specific issue. This figure may be
enough, but often fails to incorporate long-term effects of disaster and defense
problems.

Public Preference

Eichenberg and Stoll (2003) argue that citizen desires or public preference has a
strong influence over defense spending decisions. They claim that public opinion is
the “most consistently significant influence” on changes in defense spending. This can
be seen in election cycles such as the current Presidential campaigns. Defense efforts
and funding are a top rated topic for determining political candidate preference. The
candidates’ positions on defense funding have been used both as a positive for that
candidate and a negative for their rivals.

Eichenberg and Stoll (2003) found that in the United States and Europe, public
opinion did not favor defense spending that increased more rapidly than non-defense
program spending. Public opinion reacts to governmental policies and the environment
in a rational and systematic way, unlike the erratic reactions speculated by early
scholars (Eichenberg and Stoll, 2003). The problem with the generalized statement of
the public reacting in “rational and systematic” ways is that the root cause of the
reaction is not predictable. After the terrorist attacks of 9/11/01, the public was in
favor of increasing defense spending. This is a rational response to achieve the goal of

538 | P a g e
being proactively involved in attempting to prevent future attacks. The attack itself
was not planned for and funding was not readily available to deal with that situation,
although the public was willing to forgo deficit control and immediately respond to this
threat.

Wlezien (1995) offered the thermostat metaphor, stating that if the public’s threshold
was crossed (high or low), citizen demands will reflect the opposite for several years.
The thermostat analogy was supported by the findings of Eichenberg and Stoll (2003),
who note that public opinion reacted in a significantly negative manor to unpopular
changes in defense spending, with the exception of France. Goldsmith (2003) agrees
with the impact of public preference, but only in peacetime. According to Goldsmith
(2003), international factors as well as public preference have a large influence on the
defense budget in times of war. As mentioned earlier, Carter (1989) identified local
economic effects as being a significant indicator for defense budgeting during the
Reagan administration, supporting the public preference viewpoint.

Nonlinear Dynamics

Kiel and Elliott (1992) akin budgeting to nonlinear dynamics in the natural sciences.
This is an opportunity to view a social process (defense budgeting) as an open system,
strongly affected by the environment, public preference and politics. The application
of nonlinear dynamics is not new to the social sciences. Studies have successfully
applied its principles to evaluate economic equilibrium cycles, business cycles,
competitive behavior among firms, the arms race, patterns of urban growth and
noncompliant social behavior (Kiel and Elliott, 1992).

Nonlinear systems are identified by a constantly changing relationship between the


variables, such as the constant changes seen in the politics of defense. There are four
temporal classifications of nonlinear systems. Stable systems are grounded to a fixed
point and can be temporarily predicted by a mathematical equilibrium. Oscillating
systems exhibit a smooth patterned cycle, although the amplitude is unpredictable
and will vary. Unstable systems are characterized by un-patterned behavior. Finally,
random systems are completely devoid of patterns and can only be described as
chaotic. Changes in nonlinear systems produce exponential effects on the structural

539 | P a g e
configuration of the system. An example of an exponential effect in defense budgeting
would be the effect of a single terrorist attack (9/11/01) on future defense budgeting.
Based on the definition and properties that comprise nonlinear systems, long-term
predictions of nonlinear systems are highly suspect (Kiel and Elliott, 1992).

Kiel and Elliott (1992) mention Wildavsky’s 1988 recant of his earlier incrementalist
position where he claims that modern budgeting appears virtually random in nature.
The fact that budgeting is an open system (Rubin, 1990), and susceptible to external
disturbances immediately rules out the option for budgeting to be a linear system.
National defense and natural disaster preparedness are perhaps the two situations
face by the federal government subject to the most “external disturbances”. Kiel and
Elliott’s (1992) analysis of defense budgeting, total mandatory and total discretionary
domestic outlays illustrates considerable instability in the defense budget, compared
to mandatory and discretionary outlays.

Heuristics

A rule of thumb or heuristics approach to the defense budget would serve as a guide
in times of a changing environment. Kiel and Elliott (1992) analyze defense budgeting
using non-linear dynamics, and use this concept to defend budgeting as a dynamic,
ever changing system that does not fit within a neat theory framework. Defense
budgeting does not occur by inserting numbers into a formula and approving whatever
an equation spits out. The reality is closer to Carter’s 1989 statement that “Presidents
shape their defense requests with an eye to what they think Congress will approve.
Members of the authorizations or appropriations committees adjust presidential
requests to reflect congressional concerns as much as possible bearing in mind the
possibility of a veto”. This statement sums up the politics of defense budgeting, but
could be extended to budgeting in general. This rule of thumb approach exhibits the
true reality of defense budgeting and policy.

How can a true defense budgeting theory exist in an environment of uncertainty and
politics? Carter (1989) also found that U.S. senators “more consciously calculate the
economic consequences of their votes on non-procurement decisions than they do on
procurement matters.” The implication is that non-procurement matters are more

540 | P a g e
open to the public and will reflect back on the senator more than procurement
decisions. This also supports Ripley (1969), who proposed the “salience” hypothesis,
which states that party leaders are more successful on issues that are not in the
public spotlight.

Dolan (2002) assessed how accurately top level bureaucrats represent the viewpoints
of the public. This was analyzed using a survey given to both groups asking their
opinion on funding levels of various federal departments and programs, including
defense. The assumption is that non-elected public administrators are cut from the
general population; therefore an unbiased viewpoint should reflect that of the
citizenry. The concern is whether or not public administrators are more supportive of
funding programs for which they are employed in an effort to preserve their own self-
interests. In fact, the overall findings were that public administrators favored a 6.9%
increase in defense spending, compared to a 17.5% increase requested by the general
population. When looking only at defense department public administrators, a 15.1%
increase was favored, compared to the 17.5% of the general public. This shows that
employees of the defense department favor funding their own interests more so than
other public administrators, but still less than the general population. Dolan (2002)
found that bureaucrats overall were more likely to advocate decreased government
spending than the general public. This may be attributed to the bureaucrats
heightened awareness of budgeting constraints, decreasing discretionary spending and
the publics concern over the budget deficit, realizing that the only way to control the
deficit (without increasing taxes) is to control spending. The generation of heuristics
or multiple rules of thumb may be more successful than efforts have been to develop a
budgeting theory.

The Argument against a Budgeting Theory

Schick (1988) suggests and Kiel and Elliott (1992) agree that a theory of budgeting is
not realistic since most budgets, such as the defense budget, are dynamic and
changing systems. Environmental influence is unpredictable in the defense world and
can only be studied retrospectively, providing models to fit a specified time period, but
unsuitable for forecasting future needs. As stated by Kiel and Elliott (1992), “The
dynamics in the relationships between variables over time in nonlinear systems can

541 | P a g e
generate complexities that defy generalization, which underscores the challenge of
building theories relevant to complex social phenomena, such as government
budgeting.”

The relationship between appropriations and outlays holds a lot of variability (Wlezien
and Soroka, 2003). Specifically, between 1976 and 2002 the defense department
lagged as much as 40 billion dollars, taking up to two years for outlays to catch-up to
appropriations. Fluctuations in appropriations were found to mimic public preference
(Wlezien and Soroka, 2003), however the delayed outlays may not be sufficient to
address immediate needs.

Brauer (2004) disagrees with the notion that defense budgeting has a viable theory.
Brauer (2004) argues that all defense budgets and studies are being conducted with
flawed data. Wlezien and Soroka (2003) identified the problems with differences
between appropriations and outlays. Brauer (2004) acknowledges that fact, and goes
a step further proclaiming that all sources of ‘reliable’ data are incomplete and
misleading. Line item budgeting comparisons exclude many relevant categories, such
as $571 million allocated to biodefense in FY2004, categorized under ‘Health’. Brauer
(2004) found that comparing federal budget-based national defense outlay data with
the U.S. National Income and Product Accounts resulted in a difference of nearly $100
billion dollars for 2003. Even the U.S National Income and Product Accounts data
lacks full disclosure by not including the defense departments’ portion of interest
payments on the accumulated federal debt.

Palmore and Melese (2001) view national defense through game theory. This is an
accurate viewpoint in many circumstances, such as the Cold War, the Iraq War and
even terrorism in general to a certain extent. The problem with game theory, as it
relates to defense budgeting, is its conditional response to situations. Given a goal of
minimizing risk to the nation (possibly by maximizing deterrence) with the lowest cost
to taxpayers, how does one project a budget for such a theory? The U.S. position will
depend on the actions of others (which, in game theory, will depend on U.S. action).

Palmore and Melese (2001) provide an example of the potential for a missile attack
during the Cold War. Even with the limited choices of “Hi” and “Low” for both the U.S.

542 | P a g e
and USSR, which provides a matrix of four budgeting alternatives on that one topic
alone. The current U.S. budgeting cycle begins over a year prior to actual funding
allocation and outlays, rendering the system inflexible to rapidly changing needs,
which would be needed in game theory. Game theory may be a realistic and viable
defense strategy in a more flexible system, but it is not conducive to accountability
and forecasting in defense budgeting.

Conclusion

V. O. Key challenged the world for a theory of budgeting in 1940. Has anyone or any
one theory risen to the challenge? Incrementalism has correctly, yet retrospectively
described defense budgeting for specific time periods. However; incrementalism itself
has been discredited as a legitimate theory.

PPBS and PPBES are still running strong in defense budgeting according to Jones and
McCaffery in their 2005 publication. Multi-year budgeting would greatly enhance the
functionality of PPBES; however this has yet to be realized. It is interesting to note
that PPBS was declared “ended” (Pilegge, 1992), yet according to Jones and McCaffery
(2005), the highly related PPBES remains the current operating theory for defense
budgeting. PPBES is not proving to be a very effective option, given the quantity of
literature devoted to defense budgeting trade-offs and supplemental appropriations.

Budgetary trade-offs are a realistic sign of limited resources. If resources were


unlimited, then trading one service for another would not be necessary. Although
budgetary trade-offs in the defense department are realistic and necessary, the idea of
a trade-off does not constitute a budgetary theory.

September 11, 2001 did not impose any earth shattering changes on defense
budgeting theory, since it can be argued that there was not a theory in place to start
with. Even if PPBS can be considered the leading theory at the time, it did not change
to PPBES until 2003. The leading change with PPBES was to stress multi-year
budgets, which is not in effect today. Joyce (2005) sums up this topic nicely by
stating that since 2001 the budget outlook has worsened, the process of defense
budgeting has not changed, and Congress still fails to approve a budgets in a timely
manor
543 | P a g e
The original purpose of budgeting supplementals was to provide financial support
during times of unforeseen financial circumstances. The process of acquiring funding
by labeling it a “dire emergency” proved to be easier and faster than acquiring the
same funding through routine channels. This almost unrestricted access to funding
in the defense department has made a mockery of any attempt to create a valid theory
of budgeting that could accurately provide control yet accessibility to funds when
needed.

Budgeting exists simply because resources are limited. Society’s priorities determine
(to a certain extent) where financial support is assigned. While funding limits may
drive defense spending, budgetary processes do not facilitate priority setting (Domke et
al., 1983). There are multiple issues with this plan. First, who defines “society”? In
our representative democracy, elected officials are expected to speak for the majority
in their jurisdiction. The reality is that political pressures and personal beliefs tend to
taint that perfect view of decision making. The second issue is that there does not yet
exist a theory that can determine what the priorities of a society are as a collective
whole. Public preference does hold a strong influence over with defense spending,
however public preference is reactionary and does not constitute a theory of defense
budgeting.

Nonlinear dynamics is the closest to being able to explain defense budgeting; however
it is presented as a qualitative method of retrospectively analyzing defense budgets,
not as a predictor of future events or theory. Nonlinear dynamics is a paradigm, a way
of thinking or viewing a system, not a theory that can guide decision making. A key
element argued by Kiel and Elliott (1992), is that the fundamental element of
traditional theory building involves prediction, an unobtainable goal when one is
dealing with a nonlinear system involving changing relationships of the players
involved.

A rule of thumb or heuristics approach to defense budgeting is a practical viewpoint


that fits most situations. The problem with heuristics is that it is more of an action
decision-making guide rather than a budgeting theory. Heuristics are valuable as a
strategic guide in an unpredictable environment, but is not useful in determining
future funding allocations.

544 | P a g e
Theory building involves prediction, which is not always possible in the world of
defense. Theory development is solidified by validation and replication (Kiel and
Elliott, 1992). Proposed defense budgeting theories are weak at best, and do not stand
the test of time (replication). Each individual study of defense budgeting theory is
restricted to a specific time period and set of conditions. Overall, the limitations of
such a group of studies outweigh any practical significance. It is this authors’
conclusion that a functional budgeting theory for the federal defense department has
not been applied up to this point. Further, with the current structure of the federal
budgeting system, a defense budgeting theory is not realistic

References

Berry, W.D. (1990). The Confusing Meaning of Budgetary Incrementalism: Too Many
Meanings for a Single Concept. Journal of Politics, 52:167-196.

Brauer, J. (2004). United States Military Expenditure. Presented at the Conference on


Economics and Security. Technical Institute of Larissa, Greece. June:1-30.

Carter, R.G. (1989). Senate Defense Budgeting, 1981-1988: The Impacts of Ideology,
Party, and Constituency Benefit on the Decision to Support the President. American
Politics Quarterly, 17(3):332-347.

Davis, O.A., Dempster, M.A.H., & Wildavsky, A. (1966). A Theory of the Budgetary
Process. American Political Science Review, 60:529-547.

Davis, O.A., Dempster, M.A.H., & Wildavsky, A. (1974). Towards a Predictive Theory of
Government Expenditures: U.S. Domestic Appropriations. British Journal of Political
Science, 4:419-452.

Dolan, J. (2002). The Budget-Minimizing Bureaucrat? Emperical Evidence from the


Senior Executive Service. Public Administration Review, 62(1):42-50.

Domke, W.K., Eichenberg, R.C., & Kelleher, C.M. (1983). The Illusion of Choice:
Defense and Welfare in Advanced Industrial Democracies, 1948-1978. The American
Politicsl Acience Review, 77(1):19-35.
545 | P a g e
Eichenberg, R.C., & Stoll, R. (2003). Representing Defense: Democratic Control of the
Defense Budget in the United States and Western Europe. The Journal of Conflict
Resolution, 47(4):399-422.

Gerace, M.P. (2002). US Military Expenditures and Economic Growth: Some Evidence
form Spectral Methods. Defence and Peace Economics, 13(1):1-11.

Goldsmith, B.E. (2003). Bearing the Defense Burden, 1886-1989: Why Spend More?
The Journal of Conflict Resolution, 47(5):551-573.

Goyal, V.K., Lehn, K, & Racic, S. (2002). Growth Opportunities and Corporate Debt
Policy: The Case of the U.S. Defense Industry. Journal of Financial Economics,
64(1):35-59.

Jones, L.R., & McCaffery, J.L. (2005). Reform of the Planning, Programming,
Budgeting System, and Management Control in the U.S. Department of Defense:
Insights from Budget Theory. Public Budgeting and Finance, Fall:1-19.

Joyce, P.G. (2005). Federal Budgeting After September 11th: A Whole New Ballgame,
or is it Déjà Vu All Over Again? Public Budgeting and Finance, Spring:15-31.

Kanter, A. (1972). Congress and the Defense Budget: 1960-1970. The American
Political Science Review, 66(1):129-143.

Kiel, L.D., & Elliott, E. (1992). Budgets as Dynamic Systems: Change, Variations,
Time, and Budgetary Heuristics. Journal of Public Administration Research and
Theory, 2(2):139-156.

McCaffery, J., & Godek, P. (2003). Defense Supplementals and the Budget Process.
Public Budgeting and Finance, Summer:53-72.

Mintz, A. & Stevenson, R.T. (1996). Theories of Budgetary Tradeoffs. Journal of Public
Budgeting, Accounting and Financial Management, 7(4):586-612.

O Key, V. (1940). The Lack of a Budgetary Theory. The American Political Science
Review, 34(6):1137-1144.

546 | P a g e
Palmore, J., & Melese, F. (2001). A Game Theory View of Preventive Defense Against
Ballistic Missile Attack. Defense Analysis, 17(2):211-215.

Pillegge, J.C. (1992) “Budget Reforms” In Irene Rubin, ed. Handbook of Public
Budgeting, New York: Marcel Dekker, 67-94.

Ripley, R.B. (1969) Majority Party Leadership in Congress. Boston: Little, Brown.

Rubin, I. (1988). “Introduction.” In Irene Rubin, ed. New Directions in Budget Theory.
Albany: State University of New York Press.

Rubin, I. (1990). The Politics of Public Budgeting. Latham, N.J.: Chatham House.

Schick, A. (1988). “An Inquiry into the Possibility of a Budgetary Theory.” In Irene
Rubin, ed. New Directions in Budget Theory. Albany: State University of New York
Press, 59-66.

Schuler, A. (2005). Billions for Biodefense: Federal Agency Biodefense Budgeting,


FY2005 - FY2006. Biosecurity and Bioterrorism: Biodefense Strategy, Practice, and
Science, 3(2):94-101.

Wildavsky, A. (1964). The Politics of the Budgetary Process. Boston: Little, Brown.

Wlezien , C. (1995). The Public as a Thermostat: Dynamics of Public Preferences for


Spending. American Journal of Political Science, 39:981-1000.

Wlezien, C., & Soroka, S.N. (2003). Measures and Models of Budgetary Policy. The
Policy Studies Journal, 31(2):273-286.

547 | P a g e
4.7 Summary of Budgeting: Theories, Concepts, Methods, and Issues
Budget – A political and managerial instrument allocating scarce public resources
among social and economic needs.

Budget theory in particular should be able to answer questions about why particular
practices should be adopted, the importance of particular tasks, and the location of
particular tasks in a larger process. Budgeting does not currently have a theory in
this sense as it is fragmented and incomplete. The purpose of this article is to organize
and reduce the confusion by outlining some of the key assumptions, concepts,
methodologies, and current issues.

When the relationship of budgeting to society is examined for the United States, there
has been little agreement as to what budging reveals about the society. A theory that
utilizes differentiated interest groups would probably be more effective at capturing
and describing business impact and would undoubtedly be better at outlining the
impacts of other interests on the budget. (Neo-Marxist – class interests dominate
budgeting and allocation choices).

A second theoretical approach to linking budgeting and society is the public choice
school. Individual theorists have taken widely varied positions, but what they share is
the assumption that human behavior is based on individual economic rationality and
the maximization of individual benefits, or what economists call utility. Their model of
the ideal government is an extension of the marketplace where citizens can buy
exactly the amount of services they want in the combinations they prefer.

Public choice theorists emphasize the very important issue of the relationship between
what the citizen taxpayers want government to do and spend and what it actually does
and spends.

The voting mechanism posits that citizens can choose among candidates that best
reflect their own spending priorities.

The mechanism of suburbanization suggest that citizens choose their residences


based on their preference for public service packages – they move to communities

548 | P a g e
whose residents are people like themselves who want the same things from
government and who are willing to pay similar amounts of taxes to get them.

Openness of taxation implies that the scope of government and the expense would be
controlled if people clearly saw how much they were paying in taxes.

Despite some obvious strengths, public choice theory also has some weaknesses. This
theory tends to be deterministic, and has difficulty explaining change over time. The
theory ignores the importance and existence of altruism and the existence of group
goals that are different from individual goals. The third major theory that relates
budgeting to society is Incrementalism, and its related theory of interest-group
pluralism. Incrementalism argues that budgeting occurs virtually exclusively inside
government (Taft ?). Incrementalists see budgeting as a formal process based on
bargaining and technical needs, but fairly devoid of policy concerns.

The incrementalists do not envision the budget process as responding to perceived


societal problems, emerging situations, or environmental changes.

Thus, incrementalism is unable to theorize about changing budgets or budget process.

More recent theories of budgeting have not coalesced into a single school, but they
have moved toward a common understanding of how budgeting relates to the society
more broadly.

Hierarchy theory says that the top levels of the executive branch make decisions about
broad policy issues, and judge the environment, and pass that information down
through the budget office to the agencies before they make their requests.

The macro-micro budgeting theory argues that bargaining still goes on over budget
strategies, but broader policy issues are also explicitly dealt with and frame the
choices and outcomes of the bargaining.

The current consensus is that budgeting is linked to the society and to the
environment by both technical constraints and policy, and that interest groups are
sometimes important in determining outcomes. Budget theory has developed in a
dual fashion. First, theory strongly emphasized rationality and getting the most from

549 | P a g e
each dollar; then a second school grew up to refute the maximizers, arguing that very
little rational decision making went on. The current literature argues for a variety of
positions in between.

Budget - A detailed plan, based on estimates of needs and resources, for financing an
enterprise or government during a definite period, which is prepared and submitted by
an executive to a representative body.

Evolution of Public Budgeting

The U.S. Constitution did not include mention of a budget, but it required the
president to provide “a regular statement of receipts and expenditures”.

Budgeting first introduced in the Progressive Era (1890-1920) in response to


corruption and declining morality.

Municipal reformers turned to scientific methods and professionalism to eliminate


corruption.

The National Municipal League (NML) and the New York Bureau of Municipal Research
(NYB) called for uniform bookkeeping

NYB was formed by Frederick Cleveland and others, and was funded by businessmen.

Municipal reformers were the first to introduce budgeting in the U.S.

Budgeting practices in municipalities eliminated the worst forms of corruption.

STATES – Wisconsin 1912 adopted a budget measure as a joint venture between


Governor & Legislature.

New York was the first state to write the budget idea into its constitution.

Reform Proposals:

550 | P a g e
Before 1912, various techniques were recommended for centralizing responsibility for
the budget. After 1912, most reform proposals called for budget authority to rest with
the chief executive.

TAFT COMMISSION – (1910) – Suggested that the President should prepare and
present a budget to Congress, became foundation of Budget and Accounting Act of
1921. The commission reasoned that the constitutional privileges pertaining to the
receipts and expenditures and the state of the union authorizes a definite, well-
considered budget.

1913 – Taft submits a budget to Congress. It was not acted upon.

1921 – Budget and Accounting Act –

Established BOB and GAO. Each dept. to have a budget officer.

Created a centralized executive budget.

1932 – Economy Act – Pushed for installation of accounting forms, systems &
procedures.

1939 – Reorganization Act –

Moved Bureau of Budget (BOB) from Treasury dept. to the Executive Office of the
President.

BOB changed to OMB.

1974 – Budget and Impoundment Act –

Restricted impoundment authority.

Created CBO – Congressional Budget Office to counterbalance OMB.

Failed to stop budget deficits.

1985 - Gramm-Rudman-Hollings (Balanced Budget and Emergency Deficit Control


Act)

Focused on reducing the deficit and balancing the budget.


551 | P a g e
1990 – Chief Financial Officer’s Act – established CFOs in agencies.

1993 – GRPA – Govt. Performance and Results Act.

Was a product of Al Gore’s National Performance Review

Called for agency strategic planning, performance measures, outcomes

Complex and expensive to implement.

Allen Schick – Emphasis of budgeting changed over time

from Control to Management to Planning to Accountability/Results.

CONTROL – Early 1900s – Line Item Executive control, Incremental.

MANAGEMENT – 30s-50s – From recommendations of Brownlow Committee and


Hoover Commissions.

PLANNING – 60s – PPBS – efforts to rationalize budget process continued.

MBO and ZBB – 1970s.

Nothing reform-wise happened in 80s – general antipathy toward gov’t.

SEA initiative – from GASB - Service Efforts & Accomplishments.

Encouraged State & Local govts to include statements of service results in annual
financials (needed more performance info for financial reporting).

ACCOUNTABILITY - GPRA – 1993 – A Congress-initiated reform – emphasis on


planning & accountability for results. Long time horizons.

Budget Theory (See Handout Session 4 – Enzo & Amy)

Each theoretical school makes its own assumptions. (See Tyler & Willand article)

Neo- Marxists Theory:

Argue that class interests dominate budgeting and allocation choices

Argue that govt is controlled by capitalists, and they determine spending priorities.
552 | P a g e
Public Choice Theory:

Assumes

Various authors agree that there is no consensus as to an acceptable, operational


concept of budgeting.

INCREMENTALISM :

Dominant budgeting method used to date.

Wildavsky, Lewis & Lindblom support incrementalism as a theory.

HIERARCHY THEORY:

Argues that top-levels of executive branch make decisions about broad policy issues,
then pass that info down through the budget office to agencies before they make their
requests.

MACRO-MICRO BUDGETING THEORY:

Argues that bargaining still goes on over budget strategies, but broader policy issues
frame the choices and outcomes of bargaining.

Economic policies, priorities, spending ceilings and assumptions about the growth of
the economy are made by the budget committees and guide the decisions of other
committees.

Budget Systems:

Line Item

Control-based – Executive has line-item control over expenditures.

Inputs are the single scope of analysis.

Decision-units are simply objects of expenditure.

Incremental policy-making style

Performance Budgeting
553 | P a g e
Management is KEY.

Focus on Inputs and Outputs

Decision units – Activities of agency

Incremental policy-making style

PPB

PLANNING is KEY

Scope is on Inputs, outputs, effects and alternatives (managers submit alternative


budgets)

Decision units – purposes of agency/mission

Centralized – cuts Congress out of process.

ZBB

TBB – Target based budgeting (Rubin). Revenue projections need to be accurate.

PBB - Performance-Based Budgeting (See Session 7 handout from Agatha & Amy)

Results-oriented. Focus on outcomes and accountability

Strategic Planning, Management and Goal Attainment (established by top admin.)

Scope – Inputs, Outputs AND Outcomes

GPRA (1993) is the legislation that led to the adoption of PBB at Fed level.

Agencies must come up with: Strategic plans, goals, performance reports

Unlike PBB, Congress is included in the process.

Clinton told Gore to perform the Nat. Performance Review. Gore looked at other
countries that were calling their new budgeting system “New Public Management”.
Review resulted in recommendations out of which came GPRA, out of which came
PBB.

554 | P a g e
Reform Movements – See Rubin (1994) in Session 2 Handout from Mark & Jill.

Municipal Reform Movement (1870s):

Response to expansion after Civil War, corruption. NML - Large cities were important
for experimenting with different budget systems.

Research Bureau Movement (1906):

Reflected the Progressive spirit – Executives/Professionals running govt. Desire of


progressives is to move away from non-qualified people running govt. NY Bureau of
Municipal Research.

Budgeting Concepts:

Budgets are an important link between taxpayers and the public officials. Budgets tell
citizens what their taxes are being spent on, and in a general way how well their taxes
are being spent on, and in a general way how well their money is being spent. Public
budgeting is both political and technical, influenced by interest groups and agency
heads. Public budgeting is open to the environment, in the sense that it is influenced
by the economy, by public opinion, by other levels of government, by interest groups,
by interest groups, by the press, and by politicians. Descriptions of budget processes
have not been simply a list of decisions, actors, and due dates, but descriptions of the
shifting location of power. Two major dichotomies have been used to describe the
shifts in location of power: executive versus legislative, and top-down versus bottom-
up.

The overall direction of change during this century (20th) has been toward more
executive and more top-down budgeting.

Line-item budgets seem almost deterministic. They tell little if anything about the cost
or efficiency of programs provided.

These types of budgets are becoming rare

555 | P a g e
Program budgets allow public officials to clearly choose between priorities and express
them in the budget, but they do not provide a way of evaluating the relative efficiency
or effectiveness of programs.

Performance budgeting has had many difficulties, and while sometimes implemented
has seldom worked as billed, and has often been modified.

Zero-based budgeting in its full-blown form requires the agencies to put all their
budget requests into decision packages, and rank order them in importance. Then the
rankings of the agencies are gathered and arranged according to criteria that make
sense at a government-wide level.

Planning programming budgeting systems were the most integrated and fullest
statement of the linkage between planning and budging, but even when the full
system is not adopted, budgets may state goals and link spending requests with those
goals.

The emphasis on linking budgeting to goals was used with the linkage of Management
by Objectives (MBO) to budgets.

Some Budget Terms:

Tradeoffs – money being spent on one program or project can not be spent on another
at the same time.

Equity – a broad term that also applies to taxation, but in the context of spending
choices it raises the topic of welfare economics. Does government have a role in
making the distribution of income more equal than the market alone would create?

Accountability – is government doing what the taxpayers want it to do? Do the


priorities of the government, as reflected in the budget, reflect the priorities of the
taxpayers?

Balance – means the total income has to match the total outgo, with or without
borrowing.

556 | P a g e
Reform – change in a direction advocated by some groups or individuals. It does not
necessarily mean improvement!

Methodology

Studies of budget processes have normally been based on elite interviewing and
participant observation (Qualitative). However, studies on decision making have been
quantitative. Surveys have also been a continuing part of budget studies, and are
particularly useful to define changing practices at the state and local levels, where
more qualitative work has difficulty generalizing.

For the future, more qualitative studies are needed, and studies of the variety and
history of budgeting are required.

Issues

The relationships among different business interests and their impact on budgets need
further exploration.

Public choice theory postulates excessive public demands for government services, but
provides no mechanism for the choice among demands.

Hierarchy theory is supposed to tie budget process to changing environments, to be


historically rooted, but to date the school has used data only from one period of time,
and has assumed the policy input from the president.

Much more work needs to be done to link processes and decision making to outcomes.

557 | P a g e
4.8 Important Comps Questions and Student Answers
Q – 1: Other than Performance-Based Budgeting, what are the most important recent
developments in financial management? Why are they important and what
precipitated them? Explain how these recent changes may/can influence the work of
public managers and policy makers.

Indeed, performance based budgeting has been a significant development in financial


management. The initial performance based budgets were developed in the 1940s as a
result of the Brownlow and Hoover Commissions. These budgets were management-
oriented systems heavily focused on efficiency by relating costs to measured outputs.
In the 1960s Planning-Programming Budget Systems (PPBS) emerged emphasizing
multiyear planning, policy analysis, and program objectives focusing on effectiveness.
PPBS was discontinued in government in the 1970s during a period in which the
executive and legislative branches battled over who controlled the budget. Then in
1977, under the Carter administration, Zero Based Budgeting was implemented in the
federal government. ZBB was to replace the traditional incremental (line item)
budgeting systems. It was overwhelmed by changing economic events and in 19981
was rescinded by the Reagan administration. Budget deficits were the budget focus of
the 80s and the passage of the Balanced Budget and Emergency Deficit Control Act in
1985 (Gramm-Rudman-Hollings Act) began a lengthy era of dealing with deficits. With
the passage of the Government Performance Review Act in 1993, we see the emergence
of a new performance budgeting system more concentrated on outcomes. This form of
budgeting dominates both federal and state budgets and continues to exert influence
in budget decisions today. Of significance is that performance budgets, or outcome
budgets, connect the budgeting process with policy formulation, strategic planning,
program goal formulation, and program evaluation.

Since the early 1990s with the rapid implementation of performance based budget
there have been some important new developments affecting public budgets. Growing
deficits, the public’s demand for accountability, and a clear rivalry between the
President and Congress as to who should control the budget has precipitated these
developments. Some of these new developments are a result of the passage of new
laws related to public budgeting. These would include:

558 | P a g e
Budget Enforcement Act – 1990 – amended the Gramm-Rudman-Hollings Act and
created a number of procedural requirements such as “pay-as-you-go”. This meant
that after the budget had been passed, any new spending approved by Congress and
the President had to be compensated for by new sources of revenues other spending
reductions.

Credit Reform Act – 1990 – tightened requirements on federal lending and loan
guarantees.

Chief Financial Officers Act – 1990 – added new financial management responsibilities
for federal agencies including creating a CFO position to oversee agency financial
practices.

Omnibus Budget Reconciliation Act – included major reductions in defense spending


and tax increases for upper income groups and no tax cuts for the middle class.

The Line Item Veto (1996 to 1998) – For a two year period, gave the President the
power to veto specific appropriations. Found to be unconstitutional by the U.S.
Supreme Court in 1998.

Performance based budget reforms have focused attention more that ever on sharing
information and decision making in budgeting. Sharing information about what
governments do well, through performance measurement and reporting, can go a long
way in reducing cynicism. Broader participation in trading off various goals and
means to achieve them, as well as in developing performance measures, will yield
better decisions and a sharing of risks among citizens, public managers, and elected
officials.

The GPRA and state and local results acts and executive orders emphasize program
results and holding federal government agencies accountable for them. The GPRA
focuses managers’ attention on setting goals, and reporting publicly on progress made.
One of the foremost purposes of the act is to instill confidence in the public about
federal government managers’ ability to solve problems and meet citizen-taxpayers’
needs. To implement the GRPA, each agency must first develop strategic plans
covering a period of at least five years. The strategic plan must include a mission

559 | P a g e
statement, outcome related measurable goals and objectives, and plans that agency
managers and professionals intend to follow to achieve these goals through their
activities and through their human, capital, information, and other resources. Those
in the agency must consult Congress and others interested in or affected by the plans:
in other words, they must consult stakeholders. The new reforms incorporate most of
the goals of the previous reforms, but they seek to achieve them through decentralized
incentives that give program managers greater authority to combine resources as they
think best but that hold the managers accountable for the results.

Performance based reforms deal widely with organization change. Keep in mind that
no other decision making system has the leverage to pressure departments to improve
program management like the budget. Also, the budget has always been the place
where everyone raises questions of the efficiency, economy, effectiveness, productivity,
impact, and results of government activities. Lastly, budget offices can inspire
agencies to strengthen their programs, operating systems, and organizational
structures.

These new efforts at performance budgets appear to have a promising future; however,
some authors are not so enamored with performance based budgets. Wildavsky warns
that results that are “too good” can have adverse effects in that superb results may
result in no further funds being appropriated, as the goals will have apparently been
achieved. Caiden finds that performance may be irrelevant in making budget
decisions as political popularity and the need to balance budgets are the sole reason
for budget decisions.

For managers of programs that meet their goals and objectives and otherwise exhibit
exception performance, pay-for-performance has emerged as a strategy for responding
to increased productivity of staff. Merit pay is one method, including bonuses,
commissions and formulas for sharing savings. NOTE: For more information on the
use of merit pay, good and bad, refer to: Halachmi, A, Holzer, M. (1987). Merit pay,
performance targeting, and productivity. Review of Public Personnel Administration, 7
(Spring), 80-91.

560 | P a g e
Q – 2 : In her article treating budgeting, included within Naomi Lynn and Aaron
Wildavsky’s Public Administration: The State of the Discipline, Naomi Caiden
concludes that “Public Budgeting is undoubtedly a discipline in the dictionary
definition of the term as a ‘branch of knowledge or learning’.”

What evidence supports this assertion?

Caiden then cautions: “But if the study of public budgeting is to be more than a
collection of disparate concerns, it requires stronger themes, and theories that act not
only to unify them but also to reveal the philosophical assumptions underlying
empirical description and normative proposals.”

Q – 3: Assess the state of the “discipline” of public budgeting

Q – 4: In a renowned 1940 article, V.O. Key, Jr. lamented about “the lack of budget
theory.” Twelve years later (1952), Vernon B. Lewis attempted to construct a
normative budget theory. In one of her numerous writings, Irene Rubin noted that
“budget theory today is fragmented and incomplete….It is in the process of being
invented.” Based on the work of scholars who address this issue, discuss the extent
to which the literature of public budgeting evidences “theory.” Assess the limitations
of such a “theory of budgeting.”

Q – 5 : Cite the major federal financial and budget reforms of the past decade,
identifying the basic characteristics (provisions) of these reforms. Discuss the
implications that these budget reforms hold for public management at the federal,
state, and local levels. Discuss the implications of the value assumptions implicit in
these reforms vis a vis the fundamental values and beliefs which under gird public
administration.”

Public budgeting in the US is about accounting and financial management; it is also


about accountability and governance. A fundamental principle of the American
system of governance is separation of powers – an aspect which is important for
restraining the branches and achieving accountability in public budgeting. Protects
citizens against the budgetary abuses of excessive taxation and imprudent spending.
561 | P a g e
Budget Reform in the 1990s: The "New" Performance Budgeting

In the 1990s considerable attention has been refocused upon performance budgeting
resulting in a "new" notion of performance budgeting to use Mikesell’s term, or what
others have called "Entrepreneurial Budgeting" (Cothran, 1993; Lynch, Hwany and
Lynch, 1996). As is usually the case, however, these concepts did not suddently spring
upon the scene, but rather became the object of renewed attention and interest as
fiscal constraints persisted in many governmental entities.(9) Several things converged
though to refocus attention on performance budgeting.

One influence was the 1992 publication of Reinventing Government, How the
Entrepreneurial Spirit Is Transforming the Public Sector by David Osborne and Ted
Gaebler. They noted among other things what was widely known--Americans are
cynical about their government. (See, for example, Kanter, 1989.) As one remedy they
proposed a results-oriented budget system (although the term was not unique to
Osborne and Gaebler). The idea was to hold governments accountable for results
rather than focus upon inputs as traditional budgets and management did. Cost-
savings and entrepreneurial spirit would be rewarded. A long term view would be
facilitated in terms of strategy, costs and planning for programs. Gaebler was a former
city manager turned consultant and Osborne was a writer and consultant. Thus, they
had exposure to a wide variety of governments. From this experience they focused on
examples they found of enptrepreneuial management in government. A local
government example they cited as a "performance leader" was Sunnyvale, California
and its focus on outcomes rather than inputs (1992: 142). Other examples of
"innovation" at the federal and state level were also cited.

Interestingly, as is often the case, efforts to improve performance and increase


economy in the public sector were underway before Reinventing Government was
published.(10) At the federal level, for example, the Chief Financial Officers Act of
1990 (P. L 101-576) required the development and reporting of systematic measures of
performance for twenty three of the larger federal agencies. The Governmental
Accounting Standards Board had already examined the use of service efforts and
accomplishments (SEA) reporting for state and local government entities. Beginning in
the early 1980s it had encouraged governments to report not just financial data in

562 | P a g e
budgets and financial reports but also information about service quality and outcomes
and in 1992 issued a Preliminary Views report on SEA reporting (Wholey and Hatry,
1992) followed by another statement in 1994 (GASB, 1994).

Federal Reform Efforts in the 1990s

One thing that popular ideas like reinventing government do, however, is get the
attention of the media and political leaders. Drawing on this attention, the
Government Performance and Results Act (GPRA) of 1993 (P. L. 103-62) was passed by
Congress. The GPRA drew upon earlier antecedents, such as a report submitted
during the Reagan Administration and a bill introduced in 1991 by Senator William
Roth that had not been enacted, but was revised and became the GPRA. (See Groszyk,
1995.) This act specifically focused attention on results and performance budgeting.
The act requires federal agencies to prepare strategic plans by 1997, to prepare annual
performance plans starting with fiscal year 1999, and submit an annual program
performance report to the President and Congress comparing actual performance with
their plans beginning in the year 2000. Due to its relatively recent passage and phase-
in schedule, the GPRA will obviously need some time for assessment.

As the GPRA was becoming law, the National Performance Review (NPR) was coming to
the forefront also as an initiative for Federal government reform led by Vice President
Al Gore. The NPR embraced many of the ideas of the GPRA and added more attention
to performance and results management, calling also for a move toward budgeting
based on results.(11) The ‘New" Performance Budgeting, however, draws from earlier
performance budgeting ideas and from program budgeting/PPBS concepts as well. It
differs from those ideas in that it does not advocate crossing agency lines like program
budgeting and PPBS did, however. And, according to Mikesell (1995: 189), it does not
concentrate on tasks, activities or outputs as much as the old performance budgeting
did, rather emphasizing an outcome focus.

As a result of the activity and interest at the national level in managing for results and
performance budgeting, both the General Accounting Office (GAO) and the
Congressional Budget Office (CBO) have studied the reform proposals in recent years.
The CBO, in a 1993 study, Using Performance Measures in the Federal Budget

563 | P a g e
Process, concluded that performance measurement "is limited in its ability to bring
about substantial change" (1993: 44; see also Joyce, 1993). It noted, however, that
some of these limitations had nothing to do with commitment but rather with the
difficulty of measuring government performance itself, and particularly that of the
national government. The greatest obstacle it found was the identification of the
measures themselves, in large part because at the national level "so many programs
[are] influenced by other actors, including state and local governments, private
businesses, and individuals" (Congressional Budget Office, 1993: 44). In so far as
performance budgeting itself is concerned, the CBO, after studying state and local
government experience, concluded that performance measures did not appear to
significantly influence the allocation of budgetary resources. Rather, they were used
more to carry out budgets than to make decisions. The GAO (1993) came to similar
conclusions as well.

State Reform Efforts in the 1990s

State interest in the "New" Performance Budgeting in the 1990s is evident in a number
of ways. The National Governors’ Association (NGA), for example, published An Action
Agenda to Redesign State Government in 1993 which called for creating performance
based state government with measurable goals, such as benchmarks and performance
measures, in order to move to performance budgeting. A year later, the National
Conference of State Legislatures (NCSL) published a study entitled The Performance
Budget Revisited: A Report on State Budget Reform (Carter, 1994). Together these two
studies found interest in performance budgeting and measurement in a number of
states, including Oregon, Minnesota, Montana, Iowa, Texas, Idaho, Ohio, Florida,
Mississippi, California and Virginia, to name a few.

The NCSL, like the GAO study in 1993, found that performance measures and
budgeting had not yet attained the credibility needed to influence budget allocation
decisions at the state level, due often to constraints on time, resources and data.
Conceptual as well as political issues remain according to these studies regarding
performance measurement and budgeting. Still, performance measurement was
viewed as useful for internal agency management.

564 | P a g e
One is reminded when reading accounts of interest in concepts like performance
budgeting of Premchand’s "fortune cycle" that consists of "great anticipation, quick
acceptance, partial implementation, partial dissatisfaction, search for new techniques-
-and the rediscovery of continuing themes" (Premchard, 1981; see also Lewis, 1988).
The "rediscovery" or renewed interest in performance budgeting at the state and
national level appears to be drawing on a continuous theme in American budgeting--
the need for a way to make resource allocation decisions based on more than the
inputs that will be used to carry out public programs. That pressures would build at
the state (and local) level in particular for such devices is not surprising given their
relatively inelastic revenue sources and requirements for balanced budgets, not to
mention their proximity to the citizen and voter.

What we may be missing, however, is the slow, incremental process by which change
is occurring in governmental budgeting and management. Periodically one gets a
glimmer of evidence of change occurring, such as when changes are studied over
extended periods of time. Lee’s research, for instance, on developments in state
budgeting between 1970 and 1990 found "considerable change" such as more program
effectiveness and productivity information being required of state programs, measures
being included in budget documents, analysis being conducted by the central budget
office and legislature and so on (Lee, 1991). Earlier indications of progress being made
in state budgeting outside the glare of periodic reform proposals include changes made
in Kansas, particularly involving the introduction of planning into state government
and its linkage to state budgeting. (See Bibb, 1984) Thus, managing for results and
performance measurement are the focal point in many states as they prepare to enter
the 21st century. While they may not have actually initiated formal performance
budgeting programs, this should not obscure the direction that many of them appear
to be headed in.

The national and state interest in the "New" Performance Budgeting draws upon a
common source. That is a cynicism and loss of confidence in government in the United
States. The 1970s saw the tax and spending limits movement begin, first in California
and then in Massachusetts. Since then it has spread to a number of other states and
local governments. In many ways this movement is symptomatic of the loss of public

565 | P a g e
confidence in government. Limitations have often been most severe for local
governments, especially when coupled with mandates from higher levels of government
and existing restrictions on revenue raising capability. Accordingly, local governments
have been forced to find ways to respond to this era of limits and scarcity.

Local Reform Efforts in the 1990s

Some local governments have responded by becoming entrepreneuial. One local


government in particular has received a great deal of attention for its entrepreneuial
efforts--Sunnyvale, California. Sunnyvale was the inspiration for the Government
Performance and Results Act and was cited by Osborne and Gaebler in their book
Reinventing Government. A Council-Manager city of about 120,000 people located
south of San Francisco, Sunnyvale is unique in its application of performance
measurement and budgeting at the local government level. It has a General Plan
looking 5 to 20 years into the future. The plan comprises seven elements and 20
subelements that set goals and policies for the city. Its Resource Allocation Plan is a
10-year budget to implement the General Plan. Each year the annual budget is a
performance budget that targets specific service objectives and productivity measures
linked to the larger plan. Its budget therefore is a service oriented document rather
than the traditional line-item, input oriented budget.

In many ways, this city appears to be the embodiment of the contemporary interest in
performance budgeting. Yet, observers note that it is atypical of other cities in terms of
its demographic characteristics (Lewcock and Rogers, 1988; Mercer, 1994; and "Better
Government, A Clockwork City," 1993).

On the other side of the United States is another city which has also received some
national attention for its innovative management, Rock Hill, South Carolina
(Wheeland, 1993). Rock Hill has been actively involved with strategic planning for a
number of years. And, it has also received the Government Finance Officers
Distinguished Budget Award, most recently in 1996. This national award was begun
in 1984 for outstanding budget presentations by state and local governments. To date
over 700 local governments have received the award (including Sunnyvale). Rock Hill
is smaller (population 41,600), however, than Sunnyvale and is not located in a high

566 | P a g e
tech growth corridor. Thus it is in many ways more typical of many cities having to
struggle with a declining economic base (the demise of the textile industry) and facing
competition from nearby growth centers (in this case the Charlotte-Mecklenburg SMA
in North Carolina). Like many other cities in its population range, it has the council-
manager form of government (as does Sunnyvale, although cities over 100,000 do not
commonly use this form). Also, like others, Rock Hill has regularly sought recognition
for its financial management and budget practices. In some ways Rock Hill’s budget
might be what ICMA (1993) calls "Strategic Budgeting." This is a budget process that
is built on strategic planning and which uses the budget as a policy guide, a financial
plan, an operational guide, and a communications device with citizens and taxpayers.
Briefly, the budget includes all funds and addresses all organizational needs through
goals and objectives clearly stated and where possible measurable. A longer time
horizon than one year is used as well.

When one examines the fiscal year 1996 budget for Rock Hill and compares it with
previous years, for example, one notices one clear difference--there are no expenditure
line-items in the budget. Departmental expenditures are summarized in terms of
program goals, program objectives and performance measures. Thus, like a number of
other local governments, Rock Hill is shifting its focus to accountability and results.
While cities like Sunnyvale may be described as entrepreneuial, Rock Hill may be more
symbolic of what more local governments, particularly cities, are doing to respond to
revenue constraints and scarcity, namely engaging in more planning and management
improvement using what are viewed as "state of the art" techniques such as strategic
planning and strategic budgeting, or the "New" Performance Budgeting. Some evidence
exists that productivity and quality improvement efforts at the local level of
government are having some impact nationally as well, although more information
needs to be collected (Cope, 1995).

CONCLUDING REMARKS: Prospects for Budgeting in the 21st Century

Budgeting in the United States has experienced at least five emphases, starting with
control at the turn of the century, moving to management in the New Deal and post-
World War II period, to planning in the 1960s, prioritization in the 1970s and 1980s
and now to accountability in the 1990s. By all accounts the initial emphasis on

567 | P a g e
control has been successful if one interprets the adoption of line-item budgeting as an
indication of that emphasis. It appears that at least eight out of ten cities and counties
use line-item budgeting in some form today (Cope, 1995: 43). As Cothran writes:
"Although none of these efforts, such as performance, program, or zero-base
budgeting, entirely supplanted incremental line-item budgeting, elements of these
reforms endure in the budgeting process of many governments" (1993: 445).

Assessing the impact of the remaining reforms becomes somewhat more difficult, in
part due to the American federal system and its diversity. In addition, there are
methodological challenges to studying large numbers of governments. Survey research
is often used and as Schick (1971) notes surveys are not entirely accurate if
exclusively relied upon when assessing budget approaches. (The CBO and GAO
studies also reflect the need for other research methods, such as case studies and
expert panels.)

One thing that most scholars and practitioners agree on is that most reforms are
oversold. Consequently, because the expectations are unrealistic, assessments
conclude that they have failed. Conclusions of general failure may not be entirely
accurate, however. Rubin (1990) makes an excellent point when she notes that budget
reforms have been more successful than many people in public administration think.
While some reforms may indeed be fads, in other cases some changes have
occurred.(12) Partially this is due to overemphasis on the federal level of government
by scholars. Innovations often begin and continue at the state and local level of
government without much fanfare.

But Rubin and Rose make another point as well that we feel needs to be emphasized.
Namely, observers may look too quickly for results. As Rose has written: "The
absorptive character of government, gradually adapting and incrementally
augumenting its activities, suggests that change may more easily be measured on a
time scale congenial to a forester or a geologist than to a Congress or a White House in
a hurry" (1977: 64). Many innovations are clumsily introduced and require adaptation
(Rubin, 1990) and, ideas are frequently adapted to local circumstances and needs
(Walters, 1996). Thus, reforms or innovations have to be evaluated over a period of
time using a scale of achievement.

568 | P a g e
If we take this longer view, it appears that budget reform is alive and well in the United
States. The federal government and many states are continuing to experiment with
program and performance information as are local governments. Professional
organizations, like the Governmental Finance Officers Association, continue to nurture
change and advancement in budget presentation and financial reporting.

Cities like Sunnyvale gain attention as they illustrate a willingness to change and
experiment resulting in the use of terms like Entrepreneurial Budgeting. Perhaps more
often, cities like Rock Hill strive to advance their decision making using more
commonly understood notions of performance budgeting and performance
measurement in response to taxpayer resistance and revenue scarcity.

Thus, as the twentieth century ends, local governments are a focal point once more for
budget innovation and change. Part of the reason for this is that they were at the
forefront of the movement in the United States to establish strong executive forms of
government. Along with that movement went also the council-manager form of
government. Budgeting in the United States is heavily influenced by both the federal
system and executive-legislative conflict (Hyde, 1992: 3). Competition between the
executive and legislative branches is most evident at the national and state levels of
government. It is the local level, and then in the mid-size population range of cities,
that we find truly strong executives with weak, part-time legislators in the
professionally managed council-manager cities. Whatever the merits of the council-
manager form of government, a good case can be made that it has allowed more
innovation and reform to occur. Thus, change and continued progress can be expected
in the field of American budgeting in the twenty first century building upon both what
was begun at the turn of this century and the trends underway in the latter part of
this century.

Federal Unfunded Mandate Reform:

A First-Year Retrospective Theresa Gullo and Janet Kelly

This document, written by, describes the progress that The Unfunded Mandates
Reform Act of 1995 has made. This act took effect on January 1, 1996, and has made
two important changes about the budgetary impact of federal mandates is provided to
569 | P a g e
and used by Congress. First, the act increased the amount of information, and as a
result, placed a higher priority on preparing state and even local estimates. Second,
this act encouraged Congress to use the information by establishing new points of
order procedures.

The Unfunded Mandates Reform Act was enacted as part of the Republicans’ Contract
with America. However, many governors were reluctant to support such an
amendment. They were worried that the pressure for new programs and policies would
remain constant while the budget would decrease, and the requirements for balancing
the budget would shift the cost of new programs and policies to the states. At the 1995
annual association meeting, the governors declared that the unfunded mandates
should be protected first, and the Congressional Republicans agreed.

As stated before, one of the main goals of The Unfunded Mandates Reform Act (UMRA)
was to improve the information that Congress received about the effect of federal
legislation on state, local, and tribal governments, and the private sector. Another goal
was to make it more difficult for Congress to enforce unfunded mandates on state and
localities.

The UMRA obtains four titles: Title I, Legislative Accountability and Reform. This Title,
in general, makes it harder for the Congress to consider unfunded mandates with cost
above specific thresholds. Title II, Regulatory Accountability and Reform, requires
federal agencies to review the effect of proposed regulatory actions on the state, local,
and tribal governments when the costs of that effect exceeds one hundred million
dollars. Title III, on the other hand, forces the U.S. Advisory Commission on
Intergovernmental Relations to have three reports. These reports include: a review of
the role of mandates in intergovernmental relations; a study of the measurements and
estimating issues surrounding the costs of mandates; and finally, a review of federal
court rulings to identify requirements placed on state, local, or tribal governments.
Lastly, Title IV authorizes federal courts to compel agencies to comply with the duties
outlined in Title II.

The UMRA defines an intergovernmental mandate as “any provision in legislation,


statute, or regulation that would impose an enforceable duty upon state, local ot tribal

570 | P a g e
governments, except a condition of federal assistance; or a duty arising from
participation in a voluntary federal program.” However, this definition is unclear
because it excludes legislation that enforces the constitutional rights of individuals. To
help with this confusion, a point of order lies against any bill reported from an
authorizing committee if a mandate statement from the CBO is not attached to it. A
point of order also lies against any bill if it contains an intergovernmental mandate
with unfunded act above the threshold. However, this procedural constraint is not
self-enforcing. A member of the House or Senate can easily avoid a point of order and
pass the bill.

The UMRA has also given the CBO new responsibilities. In fact there have been two
important changes. The first change is that the CBO is encouraged to place a higher
priority on preparing state and local government estimates. Second, by establishing
the provisions on points of order, the new law protects that information. The CBO is
also required to prepare state and local government cost estimates since1982. The
Congress showed its willingness to take information seriously by using it as the basis
for points of order. In addition, the act requires the agency to prepare cost estimates
for tribal and private sector mandates and allows for more detailed analysis of both
public and private sector mandates when requested.

Of the five intergovernmental mandates that CBO identified as exceeding the fifty
million dollar threshold, only the increase in the minimum wage was enacted into law
in a form that will impose costs on state and local governments above that threshold.

One of those bills included provisions that would have amended the Fair Labor
Standards Act to require an increase in the minimum wage for all workers. The CBO
decided that such an increase would constitute an enforceable duty on all employers
and thus impose both an intergovernmental and a private-sector mandate as defined
by the UMRA.

The Health Insurance Reform, on the other hand, required parity for mental health
benefits relative to physical health benefits for group health insurance. A more limited
version of the requirement was enacted. The CBO estimated that the direct coasts to

571 | P a g e
the state and local governments of the more limited version were well below fifty
million dollars in any year.

The bill, Applying Occupational Health and Safety Standards to State and Local
Workplaces, would have changed the Occupational Safety and Health Act to make
workplace requirements applicable to the twenty-seven states where they do not
currently apply. This bill was never even considered on either the House or Senate
floor.

The Fourth bill, Securities Regulatory Reform, would have preempted state laws and
regulations regarding certain types of securities and security-related transactions. The
estimated costs of the mandates would total about fifteen million dollars annually.
This bill was later enacted into law.

The last bill constructed by the UMRA was the Immigration Reform: Driver’s License
Provisions. This bill would have required state agencies issuing driver’s licenses or
identification documents either to print Social Security numbers on those items or to
collect and verify the number before issuance. Those requirements would have been
effective within one year of the bill’s enactment. No point of order was raised against
the bill, as amended, and it was ultimately enacted into law.

Also, because of the diversity of affected entities, aggregate cost estimates were
extremely hard to make. Finally, the agency decided that it was difficult to gather
adequate data at the legislative stage of the process, because the available time was
often too short and because the likely impacts were too uncertain.

The UMRA did lead to some important changes in the estimating process. By
establishing a fifty million dollar threshold for intergovernmental mandate costs, the
act tended to simplify the job of the CBO, at least for bills with mandate costs that
clearly exceeded that level. The agencies found that one-on-one telephone contacts
were still the best tool for collecting data. They have also learned that more interest
group has increased the capacity to respond back to the CBO. The state and local
interest groups have appeared to increase their capacity to respond to the
Congressional Budget Office’s inquiries. The Federation of Tax Administrators was an
extremely useful resource for the CBO.
572 | P a g e
Just as the CBO found that the old tools were most effective, the agency also found
that the same obstacles were still the hardest to overcome. For example, separating
the incremental costs of federal actions continued to be difficult. Consequently, state
and local governments analysts have a hard time predicting how they would comply,
making it difficult for the CBO to estimate costs reliably. Before the CBO found that
state and local estimates generated little interest. If the agency did not provide a state
and local estimate for a bill, the Congress rarely asked it to prepare one. And few
controversies or questions arose about the estimates either from the Congress or from
state and local governments. The CBO operated in an environment where little
attention was paid to its state and local estimates.

That environment has changed dramatically since the UMRA has been in effect.
Representatives of state and local governments are now much more likely to approach
the CBO about mandates proposals and to assist in gathering cost data. Members of
Congress and their staff are also much more likely to request analyses of mandates
and to question the agency’s methods and assumptions. As a result, the estimating
process is much more dynamic and has helped improve the quantity and quality of the
information being provided to the Congress.

What do we know About Budgeting?

There is virtually nothing of substance about how or why budgetary decisions are
actually made. Yet the opportunities for developing and testing important propositions
about budgetary decisions are extraordinarily good and I would like to suggest a few of
the many possible approaches here.

How do various agencies decide how much to ask for? Most agencies cannot simply
ask for everything they would like to have. If they continually ask for much more than
they can get, their opinions are automatically discounted and they risk a loss of
confidence by the Budget Bureau and Appropriations subcommittees that damages
the prospects of their highest priority items. We might also inquire about the
participants' perceptions of their roles and the reciprocal expectations they have about
the behavior of others. Budgetary items are commonly adjusted on the basis of mutual
expectations or on a single participant's notion of the role he is expected to play.

573 | P a g e
By its very nature the budgetary process presents excellent opportunities for the use
of quantitative data although these must be used with great caution and with special
attention to their theoretical relevance. It would also be desirable construct a theory of
budgetary calculation by specifying the series of related factors that affect the choice
of competing alternatives by the decisions makers. This kind of theory would describe
how problems arise, how they are broken down, how information is fed into the
system, how the participants are related to one another, and how a semblance of
coordination is achieved. If changes in procedure lead to different kinds of conclusion,
one would like to be able to predict what the impact on decisions was likely to be.

The Goals of Knowledge and Reform

Concentration on developing at least the rudiments of a descriptive theory is not


meant to discourage concern with normative theory and reform. On the contrary, it is
worthwhile studying budgeting from both standpoints. Surely, it is not asking too
much to suggest that a lot of reform be preceded by a little knowledge. The point is
that until we develop more adequate descriptive theory about budgeting, until we
know something about the "existential situation" in which the participants find
themselves under our political system, proposals for major reform must be based on
woefully inadequate understanding.

Perhaps the "study of budgeting" is just another expression for the "study of
budgeting"; yet one cannot study everything at once, and the vantage point offered by
concentration on budgetary decisions offers a useful and much neglected perspective
from which to analyse the making of policy. The opportunities for comparison are
ample, the outcomes are specific and quantifiable, and a dynamic quality is assured
by virtue of the comparative ease with which one can study the development of
budgetary items over a period of years.

Q – 6: The budget may be viewed as an instrument of fiscal policy, as a means of


determining policy choices, and as a tool for managing the economy. Explain each of
these aspects of budgeting. In your answer, emphasize the views of the classical or
pre-Keynesian economists, Keynesian economists, neo Keynesians, the monetarists,
the public choice school of economy, and leading public budgeting scholars

574 | P a g e
Classical/pre-Keynesian

 Adam Smith wrote The Wealth of Nations in 1776 --- father of capitalism
 the free market is most important
 government’s role in the economy is very limited
 voluntary exchange between buyers and sellers leads to economic efficiency
 prices are set by competition in the free market based on supply and demand

Keynesian

 John Maynard Keynes wrote General Theory of Employment, Interest and


Money in 1936
 some government spending is necessary to fine tune the economy
 performance by the economy in the aggregate is more important than
individual
 so we need to look at output (GDP) , income and employment rates
 potential GNP means the full use of the factors of production (land, labor,
capital)
 compare potential GNP with actual GNP
 aggregate demand Y = C + I + G + (X-M)

C is consumption

I is investment

G is government spending

X-M is exports minus imports

 small deficits are ok---necessary to fine tune the economy


 the multiplier effect is very important---an increase in aggregate demand will
result in an even larger increase in actual or equilibrium GNP

Neo Keynesian

 Reagan and Bush One

575 | P a g e
 they called for big changes in government spending, interest rates and to
counter economic cycles
 in a recession they want more government spending and tax cuts to stimulate
the economy
 in inflation they want less government spending and tax increases to slow the
economy down
 they also use monetary policy (interest rates set by the Federal Reserve Board)
to influence the economy

Monetarists

 Milton Friedman and Alan Greenspan


 Professors at Syracuse says monetarists may be right - monetary policy may be
more effective in impacting the economy than fiscal policy
 they are also called “counter Keynesians” because they disagree with Keynes
 they say the economy is inherently stable---government fine tuning is not
necessary
 to them, the money supply is most important
 and the supply of money is controlled by interest rates which are set by the Fed

Pubic Choice

 this is a modern day theory


 followed by pre Keynsian economists
 individuals are rational---they make choices to best benefit themselves
(whether as buyers or as bureaucrats)
 politicians may claim to follow neo Keynesian policies but sooner or later they
must balance the budget

see Forrester, John P., Evolving Theories of Budgeting, pp. 101-124

See Stapleford, John E. (1992). “Economic Impact of Budgeting” in J. Rabin (editor),


Handbook of Public Budgeting. pp. 401-418. **See hard-copy of summary

Also see Ooms, V.D., Boster, R.S., and Fleegler, R.L. (1999). Handbook of Government
Budgeting, pp. 197-226
576 | P a g e
The Federal Budget and Economic Management, V.D. Ooms, R.S. Boster and R.L.
Fleegler

Handbook of Government Budgeting, pp. 197-226 (1999)

The question: Can the budget by used to manage unemployment, inflation and
economic growth?

The conclusion: The budget can be used to influence economic growth but NOT
unemployment and inflation (this may even be counterproductive).

Definitions:

Fiscal policy---choices regarding a budget’s total revenues, expenditures and deficit


(government spending)

Budget policy---choices about the broad composition of aggregate revenues and


expend.

Stabilization policy---fiscal (“fis”) & monetary (“mon”) policies to influence


unemployment (“unemp”), inflation (“infl”) and economic activity in the short term

Growth policy---using fiscal and monetary policies to affect long term changes in
productive capacity---

Monetary policy---Federal Reserve actions regarding short-term interest rates to affect


the money supply

Economic stabilization---concerned with short term unemployment and inflation---2


years max

Stabilization policy---changes the level of spending and thereby changes the level of
output---this affects unemployment and inflation

Real GDP---the total value of goods& services produced in the U.S. per year adjusted
for inflation

Potential GDP---the output the economy has the capacity to produce without
increasing inflation
577 | P a g e
Economic growth---an increase in potential GDP over years or decades

Productive factors---labor, skills, machines and their related technology---growth


policies Increase future levels of these factors---thereby increasing Potential GDP

Stabilization is concerned with the gap between real GDP and potential GDP and the
implications of the gap for inflation and unemployment---see p. 199

The goal of stabilization policy is to manage the gap so that GDP is close to its
potential (output and incomes will be high and unemployment will be low)

Growth is concerned with increasing Potential GSP

The goal of growth policy is to raise the slope of the potential GDP curve by increasing
productive factors and their technology

Unemployment, Inflation and Growth

Unemployment Rate in Feb. 05 = 5.4% in U.S. and 5.9% in TN

in Jan. 05 = 5.2% in U.S. and 5.9 % in TN

“The number of civilians at least 16 years old who are unemployed and tried to find a
job in the last 4 weeks.” (Understanding Business, p. 53)

Inflation Rate in Feb = 3.0 % in U.S.

in Jan. = 3.0% in U.S.

Involuntary unemployment has individual costs---low income, anxiety, low self respect
and social costs---crime, inequity ( it hits low income workers more)

Hysteresis---high unemployment may perpetuate itself (e.g., some European have


adapted to it)

The cost of inflation is less obvious

Both the public and the Fed have deep antipathy toward inflation

It produces large redistributions of wealth

578 | P a g e
Perception of reduced purchasing power

It engenders uncertainty

It leads to high unemployment

The economic growth question:

How much consumption should we sacrifice today to make investments that will help
the future? (p. 201)

The authors say we need to be making more future-oriented investments (due to


reductions in real incomes and baby boomers retiring)

Stabilization Goals

The federal government promotes maximum employment and stable prices (1946 and
1978 Acts)

There is much disagreement on this but consensus on the fact that unemployment
and inflation are independent in the long run. High inflation does not automatically
lead to low unemployment and low inflation does not automatically lead to high
unemployment---despite the inverse relation shown on p. 203

Remember: Full employment does not mean zero unemployment

Frictional unemployed---people who quit because they didn’t like the job

Structural unemployed---restructuring of firms or a mismatch of skills & jobs

Cyclical unemployed---due to business cycles---the most serious

Full employment means the demand and supply for labor are in balance. Therefore,
the wage inflation stays at the same rate.

So --- full employment is the lowest unemployed rate attainable without continually
rising inflation, also called the non-accelerating inflation rate of unemployment
(NAIRU)

Potential GDP means the output that would be produced at the NAIRU
579 | P a g e
If inflation and unemployment are independent --- why not pursue zero infl? --- p. 204

Probably because it would lead to severe dampening of the economy. (?)

The authors observe that most experts pursue a modest infl rate of 2-3%

Stabilization Policy

We need stabilization policy because (i) aggregate demand does not rise and fall with
full employment, and (ii) adjustments of employment and inflation to low demand is
slow.

(i) Consumer spending represents 2/3 of aggregate demand. And it shifts to reflect
changing sentiment about jobs, wages, etc. Public demand also changes as gov
spending changes (e.g., the reduction in military spending after WWII led to a
recession despite private growth)

Wages are “sticky downward”---they adjust slowly---so when demand falls, wages
remain high leading to involuntary unemployment

Inflation can only be reduced by large reductions in demand---p. 206

This requires higher unemployment and lower output---measured by the sacrifice ratio
(the fraction of a year’s potential GDP that must be sacrificed to reduce inflation 1 %---
about 4%.)

Using Fiscal Policy to Stabilize the Economy

When the economy weakens, the deficit automatically increases and when the
economy improves the deficit automatically shrinks.

Discretionary changes in fiscal policy are different. For example, the difference
between the structural deficit (which assumes high employment and is caused by the
“structure” of tax and spending programs) and the actual deficit is called the cyclical
deficit. (p. 207)

580 | P a g e
If the economy is weak and GDP is below potential the actual deficit will be larger than
the structural deficit. If the economy is strong and GDP is above potential, the actual
deficit will be smaller than the structural deficit.

Automatic stabilizers dampen the effect of changes in the budget. These include:

 Taxes
 Unemployment compensation
 Food stamps
 Medicaid

So---if military spending is reduced the secondary effects on workers will be


moderated because they will have less income on which to pay taxes and they will
receive these benefits. The government budget automatically absorbs part of the
decline in their income.

Discretionary Stabilization Policy

Rather than merely dampening the effects of changes in aggregate demand through
automatic stabilizers, discretionary changes in policy may completely reverse them.
For example, a big surge in demand may be offset by restrictive fiscal or monetary
policy (such as lower spending or higher interest rates) or a drop in demand may be
offset by expansionary policy..

This was the view 30-40 years ago. Today the experts question how effective this
approach really is. The idea that the government could (and must) step in and
stimulate the economy (refined Keynesian theory) was very popular from the mid ‘40’s
to the late ‘60’s.

But this optimism was overtaken by events:

 “Stagflation” in the late ‘60’s---high infl + high unempl (the misery index which
hit 21% in 1980)
 Oil crisis in 1973
 Grain shortages

Edward Phelps and Milton Friedman theory:

581 | P a g e
Unemployment and inflation are independent in the long run. If government policies
hold unemployment below its natural (or “equilibrium”) point based on supply
and demand of labor, inflation will go up continuously. This was the cruel dilemma---
by trying to help, the government was making things worse!

We learned that forecasting future economic conditions is very difficult

The doctrine of rational expectations---stabilization policies by the gov won’t work


because individuals will adjust their behavior in anticipation of them

Time lags between awareness of the problem and ultimate effects of the policy

Inside time lags---within the policy making process

Outside time lags---in the economy, itself

Monetary policy---short inside lags---the Fed meets every 6 weeks long outside lags---
the impact of interest rate changes on the economy may take a long time

Fiscal policy---long inside lags because legislation must be passed (the pol change may
not take effect until into the next economic cycle!) short outside lags because spending
changes quickly affect the economy

Politics play a big role in fiscal policy

Globalization has made fiscal policy less effective (e.g., expansionary fiscal policy
raises demand so interest rates go up; this attracts foreign capital causing the value of
the dollar to go up making imports less expensive which causes demand to fall)

Discretionary fiscal policy ceased being effective as an expansionary tool in the ‘80’s
and early ‘90’s due to large structural budget deficits. Monetary policy---actions by
the Fed---became the way to stabilize the economy (p. 213)

Thus, the main role of fiscal policy has now become economic growth rather than
stabilization

Growth Policy

582 | P a g e
Economic growth looks back (to the sharp slowdown in productivity in the ‘70’s) and
forward (to the impact of the baby boomers retiring).

Productive capacity is determined by how efficiently and effectively the factor inputs of

Labor hours

Human capital---education and skills

Physical capital---machines and structures

Can be combined to produce finished goods and services. Thus, the technology which
is used to bring them together is most important factor.

(Some economists argue knowledge is most important---?)

A big question is why has productivity dropped so much in the last few decades?---p.
215

Growth policy includes strategies to increase physical capital, human capital or


technology.

Using Fiscal and Budget Policies to Increase Economic Growth

Fiscal policy---impact private capital formation (it addresses the size of the deficit
thereby affecting the amount of national savings
available for private investment)

Budget policy---impact public capital formation (it alters the composition of taxes and
spending)

National growth policy in recent years has focused on deficit reduction and increasing
private capital. Deficit reduction reallocates resources from public use to private
investment. But the reallocation is not dollar for dollar.---p. 216

This pol is due to(i) the political appeal of balancing the budget, (ii) a broad consensus
that this will work, and (iii) a concern that private savings has fallen so much in recent
decades.

583 | P a g e
See p. 217---public “dissaving” has increased in recent decades due to large deficits

In the last decade (’87-’97) public saving increased

But the outlook for the deficit and improved saving remains
unfavorable due to the explosion in retirement and health care as BB’s retire

This proved to be true---9/11!

Budget Policies

National output, federal expenditures, federal tax policy and federal budget policies
focus on two things:

Consumption today OR investment to enhance growth in the future.

Budget policies can affect growth in different ways:

1. Redirect public expenditures from consumption to investment (e.g., highways,


aviation, water, environmental programs---a concern is that programs of this type are
politically motivated)

2. Tax policies (e.g., investment tax credit, capital gains, housing---two problems:
excessive demand may result from these policies causing interest rates to go up AND
these policies may reduce public saving because money has gone into private saving---
the lesson is that the budget policy may requires a coordinated fis pol to be effective)

3. Focus on human capital --- education and training are very important in
economic growth

4. Invest public funds in technology---private investment in R& D is not as


efficient

Social returns from R & D = 50%

Private returns from R & D = 20-30%

For this reason the private sector under-invests in R & D.

584 | P a g e
The public sector should increase its investment in R & D---especially in basic
research

5. Non budgetary instruments also affect economic growth (e.g., regulatory and
trade policy)

Coordination of Fiscal and Monetary Policy

The fiscal policy of the government affects the monetary policy adopted by the Fed
much more than the Fed’s policy affects the government.

They are theoretically independent but the Fed watches the government very closely
and they frequently are coordinated to pursue a single purpose (to expand or restrain
the economy).

When more than one goal is pursued at the same time, this coordination is complex.

(The Tinbergen Rule---pursuing multiple policy goals requires an equivalent number of


policy instruments)

An expansive fiscal policy (with large deficits) requires a tight monetary policy (with
increasing interest rates) to hold down inflation-p. 222

This describes late 2004 and early 2005!

Seven ¼ point increases in short term rates in 6 months

The president, congress and the Fed have different policy goals

The Fed usually worries more about the long term so it is the “inflation fighter”

Politics is always involved

See p. 223---Has the traditional role of Republicans and Democrats been


reversed?

Conclusion

1. The budget can be an effective tool to promote long term economic growth.

585 | P a g e
2. Through automatic stabilizers the budget moderates fluctuations in economic
activity.

3. Discretionary fiscal policy is not effective to bring about short term stabilization
(it may even be counterproductive) so monetary policy has become our main way to
stabilize the economy.

State and Local Budgeting: Coping with the Business Cycle M. Wolkoff

Handbook of Government Budgeting, pp. 178-196 (1999)

State and local officials can do little to influence the economic winds that blow sharply
across their bow. P. 179

But these “winds” are very important to them because:

48 state constitutions require a balanced budget

Local and state governments must maintain a good bond rating to incur debt

Local and state governments have less flexibility than fed budget makers
(interest rates and fiscal policy are controlled by the fed gov)

Economic Cycles

All economists agree that the US economy goes through cycles of expansion and
contraction.

From 1970-1996 there were four such periods---see p. 180

Different regions deal with national economic cycles differently due to different

Industrial composition

Access to markets

Raw materials

Historical happenstance

Technology may make these differences less important


586 | P a g e
See p. 182--- 1989 to 1995 unemployment rates in the 10 largest metropolitan areas
varied greatly---above and below the national average each year

See p. 183---this model shows statistically that local unemployment rates ARE
impacted by changes in the national unemployment rate

And that the rates in different SMSAs are impacted differently

For example, a 3 % increase in the national UR will cause a 6% increase in Boston but
only a 1% increase in Atlanta.

See p. 185---shows employment shares by industry in the 10 largest metropolitan


areas which lead to different experiences as national economic conditions

Linking Economic Cycles to the public Fisc

State and local fiscal fortune is directly linked to the national economy.

When times are good, people are working and the revenue coffers are full.

When times are bad, people need more help from the gov but coffers are empty.

Some jurisdictions are more sensitive to cyclical changes in the national economy

due to differences in their

1. Industrial structure

2. Expenditure responsibilities

3. Composition of their revenue sources (e.g., income taxes are very sensitive to
changes in economic conditions while property taxes are less sensitive)---see p. 187
for breakdown

For years the conventional wisdom was that state and local govs had to choose
between growth (income taxes) or stability (property taxes) ( ? ) --- p. 186

BUT now experts say that a combination of short term stability and long term growth
is possible. Taxes that have higher long-term income elasticities (income taxes) need

587 | P a g e
not vary more over the business cycle than taxes with lower elasticities (property
taxes).

Dealing With Budgetary Imbalance

State and local govs have a limited number of options for dealing with budget deficits

1. Pessimistic Budget Estimates

By assuming the least favorable economic circumstances, deficits due to


cyclical changes will be avoided and the budget will always be balanced. But
there will never be enough money and services will be far below what the
citizenry demands

Problems with this approach:

It gives much power to the revenue estimators

The level of services provided is less than what is sustainable (and needed!)

Surplus almost guaranteed-giving great power to the executive & incentive to


the legislature not to be conservative

The dashed line represents economic cycle

SS is a budget in structural surplus---deficits are rare due to pessimistic


forecasts

SD is a budget in structural deficit---surpluses are rare due to optimistic


forecasts

SB is a budget in structural balance---the cyclical deviations are symmetrical

2. Rainy Day Funds

A percentage of the total budget is set aside and not spent---frequently 5%

It is saved for future economic hard times

How do you decide when to add to it or to dip into it?

588 | P a g e
Executive decision

Legislative decision

Formula---e.g., Michigan (Wolkoff likes this best)

3. Financial Legerdemain

Unsound accounting tricks---“slight-of-hand”

Redefine the fiscal year, or

Show revenue or expenditures in another period

These are temporary fixes ---merely putting off the problem

Move operating expenses to the capital budget

Sell assets

Use money for other purposes

4. Contingency Spending and Taxing Plans

These are tax or spend policies automatically triggered by economic events (e.g., if
revenue collections are down, a tax increase kicks in OR spending is reduced,
automatically)

Problem with this approach---it can exacerbate local cyclical effects

Remember the impact of mundane, day-to-day decisions on the budget (e.g., leaving a
position vacant lowers the level of services)

Endogenous Strategies

State & local govs have three generic strategies to get off the cyclical roller coaster:

1. Revenue structure---chose sources that are less cyclically prone or that offset
each other (e.g., combine income and property taxes)

589 | P a g e
2. Use countercyclical policies---economists now think decentralized stabilization
policies may work, after all because local multipliers are larger and borders are less
“leaky” than Musgrave thought---p. 194

3. Alter the growth path---this is the flip side of Larkey & Smith’s pessimistic
budgeting---use optimistic assumptions that the economy will grow its way out of the
cycle

Problem---this requires restraint on spending

Conclusion

These strategies for dealing with economic cycles can be applied to other challenges to
the budget, such as the need for snow removal or for police overtime.

All of these are difficult to predict and exhibit year-to-year variation in budget
requirements.

Flexibility is the key!

The Growing Fiscal and Economic Importance of State and Local Governments by R.
Bahl

Government Budgeting: Theory, Process and Politics. 3rd ed. Pp. 363-379 (1984)

State and local gov = 13% of GNP in 1981 and 50% of total gov spending

What is the “proper” balance between fed and state and local gov in the U.S.?

Distribution---are subnational units big enough to influence the interpersonal


distribution of income?

Stabilization---can counter-cyclical behavior by state & local gov compromise federal


macroeconomic policy?

Allocation---is state & local gov too large (thereby discouraging private investment)? Is
it too small (not providing adequate services)?

From 1942-1976 there were four major trends:

590 | P a g e
1. The state & local sector became more important in the national economy

2. Public spending shifted to health, ed and welfare (big increases in SS)

3. State & local govs became more dependent on federal transfers of money

4. State gov has become very dominant in state & local sector

Since 1976 these trends 1-3 have been reversed.

1. The national sector became more important

2. Spending moved away from health, education and welfare

See p. 366---60% of expenditure increase from1960-76 was for hew

56% of expenditure increase from 1976-81 was for hew

(Is this decrease significant? Is five years long enough?)

3. There was less dependence on federal money

These conclusions are based on measurements of employment and expenditures

From 1954 to 1980 state and local gov employment increased 174%
fed gov employment increase only 22%

From 1959 to 1984 the fed share of local spending increased from 40% to 50%

The Economic Role of Subnational Governments

Conventional thought says that of the three functions of public budgets (stabilization,
distribution and allocation) only allocation is proper for state & local govs.

National policy is needed to stabilize the economy because of “leakages” from the state
economy (i.e., spending on goods produced in other states).

And to distribute income due to mobility (high income individuals may migrate to
avoid taxes and low income individuals may migrate to gain benefits).

591 | P a g e
Thus allocation---deciding which and how much local services will be provided and
how they will be paid for---is the proper function of subnational govs. It is also more
efficient because the preferences of local voters can be considered.

But Bahl questions this conventional thought and argues that subnational govs can
have an important impact on stabilizing the economy and in distributing income.

While subnational govs are ill-equipped to set their own growth and stabilization
policies, their reaction to national policies must be considered. Two issues are
important:

 Whether state & local budgetary decisions hurt or help federal stabilization
programs?

The studies on this question show mixed results.

Bahl’s conclusion is that “the fiscal actions of state & local govs have to
be reckoned with in the formulation of national economic policy. The
sector is simply too large a share of GNP to be ignored.” (???)

 Whether federal programs which stimulate the state & local sector result in
unintended changes in the federal fiscal system?

Yes -two times in five years.

Bahl argues that state & local budgets play an important role in
distributing income

He bases this on the size of state & local gov (40% of nondefense gov
spending and 13% of GNP)

State &local gov influence distribution of income through taxes, public assistance.

(He notes that low income families have not migrated to high payment areas).

The redistributive effect of taxes is probably less than of expenditures.

What we need from the research is the “net fiscal incidence” that measures the
benefits of expenditures and the burdens of taxes on the distribution of income.
592 | P a g e
Hard research on this question does not offer a clear answer.

But Bahl’s hypothesis is that the poor do not fare very well. They are hit by taxes, not
always offset by benefits.

Even if the research is not clear that state & local govs actually affect redistribution of
income, they express concern about it.

Is the State and Local Government Sector Too Large?

There are no guidelines on the optimal size of gov from political nor economic theory.

Three possible considerations:

1. Gov is too small compared with other industrialized countries. Bahl says this is
not a fair comparison because we do not offer the same package of public
services, and other countries are so small (the overall tax burden varies greatly
among states in the U.S.--- e.g., NY)
2. Gov is too large because it interferes with the market and retards growth. The
Laffer Curve theorizes an inverse relationship between tax rates and
government revenues---research does not support this but Bahl agrees that
high tax rates are a disincentive to capital formation which can be addressed by
changing the tax system or reducing the size of government (Is this very likely?)
3. Gov is too small because it has not corrected the unequal distribution of
income. It is argued that there is a direct relationship between the size of the
public sector and the distribution of income. In Europe there is more
redistribution of income than in the U.S. and the larger governments are more
involved in social insurance and welfare.

The conclusion---the size of the government in the U.S. should be increased to


improve income distribution.

Bahl says increasing the size of government will not likely have this effect

Also the mood in the nation seems to be toward smaller government

593 | P a g e
Q – 7: Other than Performance-Based Budgeting, what are the most important recent
developments in financial management? Why are they important and what
precipitated them? Explain how these recent changes may/can influence the work of
public managers and policy makers

Indeed, performance based budgeting has been a significant development in financial


management. The initial performance based budgets were developed in the 1940s as a
result of the Brownlow and Hoover Commissions. These budgets were management-
oriented systems heavily focused on efficiency by relating costs to measured outputs.
In the 1960s Planning-Programming Budget Systems (PPBS) emerged emphasizing
multiyear planning, policy analysis, and program objectives focusing on effectiveness.
PPBS was discontinued in government in the 1970s during a period in which the
executive and legislative branches battled over who controlled the budget. Then in
1977, under the Carter administration, Zero Based Budgeting was implemented in the
federal government. ZBB was to replace the traditional incremental (line item)
budgeting systems. It was overwhelmed by changing economic events and in 19981
was rescinded by the Reagan administration. Budget deficits were the budget focus of
the 80s and the passage of the Balanced Budget and Emergency Deficit Control Act in
1985 (Gramm-Rudman-Hollings Act) began a lengthy era of dealing with deficits. With
the passage of the Government Performance Review Act in 1993, we see the emergence
of a new performance budgeting system more concentrated on outcomes. This form of
budgeting dominates both federal and state budgets and continues to exert influence
in budget decisions today. Of significance is that performance budgets, or outcome
budgets, connect the budgeting process with policy formulation, strategic planning,
program goal formulation, and program evaluation. NOTE:

Since the early 1990s with the rapid implementation of performance based budget
there have been some important new developments affecting public budgets. Growing
deficits, the public’s demand for accountability, and a clear rivalry between the
President and Congress as to who should control the budget has precipitated these
developments. Some of these new developments are a result of the passage of new
laws related to public budgeting. These would include:

594 | P a g e
Budget Enforcement Act – 1990 – amended the Gramm-Rudman-Hollings Act and
created a number of procedural requirements such as “pay-as-you-go”. This meant
that after the budget had been passed, any new spending approved by Congress and
the President had to be compensated for by new sources of revenues other spending
reductions.

Credit Reform Act – 1990 – tightened requirements on federal lending and loan
guarantees.

Chief Financial Officers Act – 1990 – added new financial management responsibilities
for federal agencies including creating a CFO position to oversee agency financial
practices.

Omnibus Budget Reconciliation Act – included major reductions in defense spending


and tax increases for upper income groups and no tax cuts for the middle class.

The Line Item Veto (1996 to 1998) – For a two year period, gave the President the
power to veto specific appropriations. Found to be unconstitutional by the U.S.
Supreme Court in 1998.

Performance based budget reforms have focused attention more that ever on sharing
information and decision making in budgeting. Sharing information about what
governments do well, through performance measurement and reporting, can go a long
way in reducing cynicism. Broader participation in trading off various goals and
means to achieve them, as well as in developing performance measures, will yield
better decisions and a sharing of risks among citizens, public managers, and elected
officials.

The GPRA and state and local results acts and executive orders emphasize program
results and holding federal government agencies accountable for them. The GPRA
focuses managers’ attention on setting goals, and reporting publicly on progress made.
One of the foremost purposes of the act is to instill confidence in the public about
federal government managers’ ability to solve problems and meet citizen-taxpayers’
needs. To implement the GRPA, each agency must first develop strategic plans
covering a period of at least five years. The strategic plan must include a mission

595 | P a g e
statement, outcome related measurable goals and objectives, and plans that agency
managers and professionals intend to follow to achieve these goals through their
activities and through their human, capital, information, and other resources. Those
in the agency must consult Congress and others interested in or affected by the plans:
in other words, they must consult stakeholders. The new reforms incorporate most of
the goals of the previous reforms, but they seek to achieve them through decentralized
incentives that give program managers greater authority to combine resources as they
think best but that hold the managers accountable for the results.

Performance based reforms deal widely with organization change. Keep in mind that
no other decision making system has the leverage to pressure departments to improve
program management like the budget. Also, the budget has always been the place
where everyone raises questions of the efficiency, economy, effectiveness, productivity,
impact, and results of government activities. Lastly, budget offices can inspire
agencies to strengthen their programs, operating systems, and organizational
structures.

These new efforts at performance budgets appear to have a promising future; however,
some authors are not so enamored with performance based budgets. Wildavsky warns
that results that are “too good” can have adverse effects in that superb results may
result in no further funds being appropriated, as the goals will have apparently been
achieved. Caiden finds that performance may be irrelevant in making budget
decisions as political popularity and the need to balance budgets are the sole reason
for budget decisions.

For managers of programs that meet their goals and objectives and otherwise exhibit
exception performance, pay-for-performance has emerged as a strategy for responding
to increased productivity of staff. Merit pay is one method, including bonuses,
commissions and formulas for sharing savings. NOTE: For more information on the
use of merit pay, good and bad, refer to: Halachmi, A, Holzer, M. (1987). Merit pay,
performance targeting, and productivity. Review of Public Personnel Administration, 7
(Spring), 80-91.

596 | P a g e
Chapter 5
Public Policy Analysis

MPA Comprehensive Exam


Study Guide
5.1 History of Public Policy
While the study of politics has a long history, the systematic study of public policy, on
the other hand, can be said to be a twentieth century creation. It dates, according to
Daniel McCool, to 1922, when political scientist Charles Merriam sought to connect
the theory and practices of politics to understanding the actual activities of
government - that is public policy." (p.4) (see McCool, Daniel C. Public Policy Theories,
Models, and Concepts: An Anthology. Englewood Cliffs, N.J.: Prentice Hall, 1995.)

The “policy orientation” after 40 years: Review major developments and their
implications for researchers and practitioners.

Harold J. Lasswell (1951) used the action-characterizing term “policy orientation” to


distinguish it from the “policy sciences” in the first chapter of the book he edited with
Daniel Lerner, Policy Sciences: Recent Developments of Scope and Methods (1951).

 “the Praxis of Policy” - interdependence of scholarly disciplines dependent on


the knowledge of policy process and knowledge responding to the particular
intelligences needs surrounding particular policy choices.
 Emphasis on foresight and ‘development constructs’
 The policy orientation, according to Lasswell, is a problem-orientation.
Problems are complex and often involve complex answers drawing from many
disciplines. He clarified the parameters that later became decisive in analyzing
the content of what politics is: analysis must focus on problems; it must be
multi-subject and clearly normative.

Graham(1988): Lasswell was concerned for macro-level analysis and realized that
someone must apply the intelligence in a choice.

The policy sciences were initially conceived by Harold Lasswell (1951) and others as a
means to improve the quality of information to governments, as a means of improving
the governmental decision process (see deLeon, 1988). They were designed to be
problem oriented, multidisciplinary, and explicitly normative (i.e., explicitly
considering values) in their approach. Yet early on, the policy sciences were “captured”
by many of the heavily quantitative disciplines, which sought to bring the putatively

598 | P a g e
“proven powers” of the natural sciences to the social sciences. So it was not unusual
to find systems analysis, operations research, and quantitative modeling providing the
early impetus to policy research, its proponents encouraged by their widely acclaimed
successes from the Second World War. They were succeeded by welfare economists
with their particular “answer” to policy questions, typically framed in terms of cost-
benefit analysis.

As Etzioni (1988) and others have pointed out, however, their influence on public-
policy makers has largely been ancillary, because, in their orientation towards strictly
“objective” analysis (e.g., Stokey and Zeckhauser, 1978), they tended to overlook
(because they could not openly include) the normative bases of politics.

Worse yet, there is a widespread implication that the policy sciences, while widely
accepted, have become instruments protecting the status quo, that their “research,
insofar as it exercises independent influences on opinions about complex social
questions, tends over time to be profoundly conservative in its impact” (Aaron, 1987,
p. 2). As traditionally practiced, the policy sciences have been unable to effect a shift
in the structure or process of governance, because they were widely perceived to have
been co-opted by government offices, programs, and priorities.

More Programs: On the more positive side, there has been tremendous growth in the
policy analytic community, in terms of both supply and demand. On the supply side,
hardly a major university does not now have a program to train incipient policy
analysts, with a large number of schools also offering a doctoral degree. On the
demand side, there is hardly a federal agency that does not have an analytic (or
evaluative) section as part of its organization; increasingly, similar offices are
appearing on the state and local levels.

Participatory Policy Analysis: In recent years, there has been a growing awareness
of this tension within some parts of the policy analysis community. In response,
drawing in part on the writings of the Frankfurt School (see, most notably, Habermas,
1975 and 1996), policy scientists have move to a “postmodernized” policy analysis
(Danziger, 1995) and, more operationally, to a “participatory policy analysis” (Durning,
1993). The impetus of the movement (which, to be fair, hardly represents a consensus;

599 | P a g e
Lynn, 1999) is the observation that citizens perceived their opinions were no longer
important when new programs were devised or revised, and that they were largely
being excluded from the governing process. As a result, not only were they distancing
themselves from government (that is, they viewed policy as being imposed upon them,
all the while, apparently, for them) but also it was clear that the programs being
produced without their voice were nowhere near as successful as could otherwise be
the case (deLeon, 1992).

The participatory policy analysis case is relatively straightforward: Citizens


deserve a greater say in their governance, based upon the knowledge that they are
best able to articulate their special “needs.” Given this voice as a basis for action,
government can be more informed and responsive (or, in many cases, more limited),
thus promoting a more involved, engaged, and, in many perceptions, (see Barber,
1984, and Mansfield, 2001), a better citizenry. The result would be an enhanced
(maybe even a good) society.

600 | P a g e
5.2 Public Policy Defined - Various definitions
According to Thomas A. Birkland in An Introduction to the Policy Process (2001), there
is a lack of a consensus on the definition of public policy. Birkland outlines a few
definitions of public policy (Table 1.3 on p. 21):

Clarke E. Cochran, et al.: "The term public policy always refers to the actions of
government and the intentions that determine those actions".

Clarke E. Cochran, et al.: "Public policy is the outcome of the struggle in government
over who gets what".

Thomas Dye: Public policy is "Whatever governments choose to do or not do".

Charles L. Cochran and Eloise F. Malone: "Public policy consists of political


decisions for implementing programs to achieve societal goals".

B. Guy Peters: "Stated most simply, public policy is the sum of government activities,
whether acting directly or through agents, as it has an influence on the life of citizens".

Birkland indicates that the elements common to all definitions of public policy are as
follows:

 The policy is made in the name of the "public".


 Policy is generally made or initiated by government.
 Policy is interpreted and implemented by public and private actors.
 Policy is what the government intends to do.
 Policy is what the government chooses not to do.

According to Encarta, in its definition of political science, "The field of public policy
involves the study of specific policy problems and governmental responses to them.
Political scientists involved in the study of public policy attempt to devise solutions for
problems of public concern."

601 | P a g e
5.3 Public Policy Analysis outline
Lowi: The End of Liberalism

 Argues why pluralistic interest group liberalism does not fulfill the goals of the
constitution
 Interest-group liberalism (IGL) is a new public and pluralistic philosophy, which
began in the depression and solidified in the 1960s. It expects that competing
interest groups represent the people’s will to government decision makers
through competitive group interaction.
 Lowi finds fault in IGL in that like automatic regulation in economics (laissez-
faire) is imperfect, so is automatic regulation imperfect in group dynamics
(pluralistic). He also finds that IGL idealizes the conception of groups.

The results of IGL are

 The regulated hold power over the regulators (Capture theory).


 It removes legitimacy from the representative government.
 This loss of legitimacy removes the ability to plan because it takes authority to
plan.
 Example: U.S. works best in crisis when representative leader is given
authority.
 Bargaining replaces planning and is reactionary.
 Lowi’s solution is a juridical democracy and involves clear and formal law
making from elected representatives.

Three policy streams: economic, domestic, and foreign policy

Policy arenas

 John Kindon: Agendas, Alternatives, and Public Policies


 Kindon’s theory for public policy is called primeval soup
 His primeval soup has three policy streams

Mix of ideas

 Many ideas

602 | P a g e
 Some survive, die, or combine
 Combination of ideas is called coupling
 Combination of streams is also coupling
 Problems with solutions have better chance for survival

James E. Anderson: Public Policy Making: An Introduction

 Scientific policy approach


 Rational
 Ordered approach
 Ideal type
 Definitions of policy
 Public policy cycle
 Problem identification and agenda setting
 Policy environment
 Officials
 Unofficial
 Political culture
 Levels of politics
 Iron triangle
 Congressional committees
 Administrative agencies (bureaus)
 Interest groups

Charles E. Lindblom & Edward J. Woodhouse: The Policy Making Process

 Improved analysis will improve policy


 Improved intelligence will improve policy
 Movement toward public equity of voice in policy
 Definition of public policy

Deborah Stone: Policy Paradox and Political Reason

 Policy making rationality – Three pillars


 Market verse Polis
603 | P a g e
 Market vs polis value tradeoffs

Model Definition/Characteristics Traditional PA Class Theorists


Theorists

Institutionalism Policy as institutional Lowi (Moderate)–


From: output Lowi’s greatest
organization Relationship between concern is the
theory structure and policy structure of
PA policy view: outputs government. Rule
moderate to Characteristics from of law to avoid
weak. government capture theory.
Legitimacy Kingdon (Weak)
Universality Anderson (Weak)
Coersion Lindblom (Weak)
Questions Dye (Weak)
What impact does division
of responsibility have on
policy
What impact do levels of
government have on policy

Process Policy as political activity Anderson (Main –


From: Political Patterns of activity – How Well defined)
Science or decisions are made (not Kingdon (Weak)
Public content) Lindblom (Weak)
Administration Implementation is one Dye (Weak)
PA policy view: activity in the policy
moderate process ?? six tenants

Rationalism Policy as maximum social Taylor Anderson


From: (From gain (Moderate)
closed system Strict cost/benefit analysis Stone (Moderate)
business based on rationality But, her definition
model) Comprehensive is different than
PA policy view: Exhaustive knowledge of traditional modern
moderate options rationalism.
Exhaustive knowledge of Lowi (Weak)
consequences Kingdon (Weak)
An ideal – not attainable Lindblom (Weak)
Limitations of rationality Dye (Weak)
Not attainable
Ineffective – constraints of
time, information, and cost
(Janice said inefficient. I
think it is more a result of
not meeting human nature

604 | P a g e
accurately.)

Incrementalism Policy as variations on the Charles Lindblom (Main –


From: Political past Lindblom Well defined)
Science – Conservative approach to “The Science of Lowi (Weak)
Political model policy change Muddling Kingdon (Main –
PA policy view: Why incrementalism is Through” Well defined:
Strong prevalent. Public Incremental,
Rationalism is inefficient Administration Mixed, & Garbage)
Legitamacy of previous Review 19:79- Anderson
policies accepted due to 88. (Moderate)
uncertainty of Stone (Weak)
consequences of radically Dye (Moderate)
different policies.
Sunk costs in current
policies.
Politically expedient – no
boat rocking
Not thinking outside the
box

Group Theory Policy as group equilibrium Stone (Main – Not


From: Political Policy is outcome of group well defined)
Science / struggle Stone’s theory is
Sociology Characteristics the polis model.
(political model) Bargaining She is against the
PA policy view: Negotiation market model.
Moderate Compromising Kingdon (Moderate
Competing interests )
(Pluralism) Anderson
(Moderate) Agenda
setting.
Lindblom
(Moderate)
Dye (Moderate)
Lowi (Weak)

Elite Theory Policy as elite preference Lowi (Main, Not


From: Political Policy reflects the interests well defined)– Lowi
Science and value of elites (not is concerned with
(political model) general public) the loss of
PA policy view: Change in policy results authority from
Moderate from change in elite values U.S.
representative
political elites but
is more concerned
with structure

605 | P a g e
(institutionalism).
Lowi talks of the
market and polis
as the 1st two
republics. He
suggests a return
to the
constitution.
Stone (Moderate)
From an anti-
position
Dye (Moderate)

Public Choice Policy as collective decision Ostrom, Stone (Weak)


From: making by self-interested Buchanan Dye (Weak)
Economic individuals
PA policy view: Govt. must provide public
Weak goods
Externalities – imposition of
some on many (air
pollution)
Market approach of selfish
actors.

Game Theory Policy as rational choice in Kingdon (Weak)


From: Political competitive situations Stone (Moderate)
Science Choices portrayed in a From an anti
PA policy view: matrix position.
Weak Payoff – the values each Dye (Weak)
player places on each
outcome
Each player must try to
calculate the values of the
other (You must know the
mindset – the culture – of
the opponent)
Strategy is key – “minimax”
– to protect against the
opponents best play

Systems Model Policy as system output Dye (Weak)


From: The concept of system
Organization implies
theory Identifiable set of
PA policy view: institutions and activities in
Weak society that function to
transform demands into

606 | P a g e
authoritative decisions
requiring the support of the
whole society.
Elements of the system are
interrelated
Model
Input
Black box (organization)
Output
Feedback
Within the environment

Mixed Policy as a combination of Etzioni (? Kingdon (Main –


Scanning incrementalism and Spelling) You Well defined:
From: Political rationalism. must stress Incremental,
Science This is not one of Dye’s rational to me Mixed, & Garbage)
(political model models mixed as a Anderson
- Political with Why is this different from main category. (Moderate)
attempt to incrementalism? Lindblom Lowi (Weak)
explain itself to Incrementalism requires comes close but Stone (Moderate)
a rational limited rationality. So, does not make Lindblom
public) mixed scanning highlights a strong case (Moderate)
PA policy view: the limited rationality for rationality. Lindblom comes
Moderate inherent. I believe I am His rationality close to stressing
starting from a too is just an rationality but
“rational” mindset and can’t adoption of does not make a
see this one. some rational strong case for
Incrementalism combined practices into rationality. His
with other theories that are incrementalism. rationality is just
political would be just an adoption of
incrementalism, but if it some rational
had some rationality call it practices into
mixed scanning. I am incrementalism.
getting punchy. Help me
out here.

Garbage Can / Policy as an incoherent Cohen and Kingdon (Main –


Primordial mess: unclear goals, March Well defined:
Soup imperfect technology, Incremental,
From: history not understood, and Mixed, & Garbage)
Organization participants wondering in Kingdon has three
theory and out. policy streams
PA policy view: This is not one of Dye’s that open up a
Moderate models policy window:
Garbage Can – Cohen and problem, politics,
(Political with March and policy.
attempt to Everything goes into a Kingdon’s solution

607 | P a g e
explain itself to garbage can and those has two elements.
a rational things with political One, it is
public) strength come out and are patterned:
placed into policy. technically
Primordial Soup – Kingdon feasible,
Three streams guide what researched,
is born into policy implementable,
These two theories differ administratable).
because the Garbage Can Two, it meets with
theory does not have an societal values.
optimistic perspective, Lindblom
while the Primordial Soup (Moderate) The
theory does. The negative tone
Primordial Soup although a toward decision
like goulash does have making matches
elements that make it more garbage can.
structured.
Are these theories
incoherent process
theories? We did not put
Kingdon down as a process
theorist.

608 | P a g e
5.4 Definition of budget In “Public Policymaking”
In “Public Policymaking”, by James E. Anderson, the definition of budget provided is:
“A statement of estimated revenues and proposed expenditures; it is also a policy
statement and a political document”. With regard to question 1: “Explain how the
budget can be described as an expression of preferences or conflicting values, what do
we mean by the ‘budget’?”, the important points are that the budget is a “policy
statement” and a “political document”. In speaking about the federal budget, citizens,
special interest groups, legislators, agencies and the executive branch all have
preferences in terms of policies that they wish to be funded and included in the
budget. Through a complex process of negotiations, pass-backs, and compromising, a
set of policies that will be funded (the budget) is created. It is absolutely true that the
budget document (at any level of government) is the most important document in
public administration. The actors involved in budgetary activities within an agency or
organization are usually perceived as highly-important and their offices considered
repositories of great power in the organization. One of the biggest problems regarding
the budget is that there is never enough money for every individual actor to get what
they want. This is where it becomes evident that the budget is a “political document”.
The actors and committees involved in authorizing and appropriating funds hold
immense political power because their influence can make or break a program, as well
as significantly affect the operations of agencies throughout government. Congress
held the lion’s share of this power until 1921, when the federal budget process
switched to “Presidential budgeting”. Presidential control over the budget process
dominated until the Nixon years, when the general feeling took hold that the budget
was out of control. Even after the Budget Enforcement Act of 1990, when the
legislative committees were separated from the appropriations committees, political
gaming of the system still dominated the budget process and remains so up to this
day. I believe it is true that “there is no one document that contains “the budget”.
This is because, as we discussed in class, no matter how carefully the parties lay out
the expected revenues and expenditures, there will always be surprises, emergencies,
disasters and political maneuvers that change the budgetary figures and the programs
funded in it. Therefore, there literally can be no “finished” document, because it is
constantly changing and adapting to the political and real-world environment.

609 | P a g e
5.5 Public Policy in Detail
POLICY INSTRUMENTS: Forms of government intervention (as opposed to market-
style actions).

REGULATORY INSTRUMENTS – Regulations

ECONOMIC INSTRUMENTS – Subsidies, grants

INFORMATION INSTRUMENTS – Used to persuade citizenry in a certain direction.

POLICY EVALUATION/PERFORMANCE EVALUATION:

CRITERIA FOR GOOD GOVERNANCE (Bemelmans-Videc 1998):

 Criterion of DEMOCRACY – concerned with the degree to which public action is


in line with accepted values and norms.
 Criterion of LEGALITY
o EFFICIENCY
o EFFECTIVENESS

PROBLEMS WITH POLICY ANALYSIS (RAAD I):

 Much of policy analysis focuses on MEASURABLE CRITERIA.


 Most policy analysis focuses on evaluating the realization of operational short-
term goals.
 Policy analysts claim to generate usable knowledge, but this is false since most
policies and policy successes are short-term and one-sided (No long-term
perspective = no generalizable usable knowledge).
 Policy evaluation excludes one important arena – POLITICS.

610 | P a g e
5.6 Public Policy Implementation
TOP-DOWN AUTHORS

PRESSMAN & WILDAVSKY – Followed a RATIONAL MODEL APPROACH. Studied


implementation of a federal econ development proj in Oakland. Pessimistic in
tone.

 Discovered that implementation goes thru numerous agencies – implementation


becomes more difficult the more places it has to go thru. “Everyone is in
charge, no one is in charge”.
 Policy objectives are set by central policymakers.
 Implementation implied adequate bureaucratic procedures to execute policies
accurately.

VAN METER & VAN HORN – Investigated relationship between outcomes and
initial policy decisions.

 Big changes are difficult.


 Significant policy change is only possible if goal consensus is high.

SABATIER & MAZMANIAN – Top-Down. Assumed a clear separation of policy


formation from implementation.

 Established criteria for effective implementation.


 Objectives are clear and consistent
 Implementation process is structured adequately
 Officials are committed to program goals
 Interest groups are supportive.

BOTTOM-UP AUTHORS:

LIPSKY – STREET LEVEL BUREAUCRATS

 LIPSKY justified the fact that those who have direct contact with service delivery
are important.

611 | P a g e
 Control should be with central command, but should INCLUDE input from
street level bureaucrats.

ELMORE – Questioned how to study implementation. “Don’t assume that policy


makers control implementation.”

 Don’t bind the street-level bureaucrat. – Suggested BACKWARD MAPPING of


policy thru system.
 The further away from street-level policy is made, the less chance for a
successful outcome.
 Later became top-down and suggested FORWARD MAPPING along with
Backward Mapping.

HJERN & HULL – First to introduce IMPLEMENTATION NETWORKS – Where


policy execution takes place.

Analysis starts with identifying implementation networks and the way they solve
problems.

WILDAVSKY – Previously Top-Down author.

Implementation starts with central policy makers but these inputs will be changed in
their execution.

KINGDON’S MODEL OF POLICY MAKING:

John KINGDON (1984, 1995) –

 Multiple Decision Streams - Streams of problems, policies and politics coalesce


at critical junctures, opening windows for solutions.
 Policies get made into law by capitalizing on “WINDOWS” of opportunity when
the three streams (problems, policies and politics) come together. TIMING IS
EVERYTHING.
 Policy agendas are affected by active participants, processes and alternatives
becoming visible.
 Much of policy analysis has focused on the theoretical problem of agenda
building and policy development.
612 | P a g e
 What moves social problems onto the political agenda?
o Problem urgency
o Policy expertise
o Political situation:
 If all three areas merge, there is a POLICY WINDOW where action
can happen.
 RAAD says Kingdon’s model is limited because policy changes can
only be evaluated after they happen and thus, this model is not
useful for predicting what policies will go through.
 Kingdon recognizes that once a social problem is being addressed
there are a variety of factors that may prevent a solution:
Procedures, interpretation, budget constraints, etc.

LINDBLOM (1959) – The Science of “Muddling Through”.

 INCREMENTALISM

Aaron WILDAVSKY – The Politics of the Budgetary Process (HE APPLIES


LINDBLOM’S IDEAS TO BUDGETING)

 The “best known expression of INCREMENTALISM IN BUDGETING –


WILDAVSKY.
 Wildavsky says that KEY’S question is unanswerable without a comprehensive
and specific NORMATIVE theory of politics – which he thinks is impossible.
 The budgetary process is human behavior in a government setting.
 The political environment of budgeting limits choices.
 Being a good politician is the best strategy
 Traditional budgeting (INCREMENTAL) is superior to rational budgeting.

WILDAVSKY & CAIDEN – Entitlement Budgeting

 Wildavsky & Caiden note that less than 20% of the federal budget is really up
for debate, as well over half is entitlement spending.

PRESSMAN and WILDAVSKY (1973) Implementation.

613 | P a g e
 Classic study of federal programs in Oakland made implementation a new focus
for PA.
 Earlier policy planning and analysis did not consider difficulties of
implementation.
 Discovered that implementation goes thru numerous agencies – implementation
becomes more difficult the more places it has to go thru. “Everyone is in
charge, no one is in charge”.
 Policy objectives are set by central policymakers.
 Implementation implied adequate bureaucratic procedures to execute policies
accurately.

DROR – Policy Gambling: Every policy is a RISK and may not work, but it is important
to TRY and Experiment.

SELZNICK – (1949) – The Cooperative Mechanism.

 Gave us the concept of COOPTATION in policy making and human relations in


the study of the workplace.
 Studied cooptation processes at TVA – trying to gain community support –
included them in everything.

SCHICK – Criticism of Incrementalism.

BACHRACH & BARATZ – FOUR BARRIERS that have to be overcome for any policy
to be successful:

 Belief system of the population


 Procedures
 Implementation
 Administrative interpretation

LOWI – POLICIES DETERMINE POLITICS:

 Lowi’s Four Types of Policies:


o Distributive (e.g. health research, national parks)
o Regulatory (e.g. criminal, business)

614 | P a g e
o Redistributive (e.g. welfare, housing, health care)
o Constituent (e.g. benefit govt or country as a whole – army, secret
service)

MAZMANIAN & SABATIER (1983) –

 Focused on the Policy Implementation process. Considered the variables that


affect the achievement of legal objectives, such concerns as the tractability of
the problem, the ability to structure implementation, and other variables
affecting support for a particular program.

Thomas DYE - “Public Policy is what governments choose to do or not to do.”

 Decision making models help us clarify thinking. Models help us provide


explanations for policy and predict consequences.

Thomas DYE & ZEIGLER – ELITE THEORY

 Public policy reflects the values of a governing elite.


 The masses do not determine public policy through their demands
 The masses are largely passive and ill-informed.

David EASTON – Code of Relevance

 Politicization of the Professions is inescapable and desirable because it drives


the study of real current political problems.
 Research is NOT value-neutral. You look for the data to backup your position.
 Responsibility to protect human values.

Michael LIPSKY – (1980) Street-Level Bureaucracy.

 Lipsky first brought the term “street-level bureaucracy” to the attention of the
field by pointing out that public policy is determined not merely by legislators
and managers at high levels of government but by the police officer, the nurse,
and the welfare worker (among others), who engage in the direct delivery of
services.

615 | P a g e
 Lipsky writes that “the decisions of the street-level bureaucrats, the routines
they establish, and the devices they invent to cope with uncertainties and work
pressures, effectively become the public policies they carry out.”
 Finding that systemic constraints on the professional practices of street-level
bureaucrats result in confusion and conflicting demands, Lipsky offers several
important suggestions for reform, suggestions addressed to the concerns of
street-level bureaucrats but not without relevance to those at all levels of public
agencies.

Paul LIGHT – (2000) in RAAD I p. 273-275– Light conducted research on the fifty
greatest achievements of federal government in the second half of the twentieth
century, based on a survey of professors of history and govt. Lessons learned:

 Most federal policies were the outcome of policy making over a long period of
time (Lindblom – this supports the idea that most policy making is incremental
by nature). Exceptions are breakthrough policies such as Medicare and Welfare
reform.

 More than 80% of these achievements were the result of bipartisan


commitment.

 Most of these achievements involved a mix of policy strategies (federal spending,


regulatory policies, distributive).

Light argues that we are not likely to see such bold policies as those found in the
analysis above. This is because the nation’s leaders are so worried about losing their
jobs that they will not take risks. Americans are so impatient for success that no
program, however well designed and justified, can outlast the early difficulties that
face so many innovative efforts. The media are so addicted to stories of government
failure that no endeavor, however noble and well designed, can survive long enough to
achieve results. Many of the most important problems addressed in the top ten
policies above are still in need of a solution (health care, nuclear war, improving air
and water quality, reducing hunger, etc.). To the extent that the nation’s leaders avoid
the risky issues in favor of safe rewards, the public demands instant gratification
instead of long-term diligence, and the media punishes the trial and error so essential

616 | P a g e
to ultimate impact, the list of government’s greatest achievements of the next half
century will be short, indeed (Light, 2000).

Paul LIGHT – (1997) The Tides of Reform.

 The book is based on the notion that there is not too little management
reform in govt, but TOO MUCH. Congress and the Presidency have moved
effortlessly from one reform philosophy to another and back again, rarely
questioning the contradictions and consequences of each separate act. Light
used the Congressional Quarterly Almanac to identify and analyze 141 federal
management statutes signed into law from 1945 to 1994.
 Four separate philosophies or Tides of Reform:
o Scientific Management – efficiency, principles of administration,
experts, executive control (Brownlow Committee & First Hoover
Commission).
o War on Waste – economy, audits, investigations, generally accepted
practices (e.g. welfare fraud hearings).
o Watchful Eye – Goal: fairness, rights, information, whistle-blowers,
congress and the courts (e.g. Vietnam & Watergate).
o Liberation Management – Goal: Higher performance. Standards,
evaluation, outcomes, employees, teams (e.g. Gore’s National
Performance Review)

PETERS (1993) – PICKET-FENCE FEDERALISM.

KUYPER’S THREE-LEVEL POLICY ANALYSIS:

Ultimate Goals

Intermediate Goals

Instruments in the strict sense

617 | P a g e
 In his view, policy making and planning can be mapped as a GOAL TREE in
which both travel up and down between pure instruments and the ultimate
goals.

618 | P a g e
5.7 The Science of "Muddling Through"
1. There are two approaches public administrators use in the decision making
process to compare and ultimately choose among policy alternatives.
a. Rational-comprehensive model (Root Method)
 More mechanical process of choosing means that best satisfy goals
 Goals are ranked and clarified previous to choosing the means
 The process is comprehensive, time-consuming, and exhaustive of
resources
 Assumes intellectual capacities that are impossible
 Public administrators are told not to use this method, however the
literature formalizes the Root method
b. Successive limited comparisons (Branch Method)
 One expects to only partially achieve their goals
 Expect to repeat the process continually as conditions and
accuracy of prediction improve
 Administrators use the Branch Method, although they have no
formal clarification -thus the purpose of this article
2. Clarification by contrast of the Successive Limited Comparisons Model (Branch
Model)
a. Selection of value goals and empirical analysis of the needed action are
not distinct from one another but are closely intertwined
 Clarifying values in advance is ideal, but when dealing with a
complex social problem it is impossible
 Even if an administrator can follow his/her own values as a
criterion for decisions, they cannot rank them because some will
conflict with each other
 Social objectives do not always have the same relative values
 The branch method has the administrator focus attention to the
marginal or incremental values only
b. Since means and ends are not distinct, means-end analysis is often
inappropriate or limited

619 | P a g e
 Therefore, means-ends relationships are absent from the branch
method
 In the branch method means and ends are simultaneously chosen
c. The test of a "good" policy is typically that various analysts find
themselves directly agreeing on a policy (without their agreeing that it is
the most appropriate means to an agreed objective)
 Agreement on policy thus becomes the only practicable test of the
policy's correctness
 Agreement is the test of the "best" policy in both methods - the
root method requires agreement on what elements in the decision
constitute objectives and on which of these objectives should be
sought.
 The branch method only relies on agreement, wherever it can be
found
d. Analysis is drastically limited: Important possible outcomes are
neglected; Important alternative potential policies are neglected; and
important affected values are neglected
 The branch method uses simplification through limitation of policy
comparisons to those policies that differ in relatively small degrees
from policies presently in effect.
 Also, it is necessary only to study those respects in which the
proposed alternative and its consequences differ from the status
quo (branch method)
 Branch method ignores values attached to neglected consequences
as well
 Achieving a degree of comprehensiveness comes with the use of
watchdogs who protect the interests of their jurisdictions by
redressing damage done by others, and by anticipating and
heading off injury before it occurs
e. A succession of comparisons greatly reduces or eliminates reliance on
theory

620 | P a g e
 In the branch method the comparisons, together with the policy
choice, proceed in a chronological series
 Policy-making is a process of successive approximation to some
desired objectives in which what is desired itself continues to
change under reconsideration
3. Closing remarks regarding the branch method
a. Description of the branch method explains why many administrators
often feel more confident when "flying by the seat of their pants" than
when following the advice of theorists
b. Successive Limited Comparison (branch method) is indeed a method
c. Branch method contains some imperfections
 Method has no built in safe-guards for all relevant values
 May lead the decision-maker to overlook excellent policies for no
other reason than that they are not suggested by the chain of
successive policy steps leading up to the present
d. With this in mind, policies will continue to be as foolish as they are wise
(using this as well as a number of other methods)

621 | P a g e
5.8 "System Politics and Systems Budgeting"
1. Process and System

a. Process Politics is the activity by which bargains are struck and


allocations negotiated or rules of the game for the decision of budgetary
matters

b. System Politics, unlike Process Politics, is concerned with the outcome or


results, not the activity of deciding

c. Systems Budgeting manifests itself in the form of so-called PPBS


budgeting systems which are concerned with results and outcomes more
than anything else.

2. The Dominance of Process

a. Process Politics dominated the academic literature during the early 50's
and the mid 1960's

b. This centered around the strong focus on pluralistic decision making

 It held that conflict was reduced by each competing party getting


its share of the budget pie

 The process, while imperfect, was viewed to maximize a


Pareto Model by assuming that if parts of society were not
represented they would join a interest group that would represent
their needs

 It posited that it was a winning sum game where everyone got


something

c. Concern was not focused on the best allocation of resources-theorists


believed that if a decision was reached the process itself would have
already made the right decision

d. Schick argues that this was easy to do because resources were plentiful-
eventually though this would not be so as resources became scarce

622 | P a g e
3. The Systems Challenge

a. The move from process to systems requires two things

 A dissatisfaction with the outcomes resulting from the budgetary


process

 a belief that better outcomes can be achieved through a systems


approach

b. New Deal and Great Society programs with their emphasis on new
programs fostered an environment where a systems model could develop
in light of the deficiencies of the process model in goal formation

c. Conservatives and Liberals like PPB for opposite reasons-Conservatives


lack faith in public expenditures, while Liberals want to show that
government can work

4. Political Process Deficiencies

a. Pluralistic processes are based on competition among interest groups.


Therefore if the market is not purely competitive then the system is
flawed. The other sections illustrate concepts where the market is flawed

b. Public Goods must be based on the public will. The political process
model assumes the opposite- private will is the determining factor
through the power of interest groups. Therefore competition based on the
collective will is not occurring

c. Externalities create benefits that often go to the strongest power group.


For example, Air polluters may get a tax credit, paid poorer people, who
are hurt by the pollutants to clean up their discharge. The powerful
group in this case get doesn't pay for its destructive output-the powerful
group gets the benefit without the cost

d. Income Distribution exemplifies how powerful groups have political


power while the poorer segments of society do not. Therefore pure
collective will impossible due to the lop-sided amounts of political power
a few wealthy groups wield

623 | P a g e
e. During the 50's the affluent majority rose to power. This led to a tyranny
of the majority where the relatively smaller amounts of poor people were
excluded from political power

f. Ideology can be a hindrance to rational market choices since it creates a


biased set of assumptions about the nature and potential of government
programs. This can work negatively for both conservatives and liberals

g. Immobility of Political Resources hinders those seeking power from


actually getting it. Voting district lines, seniority, bureaucratic patterns,
committees, and balkanization of urban regions are examples of
structural impediments

h. Representation may also block citizens from achieving power

5. The Status of Systems Budgeting

a. System Budgeting has met with considerable misfires and


misinterpretation

b. It is a simple concept that has produced meager results

c. The political institutions are not willing to develop the model into
practical capabilities

d. The continuing focus on process has stifled the intentions of PPB


systems which is to concentrate on outcomes based on goals while
examining possible alternatives

624 | P a g e
5.9 Policy Models
INSTITUTIONAL MODEL (Policy as Institutional Output)

 Public policy is authoritatively determined, implemented, and enforced by


government institutions – Congress, Presidency, courts, bureaucracy, states,
municipalities, etc. (Legitimacy)
 Only government policies have legal obligations (Universality)
 Only government policies extend to all persons
 Only government monopolizes coercion in society – imprisoning violators
 The U.S. Constitution establishes the fundamental institutional structure for
policy making.

PROCESS MODEL (Anderson)

 One can view the policy process as a series of political activities


 Problem Identification
 Agenda Setting
 Policy Formulation
 Policy Legitimization
 Policy Evaluation – (a process)

RATIONAL MODEL (Dunn)

 Achieves “maximum social gain” – governments should choose policies resulting


in gains to society that exceed costs by the greatest amount.
 Rationalism involves sacrificed by a public policy, not just those that can be
measured in dollars.
 Efficiency is important

INCREMENTAL MODEL (Lindblom)

 Views public policy as a continuation of past government activities with only


incremental modifications.
 Policy makers generally accept the legitimacy of established programs and
tacitly agree to continue previous policies.

625 | P a g e
 Incrementalism is politically expedient – agreement comes easier
 People are pragmatic – they seldom search for the “one best way”, but instead
find a way that works.

GROUP THEORY (Stone)

 Interaction among groups is the central fact of politics


 Individuals are important in politics only when they act as part of, or behalf of,
group interest.
 Politics are really the struggle among groups to influence public policy.
 Public policy at any given time is the equilibrium reached in the group struggle.

ELITE THEORY – Policy as Elite Preference (Lowi)

 The preferences and values of the elite


 Public officials and administrators merely carry out the policies decided on by
the elite.
 In the U.S., the bases of elite consensus are the sanctity of private property,
limited government, and individual liberty.
 Public policy does not reflect the demands of the masses, but rather the
prevailing values of the elite.
 Elites desire to preserve the status quo.
 The stability of the system, and even its survival, depends on elite consensus on
behalf of the fundamental values of the system, and only policy alterations that
fall within the shared consensus are given consideration.

PUBLIC CHOICE MODEL – Policy as Collective Decision Making by Self Interested


Individuals

 Assumes that all political actors – voters, taxpayers, candidates, legislators,


bureaucrats, interest groups, parties, governments – seek to maximize their
personal benefits in politics as well as in the marketplace.
 Individuals come together in politics for their own mutual benefit and even with
selfish motives they can mutually benefit through collection decision making.

626 | P a g e
 Government must provide public goods – goods and services that must be
supplied to everyone if they are supplied to anyone.

GAME THEORY

 Opponents must be rational – they must weigh he potential costs & benefits of
their actions and choose an course of action that does not result in costs that
exceed gains.
 The study of decisions in situations in which two or more rational participants
(person, groups, government) have choices to make and the outcome depends
on the choices made by each.
 Opponents must adjust their conduct to reflect not only their own desire &
abilities, but also their expectations about what others will do

SYSTEM THEORY – Policy as System Output

 Organic system with input, throughput and output.


 Based on rationality (genuine rationality)

MIXED SCANNING (Etzioni)

 Combination of rationality and incrementalism

GARBAGE CAN MODEL (Kingdon)

 Unclear goals
 History not understood
 Participants in and out
 Imperfect technology
 Chaotic – not the same each time

MODELS OF PUBLIC DECISION MAKING (RAAD I p. 255-259)

 RATIONAL-COMPREHENSIVE MODEL – aka ECONOMIC MAN model. Assumes


the decision maker has access to all needed information, ability, intellect, time,
etc. and is able to select the ONE BEST solution. It presupposes that

627 | P a g e
individuals are out to maximize their self-interest. Values are separated from
facts.
 SIMON - ADMINISTRATIVE MAN MODEL – SIMON criticized the Economic Man
model. The reality of decision making is quite different because information is
limited and human processing capabilities are limited. Simon’s decision maker
is an ADMINISTRATIVE MAN whose analysis is based on BOUNDED
RATIONALITY and whose actions are aimed at SATISFICING rather than
maximizing. In Simon’s view, objectives and values cannot be separated.
 LINDBLOM - INCREMENTAL MODEL – LINDBLOM. “MUDDLING THROUGH”.
Decision making is a non-comprehensive process of successive and limited
comparisons. The rational-comprehensive model is fallible, too slow and costly
and can never resolve conflicts of values and interests. Lindblom’s model
provided the foundation for a more realistic analysis of BUDGETING that his
student WILDAVSKY provided in The Politics of the Budgetary Process. Values
rather than facts determine budgetary decisions (view shared by Guy PETERS).
 DROR - NORMATIVE-OPTIMUM MODEL – DROR – Dror argues that Lindblom’s
model is more realistic than Simon’s model, but is still unsatisfactory. Dror’s
model enhances rationalization, uses extra-rational input via sensitivity
sessions and brainstorming. Policy Gambling: Dror argues that every policy is
a RISK and may not work, but it is important to TRY and Experiment.
 ETZIONI - MIXED-SCANNING MODEL – Amtai ETZIONI - Distinguishes
between contextualizing decisions (which outline the basic directions of policy
substance), and BIT-BY-BIT DECISIONS or incremental steps which prepare for
or follow a contextualizing decision.
 WILDAVSKY - IRRATIONAL POLICY ANALYSIS – WILDAVSKY – Argues that
decision making is not very rational. It is very difficult to determine empirically
the degree to which decision making processes are determined by RULE OF
THUMB, perception (definition) of the situation, bias, emotion, the need for
quick results, groupthink, stress, org and cultural history, the informal role and
position of decision makers, and the physical messages (face, expressions of
doubt, non-verbal cues, etc.) of decision makers.

628 | P a g e
 COHEN, MARCH & OLSEN - GARBAGE CAN MODEL – (Universities &
hospitals) At the individual level, one has to work with conflicting groups of
people. Organized anarchy. In this type of decision making, policies and goals
are often vague and, when clear are often conflicting. Members of the org have
little knowledge of the overall work of the organization, and decision making is
erratic. This decision making model is good for professional orgs where
individuals are highly educated, opinionated, and independent.

629 | P a g e
5.10 Bureaucracy and the Presidency
Riley and Baermann note that the President does have influence on the
implementation activities of the bureaucracy. However, this influence is limited by a
number of forces. The White House can affect the way agencies operate by attempting
“to have an impact on the organizational, budgetary, and legal environments in which
agencies make their decisions.” The “political executives” or political appointees can
have influence over the bureaucracy; however sometimes this backfires if the
appointee “goes native”. Over time, and as a result of interacting with agency
personnel, appointees may develop loyalty to the agency as well as the President. In
order to have influence, the appointees must have cooperation of the policy specialists
(who influence congressional committees), in addition to balancing the pressures to be
loyal to constituencies, professional associations and policy experts.

Regarding the influence of Congress on the bureaucracy, it is important to remember


that the bureaucracy cannot operate without money, and no money can be spent
unless the Congress decides to spend it. Committees and subcommittees magnify the
individual power of representatives trying to push for issues that their particular
constituency cares most deeply about. It helps to remember that subcommittees in
the House and Senate are composed of members who are motivated to work for certain
issues because they are either representing their constituency or trying to get
reelected. As such, the bureaucracy can expect some interference and influence
attempts from Congress as well as from the President and interest groups in attempts
to shape regulations and bureaucratic action in their favor. Members of the Congress
or Senate will also be interested in trying to influence regulation coming out of the
agencies, because they affect constituents and interest groups that may pull their
campaign contributions if they see increased regulations coming down. In the case
that a particular piece of legislation or executive order is viewed as a “contract” to
implement as expressed, it is important that the agency try to shape the
administration of the legislation or order in the same fashion that it was expressed,
without compromising the existing rules and mission of the agency.

The bureaucracy must also consider the oversight role of Congress, and how the
“heat” may be turned up or down on their agency as a result of the regulations they

630 | P a g e
promulgate. The bureaucracy is able to handle these influences by keeping to their
mission as administrator, while carefully balancing the influences coming from all the
various forces in the environment. It is impossible for the bureaucracy to escape
criticism because of the nature of their role and the place they occupy in our system of
democracy. The processes bureaucracies employ in implementing programs and
decision-making are not perfect in terms of the ideal democratic participation model,
but these processes are a “work-in-progress” and we can expect to see much more
“fine-tuning” of the systems in the future.

631 | P a g e
5.11 The concept of policy evaluation
The concept of policy evaluation involves investigation of a policy or program at some
point in time after its inception, in order to assess if the policy or program is effective
at delivering the goals, outputs and/or outcomes it was intended to deliver. In the
past, some policy evaluations of particular policies and programs involved simple
unscientific and intuitive judgments based on little evidence, that were strongly
influenced by ideological and partisan interests.

A different form of policy evaluation centers on the process involved in administering a


given policy or program. A newer type of policy evaluation, called systematic
evaluation, uses social science methodology in an attempt to measure the effects on
society of a given policy or program, and to gauge how effective they are at reaching
their objectives. Today, we have several other scientific research methods and
research designs that are commonly used in policy evaluation. One research design
commonly used is “experimental design”, which compares a treatment group with a
control group. In the experimental design, both groups are randomly chosen. The
“quasi-experiment” design compares a treatment group with a “comparison” group,
one that is chosen because it has many aspects that are similar to the treatment
group. The “before-and-after” study compares the results of a program with the
conditions that existed in the policy environment prior to the adoption of the policy in
question.

Who performs policy evaluations? Much of the work is done by non-governmental


think-tanks, the media, university researchers, and interest (pressure) groups who
wish to use the information to influence future policy. In government, some
departments (Labor, HHS, Energy) use assistant secretaries who are responsible for
program evaluation. Other governmental entities have full-time specialized staff
devoted to program evaluation, and some departments and bureaus outsource
evaluation to private research organizations or universities. There are several inherent
factors in the policy process that make evaluations difficult. Determining whether a
policy or program is “working” can be very difficult. Goals may be unclear, societal
changes may have numerous causes other than the policy being evaluated, policies
may have “externalities” that affect other populations than the ones intended, and
evaluation data may be difficult to acquire. Also, agency officials may offer resistance
632 | P a g e
to evaluation efforts, and the completed evaluation may be ignored or attacked for
various reasons.

Do policy evaluations really impact change? It depends on if and how the evaluation
is used. As Anderson points out, sometimes evaluation studies are used as political
tools to delay decision-making on a certain issue or to justify and legitimate a decision
already made. Actors may also try to avoid confrontations or controversy by “passing
the buck”, and requesting that an evaluation be done. Evaluations are not likely to
impact change if they are “institutionalized” as in some government grant processes
where they are required. The reason for this is that they are required as part of the
funding scenario. They may never be read or used to improve policies. The main
purpose is that if an evaluation is done at the required intervals, the funding will
continue, and that’s the end of the evaluation’s useful life. Also, if evaluations are
used as ammunition for partisan or personal political tools, they may not accomplish
or impact any change. The reason why these uses of evaluations are not likely to
result in any change is that they are simply used for self-serving purposes, then
discarded (as in the requirement for continued funding scenario). No further use of
the information may be necessary if the primary goal is to evade criticism or to gain
political advantage for a short moment in time.

633 | P a g e
5.12 Five-Step Method for Policy Analysis
(A postpositivist, postmodernist inspired, nonrational approach to a rational
methodology.)

Step 1: Define the problem and determine its causes.

Step 2: Establish criteria to evaluate alternatives.

Step 3: Generate Policy Alternatives

Step 4: Evaluate and select Policies

Step 5: Evaluate adopted policy (evaluation research).

634 | P a g e
5.13 Important Comps Questions and Student Answers
Q – 1 What characterizes different policy arenas? Describe one typology
and discuss how it might help to conduct public policy analysis.

Theodore J. Lowi , 4 “Arenas of Power”

1. Redistributive

2. Distributive

3. Constituent (Not singled-out)

4. Regulative

Two Dimension in these 4 policy arenas:

1. Probability of Coercion and

2. Target of Coercion

From these policy arenas, which are determined by target and probably of
coercion, emerge certain identifiable types of political behavior.

Lowi’s theory has an eye toward generating theoretical prediction.

Q – 2: Discuss two views about the proper way to study policy


implementation.

Nicholas Henry, in Public Administration and Public Affairs (1991):

 The subfield of public policy is divided into 2 broad branches


o Incrementalist paradigm: substantive and focuses on description
(political scientist)
o Rationalist Paradigm: theoretical and normative, dominated by Public
Administrationists

Incrementalist:

 Charles Lindblom: described policy making as disjointed incrementalism and


“muddling through”

635 | P a g e
o Six Models
 Elite/Mass – C. Wright Mills Power Elite
 Group Model – Arthur F. Bentley The Process of Government
 Systems Model – David Easton, The Political Model
 Institutionalist Model – The ‘org. chart’ of government. Carl J.
Friedrich Constitutional Government
 Neo-Institutionalist – Lowi (4 arenas of power)
 Organized Anarchy – John W. Kingdon, policy streams, open
windows

Rationalist

 Yehezkel Dror, Metapolicy


 Public Choice
 Pareto optimality, making everyone better off, without making anyone worse off
o Exclusion/Consumption Model
o Technology Assessment Model

Nicholas Henry (1991) advocates Strategic Planning as a third approach.

 What is agency (principle – agent) theory and how can it be used to improve our
understanding of public policy making?

Q – 3: Some scholars have argued that “public policy is public


administration.” First, evaluate whether this statement accurately
captures the field to date. Then examine if this statement – or a
replacement – explains where the field appears to be heading for the
foreseeable future.

 According to Barrett & Fudge (Policy & Action, 1981) “policy doesn’t implement
itself.” Implementation transforms policy into action, operationalizing often
ambiguous policy.
 Thus, the study of “implementation theory” a terminology initially used by
Pressman and Wildavsky (1973) to bridge the gulf between politics and

636 | P a g e
administration. Focus on the normative advice feeding back into policy design,
researching lessons learned from implementation failures

 LJ O’Toole (1986, 1993)


o Emphasizes the academic backwater into which implementation research
has drifted; reviewed over 100 studies of public policy implementation
o role of implementation research for policy recommendations
o relying upon a linear model of implementation and ignores the
ambiguous nature of political policy initiatives, mult-actor contingenices
and conflict
 Kingdon
o 1990 – Implementation studies used to show folly of government policy
 Lipsky (1980)
o Emphasis on street bureaucrat, deviating from the linear, top-down
policy process; bureaucratic discretion enters equation.
 Lester Salamon (Beyond Privatization: The Tools of Gov. Action, 1989) describes
the traditional tools of policy implementation as the direct “command and
control” tools such as public enterprises, regulatory agencies

Future

 O’Toole (2000) suggests that field is alive and contributions are indirect, framed
in Ostrom’s (1999) work on institutional analysis. A stronger focus on
governance (Stone, 1989) and network analysis (Kikert et al, 1997; Rhodes,
1997)
 Schlager, (1999) Implementation studies need to reflect new and emerging
political ideas which impact public service organizations: Changing views of
citizenship, networks, governance.
 B. Guy Peters (1996) The Future of Governing: Four emerging Models &
“Governance without Governing” (1998) shows that the hollow state needs tools
and procedural instruments, such as the government-NGO partnership

637 | P a g e
 N. Henry (1991) Public Administration and Public Affairs, indicates that
strategic planning has surfaced as a practical paradigm of public policymaking
that reconciles the rational and incrementalist perspectives.

Globalization, terrorism, development, and modernity have changed the world in


which we live. Dynamic environments have the potential of creating great uncertainty
for individuals: should one invest in American corporations when it is unclear if your
money goes to increasing infrastructure or a CEO’s wallet? Is it safe for an Afghan
mother to go to the morning market without a headscarf? Should the World Bank be
allowed to “aid” failing economies by placing harsh conditions on loans that hurt local
economies and forcing the privatization of industries (which are then sold to foreign
multinationals)? All of these questions are difficult because it is unclear to individuals
what the outcomes of human interaction will be. Public policy is the study of how we
attempt to stabilize the ways we interact. From this perspective, public policy is not
only formal legislation coming from Congress or cases from the courts; public policy
encompasses both formal and informal human constructs that make the social world
more understandable.

638 | P a g e
Appendix A
Sample Comps Questions
and Students Answers

MPA Comprehensive Exam


Study Guide
Q – 1 Explain how public managers can use strategic analysis to implement
public policies. Describe some of the possible approaches or models they may
use.

First There's Strategy

Strategy involves the match between an organization and its environment. Although
some environments change faster than others, all organizations operate in a changing
environment. Certainly, the public accounting profession has seen tremendous change
over the last decade. New developments have included, for example, technological
changes, new .types of investments, increasing complexity in the tax code, personal
financial planning, and increasing liability insurance premiums.

As the environment changes, threats and opportunities are encountered. Some


organizations react to the changing environment by implementing changes in their
structure. These changes can affect the relationship between the firm and its
environment, or the changes can relate to the internal operations of the firm. Changes
relating to the relationship of a firm to its environment have more impact on the
organization's effectiveness, and changes involving the organization's internal
operations have a greater effect on the efficiency of the firm.

In general, the long-run success of a firm is more dependent on the organization's


effectiveness rather than its efficiency. As Peter Drucker stated, "It is more important
to do the right things than to do things right."

Thus, it is more important to Iong-run success to be effective--being in the right place


at the right time, offering the services demanded by the firm's market niche--than to
be efficient--achieving the most output from a given set of inputs, e.g., using the
internal resources of the firm to their maximum. It is possible that a firm doing the
right things wrong (effective but not efficient), can outperform a firm doing the wrong
things right (efficient but not effective). Concentrating on change involving the
organization's internal operations and being extremely efficient will not always insure
a firm's long-run success.

640 | P a g e
Next There's Strategic Management

Strategic management is a continuous process that works to fit an organization into


its changing environment. Strategic management is a broader concept than strategic
planning. Strategic planning (or long- range planning) is traditionally regarded as a
periodic process to develop long-range plans for the organization. Strategic
management consists of strategic decision making and strategic planning. Thus,
strategic management focuses on any strategic decision that must be made, regardless
of its time frame and the planning necessary to complement that decision.

Strategic management should not be thought of only in a long-term time frame. Many
times, changing a firm's strategies does take a long time to become operational, but
sometimes threats or opportunities emerge that must be acted upon immediateix'.
Thus, strategic management includes any decision that has strategic consequence and
developing a plan (whether short or long-range) to implement the decision.

Strategic management should also not be confused with operating management.


Operating management deals with the ongoing, day-to-day operations of the firm.
These decisions are aimed at improving the efficiency of the firm. Certainly, operating
management must not be neglected, and both areas of management responsibility
must fit together and complement one another. The strategic function and the
operation management function may be carried out by the same people, but the
strategic function is separate and distinct from the operating function.

The strategic management process involves taking advantage of the opportunities that
are made available to the organization and minimizing the threats to the organization.
As a minimum, an organization must be able to react effectively to changes over which
it has no control. With an effective strategic management function, a firm may be able
to endure change that causes threats for the competition.

And Then There's Implementation

Implementing a strategic management system is an administrative task carried out by


a group of the firm's partners and managers. At first, the task can be time consuming.
Once the basic system is in place, it will be less time consuming to keep it operating

641 | P a g e
than it was to implement. Probably the best method to generate support for the
program is to stress that a strategic management system can improve the firm's
overall performance.

Implementation of the strategic management system must be approached in a manner


indicating that the small planning group is not trying to take over the firm. It must be
made clear from the start that the group will work within the bounds of the
organization and that all members of the firm will have input into the process. Without
the support of the other members of the firm, the process may be doomed to failure.

Another way to increase support for the strategic management system is the use of
consensus decision making rather than formal vote taking. Voting on issues tends to
divide the organization rather than unite. Voting forces individuals to argue and to try
to convince others of their views. Consensus decision making fosters working together
and can lead to increased cooperation among members of the organization.

A possible stumbling block to a successful strategic management system is reluctance


to change. No benefits can be derived from brilliant strategies unless they are
implemented. One way to increase the chances for acceptance of change may be to
introduce change more frequently for relatively less important matters. As change
becomes more commonplace, if the strategic management process proposes change, it
may be more readily accepted.

The Overall Process

At first, the process should focus on formulating the business's mission and the
overall direction for the organization. With this clearly in mind, better goals and
objectives can be set for the firm to help achieve its mission. This is a difficult project
and will not result in a finished product, but rather serve as a starting point.

Early on, the process should attempt to identify the major strategic issues facing the
firm. Having this formalized process forces the leaders of the organization to focus on
the future and the strategic issues the organization may face. Without this process,
many of these issues may, be overlooked until it is too late. Then the firm can only
react to the issues rather than anticipating the situation and taking advantage of it.

642 | P a g e
The next phase of the process involves analyzing the firm's strengths and weaknesses
(an environmental analysis) and current services to clients in relation to objectives.
Insight into an organization's strengths and weaknesses can be gained by examining
informal factors such as the organization's past, its current employees, average age of
employees, the current and future partner/manager/staff ratios, salary requirements
of personnel, employees' expertise, the business's financial position, and its location.
An environmental analysis also consists of studying external factors such as the
growth patterns of the surrounding community, types of current and potential
businesses in the community, the local economy, average age of the community,
changes in client services demanded, services offered by other public accounting firms
in the community, future supplies of accountants, and new tax laws.

The final step in this phase compares the firm's current services to clients to the firm's
stated objectives. This should determine if there are gaps that can be filled or services
that should be eliminated.

At this point (or at various points during the process), progress reports should be
made to the entire firm. This serves to keep the process on track by generating input
from the entire firm, and it keeps the process out in the open so everyone feels a part
of it.

The final part of the process involves a meeting with the entire firm. This should
include a presentation by the planning group covering the mission objectives
statements and the analysis that has been done. Then the entire firm can discuss the
mission statement and objectives and finalize these documents. Finally, changes that
are to be implemented can be identified, and work groups can be set up to begin
implementation. At this point, the purpose of the strategic management process is to
monitor the progress of the changes and to receive feedback. As well, the firm will now
have a process in place that can continually monitor the environment and be ready to
promptly act on threats and opportunities that have strategic consequences.

A formalized strategic management system can be a powerful tool to achieve a better


match between its strategies and its environment. The process provides a way for the

643 | P a g e
firm to determine where it is currently and where it wants to be, and helps to
formulate a plan so the firm can achieve its objective.

Extra material

1. Values

Each sector is committed both to the values of public service (the seven principles
identified by the Committee on Standards in Public Life – selflessness, integrity,
objectivity, accountability, openness, honesty and leadership) and to the highest
academic standards, including:

 rigorous independent research, evidence, evaluation and thinking

 speaking truth to power, without fear or favor

 working across all political parties and perspectives

 exploring the inter-relationships between public, private, voluntary and


informal sectors

 taking into account perspectives from a wide range of stakeholders

 linking theory to practice, and research to action

 working in partnership and dialogue with policymakers, and managers

 linking research to development to education and learning

 aiding the translation of policies into practices

 recognizing diversity and emphasizing equalities

 encouraging thought leadership and learning

2. Changes, Complexity, Challenges

Profound political, economic, technological and social changes are taking place both
globally and locally, and facing citizens, governments, businesses, voluntary and
informal organizations with far-reaching challenges.

Deep questions are posed for the public, private, and third sectors by structural
changes like:

644 | P a g e
 Globalization of the economy, and of culture

 European integration and expansion

 The revolution in information and communications technologies

 Industrial restructuring and concentration

 Ecological and environmental change

 Ageing of the population

 Brutalization, crime and community safety

 Social exclusion and poverty

 The democratic deficit

 Diversities of identity and representation

Complex cross-cutting issues like these require governments and other organizations
to develop capabilities for:

 medium to longer term scenario based planning, with a 5 to 10 year horizon

 strategic analysis combined with lateral thinking and creativity

 translating strategies into practical policies and action programs

 knowledge generation, innovation, testing and rapid dissemination

 detailed management of implementation, logistics and operations

 leadership and management of organizational and cultural change

 mobilizing inter-organizational networks and partnerships

 involving citizens and users in service development and delivery

 multi-level working between the tiers and spheres of governance

Governments at all levels, along with other sectors and services, now therefore have to
operate in a context of continuous change, complexity and volatility.

645 | P a g e
Organizations of all kinds have to develop new more flexible and adaptive patterns of
governance and leadership, policymaking and strategy, management and service-
delivery, if they are to respond adequately to these changes and challenges.

Governments in many countries are exploring new approaches to modernization and


improvement of public services, including attempts to develop more "joined up"
citizen-centered approaches, in which government and the public service sector are
organized around the cross-cutting needs of citizens and users, rather than around
the interests of the professions and departments which have traditionally structured
government bodies.

3. The Need for a Major New Initiative

Much of the more traditional writing and teaching about public administration in the
UK is both out of date and also out of touch with the complex realities facing
policymakers and managers in government and the public service today.

There is often a failure to take sufficient account of the deep structural changes which
have taken place in the political, economic, social and technological context of public
services, over the past 5 to 10 years, and the consequences of these changes for the
role, purposes and nature of democratic governance, public policy and public
management.

Many of the traditional assumptions about public administration no longer apply, and
must be revised to provide explanations which are more appropriate, illuminating and
relevant:

 The context for public policy and management is no longer one of relative
stability, but one of continuous change and uncertainty (political, economic,
social and technological).

 Public policymakers and managers now recognize that the needs and problems
facing citizens, communities and governments are complex and diverse rather
than straightforward, that previous patterns of government intervention have
not been notably effective in resolving these problems, that the best solutions
and responses are not always known or understood, and that Governmental

646 | P a g e
policies and programs therefore have to be developed and tested in a more
provisional and reflexive way.

 The assumption that the needs of their populations were relatively


homogeneous and that Governments could therefore mass produce fairly
standardized services to meet uniform needs is being replaced by a recognition
of the range and variety of needs within the population and therefore the
necessity to develop diverse services, tailor-made to the needs of particular
groups of citizens and users within the population.

 The assumption that the primary task of Governments was to administer the
state apparatus (primarily through command and control of the bureaucracy
and its budgets and procedures) is giving way to a recognition that the primary
task of Governments is to govern their communities and economies (and that
this requires not just administration of the bureaucracy but also civic
leadership in the community).

 The assumption that the state would be the main provider of public services,
and that a watertight distinction could be made between public and private
sectors, between the state and the market is being challenged by the view that
civic leadership cannot be provided by the public sector alone, but requires new
kinds of partnerships and joint ventures between public, private, voluntary and
grassroots sectors. It recognizes the need to analyze the inter-relationships and
inter-dependencies between three spheres - state, market and civil society.

 The assumption that education and training should concentrate mainly on the
techniques and procedures for the internal administration of the public
bureaucracy is being replaced by a recognition of the need to set public
administration in the wider context of democratic governance and public policy
and management. Courses therefore need, among other things, to address
issues of civic leadership as well as public administration, policy formulation as
well as policy implementation, strategic management as well as operational
management, evaluation of the impacts, outcomes and results of public
programs as well as the measurement and control of cost inputs.

647 | P a g e
Theorizing Governance

Seven main versions and uses of the term governance have been identified in the
academic literature (Hirst, P 2000; Rhodes, R 2000):

 Governance as part of the so-called New Public Management, arising from the
marketization and sometimes privatization of public services, and the
consequent need for public authorities to "steer" the work of organizations
which they no longer own or control but are sub-contracted to deliver services.

 "Corporate governance" as a means of improving the accountability and


transparency of actions by the boards and managers of companies, especially to
wider stakeholders outside the company.

 "Good governance" seen by international development agencies like the World


Bank as a necessary condition for economic development particularly in the
third world

 "Global and regional governance" as a response to the problem of regulating


supra-national issues like global warming or world trade through the nation
state, or through inter-governmental activity.

 Governance as part of the new political economy which draws on regulation


theory to interpret the blurring of the boundaries between state, market and
civil society, and the shifting patterns of leadership and coalition between
competing interests and ideologies.

 Governance within a socio-cybernetic system where there is no single or stable


centre of control, and where continuous waves of change produce complex
fluxes and flows of power, and multiple actors and authorities.

 Governance within policy networks and inter-organizational networks in which


government itself is only one of a wide range of actors, and has to orchestrate or
negotiate with a wide range of different stakeholders. Governance in this
context is particularly complex not only because it cuts across the boundaries
between different sectors, but also because state, market and civil society are
each constructed around very different goals, values, accountabilities and
organizational forms.

648 | P a g e
F I G U R E 1: R E L A T I O N S H I P B E T WE E N S T A T E , S O CI E T Y A N D M A R K E T

Broad themes to be explored by the Institute include the implications for governance
and public management of the changing, complex and dynamic inter-relationships
between

 state and civil society – including the relationships between representative and
participatory democracy; and between producers and users of public services

 state and market – including public/private partnerships, management of


regulated industries, the role of the state in social reproduction, and in the
regulation and ordering of markets

 market and civil society – including corporate citizenship, the role and power of
consumers, the marketization of everyday life

 state, market and civil society – those areas of confluence and inter-action
between all three spheres, where the vector forces are most complex and
contradictory.

Q – 2 Criticism of bureaucratic performance seems to have reached epidemic


proportions. Although citizen complaints stem from many origins, some
observers, such as James Q. Wilson, speak to a common misconception:

The greatest mistake citizens can make when they complain of “the bureaucracy” is to
suppose that their frustrations arise simply out of governance problems.

While it is apparent that Wilson sympathizes with the governance position, others
disagree. Assess both sides of this debate, incorporating organization and
649 | P a g e
administration theory as well as related literature, and defend your own position in
the debate.

 Hamilton argued in Federalist 27 that the more people experience the actual
operation of government on the common occurrences of their lives, ‘the more it
will conciliate the respect and attachment of the community.

 Americans harbor anitgovernmental, antibureaucratic attitudes in the abstract,


but most approve in general of current governmental policies and services Meier

 Meier’s definition of governance is very different from other definitions. I am


using the term governance in the same sense that Aristotle used politics; it is
the process of governing society in a generic sense.

 The problems in American government in my view are not problems of


bureaucracy but problems of governance. In contrast to what is adequate
performance by the bureaucracy the performance of our electoral institutions
has been dismal.

 The irony of the situation is that as the electoral branches stalemate, they act
against the bureaucracy- the one part of government that has a capacity to
govern.

 The norms of democracy grant policy-making legitimacy to electoral institutions


not to bureaucracy.

 The fundamental problem of governance that has generated the continual state
of crisis in political/bureaucratic relationships is that the electoral branches of
government have failed as deliberative institutions; they have not resolved
conflict in a reasoned manner.

 Failure to establish new policy is not the only area where political institutions
have failed. At times policies have contradictory goals and these are left to the
bureaucracy to grapple with as best it can.

 The failure to resolve goal conflict with informed public policy is exacerbated by
the development of the continual campaign for office.

650 | P a g e
 Politicians compete with each other to adopt more extreme policies; the normal
tempering role of bureaucracy, the application of expertise to policy and policy
proposals is lacking.

 Policy failure then leads to greater cynicism among the public.

 Our basic problem of governance is that the long running interplay between
bureaucracy and expertise on the one hand and responsiveness and democracy
on the other hand has swung too far in the direction of democracy.

 Bureaucracy is being asked to resolve political conflict, a function it performs


poorly at best.

 The solution to the governance problem in the united States is to have more
bureaucracy and less democracy.

 Solutions

1. Replace checks and balances system with more unified political


structures

2. Lengthen the time frame for public policymaking.

3. Restrict and perhaps even eliminate political appointees.

4. Reduce the public sector reliance on the private sector.

5. Bureaucracy’s normative role in public policy suggests that bureaucracy


serves best when it exploits its information and expertise advantages.

6. Bring the institutionalized presidency under the merit system.

7. Replace the current public philosophy of neoclassical economics and its


sole value of efficiency.

8. Reorient our education programs from training entry level civil servants
to training policymakers.

 The bureaucracy, by most objective standards, is performing fairly well while


the electoral institutions seem to be deteriorating.

651 | P a g e
Wilson

 Americans fear bureaucracy’s use of discretion to guide decisions and


actions, and insist on rules, for example, particularly “at the hands
of…street-level bureaucracies that deal with us as individuals rather than as
organized groups and that touch the more intimate aspects of our lives [e.g.,
police, schools, medical institutions, prisons]. That worry is natural; in these
settings we feel helpless and The State seems omnipotent. We want these
bureaucracies to treat us fairly but we also want them to be responsive to our
particular needs…”

 How then does a society strike a reasonable balance between governance by


rules and governance by discretion? First, Wilson suggests, we must
“sensitize ourselves to the gains and losses associated with governance by
rule rather than by discretion.” We need to be aware that in America rules
induce agencies to 1) produce certain observable outcomes, 2) create offices,
procedures, and claims inside an organization that can protect precarious
values, and 3) specify minimum standards that must be met. Talented,
strongly motivated people usually will find ways of making even rule-ridden
systems work to get the job done, says Wilson. Second, if we wish to complain
about how rule-ridden our government agencies seem to be, we should direct
those complaints not to the agencies but to the Congress, the courts, and the
organized interests that make effective use of Congress and the courts.”

Q – 3 What are the major approaches to measuring service quality? How


applicable are they to the case of public agencies?

There are a number of approaches to measure service quality and many are
appropriate for public organizations. While there are great similarities between public
and private organizations, there are several methods to measure service quality used
by the private sector that are clearly not compatible with public organizations.

“Early work on municipal service quality assessment recommended multiple measures


of performance from both providers and users. Citizen satisfaction surveys have
rivaled their more quantitative counterpart, administrative performance measures, in
adoption, but the implication of survey results for action is not well understood by

652 | P a g e
managers or scholars. To achieve meaningful integrated multiple measures of service
quality, we need to explore the dimensions of citizen satisfaction and review patterns
of satisfaction across localities. We also need to understand the relationship between
administrative performance measures and citizen perceptions. This cross sectional
analysis of municipal citizen satisfaction and performance benchmark data suggests
that citizen satisfaction survey results are useful to managers in conjunction with
performance-measurement programs as part of a multiple-indicator approach to
evaluating municipal service quality. However, understanding citizen perceptions
requires a different perspective than that applied to administrative service
performance measurement.”

(Kelly, J.M., Swindell, D. (2002). A multiple-indicator approach to municipal


service evaluation: correlating performance measurement and citizen satisfaction
across jurisdictions. Public Administration Review, 62, 5, 610-621.)

Other approaches to measuring service quality are:

 Customer Interaction – works for the public sector


 Self Assessments – works for the public sector
 Mystery Shopping (now here’s an idea for a public service!)
 Organizational Report Cards

(Service Evaluation Concepts (2005).

Fred David (p.317-320) introduces us to a number of useful methods to measure


service/performance. For example, comparing expected results to actual results,
investigating deviations from plans, evaluating individual performance, and examining
progress being made toward meeting stated objectives. These are viable measures for
private and public agencies. Other procedures commonly used for measuring service
quality include: 1) comparing the agency’s performance over different time periods, 2)
comparing the agency’s performance to competitors’ (this seems particularly useful
with contracting out or privatization, that is, government comparing their performance
with a private entity), 3) comparing the agency’s performance to industry averages.
These are all good measures and can be analyzed with both quantitative and
qualitative methods.

653 | P a g e
Robert K. Yin reminds us that the case study method is best used when seeking the
“how” or “why” questions asked about a contemporary set of events, over which the
investigator has little or no control. Actually, Yin recommends the use of the dual
approaches of case study and survey methods. Therefore, the case study is a
superlative method of measuring service quality. Yin (p.8) relates that one particular
method, the randomized field trial, was designed for evaluation research and is
commonly used. While this method works well for both public and private
organizations, Yin points out that randomized field trials do not work well in a number
of situations.

A related article that is excellent for this and other public administration topics to
describe the differences between public and private organizations is:

Rainey, H.G., Backoff, R. W., Levine, C.H. (1976). Comparing public and private
oganziations. Public administration Review, March/April, 233-244.

Of importance, is page 240, section III, 3. Performance Characteristics. This short


section discusses the observations of Dahl, Lindblom, Golembieski, and Downs
concerning the performance of government organizations and administrators, usually
in comparison to business.

Performance assessment is often implemented as a series of specific performance


measures associated with certain service functions or as indicators of progress toward
a mission, as in the case of service efforts and accomplishments. These may be
thought of as internal measures of service quality because they come from a definition
of effectiveness derived and monitors by administrators. In contrast, external
measures of service effectiveness come from citizens, usually through satisfaction
surveys. Results from these kinds of measures are problematic for administrators
because it is unclear what criteria the citizens are using to evaluate effectiveness. Also
questionable is how much accurate information citizens have on which to base their
evaluations.

Administrative performance measures are widely accepted as reliable indicators of


service quality though there is considerable variation in the types of measures
employed and how local governments use them. There is not so much consensus on

654 | P a g e
the utility of citizen satisfaction surveys as a reliable indictor of government
performance, which makes this topic a bit more interesting.

Q – 4 Some scholars have argued that “public policy is public administration.”


First, evaluate whether this statement accurately captures the field to date.
Then examine if this statement – or a replacement – explains where the field
appears to be heading for the foreseeable future.

 According to Barrett & Fudge (Policy & Action, 1981) “policy doesn’t implement
itself.” Implementation transforms policy into action, operationalizing often
ambiguous policy.

 Thus, the study of “implementation theory” a terminology initially used by


Pressman and Wildavsky (1973) to bridge the gulf between politics and
administration. Focus on the normative advice feeding back into policy design,
researching lessons learned from implementation failures

 LJ O’Toole (1986, 1993)

o Emphasizes the academic backwater into which implementation research


has drifted; reviewed over 100 studies of public policy implementation

o role of implementation research for policy recommendations

o relying upon a linear model of implementation and ignores the


ambiguous nature of political policy initiatives, mult-actor contingenices
and conflict

 Kingdon

o 1990 – Implementation studies used to show folly of government policy

 Lipsky (1980)

o Emphasis on street bureaucrat, deviating from the linear, top-down


policy process; bureaucratic discretion enters equation.

 Lester Salamon (Beyond Privatization: The Tools of Gov. Action, 1989) describes
the traditional tools of policy implementation as the direct “command and
control” tools such as public enterprises, regulatory agencies

 Future

655 | P a g e
 O’Toole (2000) suggests that field is alive and contributions are indirect,
framed in Ostrom’s (1999) work on institutional analysis. A stronger focus on
governance (Stone, 1989) and network analysis (Kikert et al, 1997; Rhodes,
1997)

 Schlager, (1999) Implementation studies need to reflect new and emerging


political ideas which impact public service organizations: Changing views of
citizenship, networks, governance.

 B. Guy Peters (1996) The Future of Governing: Four emerging Models &
“Governance without Governing” (1998) shows that the hollow state needs tools
and procedural instruments, such as the government-NGO partnership

 N. Henry (1991) Public Administration and Public Affairs, indicates that


strategic planning has surfaced as a practical paradigm of public policymaking
that reconciles the rational and incrementalist perspectives.

Globalization, terrorism, development, and modernity have changed the world in


which we live. Dynamic environments have the potential of creating great uncertainty
for individuals: should one invest in American corporations when it is unclear if your
money goes to increasing infrastructure or a CEO’s wallet? Is it safe for an Afghan
mother to go to the morning market without a headscarf? Should the World Bank
invest in continually failing economies? All of these questions are difficult because it is
unclear to individuals what the outcomes of human interaction will be. Public policy is
the study of how we attempt to stabilize the ways we interact. From this perspective,
public policy is not only formal legislation coming from Congress or cases from the
courts; public policy encompasses both formal and informal human constructs that
make the social world more understandable.

Q – 5: In class Dr. R. gave us copies of 4 comps questions. One was the question
about “PA being in search of an “identity.” She told us that the answer should
include the 5 eras of PA discussed in Shafritz & Hyde’s Classics of PA.

These 5 eras are:

1. 1880s-1920s

2. 1930s-1950s

656 | P a g e
3. 1960s and 1970s

4. 1980s and 1990s

5. The present (This is not shown in the Classics book, but Dr. R. has confirmed this
is the 5th era)

Hatch, Mary Jo. Organization Theory (1997)

Four Major Perspectives

1. Classical (1900s---)

Industrialism----Adam Smith---father of capitalism---1776---Wealth of Nations---


invisible hand---div of labor (e.g., pin factory)

Post industrialism---society and organization---Wal Mart/Saturn (no Boundaries)

Sociological stream

Emile Durkheim (p. 30)---sociologist---explained structural shifts from ag. to


industrial organizations---informal orgs focus on workers social needs
Max Weber (p. 32)--- he liked law/structure father of bureaucracy--- it is a way to
rationalize the social environment---formal rationality (means or techniques) and
substantive rationality (ends or goal)---formal rationality w/o substantive rationality
leads to an iron cage---making man a cog in a machine

Karl Marx (p. 28)---theory of capital---inherent antagonism between capitalists and


workers over how to divide surplus value---workers are alienated so they must
organize

Classical Management stream

Frederick Taylor---father of scientific management---he attacked soldiering


(workers limiting their output on purpose---time/motion studies---one best way
promoted rationalization in orgs

Henri Fayol---(p. 32)---span of control---“departmentation”---unit of


control---hierarchy---espirit de corps

657 | P a g e
Chester Barnard---expanded Durkheim’s informal org---integrated goals and
motivation---contributed more to org behavior than org theory

2. Modern (1950s---)

 Kenneth Boulding (36)---hierarchy of systems---any thing w/ interrelated parts


A control or cybernetic system uses feedback (e.g., a thermostat)
 A closed system does not require additional input to operate
 An open system depends on the environment for inputs to operate

Network analysis—looks at the complex web of relationships of how the org interacts
with other orgs and with its environment

The org and its environment are totally separate---there are boundaries

The general environment --- social, cultural, legal, political, economic,


technological and physical components (p. 67)

The international and global environments also impact the org

“Buffering”---protecting the internal operations of an org from interruption by


environmental shocks such as material, labor and capital shortages (p. 91)

Environmental scanning is done to protect against these threats

Jeffrey Pfeffer and Gerald Salancik---Resource Dependent Theory (1978)

An org is vulnerable because of its need for resources (raw material,


labor, capital, equipment) from its environment --so the org is
controlled by its environment (p. 78)

So we analyze the org by starting with the resources it needs and


tracing them to their source---also look at the org’s competitors for the
same resour

Michael Hannan, John Freeman, Howard Aldrich---Population Ecology


Theory

Orgs are dependent on the environment for resources but this theory focuses on
patterns of success and failure among all orgs---not just one---it is survival of the
fittest (p. 810

658 | P a g e
Variation---changes in orgs

Selection---orgs choose certain characteristics

Retention---some survival

Philip Selznick---Institutional Theory

Orgs adapt to the values of external society---i.e, when actions are


repeated and given similar meanings by self and others (Richard Scott)
this is institutionalization---can lead to “rationalized myths”---i.e., TSU
serves A/A students from all over the U.S.

Rational decision making (p. 273)

 Define the problem


 Generate and evaluate alternatives
 Select an alternative
 Implement
 Monitor
 Evaluate

Herbert Simon---bounded rationality

When making a decision, decision makers often have

 incomplete and imperfect information


 complex problem
 limited human ability
 time pressure
 conflicting preferences

The Garbage Can Model (p. 278)

 The decision making process is very random


 Actors move in an out
 Problems, participants, solutions are all independent
 Thrown into the garbage can at random

659 | P a g e
Power and Politics—Jeffrey Pfeffer (p. 282)

Strategy process (p. 105)

Rational model---SWOT analysis---look at the org’s core competencies

Strategy formulation precedes implementation—it is top down

Emergent strategies can be bottom up

Goals---interrelated with strategies (p.119)

 Official---may be vague
 Operative---more specific

3. Symbolic Interpretive (1980s---)

Karl Weick---enactment theory---when you use concepts (i.e., organizations)


you create the thing you’re seeking to study---he is not
pragmatic—he is an interpretist (p. 41)

Conditions in the environment can’t be separated from the


perception of those conditions (p. 93)

4. Postmodern (1990s---)

 be careful, many of them wouldn’t like being put in a category


 this term includes a large variety of ideas---the key here is diversity
 it is relativistic---it abandons notions of universal truth---but it has some
standards
 fragmentation is a key theme---breakdowns in family, community & society and
threats to self identity are caused by tying to play so many roles with little
separation between them

the future will see smaller, more decentralized and informal orgs causing us to
face more ambiguity than ever---helping (and forcing) us to adapt to
more and more change---the paradox is that science has created the
means of sharing information so quickly, making orgs all the more
unpredictable (p. 45) to prepare for the post modern world we must take nothing for
granted---deconstruct everything!

660 | P a g e
Philosophy includes (see Burrell and Morgan, Sociological Paradigms and Org Analysis
1979)

epistemology (p.47)---how we know the world---the process by which we obtain


knowledge

objectivists---positivists and empiricists---independent observation is required

subjectivists---anti-positivists and idealists---all knowledge is filtered through the


observer

a third position---the process is greatly influenced by cognitive, social and


cultural forces---language is very important---postmodern

ontology---what can be known (the kinds of things that exist)

Q – 6: Describe no less than five (5) approaches/definitions/models of strategic


planning and highlight their possible advantages and limitations in general and
when applied to the public sector.

Mintzberg, Henry and Quinn, James Bryan Readings in the Strategy Process

1. The Entrepreneurial Organization---p. 244 (Mintzberg)

 simple structure
 one flamboyant leader
 little staff
 the leader creates the strategy and can adapt it as he/she deems necessary

2. The Machine Organization---p. 265 (Mintzberg)

 more complex structure


 routine work (i.e., the postal service)
 highly standardized work processes
 very regulated, bureaucratic structure

3. The Professional Organization---p. 288 (Mintzberg)

 complex work (i.e., a university; a hospital)


 work must be carried out by professionals
 but stability is important

661 | P a g e
 standard operating procedures are used

4. The Innovative Organization---p. 309 (Mintzberg)

 must be capable of sophisticated innovation (i.e., a research firm)


 adhocracies---matrix organizations (move people around a lot)
 highly organic structure
 flexibility is the key
 experts are grouped in functional units and then deployed to teams as needed

5. The Diversified Organization---p. 335 (Mintzberg)

 the most complex structure


 a set of semi-autonomous units called divisions
 very common in the private sector
 each unit is relatively free form headquarters control

The Model Approach --- the structure is most important

Chafee, Ellen Earle presents three models of strategy:

 Linear---rational, logical / closed system


 Adaptive---monitor the environment / open system
 Interpretive---social contracts based on free will / open system

Dutton, Jane

Categorization theory---merely labeling an event as an opportunity or a threat affects


information processing and motivation

Ginter, Robert

Social learning theory---behavior results from interactions of persons and situations

Hart, Stuart

Integrative framework---focuses on integrating the 5 roles played by top managers:


command, symbolic, rational, transactive, and generative

The Process Approach---the process is most important

662 | P a g e
Q – 7: The budget may be viewed as an instrument of fiscal policy, as a means of
determining policy choices, and as a tool for managing the economy. Explain
each of these aspects of budgeting. In your answer, emphasize the views of the
classical or pre-Keynesian economists, Keynesian economists, neo Keynesians,
the monetarists, the public choice school of economy, and leading public
budgeting scholars.

Classical/pre-Keynesian

 Adam Smith wrote The Wealth of Nations in 1776 --- father of capitalism
 the free market is most important
 government’s role in the economy is very limited
 voluntary exchange between buyers and sellers leads to economic efficiency
 prices are set by competition in the free market based on supply and demand

Keynesian

 John Maynard Keynes wrote General Theory of Employment, Interest and


Money in 1936
 some government spending is necessary to fine tune the economy
 performance by the economy in the aggregate is more important than individual
 so we need to look at output (GDP) , income and employment rates
 potential GNP means the full use of the factors of production (land, labor,
capital)
 compare potential GNP with actual GNP
 aggregate demand Y = C + I + G + (X-M)

C is consumption

I is investment

G is government spending

X-M is exports minus imports

663 | P a g e
 small deficits are ok---necessary to fine tune the economy
 the multiplier effect is very important---an increase in aggregate demand will
result in an even larger increase in actual or equilibrium GNP

Neo Keynesian

 Reagan and Bush One


 they called for big changes in government spending, interest rates and taxes to
 counter economic cycles
 in a recession they want more government spending and tax cuts to stimulate
the economy
 in inflation they want less government spending and tax increases to slow the
 economy down
 they also use monetary policy (interest rates set by the Federal Reserve Board)
 to influence the economy

Monetarists

 Milton Friedman and Alan Greenspan


 Dr. Sekwat says monetarists may be right----monetary policy may be more
effective in impacting the economy than fiscal policy
 they are also called “counter Keynesians” because they disagree with Keynes
 they say the economy is inherently stable---government fine tuning is not
necessary
 to them, the money supply is most important
 and the supply of money is controlled by interest rates which are set by the Fed

Pubic Choice

 this is a modern day theory


 followed by pre Keynsian economists
 individuals are rational---they make choices to best benefit themselves (whether
as buyers or as bureaucrats)
 politicians may claim to follow neo Keynesian policies but sooner or late they
must balance the budget
 also see Forrester, John P., Evolving Theories of Budgeting, pp. 101-124

664 | P a g e
Q – 7: Organizations are said to be difficult to evaluate; yet some argue that
the public nature of government organizations compounds this difficulty.
Discuss the contributing factors that make public organizations difficult to
evaluate. Cite authors who argue that public organizations are more difficult
to evaluate as well as some examples of those arguing the opposite view.

James Q. Wilson noted that for public agencies, efficiency means not only one output
considered. While there may be one major goal according to which we can measure
the performance of public agency there always are some contextual goals that public
bureaucracy must try to attain. Even if we considered only major goals, we would find
that often they are difficult to measure as some public agencies exist to supply
services that are immeasurable in economic terms hence they are not supplied by the
market.

Wildavsky, Aaron (1972). The Self-Evaluating Organization. Public


Administration Review, 32(5): 509-520.

 Evaluation and organization may be contradictory terms. The ideal organization


would be self-evaluating. Organizational structure implies stability while the
process of evaluation suggests change. Organization generates commitment
while evaluation inculcates skepticism. Evaluation speaks to the relationship
between action and objectives while organization relates its activities to
programs and clientele (pp. 509-510).

 Rather than succumb to the diseases of bureaucracy, the self-evaluating


organization will be tempted to pass them on to others. The self-evaluating
organization can split itself off into “evaluating” and “administering” parts, thus
making lower levels pay the costs of change, or it can seek to impose them on
other organizations in its environment (p. 513).

Ouchi, William G. (1980). Markets, Bureaucracies and Clans. Administrative


Science Quarterly, 25(1): 129-141.

 Bureaucracies can fail when the ambiguity of performance evaluation become


significantly greater than that which brings about market failure. A
bureaucratic organization operates fundamentally according to a system of
hierarchical surveillance, evaluation and direction. In such a system, each

665 | P a g e
superior must have a set of standards to which he can compare behavior or
output in order to provide control. These standards only indicate the value of
an output approximately and are subject to idiosyncratic interpretation. People
perceive them as equitable only as long as they believe that they contain a
reasonable amount of performance information. When tasks become highly
unique, completely integrated, or ambiguous for other reasons, then even
bureaucratic mechanisms fail. Under these conditions, it becomes impossible
to evaluate externally the values added by any individual.. Any standard which
is applied will be by definition arbitrary and therefore inequitable.

Scott, W. R. (1998). Organizations: Rational, Natural and Open Systems. Upper


Saddle River, NJ: Prentice Hall.

 Quite diverse conceptions of organizations are held by various analysts and


associated with each of these conceptions will be a somewhat distinctive set of
criteria for evaluating the effectiveness of organizations (p.344).

 Other important bases of diversity include time perspective and level of


analysis. The criterion employed may vary depending on whether a relatively
shorter or longer time frame is adopted. How critical a time frame is may
depend on how rapidly the environment is changing. Level of analysis is a
critical factor in accounting for the variations in effectiveness criteria. Our
conclusions concerning the relative effectiveness of organizations will vary
greatly depending on whether we emphasize their impact on individual
participants on the organizations itself or on broader external systems (p. 345-
346).

 It is important to emphasize that when we speak of goals in relations to


ascertaining the effectiveness of organizations, we are focusing on the use of
goals to supply evaluation criteria. Varying goals will beheld by different
participants groups and constituencies in organizations, another consideration
complicating the examination of effectiveness is the recent challenges to the
assumption that organizations necessarily exhibit a unified or consistent set of
performances (p.347-348).

666 | P a g e
 TQM has fostered ideas in which the concept of effectiveness has begun to be
displaced by an emphasis on quality and a customer focus.

 Organizations demanding high reliability place extraordinary demands on all


organizational components. They represent systems that devote much energy
and attention to monitoring their own performance- but they do not fare well
under the present conditions of increasing production pressures and reduced
resources facing many organizations (p.351).

 The effectiveness of market –controlled organizations is directly determined by


their customers: if their interests are satisfied then they will continues to
supply the inputs required by the organization; if not then they can withhold
their contributions, causing the organization to suffer and perhaps ultimately
fail. (p.351).

 Many public organizations operate in non-market environments.


Downs(1967:25&30) employs as his major criterion for defining a government
bureau the condition that the major portion of its output is not directly or
indirectly evaluated in any markets external to the organizations by means of
voluntary quid pro quo transactions. This means that there is no direct
relationship between the services a bureau provides and the income it receives
in providing them. Attention to the institutional aspects of environments will
show that such organizations are subject to extensive controls but of a different
type: controls that emphasize process over outcome indicators of performance

 Non-market organizations are particularly likely to lack clear output measures


(p.353).

 When effectiveness can be viewed as comprising such varied criteria as


flexibility, low turnover and growth, we must seek explanatory variables as
varied as what they are asked to account for. Given that many of the proposed
measures of effectiveness are uncorrelated or even negatively correlated, we
should expect to find general explanations that will distinguish effective from
ineffective organizations. Given the complexity and openness of organizations
we would not expect any single set of factors to account for organizational
effectiveness (p.361).

667 | P a g e
 Scott suggested that Ouchi’s Theory Z and Waterman’s In Search of Excellence
convey the message that organizational performance is primarily a matter of
management effectiveness- of management’s capacity to create the right culture
and motivational structure. He suggested they overlooked factors such as
industrial locations, positioning in markets, alliances, information systems and
institutional supports. Effective organizations require more than enthusiastic
managers and motivated workers.

Baile, Kenneth C. (1998). A Study of Strategic Planning in Federal


Organizations. An unpublished dissertation.

 Many writers in the area hold that to be successful, the planning and
implementation process should have specific elements that reflect the unique
nature of the organization and its environment (Eadie, 1989; Koteen, 1991).

 Others argue that the unique characteristics of public organizations demand an


approach to strategic planning that allows for bargaining, opportunism and
response to dynamic forces in the political environment (Isenberg, 1987;
Lindbloom, 1965; Miller, 1989; Mintzberg, 1987; Mintzberg, 1994).

 The reluctance to embraces strategic planning is based on concerns as the


following:

o Planning is driven, usually by the yearly appropriations cycle.

o There is less control over administrative systems in bureaucracies as


compared to businesses.

o A ‘bottom line’ does not exist for most public organizations.

o Measuring progress on many social problems is difficult.

o Laws and policy established by political authority determine what the


public organization does.

 Managers in public organizations operate in amore complex environment


compared to those in the private sector. They must deal more directly with
questions of value and democratic principles must underlie the process (Ring 7
Perry, 1985).

668 | P a g e
 Political influences have a profound effect on the process and feasibility of
achieving specific objectives (Downs, 1996).

 Importing private sector strategic planning into public organizations requires


attention to the differences in context in which planning takes place.

 Policies or strategies in public organizations are more ambiguous and more


difficult to measure and frequently address broad social issues. In private
business strategy development and implementation are primarily confined to
participants within the organization and the strategy is for internal use.
Strategies in public organizations have significant external input and
implementation depends on the cooperation of administrative and political
oversight bodies and constituent groups (Campbell & Garnett, 1989).

 William Eldridge argues that cultural distinctions dictate different approaches


to strategic planning in business and government and create different
expectations for successful implementation of strategic plans

o Governments have less competition that business.

o Customer influences is likely to be weaker in government.

o Measuring governmental work performance is more difficult.

o The rapid turnover of governmental leaders causes instabilities that


inhibit the developing and sustaining of long term strategic direction for
the organization.

o Governments have more stakeholders and are subject to greater outside


influence then are private companies.

o Governments normally have far more purposes than do private


companies

o Government supervisors are more likely to view themselves as specialists


rather than managers.

 GPRA which is the most current legally mandated management reform initiative
in the federal sector suggests government strategic planning can be identical to
business. In recent years an understanding has emerged that the federal
government needs to be run in a more business like manner than in the past.

669 | P a g e
As companies are accountable to stockholders, the federal government is
accountable to taxpayers and taxpayers are demanding as never before that the
dollars they invest in their government be managed and spent responsibly
(GAO, 1996).

 Bozeman and Strausman 91990) argue that is it crucial for public managers to
understand the nature of the political influence on the organization and how
the political influence affects the practice of managing, particularly setting the
organization’s strategic agenda.

 Ring and Perry (1985) suggest that the following propositions describe the
distinction between public and private sector strategic management processes:

o Policy directives tend to be more ill defined for the public than for private
organizations.

o The relative openness of decision-making creates greater constraints for


public executives and managers than for their private sector
counterparts.

o Public sector policy makers are generally subject to more direct and
sustained influence from a greater number of interest groups than are
executives and managers in the private sector.

o Public sector management must cope with time constraints that are
more artificial than those that confront private sector management.

o Policy legitimating coalitions are less able in the public sector and are
more prone to disintegrate during policy implementation.

 Graham Toft 91989) suggested there are constraints on the adaptation of


strategic management approaches to the public sector:

o Government is near term action oriented. The long term horizon does not
fit the normal political/budget cycle.

o Public strategy is often set by the legislature which limits an agency’s


flexibility to develop and adjust strategy.

o Public planning is subject to significant public scrutiny and demands for


participation that can dilute the focus of strategy.

670 | P a g e
o Reliable analyses are often difficult and expensive.

o The bureaucratic nature of public agencies inhibits creativity and


innovation and group problem solving- important ingredients for
successful strategic planning.

o Funding limitations and the near term perspective make it difficult to


justify jobs for strategic planners in public organizations.

o Because most public organizations are labor intensive they depend on


personnel systems and policies that often lack the flexibility necessary to
support strategic plans.

 According to Nutt and Backoff (1993) ‘the framework offers a theory of


publicness applied to strategic management that specific factors for arraying
organizations along a continuum of publicness and linking the degree of
publicness to strategic management.

 Melemid & Luck 91994) identified areas they believe must be addressed is
organizations are to adopt private sector management.

o Multiple and conflicting goals

o Constraining financial , legal, contractual and organizational


practices

o Difficulties in aligning actions of individual members of the


organizations with its goals.

Q – 8: Organizations are said to be difficult to evaluate; yet some argue that


the public nature of government organizations compounds this difficulty.
Discuss the contributing factors that make public organizations difficult to
evaluate. Cite authors who argue that public organizations are more difficult
to evaluate as well as some examples of those arguing the opposite view.

James Q. Wilson noted that for public agencies, efficiency means not only one output
considered. While there may be one major goal according to which we can measure
the performance of public agency there always are some contextual goals that public
bureaucracy must try to attain. Even if we considered only major goals, we would find
that often they are difficult to measure as some public agencies exist to supply

671 | P a g e
services that are immeasurable in economic terms hence they are not supplied by the
market.

Wildavsky, Aaron (1972). The Self-Evaluating Organization. Public


Administration Review, 32(5): 509-520.

 Evaluation and organization may be contradictory terms. The ideal organization


would be self-evaluating. Organizational structure implies stability while the
process of evaluation suggests change. Organization generates commitment
while evaluation inculcates skepticism. Evaluation speaks to the relationship
between action and objectives while organization relates its activities to
programs and clientele (pp. 509-510).

 Rather than succumb to the diseases of bureaucracy, the self-evaluating


organization will be tempted to pass them on to others. The self-evaluating
organization can split itself off into “evaluating” and “administering” parts, thus
making lower levels pay the costs of change, or it can seek to impose them on
other organizations in its environment (p. 513).

Ouchi, William G. (1980). Markets, Bureaucracies and Clans. Administrative


Science Quarterly, 25(1): 129-141.

 Bureaucracies can fail when the ambiguity of performance evaluation become


significantly greater than that which brings about market failure. A
bureaucratic organization operates fundamentally according to a system of
hierarchical surveillance, evaluation and direction. In such a system, each
superior must have a set of standards to which he can compare behavior or
output in order to provide control. These standards only indicate the value of
an output approximately and are subject to idiosyncratic interpretation. People
perceive them as equitable only as long as they believe that they contain a
reasonable amount of performance information. When tasks become highly
unique, completely integrated, or ambiguous for other reasons, then even
bureaucratic mechanisms fail. Under these conditions, it becomes impossible
to evaluate externally the values added by any individual.. Any standard which
is applied will be by definition arbitrary and therefore inequitable.

672 | P a g e
Scott, W. R. (1998). Organizations: Rational, Natural and Open Systems. Upper
Saddle River, NJ: Prentice Hall.

 Quite diverse conceptions of organizations are held by various analysts and


associated with each of these conceptions will be a somewhat distinctive set of
criteria for evaluating the effectiveness of organizations (p.344).

 Other important bases of diversity include time perspective and level of


analysis. The criterion employed may vary depending on whether a relatively
shorter or longer time frame is adopted. How critical a time frame is may
depend on how rapidly the environment is changing. Level of analysis is a
critical factor in accounting for the variations in effectiveness criteria. Our
conclusions concerning the relative effectiveness of organizations will vary
greatly depending on whether we emphasize their impact on individual
participants on the organizations itself or on broader external systems (p. 345-
346).

 It is important to emphasize that when we speak of goals in relations to


ascertaining the effectiveness of organizations, we are focusing on the use of
goals to supply evaluation criteria. Varying goals will beheld by different
participants groups and constituencies in organizations, another consideration
complicating the examination of effectiveness is the recent challenges to the
assumption that organizations necessarily exhibit a unified or consistent set of
performances (p.347-348).

 TQM has fostered ideas in which the concept of effectiveness has begun to be
displaced by an emphasis on quality and a customer focus.

 Organizations demanding high reliability place extraordinary demands on all


organizational components. They represent systems that devote much energy
and attention to monitoring their own performance- but they do not fare well
under the present conditions of increasing production pressures and reduced
resources facing many organizations (p.351).

 The effectiveness of market –controlled organizations is directly determined by


their customers: if their interests are satisfied then they will continues to
supply the inputs required by the organization; if not then they can withhold

673 | P a g e
their contributions, causing the organization to suffer and perhaps ultimately
fail. (p.351).

 Many public organizations operate in non-market environments.


Downs(1967:25&30) employs as his major criterion for defining a government
bureau the condition that the major portion of its output is not directly or
indirectly evaluated in any markets external to the organizations by means of
voluntary quid pro quo transactions. This means that there is no direct
relationship between the services a bureau provides and the income it receives
in providing them. Attention to the institutional aspects of environments will
show that such organizations are subject to extensive controls but of a different
type: controls that emphasize process over outcome indicators of performance

 Non-market organizations are particularly likely to lack clear output measures


(p.353).

 When effectiveness can be viewed as comprising such varied criteria as


flexibility, low turnover and growth, we must seek explanatory variables as
varied as what they are asked to account for. Given that many of the proposed
measures of effectiveness are uncorrelated or even negatively correlated, we
should expect to find general explanations that will distinguish effective from
ineffective organizations. Given the complexity and openness of organizations
we would not expect any single set of factors to account for organizational
effectiveness (p.361).

 Scott suggested that Ouchi’s Theory Z and Waterman’s In Search of Excellence


convey the message that organizational performance is primarily a matter of
management effectiveness- of management’s capacity to create the right culture
and motivational structure. He suggested they overlooked factors such as
industrial locations, positioning in markets, alliances, information systems and
institutional supports. Effective organizations require more than enthusiastic
managers and motivated workers.

Baile, Kenneth C. (1998). A Study of Strategic Planning in Federal


Organizations. An unpublished dissertation.

674 | P a g e
 Many writers in the area hold that to be successful, the planning and
implementation process should have specific elements that reflect the unique
nature of the organization and its environment (Eadie, 1989; Koteen, 1991).

 Others argue that the unique characteristics of public organizations demand an


approach to strategic planning that allows for bargaining, opportunism and
response to dynamic forces in the political environment (Isenberg, 1987;
Lindbloom, 1965; Miller, 1989; Mintzberg, 1987; Mintzberg, 1994).

 The reluctance to embraces strategic planning is based on concerns as the


following:

o Planning is driven, usually by the yearly appropriations cycle.

o There is less control over administrative systems in bureaucracies as


compared to businesses.

o A ‘bottom line’ does not exist for most public organizations.

o Measuring progress on many social problems is difficult.

o Laws and policy established by political authority determine what the


public organization does.

 Managers in public organizations operate in amore complex environment


compared to those in the private sector. They must deal more directly with
questions of value and democratic principles must underlie the process (Ring 7
Perry, 1985).

 Political influences have a profound effect on the process and feasibility of


achieving specific objectives (Downs, 1996).

 Importing private sector strategic planning into public organizations requires


attention to the differences in context in which planning takes place.

 Policies or strategies in public organizations are more ambiguous and more


difficult to measure and frequently address broad social issues. In private
business strategy development and implementation are primarily confined to
participants within the organization and the strategy is for internal use.
Strategies in public organizations have significant external input and

675 | P a g e
implementation depends on the cooperation of administrative and political
oversight bodies and constituent groups (Campbell & Garnett, 1989).

 William Eldridge argues that cultural distinctions dictate different approaches


to strategic planning in business and government and create different
expectations for successful implementation of strategic plans

o Governments have less competition that business.

o Customer influences is likely to be weaker in government.

o Measuring governmental work performance is more difficult.

o The rapid turnover of governmental leaders causes instabilities that


inhibit the developing and sustaining of long term strategic direction for
the organization.

o Governments have more stakeholders and are subject to greater outside


influence then are private companies.

o Governments normally have far more purposes than do private


companies

o Government supervisors are more likely to view themselves as specialists


rather than managers.

 GPRA which is the most current legally mandated management reform initiative
in the federal sector suggests government strategic planning can be identical to
business. In recent years an understanding has emerged that the federal
government needs to be run in a more business like manner than in the past.
As companies are accountable to stockholders, the federal government is
accountable to taxpayers and taxpayers are demanding as never before that the
dollars they invest in their government be managed and spent responsibly
(GAO, 1996).

 Bozeman and Strausman 91990) argue that is it crucial for public managers to
understand the nature of the political influence on the organization and how
the political influence affects the practice of managing, particularly setting the
organization’s strategic agenda.

676 | P a g e
 Ring and Perry (1985) suggest that the following propositions describe the
distinction between public and private sector strategic management processes:

o Policy directives tend to be more ill defined for the public than for private
organizations.

o The relative openness of decision-making creates greater constraints for


public executives and managers than for their private sector
counterparts.

o Public sector policy makers are generally subject to more direct and
sustained influence from a greater number of interest groups than are
executives and managers in the private sector.

o Public sector management must cope with time constraints that are
more artificial than those that confront private sector management.

o Policy legitimating coalitions are less able in the public sector and are
more prone to disintegrate during policy implementation.

 Graham Toft 91989) suggested there are constraints on the adaptation of


strategic management approaches to the public sector:

o Government is near term action oriented. The long term horizon does not
fit the normal political/budget cycle.

o Public strategy is often set by the legislature which limits an agency’s


flexibility to develop and adjust strategy.

o Public planning is subject to significant public scrutiny and demands for


participation that can dilute the focus of strategy.

o Reliable analyses are often difficult and expensive.

o The bureaucratic nature of public agencies inhibits creativity and


innovation and group problem solving- important ingredients for
successful strategic planning.

o Funding limitations and the near term perspective make it difficult to


justify jobs for strategic planners in public organizations.

677 | P a g e
o Because most public organizations are labor intensive they depend on
personnel systems and policies that often lack the flexibility necessary to
support strategic plans.

 According to Nutt and Backoff (1993) ‘the framework offers a theory of


publicness applied to strategic management that specific factors for arraying
organizations along a continuum of publicness and linking the degree of
publicness to strategic management.

 Melemid & Luck 91994) identified areas they believe must be addressed is
organizations are to adopt private sector management.

o Multiple and conflicting goals

o Constraining financial , legal, contractual and organizational


practices

o Difficulties in aligning actions of individual members of the


organizations with its goals.

Q – 9: Review the alternative definitions of “strategy” as offered by Mintzberg


and point out which of the definitions are more suitable for characterizing
strategic planning in the public sector.

Mintzberg provides five definitions of strategy:

1. Strategy as Plan: suitable for public sector

Some sort of consciously intended course of action, a guideline (or set of guidelines) to
deal with a situation. By this definition, strategies have two essential characteristics:
they are made in advance of the actions to which they apply, and they are developed
consciously and purposefully. Intended or deliberate strategy. As plan, strategy deals
with how leaders try to establish direction for organizations, to set them on
predetermined courses of action. Strategy as plan also raises the fundamental issue
of cognition – how intentions are conceived in the human brain and what intentions
really mean.

2. Strategy as Ploy:

678 | P a g e
A specific maneuver intended to outwit an opponent or competitor. As ploy, strategy
takes us into the realm of direct competition, where threats and feints and various
other maneuvers are employed to gain advantage.

3. Strategy as Pattern: suitable for public sector

A pattern in a stream of actions. By this definition, strategy is consistency in


behavior, whether or not intended. Realized or emergent strategy. As pattern,
strategy focuses on action, reminding us that the concept is an empty one if it does
not take behavior into account. Strategy as pattern also introduces the notion of
convergence, the achievement of consistency in an organization’s behavior.

4. Strategy as Position:

A means of locating an organization in what organization theorists like to call an


“environment”. By this definition, strategy becomes the mediating force, or match,
between organization and environment (the internal and external context). As
position, strategy encourages us to look at organizations in their competitive
environments – how they find their positions and protect them in order to meet
competition, avoid it, or subvert it.

5. Strategy as Perspective: suitable for public sector

Content consisting not just of a chosen position, but of an ingrained way of perceiving
the world. Strategy in this respect is what personality is to the individual. This
definition suggest that strategy is a concept. Implication that all strategies are
abstractions which exist only in the minds of interested parties. The perspective is
shared by the members of an organization through their intentions and/or by their
actions. Entering the realm of the collective mind – individuals united by common
thinking and/or behavior. As perspective, strategy raises intriguing questions about
intention and behavior in a collective context. If we define organization as collective
action in the pursuit of common mission, then strategy as perspective raises the issue
of how intentions diffuse through a group of people to become shared as norms and
values, and how patterns of behavior become deeply ingrained in the group.

Q – 10: What are the different kinds of strategies according to Mintzberg?

Planned Strategy: precise intentions are formulated and articulated by a central


leadership, and backed up by formal controls to ensure their surprise-free

679 | P a g e
implementation in an environment that is benign, controllable, or predictable (to
ensure no distortion of intentions); these strategies are highly deliberate.

Entrepreneurial Strategy: intentions exist as the personal, unarticulated vision of a


single leader, and so are adaptable to new opportunities; the organization is under the
personal control of the leader and located in a protected niche in its environment;
these strategies are relatively deliberate but can emerge as well.

Ideological Strategy: intentions exist as the collective vision of all the members of
the organization, controlled through strong shared norms; the organization is often
proactive vis-à-vis its environment; these strategies are rather deliberate.

Umbrella Strategy: a leadership in partial control of organizational actions defines


strategic targets or boundaries within which others must act; as a result, strategies
are partly deliberate (the boundaries) and partly emergent (the patterns within them);
this strategy can also be called deliberately emergent, in that the leadership
purposefully allows others the flexibility to maneuver and form patterns within the
boundaries.

Process Strategy: the leadership controls the process aspects of strategy (who gets
hired and so gets a chance to influence strategy, what structures they work within,
etc.), leaving the actual content of strategy to others; strategies are again partly
deliberate (concerning process) and partly emergent (concerning content), and
deliberately emergent.

Disconnected Strategy: members or subunits loosely coupled to the rest of the


organization produce patterns in the streams of their own actions in the absence of, or
in direct contradiction to, the central or common intentions of the organization at
large; the strategies can be deliberate for those who make them.

Consensus Strategy: through mutual adjustment, various members converge on


patterns that pervade the organization in the absence of central or common intentions;
these strategies are rather emergent in nature.

Imposed Strategy: the external environment dictates patterns in actions, either


through direct imposition or through implicitly preempting or bounding organizational
choice; these strategies are organizationally emergent, although they may be
internalized and made deliberate.

680 | P a g e
Q – 11: What arguments does he make in the Rise and Fall/The Fall and Rise of
Strategic Planning?

This article presents Professor Mintzberg’s warning that strategic thinking is as


important as strategic planning; the two are not the same and are mutually exclusive
(planning must be systematic and thinking must be creative). Strategic planning
involves analysis. Strategic thinking involves synthesis, and synthesis comes from
intuition and creativity. Planning can be done on a time schedule but thinking must
be “free to appear at any time and at any place in the organization.” Mintzberg notes
three fallacies of strategic planning: (1.) the fallacy of prediction – while repetitive
patterns and seasons do occur, predicting future events is virtually impossible; (2.) the
fallacy of detachment – strategy should be made in concert with, not detached from
those who really do the work; and (3.) the fallacy of formalization – formal procedures
stifle synthesis and creativity. He warns that organizations not go over the
formalization edge. Both analytic thinkers and creative thinkers are needed in all
organizations to develop strategic decisions.

Q – 12: Do his arguments hold true for both the public and private sectors?

This is left to interpretation, but I believe the answer is yes, these arguments hold true
for both public and private organizations. Accountability issues may differ between
the sectors, but these arguments apply to both.

Q – 13: Who are the The Big Names in Budgeting.

Brownlow, Louis 1937: Committee Report to FDR

The president needs help

Caiden, Naomi 1981: Public Budgeting Amidst Uncertainty and Instability

Uncertainty and stress must be taken into account


in budgeting.

Cleveland, Frederick A. Evolution of the Budget Idea in the United States


1918: (pp. 7-23 in Government Budgeting by Hyde)

This article was the justification of the Taft

681 | P a g e
commission which led to executive budgeting.

Forrester, John P.  Public Choice Theory and Public Budgeting:


2001: Implications for the Greedy Bureaucrat (pp.
101-116 in Evolving Theories of Budgeting,
John R. Bartle, ed.)

 Public choice theory says self interest and the


free market, explain why people (including
bureaucrats) make the decisions they do.
Public budgeting is different but public
budgeting can learn from public choice
theory.

Gulick, Luther 1937: POSDCORB

Planning, organizing, staffing, directing,


coordinating, reporting, budgeting

Harding, Warren G. 1921: signed The Budget and Accounting Act

Created the Bureau of the Budget, GAO, and


executive budgeting

Focused on accountability and control

Line item budgeting

Key, V.O. 1940: The Lack of a Budgetary Theory

“On what basis shall it be decided to allocate x


dollars to activity A instead of Activity B?”

Budgeting is “applied economics”---allocating scarce


resources

The budget must reflect the public’s interest

682 | P a g e
Keynes, John Maynard
1936:
The General Theory of Employment, Interest and
Money

Keynesian Theory: fiscal policy (government


spending
& taxing policies) can, and should be used to
influence the economy

Lewis,Verne 1952: Toward a Theory of Budgeting (see p. 29, Hyde)

He is a rationalist---but in the end concludes that


Key’s questions is unanswerable

He said “for x level of funding, y level of service can


be provided”

Doctrine of Public Utility---marginal utility;


diminishing return (i.e., battleships of poor relief)

The cost of a thing is the amount of other things we


must give up for its sake.

We must make choices---i.e., 4 or 5 tires for a car,


but not 6.

Lindblom, Charles 1959: The Science of Muddling Through

Osborne, David and Gaebler, Reinventing Government, How the Entrepreneurial


Ted 1992: Spirit is Transforming the Public Sector Led to
Government Performance and Results Act and
National Performance Review---1993

Ostrum, Vincent 1974: The Intellectual Crisis in American Public


Administration

He supported public choice theories

683 | P a g e
Rubin, Irene 1990: Budget Theory and Budget Practice: How Good the
Fit? (see p. 77, Hyde)

No single theory explains budgeting---they are


fragmented and incomplete

Budgeting is complex

Neo-Marxist, public choice, incrementalism

Rational (to maximize return), non-rational, now


combination

Schick, Allen 1966: The Road to PPB: The Stages of Budget Reform (see
p. 52, Hyde)

A classic---it traced the evolutionary cycle of


budgeting reform from line item/strict expenditure
control to performance
budget/management – work efficiency

Taft, William Howard Taft Commission presented The Need for a National
1912: Budget

Led to Budget and Accounting Act of 1921

Wildavsky, Aaron 1964/84: The Politics of the Budgeting Process

The best expression of Incrementalism---a


descriptive theory of budgeting

Budgeting is political --- Congressional influence is


very important

Budget reform is political system reform

A normative theory of budgeting is possible but very


difficult

Redistribution is behind all budget decisions

684 | P a g e
His later writings were less incrementalism-oriented

Rescuing Policy Analysis form PPBS


1969:
No one knows how to do PPBS

Willoughby, William F. 1918: The Movement for Budgetary Reform in the States
(see p. 20 in Hyde)

The public should be involved in the budget process

Part of the Progressive movement in the early 1900’s

Q – 14: What are the possible limits of quantitative policy analysis? Can
qualitative analysis help the policy analyst to overcome all of them.

Research in the social sciences uses several different methods to answer questions.
The experimental method is used with the quantification of data, the process of
converting data to a numerical format (Babbie, 396) and used in evaluation research.
Another method is quasi-experimental – non-rigorous inquiries somewhat resembling
controlled experiments but lacking key elements such as pre-and post-testing and/or
control groups (Babbie, 349). The third type of method is the qualitative evaluation

Babbie: Although quantification makes observations more explicit, it has the


disadvantage of potential loss in richness of meaning. More aligned with nomothetic
explanation (settles for partial versus full explanation – seek to identify few causal
factors that generally impact a class of conditions or events). Qualitative (purely
verbal) has disadvantage of ambiguity. Aligned more with idiographic explanation
(seek to exhaust idiosyncratic causes of a particular condition or event).

Bailey: Limits of quantitative research are such that the social scientist risks
producing reliable but insignificant “so what” results. Strict control of variables to
alleviate interpretive variances could eliminate possibility for theory building. Kuhn
says that the accumulation of anomalies leads to identification of new scientific
paradigms.

Rossman and Wilson (1985): Utilizing both quantitative and qualitative data allows
triangulation which improves the accuracy of conclusions by relying on data from

685 | P a g e
more than one method. Corroboration (seeks convergence in findings) and elaboration
(provides richness of detail). Holistic triangulation may also uncover paradox and
contradiction (initiation).

Since quantitative analysis relies on numbers or being able to convert data to an


ordinal scale, it is sometimes unrealistic. Numbers are not always available.

The paradigmatic stance helps determine the best form of analysis.

Mixed-method approach is safest. Consider deductive versus inductive.

Yin

 case study allows investigators to retain the holistic and meaningful


characteristics of real life events;

White/Adams

 technical rationality exhibits a faith in the power of science and technology to


liberate us; blind to detrimental effects of relying solely on quantification; belief
that research is rigorous and objective;
 quantitiative assumes human action is to be explained through the
development of general laws and models independent of time and space; no
need to include history and culture to account for human behavior

Box

 qualitative methodologies as alternative theoretical approaches in contrast to


positivism
 cited C. Wright mills 91959) extremist positivist position – abstracted
empiricism
 reality may consists of a complex mixture of perception, measurable
phenomena, interpretation, and assessment of the pragmatic usefulness of
theory for practice instead of solely the ability to measure phenomena
quantitatively
 advocates a particular view of knowledge

686 | P a g e
Dunn

 Problem solving is the key element of the methodology of policy analysis


 Policy analysis is partly descriptive and also normative in the aim of creation
and critique of knowledge claims about the values of policies
 Complexity – critical multiplism; triangulation
 Multiplism has advantage over rivals: “Approximating the ultimately
unknowable truth though the use of processes that critically triangulate from a
variety of perspectives on what is worth knowing and what is known” (p.6)
 problem structuring is embedded in a political process where the definition of
alternatives is the supreme instrument of power

Bednarz

 Quantitative and qualitative approaches can not be synthesized because they


occupy alternative rather than complementary philosophical spaces
 Many researchers trained in quasi-experimental methods and multivariate
analysis find it difficult to grasp the so-called qualitative and qualitative
methods did not spring from the same epistemological ground
 Qualitative knowing underlies all quantitative knowing
 Qualitative knowledge provides the context necessary for the interpretation of
quantitative data
 Quantitative scientists can be questioned for ignoring qualitative dimensions
underlying aggregate statistics and data

Dr. Linda Mayoux “Qualitative Methods”:

Qualitative methods contrast with quantitative methods in a number of important


respects which they share with participatory methods. They are:

 holistic ie development is seen as an interconnected process with many


different dimensions. A key focus of investigation is not on the different
dimensions in isolation e.g. separating out economic and social impacts, but
understanding the interlinkages and tensions between them. Whereas
quantitative methods seek to separate and simplify indicators and impact
processes in order to measure them, qualitative methods seek to understand
the complexity as a more accurate reflection of reality.

687 | P a g e
 based on recognition of multiple realities where reality is seen as inherently
subjective. The focus is on understanding different perceptions, aspirations
and interests and how these influence accounts of ' facts ' and events rather
than attempting to reduce them to one version of reality. For example women
and men may have different accounts of levels of income and/or roles in
household decision-making. Different stakeholders may have different
perceptions of power relations within organizations. Qualitative methods treat
these differences as interesting in themselves as indicators of relative power and
as possible explanatory factors in differential impacts of development
interventions.
 heuristic, interpretative and inductive ie qualitative research evolves rather
than restricts itself to predetermined questions or hypotheses. Any assessment
starts with an intensive familiarisation with the context, institutions and
policies to be assessed and progressively builds up a comprehensive
understanding of the processes involved. Because of the emphasis on
understanding complexity the scope and focus of the research are continually
redefined as understanding of different parts of the process increases and new
issues arise.
 requires in-depth face-to-face field work. Because of the need to relate all
these different dimensions together in the cumulative understanding of a
particular context, it is more difficult to delegate or divide up different parts of
the qualitative investigation between different people. Skilled (and hence more
expensive) researchers typically spend long periods in the field rather than
delegating field research and questionnaires to less-skilled enumerators,
although in the field they may closely supervise local researchers to collect less
difficult information.

In the above respects qualitative methods are broadly similar to participatory


methods. However qualitative methods also contrast with participatory
methods in a number of important respects:

 central role of the outside researcher in design, research and analysis.


Although there is the focus on multiple realities, the ways in which these are
investigated and the analysis of their significance lies largely with the

688 | P a g e
researcher rather than being an open-ended process to be determined by
participants.
 there is a focus on information from individuals: although qualitative
methods may be used to compile case studies or observe groups and
communities, there is much more of an emphasis on individual information.
This makes it possible to ask much more sensitive probing questions which
people would not like to answer in a public forum.
 the investigation records what is happening rather than seeking to
influence events: a key difference between qualitative and participatory
methods is that qualitative methods seek to understand current events rather
than intervening to change future events. Although recording individual
accounts may aim to empower people and influence policy through making
them more visible, there is no attempt to integrate qualitative research with
empowerment and policy development. This may make the data more reliable in
some respects as people are less liable to manipulate information in expectation
of beneficial outcomes or fear of unwanted consequences. ]
CONTRIBUTIONS OF QUALITATIVE RESEARCH

Because of these distinctive principles and characteristics, qualitative methods are


useful compliments to quantitative and participatory methods in order to:

Increase understanding of WHAT is happening.

 Qualitative methods are useful in informing the selection of criteria and


indicators, highlighting any limitations or complexities and hence assisting in
their interpretation.

 Qualitative methods are also often necessary to investigate more complex and
sensitive impacts which are not so easy to quantify or where quantification
would be extremely time-consuming and costly. They are also used to
investigate more sensitive issues which cannot be easily aired in the public
forum of participatory methods.

Contribute to understanding of WHO is affected in which ways.

 Qualitative methods highlight the voices of those who are most disadvantaged
in ways which might be difficult to the public and consensual nature of

689 | P a g e
participatory methods or missed in the process of aggregation of quantitative
methods.

 Qualitative methods can also be used for probing of key informants to further
investigate issues of diversity and conflict.

Analyse WHY particular impacts are occurring.

 Qualitative methods enable more probing investigation of contexts and


development processes and the complex interactions between contexts,
grassroots aspirations and strategies, institutional structures and enterprise
interventions.

Assessing HOW POLICY CAN BE IMPROVED.

 Qualitative methods are likely to be necessary in investigating more complex


and sensitive issues essential to understanding the feasibility of proposals from
participatory workshops.

Q – 15: What are the requisites necessary for researchers to conclude that a
causal relationship exists? Why is this the case?

The three main criteria for causal relationships in social research are 1) variables
must be correlated – there is an actual relationship, 2) the cause takes place before
the effect – time order, and 3) the variables are non-spurious – there is not a third
variable effecting the relationship (Babbie, 90). Note these definitions:

Correlation – An empirical relationship between two variables such that 1) changes in


once are associated with changes in the other or 2) particular attributes of one
variable are associated with particular attributes of the other. Correlation in and of
itself does not constitute a causal relationship between the two variables, but is one
criterion of causality (Babbie, 90).

Time order – We can’t say a causal relationship exists unless the cause precedes the
effect in time (Babbie, 90).

An alternate answer: there are four criteria to conclude that a causal relationship
exists. The criteria are 1) time order – If A is the cause of B, then A must precede B in
time. Also, Changes in A must occur before changes in B. So, cause must precede
effect. 2) Covariation – means that the two variables move or vary together. If A

690 | P a g e
changes and B also changes, this covariation provides some evidence that A is the
cause of B. If changes in are never accompanied by changes in B, then A cannot be
the cause of B. 3) Non-spuriousness – a relationship is an association between two
variables that cannot be explained by a third factor. 4) Theory – Not only must the
conditions of time order, covariation, and non-spuriousness be satisfied, but also a
theoretical or substantive justification or explanation for the relationship must be
provided. Theory interprets the observed covariation; it addresses the issue of how
and why the relationship occurs (Meir, K. J. & Brudney, J.L., 32-34).

Must use experimental method. Why? To determine causal versus mere correlation
(no direction).

Bednarz

 Causal knowledge about society is currently limited to the tentative partial and
probabilistic realm (Cook & Campbell)

 Complexity of causal interconnections of the social world are prohibitive

Causation and Correlation

The ability to determine causal connections in the world is important. What connects
the cause and the effect is invisible to us (Hume).1 But we can take notice of
correlations and from these sometimes draw conclusions about causal relationships.
Not all correlations exist because there is a causal relationship.

Correlations

Statements of correlation express a relation between two properties (the values of


variables) within a single population.

Smokers Non-smokers

American males 51 49

American females 34 66

691 | P a g e
From this data we assert that 51% of American males smoke and 34% of American
females smoke. The property of being an American male is positively correlated with
the property of being a smoker, and the property of being an American female is
negatively correlated with being a smoker. The population here is adult Americans and
we are comparing two variables: smoking and gender; each variable has two values.

A is positively correlated with B if and only if the percentage of As among Bs is greater


than the percentage of As among non-Bs.

A is negatively correlated with B if and only if the percentage of As among Bs is less


than the percentage of As among non-Bs.

A is not correlated with B is the percentage of As among Bs is the same as the


percentage of As among non-Bs.

Judging correlations

Attentional bias in judging correlations:

Nurses were asked to view 100 cards with patient information on them and then judge
whether there was a relationship or connection between a particular symptom and a
particular disease. Each card indicated whether the symptom was present or absent
and whether the disease was present or absent. (Smedslund, 1963)

Here is the incidence of symptom and disease for 100 patients.

Disease No disease

Symptom 37 33

No
17 13
symptom

Results:

692 | P a g e
There is no correlation here though 85% of the nurses thought there was a positive
correlation between the symptom and the disease. The present/present cell was the
best predictor of the subject's judgments; a high figure in that cell prompted a positive
judgment.

Notice that for both the symptom group and the non-symptom group about as many
have the disease as don't have the disease (slightly more have it than don't have it for
both groups; 37-33 with symptom, 17-13 without symptom). Whether you have the
disease or not, about twice as many have the symptom as don't have it.

Subjects are inclined to look only at select cells for pertinent information.

Another example: Does God answer prayers? Many say yes because many time
prayers were successful. But what about the other cells?

Another example:

Subjects were asked whether Mr. Maxwell, a fictional person they were asked to
imagine that they met at a party, was a professor. They were told he was either a
professor or an executive, and that he belonged to the Bear's Club. Subjects were then
asked what additional information they would like to have to make their judgment. For
example, what percentage of professors at the party are members of the Bear Club, or
what percentage of executives at the party were members of the Bear Club? 89% of the
subjects wanted the first piece of information, but only 54% wanted the second piece,
even though both pieces are relevant. (Also relevant is the information regarding the
percentage of professors at the party.)

The effects of prior belief in judging correlations:

Clinical psychologists sometimes use Draw-a-person tests by which the patients are
thought to projects aspects of their personalities into the drawings. Big eyes might
indicate the patient is suspicious of others or paranoid; big shoulders might indicate a
preoccupation with manliness.

Studies have shown these tests to be useless as indicators of personality traits. But in
studies in which pictures and trait-labels are associated in ways that reflect no

693 | P a g e
correlations, untrained subjects still claim to "discover" that certain traits are
correlated with certain aspects of the drawings. Even professionals maintain
confidence in them after learning of their inefficacy. Similar results apply to Rorschach
tests. Quote: "I know paranoids don't seem to draw big eyes in the research lab, but
they do in my office." (Chapman and Chapman, 1967, 1969)

Prior belief can increase attentional bias:

Subjects are told of an experiment in which boarding school children are given certain
combinations of food to see whether they affect the likelihood of getting a cold. Before
seeing the data the subjects are asked to formulate their own hypotheses. Once shown
the data, their interpretations are clearly influenced by their own hypotheses. Even
though the data reflect no correlations, subjects who hypothesized beforehand that the
type of water (bottled or tap) might be relevant to getting a cold also said they saw
such a correlation exemplified in the data. Subjects who, for example, hypothesized
that the type of mustard would cause colds would look to the mustard/cold data and
ignore the mustard/no cold data.

Causal relationships:

A causal generalization, e.g., that smoking causes lung cancer, is not about an
particular smoker but states a special relationship exists between the property of
smoking and the property of getting lung cancer. As a causal statement, this says
more than that there is a correlation between the two properties.

Some causal conditions are necessary conditions: the presence of oxygen is a


necessary condition for combustion; in the absence of oxygen there is no combustion.
"Cause" is often used in this sense when the elimination of the cause is sought to
eliminate the effect (what's causing the pain?)

Some causal conditions are sufficient conditions: the presence of a sufficient condition
the effect must occur (being in temperature range R in the presence of oxygen is
sufficient for combustion of many substances. "Cause" is often used in this sense
when we seek to produce the effect (What causes this metal to be so strong?)

694 | P a g e
Looking for special circumstances: what was the cause of the fire? Oxygen? or an
arsonist's match?

Causes are sometimes said to be INUS conditions in that they are Insufficient but
Necessary parts of an Unnecessary but Sufficient set of conditions for the effect.
Striking a match may be said to be a cause of its lighting. Suppose there is some set of
conditions that is sufficient for a match's lighting. This might include the presence of
oxygen, the appropriate chemicals in the matchhead and the striking. The striking can
be said to be a necessary part of this set (though insufficient by itself) because without
the striking among those other conditions the match would not have lit. But the set
itself, though sufficient, is not necessary because other sets of conditions could have
produced the lighting of the match.

How are causal relationships different from correlations?

1. A statement about a correlation is symmetrical while a statement about a causal


relationship is asymmetrical. If being a male is positively correlated with being a
smoker, being a smoker is also positively correlated with being male. But if smoking
causes lung cancer it needn't be the case that lung cancer causes smoking.

2. Correlations are about actual populations and are not lawlike. Causal relationships
are lawlike in the sense that they are about hypothetical populations as well as actual
populations. When A is said to be the cause of B we are saying that were there an
increase in the incidence of A there would be an increase in the incidence of B; or if A
cases were to diminish, B cases would diminish, too. (If fewer people smoked, there
would less lung cancer.) Mere correlations pertain only to actual populations. If
National League success in the Super Bowl is merely correlated with stock market
decline, then we should not expect changes in the stock market to affect the outcome
of the Super Bowl (or vice versa).

How can one form judgments about causal relationships based on statements about
correlations?

For example, there is a strong positive correlation between an increase in the number
of sex education classes and an increase in the rate of gonorrhea. Suppose we

695 | P a g e
conclude that increasing the number of sex education classes has caused the increase
in the gonorrhea rate.

(A) Is the statistical premise (the statement about the correlation) true or well
founded?

(B) What alternative explanations are available?

1. The correlation might be accidental or coincidental. Increase in the national


debt is positively correlated with an increase in the gonorrhea rate, but there is
no causal connection.

2. The relation might be spurious, both an increase in the number of sex


education classes and an increase in the rate of gonorrhea being the effects of
the same cause.

3. The causal direction might be the reverse. Could the increase in the gonorrhea
rate be causally responsible for the perceived need for more sex education
classes?

4. The causal relation might have been more complex than the conclusion
suggests. The increase in sex education classes might have caused a change in
attitudes about sex, which led to an increase in sexual activity, which led to an
increase in the gonorrhea rate.

5. The causal relation cited might be insignificant relative to other factors


responsible for the increase in the gonorrhea rate.

Is a causal relationship suggested in the cases below?

At one time there was a strong positive correlation between the number of mules in
the state and the salaries paid to professors (the more mules the lower the salaries).

There is a strong positive correlation between the number of fire trucks in a borough
of NYC and the number of fires that occur there.

There is a strong positive correlation between foot size and hand writing quality.

696 | P a g e
There is a strong negative correlation between the number of forward passes thrown in
a football game and winning the game.

Heavy coffee consumption is positively correlated with heart attacks.

Going to the hospital is positively correlated with dying.

An increase in the number of hours kids watch TV positively correlates with decrease
in SAT scores.

Marijuana use is negatively correlated with high GPAs.

Another example:

"[W]hile half the country's communities have flouridated water supplies and half do
not, ninety percent of AIDS cases are coming from flouridated areas and only ten
percent are coming from nonflouridated areas."

Any connection?

1. Communities aren't all the same size: flouridated communities (likely to be big cites)
might contain much more than half the population.

2. The relationship might be spurious: cosmopolitan/progressive attitudes might


encourage both fluoridation and lifestyles associated with AIDS

Another example:

Is there a causal relationship between class attendance and grades achieved?

"Students with the lowest attendance earned the poorest grades. Those who attended
79 percent of the classes or less ended up in the low C range; 90 percent and above
scored above a B average. Student who sat up front got 'significantly higher grades,'
but Walsh [the researcher] thinks they could be more interested in the subjects."

John Stuart Mill, A System of Logic, 1843

697 | P a g e
A is not a sufficient condition for B if A occurs without B.

A is not a necessary condition for B if B occurs without A.

The Direct Method of Agreement

Find a causal connection between an effect and a necessary condition

Which factor is always present when the effect is present?

If among the residents of a dormitory there is a rash of stomach upsets, we would


likely look for one food item that all the patients ate as the cause.

1. The conclusion applies only to the occurrences considered.

2. Only probable: other important conditions might have been overlooked; it might
have been a combination of factors

The Inverse Method of Agreement

Find a causal connection between an effect and a sufficient condition

Which factor is always absent when the occurrences of the effect are absent?

Five factory workers are found to be inefficient relative to others who are doing the
same work. The efficient workers and the inefficient workers were found to be similar
in all relevant ways except one: the inefficient were not part of a profit sharing plan.
Conclusion: profit sharing causes efficiency.

1. The conclusion applies only to the occurrences considered.

2. Only probable: other important conditions might have been overlooked; it might
have been a combination of factors

The Double Method of Agreement

Find a cause that is both a necessary and a sufficient condition

Which factor is always present when the effect is present?

698 | P a g e
Which factor is always absent when the occurrences of the effect are absent?

Eight patients have a disease and each was given some remedy or other. Four patients
who are given serum S are cured. Of those who are cured no other single remedy was
given to all. Of the four who were not cured, every patient was given at least one of the
remedies (but none the serum S). Serum S judged to be the cure.

1. The conclusion applies only to the occurrences considered.

2. Only probable: other important conditions might have been overlooked; it might
have been a combination of factors

The Method of Difference

Identify a sufficient condition among possible candidates in a specific occurrence

The factor is the only one that is present when phenomenon is present and absent
when the phenomenon is absent.

Two identical white mice in a controlled experiment were given identical amounts of
four different foods. In addition, one of the mice was fed a certain drug. A short time
later the mouse that was fed the drug became nervous and agitated. The researchers
concluded that the drug caused the nervousness.

1. Less general conclusion than the inverse method of difference, which applies to all
occurrences listed

The Joint Method of Agreement and Difference

Identify a necessary and sufficient condition that is present is a specific occurrence.

Use the direct method of agreement to isolate necessary conditions (if no factor, no
effect) and the method of difference to isolate those that are also sufficient.

1. Less general conclusion than the double method of agreement, which applies to all
occurrence listed;

699 | P a g e
George, who exercised regularly, took vitamins, and got plenty of rest, contracted a
rare disease. Doctors administered an antibiotic and the disease cleared up. convinced
that the cure was caused by either the exercise, the rest, or the antibiotic, the doctors
searched for analogous cases. Of the two that were found, one got no exercise, took no
vitamins, and got little rest. He was given the same antibiotic and was cured. The
other person, who did the same things George did, was given no antibiotic and was not
cured. The doctors concluded that George was cured by the antibiotic.

Method of Residues

"Separate from a group of causally connected conditions and phenomena those


strands of causal connection that are already known, leaving the required causal
connection as the 'residue'."

Method of Concomitant Variation

Match variations in one condition with variations in another.

Q – 17: What type of research design is most likely to enable a researcher to


conclude that a causal relationship exists? Why is this the case?

Must use experimental method. Why? To determine causal versus mere correlation
(no direction). In an experimental design, the researcher can manipulate the
independent variables.

In an experimental research study, if the researcher finds a statistically significant


difference between two or more of the groups representing different treatment
conditions, he can have some confidence in attributing causality to the IV. In non-
experimental research, the researcher has no control over the levels of the IVs (Mertler
& Vannatta, p. 2).

 How do you distinguish between internal and external validity?

Internal validity – process used to test criteria is valid (Babbie) – met by Pearson
Correlation

External validity – generalizability (Babbie) – F test tells

Also important to make certain have construct validity and reliability

Triangulation critical

700 | P a g e
Lack of internal validity refers to the possibility that the conclusions drawn from
experimental results may not accurately reflect what went on in the experiment
itself. External validity is the process of testing the validity of a measure by
examining its relationship to other presumed indicators of the same variable
(Babbie).

 Propose an experimental research design that can be utilized by public


administration analysts. In that experimental design you are to specify:

* See Dr. Stanley’s research designs on per pupil spending as examples. Babbie,
Chapter 8 good review also.

 The nature of the problem that you wish to explore and why it is
amenable to research through an experimental design.

 What is the major hypothesis that you would test; how would you
operationalize the independent and dependent variables?

Show prediction/states direction or explains past/best to state hypothesis in null


(showing no direction or relationship) so can disprove/verify falsifiability of theory
(Sir Karl Popper says the scientific status of a theory is determined by its
falsifiability, refutability, or testability). Best to keep to 1 DV and 1 IV /variables
must be measurable.

Operationalize – how collect data to measure/where find data

Conceptualize – what are you measuring/how define what measuring

“Students who participate in the A+ reading program will have similar student
achievement levels as students who do not participate in the A+ reading program.”

R OPre-Test X OPost-Test Comparison

Experimental Group OE1 XReading OE2 OE2 – OE1

Control Group OC1 OC2 OC2 - OC1

Must determine unit of analysis – determined by way research questions framed.


Here, the student is the unit of analysis.

X = achievement level

Y = A+ reading program

701 | P a g e
 How would you set up the experiment?

Experimental group and control group with pretest and post-test data.

Case studies (research strategy) are preferred strategy when how or why questions
are being posed and focus is on contemporary phenomenon within real-life context.
3 types of research questions posed are 1.) what – exploratory 2.) who and where –
descriptive 3.) how and why – explanatory. Unfortunately, case studies have low
generalizability/poor external validity. Can be both quantitative and qualitative.
Case study design must concern construct validity (studying what intended),
internal validity (nonspuriousness), external validity (generalizability), and
reliability (replicability). Multiple case study designs are preferred over single case
designs because of the possibility of direct replication. A pilot case study is helpful
to see if worthy of study.

Use linear-analytic structure in report – sequence begins with issue being studied
and a lit review followed by methods used, findings, and conclusions/implications
(Yin).

 How would you collect the data?

TCAP Test – reading test conduct pre- and post-

Snowball sampling best method for data collection (ask ?/answers → other ?s)

Population sample (ideal b/c removes chance of bias or flaws) vs. random sample
(pick every kth number – no weighting necessary when all cases have same chance
of selection – weighting should not be used since element of subjectivity)

May use Likert Scale (assign scores to patterns of responses): Strongly


Agree…Strongly Disagree (this is ordinal level)

Standardized coding scheme of ratios and % best since will not skew data – more
representative of reality (aggregate codding schemes of total #s possibly skew data)

Time series model very thorough but difficult to find consistent data

Mixed-method approach most comprehensive

Documentation, archival records, interviews, direct observations, participant


observation, physical artifacts

702 | P a g e
 How would you know that the data support or do not support the
hypothesis?

One Way ANOVA test – can only look for correlation – can’t look for causality.

Z and p scores tell whether data support the hypothesis. If z is between -1.96 and
+1.96, then safe. Outside of this range is the danger area. The p score tells the
amount of mistake you made (95% acceptance is good).

 How would your design control specifically for threats to internal and
external validity?

Employ triangulation in data collection and measurement – addresses issue of


construct validity. This checks internal validity by using multiple methodologies for
collection and external validity by using multiple methodologies for measurement.

There are 9 threats to validity (see handout), so must use many control variables such
as teachers, rural vs. metro, school funding, etc…

Must also use random selection of control group and experimental group.

Q – 18: Why is it very difficult to utilize experimental designs in the social


sciences?

There are a number of problems using experimental designs in the social sciences.
The social sciences often do not lend themselves to the controlled elements of the
experimental design. The controlling factors of the experimental design work very well
in a laboratory setting, but not so well in real-life settings. Thus, one can question
how closely the situation simulated in the laboratory resembles the processes of
everyday life. Both internal and external validity is challenged with experimental
design. In particular, three threats to internal validity are present: maturation,
reactivity, and history, and often others as there are twelve threats. External validity
of experimental designs of research is threatened by the sample of subjects upon
which the findings are based. Because of ethical and financial considerations,
experiments conducted on a random sample of individuals drawn from a well-defined
population have been rare. Funding authorities tend to take a dim view of
experimentation on human subjects and frown on the use of deception, which may be
employed in order to make an experimental setting seem more realistic to participants.
Also, the cost and practical problems of conducting an experiment on a random

703 | P a g e
sample of subjects can be prohibitive. In short, the experimental design threatens
internal and external validity, is often cost prohibitive, and existing in a laboratory
conditions may not resemble life in “real” settings.

Issues of control

Babbie:

Ethical constraints (informed consent, right to privacy/protection of identity,


participation should be voluntary and bring no harm to subject – coupled with
obligation for analysis and reporting to scientific community).

Political constraints – difficult to remain objective/intersubjectivity achieves equivalent


of objectivity for science/science should be unencumbered by personal values

Karen Payton

Ohio State University

This article addresses the strategies and tools that public administration scholars use to
understand phenomena of interest. The range of qualitative methods used has been
limited, and the kind of rigor generally associated with quantitative methods has largely
been absent in the application of their qualitative counterparts. Two conclusions are
drawn from an analysis of articles published in two respected journals: Training on
research methods in Ph.D. and M.P.A. programs should be expanded to include a
broader range of strategies and tools, and the rigorous use of a broader range of
research tools promises to better position the field of public administration to identify,
examine, and answer the many big questions that it now faces.

Key Words: qualitative research methods • empirical research • truth claims

Q – 19: What are the elements of the classical experimental design?

The elements of the classical experimental design are:

a. Randomization (R) – each subject or case has an equal chance of being


assigned to the experimental group or to the control group (random sampling)

b. Observation or Measurement 1 (pretest)

704 | P a g e
c. Administration of the experimental treatment to the experimental group, but
not the control group.

d. Observation or Measurement 2 (Posttest)

e. Compare the measurement of the two groups

Classical Experimental Design

Group Randomization Observation 1 Treatment Observation 2


Comparison

Experimental Re Oe1 X Oe2


Oe2 – Oe1

Control Rc Oc1 Oc2


Oc2 – Oc1

Step 1 Assign subjects to two or more groups, with at least one “experimental” and
one “control,” so that the groups are as comparable as possible. The best way
to assemble comparable groups is through random assignment of subjects to
groups.

Step 2 Measure all subjects on relevant variables. Although a pre-experiment


measurement or pretest is usually administered, some experimental designs do
not require a pretest.

Step 3 Expose the experimental group(s) to a treatment or stimulus, the independent


variable. Ensure that the other control group(s) is not exposed. Exposure to
the treatment should constitute the only difference between the groups.

Step 4 Measure the groups again on the requisite variables in the postexperiment
groups.

Step 5 Compare the measurements of the groups. If the independent variable does
lead to changes in the dependent variable, this result should be evident in
pretest-posttest comparisons between the experimental and control groups.
Or, if the groups are large and known to be equivalent through random
assignment, the analyst can simply compare posttest scores between the two
groups. If the causal inference is valid, these comparisons should bear out
predicted differences between the experimental and control groups.

705 | P a g e
Use X (Experimental Group) and O (Control Group)

Pretest (prior to program start), then lead intervention for a time period, then post-test

Many threats to validity exist

Q – 20: As a distinguished graduate of TSU’s Ph.D. program, the Tennessee


Legislature has contracted out your services to measure the “financial
impacts of per pupil spending on student achievement” in Tennessee,
compared to other American states. In order to determine the
associated costs of this study, the legislative sub-committee on
Education Finance in Tennessee has asked you to prepare a proposal for
this research project. Construct a research design that would
adequately measure the previously stated phenomenon and the costs
associated with such a study. Your research proposal should include
the following components:

a. Literature Review – What is the generic public administration literature to


which you would refer in this consulting assignment?

The dependent variable in this study is student achievement; therefore, I would focus
my literature review on this topic. There is a large body of literature on this
particular topic as with most topics related to education. I would examine the
literature in ERIC, education finance journals, U.S. Census Bureau, U.S.
Department of Education, and NCES (National Center for Education Statistics).
Also, public administration literature on public finance would provide some good
references for school funding and budgeting.

Use refereed and top-tier journals.

b. Hypotheses – a section containing the hypotheses tested in the manuscript


(written as null hypotheses). Provide three hypotheses minimum.

Three likely hypotheses, using “states” as the unit of analysis are:

 States with higher than average per pupil spending will have similar student
achievement results as states with lower than average per pupil spending.

706 | P a g e
 The average per pupil spending of the states in the Southeast will have
similar student achievement results as the average per pupil spending of the
states in the Northeast.

 States with similar state demographics as Tennessee, but higher per pupil
spending will have similar student achievement as Tennessee.

c. Data – Conceptual definitions: What variables are you going to use in the
manuscript and why?

 Student Achievement – as determine by the standards of No Child Left Behind


for 2004, which is standard for all states. (Interval Scale)

 Per Pupil Spending – As determined by the formula used by each state. (Interval
scale)

 State Demographics (Ho3) – Race, Gross State Product, No. Living in Poverty,
Political Affiliation, etc. (some nominal, ordinal, interval scales)

d. Operational definitions: Where you are going to collect the data you plan to
use in your manuscript.

Data collection of the variables will be from the following sources:

 Student Achievement – National Center of Education Statistics, 2004

 Per Pupil Spending – Education Weekly, or U.S. Census Bureau

 State Demographics – U.S. Census Bureau, 2004.

e. Research Methods – The statistical methods you could use to test the data
and why you believe this technique is the most appropriate for your
manuscript.

Determining correlation, not causality so use ANOVA.

f. Anticipated Findings – The anticipated findings of you study.

Anticipate rejecting all three null hypotheses which in essence states that states with
higher per pupil spending will have higher student achievement results than states
with lower per pupil spending, regardless of demographics.

g. Limitations – The anticipated limits of your study.

707 | P a g e
The method to compute per pupil spending is different in many states. Therefore, the
analysis will be adversely affected. External validity, that is generalizability, may
suffer as a result of the different methods to compute per pupil spending. Also, the
demographic data may be used to establish comparable states (null hypothesis 3)
in a demographic sense, but state policies related to education and education
funding may be different. Totally comparable states would have similar
demographics and education policies.

h. Conclusion – The anticipated policy implications of the research and the


anticipated costs of the study.

Policy implications should be evident, that is, states concerned with student
achievement will have to spend comparable per pupil spending amounts as other
similar states with higher student achievement. In short, higher per pupil
spending equals higher levels of student achievement. A related policy is to
determine a new, more equitable means of computing per pupil spending.

708 | P a g e
Appendix B
NC State Comps Questions

MPA Comprehensive Exam


Study Guide
Competency guide for Public Organization Theory (Romanova)

Classical Organization Fayol (1949) General Principles of Management in


Theory General and Industrial Management
Taylor(1916) Principles of Scientific Management in
Bulletin of the Taylor Society
Weber (1946) Bureaucracy in Essays in Sociology Gulick
(1937) Papers on the Science of Administration
Neoclassical Barnard (1966)The Functions of the Executive Merton
Organization Theory (1957) Social Theory and Social Structure
Simon (1946) Proverbs of Administration, PAR 6 1946
Modern Structural Burns & Stalker (1961) The Management of Innovation
Organization Theory Blau & Scott (1962) Formal Organizations
Walker and Lorch(1968) Organizational
Choice: Product versus Function, Harvard Business
Review, Nov. 1968
Mintzberg (1979) The Structure of Organizations Jaques
(1990) In Praise of Hierarchy, Harvard Business Review
Jan.1990
Decision Theory Charles Lindblom (1959):
The Science of Muddling Through
Rational Choice Theory Anthony Downs (1967):
Inside Bureaucracy
William Niskanen (1971): Bureaucracy
and Representative Government
Elinor Ostrom (1998): A Behavioral
Approach to the Rational Choice Theory of Collective
Action (in APSR 92:1-22)
Human Resource Follet (1926)Scientific Foundations of Business
Theory, or the Administration
Organizational Behavior Roethlisberger (1941) Management and Morale Maslow
Perspective (1943) A Theory of Human Motivation, Psychological
Review 50 1943
McGregor (1957) The Human Side of Enterprise
Systems Theory and Katz & Kahn (1966) The Social Psychology of
Organizational Organizations
Economics Thompson (1967) Organizations in Action
Theories of Governance Donald Kettl (2000): The Global Public Management
Revolution: A Report on the Transformation of
Governance
Laurence Lynn et al (2001): Improving Governance: A
New Logic for Empirical Research
B. Guy Peters and John Pierre (1998): Governance
Without Government? (JPART 2:223-243)
H. George Frederickson (1999): The Repositioning of
American Public Administration in PS 32(4)701-711
Power and Politics in Pfeffer (1981) Power in Organizations
Organizational Theory Kanter (1979) Power Failure in Management Circuits,
Harvard Business Review, July 1979
710 | P a g e
Mintzberg (1983) Power in and around organizations
Theories of Political Dwight Waldo (1948);
Control of Bureaucracy The Administrative State
Michael Lipsky (1980):
Street Level Bureaucracy
Wood and Waterman (1994):
Bureaucratic Dynamics: The Role of Bureaucracy in a
Democracy
Theories of Bureaucratic Dwight Waldo (1952): Development of Theory of
Politics Democratic Administration
Graham Allison (1971):
Essence of Decision
Harold Seidman (1998): Politics Position and Power: The
Dynamics of Federal Organization
Organizational Culture Schein (1985) Organizational Culture and Leadership
and Sense Making Louis (1983) Organizational Symbolism
Theories of Public James Q. Wilson (1989):
Institutions Bureaucracy: What Government Agencies Do and Why
They Do it.
James March and Johan Olsen (1989): Rediscovering
Institutions
763 Competency Guide

You should have a strong grasp of the literature read in the course. Additional
readings are noted in parentheses next to the subject matter with which I wish you to
be familiar.

You should know the following units of analysis and theoretical approaches:

 Individual rational choice and its criticisms (Donald P. Green and Ian
Shapiro, Pathologies of Rational Choice Theory: A Critique of Applications in
Political Science (New Haven: Yale University Press, 1994), ch. 1).
 Group theories: pluralism, corporatism (see Phillippe Schmitter “Still the
Century of Corporatism?” Review of Politics 36 (1974): 85-13 1), Olson’s
selective benefits theory, Marx’s theory
 Institutions (see also Gary Cox “On the Effects of Legislative Rules.” Legislative
Studies Quarterly 25 (2000):169-192); James G. March and Johan P. Olsen,
“The New Institutionalism: Organizational Factors in Political Life,” American
Political Science Review 78 (1984): 734-49.

You should know the following macro -theories of the policy process:

711 | P a g e
 Institutional rational choice
 Advocacy coalitions
 Diffusion/innovation
 Stages approach
 Policy streams

You should have some familiarity with these important theoretical questions:

 Agenda setting (Gary Cox, “Agenda Setting in the U.S. House: A Majority Party
Monopoly?” Legislative Studies Quarterly, 26 (2001): 185-210.
 Representation (James A. Stimson, Michael MacKuen, Robert S. Erikson,
“Dynamic Representation”, American Political Science Review 89 (1995): 543-
65.
 Principal-Agent Theory

COMPETENCY GUIDE FOR FOUNDATIONS OF PUBLIC ADMINISTRATION

One broad question will be completed for this portion of the exam. Students will have
two questions from which to choose. The answer will require synthesis of the material
used to answer the following questions and definitions of key concepts given below:

 What is the nature of public administration, and how has it developed as a


field? What are the tensions between the big and little questions of public
administration?

 What are the issues in the relationship between the public, politicians, and
administrators? To whom are administrators accountable?

 What are the tensions between democracy and "bureaucracy," and how can
they be reconciled?

 What are the tensions between promoting administrative self-interest and the
public interest, and how can they be reconciled?

 What is the normative base of public administration, i.e., values that guide
practice? How does one reconcile “neutrality” and commitment to values in
public administration? Is ethics essential to public administration?

 What does the “public” in public administration really mean? What is the

712 | P a g e
impact of the modalities of implementing public administration on this concept?

 How does public administration legitimize its activities?

 What are the alternative bases of administrative ethics?

Key Concepts

Accountability

Bureaucracy

Citizen participation; how administrators relate to citizens

Complementarity - Political-administrative relations

Democracy

[Dichotomy] Political-administrative relations

Ethics

Internalized / External control: complexities of accountability [e.g., Principal-Agent


and/or principled agent]

Governance, new

History

Implementation, Networks, and Nonprofits in Public Administration

Intellectual issues: PA as field of study

Leadership

Legitimacy

Models of PA Evolution, overlap, and coexistence of models

Classical model – Differentiation of functions

Orthodox model – bureaucracy/principles of administration/scientific


management

Political and institutional models

Normative model

 Waldo

713 | P a g e
 New PA

 Refounding/efforts to establish legitimacy of public administration

Public Choice/NPM

 Principal-Agent

Neutrality: active/expansive definition versus passive/narrow definition

Public interest vs. self-interest

Public vs. Private Administration

The Public

Roles and responsibilities

State agent/citizen agent

Ph.D. Methods Prelims Questions for Summer 2002

1) Research Design has been defined as largely defined as a matter of control. Discuss
statistical versus physical control and relate each to a specific type of research design,
for example, experimental versus quasi_experimental design. What is internal
validity? What is external validity? List and define the principle threats to internal
validity. Can a study have high internal validity and lack external validity? Explain
and provide a concrete example.

2) Much research in the social sciences is based on taking a sample of


respondents, testing hypotheses about the relationships found in the sample data,
and, then making some inference about the likelihood that the relationship found in
the sample data would be true if the entire population had been tested. In th is
regard, define and discuss the following: (a) probability samples vs non_probability
samples. (b) The differences between statistical significance and substantive
importance. (c) Power, Type I and Type II error. (d) The null hypothesis. (e)
Reliability and validity of measures.

3) What is the difference between randomization and random sampling? Relate each
to internal and external validity, and, to the concept of control. Why do researchers
have more confidence in saying that the independent variable affec ted the
dependent variable in a research design in which randomization has taken place?
714 | P a g e
4) Women faculty members at Waterford Central University (WCU) have filed a sex
discrimination suit against the university. You are a newly_hired associate in the firm
that represents the plaintiffs. The deposition of the university's expert has already
been taken. The transcript reveals that the expert performed two multiple regression
analyses on the faculty salary data. In the first, salary was the dependent variable
with the independent variables being:

(1) the mean salary of the department

(2) experience as an instructor at any university, including WCU

(3) experience as a faculty member other than at WCU

(4) experience as a full professor at WCU

(5) experience as an associate professor at WCU

(6) experience as an assistant professor at WCU

(7) possession of a doctoral degree other than a Ph.D.

(8) possession of a Ph.D. degree

The expert's second regression analysis is identical to the first except that sex is
added in the form of a dummy variable having the value of 1 for males. The coefficient
for the sex variable in the university's second analysis is $694 (per year) and its
associated t_ratio is 2.3. The first model (without sex as a factor) had an r2 of .524
percent while the second model (with sex as a factor) had an r2 of .532. According to
WCU expert's testimony, the $694 coefficient for sex in the second model does not
indicate discrimination against women because the model with sex as a factor
explained only .8 percent more of the total variation around the average salary than
did the model without sex:

The researcher also applied the first multiple regression model, without the sex
variable, to each faculty member to get a prediction of the salary that each shou ld be
earning. They then compared each prediction to the faculty person's actual salary.
Some, 343 men were paid less, and 302 more, than the model predicted they would
be paid, while 61 women were "underpaid" and 55 "overpaid."

715 | P a g e
A more senior associate has found a case that went to trial on virtually identical
facts to the present one. In that earlier suit, the defendant prevailed in a bench trial.
The partner directing the litigation solicits your comments. She worriedly notes that
"If I understand what the WCU expert is saying, I'm troubled myself that adding the
gender variable has hardly any effect. Still, I thought that the $684 coefficient was
pretty good for us." What can you tell the partner that be useful either in assessing
what's going on here or in attacking the thrust of the expert's argument?

5) Describe the use of principal components analysis (PCA) as a dimension_reduction


technique. Comment on the following in this description: a) selecting the number of
dimensions / components; b) calcul ating new data or new variables from the
eigenvectors; c) the relation between the new components, and the relation of the new
components to the multivariate indicators.

6) Interpret the logistic regression output in the appendix, which in which a


researcher is trying to assess whether four survey items (Q1 through Q4) differentiate
husbands and wives (SPOUSE, 1= husband, 2= wife). (Note – Appendix did not
reproduce clearly and is not included here)

7) Describe the use of structural equation modeling in testing h ypothesis of causal


structure and latent structure in a multivariate set of data. Comment on the following
the difference between the “structural” and the “measurement” model. Also, comment on
the following aspects of this technique: i) constructing path m odels and equations; ii)
the problem of model identification; and iii) estimates of model fit. Distinguish (or
compare similarities) principal components analysis, factor analysis, confirmatory
factor analysis, and structural equation modeling in how the t echniques estimate
unknown (latent) variables from known (indicator, overt) variables.

Summer 2003 Written Preliminary Examination – PhD in Public Administration


Methodology

1) Research Design has been defined as largely as a matter of control. Discuss


statistical versus physical control and relate each to a specific type of research design,
for example, experimental versus quasi -experimental design. What is internal
validity? What is external validity? List and define the principle threats to internal

716 | P a g e
validity. Can a study have high internal validity and lack external validity? Explain
and provide a concrete example.

2a) Much research in the social sciences is based on taking a sa mple of


respondents, testing hypotheses about the relationships found in the sample data,
and, then making some inference about the likelihood that the relationship found in
the sample data would be true if the entire population had been tested. In this
regard, define and discuss probability samples vs non -probability samples.

2b) Which sampling design would best address the following? You are a city
manager and you want citizen feedback on a new diversity program. You have a
very small proportion of African Americans, Hispanics and Asians in your city, yet
you want to be sure you get feedback from them in a large -scale citizen survey.

3) What is the difference between randomization and random sampling? Relate each
to internal and external validity and to the conce pt of control. Why do researchers
have more confidence in saying that the independent variable affected the dependent
variable in a research design in which randomization has taken place? What is the
only type of research design in which random assignment of subjects to treatment
groups is possible?

4) When is multicollinearity important in multivariate statistical procedures? When is it


unimportant? Explain how multicollinearity is tested. If multicollinearity is found,
what can be done about it?

5) What is cross-validation? Although always desirable, when is it most needed


and why?

6) Non-constant error variance may indicate violation of which common assumption of


statistical procedures in the multiple general linear hypothesis family?

7) Outliers are undesirable because they imply violation of which two common
assumptions of statistical procedures in the multiple general linear hypothesis
family?

8) Explain attenuation.

9) Describe the use of principal components analysis (PCA) as a dimension -reduction


technique. Comment on the following in your description: a) selecting the number of
717 | P a g e
dimensions / components; b) calculating new data or new variables from the
eigenvectors; c) the relation between the new components, and the relation of the new
components to the multivariate indicators.

10) If an independent variable has a logistic coefficient of 1.476, how can this be
interpreted in a plain-English sentence that renders some intuitive meaning to the
reader.

11) Describe the use of structural equation mo deling in testing hypotheses of


causal structure and latent structure in a multivariate set of data. Comment on the
following the difference between the “structural” and the “measurement” model. Also,
comment on the following aspects of this technique: i) constructing path models and
equations; ii) the problem of model identification; and iii) estimates of model fit.
Distinguish (or compare similarities) among principal components analysis, factor
analysis, confirmatory factor analysis, and structural equat ion modeling in how the
techniques estimate unknown (latent) variables from known (indicator, overt)
variables.

Foundations, Theory, and Policy

June 27, 2005

I. Organizational Theory: Answer the following question

Compare and contrast the pre-theoretic frameworks – Systems,


Humanistic/Organizational Behavior, Rational Choice, and Post-Modern – in
light of a specific problem or substantive issue.

II: Policy: Choose question A or B

A. Describe the punctuated equilibrium and innovation/diffusion theories of


the policy process. For each one, explain the principal unit of analysis, the
critical actors and what drives their preferences and behavior, their
characterization of policy change, the nature of collective action, the part of
the policy process on which most attention is focused, and whether or not
governmental institutions are important. Then think about the lottery issue
in North Carolina. Briefly explain how well each theory describes the lack of
policy change on this issue.
718 | P a g e
B. When we think of the policy process, we often divide it into two principal
types of actors: societal and governmental. Briefly discuss Olson’s theory of
group behavior. How does he understand the behavior of groups in the
policy process and explain those that he believes are able to turn their
preferences into policy outputs. Now think of governmental institutions.
Whose preferences within government are closest to outputs according to
Krehbiel (Pivotal Politics). Explain yourself.

III. Foundations: Choose question A or B

A. There has been considerable debate about whether public administration is


capable of renewal from within, on the one hand, or resists change and
must be “reformed” from the outside, on the other. What are the major
conceptual models of public administration that would support each
position? How would you interpret the response of administrators in the
United States with “reinventing government” in terms of the “renewal from
within/reform imposed from outside” debate?

B. Neutrality has been defined narrowly and broadly in the public


administration literature. How would neutrality be defined in each of these
ways and which models and/or concepts are associated with each
definition? In view of this discussion, how would you assess the neutrality
of (a) city managers who view themselves as more influence than elected
officials in policy making and (b) administrators who share information with
reporters about the misdeeds of political superiors, e.g., like Mark Felt in his
deep throat role?

July 17, 2006: PhD in Public Administration Preliminary Examination

Foundations Answer 1A or 1B (cite relevant bibliography).

1A. In their interactions with elected officials, some have argued that public
administrators may be either instruments or usurpers, agents or trustees,
servants or masters, and tools or independent bodies. What is the significance
of all these distinctions taken together, and what conceptual models support
the alternative views? In contrast, what does it mean to view the political-

719 | P a g e
administrative relationship as a two-way street or a complementary
relationship? What empirical evidence supports this alternative view of
administrators in their relationship with elected officials?

1B. What is the difference between “value neutrality” as a quality of public


administrators and “political neutrality,” and how is each reflected in behavior?
What difference does it make to the nature of democratic governance which
kind of neutrality is displayed by public administrators? How would the
definition of neutrality affect the role of administrators in formulating public
policy? Examine the conceptual models that support each approach to
neutrality, and discuss the ethical implications of each approach?

Organization Theory

2. Compare and contrast Classical and Neo-Classical Organization Theory.


Describe what makes up each. How are they similar? How do they differ? What
are the limitations of these theories? How are they still relevant for today’s
research?

Public Policy Answer 3A or 3B

3A. Describe the punctuated equilibrium and innovation/diffusion theories of


the policy process. For each one, explain the principal unit of analysis, the
critical actors and what drives their preferences and behavior, their
characterization of policy change, the nature of collective action, the part of the
policy process on which most attention is focused, and whether or not
governmental institutions are important. Then think about the lottery issue in
North Carolina, briefly, how does innovation/diffusion explain change on this?
Again, briefly, How can punctuated equilibrium help explain policy change on
the tobacco issue over the past 15 years?

3B. When we think of the policy process, we often divide it into two principal
types of actors: societal and governmental. Briefly discuss Olson’s theory of
group behavior. How does he understand the behavior of groups in the policy
process and explain those that he believes are able to turn their preferences

720 | P a g e
into policy outputs. Now think of governmental institutions. Whose
preferences within government are closest to outputs?

REVIEW GUIDE FOR PA METHODS

Date: 5/17/07:

Subject to change and updating.

Updates may be available at http://www2.chass.ncsu.edu/garson/pa765/ch15.htm.

Sample items are available at


http://www2.chass.ncsu.edu/garson/pa765/sampleexam.htm

The following topics are covered in the methods test item data bank. Topics listed are
general topics. Individual test items may include reference to more detailed aspects of
general topics.

RESEARCH DESIGN

purpose of residual analysis

standardizing data

levels of measurement

normal curve & confidence levels

types of ordinal scales

types of validity

threats to validity

types of reliability

reliability analysis

testing for normality

testing for homoscedasticity

721 | P a g e
testing for linearity

testing for unidimensionality

data transformation

data screening

SIGNIFICANCE

assumptions of significance testing

significance v. power

type I and II errors

confidence intervals

chi-square tests

t-tests

CORRELATION/ PARTIAL CORRELATION

assumptions of correlation

attenuation

types of explanation & suppression

control variables

canonical correlation

REGRESSION

assumptions of regression

standardized and unstandardized b

R-square

significance of b and of model

722 | P a g e
WLS regression

ANOVA FAMILY

assumptions of anova

homogeneity of variance

covariates

within- and between-group designs

FACTOR ANALYSIS & RELATED

types of extraction

types of rotation

communality

eigenvalues

factor loadings

confimatory factor analysis in SEM

cluster analysis

multi-dimensional scaling

LOG-LINEAR ANALYSIS

assumptions

sampling adequacy

likelihood ratio tests

parsimony

types of models

link function

723 | P a g e
odds and odds ratios

LOGISTIC REGRESSION

assumptions

logits

odds ratios

likelihood ratio test

significance

maximum likelihood

binomial vs. multinomial

STRUCTURAL EQUATION MODELING

assumptions

comparison with OLS regression

types of variables modeled in AMOS

model development approaches

path significance tests

modification indexes

critical ratios

assigning a metric

confirmatory factor analysis

measurement vs structural model

goodness of fit measures

checking cross-group invariance

724 | P a g e
identification of models

725 | P a g e
Appendix C
Rutgers University Comps
Questions

MPA Comprehensive Exam


Study Guide
Day 1 Morning, March 31, 2009

Institute of Government

Doctoral Comprehensive Examination

Spring 2009

Please write ONLY your Student ID# on your answer sheet.

Answer the following question in its entirety:

In recent years much attention has been given to the issues of equity in funding
and performance of children in public schools. The Tennessee Supreme Court
in 1993’s Tennessee Small School systems, et al. v. Ned Ray McWherter, et al.
ruled that the:

State Constitution imposes upon the General Assembly the obligation to


maintain and support a system of free public schools that afford
substantially equal educational opportunities to all students.

The small school districts that sued the state for equity in funding were
primarily poorer counties or cities or special school districts that did not have a
substantial taxing base in their communities.

It is now 12 years after the implementation of the funding equity formula and
you have been called upon to design a study to see if equity in funding has led
to comparable performance between the small school systems and non-litigants
in the case.

1. How would you operationalize performance? What are the component


parts that would comprise a valid, reliable measure of performance in the
context of the lawsuit?

2. How would you operationize equity? What are the component parts that
would comprise a valid, reliable measure of equity in the context of the
lawsuit?

727 | P a g e
3. What type of research design would you construct to do this study? What
would be your unit of analysis? Why? What would be your sampling
frame?

Day 1 Afternoon, March 31, 2009

Institute of Government

Doctoral Comprehensive Examination

Spring 2009

Please write ONLY your Student ID# on your answer sheet.

Answer one of the following questions.

1. The budget may be viewed as an instrument of fiscal policy, as a means of


determining policy choices, and as a tool for managing the economy. Explain
each of these aspects of budgeting. In your answer, emphasize the views of the
classical or pre-Keynesian economists, Keynesian economists, neo-Keynesians,
the monetarists, the Public Choice school economists, and the leading public
budgeting scholars.

2. In his renowned 1940 article, V.O. Key, Jr. lamented about “the lack of budget
theory.” Twelve years later (1952), Verne B. Lewis attempted to construct a
normative budget theory. Today the quest for a budget theory continues. In one
of her numerous writings, Irene Rubin noted that, “budget theory today is
fragmented and incomplete…. It is in the process of being invented.” Why is
budget theory still viewed incomplete and fragmented? Briefly discuss the major
competing theories of budgeting (distinguishing between normative and
descriptive theories) treated in the budgeting literature.

Day 2 Morning, April 1, 2009

Institute of Government

Doctoral Comprehensive Examination

Spring 2009

728 | P a g e
Please write ONLY your Student ID# on your answer sheet.

Answer one of the following questions.

1. Examine the arguments for the legitimacy of the administrative state. Evaluate
the significance of this debated to the development of the field.

2. It has been argued that Chester Barnard and Max Weber-two very different
persons intellectually-are the founders of administrative theory. Discuss their
specific contributions. In view of the wide array of perspectives on
administration currently available, is it meaningful to trace the roots of this
field back to them? Illustrate your point by referring to several theoretical
perspectives on administration.

Day 2 Afternoon, April 1, 2009

Institute of Government

Doctoral Comprehensive Examination

Spring 2009

Please write ONLY your Student ID# on your answer sheet.

Answer the following question in its entirety.

Other than performance-based budgeting, what are the most important recent
developments in financial management? Why are they important and what
precipitated them? Explain how these recent changes may/can influence the work of
public managers and policy makers.

School of Public Affairs and Administration


Rutgers-Newark

Sample Comprehensive Exam Questions

Comp #1 – Theoretical Foundations

601 The Study of Public Organizations

729 | P a g e
A. 1. Assuming that the body of knowledge which comprises public administration is
itself comprised of a series of at least five types of insights, and that each of those
insights continues to be relevant to contemporary problem solving, analyze the central
problem of the “privatization model” through multiple theoretical lenses. Cite the
appropriate literature as to the importance of the concepts you have chosen to
emphasize.

Or

A.2. How would you explain to proponents of privatization that their theories have
insufficient explanatory power, and that a more systemic view of public administratio n
would prove more efficacious in the long run? . Discuss competing assumptions
from an interdisciplinary perspective, and assess the dynamic nature of the
“state-of-the-art” of the field of public administration with particular attention to
elements which both mainstream public administration theorists and government’s
critics may have overlooked.

OR

A. 3. How do the basic themes of American public administration apply to the events of
Sept. 11, 2001? Address this question in terms of organizational capacity, or lack
thereof, in basic elements of public administration at the federal and municipal
levels.

A. 1. Assuming that the body of knowledge which comprises public administration is


itself comprised of a series of at least five types of insights, and that each of those
insights continues to be relevant to contemporary problem solving, analyze the central
problem of the “privatization model” through multiple theoretical lenses. Cite the
appropriate literature as to the importance of the concepts you have chosen to
emphasize.

Or

2. How would you explain to proponents of privatization that their theories have
insufficient explanatory power, and that a more systemic view of public
administration would prove more efficacious in the long run? . Discuss competing
assumptions from an interdisciplinary perspective, and assess the dynamic

730 | P a g e
nature of the “state -of-the-art” of the field of public administration with particular
attention to elements which both mai nstream public administration theorists and
government’s critics may have overlooked.

OR

A. 3. How do the basic themes of American public administration apply to the events of
Sept. 11, 2001? Address this question in terms of organizational capacity, or la ck
thereof, at the federal and municipal levels.

A. 1. Assuming that the body of knowledge which comprises public administration is


itself comprised of a series of at least five types of insights, and that each of those
insights continues to be relevant to contemporary problem solving, analyze the central
problem of the “privatization model” through multiple theoretical lenses. Cite the
appropriate literature as to the importance of the concepts you have chosen to
emphasize.

Or

A. 2. How would you explain t o proponents of privatization that their theories have
insufficient explanatory power, and that a more systemic view of public administration
would prove more efficacious in the long run? . Discuss competing assumptions
from an interdisciplinary perspective, and assess the dynamic nature of the
“state-of-the-art” of the field of public administration with particular attention to
elements which both mainstream public administration theorists and government’s
critics may have overlooked.

OR

A. 3. How do the basic themes of American public administration apply to the events of
Sept. 11, 2001? Address this question in terms of organizational capacity, or lack
thereof, at the federal and municipal levels.

A. 1. Assuming that the body of knowledge which comprises public administration is


itself comprised of a series of insights, and that each of those insights continues to
be relevant to contemporary problem solving, analyze the central problem of the

731 | P a g e
“business model” through multiple theoretical lenses. Cite the a ppropriate literature
as to the importance of the concepts you have chosen to emphasize.

Or

A. 2. How would you explain to proponents of privatization that their theories


have insufficient explanatory power, and that a more systemic view of public
administration would prove more efficacious in the long run? . Discuss
competing assumptions from an interdisciplinary perspective, and assess the
dynamic nature of the “state -of-the-art” of the field of public administration with
particular attention to elements which both mainstream public administration
theorists and government’s critics may have overlooked.

A. 1. Assuming that the body of knowledge which comprises public administration is


comprised of a series of insights, and that each of those insights continues to be
relevant to contemporary problem solving, analyze the central problem of the
“business model” through multiple theoretical lenses. Cite the appropriate
literature as to the importance of the concepts you have chosen to emphasize.

or

A. 2. Define the field of public administration in no more than one hundred words.
Then defend your definition, arguing for the central nature of the concepts in your
definition, and citing the appropriate literature as to the importance of the concepts
you have chosen to emphasize.

602Decision Making and Policy Analysis

A. 1. Assuming that the body of knowledge whi ch comprises public administration is


itself comprised of a series of at least five types of insights, and that each of those
insights continues to be relevant to contemporary problem solving, analyze the central
problem of the “privatization model” through multiple theoretical lenses. Cite the
appropriate literature as to the importance of the concepts you have chosen to
emphasize.

Or

732 | P a g e
A.2. How would you explain to proponents of privatization that their theories have
insufficient explanatory power, and that a more systemic view of public administration
would prove more efficacious in the long run? . Discuss competing assumptions
from an interdisciplinary perspective, and assess the dynamic nature of the
“state-of-the-art” of the field of public administration with particular attention to
elements which both mainstream public administration theorists and government’s
critics may have overlooked.

OR

A. 3. How do the basic themes of American public administration apply to the events of
Sept. 11, 2001? Address this question in terms of organizational capacity, or lack
thereof, in basic elements of public administration at the federal and municipal
levels.

B. 1. From an policy -analytic point of view, compare the role of empirical and
normative information in the phases of the policy cycle (policy agenda -setting, policy
formulation, policy adoption and legitimation, implementation, and evaluation) and
discuss the interplay of methods appropriate to each.

Or

B. 2. . Explain the three most im portant challenges confronting knowledge utilization


in public managerial decision-making and examine their implications for the practice
of policy analysis.

B. 1. From an policy-analytic point of view, compare the role of empirical and


normative information in the phases of the policy cycle (policy agenda -setting, policy
formulation, policy adoption and legitimation, implementation, and evaluation) and
discuss the interplay of methods appropriate to each.

Or

B. 2. . Explain the three most important challenges confront ing knowledge utilization
in public managerial decision-making and examine their implications for the practice
of policy analysis.

733 | P a g e
B. 1. Focusing on the need for usable knowledge, compare and discuss the logic of
policy decision-making with that of positiv ist-oriented policy analysis. How, in your
view, might decision-making and policy analysis best be brought into a mutually
beneficial relationship?

Or

B. 2. Examine the differences between outcome and process oriented policy


evaluation and suggest how they might be utilized together to improve the policy
decision-making process.

B. 1. Discuss the differences between the positivist and postpositivist approaches


to policy analysis. How, in your view, should the tensions between these two
perspectives bed dealt with ? Explain.

or

B. 2. . Despite Lasswell's call for a "policy science of democracy," policy


science, as it came to be practiced, has often been described as "technocratic"
and "apolitical." Explain the ways in which policy science is seen to work against
democratic decision-making. And how might we begin to mitigate this tension
between expertise and democracy?

603 Public Administration in a Democratic Society

C. 1. The classic theory of bureaucracy formulated by Max Weber identifie s certain


formal dimensions of administration that presumably remain constant over time and
in different social systems. Other thinkers have argued that the functions of
administration change in response to the orientation of the state (liberal,
authoritarian or welfare state, for example) or economy. Discuss various
perspectives on this issue and what each implies for how we understand the role of
public administration in democratic societies.

Or

C.2. The public administration literature has paid more atte ntion to normative
questions of ‘administration’ than to what we mean –or should mean– by ‘public.’
How should the ‘public’ in public administration be understood? What challenges

734 | P a g e
are posed to this concept in market societies? Illustrate this understanding or
these challenges in a particular substantive area (environment, race, sex, poverty,
etc.).

C. 1. The classic theory of bureaucracy formulated by Max Weber identifies certain


formal dimensions of administration that presumably remain constant over time and
in different social systems. Other thinkers have argued that the functions of
administration change in response to the orientation of the state (liberal,
authoritarian or welfare st ate, for example) or economy. Discuss various
perspectives on this issue and what each implies for how we understand the role of
public administration in democratic societies.

(b) The public administration literature has paid more attention to normative
questions of ‘administration’ than to what we mean –or should mean– by ‘public.’ How
should the ‘public’ in public administration be understood? What challenges are
posed to this concept in market societies? Illustrate this understanding or these
challenges in a particular substantive area (environment, race, sex, poverty, etc.).

C. 1. How does its economic or political context change the role of public
administration? With specific reference to their different economic and political
institutions, compare the role of administration in ancient or feudal society, market
capitalism and the corporate or Keynesian welfare state.

Or

C.2. How can public participation in governance be improved? A number of


thinkers have faulted democracy for the dominant role of business, t he over-reliance
on technical experts, and low-levels of civil education and/or participation in
governance at all levels. Responses to these criticisms include ‘deliberative democracy”
and so-called “e-governance”. Discuss the pros and cons of these or related attempts
to ‘fix’ democracy. Consider what proposals for reform imply for public
administration.

Or

C.3. Marxists, pluralists, and managerial/elite theorists present different stories


about the nature of governance in market societies. Discuss these compe ting views

735 | P a g e
and what each implies for the role of public administration

C. 1. From the theorists you have studied, both classic and modern, select three with
competing views of how democracy, administration/bureaucracy and markets are
related. Briefly describe each view and for each identify a researchable question that
might help determine whether the theory is correct and how you might go about
answering it. For example, for Marx, you might ask: Do industrial workers support
radical change?

C.2. Pluralist, managerial/elite, and class theories reco gnize the importance of the
(l)State, (2)Capitalism, (3) Democracy (4) interest groups, and (5) Administration. But
they are assigned very different meaning and importance by each perspective.
Employing the vocabul ary associated with the perspective as closely as possible,
compare how each of the 3 approaches conceptualize these phenomenon.

C. 3. Competing interest groups increasingly claim special privileges from the state
because of past disadvantage, disability a nd/or discrimination, including privileged
access to education, the electoral process and employment. Select a claim associated
with disadvantage or inequality (e.g. affirmative action, equal pay, the ADA,
redistricting, and so forth) and discuss how it bears on public administration.

C. 1. Among the key factors that shape the making and administration of public
policy in democratic societies are inequality, bureaucracy, the influ ence of
business and professional elites, and the growing importance of cultural diversity.
Basing your answer on the theoretical literature, discuss the importance of two of
these factors for public administration.

or

C.2. Classic as well as contemporary theories of democratic governance


recognize the importance of (l) the State/Administration, (2) Markets, (3)
Knowledge/Information, and (4) Society/Participation. But they are assigned very
different meaning and importance by each perspective. Select two m ajor
perspectives (e.g. class, statist, technocratic, managerial/elite, pluralist, etc.).
Employing the vocabulary associated with the perspective as closely as possible,

736 | P a g e
compare and contrast how major proponents of the perspective conceptualize these
phenomenon and their interrelationships.

611 Administrative Politics

Describe the role of the bureaucracy in the U.S. policy -making process. Pay
particular attention to bureaucracy’s role in the American constitutional system,
and to the impact of politics on bureaucratic policy making.

Comp #2 - Performance Application

604Performance Improvement in Public Administration

D. 1. Public sector productivity improvement strategies are of differing interest


to the practitioner as opposed to the academic.

From the practitioner's point of view -- what are the techniques or strategies that
promise the most productivity improvement and how have they been implemented in
the public sector?

From the academic's viewpoint -- what are the issues that offer the large st
research opportunities?

OR

D. 2. What are the central research questions concerning the relationship between
innovation and organizational productivity? Please discuss the questions and authors
according to the process model and by level of analysis. Wh at are the
recommendations emerging from the research to create an innovation-friendly
organization?

OR

D.3. Please trace the history and development of public administration as a


discipline, paying special attention to the importance of the concepts

Of quality, productivity and efficiency to that history. Then place American


public administration within a global context: how has our history been

737 | P a g e
Influenced, and in turn influences, global public management. Finally, what are
your predictions for the future path of the field?

D. 1. Public sector productivity improvement strategies are of differing interest


to the practitioner as opposed to the academic.

From the practitioner's point of view -- what are the techniques or strategies that
promise the most productivity improvement and how have they been implemented in
the public sector?

From the academic's viewpoint -- what are the issues that offer the largest
research opportunities?

OR

D. 2. What are the central research questions concerning the relationship between
innovation and organizational productivity? Please discuss the questions and authors
according to the process model and by level of analysis. What are the
recommendations emerging from the research to create an innovation-friendly
organization?

OR

D.3. Please trace the history and development of public administration as a


discipline, paying special attention to the importance of the concepts

Of quality, productivity and efficiency to that history. Then place American


public administration within a global context: how has our history been
Influenced, and in turn influences, global public management. Finally, what are
your predictions for the future path of the field?

D. 1. The movement to privatize many government activities has led some to


question the appropriate role of government in society. Some public administration
theorists have been very concerned about the emphasis on privatization in the
public administration lite rature, what are the major concerns? Historically, what
has been the private sector involvement in producing public goods and services in
this country? And how has the American experience compared to other countries in
Asia or Europe? Finally, if privatiza tion strategies are pursued by state and local

738 | P a g e
governments, what recommendations have emerged from the PA literature.

OR

D. 2. What are the central research questions concerning the relationship between
innovation and organizational productivity? Please discuss the questions and authors
according to the process model and by level of analysis. What are the
recommendations emerging from the research to create an innovation-friendly
organization?

OR

D.3. Develop and defend a comprehensive framework for productivity


improvement in the public sector, supporting your conceptualization with specific
and extensive citations from the literature.

605Government Budgeting and Resource Acquisition

E. Answer either E.1. or E.2. or E. 3:

E. 1. Performance budgeting advocates have proposed an alternative to the traditional,


line-item methods of budgeting, first, to reorient many of traditional budgeting’s
incentives and, second, to integrate budgeting with management. Describe the
performance budget in these terms. Account for performance budgeting’s spread
across nations and lower -level jurisdictions. Critically evaluate performance
budgeting’s long-term prospects.

OR

E. 2. Taking into account the literature in the area, how do budgeteers make sense
of reality in making resource allocations? What limits exist in using rational analysis in
budget decisions? In addition to or instead of rational analysis, when do budgeteers
“interpret” and “narrate” the phenomena they observe in making resource
allocations? Illustrate from the literature you’ve read. What practical implications
do interpretation and sense -making have for public administration and public
budgeting?

OR

739 | P a g e
E. 3. Budget reform movements have tended to produce reforms in thinking
applicable not only to budgeting but also to public administration generally. That is,
both budgeting and public administration have tried to deal with accountability
issues at about the same period. Both have focused on decision-making. Both have
tried to deal with government performance.

a.characterize the budget and public administration reforms that have emerged
since the Progressive era and the issues they have both dealt with.

b. assess whether budgeting reforms and general public administration


reforms have worked together to further changes and improvements in the ways
government operates or whether the reforms have worked at cross purposes.

c. argue whether budgeting and management are fundamentally dealing with


the same issues or different ones and assess whether budgeting and
management are part of the same or different areas of public administration
theory.

E. 1. Performance budgeting advocates hav e proposed an alternative to the traditional,


line-item methods of budgeting, first, to reorient many of traditional budgeting’s
incentives and, second, to integrate budgeting with management. Describe the
performance budget in these terms. Account for performance budgeting’s spread
across nations and lower -level jurisdictions. Critically evaluate performance
budgeting’s long-term prospects.

OR

E. 2. Taking into account the literature in the area, how do budgeteers make sense
of reality in making resource allocations? What limits exist in using rational analysis in
budget decisions? In addition to or instead of rational analysis, when do budgeteers
“interpret” and “narrate” the phenomena they observe in making resource
allocations? Illustrate from the literature y ou’ve read. What practical implications
do interpretation and sense -making have for public administration and public
budgeting?

OR

E. 3. Budget reform movements have tended to produce reforms in thinking

740 | P a g e
applicable not only to budgeting but also to public administration generally. That is,
both budgeting and public administration have tried to deal with accountability
issues at about the same period. Both have focused on decision-making. Both have
tried to deal with governmen t performance.

a. characterize the budget and public administration reforms that have emerged
since the Progressive era and the issues they have both dealt with.
b. assess whether budgeting reforms and general public administration reforms
have worked together to further changes and improvements in the ways
government operates or whether the reforms have worked at cross purposes.
c. argue whether budgeting and management are fundamentally dealing with the
same issues or different ones and assess whether budgeting an d
management are part of the same or different areas of public administration
theory.

E. 1. Anthony Downs (“Why the government budget is too small in a democracy”) and
James Buchanan (“Why does government grow?”) present two opposing views on the
budget problem. Outline their arguments and the specific points on which they
differ. Design and present a research proposal in which you resolve specific points
on which they differ.

OR

E.2. Performance budgeting adherents have proposed an alternative to the tradi tional,
line-item methods of budgeting, first, in dealing with the many of traditional
budgeting’s incentives and, second, by integrating budgeting with management.
Describe the performance budget in these terms. Critically analyze performance
budget’s long-term prospects.

OR

E. 3. V. O. Key asked the basic question for budgeting researchers and theorists. What
was that question? What did he mean by the question?

Wildavsky and others have outlined and defended a budget theory that consists of
roles and as such is a behavioral approach to budgeting. Meyers and others have
outlined a budget theory that is structural and follows a functionalist approach to

741 | P a g e
budgeting. Contrast these two theories. Assess each in terms of its explanatory power
in answering the basic question put forth by V. O. Key.

E. 4. Budget reform movements have tended to produce reforms in thinking


about public administration generally. That is, both budgeting and public
administration generally have tried to deal with accountability issues at about the
same period. Both have focused on decision making and process. Both have tried
to deal with government performa nce.

a) characterize the budget and public administration reforms and what issues
they have both dealt with since the Progressive era.

b) assess whether budgeting reforms and general public administration reforms


have worked together to further changes and impro vements in the ways
government operates or whether the reforms have worked at cross purposes.

c) argue whether budgeting and management are fundamentally dealing with


the same issues or different ones and assess whether budgeting and
management are part of the same or different areas of public
administration theory.

E. 1. Anthony Downs (“Why the government budget is too small in a democracy”) and
James Buchanan (“Why does government grow?”) present two opposing views on the
budget problem. Outline their arguments and the specific points on which they
differ. Design and present a research proposal in which you resolve specific points
on which they differ.

OR

E. 2. Performance budgeting adherents have proposed an alternative to the traditional,


line-item methods of budgeting, first, in dealing with the many of traditional
budgeting’s incentives and, second, by integrating budgeting with management.
Describe the performance budget in these terms. Critically analyze performance
budget’s long-term prospects.

OR

E. 3. V. O. Key asked the basic question for budgeting researchers and theorists. What

742 | P a g e
was that question? What did he mean by the question?

Wildavsky and others have outlined and defended a budget theory that consists of
roles and as such is a behavioral approach to budgeting . Meyers and others have
outlined a budget theory that is structural and follows a functionalist approach to
budgeting. Contrast these two theories. Assess each in terms of its explanatory power
in answering the basic question put forth by V. O. Key.

606 Administrative Law

F Answer either F.1. or F.2:

F. 1. “Due process,” as we know, is an elastic concept, but even so it is difficult to


reconcile the Court’s holding in Goldberg vs. Kelly (the welfare case) with its one six
year later in Mathews v. Eldridge (involving the poor person who disability payments
were terminated). What factors can you cite to account for these disparate rulings.

OR

F.2. Cann, in his Administrative Law text, observes that “Congress has the power to
control agencies but lacks the will; the President has the will to control them, but
lacks the power.” What did Cann mean? Please explain, with reference to cases we have
briefed and discussions we have shared.

F. 1. “Due process,” as we know, is an elastic concep t, but even so it is difficult to


reconcile the
Court’s holding in Goldberg vs. Kelly (the welfare case) with its one six year later in
Mathews v. Eldridge (involving the poor person who disability payments were
terminated). What factors can you cite to account for these disparate rulings.

OR

F.2. Cann, in his Administrative Law text, observes that “Congress has the power to
control agencies but lacks the will; the President has the will to control them, but
lacks the power.” What did Cann mean? Please expl ain, with reference to cases we have
briefed and discussions we have shared.

743 | P a g e
F. 1. “Due process,” as we know, is an elastic concept, but even so it is difficult to
reconcile the Court’s holding in Goldberg vs. Kelly (the welfare case) with its one six
year later in Mathews v. Eldridge (involving the poor person who disability payments
were terminated). What factors can you cite to accunt for these disparate rulings.

OR

F.2. Cann, in his Administrative Law text, observes that “Congress has the power to
control agencies but lacks the will; the President has the will to control them, but
lacks the power.” What did Cann mean? Please explain, with reference to cases we have
briefed and discussions we have shared.

F. 1. "Our governmental structure is based upon the sepa ration-of-powers, and


we have discussed several cases in which the Courts have disallowed actions on
the ground that one branch has usurped, or encroached upon, the powers of
another. At the same time, the Supreme Court has long recognized that the branche s
are not hermetically sealed, and must occasionally share or delegate power.

Please discuss some cases in which the High Court has, and has not, allowed some
breaches of the "walls of separation" between the three branches, and explain its
rationale for so doing."

F. 2. In the context of administrative law, what, essentially, is meant by "due


process"? Please discuss, incorporating when appropriate actual cases we have
discussed in class.

612 Performance Measurement and Evaluation

The belief that individuals sh ould be given a voice in their governance appeals to our
democratic ideals. While there appears to be universal agreement that the
involvement of citizens in the decision making process of government is a good idea,
there is little agreement as to the best way to achieve meaningful involvement. There
are many ways to consult with the public and get a sense of what they see as
problems and opportunities; it is quite another thing to actively engage citizens in the
decision-making process and more explicitly the measurement of government
performance. Some theories advocate direct and active influence in decision
making, while others suggest more indirect approaches. Which approach do you

744 | P a g e
advocate and why? Be sure to discuss the advantages and disadvantages of including
citizens in the measurement of government performance. Include appropriate
references to support your argument.

Much discussion has centered on the appropriate relationship between citizens


and public administrators. Discuss the key elements of t he various relationships,
as presented in the literature, as well as the strengths and weaknesses of each
relationship. In your opinion, what relationship is most appropriate for a well
functioning society and why? Support your choice with relevant argumen ts from
the readings. Draw a model/diagram of what your citizen/government
relationship looks like and explain the model.

Compare and contrast the following: Classic public administration

New public management

New public service

Discuss the core values of each model of administration as well as the strengths and
weaknesses of each model. After discussing these models, present your model for a
new public administration. Your response must be supported by the literature and
relevant citations.

745 | P a g e
Appendix D
TSU Comps Questions

MPA Comprehensive Exam


Study Guide
TSU: Ph.D. Public Administration Comprehensive Exams

Spring 2009 Thursday, February 26, 2009

QUESTION ONE: Identify, describe and explain the pivotal works and recurring themes
in the literature about policy implementation. Has any scholarly consensus been reached?

The public policy discourse in public administration involves several components and centers
on multiple questions: What are policies? What forms do policies take? Who makes policies?
What is the policy formulation process? How are policies implemented? Is policy
implementation effective? How do we evaluate public policy effectiveness? This discussion will
involve examining these essential elements of the public policy discourse while finally
addressing concerns, theories, and dominant themes in implementation research. However, this
author will examine who makes policies and how policies are made, as this discussion is
essential to understanding how policies are eventually implemented (or not).

Defining Policy and Policy Actors

A public policy could be defined as a purposive course of action designed to impact some social,
political, or economic issue that affects the general public (Anderson 2006). Public policies
may take the form of rules and regulations, legislative bills, budgeting and financial
allocations, or legal decisions. Public policies are formulated through courts, administrative
agencies, state and federal legislatures, executive offices (President or Governor), as well as
citizens and public interest groups (Anderson 2006). With the complexity of policy forms and
actors, Lindblom suggests that just agreeing on a particular public policy may be viewed as a
sign of success in public policy making, not necessarily whether or not the policy will have the
most impact on a particular public issue or concern.

Public Policy Theories

Because the policy formation process involves multiple actors—interest groups, politicians,
administrators, citizens, committees—some scholars suggest that agreement on policy
decisions is a viable goal and one that is the most policy makers can hope for (Lindblom). With
this in mind, it is at least somewhat understandable that often the policy formation process and
the implementation process are thought to be separate and distinct. If it takes such a
concentrated effort to reach a consensus on a policy in the first place, is there time, energy or
effort to confront the implementation specifics? Or should those specifics be worked out through
administrative agencies? It has often been assumed that the policy formation process and

747 | P a g e
implementation process are distinct; however, much like the politics-administration dichotomy
(Wilson, Goodnow, Waldo) public administration has moved past the “either/or” framework
of most public administration arenas and recognizes that dichotomous relationships between
policy makers and implementers no longer exist.

The following public policy theories center on how policies are made and who has the most
influence on policy decisions. In group theory, policies are developed by predominant interest
groups. Groups that have the most political power and access to political processes will see
their demands met through public policies. As an example, Wildavsky discusses the political
power of older adults through groups such as the AARP and their influence on policies such as
Social Security and Medicare. Wi l davsky would argue that these entitlement programs are not
likely to be significantly altered because of the relative power of groups that have a vested
interest in preserving them. Lowi, in The End of Liberalism , strongly criticizes interest
group politics and pluralism and maintains that the competing interests of various groups
ultimately stifles government progress. Lowi suggests that when power in the government is
decentralized and „ everyone has power‟ in the form of groups then power is “watered
down.” Thus, if so many groups are competing for power, no dominant power emerges and
government is stifled (Lowi).

Elite theory (Dye and Zeigler) argues that powerful players—particularly in the economic and
political arena—determine the course of public policies. In other words, a select few in the
capitalist democracy determine policy directions and will steer policy decisions toward more
favorable outcomes for the elite (corporations, high-ranking government officials, etc.). An
example of this policy theory might include tax cuts for the wealthy while benefits for
economically disadvantaged groups may be less generous. The economically and politically
disadvantaged groups (i.e. Food Stamp or TA N F recipients) have little social, economic, and
political power to impact public policies.

Kingdon‟s policy streams model also describes how multiple influences and actors
converge to make public policy decisions. The policy streams model is described as multiple
streams—“streams” being problems, policies and new ideas, participants, and politics—that
converge together in a policy window or an opportunity to form public policy. Kingdon analyzes
what makes a policy window opportunity emerge (or as he says, „hot‟) in a particular time
while other policy choices, while being equally important, may not make it to the policy agenda.
For example, two years ago, immigration reform was a prominent topic on the policy agenda at
the federal level. When federal reforms were not instituted, states and local governments (including
748 | P a g e
Nashville) began instituting policies directed at curtailing illegal immigration, punishments for
businesses that hired illegal workers, and English-first language proposals. Now with emerging
policy concerns centered on the continued downward spiral of the economy—immigration
reform has taken a “back seat” to other policy considerations. Another example of
Kingdon‟s policy streams model would include welfare reform in 1996—public perceptions,
political agendas, and policy reform ideas converged to develop a massive overhaul of welfare in
the United States.

Policy Decision -Making Models

Understanding implementation of policy studies should also involve an understanding of how


policy decisions are made or the policy formation process or cycle (Anderson, Stone,

K i ngdon). Some scholars argue that an integration of implementation considerations should be


inherent in public policy formation (Wildavsky & Pressman; Elmore); however, much of the
public policy formation literature primarily outlines the formation process while focusing less
on the process of implementation.

The decision-making element of policy formation is of central concern in public policy research.
For example, several models exist to explain decision-making processes in public policy
formation: rational-comprehensive model; garbage can model (Cyert & March); mixed- scanning
(Etzioni). The rational-comprehensive model includes defining the problem, consideration of
policy alternatives and consequences, deciding between policy alternatives, then finally, policy
enactment. Herbert Simon advocates that bounded rationality of individuals limits „ rational‟
decision making. To Simon, bounded rationality suggests that public actors are limited in
their decision-making capacities because they have access to limited information or the

information may be filtered by the individual‟s own constructs and experiences. In


other words, individual actors do not always make „ rational‟ decisionsthey are
limited by time, circumstance, experience, and information access; therefore, rational
decision making is limited by these boundaries (Simon).

Because of the bounded rationality constraints of actors, Simon would argue that many policy
decisions result in administrators and policy officials „ muddling through‟ or what
Lindblom would characterize as incrementalism. The concept of incrementalism is considered a
descriptive theory in policy formation. In other words, prominent scholars like Wildavsky and
Lindblom would argue that incrementalism is „ what really happens‟ in policy

749 | P a g e
formation, thus,

normative theories of how policies should be made (i.e. rational-comprehensive approach to


policy formation) are not realistic.

The garbage can model suggests that policy decisions are made in „organized anarchies.‟An
example might involve a university administrator who makes policy decisions and justifies the
rationale later. In the garbage can model, there are various forms of information that flow into the
garbage can of policy formation, these information flows are essentially combined, and the
resulting policy is a combination of various information sources and influences that
„ flow out‟ of the proverbial garbage can.

Mixed-scanning (Etzioni) involves scanning the environment for the most pressing or seemingly
relevant policy issues, then addressing those policy issues first. In this way, some issues are
isolated and addressed by public policy, but not all. The disadvantage to this approach is that
there may be a tendency to focus on the bigger problems/issues or even crises while neglecting
other public policy concerns.

Public Policy Implementation: Top -Down, Bottom -Up, Hybrid, and Principal -
Agent Approaches

Anderson (2006) would note that the policy cycle has several stages, including defining the
problem and agenda setting, consideration of and decisions between possible policy
alternatives, implementation, and then evaluation of policy progress (success, failure, or „the
jury is still out‟).

There are several approaches to policy implementation which will be discussed in the remainder
of this paper—top down approaches, bottom up approaches, and hybrid approaches. Top down
approaches to policy implementation involve a strong hierarchical structure and would purport
that policies are implemented through higher level administrators, who are ultimately
responsible for the success or failure of policy implementation. In his early work, Aaron
Wildavsky was considered a “top-downer.” In Pressman and Wildavsky‟s landmark
research on policy implementation in Oakland, California, they questioned why federal
work policies in Oakland failed. Their assumptions centered on what administrative and
political officials did or did not do correctly in the policy formation process and advocated that
if policy structure is viable, then implementation should not be a problem. Pressman and
Wildavsky looked for „ holes‟ in the policy formation, structure, and development

750 | P a g e
process to explain the failure of federal policies in Oakland.

Lipsky—in his now classic work on „ street-level bureaucrats‟ suggested that policy

implementation may occur more so at the bottom-level of the administrative agency than may be
assumed. “Street-level bureaucrats” for Lipsky are the fire fighters, police officers,
social workers, and food stamp case workers that have direct contact with the public being
served by the policy. Lipsky argued that street level bureaucrats may have too much discretion
when it comes to policy implementation and have more influence on policy outcomes than
previous research outlined. While Lipsky‟s study focused on the impact that lower
level bureaucratic workers have on policy implementation, the ultimate conclusion from his
studies was that too much discretion on the part of the street-level bureaucrat was negative.
In other words, Lipsky‟s research supported (for him) that street-level bureaucrats have too
much discretion in policy implementation. For example, the street-level bureaucrats‟
perceptions of clients may negatively impact effective implementation of the designed public
policy. As an example, if a police officer or social worker is racist, will they be effective
implementers of public policy? If a welfare case worker perceives a client to be reliable (or
non-reliable), how will this impact the client‟s benefits and other considerations in
implementing the policy rules and guidelines of TANF?

A third approach—the hybrid approach—is another perspective on policy implementation


research and considered the contemporary culmination of early public policy implementation
studies. This approach ultimately combines both the top-down and bottom-up approach to
policy implementation. In the hybrid approach, of which Wildavsky, as well as Elmore
eventually converted to—both top-level officials in the hierarchical structure of the public
bureaucracy and political arena, as well as the bottom-uppers or street-level bureaucrats, have
significant influence on the implementation of public policies. In the hybrid approach, both the
top-down and bottom up perspectives on implementation are valued. From the top-down
approach, the political support of the policy, the structure and design of the policy (policy
formation), as well as administrative resources to carry out public policies (budgeting, personnel,
etc.) are important in successful public policy implementation. At the same time, the bottom-up
perspective, which involves the direct contact of agency personnel and bureaucratic workers with
their constituencies (i.e. the public or clients) also matters. Street-level bureaucrats are, after
all, the face of the government to the people, thus, their impact on public policies cannot be
marginalized (Lipsky, Halachmi, Easton).

751 | P a g e
Public policy implementation studies can also be informed by other theories, namely
principal -agent theory. For purposes of this discussion on public policy
implementation, let‟s assume that the “principals” are policy makers, such as
administrative managers and politicians while “agents” are in charge of implementing
public policies through various mechanisms.

Forrester would suggest that agents may act in their own self interests in order to preserve power,
status, or resources and withhold information from the principal (moral hazard). As applied to
public policy implementation, let‟s take the example of the state of Tennessee policy
makers (principals) contracting out work placement services of TAN F/Families First (aka
welfare) clients. In other words, the state has private agencies finding work for TANF clients
rather than the Tennessee Department of Human Services. If the work placement agencies (in
principal agent theory, the „ agents‟) act in their own interests by cutting costs or lack
the staff to help TANF recipients locate gainful employment, these actions inhibit successful
policy implementation.

Future Directions of Implementation Research

Scholarly consensus can be reached on the value of all public policy implementation
approaches—top-down, bottom-up, and hybrid and even the inclusion of other theories, such as
principal agent theory. The hybrid approach has had the most converters, such as Wildavsky and
Elmore, and appears to be more descriptive of implementation reality. While the hybrid
approach in public policy implementation may lead to further developments in public policy
research—not only for the implementation discourse, but for the policy formation discussion,
it is this author‟s position that even the hybrid approach may no longer be sufficient. Since
many public policies are implemented by a variety of actors—not just in public agencies, but
increasingly in private agencies as well—should there be also be a top-down, bottom-up, and
“sideways” (private organization implementers) approach to public policy implementation?

If public administration research points to the influences of both top-level (politicians and
administrators), as well as bottom-level public officials (agency representatives, case workers,
social service agency personnel), in addition to other actors in the private sector now involved in
the delivery of public services, then should it not follow that all of these groups should be
involved in public policy formation? If we refer back to various policy-making models, such as
elite theory or group theory, then it may be suggested that Lipsky‟s street-level bureaucrats will
still have limited input in policy formation while ultimately still impacting the policy

752 | P a g e
implementation process. In addition, if the key players (administrative and political, personnel
and agency representative, as well as private agency representatives) are all involved in the
policy formation process in order to impact implementation, then it is this author‟s
contention that public policy implementation research will expand to include as many theories
and perspectives as public policy formation and decision-making models.

TSU: Ph.D. Public Administration Comprehensive Exams

Spring 2009 Thursday, February 26, 2009

QUESTION TWO: In his classic 1940 APSR article, V.O. Key discussed the absence of a
theory of budgeting that addresses the fundamental question, “On what basis shall
it be decided to allocate x dollars to activity A instead of activity B?” Discuss the
development of scholarly attempts to remedy this lack, citing relevant literature. Would
you argue that a useful theory or theories of budgeting have been developed? Why or why
not? Have any of these theories adequately distinguished the differences between capital
and operational budgeting?

V.O. Key‟s often cited question--“On what basis shall it be decided to allocate x
dollars to activity A instead of activity BI ”—forms the basis for trying to determine if
there is, in fact, a budgeting theory or if public administration is still working toward one. While
there have been significant scholarly attempts to remedy this lack of budgetary theory, some
would advocate that a “theory of budgeting is a theory of politics” (Wildavsky) and
that incrementalism is the primary method through which budgeting decisions are made.
With those concepts in mind, additional attempts at developing a theory of public budgeting
are unnecessary (Wildavsky). This paper will discuss not only various budgeting theory
proposals, but also budgeting concepts and historical changes in U.S. public budgeting which
ultimately provide a background to the academic discourse on public budgeting theory.

Public Budgeting Concepts

A public budget is an accounting, political, policy, and management tool (Rubin). A


budget—in a very general sense—is designed to balance expenditures and revenues (or expenses
and income, for example, in a household budget). However, public budgeting involves more than
just a simple balancing of expenses and revenue. Because a public budget involves decision-
making, policy choices, and resource allocation for a society, the public budget involves value
decisions that affect a large group of people (Wildavsky; Rubin; Caiden; Mintz & Stevenson).
Historical budgeting reforms have emphasized various utilities of the budge t—as a control
753 | P a g e
instrument, an administrative instrument, an accounting instrument, a political instrument, and a
policy instrument. During various periods of history and as American society has changed, the
conceptualization of how the public budget is to be used has varied greatly.

A History of the Public Budget in the United States (and a Possible Explanation
for the Lack of an Overarching Budgeting Theory?)

A discussion of the history of public budgeting in the United States is essential in


understanding the evolution of budgeting theories in public administration. As Raadshelders
would point out, American society has expanded, thus, so have government servic es. As a result,
these combined forces have greatly impacted the conceptualization of the public budget and its
purpose. Many authors, such as Janet Kelly, Tyer and Wiland, as well as Aaron Wildavsky and
Irene Rubin, have written about the historical evolu tion of public budgeting in the United
States. The history of budgeting has been shaped by Presidential and societal
goals/issues/changes, thus, it is this author‟s contention that this variation in reforms and
budgeting adaptations throughout American history is one of the possible reasons that a
unifying budgeting theory is difficult to ascertain. In other words, Key‟s (1940) question,
“on what basis shall x dollars be allocated for activity A instead of activ ity B” could be
answered as “it depends on what period of history and what President we are talking
about.” It is not this author‟s intention to sound flippant about the development of a
budgeting theory; however, it is apparent that reviewing the evolving history of the U.S. public
budget suggests that one answer to Key‟s question might be “it just depends...on who is in
control and where society IS at that moment” (author‟s quote and wording).

The Taft Commission in 1912 was the first to suggest that a federal budget was a necessary
ingredient in directing the affairs of the federal government, and Frank Goodnow and William
Willoughby—central figures in public administration history—were proponents of this initiative.
Although Willoughby and Cleveland suggested the need for a federal budget to direct government
programs and initiatives, it was not until the Budgeting and Accounting Act (BAA) of 1921 that
the federal budget was implemented and began to form the basis for the federal budgeting
system today. Prior to this period, cities and municipalities had budgeting mechanisms in place,
primarily to curtail corruption at the local level, but the federal budget was in its primary
stages. During this time period, the public budget was primarily an instrument of control and
line-item budgeting was the dominant budgeting form (Tyer and Wiland).

Under President Hoover, budgeting focused on performance, or “not on what

754 | P a g e
government buys, but what it does” so government activities were the primary focus
(Tyer and Wiland).

During the 1960s and under President Lyndon Johnson, budgeting was used as a planning tool
under the Programming, Planning, and Budgeting System (PPBS). The primary aim of PPBS
was multi-year planning to determine program budgets. However, PPBS was replaced by Zero-
Based Budgeting (ZBB) under President Jimmy Carter‟s administration. Under
ZBB, program and administrative budgets had to essentially “start over” each year—
budgets could not use the previous year‟s budget to determine the current or future
budgets, thus, ZBB required that agencies justify their programs and expenditures year
after year. By the 1980s, ZBB ceased under President Ronald Reagan and there were efforts
at curtailing the federal budget deficit, thus a series of acts were implemented to address these
issues. In the 1990s under the Budget Enforcement Act, budget expenditures had to be balanced
with increases in taxes or cutting other expenses, but by 2002 the BEA was not re-instated.
Since then, the U.S. federal government has spent more and taken in less. As a result, the
United States is currently facing an enormous budget deficit, thus, annually increasing the
total federal debt (Rubin).

Many budgeting theories center on how the budget is a reflection of broad economic and political
changes in a society, thus, a socio-historical overview is important in understanding public
budgeting theoretical development. Janet Kelly discusses budgeting phases and reforms
through changes in society or what she refers to as “public-regarding” and “private-
regarding.” As an example, the New Deal period was a reflection of “public-
regarding” in budgeting decisions whereby resource allocations were made in
concurrence with dominant policy decisions at the time that aimed to institute economic and
work assistance to the middle class and poor. Because many New Deal policies were aimed at
lifting the country and its citizens out of the Great Depression, public-regarding was the
dominant theme in budgeting during this ti me period (Kelly). The 1980s and 1990s were
considered a time of private-regarding for the following reasons: 1) the lack of trust in
government performance and accountability, 2) more emphasis on private agencies taking on
tasks previously filled by government agencies, 3) tax cuts and „ trickle-down economics.‟
These examples refer to the shift in resource allocation from the public to the private
sector, as well as general budgeting policies that focused on expanding private agency roles in
the delivery of government services and emphasizing government accountability and
entrepreneurship (Kelly).

755 | P a g e
An Examination of Viable Budgeting Theories

Irene Rubin is a seminal author in public budgeting who has not only influenced the discourse
on budgeting theory, but also budgeting concepts and challenges as well. For Rubin, budgeting
reflects societal theories, thus, she discusses Neo -Marxist, Incrementalism, and Public-
Choice budgeting theories. For Rubin, Neo -Marxist budgeting theory reflects a
conceptualization that the most powerful economic and political actors in society direct the
budget. Neo-Marxist theory has its root in Critical and Conflict paradigms (Babbie; French,
Spears, Stanley; Burrell and Morgan) in that there is a focus on social injustice—powerful
groups “win” while less-powerful groups (i.e. racial/ethnic minorities, women, the poor) are
disadvantaged. Under the Neo-Marxist theory of budgeting, the economic and politically
powerful will have their interests represented in the public budget more so than disadvantaged
groups. As a practical example of how this theory might explain contemporary events, tax cuts
for the very wealthy or an expansion of defense spending would be reflections of the economic
elite and military power elite (Dye and Zeigler; C. Wright Mills). This theory would also explain
why many social programs that serve the poor, such as Food Stamps, TANF/welfare, or housing
assistance may receive a lower percentage of the federal budget pie (less than 4% collectively).

Incrementalism is a “favorite” budgeting theory throughout the literature as it is deemed


by many to reflect a descriptive theory of budgeting or an examination of what really happens in
budgeting vs. normative theories that suggest what „ should‟ happen. Aaron Wildavsky
and Charles Lindblom are proponents of incrementatlism—in public budgeting and in public
policy. For them, incrementalism reflects what policy makers really do as it relates to public
budgeting. Furthermore, Wildavsky would argue that budgeting is an inherently political
process, thus, the best way to “satisfice” (H. Simon) or “muddle through” (Lindblom)
the political process of budgeting without a politician risking a chance for re-election would
involve making incremental changes to the budget rather than any massive overhauls in policies
or programs. Wildavsky and Paul Light would note that while major policy decisions that
inevitably affect budgeting allocations have been made (i.e. Medicare, Medicaid, Social
Security), most policy and budgeting decisions are incremental in nature.

A public-choice theory of budgeting suggests that individual actors or “greedy


bureaucrats”(Forrester), act in their own self interests to maximize their utility. In public-
choice theory, actors support policy and budgeting decisions that reflect their own self interests
rather than the public interest, whether we are referring to individuals or bureaucratic
administrators or personnel. Under this theoretical umbrella—everyone is out for themselves (or
756 | P a g e
their agency).

Other budgeting theories include Mintz and Stevenson‟s „ theory of budgetary tradeoffs‟
or what may be referred to as the „ guns versus butter‟ argument. In this theory,
defense

allocations may be made at the expense of programs for the poor, thus, budgetary decisions
reflect a „tradeoff‟ which essentially reflects a value system—do we want better defense or
more resource allocations for the poor? Mintz and Stevenson found that this theory does not
always hold true, but it is relevant in discussing value-based decision making in public
budgeting allocations.

Institutional economics theory has also informed public budgeting theory through principal-
agent (Forrester; Eisenhardt; Moe) and transaction cost theory (Whicker and Mo). Also, theories
of agency mission reflect public budgeting allocations. If the agency is distributive,
redistributive, or regulatory—these various agency missions will reflect varying budgetary choices.

Viable Public Budgeting Theories: Do They Already Exist?

Because this author tends to view public budgeting as a reflection of public policy choices, it is
this author‟s position that viable public budgeting theories already exist. Perhaps
because of this author‟s own paradigmatic view, which is essentially the Conflict
paradigm (Babbie; French, Spears, Stanley), public budgeting is reflective of existing theories—1)
public budgeting as politics and 2) public budgeting as a conflict between competing interest
groups (i.e. Neo-Marxist, theory of budgetary tradeoffs). Budgeting as politics is relevant and
applicable (Wildavsky; Peterson, 2003) while public budgeting as reflections of socio-historical
changes (as well as Presidential choice) is also reflective of political influences (Tyer and
Wiland; Rubin). However, for this author, budgeting as a political process is not explanation
enough. A theory of budgeting centered in politics must also be combined with Neo-Marxist
theory (Rubin) or a theory of budgetary tradeoffs (Mintz & Stevenson). Put as succinctly as this
author can, it is my position that public administrators may be informed by the
interconnectedness of both the political and Neo-Marxist public budgeting theories, whereby
political preferences and power for budgeting resources may be understood in a framework of
social justice, struggles for power among advantaged and disadvantaged groups. Based on those
assumptions, theories of viable budgeting theories already exist (in this author‟s view).

Distinctions in Budgeting Theory between Capital and Operational Budgets

757 | P a g e
Capital and operational budgeting have traditionally been separated in the federal budget process
to eliminate issues where capital projects may be initiated but not completed due to inadequate
funding. However, the public budgeting theories that this author is familiar with do not
distinguish between capital and operational budgets. It is this author‟s contention that
capital budgeting seems to be less transparent than operational budgeting and flies under the
radar in budgeting decisions and out of the „ black box.‟ While it is understandable
that the federal government would not want to initiate building or infrastructure projects
without being able to complete these projects as a result of inadequate funding, thus the need for a
capital budget, it would seem that budgeting theories should incorporate both types of public
budgets.

The Future of Budgeting Theory (if there is one)

Naomi Caiden (from her work: The Great Unraveling) would suggest that contemporary public
budgeting and theory is characterized by uncertainty. The public is more resistant to tax
increases (thus, revenue increases for budgeting) while at the same time, th e federal government
is increasingly taking on more debt burden and entitlement spending (Caiden; Rubin; Peterson;
Wildavsky). This current system of budgeting in the United States (increased spending while not
being offset by increased revenues or spending cuts) cannot be sustained over the long-term and
can (or will) cause federal budget instability. Caiden would also suggest that this new wave of
instability within the budget inhibits the development of a budgeting theory. To develop a
budgeting theory, the public budget must become more stable (Caiden).

However, as this author has noted, public budgeting, even amidst instability, could still be
understood in the context of political decisions and power advantages. To illustrate —in the
current economic crisis in the United States, even amidst public concerns over the rising
national debt, there is still a political resistance to raising revenues (i.e. raising taxes). The idea of
raising taxes is politically unpopular, even in the midst of great uncertainty about the
future of entitlement programs on which much of the United States population depends, not to
mention concerns about the rising debt for future generations (Caiden; Peterson, 2003). In
addition, Cai den notes that even if concerns over the national debt are not central, concerns
over the interest on the debt is a concern in the federal budget pie. Currentl y the interest on the
national debt comprises as much (or maybe as much by now) of the federal budget pie as
Medicaid. Peterson (2003) notes that many middle-class Americans benefit more from federal
entitlement programs, such as Medicare and Social Securi ty, and have traditionally had the
power to resist any changes or overhauls in these programs. These trends, again, point to the
758 | P a g e
political unpopularity of reforming entitlement programs, raising taxes, even in the midst of fears
that the U.S. federal government has officially over-extended itself financially.

Given contemporary events and budgeting challenges, public administration discourse on public
budgeting issues should be enhanced and expanded. While the search for a possible unifying
theory of budgeting may still be on the horizon, practitioners in public administration are
currently dealing with more practical issues involving securing funding for government
services. Focusing on how to help public agencies navigate these uncertain budgetary times may
prove to be more challenging for public administration researchers and theorists than agreeing
on or determining a theory of budgeting.

TSU: Ph.D. Public Administration Comprehensive Exams

Spring 2009 Friday, February 27, 2009: DAY TWO

QUESTION THREE: It has been argued that the field of public administration theory is in
crisis. Do you agree with this argument? In your answer, address the paradigmatic issues
that gave rise to this argument and explain any relationship you see between th ese
paradigmatic issues, the status of the field, and future theory development.

Vincent Ostrom was one of the first to suggest that there was an “intellectual crisis”
in public administration. Behn said the field needed “big questions” to focus on the macro
issues public administration should begin to address—starting with public management--in
order to really move forward as a field. Neumann said Behn was correct that public
administration needed to address some “big questions” but Neumann suggested that
those questions should center on public organizations—their roles, processes, efficiency,
motivation, etc. Kuhn suggested that for a field to be considered in the “normal science”
context, it must have the elements necessary for a paradigm, so with that concept in mind, is
public administration really a science, a discipline, a field—what is it? Waldo suggested that
public administration was more like a profession, similar to medicine. Denhardt and others
have suggested that most of the students that have emerged from U.S. public administration
programs have produced research that is more applicable to their current professional roles and
that Ph.D. public administration graduates are more interested in advancing their professional
status and padding their resumes rather than advancing the field with new and scholarly
research in Public Administration.

While all of the above examples suggest that it may not be clear what the future of Public
Administration research and education looks like, or what the dominant paradigm in public

759 | P a g e
administration is or whether public administration is a discipline, a science, or a profession,
what is clear is that public administration research and literature has dedicated a considerable
amount of research time trying to figure out exactly what IT is. Whether that is considered a
crisis is dependent upon one‟s perspective.

This author would argue that public administration is in fact an academic discipline while also
being a field of practice. By definition a discipline has a body of research and knowledge that is
accepted among its participants and is considered a viable educational area that is legitimized
through academia. Even though many public administration scholars may not agree on various
theories, paradigms, or directions in the field, it still does not negate the fact that public
administration is an academic discipline. Therefore, if public administration is considered an
academic discipline, the next question involves determining the identify of that discipline. It is
also this author‟s position, that there is a crisis in public administration, but not one
that involves paradigms or theories (more on this at the end... last section).

Theoretical and Paradigmatic Evolution in Public Administration

What should public administration be? According to Frederickson, public administration should
advocate for efficiency, effectiveness, and social equity. Raadshelders and Svara would suggest that
public administrators are indirectly accountable to the public, but that protecting the public
interest should be the guiding force in public administration. With these tenets of public
administration in mind, what are the theories that help explain public administration as a
practice and as a discipline? In Frederickson and Smith, they discuss many of the various
theories that have been used to characterize public administration:

 public institution theory (public organizations are part of a larger network of social
institutions)

 bureaucratic politics theory (politics is inherent in bureaucracies and through


administration, thus these two elements are strongly interconnected)

 public choice theory (bureaucracy undermines democracy; bureaucrats are self interested;
market should be more involved than government)

 decision-making theory (how are political and administrative decisions made in the name
of the public interest)

 public management theory (focus is on fostering effectiveness and efficiency in public


agencies)

760 | P a g e
 public organization theory (what are the processes and motivations in a public organization;
how does the structure and organizational culture influence public organization efficacy)

In addition to these theories, public administration has also been thought to coincide with models
of government (Stillman; Peters).

Public Administration Reflects Models of Government

Stillman would suggest that the academic field of public administration has followed the path of
state development. Stillman cites four models of public administration aligned with state
development—no state, bold state, pre -state, and pro -state. Similarly, Peters
postulates that there are other models of government that public administration practice and
focus has coincided with: market, participatory, deregulated, and flexible government .
Side-by-side comparisons of both Stillman and Peters‟ models show strong
similarities. For example, in Stillman‟s Bold State and Peters‟ Market model, there is an
emphasis on a strong executive and use of market forces in the delivery of government
services. Both the Market and No-State models are reminiscent of public choice theory. On
the opposite end of the model continuum is Stillman‟s Bold State and Peters‟
Participatory Government—both of which include a strong bureaucratic structure,
emphasis on citizen participation, a belief in government and its ability to address social issues.

In Stillman‟s Pro-State and in Peters‟ Deregulated government, there is more


inclusion of professional experts or what Stillman refers to as „ technocrats‟—the focus is on
contracting out services in the way of government consultants, technical skills, information
management, and other specialty areas. In other words, these models reflect a government partially
controlled by professionals in various business and technical fields who claim to be experts, but
who, according to Stillman, are tied to both the public and private sector and have no strong interest
in maintaining the public good, as they are only there for the job and to carve out their niche in their
specialty area.

In both Peters‟ and Stillman‟s typologies, public administration is situated


within models of states and governments—thus public administration practice resembles
government structure and goals.

Paradigms: What Are They?

What is a paradigm? One answer might be, “it depends on who you ask.” The “what
is a paradigm” discussion has been plagued with semantics issues. Even Thomas Kuhn

761 | P a g e
(considered the main culprit/genius in beginning the paradigm discussion among sciences)
and others (Rainey; Henry; French, Stanley, Spears) may have differing versions of exactly
what a paradigm is. While many define what a paradigm is in a somewhat similar way—that
is, a paradigm is a way of viewing, a perspective on the world, a perspective on reality and the
way things work—most of the disagreement comes from the way paradigms are
conceptualized.

The use of the word paradigm is present in many academic disciplines, but how it is used and
what it really means is the central question. However, across disciplines most would agree that
understanding one‟s own paradigm is essential in understanding other paradigmatic
views. A logical next question might be, why is it even important to understand one‟s
own paradigm? If we know that we view the world through one paradigmatic lens,
it helps us see the “other side” or another‟s view which can be useful in
understanding society and government, thus, this could be a positive direction in
understanding public policies and informing public administration. In the social sciences,
one‟s paradigm informs many facets of research—the research question, the research
methodology, the research focus, etc. For example, if I understand my paradigm to be within the
conflict perspective (Babbie; French, Spears, Stanley; Burrell and Morgan) then this explains
why I am interested in social injustice, why I may choose the research projects that I do, and
why I value research that uses quantitative and qualitative perspectives, but I also understand
that social events are interpreted by actors and researc hers in different ways. With these
concepts in mind, how might ones‟s paradigmatic view shift or be altered?

For one‟s paradigm to be altered, resulting in a paradigm shift, all previous


assumptions about an issue or social phenomena must be overturned. To illustrate with an
example from the natural sciences, previously the world was thought to be flat and this was
the dominant paradigm. Later, a new truth was discovered —the world in fact was not flat but
round—thus replacing the previous paradigm or causing a paradigm shift (Kuhn). Unlike the
natural sciences, however, social science “truths” are not necessarily replaced or
completely overturned—they may still be on the periphery and not completely abandoned.
As a public administration example of how social science paradigms may just move to places
of greater prominence rather than being replaced (Kuhn), let‟s use Henry‟s version of
paradigms. Once the predominant concepts in scientific management (during
Taylor‟s time) were abandoned to include more of a human element, the concepts of

762 | P a g e
achieving efficiency in organizations were not entirely abandoned —it was still true that there was
a desire for organizations to become more efficient, there was just a progression in thought about
how this might be accomplished.

Now that we have what some paradigm concepts out of the way, let‟s evaluate various
paradigm developments in public administration.

Paradigms in Public Administration: Do We Have One?

Nicholas Henry suggested that public administration has five paradigms, which
essentially divide the discipline into time periods and focus areas:

 Public administration as politics/administration dichotomy

 Public administration as scientific management

 Public administration as administrative science

 Public administration as political science

 Public administration as public administration

Henry‟s categorization of public administration reflects the different areas of public


administration focus during specific time periods. For example, in the early stages of public
administration, Woodrow Wilson and Frank Goodnow‟s concepts of the politics-
administration dichotomy (or the idea that politics and administration should be separate)
dominated the field. Later as scientific management and the work of Frederick Taylor and Henri
Fayol became centerpieces of organizational efficiency (modeled after many industrial
practices), Henry declares this the scientific management period of public administration.
Later public administration tries to return to a focus on politics, and Waldo and Wildavsky
would be key theorists in this area suggesting that politics and administration cannot be separated
and political implications and influences are inherent in bureaucratic processes.

French, Spears, and Stanley (2005) disagree with Henry‟s characterizations of the
paradigms in public administration. Based on Kuhn‟s definition, French, Spears,
and Stanley (2005) suggest that a paradigm must encompass several components: ontology,
epistemology, and methodology . In French, Spears, and Stanley (2005), they denote five
paradigms of public organizations and describe the ontology (view of reality); epistemology (how
we know reality); and methodology (methods we use to determine reality) within each of these
paradigms. The five paradigms of public organization theory in French, Spears, and Stanley (2005)

763 | P a g e
include: 1) positivist; 2) post-positivist; 3) interpretivist; 4) conflict; and 5) post-modern.

Each of these paradigms expresses views on reality, how one comes to know and interpret
reality, and preferred methodologies in these views. As an example, in the positivist paradigm,
more common in the natural sciences, there is a reality that can be understood and determined
through objective research and causality can be determined. In the post-positivist paradigm,
reality can be known but with limitations, yet, research can inform reality because the research
can be objective. In the conflict paradigm, there is a focus on social justice, inequality, particularly
with certain population groups (i.e. racial, gender, economic, etc.). In the conflict view,
research can inform some social realities but these are socially constructed realities and may
not necessarily be understood through quantitative or experimental research.

Others (Rommels and Christianens) suggest that a paradigm for public administration is not
necessary and we should stop looking for it. Their hope is that public administration students
will move away from the influences of their professors and previous researchers‟
perspectives and break out of the “we have to find a paradigm” paradigm in public
administration.

Future Directions: Does Public Administration Need a Paradigm or a Public


Relations Overhaul?

When first embarking on this Ph.D. program in Public Administration, this author began
reading about the identity crisis in public administration and it was this author‟s
assessment that there was no “intellectual crisis” (Ostrom) in public administration for the
following reasons:

1) public administration is valuable as a multi-disciplinary field and is only enhanced by


drawing on the fields of business, psychology, sociology and economics

2) current theories (bureaucratic politics, public institutions, public management, etc.)


within the field are sufficient to encompass key focus areas in public administration
and are still relevant

3) public administration is driven by Frederickson‟s ideas of what the field should


strive for: efficiency, effectiveness, and social equity

4) government is good, it can do great things, and public administrators should strive to serve
the public interest through both practice and research (Raadschel ders; Svara; Frederickson;
D enhardt)
764 | P a g e
While this author has not abandoned these previous perceptions of what public administration
is or should be, it is true that I am skeptical about the future of the field, but not because there
is a lack of an overarching paradigm or continued disagreements about overarching theories.
Where this author feels a level of concern for public administration lies in the following areas:

1) continued public distrust in government and the lack of value that much of the public
places on public administration and government roles and services (particularly in the
wake of this current economic crisis)

2) political resistance in making sweeping changes to address major social problems,


thereby paralyzing public administrators (i.e. public and political cries not to raise taxes,
reform Social Security, Medicare or other programs, thus leaving these areas vulnerable
to funding constraints)

3) the development of applicable research in graduate studies within public administration


that can truly be used to assist public agencies, or the lack of connection between
research and practice

Public Administration and government in general needs a public relations overhaul. Public
administrators continue to try and legitimize the field with paradigms, theories, etc. but in reality
the field will be legitimized when the public legitimizes it—and this may only be accomplished
through public relations, media, and strong efforts to bridge the ties between theory/research and
practice. Raadshelders would suggest that public administration is legitimized because
government is a social construction; however, even if government is legitimized, the public
perception of government still matters, and it continues to be less than ideal (for public
administrators anyway). If public administration begins to “feel” legitimized in the
public‟s eyes, then we can have as many paradigms and theories as we want...just as other
disciplines, such as economics, sociology, psychology have. These disciplines struggle with
various theories and paradigms and research initiatives, yet they do not suffer from an
identity crisis. In this author‟s view that is because these fields have gotten over their inferiority
complex. Once public administration is truly legitimized, trusted, and valued by the public,
public administration will no longer suffer from an identity crisis and it can have as many
paradigmatic and theoretical debates as other disciplines... without these debates being a
threat to its identity as a valuable and important field.

TSU: Ph.D. Public Administration Comprehensive Exams

Spring 2009 Friday, February 27, 2009: DAY TWO


765 | P a g e
QUESTION FOUR: To enhance accountability President Obama has asked you to study
various approaches to accounting in government and nonprofit organizations. What
approaches would you consider? What are the strengths and weaknesses of each approach?
Which approach/method would you recommend to the President? Given the Sarbanes -Oxley
Act would this approach be as useful in the case of both government and nonprofit
organizations?

President Obama has a strategic vision for the United States which includes restoring the
country’s economic stability while simultaneously returning to a focus on federal
government responsibility and accountability. While President Obama has asked the public and
private businesses to support, as well as assist, in the responsibility of lifting the country out of
the current economic crisis, he has also promised organizational and financial accountability, as
well as transparency, within the federal government.

Government accountability for the public’s financial resources has never been as
important as it is in the current political and economic context. Part of the challenge in managing
organizations involves accountability, and President Obama must meet this challenge. If he fails to
show accountability or transparency in the distribution of the country’s financial
resources, what will be the implications, not only for his legacy, but for the
country’s economic climate?

Including Financial Accountability in th e Strategic Planning Process

In this new Presidential administration, it is clear that President Obama and his staff will need
to encompass Mintzberg’s concepts on strategy and vision—have a plan and a long-term
direction for the country in mind, while also allowing for creativity in solving complex social,
economic, and political problems. As evidenced by recent cases of corporate fraud—on Wall
Street and in the housing loan industry, as well as various “Ponzi” schemes—it is evident that
Sarbanes-Oxley has not been sufficient to prevent these types of accounting frauds. At the
same time, with the current economic crisis and public concern over the ever-expanding
government role in banking and other economic industries, as well as an increase in federal
spending and debt, it is clear that accountability measures will need to be put in place in order
to prevent government abuse and waste of taxpayer dollars. Part of the strategic planning process
involves developing management strategies that will address long-range goals and objectives. In
this new Presidential administration, the strategic planning process must involve financial and
accounting transparency, as well as accountability.

766 | P a g e
Strategic planning involves long-range planning that encompasses determining the
organization’s mission and objectives, analyzing internal strengths (organizational
culture, accounting methods, personnel, structure), as well as analyzing external threats to the
organization (for private—competition, costs of acquiring goods and services and for public—
funding availability, political environment, social environment, etc.).

To Mintzberg, strategy involves a vision, a long-range conception of where the organization, and
in the case of the federal government, where the country is going. A strategy can also be a ploy
(outsmarting competitors), plan, or a pattern (Mintzberg). A strategic plan can assist an
organization (in this case, the federal government) in determining a viable direction and how
it will get there. While some authors (David; Quinn) address formalizing strategy through the
strategic planning process, Mintzberg advocates for the strategic manager to not lose focus on
adaptability and flexibility. Since a strategy involves a vision, a strategic plan must also be
flexible and adaptable to changes in the environment. Strategic planning has become a formalized
process in many organizations that Mintzberg would argue may inhibit creativity. While
organizations do need leaders who can “stick with the plan” and steer the organization in a
purposeful direction, an organization needs creative leaders than can help the organization
adapt to changes in their environment (Mintzberg). As a current example, Mintzberg has
developed a program that he suggests is better at developing effective leaders than many current
MBA programs, which in his view, are great at teaching students about business but not how
to be effective leaders of organizations. Mi ntzberg suggests that business/organization leaders
are not made in the classroom but in their environment. Leaders must be able to adapt to their
environment by assessing both external and internal strengths and weaknesses. While
managers must focus on the external environment, they must be equally in tune with the
internal processes and structure of their organization, which includes elements such as,
organizational culture, personnel resources, and accounting methods.

A Public Organization Challenge : Managing the Public’s Money

Many strategies for managing the public’s money have been utilized since the 1921 Budget
and Accounting Act created a federal system of budgeting (Willoughby; Cleveland, Goodnow).
President Hoover first emphasized the need for performance budgeting and a focus on
“what the government does, not what it buys.” Other Presidential terms also shed
light on various methods for managing the public’s finances and a focus on
accountability. During President Lyndon Johnson’s administration, PPBS or

767 | P a g e
Programming, Planning, and Budgeting S ystem was put into place in all federal agencies. This
budgeting system was a reflection of strategic planning at the federal level, manifesting itself in a
budgeting and accounting mechanism. The design of PPBS encompassed multi-year
planning and the determination of a program’s goals and objectives to determine funding and
budgeting allocations (Tyer and Wiland; Rubin). The strength of PPBS involved multi-year
planning which allowed a public agency to conceptualize long-term goals and strategies for
achieving organization objectives. During the 1970s ZBB or Zero -Based Budgeting was
instituted by President Jimmy Carter, modeled after a Texas Instruments budgeting system.
ZBB required agencies and programs to justify their programs year after year. Inste ad of
incremental increases each year, under ZBB program budgets began with a budget balance of
zero and started the justification process (Tyer and Wiland; Rubin). Performance -based
budgeting (PBB) was a Clinton era system which focused on program outcome s. However, there
are several considerations in measuring outcomes within public organizations or of
government services, such as, how do you measure the value of outcomes in public services
(Halachmi)? Additionally, is the cost—in time and financial resources—of measuring
outcomes in government agencies and for public programs worth it? In a practical example,
let’s assume that through the Tennessee Department of Human Services there is a
performance-based budgeting system in place to measure outcomes of the TAN F/Families First
welfare policy. If the Tennessee state legislature determines that in order to sustain budget
funding for the Tennessee Department of Human Services, the organization must reach a
particular performance criteria—for example, 75% of TAN F/Families First clients must be
working within two years. If this is the case and these are the conditions, what are the
challenges in measuring this performance criteria or outcomes of the current Families First
program? If administrative agencies are dependent on showing “successful outcomes” in
order to continue receiving funding, several questions must be addressed:

 How will the data be collected? What will it cost for the data to be collected, analyzed, and
evaluated?

 Is “must be working” for TANF clients conceptualized as full-time or part-time work?

 Will there be too much discretion of the agency in calculating the numbers?

 How does the agency determine what the outcomes should be? Should the outcomes
include: Work hours? Work income? Families being lifted out of poverty?

 Even if the agency meets the “outcomes” requirement of 75% of TANF

768 | P a g e
 clients in the workforce—are those clients still living in poverty? Are their basic needs
being met? In other words, what value are the outcomes if the overarching social issue (lifting
TANF recipients out of poverty) is not accomplished?

These are the challenges in measuring performance and accountability in government


agencies—determining how to place value on “successful outcomes” in programs, as
well as determining whether the cost of measuring and collecting data on outcomes will prove to
be a productive and efficient use of resources (Halachmi). Another challenge will involve sustaining
the funding for continued measurement of program outcomes.

Even considering the challenges of measuring accountability in government services and


programs, it is this author’s view that President Obama must institute federalbudget
accounting and financial methods that ensure accountability. However, if a history of public
budgeting tells us anything, it is that these reforms may not prove or show what they are
designed to—program success and efficient use of taxpayer funds. Through the recent economic
stimulus/economic recovery package, President Obama is accountable to the public for where
the money goes—and to whom. The financial industry bailout from 2008 administered by the
Bush administration is already encountering immense scrutiny for lack of oversight and
accountability. Furthermore, the economic and budgeting allocations that the stimulus plan
(thus federal debt) has committed the U.S. taxpayers to will surpass President Obama’s tenure
as Chief Executive. With that in mind, will funding for measuring outcomes and showing
accountability be sustainable? Public budgeting history informs us that any budgeting reforms,
program or financial systems put in place will most likely be replaced by the new
administration.

It is this author’s view that a hybrid approach, derived from previous budgeting and
accounting systems, must be developed to assist the Obama administration in ensuring
accountability and efficient use of the public’s resources. Unlike PPBS, the objectives and
goals for programs and agencies should be targeted for measurement in 12-16 months. Funding
must be made available to account for the measurement of accomplished objectives for the
economic recovery and stimulus plans (Drucker’s Management by Objectives). For example, if
the plan includes a goal of 2 million unemployed workers being placed into employment by the
beginning of 2010 then this objective should be measured by that deadline.

769 | P a g e
Appendix E
University of Georgia Comps
Questions

MPA Comprehensive Exam


Study Guide
MPA Comprehensive

Examination Fall 2007

Directions: Answer one (1) question from each of the following sets.

PUBLIC ADMINISTRATION AND DEMOCRACY

1. From your understanding of Woodrow Wilson's 1887 essay and the works of
Frederick Taylor, Max Weber, and Luther Gulick, develop an essay in which you
explore the possible logical connections between these scholars. What common
themes, if any, exist in the works of each of these individuals? Do any of the major
arguments developed by each complement works by the others? If so, explain the
connection.

2. Develop an essay in which you respond to the following assertion: public


administrators, as they go about the ir tasks of implementing the law, should make
value judgments designed to further equity in the provision of public services. How
could some scholars and administrators argue in support of this view? Why would
some observers find this use of administrative power undesirable? What is your view
on the issue?

PUBLIC PERSONNEL ADMINISTRATION

1. You recently went on a job interview for the position of Assistant Personnel
Director with a large local government. In the intervi ew the Director asked you how
your Masters in Public Administration prepared you for the position and if your
degree provided you with unique insights into human resources management issues
in the public sector. He went on to ask if you felt you were more or less prepared
than an applicant with a Masters in Human Resources Management. Write an essay
that explains how you would respond to the Director.

2. In merit systems, applicants for employment are screened for selection on the
basis of their performance on open competitive examinations. What are the various
types of examinations that are commonly used in the public sector? Ideally, how
would such examinations be developed? What is the concept of examination validity,
and how do we attempt to determine the v alidity of examinations used for selection

771 | P a g e
purposes? Which approach to determining examination validity is most useful, and
why is that the case?

PUBLIC BUDGETING AND FINANCIAL ADMINISTRATION

1. Governments normally collect revenue through their sovereign taxing power.


However, government also raises some revenue through voluntary exchange.
Lotteries are an example of such a voluntary exchange and have become a popular
source of revenue for state governments. In the State of Georgia, the lottery has
been used since 1993 to support the HOPE scholarship, which provides financial aid
to qualified students for post-high school study. However, scholars, practitioners, and
citizens express several concerns about the use of lottery proceeds to fund
government programs. Based upon your knowledge of the standard tax evaluation
criteria discuss the pros and cons of using lotteries as a government revenue source.
Cite relevant literature as appropriate.

2. The history of public budgeting is replete with efforts to integrate program


and performance information into the resource allocation process. What have
been the motives for such efforts? What have been the principal systems or
techniques used? What is the empirical evidence about the degree of success of
such efforts?

PUBLIC MANAGEMENT

1. Privatization of public services and activities has been one of the major
trends in governments around the world across the last several decades. Write an
essay on privatization in which you describe the trend and discuss the kinds of
functions or activities that governments are privatizing. Suggest reasons for the
privatization trend, and any evidence that you know regarding the impact of
privatization. Also, please discuss a ny significant issues that privatization raises
for public management.

2. A topic that often surfaces in public management is reform. What have been
some of the major reform movements that have affected public management over
the past century? What important changes have the various reform movements
advocated for public management? To what extent have these changes been
implemented and sustained over time in public management? Which of the suggested

772 | P a g e
changes do you consider most useful? Which of the suggested c hanges do you
consider least useful? Please explain all answers and refer to the relevant literature.

MPA Comprehensive Examination

Spring 2008

Directions: Answer one (1) question from each of the following sets.

PUBLIC ADMINISTRATION AND DEMOCRACY

1. It is often said that there is no dichotomy between politics and


administration. But is that really accurate? What about, for instance, the
idea of a merit-based civil service system insulated from partisan political
decision making? Is this idea old fashioned? Still relevant? If public
administration is involved in politics, are there some forms of politics which
can and should be kept out of administration? If so, explain the distinction
and use examples. If not, is there any legitimate justification for a
professional MPA degree?

2. Wallace Sayre once wrote that public and private administration are alike
in all unimportant respects. What is meant by such a statement? Is it
accurate? How do you assess the claim that administration is a generic
subject that can and should be studied and generalized about in a broad
sense rather than sector by sector?

PUBLIC PERSONNEL ADMINISTRATION

1. The need to find effective ways to counter discrimination on the basis of


factors such as race, ethnicity, and sex has been an imp ortant issue on our
policy agenda for a considerable number of years. Develop an essay in which
you discuss the evolution of anti-discrimination policy with respect to
public employment. What is the legal foundation of equal employment
opportunity policy in the public sector? Under what circumstances might an
employer be vulnerable to discrimination charges? As a public manager, what
issues would you pay particularly close attention to if you wished to avoid
allegations of discrimination?

773 | P a g e
2. What do you see as the most important components or aspects of a modern
merit system for public personnel administration, and why do you find them
to be important? How do we attempt to implement the merit principle in the
civil service? What impact, if any, do you believe currently popular civil
service reforms will have for the merit principle? What is your forecast for
the future of merit in public service?

PUBLIC BUDGETING AND FINANCIAL ADMINISTRATION

1. You recently have been hired by the Department of Bud get and Finance of a
medium size county government. The county manager believes negative public
perceptions of the national debt, now at approximately $8.4 trillion, may
jeopardize the success of an upcoming county bond referendum. She asks
you to prepare a memorandum which distinguishes county debt from national
debt, and delineates the advantages of bonded indebtedness as a method for
funding county projects. Your memorandum will be used in the manager’s
efforts to promote the bond referendum. What would you write?

2. What is capital budgeting? How does a capital budget differ from an operating
budget? How is a capital budget related to an operating budget? Why do
state and local governments do capital budgeting? What are some common
problems or difficulties associated with capital budgeting?

PUBLIC MANAGEMENT/ORGANIZATIONAL THEORY

1. What have you learned about theories of motivation? How are motivation
theories generally categorized? Identify four different theories of motivation and
describe them in detail. What assumptions do they make? What lessons or
insights do they provide for public managers?

2. Organizational theory has developed over many decades. Please develop an


essay in which you discuss that development. What was the nature of the
earliest theories? What assumptions did those theories make about people and
organizations? How were later theories different? How would you describe the
current status of organizational theory ?

MPA Comprehensive Examination


Summer 2008

774 | P a g e
Directions: Answer one (1) question from each of the following sets.

PUBLIC ADMINISTRATION AND DEMOCRACY

1. Reviewing the last 5 years of comprehensive exams, the most frequently


asked question in this section of the exam concerns the
politics/administration dichotomy. Why is that? In other words, why has the
discussion of this dichotomy been so central to the field of public
administration? Is this a uniquely American concern? Discuss h ow the
dichotomy has shaped the theories and research of the field.

2. In his classic article “Government is Different,” Paul Appleby asserts that the
administration of government is different from every other activity in society. In
what way is administration in the public sector different from administration in
the private sector? Consider some core processes of administration: financial
administration, strategic planning, and personnel administration and discuss
how these processes are different in the public sector.

PUBLIC PERSONNEL ADMINISTRATION

1. There are numerous constraints on personnel administration in the public


sector that are not present in the private sector. Suppose you were discussing
personnel issues with a colleague from the business school. Your friend asks
you to explain why public personnel administration is different from personnel
management in the private sector. How would you respond? Please give several
examples of differences and explain their impact and why they are present. Be
sure your answer focuses specifically on personnel administration.

2. Currently, the majority of public employees possess a “property right/interest” in


their positions. Explain what is meant by a “property right/interest” and
discuss the history of this protection for public employees. How is this right
established? How does this affect the work of public personnel administrators?
Discuss how recent civil service reforms in some locations have altered this
right.

775 | P a g e
ORGANIZATION THEORY - PUBLIC MANAGEMENT

1. Leadership is a topic that is often discussed in the literature of public


management but little understood. Write an informed essay in which you
define and discuss the role of leadership in public organizations. What do you
consider the most persuasive theories of organizational leadership? Why? How is
leadership similar, to or different than, related concepts such as management
and administration?

2. Some organization and manageme nt scholars take a “generic” approach to their


topic, arguing that we need general theory that applies across organizational
settings. They typically reject the usefulness of distinctions such as those often
drawn between public, private, and nonprofit org anizations. Other scholars,
however, contend that managing in a government agency has distinctive
challenges and requirements. Discuss the generic orientation, its pros and
cons, and evidence both for and against it.

PUBLIC BUDGETING AND FINANCIAL ADMINIST RATION

1. Since the start of modern public budgeting in the U.S. in the early 20 th

century, scholars and practitioners have elaborated on the fundamental


principles of public budgeting. Name any three of those principles, elaborate
on their main features, and, citing examples from the budgets of state or local
governments, discuss whether these principles are still applicable and used in
current practices.

2. Recently some political figures in the state of Georgia have proposed to


replace the local property tax for schools with a three percentage point
increase in the sales tax. Analyze the feasibility of this proposal, and its
potential consequences for public finance, using the evaluation principles that
you have learned. Use hypothetical cal culations to help illustrate your points
when possible.

Doctoral Examination
Public Budgeting and Finance
Fall 2004

Morning Session

776 | P a g e
Part I: Please answer one of the following two questions:

1. What have been the contributions during the past 30 years of the f ollowing laws
to the development of the federal budget process: Congressional Budgeting and
Impoundment Control Act (1974), Balanced Budget and Emergency Deficit Control
Act (1985), Budget Enforcement Act (1990), Chief Financial Officers Act (1990),
Government Performance and Results Act (1993) and Line -Item Veto Act (1996)?
What were the purposes and principal features of each act? How have they
influenced the budgetary roles of Congress and the president? What has each act
achieved; how has it influenced federal budget policy? Cite the most authoritative
literature at appropriate points in your essay.
2. Identify what you judge to be the ten most important peer reviewed journal articles
on government budgeting and finance written in the past 5 years. Wh at are your
selection criteria? What are the major themes, methods, conclusions and
contributions of each article? What has been the collective impact of these ten
articles on the field of government budgeting and finance?

Afternoon Session

Part II: Please answer one of the following two questions:

1. During the past 35 years lotteries have again become a popular source of
revenue for state governments. However, several concerns have been expressed
about the use of lottery proceeds for funding government programs. They
include: (1) adequacy-lottery proceeds generally contribute a small share of total
state revenues, (2) administrative efficiency-lottery proceeds generally are more
expensive than tax revenues to collect; (3) stability-lottery proceeds generally are
an unstable and unpredictable source of revenue; (4) equity-lotteries, if
regarded as a tax rather than a voluntary payment, are regre ssive in that they
place a greater relative burden on low income players, than on high income
players, and (5) fungibility-lottery proceeds may be substituted for state general
funds. An empirical literature has evolved around several of these concerns. What
are the most important pieces of empirical research about lotteries? What were
their methods and findings? What important empirical questions about state
lotteries remain to be addressed?

777 | P a g e
2. Citizens of Athens-Clarke County entering the voting booth on Nov ember 2, 2004
will be given the opportunity to vote Yes or No on the following question: “Shall a
special 1 percent sales tax be imposed in the special district of Athens -Clarke
County for a period of time not to exceed six years and for the purpose of rai sing
an estimated amount of $122,000,000 for [various projects listed]?” Describe the
principal features of the Special Local Option Sales Tax (SPLOST) in Georgia.
Applying standard tax evaluation criteria, assess the advantages and
disadvantages of SPLOST.

Part III: Please answer one of the following two questions:

1. Assume you are serving in an advisory role to a country that is in transition from
a planned economy to a market economy. Your task is to design an
intergovernmental financial system and local budgeting processes. Describe the
design for the system and processes, as well as the necessary steps and pre -
conditions for implementing the design. Include in your discussion any existing
literature and research on this topic.
2. During the recent period of fiscal stress in the American states, budget
stabilization, or so-called rainy day, funds have received increased attention from
policy makers and researchers. With specific reference to the existing literature on
this subject, assess the role of budget s tabilization funds in state government
budgeting.

Doctoral Examination
Public Budgeting and Finance
Fall 2005

(Morning Session)

Part I: Please answer one of the following two questions.

1. Since the approval of the Local Option Sales Tax by the Georgia Genera l
Assembly, most of the 159 Georgia counties have adopted the LOST, SPLOST
and/or ELOST programs. These programs have had obvious impact on the
finances of the counties. Elaborate on the observed effects of adopting one or all
of these programs by the counties, citing literature and providing an empirical
model that you may use if you are to conduct a study of these programs.

778 | P a g e
2. Public budgeting has proved to be a rich source of debate among scholars and
practitioners alike. The purposes, methods and outcomes have been challenged
repeatedly. As a result, budget reforms are common, and some might say too
frequent. Budget reforms such as line -item budgeting, zero-based budgeting, and
performance- based budgeting were all developed in response to perceived
shortcomings in some other system. They become popular only to become the
target of later reform efforts. Choose one such reform; discuss its theoretical
origins, its proponents’ critiques of prev ious methods, and the empirical evidence
from the scholarly literature with respect to the outcomes the reform generated.

(Afternoon Session)

Part II: Please answer one of the following questions.

1. Several local governments in Georgia are currently considering a policy of tax


“freezing.” Under such a policy, assessed value would be “frozen” at the time of
purchase and would remain unaltered until the next change of ownership, at
which time a new assessment would be applied, to be followed by another freeze
that would remain in force until the next change of ownership, etc. Tax rates might
change from year to year, but property assessments would not. Discuss the
motivations for, and fiscal implications of, such a local government tax policy. Do
you recommend adoption of such a policy? Why?
2. In recent years, there has been considerable interest in performance budgeting.
What is performance budgeting? What are its antecedents? With specific reference
to empirical research on the subject, how successful has it been in penetrating
traditional budgeting practices at the national and state levels in the U.S.? What
recommendations would you make to national and provincial governments in
China regarding adoption and implementation of performance budgeting?

Part III: Please answer one of the following questions.

1. What have been the contributions of the following laws to the development of the
federal budgetary process: the Budgeting and Accounting Act of 1921, the
Congressional Budget and Impoundment Control Act of 1974, the Balanced
Budget and Emergency Deficit Control Act of 1985, the Budget Enforcement Act
of 1990, and the Line -Item Veto Act of 1996? How have they influenced the

779 | P a g e
budgetary roles of Congress and the president? How has each influenced federal
budgetary policy? Cite the most authoritative literature at appropriate points in
your essay.
2. Congressman John Linder (R -GA) and others have proposed a national sales tax as
an alternative to the current federal income tax. Applying standard criteria for the
evaluation of taxes assess the national sales tax proposal.

Doctoral Examination
Public Budgeting and Finance
Fall 2006
(Morning Session)

Part I: Please answer one of the following two questions.

1. According to a recent Wall Street Journal story (Sept. 7, A20), the hottest issue in
the fall 2006 Democratic primary for governor of Massachusetts is income tax
cuts. Two of the three Democratic candidates advocate cutting the state' s flat-rate
individual income tax to 5 percent. In fact, the general inclination towards
proportional taxation has been gaining momentum since the 1970s, with California's
Proposition 13 triggering a domino effect reversing the, progressive taxation
principle that had been on the rise then. T'iseuss
the change in attitudes among Americans in the past century towards progressive
and proportional taxation, as well as the ability -to-pay and benefit principles of
taxation at the state and local levels. Cite important literature on taxation, growth
of the public sector, and public financial administration.
2. In July 2006 the governor of New Jersey fought a battle with the state legislature over
an increase in the state sale s tax and the use of revenue it generated. The
governor wanted to use revenue from the increased sales tax to balance the state's
budget. The legislature wanted to use most of the additional money for property tax
relief. This battle involved almost every aspect of the relationship between the
executive and legislative branches over budgetary institutions such as budgetary
authority, balanced budget requirements, gubernatorial veto powers, legislative
control of the purse, budgetary procedures, budgetary impasses, and so on. The
dispute also reveals changing attitudes among politicians towards revenue
portfolios, i.e., what tax types and tax structures to adopt. Write a comprehensive

780 | P a g e
essay citing important, representative literature, on public budgeting at t he state
level. Your discussion should cover three of the budgetary institutions mentioned
above. Then design a study of any one of those institutions, using both qualitative
and quantitative methods. Do not forget to mention data collection strategies.

(Afternoon Session)

Part II: Please answer one of the following questions.

1. Public budgeting has proved to be a rich source of debate among scholars and
practitioners alike. The purposes, methods, and outcomes have been challenged
repeatedly. As a result, budget reforms are common, and some might say too frequent.
Budget reforms such as planning programming budgeting systems (PPBS), zero-based
budgeting, and performance budgeting were all developed in response to perceived
shortcomings in some other system. They became popular only to become the target of
later reform efforts. Choose one of these reforms and discuss its theoretical origins, its
proponents' critiques of previous methods, and the empirical evidence from the scholarly
literature with respect to outcomes the reform generated.
2. Several states have declared sales tax holidays, in which the state does not collect
sales tax on certain items for a short period of time. In most instances, the holiday has been
for clothing and the period has been a week or ten days in August, the idea being to give
a "back-to-school" discount as families get ready for the new year. Discuss the
implications of these holidays with respect to the broader tax issues of yield, equity,
administration and compliance, and economic impact. Use examples from the literature
to support your analysis.

Part III: Please answer one of the following questions.

1. The line-item or object of expenditure budget has been attacked by academic observers of
the budgetary process. Nevertheless, it survives, in one form or another, internationally
and at all levels of government in the U.S. What is the origin of this budgetary structure?
What are the primary criticisms of it? What factors account for its broad acceptance and
continued use?
2. Local governments rely primarily on property taxes as their primary revenue source. The
administration of this tax is relatively complex because it requires estimation
(assessment) of the relative value of properties. If assessments are not done regularly and

781 | P a g e
accurately, valuation estimate disparities are produced. These valuation disparities create
taxation inequities, that is, the tax burden (tax paid) is not distributed in an equitable
manner. Please describe the way you would compare the tax burden of different
households and discuss the equity criteria you would apply.

Doctoral Examination
Public Management
Fall 2006
(Morning Session)

Part I: Please answer one of the following two questions. You have three hours to
complete your answer.

1. Choose two theories or conceptual frameworks that have relevance for public management
theory and research. Examples include principal-agent models, transaction cost theory,
contingency theory of organizational design, institutionalization theory from organizational
sociology, a theory about a particular topic or concept, such as a motivation theory or a
leadership theory, or other important theories or frameworks that you choose. Explain each
theory and discuss its applicability to public management theory and research. Critique
each theory or framework, explaining ways in which it is valuable or not valuable, and
the theoretical and research questions it helps to answer, or fails to answer.
2. Organization theorists have heavily emphasized the influence of an organization's external
environment on the organization's structure, processes, leadership imperatives, and other
organizational dimensions. They have tended to emphasize such environmental
characteristics as uncertainty, "institutional" influences, and evolutionary
processes. Scholars and experts concerned with public management often contend that
government organizations tend to face distinctive environmental influences, components,
and dynamics. Discuss this difference in perspective on organizational environments.
Why is there a difference between more "generic" organization and management theory,
and public management perspectives? What do the public management scholars point
to as the distinctive environmental characteristics and influences on public organizations,
and what do they say about their effects on organizations and the people in them?

(Afternoon Session)

Part II: Please answer one of the following two questions:

782 | P a g e
1. Leading and motivating employees in government organizations is a major topic in
public management. Describe two theories of leadership, or two theories of
motivation, or one motivation theory and one leadership theory, that can help in the
analysis of this topic. Show how they can help in the analysis. Discuss whether
distinctive characteristics of the public sector need to be considered in applying
the theories.
2. Government undergoes "tides of reform" according to Paul Light, and a sizeable
literature has developed on large scale change or "transformational" change in
government agencies. Write a scholarly essay on governmental administrative
reform, or on large scale organizational change in government agencies, or both.
What factors motivate and influence the initiation of reform and/or change? What
determines the degree of success or failure of such initiatives? How do these topics
fit into the theory and analysis of public management, and what is the role of
public management in relation to these initiatives?

III. Please answer one of the following two questions:

1. What is public management? Is it a genuinely new area of academic inquiry, or is


it merely an old subject that is being dusted off and recycled? Explain carefully
the relationship between the research field of "public administration" and that of
"public management." Is the latter a synonym for the former? Is it a particular kind
of specialization within public administration? Does it designate a coherent
theoretical and/or methodological approach? In answering these questions, be sure
to ground your definition of public management firmly in the literature.
2. Many books on organizational theory and behavior are advertised as collections of
"classics" in the field. For the purpose of understanding public management, select
your own set of classic readings in this field, consisting of five to seven sources.
Explain why you chose these sources, and why they have special relevance and
importance for public management. Identify any common themes that underlie
your sources.

Doctoral Examination
Public Management
Fall 2007
(Morning Session)

783 | P a g e
Part I: Please answer one of the following two questions. You have three hours to
complete your answer.

1. The U. S. Office of Personnel Management regularly conducts the Federal


Human Capital Survey that elicits responses from over 100,000 federal employees
about their personal work attitudes and experiences, their leadership, “performance
culture,” and job satisfaction. The Partnership for Public Service widely
publicizes a list of the “best places to work” in the federal government, based on
the employees’ responses to these surveys in the agencies. The Partnership has
ceremonies recognizing the leaders fro m the agencies designated as best places to
work. Some agency managers report that they receive pressures from superiors
to find ways to improve the scores of their organizational units.

Whether or not you are familiar with these developments, write an essay
discussing the pros and cons of such processes, from the perspective of scholarly
literature and research. What is positive and beneficial about such use of employee
surveys, and how does
oes this relate to developments in the literature of public
administration and organizational behavior? Discuss three important theories or
concepts that should be considered as guides for developing such surveys and the
questions to be included in them. D iscuss drawbacks or cautionary considerations
that using such surveys can warrant.

2. The U. S. Office of Management and Budget has been using the Performance
Assessment Rating Tool (PART) for some years now, and has completed PART
evaluations of over 800 federal programs. According to the OMB website, the
PART includes questions about the following aspects of the program’s
performance:
 The first section of questions asks whether a program's purpose is clear
and whether it is well designed to achieve it
its objectives.
 The second section involves strategic planning,, and weighs whether the
agency establishes valid annual and long -term
term goals for its programs.
 The third section rates the management of an agency’s program, including
financial oversight and prog
program improvement efforts.
 The fourth section of questions focuses on results that programs can report

784 | P a g e
with accuracy and consistency.

The program administrators respond to questions about these criteria, and OMB
representatives make an effort to validate the responses. Whether or not you are
familiar with PART, write an essay evaluating this approach to assessing program
performance from the perspective of the literature on organizational effectiveness.
What approaches to, or theories of organizational effect iveness does PART imply (for
example, the goal model, the internal process model, and/or others)? Discuss the
pros and cons of these theories or approaches. What are their strengths and
limitations? What assumptions about achieving effectiveness do they involve, and
how well-founded are those assumptions? In assessing and achieving program
effectiveness, what is good about the list of criteria mentioned above, and what is
missing or inadequate? Also discuss the implications of the government and
public sector context for such performance assessments. What are the most
serious consequences and implications of the governmental context, for
assessing and achieving performance of a government program?

(Afternoon Session)

Part II: Please answer one of the following two questions:

1. Name three contemporary scholars who are regarded by you as making


important contributions to public management. Explain how their work explicates
key aspects of the subject and what kinds of research should be done to follow up
on this work. Cite key literature as appropriate.
2. Define public management and highlight its linkages and connections across
various disciplinary lines. In particular, explain carefully the relationship
between the research field of “public administration” and that of “public
management.” Is the latter a synonym for the former? Is public management a
particular kind of specialization within public administration? Does the study of
public management designate a coherent t heoretical and/or methodological
approach? In answering these questions, be sure to ground your definition of
public management firmly in the literature.

Part III. Please answer one of the following two questions:

1. According to sociologist Marshall Meyer, t hose empirical researchers who insist

785 | P a g e
that their research has found significant differences between public and private
organizations are, in actuality, showing their incompetence in research design.
Specifically, they are working with misspecified models. The alleged differences
between public and private organizations vanish if one controls for such factors as
the size of the organization, the nature of its resource base, and its function.

Evaluate this argument and the ev idence, both supporting and refuting evidence.
Most important, do not just provide a literature review. Evaluate the technical and
logical merit of the argument as well as its epistemological implications.

2. Privatization of public services and activities ha s been one of the major trends in
governments around the world across the last several decades. Write a scholarly essay
on privatization that includes references to major scholars or experts and to
research findings to the extent possible. Describe the tre nd including its major
features, such as what governments are privatizing and how, and what are the
alternatives and patterns of privatization? Suggest reasons for the privatization
trend, and any evidence that you know about to support these suggestions. Is
there any theory that supports the trend, and if so, what is your assessment of it.
Discuss major issues that privatization rises for public management.

Doctoral Examination
Public Management
Spring 2006
(Morning Session)

Part I: Please answer one of the following two questions. You have three hours
to complete your answer.

1. Many organization and management theorists take a "generic" approach to the


topic, arguing that we need general theory that applies across organizational
settings. They typically reject the usefulness of such distinctions as public
sector vs. private sector and profit vs. nonprofit. Some scholars and experts in
public administration and political science, however, contend that leading
and managing in a government agency has distinctive challenges and
requirements. Discuss this generic orientation, its pros and cons, and evidence
and reasoning for and against it. Then, one of the most frequent claims by

786 | P a g e
those who see public management and leadership as distinctive concerns the
influence of the "political" or governmental context. How should we conceive
of this political, governmental context? How should we define it? What
should it include? What are its most significant characteristics? What are the
most important claims about how this context influences two of the following
activities or characteristics of government organizations: decision-making;
organizational change; organizational design and structure; administrative
discretion and authority; leadership in general; incentives and motivation of
leaders and/or others in the organization. State several hypotheses about the
influence of the political context (and/or governmental, institutional context) on
one or both of the two activities or characteristics that you have chosen.
Describe a research project that would test one or more of these
hypotheses, including details of the research method such as the method of
collecting evidence or data, analytical methods such as statistical method if
applicable, the nature of the sample and unit of analysis, and other important
matters you would cover in a research proposal.

2. In governments at all levels in the U.S. and other nations, the problem of
attracting, keeping, and motivating good employees poses a serious challenge.
A major contemporary concern focuses on the impending retirements of "baby
boomers" in the U.S. and similarly aging managers and employees in other
nations. These developments increase the significance of keeping people
satisfied in their work and committed to it, so that people approaching
retirement will not seek to retire early and so that government agencies can
have the ability to hire replacements for retiring people because the
replacements see the agency as an attractive place to work. Concepts from the
study of organizational behavior and organization theory clearly have relevance
to this matter, including such concepts as organizational commitment, job
design and job involvement, leadership, and work satisfaction. Choose two of
these concepts and review the research and theory pertaining to them. Describe
the most important theories, concepts, ideas and frameworks that figure
importantly in the literature on these topics. Select the most important of
these theories, concepts, or ideas that have useful implications or applications
for the challenges described above. How might they apply to analyzing those

787 | P a g e
challenges (through the types of questions on employee surveys, as one of
many examples), and/or developing policies and approaches to addressing the
challenge? For example, in what sorts of behaviors should leaders and
managers engage? What procedures and policies should organizations adopt?

(Afternoon Session)

Part II: Please answer one of the following two questions:

1. Over the last three decades, an "incentives revolution" has occurred in


management theory - one that emphasizes stock options, pay for performance,
contingent compensation, competitive tournaments, etc. What has public
management contributed to this debate? How have public management scholars
tested specific hypotheses flowing from this body of theory? What does this
theory contribute to our understanding of the short- and long-term performance
of public agencies? Propose a study that would extend our knowledge of
incentives in the public sector; review your Proposed venue of study, research
and experimental design, specific hypotheses, and the pattern of results you
expect to observe.

2. Privatization of public services and activities has been one of the major trends
in governments around the world across the last several decades. Write a
scholarly essay on privatization that includes references to major scholars or
experts and to research findings to the extent possible. Describe the trend
including its major features, such as what governments are privatizing and
how, and what are the alternatives and patterns of privatization? Suggest
reasons for the privatization trend, and any evidence that you know about to
support these suggestions. Is there any theory that supports the trend, and if so,
what is your assessment of it. Discuss major issues that privatization raises
for public management.

III. Please answer one of the following two questions:

1. What are the boundaries of public organizations? Where does public


management begin and end in the study of public sector organizations? What is
the relationship between public management (as a field) and democratic theory?
What is the relationship between public management (as a field) and general

788 | P a g e
management theory? Propose a study that illuminates where public
management begins and ends in the study of a specific organization or set of
organizations. Offer a research and experimental design, and specific testable
hypotheses.

2. Name three contemporary scholars who are regarded by you as making


important contributions to public management. Explain how their work
explicates key aspects of the subject and what kinds of research should be done
to follow up on this work. Cite key literature as appropriate.

Doctoral Examination
Public Management
Spring 2007
(Morning Session)

Part I: Please answer one of the following two questions. You have three hours to
complete your answer.

1. According to sociologist Marshall Meyer, those empirical researchers who


insist that their research has found significant differences between public and
private organizations are, in actuality, showing their incompetence in research
design. Specifically, they
hey are working with misspecified models. The alleged
differences between public and private organizations vanish if one controls for
such factors as the size of the organization, the nature of its resource base,
and its function.

Evaluate this argument an


andd the evidence, both supporting and refuting
evidence. Most important, do not just provide a literature review. Evaluate the
technical and logical merit of the argument as well as its epistemological
implications.

2. The Public Management Research Association , of which many members of the


UGA PADP faculty are members, originated in the 1 990s when Barry Bozeman
sponsored and organized the first National Public Management Research
Conference (NPMRC) at Syracuse University. The NPMRC was initiated as an

789 | P a g e
outlet for research on “public management,” that many of those researchers saw
as distinct from more traditional public administration. Whether or not you are
familiar with the history of NPMRC, discuss the distinction between “public
management” and “public admin istration.” Why might the NPMRC participants
see such a distinction, and what form might the distinction take? What topics,
types of research and/or scholarship, and theoretical bases might differ
between the two topics? Cite important scholarly works and authors to
support your observations and conclusions.

(Afternoon Session)

Part II: Please answer one of the following two questions:

1. Discuss this assertion: “The literature on leadership in organizations


contains a substantial number of cause -and-effect statements that can be
reasonably acted upon to enhance effective leadership of people in
organizations.” If you agree with the statement, justify your position with a
review of the relevant research and theory, and i ts applicability. If you
disagree, justify your position with a specific analysis and critique of existing
research and theory. Explain all answers, taking into account the relevant
literature.

2. Many proposals for improving and reforming public management i nvolve


applications of a market model to the design of public services and programs.
Reformers call for more privatization, contracting out, user fees, and other
arrangements that assume that government should use market -type
mechanisms and arrangements to improve performance. Write a scholarly
essay on the nature of the market model and proposals for it, and on its
strengths and weaknesses, including consideration of its theoretical and
research basis. What are the various applications and versions of the model
that theorists and reformers propose? Assess what we know about their value
and effectiveness. Cite appropriate research and theory.

Part III. Please answer one of the following two questions:

1. Write a scholarly essay in which you analyze the nature of the political
environment of public management and its implications for leading and

790 | P a g e
managing. What are the main elements or components of this environment?
What are their sources of influence on public
blic organizations and managers?
Discuss at least two major issues concerning their influences on or relations
with public managers
managers—accountability, discretion, responsiveness, or other
issues. Discuss the implications of the “political” environment for an
important managerial responsibility, such as leading organizational change,
motivating employees, establishing strategy, decision -making,
making, or another
topic. Cite relevant academic literature and research, including leading
authors on the topics you choose t o cover.

2. Describe a research project you would like to undertake to advance the field of
public management. It can be related to what you have discussed in preceding
questions, but it needs to extend any responses you have made to other
questions. Describe the conceptualization and theoretical import of the
project, the design and methods you would use, possible data sources, likely
results, and other major points. Explain clearly the significance of the project
in the development of the field, citing the literature and other developments
in the field that are relevant to the importance of your project.

Doctoral Examination
Public Management
Spring 2008
(Morning Session)

Part I: Please answer one of the following two questions. You have three hours to
complete your answer.

1. Identify three research questions that are now of great importance t o the field of
public management, and explain why they are important. Sketch the
important research literature that has developed on each, and summarize the
current state of the available findings. Select one of these questions and
indicate what kinds of additional work are needed to help clarify the answers
to the question.

791 | P a g e
2. Choose two theories or conceptual frameworks that have relevance for
public management theory and research. Examples include principal-agent
models, transaction cost theory, contingency theory of organizational design,
institutionalization theory from organizational sociology, a theory about a
particular topic or concept, such as a motivation theory or a leadership
theory, or other important theories or frameworks that you choose. Explain
each theory and discuss its applicability to public management theory and
research. Critique each theory or framework, explaining ways in which it is
valuable or not valuable, and the th eoretical and research questions it helps to
answer, or fails to answer.

(Afternoon Session)

Part II: Please answer one of the following two questions:

1. Consider the following quote: “Research on ‘red tape’ is much ado about
nothing. In the first place, red tape researchers have simply discovered
formalism, a topic that sociologists have been dealing with since Weber. In
the second place, the research has not been cumulative and has not been
linked to theory. It is really nothing more than a tribute to the power of blind
empiricism.” In assessing the quote, be sure to show intimate familiarity
with theory, research, instrumentation, and substantive findings.

2. What is public management? Is it a genuinely new area of academic inquiry,


or is it merely an old subject that is being dusted off and recycled? Explain
carefully the relationship between the research field of “public
administration” and that of “public management.” Is the latter a synonym
for the former? Is it a particular kind of specialization within public
administration? Does it designate a coherent theoretical and/or
methodological approach? In answering these questions, be sure to ground
your definition of public management firmly in the literature.

Part III. Please answer one of the following two questions:

1. Organization theorists have heavily emphasized the influence of an


organization’s external environment on the organization’s structure,
processes, leadership imperatives, and other organizational dimensions. They

792 | P a g e
have tended to emphasize such environmental characteristics as uncertainty,
“institutional” influences, and evolutionary processes. Scholars and experts
concerned with public management often contend that government
organizations tend to face distinctive environmental influ ences, components,
and dynamics. Discuss this difference in perspective on organizational
environments. Why is there a difference between more “generic” organization
and management theory, and public management perspectives? What do the
public management scholars point to as the distinctive environmental
characteristics and influences on public organizations, and what do they say
about their effects on organizations and the people in them?

2. Many books on organizational theory and behavior are advertised as


collections of “classics” in the field. For the purpose of understanding public
management, select your own set of classic readings in this field, consisting of
five to seven sources. Explain why you chose these sources, and why they have
special relevance and importance for public management. Identify any common
themes that underlie your sources.

Doctoral Examination
Public Policy
Fall 2006
(Morning Session)

Part I: Please answer one of the following two questions. You have three hours to
complete your answer.

Question 1:

Policy analysts have debated whether there really exists an "efficiency -equity tradeoff'
associated with public policies and programs. Explain what is meant by the
efficiency- equity tradeoff. Discuss the standard economic viewpoint (as expressed by
Weimer and Vining) on the existence of the tradeoff and contrast it with the views
of political theorists such as Stone. Give an example of a public program that has
the potential to increase both efficiency and equity.

Question 2:

793 | P a g e
Numerous models have been proposed to help us understand the policy -making
process. These include, but are not limited to, systems models, incrementalism, policy
streams, the advocacy coalition framework, and punctuated equilibrium. Develop an
essay in which you discuss each of these five approaches. What are the major tenets
of each? How does each approach help us to understand public policy making' What
research questions do they lead us to ask? Which approach or model do you believe
is most useful and why? Please reference all appropriate literature.

(Afternoon Session)

Part II: Please answer one of the following questions.

Question 1:

Given the emergence of complex, networked patterns for policy imp lementation,
what are the implications for theory development in policy implementation? In
particular, what should one conclude about the relative utility and validity of top -
down, bottom-up, and other approaches to empirical theory regarding policy
implementation?

Question 2:

Discuss the economic arguments for and against contracting out government
services. Under what general conditions is contracting out likely to be an efficient
means of governing production relationships? Address the issues of contrac t
bargaining and monitoring costs in the context of the principal -agent problem. Cite
specific examples of government services where contracting -out has been relatively
successful and cases where it has been unsuccessful. Be sure to discuss the
reasons for success (or lack thereof) in light of the general conditions that facilitate
efficient contracting-out.

Part III: Please answer one of the following questions.

Question 1:

The State Children's Health Insurance Program (SCHIP) is aimed at improving health
insurance coverage for low -income children. At its inception, states were given the
freedom to design their own SCHIP programs. Over time, states either created

794 | P a g e
standalone programs or incorporated SCHIP into existing state Medicaid programs.
Since then policy analysts have been concerned with whether SCHIP has improved the
health status, access to health care, and health care utilization of children.
Describe a research desi gn to test the effectiveness of SCRIP in improving child
health. Identify at least two challenges/problems faced by any research designed
to assess the effectiveness of SCHIP. Does your research design overcome these
problems? If yes, explain how. If not, why not?

Question 2:

The most recent government data (March 2006 Current Population Survey) indicates
that the United States has 46.6 million residents who do not have health insurance
from any source. Discuss (at least) two competing theories about equit y/fairness.
Analyze the current distribution of health insurance coverage in light of these
competing theories. Discuss both the strengths and weaknesses of your equity
theories. Briefly discuss how universal health insurance coverage (provided by the
government) would change your analysis.

Doctoral Examination
Public Policy
Fall 2007

(Morning Session)

Part I: Please answer one of the following two questions. You have three hours to
complete your answer.

Question 1:

Compare and contrast efficiency -based theories of government intervention in market


activity to equity-based theories of intervention. Be sure to cite the literature. What
do these theories imply about when it is best to let market forces simply take their
“natural” course? How does existing empirical evidence about government
performance inform the decision to intervene? When are the efficiency and equity
theories in agreement? Give examples. Critique each theory and discuss the policy
implications of each critique.

795 | P a g e
Question 2:

Numerous models have been proposed to help us understand the policy -making
process. These include, but are not limited to, systems models, incrementalism, policy
streams, the advocacy coalition framework, and punctuated equilibrium. Develop an
essay in which you discuss each of these five approaches. What are the major tenets
of each? How does each approach help us to understand public policy making? What
research questions do they lead us to ask? Which approach or model do you belie ve
is most useful and why? Please reference all appropriate literature.

(Afternoon Session)

Part II: Please answer one of the following questions.

Question 1:

Complete information is a requirement for achieving efficiency in perfectly competitive


markets. In cases where actors have a preference for discrimination, complete
information may lead to increased discrimination. For example, real estate agents often
recommend that African-Americans
Americans hide family photos when they are selling their
homes. This
s recommendation to withhold complete information is an effort to
eliminate buyer discrimination.

1. Discuss what is meant by complete information.

2. In the context of the example above, how would individual behaviors and market
outcomes differ when there is complete information from when there is
incomplete information?

3. How might market forces correct these discriminatory outcomes?

4. Develop a policy that preserves access to information and limits discriminatory


behavior.

5. What criteria would you use to evaluate policies aimed at preserving access to
information and limiting discriminatory behavior?

Question 2:

796 | P a g e
What are policy implementation networks? Trace their emergence in both the academic
literature and world of practi ce. Explain how such networks strain traditional
conceptions of public management and organizational theory, and explore their
implications for such age --old
old public administrative concerns as the politics -
administration dichotomy, the blurring of the publi c and private sectors, and the quest
for bureaucratic accountability in democratic government. In the final analysis, how
can networks be managed more effectively, and how can policy be formulated and
implemented more effectively in networked settings?

Part III: Please answer one of the following questions.

Question 1:

Describe the purpose, content, and impact of symbol manipulation in policy


formulation and adoption. Offer empirical evidence and support from theory.
Construct a research design that investigates the value and impact of symbols in
policy.

Question 2:

Charter schools are “publicly funded elementary or secondary schools which have
been freed from some of the rules, regulations, and statutes that apply to other
public schools, in exchange
e for some type of accountability for producing certain
results, which are set forth in each school's charter.” Discuss the efficiency and
equity implications of education policies that allow charter schools to operate
alongside traditional public schools. What are some potential problems with the
public funding of these schools? What other political goals (aside from efficiency and
equity) should be taken into account in policy discussions concerning charter schools?
How would you design an evaluation to test
est the effects of charter schools on
educational outcomes? What are some of the outcome variables you would use in
this type of evaluation?

Doctoral Examination
Public Policy
Spring 2006

(Morning Session)

797 | P a g e
Part I: Please answer one of the following two questions. You have three hours
to complete your answer.

Question 1:

An Individual Training Account (ITA) is essentially a voucher given by government to


qualified individuals who need occupational skills training to become gainfully
employed or re-employed. The ITA program is implemented by Workforce
Investment Boards (WIB) established by federal/state partnerships.

An ITA is provided after an assessment of individual needs, demand for labor in


selected occupations, and local WIB policies. Depending upon local policy, the ITA
may be approved by number of people within the WIB: a individual's career
advisor, a committee, a representative of the local board, or other persons. This
process will reflect what is determined locally to provide the appropriate balance
between accountability for training funds and effective customer service.

Individuals may use their ITAs to purchase training slots in any program on the
eligible program/provider list. Local Workforce Investment Boards also establish
monetary and time limits for an ITA.

Drawing on the relevant literature answer two of the following:

a. What are the main challenges to implementing the Individual


Training Accounts policy and how might they be overcome?

b. Identify and discuss an effective design for evaluating the


success/failure for the ITA program.

c. Provide an analysis of the likely individual and labor market effects of


the ITA policy.

Question 2:

Market failure, primarily public goods and externalities, as a justification for


government intervention have been around since Musgrave (1959). Since then,
considerable theoretical and empirical work has expanded the justifications for
intervention. One of the most important is the role of asymmetric information in the
design of government policy. Discuss both the import and the impact of asymmetric

798 | P a g e
information on policy development. To what extent have government policies been
reformulated in an effort to overcome information asymmetries? How have these
design changes allowed the government to get citizens to reveal information to
alleviate the problem?

(Afternoon Session)

Part II: Please answer one of the following questions.

Question I :

Much of the debate in public policy centers on trading efficiency for other desirable
policy outcomes. Most economic models focus on the loss of efficiency from government
intervention, while political and policy models may focus more attention on equity,
fairness and justice. Clearly and succinctly define the terms efficiency, equity,
fairness and justice. Must these criteria be traded off against each other? If so,
provide a

relevant example of the tradeoff and analyze the process under which the trade off
was made. If not, under what conditions may we achieve more of all of these things?
Discuss whether the principle is generalizable. Please provide relevant examples.

Question 2:

Questions about equal treatment in the field of education arise not only with
respect to race, but also with respect to other matters. Of particular importance
are issues dealing with school financing, language, handicap, and the number of
years of schooling provided. Choose one of the substantive areas above and
discuss the policy tools available to policy-makers to achieve "equal treatment."
Be sure to define what you mean by equal treatment. Are the "tools" different under
different definitions of equal treatment.

Part III: Please answer one of the following questions.

Question I :

In their 2002 volume Implementing Public Policy, Michael Hill and Peter Hupe
remark that policy "implementation was and continues to be a concern of many

799 | P a g e
writers who do not talk about 'implementation' per se, and indeed may approach it
from very different backgrounds to the public administration specialists who do so."
Are they correct? What kinds of research can and should be relevant to those
interested in policy implementation? What kinds of theory building holds the most
promise for a general understanding of policy implementation?

Question 2:

The private sector provision of what has been traditionally provided by the
government has become increasingly common. Contracting to the private sector is
seen by some analysts as a panacea for bureaucratic inefficiency. Discuss the
problems that typically arise when bureaucratic supply is the sole form of
production. Provide a detailed exposition of the sources of inefficiency. Describe
the rationale as to why "contracting out" may fix these problems, and describe
ways in which the efficiency theory of the private sector may not hold.

Doctoral Examination
Public Policy
Spring 2007

(Morning Session)

Part I: Please answer one of the following two questions. You have three hours to
complete your answer.

Question 1:

Policy theorists have long debated whether there really exists an “efficiency -equity
tradeoff” with respect to market outcomes and the impact of social interventions.
Summarize the opposing views. In particular, explain the idea of the efficiency -equity
tradeoff and what factors are responsible for it according to the standard neoclassical
economic view. Discuss criticisms of the economic viewpoint, especially in view of the
limitations of the efficiency concept and/or the potential for public interventions to
increase both efficiency and equity in some cases. Under what circumstances might it
be possible for social scientists to reach a stronger consensus on the relevance of
the efficiency-equity tradeoff?

800 | P a g e
Question 2:

Numerous models have been proposed to help us understand the policy -making
process. These include, but are not limited to, s ystems models, incrementalism, policy
streams, the advocacy coalition framework, and punctuated equilibrium. Develop an
essay in which you discuss each of these five approaches. What are the major tenets
of each? How does each approach help us to understan d public policy making? What
research questions do they lead us to ask? Which approach or model do you believe
is most useful and why? Please reference all appropriate literature.

(Afternoon Session)

Part II: Please answer one of the following questions.

Question 1:

As a policy advisor you have been commissioned to evaluate the distribution of


income in a country. You are told the following:

(a) In 1990, mean income = median income.

(b) In 2000, mean income > median income.

Note that (a) suggests that the income distribution is likely symmetric, whereas (b)
points to inequality in the income distribution (so that either the rich are getting
richer, or there are a few individuals with high levels of income). In order to make
unambiguous conclusions about the income distributions in 1990 and 2000 is this
information sufficient? If not, what additional statistics would you request? How
would you use those additional statistics? Explain clearly. What are the policy
implications of this study?

Question 2:

Scholars
holars have written of the "cases/variables" problem in implementation research.
What is this problem? Is it a real issue or a trivial concern? If significant, why is
it a challenge? If not, why not? Give examples from the research literature, and be
specific
fic in your critiques. Give your suggestions about how the problem can be
handled, or at least addressed, in empirical research.

801 | P a g e
Part III: Please answer one of the following questions.

Question 1:

The mayor of a remote village asked a group of researchers to study whether a new
public health clinic (the only one in the village) was effective in improving the health
of its citizens. The researchers surveyed a simple random sample of villagers after
the health clinic has been operating for 12 months. The data yielded mean LDL (bad
cholesterol) levels for the villagers who had gone to the clinic (group A) and for the
villagers who never visited the clinic (group B). The researchers found that mean
LDL for
or group A was substantially higher than mean LDL for group B. They thus
concluded that the clinic was bad for heart health, and recommended to the mayor
that the clinic be shut down immediately. Develop an essay in which you evaluate
the validity of the researchers’
esearchers’ conclusions. Describe an alternate research design
that could have been used to study the effectiveness of the health clinic. Discuss
advantages and disadvantages of your design relative to the one used by the
aforementioned researchers.

Question 2:

Choose a specific policy issue or area and justify (using relevant citations in the
literature) a particular analytic approach to informing public decision makers
about policy alternatives. How does your analytic approach incorporate issues of
economic valuation, risk, and impact, as well as "normative" issues. What criticisms are
there of the analytic approach you chose? How are the shortcomings of your analytic
approach likely to bias the assessment of alternatives and policy recommendations?
universal health insurance coverage (provided by the government) would change your
analysis.

Doctoral Examination
Public Policy
Spring 2008
(Morning Session)

Part I: Please answer one


ne of the following two questions. You have three hours to
complete your answer.

802 | P a g e
Question 1:

Within the broad literature on “innovation,” most of which is rooted in sociology and
economics and focuses on technological innovation, there is the much smaller
literature on “policy innovation.” What, if anything, does the policy innovation
literature provide that is not already included in the generic innovation literature.
What are the predominant theories of policy innovation? What are the most
important empirical findings of the policy innovation literature? How is “policy
innovation” different from “policy invention” and “policy change?”

Question 2:

How would you define the field of public policy? What is the substance of the field as
an area of academic interest? What is the purpose of systemic study in this field?
How is policy as a field distinct from the study of politics or public administration, or
are there any meaningful dist
distinctions?
inctions? What do you see as some of the major
contributions that have come from the study of public policy? What is your
assessment of the field?

(Afternoon Session)

Part II: Please answer one of the following questions.

Question 1:

What is the utility of models that depict policymaking as a cycle? Alternatively, what
is the utility of models that depict policymaking as the confluence of a set of streams?
Has one of these approaches been more valuable for generating research that is
theoretically
heoretically grounded and substantively relevant than the other? If so, identify which
one and explain why. If not, explain why not?

Question 2:

In the U.S. it is currently illegal for individuals to sell one of their kidneys; they may
donate a kidney to someone they choose but cannot receive compensation. Patients
who require a kidney transplant are placed on a waitlist for a deceased donor’s
organ. However, there is a shortage of kidneys and many patients die while waiting for
a donor. One solution to this
is problem is to legalize the individual sale of kidneys.

803 | P a g e
Present an economic evaluation of that proposed solution. Be sure to highlight both
the pros and cons of this policy. Explain any equity concerns this policy may raise.
Suggest an alternative solution for improving the shortage in the U.S. kidney
market. How do you expect your suggested policy to reduce the shortage in the
kidney market? Compare the effect on the kidney market of your suggested policy to
the one described above. Is one policy clearly superior to the other? Explain.

Part III: Please answer one of the following questions.

Question 1:

The Bush administration has proposed “cap and trade” permits for mercury (a
poisonous heavy metal) emissions from coal -fired power plants. Because mercury is
heavy, it accumulates near the plants producing the emissions. Assume the harm
to individuals near these plants can be offset by the benefits to those further away.
Under this assumption, to what extent does the administration’s proposal pose an
ethical dilemma? What happens if people living near the plants actually are
compensated? Does this change the nature of the dilemma? Use this example to
summarize the arguments against using a utilitarian ethic in policy analysis.
Discuss how cost-benefit analysis embodies the utilitarian ethic. How might you
alter cost/benefit analysis to address these concerns?

Question 2:

In recent years, policy implementation as a research field has seemingly been in


decline. Assess the validity of this statement. What specific steps are needed to move
the field of policy implementation research forward?

Doctoral Examination
General Public Administration
Fall 2005
Morning Session

Part I: Please answer one of the following two questions. You have three
hours to complete your answer.

1. The growth of government has led many observers to worry that government

804 | P a g e
bureaucracies lie outside the reach of democratic accountability. Given the
existence and effects of bureaucratic discretion, how can we be sure that
bureaucrats will act responsively and be held accountable? Drawing from the
classic and contemporary literatures, explain how various scholars have tried to
reconcile the need for bureaucratic discretion with the imperative for
bureaucratic responsiveness and accountability. W hich the various approaches
to achieving bureaucratic responsibility and accountability seems most promising
to you, and why?

2. Identify three “big questions” in Public Administration that should – and


probably will – receive sustained attention from the research community over
the next decade. Explain why each question is of sufficient gravity to be on this
list, assess the current state of knowledge on the question, and identify some
promising directions for future re search.

Afternoon Session

Part II: Please answer one of the following questions.

1. Identify two basic lines of theorizing that have contributed fundamentally to the
field of public administration during the last 25 year. Assess the principal strengths,
weaknesses, and potential of each; identify some of the principal theorists associated
with each; and indicate what kinds of issues facing the field can best be addressed by
building on these types of theory.

2. It has been argued that public administration does n ot operate in a vacuum but is
deeply influenced by its social, political, and economic environment. Thus, the
practice and study of public administration evolve with the changing landscape or
“ecology” of government. Drawing from relevant literature, trace the theoretical
evolution of public administration and analyze important ways in which its
environment or ecology have influenced that evolution.

Part III: Please answer one of the following questions.

1. Some specialists in public administration have been a rguing in recent years


that some of the core issues in the field are best reframed as matters of
“governance” rather than “public administration.” Others are prone to comment

805 | P a g e
that “governance” involves more than “governments.” What, exactly, is going on
here in terms of conceptual and theoretical emphases? Do the proponents of
the study of governance have a point? Are they missing important points?
Please cite relevant literature in your response.

2. Wallace Sayre once wrote that public and private administrat ion are alike in all
unimportant respects. What is meant by such a statement? Is it accurate? How do
you assess the claim that administration is a generic subject that can and
should be studied and generalized about in a broad sense rather than sector by
sector? Refer to the research literature in supporting your answer.

Doctoral Examination
General Public Administration
Fall 2006

Morning Session

Part I: Please answer one of the following two questions.

1. In an article published in the Public Administration Review in 1947 Robert Dahl


assessed the state of the field of public administration, particularly in terms of the
prospect of a true science of public administration. He argued that the field then faced
"three problems" which would have to be addressed before public administration could
merit scientific status. These were: (1) clarifying the place of normative values in the
field; (2) developing a better understanding of human nature in administrative settings,
thus providing better predictability of behavior; and (3) generating a sophisticated
comparative study of public administration. On the basis of these criteria (and, if
appropriate, any others that you care to add), how far has public administration come
toward the goal of achieving truly scientific stature? Clarify and defend your
assessment with reference to your knowledge of the research literature of the field.

2. In 1990, the public management literature, as represented in the major U.S public
administration and policy journals, included but a single empirical study of
"organizational red tape." This single empirical paper (Buchanan, 1975)
operationalized red tape as questionnaire respondents' views about the amount and
rigidity of organizational structures. In other words, this early study measured red tape
as a perceptual variable not much different from subjective assessments of the more

806 | P a g e
venerable and well-researched organizational formalization construct.

Since 1990, more than forty papers have been published dealing with one or another
red tape construct. In considering this literature, answer the following questions:

a. What is the prevailing concept(s) of red tape and what are the most common
approaches to measurement and operationalization?

b. What research methods have been used (and should be used) to understand
organizational red tape?

c. What research-based propositions about red tape seem to have the most
convergent validity?

d. As a result of work on red tape, do we now know anything about organizations


not already covered in the literature on organizational structure and
formalization?

In each case provide criticisms of the research literature and, when possible, its
correspondence to the theoretical literature.

Afternoon Session

Part II: Please answer one of the following questions.

1. Acme University Press has commissioned you to write an introductory -level textbook
on Public Administration that can be used to teach graduate courses in the field.
The purpose of this book is to survey the major currents in the field and provide a
solid foundation for beginning students, some of whom have prior coursework
and/or work experience in Public Administration. Sketch an introductory
chapter that includes discussion of the major theoretical perspectives and
important works that will be covered. Also, provide an outline showing the
chapters included in the book. In completing these tasks, take into account the
interdisciplinary nature of the field, attempt to strike a proper balance between
theory and practice, and try to make the book appeal to the diverse range of
students who are now pursuing graduation education in PA.

2. The field of Public Administration and several cl osely related disciplines (Political
Science, Political Economy, etc.) have long been concerned about the existence

807 | P a g e
and effects of bureaucratic discretion in democratic government, and the apparent
difficulty of ensuring that bureaucrats — who are non-elected public officials — will
act responsibly and be held accountable. Drawing from the classic and contemporary
literatures, explain how various scholars have tried to reconcile this problem of
bureaucratic discretion, responsibility, and accountability. Wh ich of these
approaches seems most promising to you?

Part III: Please answer one of the following questions.

1. At the recent American Political Science conference Dr. Kenneth J. Meier gave a
lecture entitled "The Public Administration of Politics or What Pol itical Science
Could Learn From Public Administration." His lecture suggested ways in which
political science as a discipline could benefit from lenses and methods used in
public administration. His discussion raised several questions about public
administration as a discipline. First, what are the key questions in the discipline?
How do these questions distinguish public administration from other disciplines?
Finally, take the question of bureaucratic control, what could other disciplines learn
from public administration's study of bureaucratic control? Pen an essay that
addresses each of these questions. Be sure to provide clear examples from the
literature of public administration.

2. The subject of democratic accountability is one of the defining topics of the study
of public administration. Discuss the tensions between democratic accountability
and other specific substantive values. That is, what do we trade off when we pursue
democratic accountability and, conversely, what do we lose when we emphasize it in
administration and policy? In your answer, discuss both the intellectual history of
the debate and its current major issues. Ground your answer in the literature on
the topic.

Doctoral Examination
General Public Administration
Fall 2007

Morning Session

Part I: Please answer one of the following two questions.

808 | P a g e
1. Please identify what you consider the single most “promising” body of public
administration research to have emerged in the p ast 10 years. For, present
purposes, let us define “promising” as encompassing at least the following
criteria: (1) dealing with issues central to public administration; (2) providing
strong evidence and/or compelling theoretical explanation of public
administration phenomena; (3) influencing public administration scholars to
such an extent that many have begun to take up the topic (i.e. no one-person
research agendas qualify, no matter how brilliant). If possible, pick a topic that
is not the signature research of faculty members in our department. In your
response, be sure to tell us exactly why this body of work is so promising,
what it has achieved, what it has yet to achieve and how it can be improved
upon. Naturally, you should identify and briefly discuss the articles or books
that best illustrate this promising body of work.

2. In an article published in the Public Administration Review in 1947, Robert


Dahl assessed the state of the field of public administration, particularly in
terms of the prospect of a true science of public administration. He argued
that the field then faced "three problems" which would have to be addressed
before public administration could merit scientific status. These were: (1)
clarifying the place of normative values in the field; (2) developing a better
understanding of human nature in administrative se ttings, thus providing
better predictability of behavior; and (3) generating a sophisticated
comparative study of public administration. On the basis of these criteria
(and, if appropriate, any others that you care to add), how far has public
administration come toward the goal of achieving truly scientific stature?
Clarify and defend your assessment with reference to your knowledge of the
research literature of the field.

Afternoon Session

Part II: Please answer one of the following questions.

1. At the recent meeting of the American Political Science Association, Professor


Larry Lynn presented a lecture entitled, “New Frontiers of Public
Administration: The Practice of Theory and the Theory of Practice.” In the

809 | P a g e
lecture he suggested that the study of public administration
ministration could gain
significantly from the field of economics. Do you agree or disagree with his
assessment? In general, in what ways might one argue that the study of public
organizations and policy can be informed by economics? Can you think of
any published work associated with public administration that has been
influenced by the academic perspective of economics? Are there questions
previously examined in public administration that could be better addressed if
they were reexamined through
ough the lens of economics? Pick specific examples
and support your views with relevant literature.

2. Explain individually, and compare and contrast, the following concepts as


they relate to both the analysis and reform of the public sector: (a)
Governance, (b) New Public Management, and (c) Networks.

Part III: Please answer one of the following questions.

1. Trace the theoretical approaches to controlling the public bureaucracy, and


provide an example highlighting the present day challenges associated with
administrative responsibility and bureaucratic accountability. What are the
implications of this issue for public administration scholars and practitioners?

2. Woodrow Wilson noted, “It is the objective of administrative study to


discover, first, what government can properly and successfully do, and
secondly, how it can do these proper things with the utmost possible efficiency
and at the least possible cost either of money or energy. On both these points
there is obviously much need for light amon g us; and only careful study can
supply that light.” Reflect upon this quote, and trace the theoretical evolution
of the study of administration. Highlight the important theorists, theories, and
bodies of work. What implications does this evolution offer i n terms of future
public administration research as well as pedagogical focus?

810 | P a g e
Doctoral Examination
General Public Administration
Spring 2005

Morning Session

Part I: Please answer one of the following two questions.

1. Some years ago a scholar named Richard Elmore criticized the field of public
administration by referring to it as Aa collection of discrete and unrelated
subjects in search of an intellectual focus, preoccupied with institutional
description rather than analysis, and lacking in sufficient intellectual rigor to
command the respect of other academic disciplines or the public at large. @ Write
an essay in which you assess the accuracy of Elmore =s critique, carefully
considering the state of scholarship and kno wledge in public administration. Agree
or disagree with Elmore=s position, explaining your own position with a careful
review of the most important theories, intellectual positions, research streams, or
other important aspects of the field of public administration. Show how these
developments in the field rebut Elmore or support him. Whether you agree or
disagree with Elmore, what steps should scholars in public administration take
at this point to improve the field?

2. One of the classic issues for public adm inistration and public bureaucracy has
concerned how proactive a public administrator should be in orchestrating policy
change, as opposed to waiting for guidance from elected officials. Write a
scholarly essay on this question. To what extent should public administrators be
proactive, and what does the evidence indicate about whether they actually are
proactive? Cite and review major scholars, scholarly groups, or theories,
concerning these questions (possibly including, for example, public choice,
Blacksburg Manifesto, New PA, Friedrich and Finer, New Public Management,
literature on leadership in the public sector, or many others). In your essay,
consider the issues for democratic governance that relate to this question of how
proactive a public administrator should be.

Afternoon Session

Part II: Please answer one of the following questions.

811 | P a g e
1. We read and hear that public administrators should be held accountable. What
does being held accountable mean? How is this accomplished in American
governments? How well does this process of being held accountable work? Please
cite literature as appropriate.

2. Principal-agency theory centers on the twin dilemmas of adverse selection and


moral hazard. How have public organizations sought to solve these dilemmas? How
is it different from (or similar to) how businesses have sought to solve these
problems? Cite relevant literature throughout.

Part III: Please answer one of the following questions.

1. The politics/administration dichotomy, which was once dogma for the emerging
discipline of public administration, is not as simple a concept as some of the
earliest scholars in the field may have suggested. What gave rise to the dichotomy
and what brought on subsequent challenges to its validity? Is there a true
dichotomy between politics and administration? What are the most serious
implications of the intermixture of administration and politics? What can you say
about the political dynamics of public adminis tration in a democratic society?

2. Public administration has drawn on several other fields in its history and
development as an academic discipline. Select two fields that you think have
made particularly large contributions to developing the field of public
administration. Explain or justify why you have selected those two fields. Show
how these other fields have affected public administration theory or thinking.
Please cite examples of ideas and authors from these other fields.

Doctoral Examination
General Public Administration
Spring 2006

Morning Session

Part I: Please answer one of the following two questions.

1. The field of Public Administration has been criticized for a lack of theory
building. Discuss whether this claim is valid. Assess the development of theory in
one particular area of public administration research. Identify a research design

812 | P a g e
that will provide a new test for the theory you have identified. Explain important
features of the design, such as the main concepts and variables, the method,
and how it could be carried out (for example, the nature of the sample, the data
collection method, and plans for statistical analysis). Provide justification for why
your research design is worthwhile; in other words, explain how the execution of
your research design will add to the existing knowledge.

2. In many nations in recent decades, governments have sought to develop


increasing "flexibility" in their public administrative systems, such as their
purchasing and procurement systems and their personnel systems. In the U.S.,
for example, the federal government has sought to loosen the rules and
procedures for purchasing and procurement and for personnel procedures such as
hiring and compensation. Write a scholarly essay in which you analyze and assess
the prospects for such efforts at flexibility. What forces and imperatives put existing
rules and procedures in place in the first place? Include a description of scholarly
work on the origins and nature of rules, procedures, and "red tape" in public
administration. What are the impediments to loosening the current rules and
procedures? What theory and research can we draw on, or do we need, to analyze
prospects for more flexibility and the process of moving toward it?

Afternoon Session

Part II: Please answer one of the following questions.

1, Woodrow Wilson and other early thinkers in American public administration


said that administration and politics are distinct. John Gaus, a major contributor
to the field several decades later, said that a theory of public administration meant
at that time a theory of politics also. Indeed, Dwight Waldo endorsed that view
during the 1980s. Which perspective do you regard as more accurate and
insightful? Are both valid? Neither? Support your response by discussing some
recent research in the field.

2. Much of the debate over the legitimacy of administrative agencies centers on the
scope of administrators' discretion. Why is that the case? How can the dilemma
over administrative discretion be explained using principal-agent theory? What
principles or tenets of administrative law are most likely to constrain or expand

813 | P a g e
administrative discretion? Finally, since constrains on administrative discretion may
result in the loss of efficiency, consider what countervailing values exist to justify them.

Part III: Please answer one of the following questions.

1. Insiders often talk about the alleged chasm between Public Administration
scholarship and practice. Some contend the chasm does not exist. Others believe the
chasm exists but serves a good purpose — making scholarship more free-ranging and
productive. Still others believe the chasm exists and is a serious problem — they say
good scholarship should inform the world of practice. What do you think about this
alleged chasm and what is your preferred course of action? For example, depending
on your position, you might explain how to correct widespread misperceptions about
the chasm, or you might outline some ways to bridge the gap between scholarship
and practice. In formulating your answers, draw from the relevant literature.

2. The political scientist Wallace Sayre once remarked, "the public and private
sectors are fundamentally alike in all unimportant respects." What, exactly, did Sayre
and others who have quoted him mean? (For example, consider Graham T. Allison's
well-known and often reprinted paper that carries this title.) Drawing from the
literature and your personal experience, describe any public/private differences that
seem to have significant effects on the study and practice of modern-day Public
Administration. Explain these effects. Assess the evidence that these differences
actually exist and have the effects you describe.

Doctoral Examination
General Public Administration
Spring 2007

Morning Session

Part I: Please answer one of the following two questions.

1. In many nations in recent decades, governments have sought to develop


increasing “flexibility” in their public administrative systems, such as their
purchasing and procurement systems and their personnel systems. In the
U.S., for example, the federal government has sought to loosen the rules and
procedures for purchasing and procurement and for personnel procedures

814 | P a g e
such as hiring and compensation. Write a scholarly essay in which you
analyze and assess the prospects for such efforts at flexibility. What forces
and imperatives put existing rules and procedures in place in the first place?
Include description of scholarly work on the origins and nature of rul es,
procedures, and “red tape” in public administration. What are the
impediments to loosening the current rules and procedures? What theory
and research can we draw on, or do we need, to analyze prospects for more
flexibility and the process of moving tow ard it.

2. Woodrow Wilson’s vision was to foster the study of administration in hopes


of establishing a science of administration. As such, his objective was “to
straighten the paths of government, to make its business less un -businesslike,
to strengthen and purify its organization, and to crown its duties with
dutifulness.” Reflecting upon this quote, trace the theoretical evolution of the
study of public administration and highlight what you see as major
developments. What issues must scholars address if pub lic administration is
to become a science? In your view, has Wilson’s objective of developing a
science of public administration been achieved? Please note important
theorists, theories, and bodies of work to support your response.

Afternoon Session

Part II: Please answer one of the following questions.

1. The politics-administration dichotomy is among the most discussed


elements of canonical public administration. Explain it, its significance as a
normative ideal, and its insufficiencies as a positive theory. Drawing from the
literature, construct, discuss, and defend an alternative to the politics -
administration dichotomy that has both normative and positive appeal.
Describe a method of testing your theory in its positive sense.

2. The political scientist Wallace Sayre once remarked, “the public and private
sectors are fundamentally alike in all unimportant respects.” What, exactly,
did Sayre, and others who have quoted him, mean? (For example, consider
Graham T. Allison’s well-known and often reprinted paper that carries this
title.) Drawing from the literature on empirical comparisons of public and

815 | P a g e
private organizations, and from your personal experience, describe any
public/private differences that seem to have significant effects on the study
and practice of modern
modern-day
day Public Administration. Explain these effects.

Part III: Please answer one o


of the following questions.

1. Democracies and other forms of government have extensive bureaucracies.


How are bureaucracies in a democracy similar to and different from
bureaucracies in governments that are not democracies? Do bureaucracies face
any special challenges
hallenges in democratic and non-democratic
democratic forms of
government? If so, what are they? Please cite appropriate literature in your
answer.

2. Identify three lines of theoretical and empirical work currently under active
and productive research in public adminis tration. What research questions do
they focus on? What do the findings suggest? What issues need to be explored
for this work to have significant impact on the field? Cite and use relevant
literature.

Doctoral Examination
General Public Administration
Spring 2008

Morning Session

Part I: Please answer one of the following two questions.

1. Wallace Sayre once wrote that public and private administration are alike
in all unimportant respects. What do you think Sayre meant by that
statement? Is the statement accurate? How do you assess the claim that
administration is a generic subject that can and should be studied and
generalized about in a broad sense rather than sector by sector? What does
the research literature say on this issue.

2. Writing just four


our years after the end of World War II, Herbert Simon, Donald
Smithburg and Victor Thompson in their path -breaking
breaking textbook Public
Administration explained the origins of government agencies and the
expansion of U.S. federal government executive branch as follows:

816 | P a g e
“It is not surprising the new government agencies... mostly

originate in periods of government expansion. Wars,


depressions, and similar crises create new problems
requiring governmental solutions and temporarily weaken
the forces opposing broade r government programs.” (p.
37)

Does this statement provide a valid explanation for contemporary initiation,


growth, and contraction dynamics of U.S. federal government agencies? In
what respects, if any, does it remain apt? In what ways, if any, does it fail to
cover contemporary policy and management realities? To what extent does it
conform to the analyses of more recent scholars who have studied the growth
dynamics of government (and, if relevant, non - government) organizations?
Compare and contrast specific theories and theorists.

Afternoon Session

Part II: Please answer one of the following questions.

1. Discuss the concept of bureaucracy as an organizational strategy. What are


the major characteristics of bureaucracy? What would you say are the
strengths or advantages of bureaucratic organizations? What disadvantages
exist? What alternative organizational models have been proposed, and who
are the scholars that have been most closely identified with those
alternatives?

2. Much of the history of the field of public administration has centered on the
question of how to hold unelected public officials accountable for their actions
and insure democratic values. Briefly trace the history of the discipline of publi c
administration and explain how the question of accountability helped shape
that history? Given the history of the field, discuss the big questions that will
likely shape the near future of the discipline. What theories and methods will
be important to answering those questions?

Part III: Please answer one of the following questions.

817 | P a g e
1. Over the years, various scholars have argued that public administration is --
or should be -- variously, a discipline, an interdisciplinary field, a
multidisciplinary field, or a profession. From your perspective, which of these
is public administration as a research field? Which should it be? Why?
Define your terms and cite relevant research literature.

2. Privatization, also referred to as “out -sourcing”, and “contracting -out” has


frequently been touted as a mechanism to improve the delivery of
government services. The concept has been promoted heavily by politicians,
scholars, and practitioners, and while it may have benefits, some observers
note difficulties associated with it. Substantial problems have been
documented, for example, in the past few years during the on -going war in
Iraq where the contractors have often failed to meet their obligations. Develop
an essay in which you discuss the literature on contracting -out and
summarize the arguments for and against it.

818 | P a g e
Appendix F
Reading List and Online
Resources

MPA Comprehensive Exam


Study Guide
PA Required Reading

Frederickson, George. Spirit of Public Administration. Jossey-Bass. (referred to


hereafter as Fred)

Peters, Guy. Future of Governing: Four Emerging Models. University Press of Kansas,
2nd edition.

Raadschelders, Jos. Government: A Public Administration Perspective. Armonk.


(hereafter referred to as: Raad I) Handbook of Administrative History. Transaction
Publishers. (hereafter referred to as Raad

II) Shafritz, Jay M. and Albert Hyde (editors). Classics of Public Administration.
Harcourt, Brace: 5th edition. (hereafter referred to as S & H)

Stillman, Richard J. Preface to Public Administration: A Search for Themes and


Direction. Chatelaine.

Van Wart, Montgomery. Changing Public Sector Values. Garland.

Recommended Reading PA

The following texts will either form the basis of lectures, handouts or book reports.
Starred books will be reported in class by students.

Farmer, David John. The Language of Public Administration. University of Alabama


Press.

Frederickson, H. George and Kevin B. Smith. The Public Administration Theory Primer.
Westview.

Goodsell, Charles T. The Case for Bureaucracy: A Public Administration Polemic. 4th
edition. Washington, D. C.: Congressional Quarterly Press, 2004.*

Kuhn, Thomas. The Structure of Scientific Revolutions. Chicago: second edition or


later.

Light, Paul C. The Tides of Reform: Making Government Work 1945 – 1995. Yale
University Press.*

820 | P a g e
Marini, Frank (editor). Toward a New Public Administration. Chandler.*

Ostrom, Vincent. The Intellectual Crisis in American Public Administration. University


of Alabama.*

Rohr, John. To Run a Constitution. University of Kansas.*

Waldo, Dwight. The Administrative State. Holmes and Meier, second edition.*

Wamsley, Gary L. et al. Refounding Public Administration. Sage*

Public Administration
Required texts:
Frank Goodnow, Politics and Administration
Dwight Waldo, The Administrative State
Camilla Stivers, Bureau Men, Settlement Women
O. C. McSwite, Legitimacy in Public Administration: A Discourse Analysis
John Rohr, To Run a Constitution
Kenneth Meier and Larry O’Toole, Bureaucracy in a Democratic State
Steven Maynard-Moody and Michael Musheno, Cops, Teachers, Counselors
Eric Klinenberg, Heat Wave
David Farmer, The Language of Public Administration
Graham Burchell, Colin Gordon, and Peter Miller (eds.), The Foucault Effect
Supplementary text: Camilla Stivers (ed.), Democracy, Bureaucracy & the Study of
Administration.

TOPICWISE SUGGESTED READINGS

Introduction to theory of public budgeting

Rubin, IS. 1997. Budgeting: Theory and Concepts. In Public Budgeting and Finance,
ed. Robert T. Golembiewski and Jack Rabin, 185-202. 4th ed. Marcel Dekker.

Key, VO. 1940. The Lack of a Budgetary Theory. The American Political Science Review
34, no. 6 (December): 1137-1144.

Straussman, JD. 1985. Key,V.O.’s The Lack of a Budgetary Theory - Where Are We
Now. International Journal Of Public Administration 7, no. 4: 345-374.

821 | P a g e
Bozeman, B and JD Straussman. 1982. Shrinking Budgets and the Shrinkage of
Budget Theory. Public Administration Review 42, no. 6: 509-515.

LeLoup, LT. 1997. The Myth of Incrementalism: Analytical Choices in Budgetary


Theory. In Public Budgeting and Finance, ed. Robert T. Golembiewski and Jack Rabin,
505-520. 4th ed. Marcel Dekker.

White, J. 1994. (Almost) Nothing New Under the Sun: Why the Work of Budgeting
Remains Incremental. Public Budgeting and Finance 14, no. 1: 113-134.

Budget process theory I

Wildavsky, A. 1992. "Political Implications of Budget Reform": A Retrospective. Public


Administration Review 52, no. 6 (December): 594-599.

Burns, RC and RD Lee. 2004. The Ups and Downs of State Budget Process Reform:
Experience of Three Decades. Public Budgeting and Finance 24 (September): 1-19.

Smith, RW and M Bertozzi. 1998. Principals and Agents: An Explanatory Model for
Public Budgeting. Journal of Public Budgeting, Accounting and Financial Management
10, no. 3: 325-353.

Ferejohn, J, And K Krehbiel. 1987. The Budget Process and the Size of the Budget.
American Journal of Political Science 31, No. 2 (May): 296-320.

Caiden, N. 1989. Budgeting for Time-Bombs: Recent General Accounting Office


Reports on the Crises of the Nuclear Weapons Complex and the Savings and Loan
Industry. Public Budgeting & Finance 9, no. 4 (December): 83-92.

Jones, BD, FR Baumgartner, and JL True. 1998. Policy, punctuations: US budget


authority, 1947-1995. Journal Of Politics 60, no. 1 (February): 1-33.

Jordan, M. 2003. Punctuations and agendas: A new look at local government budget
expenditures. Journal of Policy Analysis and Management 22 , no. 3 : 345-360.

Breunig, C, and C Koski. 2006. Punctuated equilibria and budgets in the American
states. Policy Studies Journal 34, no. 3: 363-379.

Reddick, CG. 2002. Testing Rival Decision-Making Theories on Budget Outputs:


Theories and Comparative Evidence. Public Budgeting & Finance 22, no. 3: 1-25.

822 | P a g e
Budget process theory II

Yellen, JL. 1989. Symposium on the Budget Deficit. The Journal of Economic
Perspectives 3, no. 2 (Spring): 17-21.

Barro, RJ. 1989. The Ricardian Approach to Budget Deficits. Journal of Economic
Perspectives 3, no. 2 (Spr): 37-54.

Auerbach, AJ, and WG Gale. 2000. Perspectives on the Budget Surplus. National Tax
Journal 53, no. 3 (September): 459-472.

Conway, KS. 1999. Are Workers "Ricardian"? Estimating the Labor Supply Effects of
State Fiscal Policy. PUBLIC FINANCE REVIEW 27, no. 2 (March): 160-193.

Budgets and fiscal policy

Toder, EJ. 2000. Tax Cuts or Spending - Does it Make a Difference? National Tax
Journal 53, no. 3: 361-371.

True, JL. 1995. Is The National Budget Controllable? Public Budgeting and Finance 15
(June): 18-32. doi:10.1111-1540-5850.01037.

Weidenbaum, ML. 1997. Budget "Uncontrollability" as an Obstacle to Improving the


Allocation of Government Resources. In Public Budgeting and Finance, ed. Robert T.
Golembiewski and Jack Rabin, 415-444. 4th ed. Marcel Dekker.

Savage, JD. 1994. Deficits and the Economy: The Case of the Clinton Administration
and Interest Rates. Public Budgeting and Finance 14, no. 1: 96-112.

Su, TT, MS Kamlet, and DC Mowery. 1993. Modeling United-States Budgetary and
Fiscal-Policy Outcomes - A Disaggregated, Systemwide Perspective. American Journal
of Political Science 37, no. 1 (February): 213-245.

Wildavsky, A. 1993. Norms and Rules to Facilitate Convergence on Budget Balance.


Public Administration Review 53, no. 1 (February): 28-30.

Caiden, N. 1998. The Rhetoric and Reality of Balancing Budgets. In Handbook of


Government Budgeting, 227-252. Jossey-Bass Nonprofit & Public Management Series.
Chichester, UK: John Wiley and Sons Ltd.

823 | P a g e
Ippolito, DS 1993. The Budget Process And Budget Policy - Resolving The Mismatch.
Public Administration Review 53, no. 1 (February): 9-13.

Public debt

Kowalcky, LK and LT LeLoup. 1993. Congress and the Politics of Statutory Debt
Limitation. Public Administration Review 53, no. 1 (February): 14-27.

Posner, P. 2001. Can Democratic Governments Save? Experiences of Countries with


Surpluses. Public Budgeting & Finance 21, no. 2: 1-28.

Brecher, C, RD Horton, and DM Mead. 1994. Budget Balancing in Difficult Times: The
Case of the Two New Yorks. Public Budgeting & Finance 14, no. 2: 79-102.

Poterba, JM. 1995. Balanced Budget Rules and Fiscal Policy: Evidence from the
States. National Tax Journal 48, no. 3: 329-336.

Hale, GE. 1979. Federal Courts and the State Budgetary Process. Administration &
Society 11, no. 3 (November 1): 357-368.

Harriman, L and JD Straussman. 1983. Do Judges Determine Budget Decisions?


Federal Court Decisions in Prison Reform and State Spending for Corrections. Public
Administration Review 43, no. 4 (August): 343-351.

Courts and budgets

Straussman, JD. 1986. Courts and Public Purse Strings: Have Portraits of Budgeting
Missed Something? Public Administration Review 46, no. 4 (August): 345-351.

Duncombe, WD and JD Straussman. 1994. Judicial Intervention and Local Spending -


The Case of Local Jails. Policy Studies Journal 22, no. 4 (Winter): 604-616.

MacManus, SA and PA Turner. 1993. Litigation as a Budgetary Constraint: Problem


Areas and Costs. Public Administration Review 53, no. 5 (October): 462-472.

Budget accountability and performance measures

Joyce, PG. 1993. Using Performance Measures for Federal Budgeting: Proposals and
Prospects. Public Budgeting and Finance 13, no. 4: 3-17.

824 | P a g e
Stiefel, L, R Rubenstein, and AE Schwartz. 1999. Using Adjusted Performance
Measures for Evaluating Resource Use. Public Budgeting & Finance 19: 67-87.

Melkers, J and K Willoughby. 2005. Models of Performance-Measurement Use in Local


Governments: Understanding Budgeting, Communication, and Lasting Effects. Public
Administration Review 65, no. 2 (March): 180-190.

Moynihan, DP. 2006. What Do We Talk About When We Talk About Performance?
Dialogue Theory and Performance Budgeting. Journal of Public Administration
Research and Theory 16, no. 2 (April 1): 151-168.

McNab, RM. and F Melese. 2003. Implementing the GPRA: Examining the Prospects
for Performance Budgeting in the Federal Government. Public Budgeting & Finance 23,
no. 2 (June): 73-95.

State Budget Issues I: Structures and actors

Kearns, PS. 1994. State Budget Periodicity - An Analysis of the Determinants and the
Effect on State Spending. Journal of Policy Analysis and Management 13, no. 2: 331-
362.

Abney, G and TP Lauth. 1985. The Line-Item Veto in the States: An Instrument for
Fiscal Restraint or an Instrument for Partisanship? Public Administration Review 45,
no. 3 (June): 372-377.

Alm, J and M Evers. 1991. The Item Veto and State Government Expenditures. Public
Choice 68: 1-15.

Lauth, TP. 1996. The Line-item Veto in Government Budgeting. Public Budgeting &
Finance 16: 97-111.

Reese, CC. 1997. The Line-item Veto in Practice in Ten Southern States. Public
Administration Review 57, no. 6: 510-516.

Thompson, JA. 1987. Agency Requests, Gubernatorial Support, and Budget Success
in State Legislatures Revisited. The Journal of Politics 49, no. 3 (August): 756-779.

Willoughby, KG and MA Finn. 1996. Decision Strategies of the Legislative Budget


Analyst

State Budget Issues II: Risk and budget stabilization


825 | P a g e
Weber, JA. 1997. Certainty and Uncertainty in Decision Making: a Conceptualization.
In Public Budgeting and Finance, ed. Robert T. Golembiewski and Jack Rabin, 449-
474. 4th ed. Marcel Dekker.

Douglas, JW and RK Gaddie. 2002. State Rainy Day Funds and Fiscal Crises: Rainy
Day Funds and the 1990-1991 Recession Revisited. Public Budgeting & Finance 22,
no. 1 (March): 19-30.

Joyce, PG. 2001. What's so Magical About Five Percent? a Nationwide Look at Factors
That Influence the Optimal Size of State Rainy Day Funds. Public Budgeting & Finance
21, no. 2 (June): 62-87.

Hou, YL. 2006. Budgeting for Fiscal Stability over the Business Cycle: A
Countercyclical Fiscal Policy and the Multiyear Perspective on Budgeting. Public
Administration Review 66, no. 5 (October): 730-741.

Peng, J. Managing the Risk of Variable-rate Debt in the Public Sector. Municipal
Finance Journal 23, no. 4: 1-16.

Taxation issues

Mikesell, JL. 1992. State Sales Tax Policy in a Changing Economy: Balancing Political
and Economic Logic Against Revenue Needs. Public Budgeting & Finance 12, no. 1: 83-
91.

Berg, J, JT Marlin, and F Heydarpour. 2000. Local Government Tax Policy: Measuring
the Efficiency of New York City’s Tax Mix, FYs 1984-1998. Public Budgeting and
Finance 20, no. 2: 1-14.

Andreoni, J, B Erard, and J Feinstein. 1998. Tax Compliance. Journal of Economic


Literature 36, no. 2 (June): 818-860.

Slemrod, JB, and S Yitzhaki. 2000. Tax Avoidance, Evasion, and Administration.
Working Paper. National Bureau of Economic Research.

Forest, A, and SM Sheffrin. 2002. Complexity and Compliance: An Empirical


Investigation. National Tax Journal 55: 75-88.

Ladd, HF. 1975. Local Education Expenditures, Fiscal Capacity, and Composition of
Property Tax Base. National Tax Journal 28, no.2: 145-158.

826 | P a g e
Goodspeed, TJ. 1998. The relationship between state income taxes and local property
taxes: Education finance in New Jersey. National Tax Journal 51, no.: 219-238.

Fischel, WA. 2001. Homevoters, municipal corporate governance, and the benefit view
of the property tax. National Tax Journal 54, no. 1: 157-173.

Debt finance and municipal bond market I

Hildreth, WB and CK Zorn. 2005. The Evolution of the State and Local Government
Municipal Debt Market Over the Past Quarter Century. Public Budgeting & Finance 25,
no. 4s (December): 127-153.

Marlin, MR. 1994. Did Tax Reform Kill Segmentation in the Municipal Bond Market?
Public Administration Review 54, no. 4 (August): 387-390.

Clingermayer, JC and BD Wood. 1995. Disentangling Patterns of State Debt


Financing. The American Political Science Review 89, no. 1 (March): 108-120.

Bahl, R and W Duncombe. 1993. State and Local Debt Burdens in the 1980s: A Study
in Contrast. Public Administration Review 53, no. 1 (February): 31-40.

Debt finance and municipal bond market II

Denison, DV. 2001. Bond Insurance Utilization and Yield Spreads in the Municipal
Bond Market. Public Finance Review 29, no. 5 (September): 394-411.

Poterba, JM. 1995. Capital Budgets, Borrowing Rules, and State Capital Spending.
Journal of Public Economics 56, no. 2 (February): 165-187.

Hsueh, LP and DS Kidwell. 1988. Bond Ratings: Are Two Better than One? Financial
Management 17, no. 1 (Spring): 46-53.

Kidwell, DS, EH Sorensen, and JM Wachowicz. 1987. Estimating the Signaling


Benefits of Debt Insurance: The Case of Municipal Bonds. The Journal of Financial and
Quantitative Analysis 22, no. 3 (September): 299-313.

Clarke, W and RL Bland. 2000. State Guarantees for School Debt and the Texas
penalty. Municipal Finance Journal Summer: 1-12.

827 | P a g e
The following books may serve as useful references for specific topics.
Joyce, P.G., Lee, R.D., Jr. and R.W. Johnson. 2007. Public Budgeting Systems.
Rubin, I.S. 2005. The Politics of Public Budgeting: Getting and Spending, Borrowing
and Balancing.
Wildavsky, A. and N. Caiden. 2003. The New Politics of the Budgetary Process (5th
ed.).
Livingston, M. 1996. Money and Capital Markets.
Lamb, R. and S.P. Rappaport. 1987. Municipal Bonds.
Mikesell, J. 2006. Fiscal Administration.
Miller, G.J. 1991. Government Financial Management Theory.
Finkler, S.A. 2004. Financial Management for Public, Health and Not-for-Profit
Organizations.

PA Authors Summary with their eras

1880's- Public Service Code Cadre of Administrators Woodrow


1920's Wilson
(schools)
Ellwood
Cubberly

Authority of Administrative Law Goodnow

1900- Scientific Management as Structure of Institution Max Weber


1930's (schools)
Raymond
Callahan

Scientific Management as Logic of Production Frederick


Taylor
(schools)
Raymond
Callahan

828 | P a g e
Management as Functions Gulick

1920- Human Relations as Reducing Fatigue and Monotony Illumination &


1930's Wiring Studies

Human Relations as Giving Orders Follett

Human Relations as Cohesion & Incentives Barnard


(schools) John
Dewey

1930's Public as Interest Groups Herring

Reform as Massive TVA Project of "democratic" Selznick


Involvement Lilenthal

1930's the Organization of Executive and Reorganization of Gulick


Authority Brownlow

REASSESSMENTS

1910- Scientific Inquiry as Experiential Learning Child Dewey


1950's Centere as Humane Relations

1940's The Bureaucratic Personality Merton

1970's Public Service and Collective Bargains Mosher

1980's Ethics of Neutrality and Objectivity Thompson

1980's Impossibility of Coherent Theory of Administering Rosenbloom


Public Institutions

829 | P a g e
Step Three Writers

1940's - "applied behavioral science" Herbert Simon


l950's (specify conditions that embed a particular decision)
* back to Gulick, Brownlow Committee

"nature of worker and productivity" Douglas


( managerial perceptions and expectations) McGregor
* back to Merton and Barnard, forward to Zuboff

"incremental muddle and successive approximations" Charles


(branch from ongoing, reference to allocation cycle) Lindblom
* back to Selznick

1960's "open systems and energy transfer cycle" Katz and Kahn
(stress and adaptation, input-output-feedback)

"life cycle of the bureau" Downs


( start up and maturing to incumbents and incentives)
* back to Lindblom

"its' marble cake, not layer cake" Grodzins


(tracing specific issues defines interest groups and
government)
* back to Simon

" implementation differs from formation intent" Pressman and


( actual impact of policy a bureaucratic translation, Wildavsky
seredipity)
* back to Lindblom and Downs

1980's " POSDCORB as managing functions: public and Allison


private management [functions] are at least as different

830 | P a g e
as they are similar, and the differences are more
important than the similarities"
(analyze specific cases to identify best & worst
practices)
* back to Gulick versus Simon

Step Four Writers

1969 end to interest group liberalism as not true populist Lowi


democracy and it assumes all values negotiable...plans
without standards.. lack of democratic forms...privilege
in implementation

1971 social equity guides new public administration..work to Frederickson


redress the deprived minorities; focus upon processes
of distribution, integration, boundary spanning and
socio-emotional commitments

1972 organizations evaluating selves lead to change, not Wildavsky


status quo, Evaluation, Inc. to identify social needs, do
policy as advocacy, join knowledge with power, be
skeptical rather than committed.

1967 "new professional role of analyst; systems plus political Dror


science, psychology, maturity, idealistic realism to
make somewhat better decisions in public policy
making"

1969 "rescuing analysis from PBBS formula; domestic goals Wildavsky


not easy, budgeting not stated operationally, no impact
of expected results, not for mass implementation in all
federal government"

831 | P a g e
1976 sinful policy analysis is being in a rut, too far away, Meltsner
late for process, superficial, topical, change for sake of
changing and not able to stand political tests in
implementation.

Step Five Writers

1978 "Growth is a common denominator that links Levine


contemporary management theory to its historical
antecedents and practices with public policy
choices....decline forces the logic for rationally
structured organization on its end and upside down. "
See problem depletion, organizational atrophy, political
vulnerability and environmental entrophy. Compare
with Downs "mature" bureau and terminating
discussions, especially Levine's tactics to smooth or
resist decline.

1980 Public service employees such as teachers, police, Lipsky


social workers, public lawyers and health workers are
imporatnt as street level bureaucrats because their
activities define the scope and function of public
services delievered and have direct impact on peoples
lives.

1981 Classic budget depends upon stability derived from Caiden


accurate prediction of revenues and costs and
knowledge of future output. It works best where yearly
adjustments are marginal. Yet, contemporary
uncertainties arise from novelty, forecasting, the
annual perspective, centralized bureaucratic control,
size, erosion of accountability and reflect the larger

832 | P a g e
American crisis. See Meltzner and Wildavsky
discussions.

1987 Public administrators resist the privatisation challenge Moe


because they cannot define its limits or the
distinctiveness of the public sector. This ambiguity
challenges the public assertion behind the TVA and
FADA as "distinctive sovereign." See the Allison,
Lilenthal and Lowi discussions.

Intergovernmental relations can be viewed conflict (before 1930), Wright


cooperative, concentrated, creative, competitive, calculative(70-
80's) and contractive(80-90's). Another way to reconsider the
meanings of constitutional-legal and resource
expansion,/contraction. Compare with Grodzins and Mosher.

PA Authors and Publications

AUTHOR PUBLICATION
1 Allison, Graham T. Public and Private Management: Are they
Fundamentally Alike in All Unimportant Respects?,
1980
2 Appleby, Paul Government is Different, 1945
Appleby, Paul The Role of the Budget Division, 1957
3 Argyris, Chris Interpersonal Competence and Organizational
Effectiveness, 1962
4 Barnard, Chester The Functions of the Executive, 1948
5 Barzelay, Michael Breaking through Bureaucracy, 1992
6 Bennis, Warren Organizations of the Future, 1967
7 Buchanan, James & Gordon The Calculus of Consent, 1962
Tullock
8 Burrell & Morgan Sociological Paradigms and Organizational analysis,
1985, 1979
9 Caiden, Naomi Processes, Policies and Power: Budget Reform, 1993
10 Cleveland, Frederick Evolution of the Budget Idea in the US, 1915
11 Dahl, Robert The Science of Public Administration, 1947
12 Daneke, Gregory A Science of Public Administration, 1990
13 Denhardt, Robert Theories of Public Organization, 1993
Denhardt, Robert In the Shadow of Organization, 1981
Denhardt, Robert Toward a Critical Theory of Public Organization, 1981

833 | P a g e
14 Downs, Anthony The Life Cycles of Bureaus, 1967
Downs, Anthony Why the Government Budget is Too Small in a
Democracy, 1959
15 Dror, Yehezel Ventures in Policy Sciences, 1971
16 Fayol, Henri General and Industrial Management, 1949
17 Follett, Mary Parker The Giving of Orders, 1918
18 Fredrickson, H. George The Spirit of Public Administration, 1977
Fredrickson, H. G. & Smith, The Public Administration Theory Primer, 2003
K. B.
Fredrickson, H. George Toward a New Public Administration, 1971
19 Goodnow, Frank Politics and Administration: A Study in Government,
1900
20 Goodsell, Charles The Case for Bureaucracy, A Public Administration
Polemic, 1983
21 Gulick, Luther Notes on the Theory of Organization, 1937
22 Henry, Nicholas The Paradigms of Public Administration, 1975
23 Herring, Pendleton Public Administration and the Public Interest,1937
24 Holzer, Marc Productivity and Quality Management, 1995
25 Jaffee, David Organization Theory: Tension and Change, 2001
26 Jaques, Elliott In Praise of Hierarchy, 1990
27 Katz, Daniel & Kahn, Robert Organizations and the System Concept (1966)
28 Kaufman, Herbert Administrative Decentralization and Political Power,
1969
29 Kettl, Donald The Global Public Management Revolution…, 2000
30 Key, V. O. The Lack of a Budgetary Theory, 1940
31 Kingdon, John Agendas, Alternatives and Public Policy, 1995
32 Kuhn, Thomas The Structure of Scientific Revolutions, 1962
33 Lasswell, Harold Politics: Who Gets What, When, How?, 1936
34 Lewis, Verne B. Toward a Theory of Budgeting, 1952
35 Light, Paul The Tides of Reform, 1997
36 Lindbloom, Charles The Science of Muddling Through, 1959
Lindbloom, Charles Still Muddling, Not Yet Through, 1979
37 Lipsky, Michael Street Level Bureaucracy, 1980
38 Lowi, Theodore The End of Liberalism, 1979
39 March, James & Johan Olson The New Institutionism: Organizational Factors in
Political Life, 1984
March, James & Johan Olson Democratic Governance, 1995
40 Marini, Frank Toward a New Public Administration, 1971
41 Maslow, A. H. A Theory of Human Motivation, 1943
42 McGregor, Douglas The Human Side of Enterprise, 1960, 1957
43 Meier, Kenneth Representative Bureaucracy: An Empirical Analysis,
1975
44 Mintzberg, Henry The Structuring of Organizations, 1979
Mintzberg, Henry The Fall & Rise of Strategic Planning, 1994
Mintzberg & Quinn Readings in the Strategy Process
45 Moe, Ronald C. Exploring the Limits of Privatization

834 | P a g e
46 Mosher, F. C. Basic Documents of American Public Administration,
1976
47 Niskanen, William A. Bureaucracy and Representative Government, 1971
48 Osborne, David & Ted Reinventing Government, 1992
Gaebler
49 Ostrom, Vincent The Intellectual Crisis in American Public
Administration, 1973
50 Peters, B. Guy The Future of Governing, 1996
Peters, B. Guy The Politics of Bureaucracy, 1995
51 Pfeffer, Jeffrey Power in Organizations, 1978
52 Raadschelders Handbook of Administrative History, 2000
53 Rosenbloom, David H. Public Administrative Theory and the Separation of
Powers, 1983
54 Rohr. John To Run a Constitution: The Legitimacy of the American
State, 1986
55 Rubin, Irene New Directions in Budget Theory, 1988
Rubin, Irene Budgeting: Theory, Concepts, Methods, Issues, 1992
56 Schick, Allen The Road to PBB: The Stages of Budget Reform, 1966
Schick, Allen Capacity to Budget, 1990
Schick, Allen The Federal Budget: Politics, Policy Process, 1995
57 Scott, W. Richard Organizations: Rational, Natural and Open systems,
1998
58 Selznick, Phillip The Cooptative Mechanism, 1949
59 Simon, Herbert Administrative Behavior, 1947
60 Stillman, Richard J. Preface to Public Administration, 1999
61 Stivers, Camilla Toward a Feminist Perspective in Public Administration,
1990
62 Stone, Deborah Policy Paradox: The Art of Political Decision Making,
2002
63 Taylor, Frederick Winslow Scientific Management, 1912
64 Tiebout, Charles A Pure Theory of Local Expenditures, 1956
65 Van Wart, Montgomery Changing Public Sector Values
66 Waldo, Dwight The Administrative State,1946
67 Wamsley, Gary, James Wolf Refounding Democratic Public Administration: Modern
Paradoxes….1996
68 Weber, Max Bureaucracy, 1922
69 White, Leonard Introduction to the Study of Public Administration, 1926
70 Wildavsky, Aaron The Budgetary Process, 1984
Wildavsky, Aaron A Budget for all Seasons? Why the Traditional Budget
Lasts, 1978
Wildavsky & Caiden The New Politics of the Budgetary Process,
71 Willoughby, William F. The Movement for Budgetary Reform in the US, 1918
72 Wilson, James Q. Bureaucracy: What Government Agencies Do and Why
They Do It, 1989
73 Wilson, Woodrow The Study of Public Administration, 1887
74 Yin, Robert Case Study Research, 2003

835 | P a g e
Public Administration M.P.A. /Ph.D. Reading List

I. Defining Public Administration

Woodrow Wilson. (1887). “The Study of Administration,” Political Science Quarterly *

Leonard White. (1926). Introduction to the Study of Public Administration

Robert A. Dahl. (1947). “The Science of Public Administration: Three Problems.” Public
Administration Review 7(1):1-11.

Wallace Sayre. (1958). “Premises of Public Administration: Past and Emerging.” Public
Administration Review, 18(2): 102-5 X

Paul Van Riper. (1983). “The American Administrative State: Wilson and the
Founders—An Unorthodox View.” Public Administration Review, 43(6): 477-490.

Dwight Waldo. (1985). The Enterprise of Public Administration: A Summary View

Gregory A. Daneke. (1990). “A Science of Public Administration?” Public Administration


Review 50(3):383-392.

Richard Stillman II. (1991). Preface to Public Administration

Charles T. Goodsell. (1992). “The Public Administrator as an Artisan.” Public


Administration Review 52(3):246-253.

Francis X. Neumann, Jr. (1996). “What Makes Public Administration a Science? Or,
Are Its ‘Big Questions’ Really Big?” Public Administration Review 56(5):409-415.

John J. Kirlin (1996). “The Big Questions of Public Administration in a Democracy.”


Public Administration Review 56(5):416-423.

Paul P. Van Riper. (1997). “Some Anomalies in the Deep History of U.S. Public
Administration.” Public Administration Review 57(3):218-222.

H. George Frederickson. (1997). The Spirit of Public Administration X


836 | P a g e
Richard F. Callhan. (2001). “Challenges of (Dis) Connectedness in the ‘Big Questions’
Methodologies in Public Administration.” Public Administration Review 61(4):493-499.

O. C. McSwite. (2002). Invitation to Public Administration

II. Intellectual Evolution of Public Administration

Politics-Administration Dichotomy and the Scientific Principles of Public


Administration

Woodrow Wilson. (1887). “The Study of Administration,” Political Science Quarterly *

Frank Goodnow. (1900). Politics and Administration

Frederick Taylor. (1911) Shop Management; also in Transactions 24 (June 1903):


1337-1456.

Frederick Taylor. (1911). The Principles of Scientific Management X

Luther Gulick. (1937). “Notes on the Theory of Organization.” In Papers on the Science
of Administration edited by Luther Gulick and Lyndall Urwick*

Henri Fayol. (1949). General and Industrial Management

Lyndall Urwick. (1943). The Elements of Administration

Wallace Sayre. (1958). “Premises of Public Administration: Past and Emerging.” Public
Administration Review, 18(2): 102-5 X

John F Dalrymple. (2001). “From F Winslow Taylor to W. Edwards Deming: Over a


Century of Progress?” (unpublished conference paper,
http://www.cmqr.rmit.edu.au/staff/jdalr.html)
Rebuttals to the Politics-Administration Dichotomy and the Principles

Herbert Simon. (1946). “The Proverbs of Administration,” Public Administration


Review, 6(Winter): 53-67.*

837 | P a g e
Herbert Simon. (1947). Administrative Behavior: A Study of Decision-Making
Processes in Administrative Organizations

Robert A. Dahl. (1947). “The Science of Public Administration: Three Problems” Public
Administration Review, 7:1-11.

Dwight Waldo. (1948). The Administrative State: A Study of the Political Theory of
American Public Administration

Paul Appleby. (1949). Policy and Administration

Paul Van Riper. (1984). “The Politics-Administration Dichotomy: Concept or Reality?”


In Rabin and Bowman, Eds., Politics and Administration: Woodrow Wilson and
American Public Administration.

James Svara. (2001). “The Myth of the Dichotomy: Complementarity of Politics and
Administration in the Past and Future of Public Administration.” Public Administration
Review, 61: 176-183.

Mark Rutgers. (2001). “Splitting the Universe: On the Relevance of Dichotomies for the
Study of Public Administration.” Administration & Society 33: 3-20

The Waldo-Simon Debate

Dwight Waldo. (1952). “Development of Theory of Democratic Administration,”


American Political Science Review 46(1):81-103.

Herbert Simon. (1952). “Development of Theory of Democratic Administration: Replies


and Comments” American Political Science Review 46(2): 494-496 (Reply to Waldo).

Dwight Waldo. (1952). “Development of Theory of Democratic Administration: Replies


and Comments.” American Political Science Review 46(2):501-503 (Reply to Simon).

Michael M. Harmon. (1989). “The Simon-Waldo Debate: A Review and Update.” Public
Administration Quarterly (Winter): 437-451.

838 | P a g e
Beyond the Politics-Administration Dichotomy and the Scientific Principles: The
Intellectual Evolution of the Field

Dwight Waldo. (1955). The Study of Public Administration

Ralph Clark Chandler, (ed.). (1987). A Centennial History of the American


Administrative State.

Laurence J. O’Toole, Jr. (1987). “Doctrines and Developments: Separation of Powers,


the Politics-Administration Dichotomy, and the Rise of the Administrative State.”
Public Administration Review 47(1):17-25.

Brian Fry. (1989). Mastering Public Administration: From Max Weber to Dwight
Waldo.

Frank P. Sherwood. (1990). “The Half-Century’s ‘Great Books’ in Public


Administration,” Public Administration Review, 50(2): 249-264.

Camilla Stivers. (1990). “Toward a Feminist Theory of Public Administration,” Women


and Politics, 10(4):49-65.

Robert T. Golembiewski. (1996). “The Future of Public Administration: End of a Short


Stay in the Sun? Or a New Day A-Dawning?” Public Administration Review 56(2):139-
148.

H. George Frederickson. (1999). “The Repositioning of American Public


Administration,” PS: Political Science & Politics 32:701-711.

Robert B. Denhardt. (2000). Theories of Public Organization. X

Donald F. Kettl. (2000). “Public Administration at the Millennium: The State of the
Field.” Journal of Public Administration Research and Theory 10(1):7-34.

Laurence E. Lynn Jr. (2001). “The Myth of the Bureaucratic Paradigm: What
Traditional Public Administration Really Stood For.” Public Administration Review,
61(2):144-160.
839 | P a g e
James Carroll and H. George Frederickson. (2001). “Dwight Waldo, 1913-2000,” Public
Administration Review 61:2-8.

H. George Frederickson and Kevin Smith. (2003). The Public Administration Theory
Primer

III. Constitutional Foundations of Public Administration and other Legitimacy


Sources

‘Publius’ [Alexander Hamilton, James Madison, and John Jay]. (Appeared in various
U.S. newspapers, October 1787-April 1788). The Federalist Papers

Lynton Caldwell. (1944). The Administrative Theories of Hamilton and Jefferson

John Schaar. (1964). “Some Ways of Thinking About Equality,” Journal of Politics
26:867

Vincent Ostrom. (1973). The Intellectual Crisis in American Public Administration X

John Rohr. (1986). To Run a Constitution

Gary L. Wamsley et. al. (1990). Refounding Public Administration

Theodore J. Lowi. (1993). “Legitimizing Public Administration: A Disturbed Dissent.”


Public Administration Review 53:261-264.

Camilla Stivers. (1993, 2002). Gender Images in Public Administration: Legitimacy and
the Administrative State.

Gary L. Wamsley et. al. (1996, 2004). Refounding Democratic Public Administration:
Modern Paradoxes, Postmodern Challenges X

O.C. McSwite. (1997). Legitimacy in Public Administration: A Discourse Analysis

840 | P a g e
IV. Bureaucracy and Bureaucratic Behavior

Max Weber. (1946). From Max Weber: Essays in Sociology. Translated, edited, and
introduced by H. H. Gerth and C. Wright Mills.

Peter Blau. (1956). Bureaucracy in Modern Society.

Robert K. Merton, et al., eds. (1952). Reader in Bureaucracy, particularly important is


Merton’s classic 1940 article on “Bureaucratic Structure and Personality.” * X

Victor Thompson. (1961). Modern Organization.

Michael Crozier. (1964). The Bureaucratic Phenomenon.

Anthony Downs. (1967). Inside Bureaucracy. X

Charles Perrow. (1972). Complex Organizations.

Michael Lipsky. (1980). Street-Level Bureaucracy: Dilemmas of the Individual in Public


Services X

Charles Goodsell. (1983). The Case for Bureaucracy.

James Q. Wilson. (1989). Bureaucracy: What Government Agencies Do and Why They
Do It.

William Gormley. (1989). Taming the Bureaucracy

Michael Barzelay. (1992). Breaking Through Bureaucracy

V. Public and Private Sector Differences. What is the “Public” in Public


Administration?

Paul Appleby. (1945). Big Democracy. The best known chapter of the book is
“Government is Different,” which is included in the Classics of Public Administration.*

841 | P a g e
Graham T. Allison. (1980). “Public and Private Management: Are They
Fundamentally Alike in All Unimportant Respects?”*

Barry Bozeman. (1987). All Organizations are Public.

J. Chandler. (1991). “Public Administration and Private Management: Is There a


Difference?” Public Administration 69:385-92

George Boyne. (2002). Public and Private Management: What’s the Difference?”
Journal of Management Studies, 39(1): 97-122.

New Public Management—Market-based Public Administration

Christopher Hood. (1991). “A Public Management for All Seasons?” Public


Administration 69:3-9.

David Osborne and Ted Gaebler. (1992). Reinventing Government: How the
Entrepreneurial Spirit is Transforming the Public Sector.

Al Gore. (1993). Creating a Government that Works Better and Costs Less: Report of
the National Performance Review.

Christopher Hood. (1994). “The New Public Management in the 1980s.” Accounting,
Organizations and Society 20:2/3:93-109

Patrick Dunleavy and Christopher Hood. (1994). “From Old Public Administration to
New Public Management.” Public Money and Management 7:9-16

Robert D. Behn. (1995). “The Big Questions of Public Management.” Public


Administration Review 55(4):313-324.

Laurence E. Lynn, Jr. (1998). “A Critical Analysis of the New Public Management.”
International Public Management Journal, 1(1): 107-123

842 | P a g e
Robert D. Behn. (1998). “The New Public Management Paradigm and the Search for
Democratic Accountability.” The International Public Management Journal 1(2):131-
164.

Linda Kaboolian. (1998). “The New Public Management,” Public Administration Review
58(3):189-193.

Richard Box, et al. (2001). “New Public Management and Substantive Democracy.”
Public Administration Review 61(5):608-619.

Privatization

Emanuel S. Savas, ed. (1977). Alternatives for Delivering Public Services.

Ted Kolderie. (1986). “The Two Different Concepts of Privatization.” Public


Administration Review 46:285-291

Donald F. Kettl. (1988). Government by Proxy: (Mis?)Managing Federal Programs.

H. Brinton Milward and Keith Provan. (2000). “Governing the Hollow State.” Journal of
Public Administration Research and Theory 10:359-379.

New Public Administration

Frank Marini, ed. (1971). Toward a New Public Administration: The Minnowbrook
Perspective. X

Dwight Waldo, ed. (1971). Public Administration in a Time of Turbulence

H. George Frederickson. (1980). New Public Administration

Louis C. Gawthrop and Jeffrey S. Luke. (1989). Minnowbrook: The Search for a New
Reality.” Public Administration Review. 49(2); Special Issue: Minnowbrook II. Changing
Epochs of Public Administration:194-196.

843 | P a g e
Curtis Ventriss. (1989). “Toward a Public Philosophy of Public Administration: A Civic
Perspective of the Public.” Public Administration Review 49(2); Special Issue:
Minnowbrook II. Changing Epochs of Public Administration:173-179.

H. George Frederickson. (1996). “Comparing the Reinventing Government Movement


with the New Public Administration.” Public Administration Review 56(3):263-270.

H. George Frederickson. (1997). The Spirit of Public Administration

VI. The Public Interest, Accountability, and Administrative Discretion

Pendleton Herring. (1936). Public Administration and the Public Interest

Glendon A. Schubert. (1961). The Public Interest: A Critique of the Theory of a Political
Concept

Carl J. Friedrich, (ed.). (1967). The Public Interest

Theodore Lowi (1979). The End of Liberalism.

Michael Lipsky. (1980). Street-Level Bureaucracy: Dilemmas of the Individual in Public


Services X

Charles Goodsell (1983). The Case for Bureaucracy X

Barbara Romzek and Melvin Dubnick. (1987). “Accountability in the Public Sector:
Lessons From the Challenger Tragedy,” Public Administration Review, 47:227-238

Charles Goodsell. (1990). “Public Administration and the Public Interest.” In Gary L.
Wamsley et. al. (Eds.), Refounding Public Administration.

The Friedrich-Finer Debate: How to Hold Bureaucracy Accountable?

Carl J. Friedrich. (1940). “Public Policy and the Nature of Administrative


Responsibility.” In Public Policy edited by Carl J. Friedrich and Edward S. Mason.

844 | P a g e
Herman Finer. (1941). “Administrative Responsibility in Democratic Government,”
Public Administration Review, 1(4): 335-350.

VII. Intergovernmental Relations and Federalism

Morton Grodzins. (1966). The American System: A New View of Government in the
United States, ed. By Daniel J. Elazar.

Daniel J. Elazar. (1966). American Federalism: A View From the States

Deil S. Wright. (1978). Understanding Intergovernmental Relations.

Martha Derthick. (1987). “American Federalism: Madison’s Middle Ground in the


1980s.” Public Administration Review 47:66-74.

Deil S. Wright. (1990). “Federalism, Intergovernmental Relations, and


Intergovernmental Management: Historical Reflections and Conceptual Comparisons.”
Public Administration Review 50(2):168-178.*

VIII. Conflicting Public Administration Values: The Efficiency-Equity Debate

Anthony Downs. (1957). An Economic Theory of Democracy

Frank Marini. (1971). Toward a New Public Administration: The Minnowbrook


Experience X

David K. Hart. (1974). “Social Equity, Justice, and the Equitable Administrator.” Public
Administration Review 34:3-10.

G. W. Downs and P.D. Larkey. (1986). The Search for Government Efficiency

J. H. Knott and G. J. Miller. (1987). Reforming Bureaucracy

Gary L. Wamsley et. al. (1996, 2004). Refounding Democratic Public Administration:
Modern Paradoxes, Postmodern Challenges X

845 | P a g e
Decision-Making: Emphasizing the Bounds of Rationality

Mary Parker Follett. (1926). “The Giving of Orders.” From Scientific Foundations of
Business Administration, by Henry C. Metcalf (ed.).* X

Mary Parker Follett. (1941). Dynamic Administration: The Collected Papers of Mary
Parker Follett, edited by Henry C. Metcalf and Lyndall Urwick

Herbert Simon. (1947). Administrative Behavior: A Study of Decision-Making


Processes in Administrative Organizations

Philip Selznick. (1949). “The Cooptive Mechanism,” in TVA and the Grass Roots*

James G. March and Herbert Simon. (1958). Organizations

Charles Lindbloom. (1959). “The Science of Muddling Through,” Public Administration


Review. 19:79-88. X

Herbert Simon. (1965). “Administrative Decision Making.” Public Administration


Review 25:31-37.

Amitai Etzioni. (1967). “Mixed-Scanning: A ‘Third’ Approach to Decision Making.”


Public Administration Review 27:385-392.

Anthony Downs. (1967). Inside Bureaucracy. X

Graham Allison. (1969). “Conceptual Models and the Cuban Missile Crisis.” American
Political Science Review, 63: 689-718.

Graham Allison. (1971). Essence of Decision: Explaining the Cuban Missile Crisis.

Michael Cohen, James March, and Johan Olsen. (1972). “ A Garbage Can Model of
Organizational Choice,” Administrative Science Quarterly 17:1-25.

Charles Lindbloom. (1979). “Still Muddling, Not Yet Through,” Public Administration
Review 39: 517-526.

846 | P a g e
Michael M. Harmon. (1989). “‘Decision’ and ‘Action’ as Contrasting Perspectives in
Organization Theory.” Public Administration Review 49(2):144-152.

Johan Olsen. (2001). Garbage Cans, New Institutionalism, and the Study of Politics.”
American Political Science Review 95(1):191-198.

Rational or Public Choice: Emphasizing Pure Rationality

Charles Tiebout. (1956). “A Pure Theory of Local Expenditures.” Journal of Political


Economy, 44:416-424. X

James K Buchanan, and Gordon Tullock. (1962). The Calculus of Consent

Gordon Tullock. (1965). The Politics of Bureaucracy.

Mancur Olson. (1965). Logic of Collective Action.

William Niskanen. (1971). Bureaucracy and Representative Government

Vincent Ostrom and Elinor Ostrom. (1971). “Public Choice: A Different Approach to
the Study of Public Administration.” Public Administration Review 31:203-216.

Vincent Ostrom. (1973). The Intellectual Crisis in American Public Administration X

Emanuel Savas, ed. (1977). Alternatives for Delivering Public Services. X

Vincent Ostrom. (1987). The Political Theory of a Compound Republic (2nd. Edition).

William Lyons and David Lowery. (1989). “Governmental Fragmentation Versus


Consolidation: Five Public Choice Myths About How to Create Informed, Involved and
Happy Citizens,” Public Administration Review (Nov.-Dec.):533-543.
IX. Public Human Resources and Personnel Administration

Dorman B. Eaton. (1880). “Civil Service Reform in Great Britain.” *

Carl Fish. (1904). The Civil Service and the Patronage

847 | P a g e
Donald Kingsley. (1944). The Representative Bureaucracy: An Interpretation of the
British Civil Service.

Frederick C. Mosher. (1968). Democracy and the Public Service

Samuel Krislov. (1974). Representative Bureaucracy.

Hugh Heclo. (1977). A Government of Strangers

Walter Broadnax. (2000). Diversity and Affirmative Action in Public Service.

Norma M. Riccucci (2002). Managing Diversity in Public Sector Workforces

Julie Dolan and David H. Rosenbloom, eds. (2003). Representative Bureaucracy:


Classic Readings and Continuing Controversies

X. Public Budgeting and Fiscal Administration

William F. Willoughby. (1918). The Movement for Budgetary Reform in the States *

V. O. Key Jr. (1940). “The Lack of a Budgetary Theory.” American Political Science
Review 34:1137-1144.*

Verne B. Lewis. (1952). “Toward a Theory of Budgeting.” Public Administration Review


26 (December)

Charles Tiebout. (1956). “A Pure Theory of Local Expenditures.” Journal of Political


Economy 44:416-424. X

Charles Lindbloom. (1959). “The Science of Muddling Through,” Public Administration


Review, 19:79-88. X

Aaron Wildavsky. (1964, 1972, 1979, 1984). The Politics of the Budgetary Process

Allen Schick. (1966). “The Road to PPB: The Stages of Budget Reform.” Public
Administration Review*
848 | P a g e
James Buchanan, Charles Rowley, and Robert Tollison, eds. (1987). Deficits.

Aaron Wildavsky. (1988, 1992). The New Politics of the Budgetary Process

Irene S. Rubin (ed.). (1988). New Directions in Budget Theory

Irene S. Rubin. (1989). “Aaron Wildavsky and the Demise of Incrementalism,” Public
Administration Review, 49(1): 78-81.

Beryl Radin. (2000). “The Government Performance and Results Act and the Tradition
of Federal Management Reform: Square Pegs in Round Holes?” Journal of Public
Administration Research and Theory, 1:111-136

XI. Organization Theory

Frederick Taylor. (1911). The Principles of Scientific Management X

Mary Parker Follett. (1926). “The Giving of Orders,” from Scientific Foundations of
Business Administration, by Henry C. Metcalf (ed.).* X

Chester Barnard. (1938). The Functions of the Executive

F. J. Roethlisberger and William J. Dickson. (1939). Management and the Worker

Robert K. Merton, et al., eds. (1952). Reader in Bureaucracy, particularly important is


Merton’s classic 1940 article on “Bureaucratic Structure and Personality.” * X

Abraham H. Maslow. (1943). “A Theory of Human Motivation.” Psychological Review


50:370-396; * also Motivation and Personality (1954).

Herbert Simon. (1947). Administrative Behavior: A Study of Decision-Making


Processes in Administrative Organizations X

Douglas M. McGregor. (1960). The Human Side of Enterprise

849 | P a g e
Daniel Katz and Robert Kahn. (1966). The Social Psychology of Organizations. See
especially “Organizations and the System Concept,” reprinted in Classics of
Organization Theory by Jay M. Shafritz and J. Steven Ott (3rd edition).

James D. Thompson. (1967). Organizations in Action.

Henry Mintzberg. (1979). The Structure of Organizations. See especially “The Five
Basic Parts of the Organization,” reprinted in Classics of Organization Theory by Jay
M. Shafritz and J. Steven Ott (3rd edition).

Gareth Morgan. (1996). Images of Organization.

Robert B. Denhardt. (2000). Theories of Public Organization. X

Tor Hernes and Tore Bakken. (2003). “Implications of Self-Reference: Niklas


Luhmann’s Autopoiesis and Organization Theory.” Organization Studies 24(9):1511-
1535.

Organization Culture

Linda Smircich (1983). “Concepts of Culture and Organizational Analysis.”


Administrative Science Quarterly 28:339-358.

Stephen R. Barley. (1983). “Semiotics and the Study of Occupational and


Organizational Cultures.” Administrative Science Quarterly 28:393-413.

J. Steven Ott. (1988). The Organizational Culture Perspective.

Edgar Schein. (1992). Organizational Culture and Leadership.

Joanne Martin. (1992). Culture in Organizations: Three Perspectives.

XII. Leadership and Management

Ralph M. Stogdill. (1950). “Leadership, Membership, and Organization.” Psychological


Bulletin 47(January):1-14.

850 | P a g e
Cecil A. Gibb. (1954). “Leadership,” in Gardner Lindzey, ed., Handbook of Social
Psychology, vol. 2:877-920.

Ralph M. Stogdill and Alvin E. Coons, eds. (1957). Leader Behavior: Its Description
and Measurement.

Philip Selznick. (1957). Leadership in Administration.

Fred Fiedler. (1965). “Engineer the Job to Fit the Manager.” Harvard Business Review
43(September):115-122.

Victor Vroom and Phillip W. Yetton. (1973). Leadership Decision Making.

Victor Vroom and Arthur G. Jago. (1978). “On the Validity of the Vroom-Yetton Model.”
Journal of Applied Psychology 63(April):151-162.

Robert R. Blake and Jane S. Mouton. (1978). The New Managerial Grid.

R. H. George Field. (1982). “A Test of the Vroom-Yetton Normative Model of


Leadership.” Journal of Applied Psychology 67(October):523-532.

Victor Vroom and Arthur G. Jago. (1988). The New Leadership.

Gary A. Yukl. (2001). Leadership in Organizations.

David G. Carnevale. (2002). Organizational Development in the Public Sector.

John Antonakis, Anna T. Cianciolo, and Robert J. Sternberg, eds. (2004). The Nature
of Leadership.

XIII. Public Sector Ethics

John A. Rohr. (1978, 1989). Ethics for Bureaucrats: An Essay on Law and Values.

Montgomery Van Wart. (1996). “The Sources of Ethical Decision Making for
Individuals in the Public Sector.” Public Administration Review 56(6):525-533.
851 | P a g e
John A. Rohr. (1998). Public Service, Ethics, and Constitutional Practice.

Louis C. Gawthrop. (1998). Public Service and Democracy: Ethical Imperatives for the
21st Century.

Montgomery Van Wart. (1998). Changing Public Sector Values.

Terry L. Cooper. (1990, 1998). The Responsible Administrator: An Approach to the


Ethics of the Administrative Role.

Donald C. Menzel. (1999). “Rediscovering the Lost World of Public Service Ethics: De
We Need New Ethic for Public Administrators?” Public Administration Review
59(5):443-447.

Terry L. Cooper. (2004). “Big Questions in Administrative Ethics: A Need for Focused,
Collaborative Effort.” Public Administration Review 64 (4), 395-407.

John A. Rohr. (2004). “On Cooper's ‘Big Questions’” Public Administration


Review 64 (4), 408-409.

XIV. Transnational Public Administration and Governance

Fred W. Riggs. (1964). Administration in Developing Countries: The Theory of a


Prismatic Society.

Fred W. Riggs. (1968). “Administration and a Changing World Environment.” Public


Administration Review 28(4): 348-361.

Ferrel Heady. (1984, 1996). Public Administration: A Comparative Perspective

Andres Perez Baltodano. (1997). “The Study of Public Administration in Times of


Global Interpenetration: A Historical Rationale for a Theoretical Model.” Journal of
Public Administration Research and Theory 7(4):615-638.

852 | P a g e
Eric Welch and Wilson Wong. (1998). “Public Administration in a Global Context:
Bridging the Gaps of Theory and Practice between Western and Non-Western Nations.”
Public Administration Review 58(1):40-49.

Mark R. Rutgers. (2001). “Traditional Flavors? The Different Sentiments in European


and American Administrative Thought.” Administration & Society 33(2):220-244.

Jamil A. Jreisat. (2002). Comparative Public Administration.

Donald E. Klingner. (2004). “Globalization, Governance, and the Future of Public


Administration: Can We Make Sense Out of the Fog of Rhetoric Surrounding the
Terminology?” Public Administration Review 64(6):737-743.

Mark R. Rutgers (2004). “Comparative Public Administration: Navigating Scylla and


Charybdis—Global Comparison as a Translation Problem.” Administrative Theory &
Praxis 26(2):150-168.

XV. The Emerging Paradigm: From Logical Positivism to Interpretivism and


Discourse-Based Public Administration

Max Horkheimer. (1972). “Traditional and Critical Theory.” In Critical Theory: Selected
Essays, translated by Matthew J. O’Connell et al., pp. 1188-243.

Max Horkheimer. (1974). Eclipse of Reason.

Robert B. Denhardt. (1981). “Toward a Critical Theory of Public Organization.” Public


Administration Review 41(6):628-635.

David J. Farmer. (1995). The Language of Public Administration: Bureaucracy,


Modernity, and Postmodernity.

Charles J. Fox and Hugh T. Miller. (1996). Postmodern Public Administration: Toward
Discourse.

Charles J. Fox. (1996). Reinventing Government as Postmodern Symbolic Politics.”


Public Administration Review 56(3):256-262.
853 | P a g e
Gary L. Wamsley et. al. (1996, 2004). Refounding Democratic Public Administration:
Modern Paradoxes, Postmodern Challenges X

XVI. Theory and Classics

Alexis (Charles Henri Clerel) de Tocqueville. (1835 and 1840). Democracy in America.
First published in French in 1835 (vols. 1 & 2) and 1840 (vols. 3 & 4). English
translation published in the U.S. in 1838 (Part I) and 1840 (Part II).

Peter L. Berger and Thomas Luckman. (1966). The Social Construction of Reality

Thomas Kuhn. (1962, 1970). The Structure of Scientific Revolutions

Paul Davidson Reynolds. (1971). A Primer in Theory Construction

Alberto Guerreiro Ramos. (1981). The New Science of Organizations.

Bent Flyvbjerg. (2001). Making Social Science Matter

XVII. General Reference

Howard E. McCurdy. (1986). Public Administration: A Bibliographic Guide to the


Literature.

Ralph C. Chandler. (1987). A Centennial History of the American Administrative State.

Daniel W. Martin. (1989). The Guide to the Foundations of Public Administration.

Ali Farazmand. (1990). Handbook of Comparative and Development Public


Administration.

J. Steven Ott, Albert C. Hyde, and Jay Shafritz. (1991). Public Management: The
Essential Readings.

Jay M. Shafritz and J. Steven Ott. (2001). Classics of Organization Theory.

854 | P a g e
Frank J. Thompson. (2003). Classics of Public Personnel Policy.

B. Guy Peters and Jon Pierre. (2003). Handbook of Public Administration

Jay Shafritz, Albert Hyde, and Sandra Parkes. (2004). Classics of Public
Administration.

Key Journals (be familiar with last five years of these journals):

Journal of Policy Analysis and Management

Policy Sciences

Journal of Public Administration Research and Theory

Policy Studies Journals and Reviews

Public Administration Review

Amercian Review of Public Administration

American Political Science Review

Journal of Economic Perspectives

Journal of Public Economics

Nobel Lectures (in economics) for last ten years (published in Am Economic Review
and Journal of Public Economics alternately)

Readings are divided into the following sections:

1. Philosophy of Natural Science

2. Philosophy of Social Science

3. Public Policy Theory / Political Science

4. Decision Making Theory

5. Agenda Setting and Participation

6. Microeconomics / Public Finance

7. Public Choice

8. Organization Theory
855 | P a g e
9. Public Management / Administration

10. Policy Implementation

11. Policy Evaluation

12. Research Design and Methods

1. Philosophy of Natural Science

Chalmers, A.F. What is this thing called science? 3rd edition. Indianapolis: Hackett
Publishing Company, Inc. 1999.

Hempel, Carl G., “Studies in the Logic of Confirmation” in Aspects of Scientific


Explanation and Other Essays in the Philosophy of Science, NY: Free Press, pp. 3-51,
1965.

Hempel, Carl G., “Inductive Inconsistencies” in Aspects of Scientific Explanation and


Other Essays in the Philosophy of Science, NY: Free Press, pp. 53-79, 1965.

Hempel, Carl G., “Laws and their Role in Scientific Explanation” in Philosophy of
Natural Science, Prentice Hall, pp. 47-69, 1966.

Kuhn, Thomas. The Structure of Scientific Revolutions, 2nd edition. Chicago, IL:
University of Chicago Press, 1970.

Lakatos, "Falsification and the Methodology of Scientific ResearchPrograms," in


Criticism and the Growth of Knowledge, edited by I. Lakatos and A. Musgrave
(Cambridge: Cambridge University Press, 1970), pp. 91 - 195.

Nersessian, N. J. How do scientists think? Capturing the dynamics of conceptual


change in science. In Giere, R. N. (ed.) Cognitive Models of Science. Minnesota Studies
in the Philosophy of Science, 15, University of Minnesota Press, 1992, pp 3-45.

Popper, K. (1959). The Logic of Scientific Discovery, London: Unwin Hyman, pp. 27-
123; 265-281.

Woodward, J. and Goodstein, D. (1996) “Conduct, Misconduct and the Structure of


Science.” American Scientist, Vol. 84, No. 5 (Sept/Oct), 479-90.

2. Philosophy of Social Science

856 | P a g e
Braybrooke, D. "Three Sides of Social Science." Philosophy of Social Science.
Englewood Cliffs, NJ: Prentice Hall. 1987.

Hawkesworth, M.E. Theoretical Issues in Policy Analysis. Albany: SUNY Press, 1988.

Jennings, Ethics, the Social Sciences and Policy Analysis (Chapter 1). New York:
Plenum Press, 1983.

Paris, DC and J.F. Reynolds, The Logic of Policy Inquiry. New York: Longman, 1983.

Rosenberg, A. Economics - mathematical politics or science of diminishing returns?


Chicago: University of Chicago Press, 1992.

3. Public Policy Theory/ Political Science

Allison, Graham T. "Conceptual Models and the Cuban Missile Crisis" The American
Political Science Review Vol. 63, Issue 3, 689-718. 1969

Dahl and Lindblom. Politics, Economics and Welfare (1992) New York, Harper deLeon,
P. Advice and consent: The development of the policy sciences. New York: Russell Sage
Foundation, 1988.

Dror, Yehezkel. Public Policymaking Reexamined, 1983.

Dror, Yehezkel. Design for Policy Sciences (1971) New York, American Elsevier Pub.
Co.

Dryzek, John S. Discursive Democracy: Politics, Policy and Political Science (1994)

Dye, Thomas. Understanding Public Policy. Englewood Cliffs, N.J.: Prentice Hall. 1998

Easton, David. A Framework for Political Analysis. Englewood Cliffs, NJ: Prentice-Hall,
Inc., 1965.

Edelman, Murray J. The Symbolic Uses of Politics. Urbana: University of Illinois Press,
1964.

Fenno, Richard. The Power of the Purse: Appropriations Politics in Congress (1966)

Boston, Little, Brown Lane, Jan-Erik. The Public Sector : Concepts, Models and
Approaches, London : Sage, 1993

Lasswell, Harold D. A Pre-view of Policy Sciences. New York: American Elsevier, 1971.

857 | P a g e
Lerner, Daniel and Lasswell, Harold D. with the editorial collaboration of Harold H.
Fisher [and others].. The Policy Sciences. Stanford, Stanford University Press (1951)

Lindblom, Charles. Politics and Markets (1977) New York: Basic Books.

Lowi, Theodore J. (1969) The End of Liberalism: Ideology, Policy, and the Crisis of
Public Authority, New York: W.W. Norton & Company, Inc.

McCool, Daniel C. Public Policy Theories, Models, and Concepts: An Anthology.


Englewood Cliffs, NJ: Prentice-Hal, Inc, 1995.

Olsen, Mancur. The Logic of Collective Action: Public Goods and the Theory of Groups.

Cambridge, MA: Harvard University Press, 1965.

Ripley, Randall. G.A. Franklin. Congress, the Bureaucracy, and Public Policy
Homewood, IL: Dorsey Press, 1976.

Rittel, Horst W.J. and Melvin Webber, "Dilemmas in a General Theory of Planning",
Policy Sciences 4(1973), 155-169.

Sager, Tore "Planning and the Liberal Paradox: A Democratic Dilemma in Social
Choice," Journal of Planning Literature, Vol. 12, No. 1 (August 1997).

Sandler, Michael. Democracy's Discontent: America in Search of Public Philosophy


Theodoulou, Stella Z., M.A. Cahn. Public Policy: The Essential Readings Englewood
Cliffs, NJ: Prentice-Hall, Inc. 1995.

Wilson, James Q. 1989. Bureaucracy: what government agencies do and why they do
it. New York: Basic Books.

4. Decision Making Theory

Cohen, M.D., J.G. March, and J.P. Olsen "A Garbage Can Model of Organizational
Choice"

Administrative Science Quarterly, 17,1, pp. 1-25. 1972

Hastie, Reid, and Dawes, Robyn M. Rational Choice in an Uncertain World : The
Psychology of Judgement and Decision Making. Sage Publications, 2001.

Janis, Irving and Leon Mann. Decision Making : A Psychological Analysis of Conflict,
Choice and Commitment (1985) New York : Free Press

858 | P a g e
Kahneman, Daniel. Experienced Utility and Objective Happiness: A Moment-Based
Approach.

Chapter 37 in: D. Kahneman and A. Tversky (Eds.) Choices, Values and Frames. New
York:

Cambridge University Press and the Russell Sage Foundation, 2000

Lindblom, Charles E., "The Science of Muddling Through." Public Administration


Review 19 (Spring 1950): 79-88.

Schick, Allen The Federal Budget: Politics, Policy, Process Washington, D.C. :
Brookings Institution, 1995.

Simon, H.A. Administrative Behavior: A Study of Decision-Making Processes in


Administrative Organizations, 4th Edition. New York: Free Press, 1997.

Schon, Donald. The Reflective Practitioner : How Professionals Think in Action (1984)

Stone, D. Policy Paradox: The Art of Political Decision Making, NY: W. W. Norton &
Company, Inc., 1997.

Weiss, C.H. and M.J. Bucuvalas. "Truth Tests and Utility Tests: Decision-Makers
Frames of Reference for Social Science Research". American Sociology Review, 45,
1980, p. 302-312.

Weiss, C.H. and M.J. Bucuvalas. Social Research and Decision Making. New York:
Columbia University Press (1980).

5. Agenda Setting and Participation

Dahl, Robert A. Pluralist Democracy in the United States: Conflict and Consent
Chicago: Rand McNally & Company. 1967.

Kingdon, John W., Agendas, Alternatives, and Public Policies Boston: Little, Brown
and Company, 1984.

Putnam, Robert. The Comparative Study of Political Elites. Englewood Cliffs, N.J.:
Prentice-Hall, 1976

Wildavsky, Aaron. N. Caiden. The New Politics of the Budgetary Process, 3rd edition
Addison- Wesley Longman, Inc. 1997.

859 | P a g e
6. Microeconomics/Public Finance

Chiang, A.C. Fundamental Methods of Mathematical Economics, 3rd Edition McGraw


Hill, 1984.

Frank, R.H. Microeconomics and Behavior, 3rd Edition, McGraw-Hill, 1997.

Nelson, Richard. The Sources of Economic Growth (1996) Cambridge, Mass.: Harvard
University Press.

Nicholson, W. Microeconomic Theory: Basic Principles and Extensions, 7th Edition.


Dryden, 1998.

North, Institutions, Institutional Change and Economic Performance Cambridge, New


York: Cambridge University Press, 1990.

Rosen, Harvey. Public Finance. Irwin/McGraw-Hill. 1998.

Varian, Hal R. Microeconomic Analysis. 3d ed. New York: Norton & Co. 1992

Varian, Hal R. Intermediate Microeconomic Analysis: A Modern Approach, New York:


W.W. Norton, 1999.

7. Public Choice

Buchanan, James M. Tullock, G. The Calculus of Consent: Logical Foundations of


Constitutional Democracy The University of Michigan Press, Ann Arbor, MI. 1965.

D. Kahneman and A. Tversky (Eds.) Choices, Values and Frames. New York:
Cambridge University Press and the Russell Sage Foundation, 2000

McNutt, P.A. The Economics of Public Choice, Edward Elgar Publishing, 1996

Mueller, D.C. Public Choice II, Revised Edition. Cambridge Univ. Press, 1989

Niskanen, William A., Jr. Bureacracy and Representative Government Chicago: Aldine
Publishing Company. 1971.

8. Organization Theory

Bozeman, Barry. All Organizations are Public: Bridging Public and Private
Organizational Theories. San Francisco: Jossey-Bass. 1987.

Bozeman, Barry. Bureaucracy and Red Tape, Englewood Cliffs, NJ: Prentic-Hall. 1999

860 | P a g e
Daft, RL 1995. Organization Theory and Design. St. Paul, Minn.: West.

Downs, A. 1967 Inside Bureaucracy. New York: Little, Brown.

Goodsell, CT 1994. The Case for Bureaucracy. NJ: Chatham House.

Hall, RH 1996. Organizations: Structure and Process. (6th ed.) Upper Saddle River,
NJ: Prentice Hall.

Katz, D. and Kahn, R.L. The Social Psychology of Organizations. New York: John
Wiley, 1966.

March, J. and Simon, H. (1958) Organizations. New York: Wiley.

Morgan, Gareth Images of Organizations, 2nd Edition. Sage. 1996

Perry, JL and HG Rainey. "The Public Private Distinction in Organization Theory: A


Critique and Research (date?)

Strategy." Academy of Management Review 13: (2) 182-201 APR 1988

Rainey, HG. 1997. Understanding and Managing Public Organizations. (2nd ed.) San
Francisco: Jossey-Bass.

Scott, W.R. (1998) Organizations: Rational, natural, and open Systems. 4th ed. (NJ:
Prentice Hall).

Thompson, J.D. Organizations in Action. McGraw-Hill, 1967.

Thompson, V.. Modern Organization. New York: Knopf, 1961.

Williamson, Oliver E. 1981. "The Economics of Organizations: A Transaction Cost


Approach."

American Journal of Sociology, 87:548-577.

9. Public Management/Administration

Bozeman, Barry (1993) "A Theory of Red Tape" Journal of Public Administration
Research and Theory, 3(3): 273-303.

Bozeman, Barry. (Ed.) 1993. Public Management: State of the Art. San Francisco:
Jossey-Bass.

861 | P a g e
Bozeman, Barry 1993. "Theory, Wisdom, and the Character of Knowledge in Public
Management: A Critical View of the Theory-Practice Linkage" In Public Management:
State of the Art. San Francisco: Jossey-Bass, Barry Bozeman/Landsbergen "Truth and
Credibility in Sincere Policy Analysis", Evaluation Review, 1989

Bozeman, Barry. Public Management and Policy Analysis New York: St. Martin's Press,
1979

Gulick, L. "Notes on the Theory of Organization. In L. Gulick and L. Urwick (eds.),


Papers on the

Science of Administration. New York: Institute of Public Administration. 1937.

Heclo, H. A Government of Strangers. Washington, DC: Brookings, 1977.

Lin, N. and Wildavsky, A. Public Administration: The State of the Discipline (1990)

Lynn, LE Public Management as Art, Science and Profession. Chatham NJ: Chatham
House. 1996

Moore, Mark. Creating Public Value : Strategic Management in Government (1997)

Rohr, J.A. To Run a Constitution : The Legitimacy of the Administrative State. Univ. of
Kansas Press, 1986.

Savas, Emanuel. Privatization and Public-Private Partnerships Chatham House (1999)

Simon, H.A., Thompson, V. and Smithburg, D.W. Public Administration, Revised. New
Brunswick, NJ: Transactions, 1991.

Waldo, D. The Administrative State, 2nd Edition. New York: Holmes & Meier, 1984.

Weber, Max. 1970 [1920] "Bureaucracy." In Oscar Grusky and George A. Miller (eds.).
The Sociology of Organizations: Basic Studies. New York: Free Press.

10. Policy Implementation

Bardach, Eugene. The Implementation Game: What Happens after a Bill Becomes a
Law Cambridge, MA: MIT Press. 1977

Lipsky, Michael. Street-Level Bureaucracy: Dilemmas of the Individual in Public


Services, New York: Russell Sage Foundation, 1980.

862 | P a g e
Mazmanian and Sabatier. Implementation and Public Policy. Glenview, IL: Scott,
Foresman, 1983.

Pressman, Jeffery L. and A. Wildavsky. "Implementation: How Great Expectations in


Washington are Dashed in Oakland" in Implementation, Pressman and Wildavksy,
eds. Berkley: University of California Press. 1984.

Sabatier, Paul and Hank Jenkins-Smith, Policy Change and Learning: An Advocacy
Coalition Approach. Boulder, Colo.: Westview Press, 1993.

Paul Sabatier, ed. (1999). Theories of the Policy Process. Boulder, CO: Westview Press.

11. Policy Evaluation

Dryzek, John and Davis Bobrow, Policy Analysis by Design. Pittsburgh, Pa: University
of Pittsburgh Press, 1987.

Dunn, William. Policy Analysis. Prentice Hall: 1993.

Lindblom, Charles E. and David K. Cohen. Usable Knowledge: Social Science and
Social Problem Solving. New Haven: Yale. 1979.

Lindblom, Charles. Strategy of Design: Policy Evaluation as a Social Process Free


Press (1970) Majone, Giandomenico. Evidence, Argument and Persuasion in the Policy
Process New Haven, CT: Yale University Press. 1989

Munger, Michael, Analyzing Policy: Choices, Conflicts, and Practices. WW Norton


Press, (2000)

Patton, Carl and Sawicki, David. 1993. Basic Methods of Policy Analysis and Planning.
2nd edition. Englewood Cliffs, NJ: Prentice Hall.

Randall Ripley, Policy Analysis in Political Science. New York: Nelson-Hall, 1985

Shaddish, W.R., Cook, T.D. and Leviton, L.C. Foundations of Program Evaluation:
Theories of Practices, Sage, 1993.

Weimer, David L. and Vining, Aiden R. 1992. Policy Analysis: concepts and practice.
2nd edition.

Englewood Cliffs, NJ: Prentice Hall.

863 | P a g e
Weiss, Carol. Evaluation Research: Methods of Assessing Program Effectiveness Upper
Saddle River, NJ: Prentice Hall. 1998.

12. Research Design and Methods

Babbie, The Practice of Social Research (Chapter 2-4) Belmont, CA : Wadsworth Pub.
Co., 1998.

Campbell, Donald T. and Julian C. Stanley. Experimental and Quasi-Experimental


Designs for Research by Paperback (June 1966). Houghton Mifflin College; ISBN:
0395307872.

Cook, Thomas, D.T. Campbell. Quasi-experimentation: Design & Analysis Issues for
Field Settings. Boston: Houghton Mifflin, 1979.

Degroot, Morris H. and Mark J. Schervisch. Probability and Statistics, 3rd edition. ©
2002 / 0-201-52488-0 / Addison-Wesley OR

King, Gary, Robert O. Keohane, and Sidney Verba. Designing Social Inquiry: Scientific
Inference in Qualitative Research. Princeton University Press, 1994 (e-book 2001).

Kennedy, P. A Guide to Econometrics, (4th Edition), Blackwell, 1998.

Maddala, G.S. Introduction to Econometrics, (3rd edition), Wiley, 2001.

Mendenhall, William, and Sincich, T.T. A Second Course in Statistics: Regression


Analysis, 5th Edition. Upper Saddle River, NJ: Prentice-Hall, Inc. 1996

Miles, M.B. and Huberman, A.M. 1994. Qualitative Data Analysis. 2nd edition.
Thousand Oaks, CA: Sage.

Nachmias, David and C. Nachmias. Research Methods in the Social Sciences, 3rd
edition. New York: St. Martin's Press. 1987.

Neter, John, M.H. Kutner, and C.J. Nachtsheim. W. Wasserman. Applied Linear
Statistical Models, 4th Edition. Chicago, IL: Irwin. 1996.

Rosenthal, Robert and R. L. Rosnow. Essentials of Behavioral Research: Methods and


Data Analysis New York: McGraw-Hill Book Company, 1984.

Sieber, Sam. "The Integration of Fieldwork and Survey Methods" American Journal of
Sociology Vol 78 No 6.

864 | P a g e
Singleton, Jr., R. A. and B.C. Straits. Approaches to Social Research. Third Edition.
New York and Oxford: Oxford University Press, 1999. ISBN 0-19-510525-7.

Sudman, Seymour, N.M. Bradburn. Asking Questions: A Practical Guide to


Questionnaire Designs. San Francisco, CA: Jossey-Bass Inc. Publishers. 1982

Weiss, R.S. 1994. Learning from Strangers: the art and method of qualitative interview
studies. Free Press.

Woolridge, J.M. Introductory Econometrics, South-Western, 1999.

Yin, Robert. Case Study Research, Design and Methods. Third Edition. Newbury Park,
Sage Publications, 2002.

865 | P a g e

Das könnte Ihnen auch gefallen